147 34 10MB
English Pages [2634] Year 2023
Become An Author or Editor StatPearls is an academic teaching project developed by thousands of health professionals worldwide. Our goal is to provide inexpensive, up to date, peer reviewed study questions for all healthcare specialties. These questions are available in Apps, eBooks, and an online learning system that includes the opportunity to earn CME/CE. We need your help improving questions. Please identify questions that need revision by flagging a question. In addition, please consider becoming an author or editor. In as little as 2-3 hours you can make a substantial contribution to medical education and you will receive free CME/CE for your contribution! Please go to www.statpearls.com to sign up.
Learn More Want to learn more? Please visit our online learning system at www.statpearls.com for additional FREE questions, color pictures, and more detailed explanations.
Introduction NOTE: The intent of StatPearls review books is to identify knowledge deficits and assist you in the learning process. Review books are not intended to be a source of the knowledge base of medicine. The authors or editors do not warrant the information is complete or accurate. The reader is encouraged to verify each answer in several references. All drug indications and dosages should be verified before administration. Copyright 2023 StatPearls Publishing, LLC. All rights reserved, including the right of reproduction, in whole or in part, in any form.
http://www.StatPearls.com
Congratulations! This StatPearls book will help improve your fund of knowledge. Our books are designed to help you identify and resolve knowledge deficits. To achieve this goal the text is written in a multiple-choice format with answers and explanations. Explanations contain additional information intended to reinforce your knowledge. Emphasis has been placed on covering facts that are easily overlooked, quickly forgotten, and often show up in specialty reviews. For further information, all explanations are electronically linked to http://www.StatPearls.com . We suggest you note all questions answered incorrectly to allow you to rapidly review the content prior to your exam. We appreciate your comments, suggestions, and criticisms. Some answers and explanations may be in variance with your own knowledge. This is usually attributable to variability among sources; however, please make us aware of any potential errors you find as we update our books often. We appreciate your input
in regard to format, content, or presentation. Please contact us at [email protected] or better yet, click on the flag button after each question to give us specific input on any errors or omissions. We look forward to hearing from you. Good luck with learning your specialty!
StatPearls eBooks StatPearls books were created exclusively for use on eBook readers. Because we don't publish our books in paper format, we have the luxury of optimizing them for eBook use.
How are our books optimized for the viewing on an eBook Reader?
1.
Efficient User Interface: Each multiple choice question is on a page by itself so you can see the question and four multiple choice answers, but you can't see the correct answer until you flip the page. This would be a complete waste of paper in an ordinary book, but it makes for an easy to use eBook. You don't have to cover up the correct answer with a piece of paper. Most devices have a dedicated button for page turning so with the click of a button, you can see the correct answer.
2.
Research Links: After each explanation we include web links to search for more information on the key concepts for that question. If you miss a question, this is when you probably want to do more research on the question concepts. This would be onerous and require a lot of typing using a paper book, but is a single click on an eBook reader, PC, or Mac.
3.
Flag a Question: Medical standards and drug standards change. Also, human errors can occur resulting in questions that have issues. StatPearls strongly believes in Continuous Quality Improvement (CQI). Flagging a question gives you, the medical professional, the ability to tell us if something is wrong with a question and we will correct it and get back to you with an updated book (providing you give us a phone number or email address.) If you don't provide us with contact information, you can check
for a book update using the normal methods from the books store and you will soon find an update with the question corrected by one of our health professionals.
4.
Volume of Questions: When we make a book we publish every question we have on a topic (up to 2,000 per book). This would be ridiculously expensive to do with traditional book publishing, but in the eBook world it costs no more to publish 2,000 questions than 1,000 questions. You will find that in most of our specialties, we dwarf the competition with breadth of material. This makes our books "just what the doctor ordered" for those wishing to go the extra mile when studying.
STATPEARLS AUTHORS AND EDITORS Over 6,000 physician, nursing, and allied health professionals authored a database of over 50,000 classified multiple-choice questions and explanations. Each question has been peer reviewed by two health professionals and a pharmacist. All questions are reviewed annually and updated through a peer review process. WE APPRECIATE YOUR COMMENTS! We appreciate comments, suggestions, corrections and additional contributing authors and editors. Please e-mail us at: [email protected].
Oncology Specialty Review and Self-Assessment StatPearls Book ID: MD_ONCOLOGYSPECIALTY_REVIEW_AND_SELF_ASSESSMENT2022-10-27 Version: 10/27/2022
Editors in Chief Hani Babiker, MD; Assistant Professor; Director; University of Arizona Cancer Center; United States, Tucson, AZ, USA Anup Kasi, MD, MPH; Assistant Professor; University of Kansas; United States, Westwood, KS, USA Shiva Kumar Mukkamalla, MD, MPH; Clinical Assistant Professor; Presbyterian Healthcare Services; United States, Rio Rancho, NM, USA
Contributing Editors Shahab Abid, MD, PhD; Professor; Aga Khan Hospital; Karachi, Pakistan Ayham Aboeed, MD; Assistant Professor; Kern Medical / UCLA; Bakersfield, CA, USA Rotimi Adigun, MBBS, MPH; Associate Professor; Director of Pathology Program; University of Health Sciences, Antigua; St John's, Antigua & Deps Narothama Aeddula, MD, FASN; Clinical Assistant Professor; Deaconess HS, IN University School Med; Evansville, IN, USA Manuj Agarwal, MD; Assistant Professor; University of Pennsylvania; Glen Mills, PA, USA Rajni Ahlawat, MD; Assistant Professor; Marshfield Clinic Un Wisconsin-Madison; Marshfield, WI, USA Faran Ahmad, MD; Attending; Creighton University Medical Center; Omaha, NE, USA Iftiker Ahmad, MD; Michigan State University, College of Human Medicine; East Lansing, MI, USA
Aadil Ahmed, MD; Assistant Professor; Rush University Medical Center; Chicago, IL, USA Hossein Akhondi, MD, FACP; Associate Professor; University of Nevada; Las Vegas, NV, USA Nora Al Aboud, PhD; Assistant Professor; Umm Al-Qura University; Makkah, Saudi Arabia Ahmad Al Aboud, MD; Consultant, Professor; Chairman of Dermatology Department; King Abdullah Medical City; Makkah, Saudi Arabia Nader Al Obaidi, PharmD; KAMC; Mecca, Saudi Arabia Mohammed Al-Dhahir, MD; Strong Memorial Hospital/ University of Rochester / visiting physician; West Henrietta, NY, USA Ghaith Al-Eyd, MBChB, MSc, PhD; Associate Professor; Nova Southeastern University; Coral Springs, FL, USA Ozer Algan, MD; Practicing Physician; Oklahoma City, Alsadiq Al-Hillan, MD; Fellow; Baylor college of medicine; Houston, TX, USA Fahad Alkherayf, MBBS, MD, MSc, CIP, FRCSC; Assistant Professor; University of Ottawa; Ottawa, Canada
Saud Alsubait, MD; Fellow; Michigan State University; Lansing, MI, USA Neha Amin, MD; Assistant Professor; University of Maryland School of Medicine; Baltimore, MD, USA Jason An, MD; Attending; Associate Program Director; Riverside Community Hospital/UCRiverside; Riverside, CA, USA Catherine Anastasopoulou, MD, PhD; Associate Professor; Chair; Einstein Medical Center; Elkins Park, PA, USA Claudio Andaloro, MD; Santa Marta e Santa Venera Hospital; Catania, Italy Fatima Anjum, MD; Clinical Associate Professor of Medicine; Hofstra University, Zucker School of Medicine; Manhasset, NY, USA Faiz Anwer, MD; Associate Professor; Cleveland Clinic; Cleveland, OH, USA Komal Arora, MD; Practicing Physician; United Health Services Hospital; Hermitage, PA, USA John Ashurst, DO, MSc; Program Director; Kingman Regional Medical Center; Kingman, AZ, USA
Leela Athalye, DO; Private Practice; Newport Beach, CA, USA Christopher Atkinson, MD; Assistant Professor; Chief; Wilford hall ambulatory surgical center; San Antonio, TX, USA Maximos Attia, MD; Faculty Associate; Guthrie/RPH Family Residency Program; Sayre, PA, USA Alexandre Azevedo, MD, MSc; Hospital Santa Marcelina; São Paulo, Brazil Muhammad Aziz, MD; Instructor; Fellow; University of Toledo Medical Center; Toledo, OH, USA Hani Babiker, MD; Assistant Professor; Director; University of Arizona Cancer Center; Tucson, AZ, USA Talel Badri, MD; Associate Professor; University of Tunis El Manar; Tunis, Tunisia Tushar Bajaj, MD; Practicing Physician; University of Chicago; Chicago, IL, USA Kevin Banks, MD; Assistant Professor; Uniformed Services University; San Antonio, TX, USA Hassana Barazi, MD, MBA; Assistant Professor; WVU School of Medicine; Morgantown, WV, USA
Michael Barrett, MB, BAO, BCh, BMedSci; Consultant; Our Lady's Children's Hospital; Dublin, Ireland {Republic} Khalid Bashir, MD; Associate Professor; Chief; Creighton University School of Medicine; Omaha, NE, USA Kalgi Baxi, MBBS, MD; Consultant Dermatologist; Gujarat University;GCS Medical College; Ahmedabad, India Paul Bedocs, DO; Associate Clinical Professor; Ohio University; Avon, OH, USA Lisa Bellin, MD; Associate Professor; Univ of Tennessee Health Science Center; Nashville, TN, USA Rene Bermudez, DO; Attending; Director; Sampson Regional Medical Center; Advance, NC, USA Harshil Bhatt, MD; Goshen Hospital; Goshen, IN, USA Priyanka Bhattacharya, MD, FACP; Clinical Assistant Professor of Medicine; Hospitalist; University of Pennsylvania; Philadelphia, PA, USA Kamal Bhusal, MD; Associate Professor; Louisiana State University HSC; Shreveport, LA, USA Beenish Bhutta, MBBS, MD; Chief Resident; Sheikh Zayed Hospital, Lahore; Lahore, Pakistan
Sameh Boktor, MD, MPH; Penn State College of Medicine; Mechanicsburg, PA, USA Pradeep Bollu, MD; Assistant Professor; Associate Program Director; University of Missouri; Columbia, MO, USA Sophie Bondje, MBChB; Lister Hospital; London, United Kingdom Kristin Booth, DO; Michigan State University, POH Regional Medical Center; Clarkston, MI, USA Vaibhav Bora, MBBS, FASE, FASA; Associate Professor; Director Critical Care Medicine; Augusta University; Augusta, GA, USA Bruno Bordoni, PhD; Professor; PhD; Foundation Don Carlo Gnocchi IRCCS; Milano, Italy Bracken Burns, MS, DO; Professor of Surgery; Director of Trauma and Critical Care; East Tennessee State University (ETSU); Johnson City, TN, USA Burt Cagir, MD, FACS; Professor; TCMC, Robert Packer Hospital; Sayre, PA, USA Ahmet Can, MD; Adjunct Clinical Assistant Professor; Program Director; Lake Erie College of Osteopathic
Medicine (LECOM); Elmira, NY, USA Yvonne Carter, MD; Assistant Professor; Sinai Hospital; Baltimore, MD, USA Damian Casadesus, MD, PhD; Assistant Professor; Jackson Memorial Hospital; Miami Beach, FL, USA Marco Cascella, MD; Professor; Istituto Nazionale Tumori - IRCCS - Fondazione Pascale, Via Mariano Semmola 80100, Napoli. Italy; Napoli, Italy Sebastiano Cassaro, MD; Associate Clinical Professor; Surgery Residency Program Director; UN California Irvine / Kaweah Delta; Visalia, CA, USA Nathan Cecava, MD; Adjunct Assistant Professor; MSK Radiology Fellowship PD; San Antonio Uniformed Services HEC; San Antonio, TX, USA Raja Chandra Chakinala, MD; Guthrie Robert Packer Hospital; Sayre, PA, USA Aaron Chambers, DPM; Fellow; Naval Hospital Bremerton; Silverdale, WA, USA Onyee Chan, MD; Un of AZ, Banner Un Medical Center; Tucson, AZ, USA
Chi-Hwa Chan, BDS, MBChB; Luton and Dunstable University Hospital; Luton, United Kingdom Richard Chang, MD; Assistant Clinical Professor; University Hospitals-Cleveland Medical Center; Chagrin Falls, OH, USA Sujith Cherian, MD; Assistant Professor; U T HealthMcGovern Medical School; Houston, TX, USA John Childs, MD; Assistant Professor; Pathology Residency Program Director; Walter Reed National Military Medical Center; Bethesda, MD, USA Christopher Cook, DO; Co Program Director; Wilmington, NC, USA Jeffrey Cooper, MD; Associate Professor; Director; University of Nebraska Medical Center; Omaha, NE, USA Jean Coviello, DO; Professor; Staff Hematopathologist; SAMMC; Schertz, TX, USA Christina Cox, MD; Private practice; Coliseum Health Systems; Macon, GA, USA Jonathan Crane, DO; Dermatology Residency Program Director; Sampson Regional Med Ctr / Campbell Univ; Wilmington, NC, USA
David Crotzer, MD, MS; Clinical Assistant Professor; Creighton University; Omaha, NE, USA Craig Czyz, DO; Professor; Dept. Chair, Section Chief; Ohio University; Columbus, OH, USA Mauricio Danckers, MD; ICU Medical Director; Aventura Hospital and Medical Center; Miami, FL, USA Orlando De Jesus, MD; Professor; University of Puerto Rico, Medical Sciences Campus, Neurosurgery Section; San Juan, Puerto Rico Linda Deluxe, MD; Attending; Elmhurst Hospital Center; Elmhurst, NY, USA Jeffrey Deppen, DO; Attending Surgeon; Mclaren Lansing; Lansing, MI, USA Dhaval Desai, MBBS; Private Practice; Bronx, NY, USA Raffaela Di Napoli, MD; Institut Jules Bordet -Brussels, Belgium; Bruxelles, Belgium Christopher Dickson, MB ChB, BMedSci; Respiratory Registrar; Royal Devon and Exeter NHS Foundation Trust; Exeter, United Kingdom Elvita Dominique, DO; Program Director; Wyckoff Heights Medical Center; Brooklyn, NY, USA
Joseph D'Orazio, MD; Assistant Professor; Director; Lewis Katz School of Medicine at Temple University; Philadelphia, PA, USA Anterpreet Dua, MD; Associate Professor; Augusta University; Augusta, GA, USA Ramin Dubey, PhD; Postdoctoral Scholar; Stanford School of Medicine; Stanford, CA, USA Hieu Duong, MD; Private Practice; Aventura, FL, USA Peter Edemekong, MD, MPH; Faculty; NOVA Southeastern Un College of OM; Jupiter, FL, USA Christopher Edwards, MD; Assistant Professor; University of Arkansas; Little Rock, AR, USA Faouzi El Mezni, MD; Private Practice; Tunis, Tunisia Prabhu Emmady, MD, MBA, CPE; Adjunct Assistant Professor of Neurology; UNC school of Medicine, Atrium Health; Concord, NC, USA Fabiola Farci, MD; Pisa University; S.G. Monreale, Italy Mary Fatehi, MD; Attending; Director, Gynecologic Oncology; Nassau University Medical Center; East Meadow, NY, USA
Saira Fatima, MBBS, FCPS; Assistant Professor; Aga Khan University; Karachi, Pakistan Troy Ferguson, DO; Michigan State University; Lansing, MI, USA Sarah Ferrer-Bruker, DO; VCOM/Orange Park Medical Center; Orange Park, FL, USA Eric Fillman, MD; Assistant Professor; San Antonio Military Medical Center; Cibolo, TX, USA Ajeet Gajra, MD; Professor; SUNY Upstate Medical University; Syracuse, NY, USA Karen Garfield, MD, MS, BA; Mount Sinai Hospital; New York, NY, USA Rick Gemma, DO; Private Practice; NA, _, USA Valerie Gerriets, PhD; Assistant Professor; California Northstate University College of Medicine; Elk Grove, CA, USA Abhijeet Ghatol, MD; Mclaren Hospital, Flint, MI; Troy, MI, USA Al Giwa, LLB, MD, MBA, MBE; Associate Professor; Icahn School of Medicine at Mount Sinai; New York, NY, USA
Pouyan Gohari, MD; Private Practice; East Meadow, NY, USA Marcus Goodman, DO; Program Director; Residency; Philadelphia College of Osteopathic Med.; Roswell, GA, USA Marisa Gossweiler, DO; Brooke Army Medical Center; Garden Ridge, TX, USA Krishna Kumar Govindarajan, MCh (Pediatric Surgery), MS (Gen Surg); Additional Professor; JIPMER; Pondicherry, India Amandeep Goyal, MBBS, MD; Hospitalist; University of Kansas Medical Center; Kansas City, MO, USA Gary Gross, MD; Residency Program Director; Lewis Gale Hospital Montgomery; Salem, VA, USA Craig Grossman, MD, PhD, MSCE; Assistant Attending; Memorial Sloan Kettering Cancer Center; Commack, NY, USA Mounika Gunduru, MD; Wayne State University / Detroit Medical Center; West Bloomfield, MI, USA Gunjan Gupta, MD, MS; IGMC; Shimla, India
Mohit Gupta, MD; Assistant Professor, CCLCM and CWRU; Cleveland Clinic Foundation; Cleveland, OH, USA Nishant Gupta, MD; Assistant Professor; Columbia University at Bassett Medical Center, NY, USA; New York, NY, USA Neha Gupta, BDS, DMD, GPR; Private Practice; Westlake, OH, USA Vikas Gupta, MD, MPH; South Carolina Dept of Mental Health; Columbia, SC, USA Nilmarie Guzman, MD; Professor; Program Director TY; Orange Park Medical Center; Jacksonville, FL, USA Mary Hanley, DO; Associate Professor; Pro Dir Undersea and Hyperbaric; Un of N En Coll of Osteo Med; Charleston, SC, USA Matthew Hanlon, MD; Assistant Professor; Associate Program Director; SUNY Upstate Medical University; Marcellus, NY, USA Bonnie Harmer, PhD, MSN, MSEd; Associate Professor; Saginaw Valley State University; Midland, MI, USA Muhammad Haseeb, MBBS; Clinical Instructor; University of Pittsburgh; Pittsburgh, PA, USA
Michelene Hearth Holmes, MD; Asstistant Professor; UNMC; Omaha, NE, USA Matthew Hefner, MD; Louisiana State University; Shreveport, LA, USA Daniel Heller, MD; Attending; Aventura Hospital and Medical Center; Aventura, FL, USA William Henderson, MD; Attending; SOVAH Health Danville; Danville, VA, USA Jeffery Hogg, MD; Professor; West Virginia University School of Med.; Morgantown, WV, USA Danielle Horn, MD; Assistant Professor; Program Director; University of Miami; Miami, FL, USA Robert Hostoffer, DO, PhD; Associate Professor; Case Western University; Highland Heights, OH, USA Sameh Hozayen, MD, MSc; Adjunct Clinical Faculty; Chief of Internal Medicine; Sovah Health; Greensboro, NC, USA Ronald Hsu, MD; Professor; Univ of Calif, Davis, School of Medicine; Roseville, CA, USA Brian Jack, MD; Professor; Chair; Boston University School of Medicine; Boston, MA, USA
Tibb Jacobs, PharmD; Clinical Associate Professor; University of Louisiana Monroe; Shreveport, LA, USA Prachi Jain, MDS; Pandit B. D Sharma University of Health Sciences, Rohtak; Rohtak, India Arif Jan, MD; Associate Professor; Chief Academic Hospitalists; Drexel University; Voorhees, NJ, USA Shraddha Jatwani, MD; Albert Einstein Medical Center; Wynnewood, PA, USA Trishal Jeeva-Patel, MBBCH (WITS), FCOphth(SA), MMEDc; Fellow; University of Toronto; Toronto, Canada Andrew Jenzer, DDS; Oral and Maxillofacial Surgery; Womack Army Hospital; Southern Pines, NC, USA Jordan Jeong, DO; Professor; Assistant Program Director; Coney Island Hospital; Brooklyn, NY, USA Melenda Jeter, MD, MPH; Professor; UT MD Anderson Cancer Center; Houston, TX, USA Ishwarlal Jialal, MD, PhD, FRCPATH, DABCC; DIRECTOR, CLINICAL CHEMISTRY; VA MEDICAL CENTER, MATHER , CA; Davis, CA, USA Savio John, MD; Associate Professor; Director; SUNY Upstate Medical University; Syracuse, NY, USA
William Jones III, MD; Assistant Professor; Associate Program Director; South Texas Veterans Health Care System; San Antonio, TX, USA Zachary Joos, MD; Un of Utah Hosp, John A Moran Eye C; Salt Lake City, UT, USA Robert Joseph, DPM, PHD; Assistant Professor; Department Chair; Rosalind Franklin University; Lake Bluff, IL, USA Virginia Kalish, MD; Assistant Professor; Fort Belvoir Community Hospital; Alexandria, VA, USA Feroze Kaliyadan, MD, DNB,FRCP (London), European Board DV; Assistant Professor; King Faisal University; Hofuf, Saudi Arabia Avyakta Kallam, MBBS; Assistant Professor; University of Nebraska Medical Center; Omaha, NE, USA Steven Kane, MD; Attending; Chairman; Wellstar Atlanta Medical Center; Atlanta, GA, USA Harsha Karanchi, MD; Assistant Professor; Louisiana State University HSC; Shreveport, LA, USA Sanaz Kashan, MD; Assistant Professor; Program Director; Aventura Hospital; Weston, FL, USA
Sarang Kashyap, MD, MBBS; Assistant Professor of Surgery; Beckley Appalachian Regional Healthcare Hospital; Mt Hope, WV, USA Anup Kasi, MD, MPH; Assistant Professor; University of Kansas; Westwood, KS, USA Kirandeep Kaur, MBBS, DNB, FICO, MNAMS, MRCS; Consultant; Dr.; Aravind Eye Hospital, Pondicherry; Amritsar, India Harleen Kaur, MBBS; Assistant Professor of Radiology; University of Arkansas for medical sciences; Little Rock, AR, USA Cecylia Kelley, DO; Clinical Associate Professor; Inspira Health Network; Haddonfield, NJ, USA Alysia Kemp, MD; Attending; Karmanos Cancer Institute Wayne State University; Mt Clemens, MI, USA Shachar Kenan, MD; Assistant Professor; Attending Faculty; New Hyde Park, Ana Keppke, MD; Kettering Health Network; Dayton, OH, USA Mikelle Kernig, DDS, MS; Associate Professor; Staff Oral and Maxillofacial Pathologist; Uniformed Services
University HS, SAMMC; Fort Sam Houston, TX, USA Karam Khaddour, MD; Rosalind Franklin University of Medicine; McHenry, IL, USA Farhan Khan, MD; Assistant Professor; University of Tennessee; Memphis, TN, USA Yusuf Khan, MBBS, MD; Assistant Professor; Dr.; Vision (Alfarabi) College of Medicine, Riyadh, KSA; Riyadh, Saudi Arabia Paras Khandhar, MD; Assistant Professor; Beaumont Health System; West Bloomfield Township, MI, USA Robert Killeen, MD; University of Illinois; Holiday, FL, USA Catherine Kim, MD; Penn Medicine and Virtua; Voorhees, NJ, USA Kevin King, MD; Practicing Physician; Emergency Medicine Physician; Pittston, _, USA Arpan Kohli, MD; Assistant Professor; Associate Program Director; West Virginia University; Morgantown, WV, USA Noah Kondamudi, MD, MBA, DM, DCh, FAAP, FACEP, CPE; Clinical Associate Professor; MD. MBA; Icahn School of
Medicine at Mount Sinai , The Brooklyn Hospital Center; West Orange, NJ, USA Nebu Koshy, MD, MBBS; Louisiana State University HSC; Shreveport, LA, USA Jiten Kothadia, MD; Assistant Professor; Transplant Hepatologist; University of Tennessee Health Science Center, James D. Eason Transplant Institute; Germantown, TN, USA Supriya Koya, MD; Genesis Care; tulsa, OK, USA Kevin Krauland, MD; Assistant Professor; Associate Program Director; Brooke Army Medical Center; Seguin, TX, USA Karthik Krishnamurthy, DO; Associate Professor; Residency; Orange Park Medical Center; Jacksonville, FL, USA Koyamangalath Krishnan, MD; Professor; Dishner Endowed Chair in Medicine; East Tennessee State University; Johnson City, TN, USA Sandeep Kumar, MD; Attending; Chief of Dermatology; VA Medical Center; Lexington, KY, USA
Abhishek Kumar, MD; Assistant Professor of Medicine; Albert Einstein College of Medicine; Bronx, NY, USA Gary Kwartowitz, DO; Attending; McLaren Oakland; Clarkston, MI, USA Chris Kyriakopoulos, MD, PhD; Consultant; University Hospital of Ioannina; Ioannina, Greece Danielle Ladie, MD, MPh; Associate Program Director; UPMC Pinnacle; Harrisburg, PA, USA Keith Lagnese, MD; Assistant Clinical Professor; Chief Medical Officer; University of Pittsburgh/UPMC; Pittsburgh, PA, USA Chad LaGrange, MD; Associate Professor; Malashock Chair of Uro Sur; University of Nebraska Medical Center; Omaha, NE, USA Kamolyut Lapumnuaypol, MD; BIDMC, Harvard University; Jamaica Plain, MA, USA Savita Lasrado, MBBS, MS, Diplomate NB; Assistant Professor, Consultant; Father Muller Medical College, Mangalore, India; Mangalore, India Jacqueline Le, MD; Assistant Medical Director; Desert Regional Medical Center; Palm Springs, CA, USA
Yehuda Lebowicz, MD; Assistant Professor; Marshall SOM; Huntington, WV, USA Lawrence Lee, MD; Assistant Professor; MD; Indianapolis, IN, USA Manidhar Reddy Lekkala, MBBS; University of Rochester Medical Center; Rochester, NY, USA Stephen Leslie, MD; Associate Professor of Surgery, Urology; Creighton University Medical Center; Omaha, NE, USA Catherine Lewis, MD, PhD; Attending; Norton Community Hospital; Norton, VA, USA Kevin Lillehei, MD; Professor; Chairman; University of Colorado Department of Neurosurgery; Aurora, CO, USA Faten Limaiem, MBBS; Assistant Professor; Associate Professor; University of Tunis El Manar, Tunis Faculty of Medicine; Tunis, Tunisia Noureddine Litaiem, MD; University of Tunis El Manar; Tunis, Tunisia Peter Lopez, MD; Clinical Assistant Professor; MSU; Bloomfield Hills, MI, USA
Wilfredo Lopez-Ojeda, RT(R), MS, PhD; Associate Professor; Clinical Anatomist; Kaiser Permanente School of Medicine; Pasadena, CA, USA Saran Lotfollahzadeh, MD; MD, General Surgeon, Pediatric Surgery Sub-specialist; Research Scientist; Renal and Vascular Section, Department of Medicine, Boston University School of Medicine, Boston, MA 02118, USA; Boston, MA, USA David Lynch, MD; Assistant Professor; SAMMC; Ft Sam Houston, TX, USA Joe M Das, MRCS, MCh, DNB, FRCSEd (Neurosurgery); Consultant Neurosurgeon; Bahrain Specialist Hospital, Juffair, Bahrain; Juffair, Bahrain Elizabeth Maani, MD; Assistant Professor; UT Health San Antonio / South Texas VA; Boerne, TX, USA Patricia Maani-Fogelman, DNP; Geisinger Medical Center; Danville, PA, USA Gauri Mahabal, MD; Assistant Professor; Christian Medical College; Vellore, India Kunal Mahajan, DM Cardiology; Holy Heart Advanced Cardiac Care Centre; Rohtak, India
Heba Mahdy, MBBCH, DOWH, MRCPI; MD; Toronto, Canada Haroon Javaid Majid, MBBS, FRCS; Professor of Surgery; Shaikh Zayed Medical Complex; , Pakistan Ahmad Malik, MD, MBBS; Private Practice; Brooklyn, NY, USA Jere Mammino, DO; Professor; Kansas City College of Osteopathic Med; Oviedo, FL, USA Ankit Mangla, MBBS, MD; Assistant Professor; Case Western Reserve University; Parma, OH, USA Suparna Mantha, MBBS; Gandhi Medical College; Urbana, IL, USA Sumant Mantri, DNB; Senior Consultant; Apollo Hospital Bangalore; Bangalore, India Omar Marar, MD; Assistant Professor; General and Colorectal Surgeon; Central Michigan University; Bay City, MI, USA Brian Markovich, MD; Assistant Professor; West Virginia University; Morgantown, WV, USA Kelly Mason, DO; Attending; McLaren; Pontiac, MI, USA
Thomas Mazzoni, DO, RPh; Clinical Instructor; Cooperman Barnabas Medical Center/Rutgers University; Iselin, NJ, USA Terri McHugh, DO; Attending; Medical Director, Oncology Genetics; Main Line Health Care; Wynnewood, PA, USA Brett McKeon, MD; Miami Springs, FL, USA Ritesh Menezes, MBBS, MD, Diplomate NB; Professor; College of Medicine, Imam Abdulrahman Bin Faisal University, Dammam, Saudi Arabia; Dammam, Saudi Arabia Fassil Mesfin, MD, PhD, FACS, FAANS; Practicing Physician; Director; MU School of Medicine; Columbia, MO, USA Prerna Mewawalla, MD; Assistant Professor; Assistant Director; Allegheny Health Network Cancer Inst; Pittsburgh, PA, USA David Minter, DO; MSU-McLaren Oakland; Royal Oak, MI, USA Prasenjit Mitra, MBBS, MD, PhDc; Assistant Professor; All India Institute of Medical Sciences Jodhpur; Jodhpur, India
Mouna Mlika, MD; Associate Professor; Associate Professor in Pathology; University of Medicine of Tunis; tunis, Tunisia Oranus Mohammadi, MD; Aventura Hospital and Medical Center; Miami, FL, USA Shamim Mohiuddin, MBBS, MD; Assistant Professor, Consultant; Imam Abdulrahman Bin Faisal University, Dammam; Dammam, Saudi Arabia Dulabh Monga, MD; Assistant Professor; Temple University; Pittsburgh, PA, USA James Morris, MD; Associate Professor of Medicine; Program Director; LSU Health Shreveport; Shreveport, LA, USA Fady Moustarah, MD, MPH; Associate Professor; Central Michigan University; Saginaw, MI, USA Vasudeva Ranjit Mudipalli, MD, MS; Christu Santa Rosa , San Antonio; San Antonio, TX, USA Matthew Mueller, DO, MPH; Practicing Physician; Intensivist; University of California San Diego; Washington, DC, USA
Sandeep Mukherjee, MD, MPH; Professor; Chief of Gastroenterology; Creighton University Medical Center; Omaha, NE, USA Shiva Kumar Mukkamalla, MD, MPH; Clinical Assistant Professor; Presbyterian Healthcare Services; Rio Rancho, NM, USA Francesk Mulita, MD, MSc, PhDc; General University Hospital of Patras; Patras, Greece Sanjana Mullangi, MD; Attending; Hillcrest Medical Center; Tulsa, OK, USA Sunil Munakomi, MS Surgery, MCh Neurosurgery; Associate Professor; Kathmandu University; Chitwan, Nepal Patrick Murphy, DO; Desert Regional Medical Center; Palm Springs, CA, USA Shivaraj Nagalli, MBBS, MD, FACP; Academic Hospitalist; MD, FACP; Yuma Regional Medical Center; Alabaster, AL, USA Joseph Nahas, MD; Assistant Professor; Creighton; Omaha, NE, USA
Minhthao Nguyen, DO; University Hospitals Richmond and Bedford Medical Center; Cleveland, OH, USA Hoang Nguyen, RPH, MD, PhD; Assistant Professor; NOVA Southeastern University; Clearwater, FL, USA Vinod Nookala, MD; Associate Professor; Associate Program Director; UPMC Pinnacle; Harrisburg, PA, USA Amanda Oakley, MB ChB, FRACP, PGDipHealInf; Adjunct Associate Professor; University of Auckland; Hamilton, New Zealand Eloka Okoye, MBBS, Associate Fellow, NPMCN, MRCS; Senior Clinical Fellow; Stockport NHS Foundation Trust; Manchester, United Kingdom Oluwafunmilola Okuyemi, MD, MSCI, FACS; Assistant Professor; University of Nevada Las Vegas School of Medicine; Las Vegas, NV, USA Santiago Ortiz-Perez, PhD; Consultant; University Hospital Virgen de las Nieves; Granada, Spain Jacqueline Oxenberg, DO; Geisinger Wyoming Valley; Wilkes-Barre, PA, USA Sreelakshmi Panginikkod, MD; Practicing Physician; University of Massachusetts; SHREWSBURY, MA, USA
Maansi Parekh, MBBS, DNB; Thomas Jefferson University; Philadephia, PA, USA Mayur Parmar, PhD; Assistant Professor; Nova Southeastern University; Clearwater, FL, USA Bhupendra Patel, LRCP, MRCS, MBChB, DORCS, FRCOphth, FRCS; Professor; Chief; University of Utah; Salt Lake City, UT, USA Preeti Patel, PharmD; Medical Writer; Monroe, NJ, USA Jayesh Patel, MBBS; Fellow; University of Iowa Hospitals and Clinics; Rochester, NY, USA Surabhi Pathak, MD; Kings Daughters Medical Center; Ashland, KY, USA Manju Paul, MD; Assistant Professor; SUNY Upstate Medical University; Syracuse, NY, USA Anthony Pearson-Shaver, MD, MHSA; Clinical Professor; Pediatric Residency Program Director; Nova Southeastern University; Loxahatchee, FL, USA Tom Pepper, MBBS, BDS; St George's Hospital; London, United Kingdom Christopher Pfeifer, DO; General Surgery Program Director; Henry Ford Allegiance Health; Jackson, MI, USA
Jose Pico, MD; Residency Director; Saint Lucie Medical Center; Port Saint Lucie, FL, USA Leela Sharath Pillarisetty, MD; Midland Memorial / Texas Tech University; Midland, TX, USA Rahul Polineni, MD; Attending; Bakersfield, CA, USA Charles Preuss, BSPharm, PhD; Associate Professor; University of South Florida; Tampa, FL, USA Jennifer Pruskowski, PharmD, BCPS, BCGP, CPE; Assistant Professor; University of Pittsburgh; Pittsburgh, PA, USA Yana Puckett, MD, MPH, MBA, MSc; Assistant Professor; Complex General Surgical Oncology; West Virginia University School of Medicine; Waunakee, WI, USA Sandeep Puranik, MBBS, FAAP, ATSF; Assistant Professor of Clinical Pediatrics; Director of Fellowship Education; Riley Hospital For Children, Indiana University; Zionsville, IN, USA Maitreyee Rai, MD; Crozer Chester Medical Center; Upland, PA, USA Moses Raj, MD; Associate Professor; Temple University school of Medicine; Pittsburgh, PA, USA
Avais Raja, MD, MSc; University of Western States, Portland, Oregon; Southgate, MI, USA Venkat Rajasurya, MD, MPH; Assistant Professor; SIU School of Medicine; Springfield, IL, USA Poornima Ramadas, MBBS; Assistant Professor; LSU Ochsner; Shreveport, LA, USA Kamleshun Ramphul, MBBS , MD; Shanghai Jiao Tong University; Triolet, Mauritius Michael Ramsey, MD; Director, Mohs Surgery Fellowship; Geisinger Health System; Danville, PA, USA Davinder Ramsingh, MD; Associate Professor; Loma Linda University Medical Center; Loma Linda, CA, USA Sudha Ranganathan, MD; Assistant Professor; LSU Health Sciences- UH Conway; Monroe, LA, USA Ronald Rapini, MD; Professor; Chairman; University of Texas McGovern Medical School at Houston; Houston, TX, USA Prashanth Rawla, MD; Attending; Sovah Health, Martinsville, VA; Martinsville, VA, USA Appaji Rayi, MD; Staff Neurologist; Charleston Area Medical Center; Charleston, WV, USA
John-Paul Regan, DO; Fellow; St. Joseph's Regional Medical Center; Jersey City, NJ, USA Anis Rehman, MD; Medical Director; District Endocrine/Sentara Northern Virginia Medical Center; woodbridge, VA, USA Elizabeth Reilly, MD; George Washington University Medical Center Washington, D.C.; Auckland, New Zealand Wanda Reygaert, BS, MS, PhD; Associate Professor; Oakland Un William Beaumont Sch of Med; Rochester, MI, USA Ashley Rice, DO; CUSOM/Sampson Regional Medical Center; Wilmington, NC, USA Antonio Riera March, MD; Professor; University of Puerto Rico School of Medicine; San Juan, Puerto Rico Tiffany Roberts, PhD; Assistant Professor; University of Louisville; Lanesville, IN, USA Heesun Rogers, MD, PhD; Assistant Professor; Cleveland Clinic; Cleveland Clinic, OH, USA Venkata Rokkam, MD; Assistant Professor; Banner University; Tucson, AZ, USA
Yasmyne Ronquillo, MD, MS, JD; Visiting Professor; Hoopes Vision Research Center; Draper, UT, USA Derek Rosario, MB ChB, MD FRCS(Urol); Professor; Sheffield Hallam University; Sheffield, United Kingdom Juan Rosario-Collazo, MD; Jacksonville, FL, USA Praveen Roy, MD; Practicing Physician; Albuquerque, Mohan Rudrappa, MD, FACP; Assistant Professor; Lousiana State University; Shreveport, LA, USA Hrishikesh Samant, MD; Assistant Professor; Director Hepatology; Ochsner Multiorgan Transplant Center; new orleans, LA, USA Amit Sapra, MD, FAAFP; Assistant Professor; Southern Illinois University School of Medicine; Springfield, IL, USA Richard Scharf, DO; Program Director; St Barnabas Medical Center; Roselle Park, NJ, USA George Schmieder, DO; Orange Park; Orange Park, FL, USA Donald Schultz, MD; Instructor; Associate Program Director; SAUSHEC; Fort Sam Houston, TX, USA Ari Schwell, MD; Associate Professor; Temple University; Bala Cynwyd, PA, USA
Jasjit Sehdev, MD; Assistant Professor, Attending; Cooper Rowan Un SOM, Cooper UH; Camden, NJ, USA Santosh Selvarajan, MD; Assistant Professor; Thomas Jefferson University Hospital; Mount Laurel, NJ, USA Rushi Shah, MD; University of Miami; Miami, FL, USA Hira Shaikh, MD, FACP; University of Cincinnati Medical Center; Cincinnati, OH, USA Shafeek Shamsudeen, MBBS, MS, MCh; Consultant; Consultant Surgical Oncologist; MVR Cancer Centre & Research Institute; Kozhikode, India Elena Shanina, MD, PhD; Associate Professor; University of Texas Medical Branch; Houston, TX, USA Sandeep Sharma, MD; Faculty, Attending; Mery Fitzgerald Hospital; bryn mawr, PA, USA Alex Shimanovsky, MD; Assistant Professor; Brown University; Werentham, MA, USA Muhammad Shurbaji, MD; Professor; Residency Program Director; East Tennessee State University; Johnson City, TN, USA Abdul Siddiqui, MBBS; Northwell Health SIUH; Staten Island, NY, USA
Peter Silberstein, MD; Professor; Creighton University; Omaha, NE, USA Brittany Simpson, MD; Practicing Physician; Cincinnati Children's Hospital; Cincinnati, OH, USA Gurdeep Singh, MD; Our Lady of Lourdes Memorial Hospital; Vestal, NY, USA Achint Singh, MD; Associate Professor; Un of Texas Health Sci Ctr San Antonio; San Antonio, TX, USA Shikha Singh, MD; Wyckoff Heights Medical Center; Brooklyn, NY, USA Rahulkumar Singh, MD; Associate Director; Texas Health Arlington Memorial Hospital; Arlington, TX, USA Jasvinder Singh, MBBS; Johns Hopkins Bloomberg School of Health; Hummelstown, PA, USA Daulath Singh, MD; Stormont Vail Healthcare; Lawrence, KS, USA Mayank Singhal, MBBS, MD; Hospitalist and Associate Professor; Cape Fear Valley, Methodist University; Fayetteville, NC, USA Larry Siref, MD; Associate Professor; Creighton University School of Medicine; Omaha, NE, USA
Kranthi Sitammagari, MD; Associate Professor; Methodist Un, Campbell Un School of OM; Charlotte, NC, USA Travis Smith, DO; Clinical Dean; LECOM/ St. Vincent's Southside; Ponte Vedra, FL, USA Guillermo Solis-Ledesma, MD; Associate Professor; Chief of Dermatopathology; Instituto Dermatologico de Jalisco "Dr. Jose Barba Rubio"; Zapopan, Mexico Sidharth Sonthalia, MD, DNB, MNAMS, FISD; Senior Consultant; SKINNOCENCE: The Skin Clinic; Gurugram, India Vijay Srinivasan, MD; Fellow; Aventura Hospital and Medical Center; Aventura, FL, USA Christopher Stang, DO; Conway Med. Center/Campbell University; Conway, SC, USA Robert Steele, MD; Geisinger Health System; Danville, PA, USA James Stevens, MD; Morganton, Kristen Stewart, MD; Baptist Medical Center; Jacksonville, FL, USA
Kristin Sticco, DO, MS; Stony Brook University Hospital; Westbury, NY, USA Michael Stratton, MD; Clinical Assistant Professor; Program Director; LSU Shreveport Health Sciences Center; Shreveport, LA, USA Manuel Suarez, MD; Westchester General Hospital; Aventura, FL, USA Surabhi Subramanian, MD; IWK Health Centre; Halifax, Canada Anupam Sule, MD, PhD, FACP; Assistant Professor; Medical Director; St Joseph Mercy Oakland; Pontiac, MI, USA Scott Surowiec, MD; Assistant Professor; Upstate Medical University; Syracuse, NY, USA Wajihuddin Syed, MBBS, MD; Chief of Medical Oncology and Hematology; Franciscan Health Crown Point; Crown Point, IN, USA Prasanna Tadi, MD; Director; Asram Medical College, Eluru, India; Elkhorn, _, USA Laura Tanner, MS, MD; Private Practice; Wilmington, NC, USA
Melissa Tavarez, MD, MSEd; Assistant Professor; Un of Pittsburgh School of Medicine; Glenshaw, PA, USA Pramod Theetha Kariyanna, MD; State Un of New York Downstate MC; Brooklyn, NY, USA Claudwardyne Thevenin, DO; Albany Medical Center; Schenectady, NY, USA Bruce Thiers, MD; Medical University of South Carolina; Charleston, SC, USA Nishitha Thumallapally, MD; Associate Professor; Conway Medical Cancer Center, Duke Health Affiliate; Myrtle beach, SC, USA Richa Tikaria, MD; Assistent Professor; MSU; Okemos, MI, USA Koushik Tripathy, MBBS, MD, FRCS; ASG Eye Hospital, BT Road, Kolkata, India; Kolkata, India Faiz Tuma, MD, MEd, EdS; Associate Professor; Central Michigan University; Saginaw, MI, USA Aung Tun, MBBS; Division of Hematology, Mayo Clinic, Rochester, MN; Rochester, MN, USA Prabhakar Vaidya, MD; Associate Professor of Medicine; Campbell University School of OM; Cary, NC, USA
Matthew Varacallo, MD; Chief of Sports Medicine; Director of Orthopedic Robotic Surgery; Penn Highlands Healthcare System; DuBois, PA, USA Joseph Varon, MD; Professor; Chairman of the Board, Chief of Staff; The University of Texas Health Science Center at Houston, United General Hospital; Houston, TX, USA Roopma Wadhwa, MD, MHA; South Carolina Department of Mental Health; Columbia, SC, USA Jason Wallen, MD; Associate Professor; Upstate Medical University; Syracuse, NY, USA Andrew Walls, MD; Assistant Professor; San Antonio Military Medical Center; Fort Sam Houston, TX, USA Steven Warrington, MD, MEd; Practicing Physician; Chair; Kaweah Delta Medical Center; Dakota Dunes, SD, USA James Waymack, MD; Assistant Professor; Program Director; Southern Illinois University; Springfield, IL, USA Yurong Wheeler, MD, PhD; Clinical Associate Professor; Watauga Pathology Associates/ETSU; Johnson City, TN, USA
Nicole Winston, MS, PharmD, EdS; Assistant Professor; Assistant Director of CII; Augusta University Medical College of Georgia; Augusta, GA, USA Ryan Winters, MD; Associate Professor; Ochsner Health System; New Orleans, LA, USA Patrick Wolf, MD; Clinical Professor; University of Tennessee-Nashville; Brentwood, TN, USA Andrew Woolf, DO; Fellow; Mclaren Greater Lansing Hospital, MSU; Holt, MI, USA Brent Yeung, MD; Assistant Professor; UC Irvine; Corona Del Mar, CA, USA Sarah Yonder, MD; Assistant Professor; Course Director; Central Michigan University College of Medicine; Sanford, MI, USA Farah Zahra, MD; Northwestern Mchenry/RFMUS; South Barrington, IL, USA Julio Zayas, MD; Aventura Hospital and Medical Center/FIU; Miami, FL, USA Patrick Zito, DO, PharmD; Voluntary Assistant Professor; University of Miami; Miller School of Medicine; Fort Lauderdale, FL, USA
Hassam Zulfiqar, MBBS; Practicing Physician; Rawalpindi Medical University & Allied Hospitals; Islamabad, Pakistan
Authors Abdullah Abdussalam, MD, MBA; Fellow; Creighton University; Omaha, NE, USA Shogik Abramyan, MD; Resident; Upstate University; Syracuse, NY, USA Mohammed Abu Zaanona, MD; Resident; Carle Foundation Hospital/ University of Illinois at Urbana Champaign; Urbana, IL, USA Heath Ackley; Private Practice; , NY, USA Robert Acosta, MD; Assistant Professor; Jacobi Medical Center; Bronx, NY, USA Chad Adams, MD; Brooke Army Medical Center; San Antonio, TX, USA Adebayo Adeyinka, MD; Assistant Professor; The Brooklyn Hospital Center; Brooklyn, NY, USA Rotimi Adigun, MBBS, MPH; Associate Professor; Director of Pathology Program; University of Health Sciences, Antigua; St John's, Antigua & Deps Surya Aedma, MD; KUMC; Kansas City, KS, USA
Manuj Agarwal, MD; Assistant Professor; University of Pennsylvania; Glen Mills, PA, USA Priyal Agarwal, MD; Kasturba Medical College Manipal, India; Lansing, MI, USA Blessing Aghedo, MDc; American University of Barbados School of Medicine; Columbia, MD, USA Mudhasir Ahmad, MBBS, MD, DNB; Private Practice; New Delhi, India Arooj Ahmed, MD; Resident; Western Reserve Health Education/NEOMED; Youngstown, OH, USA Mahmoud Al Aaraj, MD; Fellow; Barking Havering & Redbridge University; Romford, United Kingdom Tamim Alabduladhem, MDc; Imam Abdulrahman Bin Faisal University; Dammam, Saudi Arabia Ghassan Alaranji, RCPI, MPH, FRACP; University of Auckland; Hamilton, New Zealand Sara Albagoush, MD; Creighton University; Omaha, NE, USA Hira Ali, MBBS; Ziauddin University; Karachi, Pakistan Muhammad Ashar Ali, MD; Resident Physician; Saint Clare's Denville Hospital; Denville, NJ, USA
Syed Awab Ali, MBBS, FCPS-1; Dr.; Mohsin Hospital Qamar Sialvi Road Gujrat; Gujrat, Pakistan Fadi Alkabban, MD; Iraqia University; Baghdad, Iraq Aimee Almanzar, MD; Aventura Hospital and Medical Center; Hallandale Beach, FL, USA Patricia Almeida, DO; Fellow; Aventura Hospital and Medical Center; Miami, FL, USA Mohammad Almomani, MBBS; MBBS; Jordanian Royal Medical Services; Amman, Jordan Asayel Alruwaili, BMS; Imam Abdulrahman bin Faisal University; Dhahran, Saudi Arabia Alfarooq Alshaikhli, MD; Internal Medicine Resident; University of Texas/ Rio Grande Valley at DHR; Houston, TX, USA Abbas Alshami, MD; Jersey Shore University Medical Center; Neptune, NJ, USA Ibrahim Altabakhi, MD; Private Practice; Amman, Jordan Sirhan Alvi, MBChB, FRCS(ENT); Registrar; Aintree University Hospital; Liverpool, Saudi Arabia Ragia Aly, MD; Attending; Danbury Hospital; Danbury, CT, USA
Muhammad Atif Ameer, MD; Resident; Suburban Community Hospital; Norristown, PA, USA Hiral Amin, MD; PGY-5; Resident; Bronx Care Health System; Bronx, NY, USA Amir Aminlari, MD; University of California, San Diego Health; , _, USA Mohammad Ammad Ud Din, MD; Rochester General Hospital; Rochester, _, USA Mark Amsbaugh, MD; University of Louisville; Louisville, KY, USA Claudio Andaloro, MD; Santa Marta e Santa Venera Hospital; Catania, Italy Aabha Anekar, MBBS; Mysore Medical College; Mysuru, India Fatima Anjum, MD; Clinical Associate Professor of Medicine; Hofstra University, Zucker School of Medicine; Manhasset, NY, USA Ibrar Anjum, MBBS, MD; University of Health Sciences College of Akhter Saeed; NY, NY, USA Pavan Annamaraju, MD; Assistant Professor; Loma Linda University; Abingdon, VA, USA
Marsha Antoine, MD; Private Practice; philadelphia, PA, USA Ayesha Anwar, MBBS; Allama Iqbal Medical College; West Henrietta, NY, USA Rahul Arora, MD Fellowship Palliative Med (India); Delhi State Health Mission; Bahrain Oncology Center, King Hamad University Hospital, Bahrain, India Taruna Arora, MD; PGY-3; Resident; Saint Vincent Hospital; Worcester, MA, USA Juwairiya Arshi, MD; Resident; University of Missouri at Columbia; Columbia, MO, USA Christopher Arteaga, MD; Resident; University of Arizona Banner University Medical Center Tucson; Tucson, AZ, USA Kwabena Asafo-Agyei, MBCHB; Chrisrtus Highland Medical Center; Shreveport, LA, USA Damilola Ashorobi, MD; Endocrinology Fellow; Nassau University Medical Center; East Meadow, NY, USA Nadja Auerbach, MBBS, BSc; Broomfield Hospital; London, United Kingdom
Samy Azer, MD, PhD, MEd, FACG, MPH, FRSM; Professor; Consultant; King Saud University, King Khalid UH; Riyadh, Saudi Arabia Muhammad Aziz, MD; Instructor; Fellow; University of Toledo Medical Center; Toledo, OH, USA Laila Babar, MBBS; AGH; Pittsburgh, PA, USA Hani Babiker, MD; Assistant Professor; Director; University of Arizona Cancer Center; Tucson, AZ, USA Madhu Badireddy, MD; Internal Medicine Hospitalist; Christus Santa Rosa Hospitals; San Antonio, TX, USA Syeda Bahar, MBBS; Resident; Lady Reading Hospital; NA, Pakistan Dana Baigrie, DO; Campbell University School of Medicine; Wilmington, NC, USA Ryan Bailey, DO; Resident; PBCGME / St Lucie Medical Center; Port Saint Lucie, FL, USA Tushar Bajaj, MD; Practicing Physician; University of Chicago; Chicago, IL, USA Arthur Balin, MD, PhD; Medical Director; Media, PA, USA Brian Ballard, DO; Resident; McLaren Macomb Hospital; Rochester, MI, USA
Hiral Banker, MBBS,MD; Le Bonheur Children's Hospital, UTHSC; Memphis, TN, USA Pankaj Bansal, MBBS, MD, RHMSUS, CCD, FACP; Assistant Professor; Mayo Clinic Health System; Eau Claire, WI, USA Jacqueline Baude, BA; University of Nebraska Medical Center; Papillion, NE, USA Samuel Bechmann, MD; Associate Program Director; New Jersey Medical School; Brooklyn, NY, USA Andrea Bell, DO; Michigan State University College of Osteopathic Medicine; Westlake, Brooke Bell, MD; San Antonio Uniformed Services Health Education Consortium; San Antonio, TX, USA Mohammed Ben Aziz, MD; Deputy Chief; ULB: Institut Jules Bordet (cancer research); Bruxelles, Belgium Conrad Benedetto, DO; Western University of Health Sciences; Newport Beach, CA, USA Courtney Bernett, DO; Private Practice; Green Cove Springs, FL, USA Michael Beshara, MBChB; Other; New York, GA, USA
Jatin Bhatia, MBBS, DORL, ECFMG, MD, PDCR; Rural Medical College; Pune, India Mamta Bhatnagar, MD, MS; Assistant Professor; Medical Director; UPMC; Pittsburgh, PA, USA Harshil Bhatt, MD; Goshen Hospital; Goshen, IN, USA Ruchi Bhatt, DO; Nassau University Medical Center; Bronx, NY, USA Prianka Bhattacharya, MD; Lankenau Medical Center; Wynnewood, PA, USA Poonam Bhyan, MD; Practicing Physician; Fayetteville, Michael Bishop, DO; Resident; Mercy St. Vincent Medical Center; Holland, OH, USA Karlyle Bistas, BSc; Medical University of the Americas; St. Catharines, Canada Michael Bodine, DO; Orthopedic Surgery Resident; Michigan State University - McLaren Macomb Hospital; Chesterfield, MI, USA Myron Bodman, DPM; Associate Professor; Kent State University CPM; Fairview Park, OH, USA Srinivas Bojanapu, MBBS, MS; Sir Ganga Ram Hospital, New Delhi; Bengaluru, India
Michael Bono, MD; Professor of Emergency Medicine; Vice Chair, Department of Emergency Med; Eastern Virginia Medical School; Suffolk, VA, USA Philip Bouchette, MD; Louisiana State University HSC; Shreveport, LA, USA Josiah Brandt, MDc; University of Tennessee Health Science Center; Memphis, TN, USA Clayton Brittingham, DO; Walter Reed National Military Med Center; Fairfax, VA, USA Melina Brizuela, BDS; Advanced Integrated Dentistry; International University Of Catalunya (UIC); Barcelona, Spain Falon Brown, DO; CUSOM/Sampson Regional Medical Center; Wilmington, NC, USA Mikayla Brown, BA; Practicing Physician; MD; Stony Brook University School of Medicine; Vestal, NY, USA Christopher Buckley, JD; University of Nebraska Medical Center; Omaha, NE, USA Deepa Budh, MD; Saurashtra University - MP Shah Medical College; Bronx, NY, USA
Serghei Burcovschii, MD; Kern Medical; Bakersfield, CA, USA Matthew Burton, MD; University of Missouri Healthcare; Columbia, MO, USA Kiran Butt, MBBS, FCPS; Rawalpindi Medical University; Rawalpindi, Pakistan Wajeeha Saeed Butt, MD; Hospitalist; JFK Medical Center; West Palm Beach, FL, USA Colin Byrd, DO; McLaren Oakland; Shelby Township, MI, USA Christopher Calandrella, DO; Assistant Professor; Associate Chairman; North Shore Manhasset; Manhasset, NY, USA Miles Campbell, BS; CUSOM; Eden Prairie, MN, USA Anthony Capria, MD; Nassau University Medical Center; Floral Park, NY, USA Stephanie Carr, DO, MBA; Marshfield Medial Center; Marshfield, WI, USA Jake Carter, MD; Practicing Physician; Syracuse, Jarett Casale, DO; Resident physician; PGY-3; Campbell University School of Osteopathic Medicine; Cary, NC,
USA Jesse Casaubon, BS, DO; Practicing Physician; Brooklyn, Marco Cascella, MD; Professor; Istituto Nazionale Tumori - IRCCS - Fondazione Pascale, Via Mariano Semmola 80100, Napoli. Italy; Napoli, Italy Murray Casey, MD, MS, MBA, PhD; Professor; Creighton University; Omaha, NE, USA Mary Cataletto, MD; Professor; NYU Winthrop University Hospital; Brookville, NY, USA Juan Cedeno, MD; Resident; Mount Sinai Medical Center; Miami, FL, USA Rebanta Chakraborty, MD; Attending; Steward Medical\Good Samaritan Medical; Watertown, MA, USA Onyee Chan, MD; Un of AZ, Banner Un Medical Center; Tucson, AZ, USA Ahmad Charifa, MD; Yale School of Medicine; New Haven, CT, USA Hammad Chaudhry, MBBS; Private Practice; Mirpur, Adithya Chennamadhavuni, MBBS; Oncology Fellow; University of Iowa; Coralville, IA, USA Anusha Chidharla, MD; UICOMP; Overland Park, IL, USA
Venu Chippa, MD; Assistant Professor; Indiana university; Newburgh, IN, USA Shalini Chiravuri, MD; Hospitalist; Conemaugh Memorial Medical Center; Johnstown, PA, USA Min Cho, DO; McLaren Greater Lansing; Lansing, MI, USA Shashank Cingam, MD; Practicing Physician; University of New Mexico Comprehensive Cancer Center; Albuqueruque, LA, USA Shane Clark, DO; Private Practice; Holt, MI, USA Megan Cochran, DO; Doctors Hospital - Ohio Health; Galloway, OH, USA Philip Cohen, MD; Professor; Touro University California College of Osteopathic Medicine; San Diego, CA, USA Jeffrey Cooper, MD; Associate Professor; Director; University of Nebraska Medical Center; Omaha, NE, USA Danielle Cooper, MD; Associate Professor; Residency Program Director; LSU Health Sciences Center; Shreveport, LA, USA Jonathan Crane, DO; Dermatology Residency Program Director; Sampson Regional Med Ctr / Campbell Univ; Wilmington, NC, USA
David Crotzer, MD, MS; Clinical Assistant Professor; Creighton University; Omaha, NE, USA Brian Cuzzo, BS, BA; Private Practice; Syracuse, NY, USA Meghan Czajka, DO; Henry Ford Allegiance Hospital; Jackson, MI, USA Rajeev Dalal, DO; Resident Physician; Penn State Health Milton S. Hershey Medical Center; York, PA, USA Catriona Daly, MBBS; University College Dublin, Ireland; Taunton, United Arab Emirates Heeransh Dave, MD; Tver State Medical University, Tver, Russian Federation; Aventura, FL, USA Donald Davis, MD; Wellstar Atlanta Medical Center; Duluth, GA, USA Orlando De Jesus, MD; Professor; University of Puerto Rico, Medical Sciences Campus, Neurosurgery Section; San Juan, Puerto Rico Jay Desai, MBBS; Clinical Research Associate; University Of Tennessee Health Science Center; Memphis, TN, USA Bhavana Devanabanda, MDc; Private Practice; New York, Arvind Reddy Devanabanda, MD; Fellow; Zucker School of Medicine; New York, NY, USA
Richard Dittrich, BSc, MUDr; Southend University Hospital NHS Foundation Trust; Southend-on-Sea, United Kingdom Mehdi Djekidel, MD; Division Lead Nuclear Medicine; Sidra Medicine; , MI, USA Chaddie Doerr, BSN, MS, CRNA; Clinical Instructor; University of Louisville; Louisville, KY, USA Laurena Dongmo Fotsing, MDc; Private Practice; San Jose, CA, USA Annabelle Dookie, DPM; Assistant Professor; DPM; Dr. William M. Scholl College Podiatric; Libertyville, IL, USA Jennifer Dotson, DO; Marshall University; Catlettsburg, KY, USA Doug Duffee, MD, Mdiv; ADJ Associate Professor; Rocky Vista University and Parkview Medical Center; Pueblo, CO, USA Scott Dulebohn, MD; Practicing Physician; ABCD; Johnson City, Scott Dulebohn, MD; Practicing Physician; Johnson City, NJ, USA Anne Dulski, DO; Kent Hospital; Warwick, RI, USA
Alper Duran, MD; UTMB; Galveston, TX, USA Ceren Durer, MD; SUNY Upstate Medical University Hospital; Syracuse, NY, USA Seren Durer, MD; SUNY Upstate Medical University; Syracuse, NY, USA Rina Eden, DO; Brooke Army Medical Center; Ocean Springs, MS, USA Nimra Ehsan, MBBS; Khyber Medical University, Peshawar, Pakistan; Peshawar, Pakistan Abbinaya Elangovan, MD; Case Western Reserve / MetroHealth; Parma, OH, USA Ahmed Elmaaz, MD; Resident; New York Medical College- Metropolitan Hospital; New York, NY, USA Anna Elseth, DO; Resident; Dwight D. Eisenhower Army Medical Center; Ft. Gordon, GA, USA Ghada Elshimy, MD; Fellow; University of Arizona; Phoenix, AZ, USA Joshua Engelsgjerd, MD; University of Nebraska Medical Center; Omaha, NE, USA Jake Fagan, MD; Geisinger Health System; Danville, PA, USA
Fabiola Farci, MD; Pisa University; S.G. Monreale, Italy Nicole Farha, BS; University of Kansas School of Medicine; Wichita, KS, USA Maria Fariduddin, MBBS; MD at Home; Chicago, IL, USA Mustafa Farooq, MDc; Medical College of Georgia at Augusta University; McDonough, GA, USA Mary Fatehi, MD; Attending; Director, Gynecologic Oncology; Nassau University Medical Center; East Meadow, NY, USA Rawish Fatima, MBBS; Private Practice; Kansas City, MO, USA Dalton Fazekas, MD; University of Miami / Jackson Memorial Hospital; Miami, FL, USA Ricardo Fernández-de Thomas, MD; Resident; University of Puerto Rico, Medical Sciences Campus, Neurosurgery Section; Guaynabo, Puerto Rico James Flynn, MD; California Northstate University; Danville, CA, USA Michael Foss, BS; Private Practice; Gainesville, FL, USA Michaelia Fosses Vuong, DO; Southern Illinois University; Springfield, IL, USA
Josephine Fowler, MD, MSc; Associate Professor; Vice Chair of Clinical Services; University of Massachusetts; Marlborough, MA, USA Ryan Gall, MD; NCC; Rockville, MD, USA Mounika Gangireddy, MD; UPMC pinnacle; York, PA, USA Karen Garfield, MD, MS, BA; Mount Sinai Hospital; New York, NY, USA Sriram Garikapati, MDc; St. George's University School of Medicine; Queens Village, NY, USA Craig Garofola, DO; Lewis Gale Montgomery; Christiansburg, VA, USA John Gay, DO; Lewis Gale Hospital Montgomery; Blacksburg, VA, USA Valerie Gerriets, PhD; Assistant Professor; California Northstate University College of Medicine; Elk Grove, CA, USA Cynthia Giraldo, MD; Practicing Physician; Dr.; San Antonio, Joann Gold, BS, BA; FIU/Herbert Wertheim College of Medicine; Coral Gables, FL, USA
Mario Gonzalez, MD; University of Puerto Rico School of Medicine Department of Otolaryngology Head and Neck Surgery; San Juan, Puerto Rico Luis Gonzalez-Mosquera, MD; Nassau University Medical Center; Mineola, NY, USA Sherry Gossman, BSN; Clinical Instructor; Point Loma Nazarene College San Diego, CA; Elkhorn, NE, USA William Gossman, MD; Professor; Creighton University; Omaha, NE, USA Brittney Graham, DO; BAMC/ SAMMC; San Antonio, TX, USA Keva Green, MD; West Virginia University; morgantown, WV, USA Karl Greenblatt, BA; Private Practice; Cincinnati, OH, USA Joshua Greene, MD; Henry Ford Health System; Detroit, MI, USA Ashley Greenwood, BSc, MD; University of Nebraska Medical Center; Omaha, NE, USA W Grimes, MD; Clinical Associate Professor; LSU School of Medicine Shreveport; Shreveport, LA, USA
James Griscom, MD; University of Tennessee/St. Thomas; Nashville, TN, USA Dayton Grogan, MDc; Medical College of GA; Griffin, GA, USA Paul Gruber, MD; Private Practice; Saint Louis, MO, USA Sai Giridhar Gundepalli, MBBS; Private Practice; Omaha, NE, USA Mounika Gunduru, MD; Wayne State University / Detroit Medical Center; West Bloomfield, MI, USA Neha Gupta, BDS, DMD, GPR; Private Practice; Westlake, OH, USA Bharat Gurnani, MBBS, DNB, FCRS, FICO, MRCS Ed, MNAMS; Practicing Physician; Dr; Aravind Eye Care System; Amritsar, India Kevin Guthmiller, MD; Assoc Prof of Clinical Anesthesiology; Director, Inpatient Pain Service; USC Keck School of Medicine; Los Angeles, CA, USA Aida Haddad, MD; Indiana University School of Medicine; Indianapolis, IN, USA Jessica Hafner, DO; Holland Hospital; Hamilton, MI, USA
Wissem Hafsi, MD; University Assistant; Faculty of Medicine of Tunis, Tunisia; Bardo, Tunisia Kelly Hall, MD; Fellow; University of Texas Health Science Center at Houston; San Antonio, TX, USA Hussein Hamad, MD; Clinical Assistant Professor; University Hospitals/Case Western Univ; Ashland, OH, USA Sahar Hamrang-Yousefi, MBBS, BSc, MRCS (DOHNS); Hammersmith Hospital, London; London, United Kingdom Angela Han, DO; Private Practice; Huntingdon Valley, PA, USA Caroline Hana, MD, MSci; University of Miami; Boynton Beach, FL, USA Nazma Hanif, MBBS; Practicing Physician; Oakbrook Terrace, Mawiyah Haq, MBBS; St. George's, University of London Medical School, London, UK; London, United Kingdom Blair Harris, DO; Resident; Pacific Northwest University; Wilmington, NC, USA Vishwajit Hegde, MBBS; Private Practice; Kumta, India
Jonathan Heistein, MD; Private Practice; Fort Worth, TX, USA Stephen Hendriksen, MD; Hennepin County Medical Center; Minneapolis, MN, USA Vincent Herlevic, MD; PGY - 6; Gastroenterology Fellow; Ochsner LSU Shreveport; Shreveport, LA, USA Nicholas Herold, DO; PGY1; Resident; Univeristy of Central Florida; Redondo Beach, CA, USA Gregory Hildebrand, BS; University of Kansas Medical Center; Kansas City, KS, USA Asif Hitawala, MD; Cleveland Clinic Foundation; Cleveland, OH, USA Bonnie Hodge, BS; University of Mississippi Medical Center; Jackson, MS, USA Christopher Hogan, MBBS, BSc; West Middlesex Hospital; London, United Kingdom Nicholas Horan, MD; Carolinas Rehabilitation; Elkhorn, NE, USA Dylon Howard, DO; Private Practice; Clinton, NC, USA Ronald Hsu, MD; Professor; Univ of Calif, Davis, School of Medicine; Roseville, CA, USA
Marshall Huang, MD; University of Utah; Salt Lake City, UT, USA Martin Huecker, MD; Associate Professor; Research Director; University of Louisville; Louisville, KY, USA Jaime Huffman, PhD, RN; Associate Professor; University Center, MI, USA Melanie Hundt, BA; SUNY Upstate Medical University; Syracuse, NY, USA Madiha Huq, DO, MS; PGY-3; University Hospitals; Brunswick, OH, USA Phillip Hynes, MD; Private Practice; Lincoln, NE, USA Michele Iguina, MD; Aventura Hospital and Medical Center; Pembroke Pines, FL, USA Asad Ikram, MD; Fellow; University of New Mexico; Albuquerque, NM, USA Qurrat Ul Ain Iqbal, MBBS; Shaikh Zayed Hospital; Lahore, Pakistan Jumping Jack, MD; Associate Professor; MD; M.S University; Vadodara, India Sumeet Jain, MD, MBA; Upstate Medical University; Syracuse, NY, USA
Manisha Jain, MBBS; MUHS, Nashik; THANE(W), India Zohaib Jamal, MBBS; Rawalpindi Medical University, Rawalpindi; Rawalpindi, Pakistan Asma Jamil, MBBS, MD; Riverside Community Hospital; Riverside, CA, USA Apoorva Jayarangaiah, MD; Assistant Professor; Wake Forest University; Brooklyn, NY, USA Suman Jha, MBBS; B.P. Koirala Institute of Health Sciences; Mahottari, Nepal Donavon Johnson, MD, BBA; Ross University; Regina, Canada Kelly Johnson-Arbor, MD; MedStar Georgetown University Hospital; Washington, DC, USA Zachary Joos, MD; Un of Utah Hosp, John A Moran Eye C; Salt Lake City, UT, USA Rebecca Jordan, DO; Geisinger; Wilkes Barre, PA, USA Catherine Joyce, BS; University of Kansas School of Medicine; Overland Park, KS, USA Felix Jozsa, MBBS, BMus; Imperial College NHS Healthcare Trust; London, United Kingdom
Angel Justiz Vaillant, MD, PhD; University of the West Indies; St. Augustine, Trinidad & Tobago Norah Kairys, MD; Temple University Hospital; Philadelphia, PA, USA Feroze Kaliyadan, MD, DNB,FRCP (London), European Board DV; Assistant Professor; King Faisal University; Hofuf, Saudi Arabia Arjun Kalra, MD; Central Michigan University College of Medicine; Ann Arbor, MI, USA Brian Kaltenecker, DO, MS; Campbell University Cape Fear Valley; Fayetteville, NC, USA Payvand Kamrani, BS; Philadelphia College; Chapel Hill, NC, USA Tejaswi Kanderi, MD; UPMC Pinnacle; Camp Hill, PA, USA Vijaya Kankanala, MBBS, MD; Medical Director; Presbyterian Hospital; Albuquerque, NM, USA Venkata Kanukollu, MBBS, MS, FRCS; Associate Professor; Darlington Memorial Hospital; Rajahmundry, India Erica Kao, MD; Brooke Army Medical Center; San Antonio, TX, USA
Mayank Kapoor, MBBS; Practicing Physician; All India Institute of Medical Sciences (AIIMS) Rishikesh; Uttarakhand, India Hatem Kaseb, MD, PhD, MPH; Cleveland Clinic; Beachwood, OH, USA Sarang Kashyap, MD, MBBS; Assistant Professor of Surgery; Beckley Appalachian Regional Healthcare Hospital; Mt Hope, WV, USA Varidh Katiyar, MBBS, MCh; Post Mch Fellow; All India Institute of Medical Sciences; Delhi, India Mohammad Kattuoa, MBBS; Resident; Yarmouk University; Amman, Jordan Alysia Kemp, MD; Attending; Karmanos Cancer Institute Wayne State University; Mt Clemens, MI, USA Meghana Kesireddy, MBBS, MD; University of Texas Medical Branch; Galveston, TX, USA Karam Khaddour, MD; Rosalind Franklin University of Medicine; McHenry, IL, USA Niloufar Khanna, MDc; California Northstate University; Cupertino, CA, USA
Swapnil Khare, MD; Assistant Professor; Indiana University; Carmel, IN, USA Ahmed Khattab, MD; Practicing Physician; Pittsburgh, Jakub Khzouz; Private Practice; Madaba, Jordan Ariel Kidron; Nova Southeastern College of Osteopathic Medicine; Aventura, FL, USA Robert Killeen, MD; University of Illinois; Holiday, FL, USA Catherine Kim, MD; Penn Medicine and Virtua; Voorhees, NJ, USA Felisha Kitchen, MD; Other; Coliseum Medical Center; Macon, GA, USA James Knight, MBBS, BA; Specialist Registrar; King's College Hospital London; Guildford, United Kingdom Logan Kolb, DO; Private Practice; Jacksonville, FL, USA Sam Konkol, MD; Resident; University of Virginia; Charlottesville, VA, USA Antony Koroulakis, MD; University of Maryland; Potomac, MD, USA Michael Kortz, DO, MBA; Practicing Physician; Dr.; University of Colorado Department of Neurosurgery;
Centennial, CO, USA Kassie Kostecki, DO; Private Practice; Wellington, FL, USA Sean Kostolansky, DO; Private Practice; Springfield, IL, USA Andrew Kowalski, DO; Mclaren Macomb; Saint Clair Shores, MI, USA Kevin Krauland, MD; Assistant Professor; Associate Program Director; Brooke Army Medical Center; Seguin, TX, USA Mridula Krishnan, MBBS; Creighton University; Omaha, NE, USA Marissa Krugh, DO; Campbell University; Holly Springs, NC, USA Pujitha Kudaravalli, MBBS; Private Practice; Syracuse, NY, USA James Kuhn, MD; Aventura Hospital and Medical Center; Miami, FL, USA Matthew Kuivenhoven, DO; McLaren Oakland; Clawson, MI, USA
Rahul Kumar, MBBS; Sharda Hospital; Greater Noida, India Anila Kumar, DO; Westchester Medical Center; Tarrytown, NY, USA Heidi Kurn, DOc; Midwestern University - CCOM; Chicago, IL, USA Geoffrey Kwitko, MD; Assistant Clinical Professor; University of South Florida; Tampa, FL, USA Harry Kyriacou, MDc; University of Cambridge; Ilford, United Kingdom Asad Lak, MD; Post-Doctoral Research Fellow; Brigham and Women's Hospital, Harvard Medical School; Boston, MA, USA Anish Lamichhane, MD; PGY 1; Resident; Lincoln medical and mental health center; New york, NY, USA Haley Lanser, DO; Private Practice; Clawson, Tasnim Lat, DO; Baylor Scott and White MC - Temple; Temple, TX, USA Mark Law, MD; Associate Professor; Childrens of AL, Un of AL at Birmingham; Vestavia Hills, AL, USA
Cuong Le, DO; Practicing Physician; DO; Hackensack University Medical Center- Palisades; North Bergen, NJ, USA Anna Lee, MD, MPH; Private Practice; Brooklyn, NY, USA Manidhar Reddy Lekkala, MBBS; University of Rochester Medical Center; Rochester, NY, USA Stephen Leslie, MD; Associate Professor of Surgery, Urology; Creighton University Medical Center; Omaha, NE, USA Haley Lewis, DO, MPH; North Fulton; Atlanta, GA, USA Lanxin Li, MD; Stony Brook University Hospital; Nesconset, NY, USA Dawn Light, MD, MPH; Associate Professor; WSU-SOM, USUHS, Dayton Childrens Hosp; Dayton, OH, USA Faten Limaiem, MBBS; Assistant Professor; Associate Professor; University of Tunis El Manar, Tunis Faculty of Medicine; Tunis, Tunisia Noureddine Litaiem, MD; University of Tunis El Manar; Tunis, Tunisia Ajeet Kumar Lohana, MBBS; Instructor; Aga Khan University Hospital; Karachi, Pakistan
Richard Lopez, DO, BME; Director Trauma & Surgical Critical Care; Geisinger Medical Center; Wilkes-Barre, PA, USA Saran Lotfollahzadeh, MD; MD, General Surgeon, Pediatric Surgery Sub-specialist; Research Scientist; Renal and Vascular Section, Department of Medicine, Boston University School of Medicine, Boston, MA 02118, USA; Boston, MA, USA Dhruv Lowe, MD; SUNY Upstate Medical University; Syracuse, NY, USA Jacqueline Lykstad, BS; Midwestern University - AZCOM; Scottsdale, AZ, USA Joe M Das, MRCS, MCh, DNB, FRCSEd (Neurosurgery); Consultant Neurosurgeon; Bahrain Specialist Hospital, Juffair, Bahrain; Juffair, Bahrain Oswaldo Madrid, MD; Universidad Central de Venezuela Caracas, Venezuela; Springfield, IL, USA Samuel Magny, MD; Harbor UCLA Medical Center; Torrance, CA, USA Abigail Magovern, MD; Royal College of Surgeons in Ireland; Pittsburgh, PA, USA
Heba Mahdy, MBBCH, DOWH, MRCPI; MD; Toronto, Canada Keri Maher, MS, DO; University of Arizona; Tucson, AZ, USA Ejaz Mahmood, MBBS; Private Practice; Philadelphia, PA, USA Hafsa Majeed, MD; Internal Medicine Resident; Icahn School of Medicine at Mount Sinai, Elmhurst Hospital; Jackson Heights, NY, USA Vidit Majmundar, MBBS; Civil Hospital Ahmedabad; Ahmedabad, India Robert Maldonado, MD; Resident; Georgetown University Hospital - National Rehabilitation Hospital; Washington, DC, USA Ahmad Malik, MD, MBBS; Private Practice; Brooklyn, NY, USA Bilal Malik, MD, H.BSc; MSU/ Mclaren Flint; , MI, USA Fnu Manas, MBBS; Griffin Prevention Research Center; Inverness, IL, USA Ankit Mangla, MBBS, MD; Assistant Professor; Case Western Reserve University; Parma, OH, USA
Nicholas Mankowski, BS; Tulane University School of Medicine; New Orleans, LA, USA Biagio Manna, DO; RWJUH/Barnabas Health System; Mercerville, NJ, USA Trina Mansour, MD; UC San Francisco; Winchester, CA, USA Edward Margolin, MD; Associate Professor; University of Toronto; Toronto, Canada Julianna Martel, BA; Tulane University School of Medicine; New Orleans, LA, USA Wajeed Masood, MD, FACP, CHCQM-PHYADV; Assistant Professor; Medical Director; Methodist University, Campbell University School of Osteopathic Medicine.; Fayetteville, NC, USA Samip Master, MD; Assistant Professsor; Louisiana State University HSC; Shreveport, LA, USA Christopher Mayer, MD, MS; Resident; Housestaff I; Albany Medical College; Los Angeles, CA, USA Miguel Mayol Del Valle, MD; Resident; Chief Resident; University District Hospital, Puerto Rico Medical Center; Guaynabo, Puerto Rico
Hiba Mazhar, MB, BCh, BAO; Private Practice; Cleveland Heights, OH, USA Thomas Mazzoni, DO, RPh; Clinical Instructor; Cooperman Barnabas Medical Center/Rutgers University; Iselin, NJ, USA Brianna McDaniel, DO; Sampson Regional Medical Center; Wilmington, NC, USA Miguel Medina Pabón, MD; University Industrial Santander / University CES; Medellin, Colombia Anthony Mercadante, BA; University of Yale; Jupiter, FL, USA Marcelle Meseeha, MD; Private Practice; Sayre, PA, USA Fassil Mesfin, MD, PhD, FACS, FAANS; Practicing Physician; Director; MU School of Medicine; Columbia, MO, USA Lynn Messersmith, DO; Brooke Army Medical Center; San Antonio, TX, USA Alyssa Miceli, DO; Private Practice; Jacksonville, FL, USA Dana Mincer, DO; Private Practice; Holland, PA, USA Mouna Mlika, MD; Associate Professor; Associate Professor in Pathology; University of Medicine of Tunis;
tunis, Tunisia Oranus Mohammadi, MD; Aventura Hospital and Medical Center; Miami, FL, USA Shahid Mohammed, MBBS; Clinical Assistant Professor; Core Faculty; Michigan State University/Spectrum Health; Grand Rapids, MI, USA Praseeda Moleyar-Narayana, MD; Resident; LSU Health Shreveport; West Monroe, LA, USA Andres Monserrate, MD; Resident; University of Puerto Rico School of Medicine; Guaynabo, Puerto Rico Christine Moore, DO, BA; East Tennessee State University; Johnson City, TN, USA Leila Moosavi, MD; Kern Medical Center; Bakersfield, CA, USA Majid Moshirfar, MD; Tenured Professor; University of Utah/John Moran Eye Center; Hoopes Vision/HDR Research Center; Utah Lions Eye Bank; Draper, UT, USA Fady Moustarah, MD, MPH; Associate Professor; Central Michigan University; Saginaw, MI, USA Ateeq Mubarik, MD; Oak hill hospital; Spring Hill, FL, USA
Shiva Kumar Mukkamalla, MD, MPH; Clinical Assistant Professor; Presbyterian Healthcare Services; Rio Rancho, NM, USA Francesk Mulita, MD, MSc, PhDc; General University Hospital of Patras; Patras, Greece Sanjana Mullangi, MD; Attending; Hillcrest Medical Center; Tulsa, OK, USA Tessa Mullins, DO; Lewis Gale Hospital - Montgomery; Blacksburg, VA, USA Robert Murgia, DO, MA; VCOM/LewisGale Hospital Montgomery; Blacksburg, VA, USA Morgan Murphrey, BS; Creighton University, School of Medicine; Sacramento, CA, USA Heather Murphy-Lavoie, MD; Practicing Physician; University Medical Center, LSU Medical School; New Orleans, LA, USA Sierra Musick, MD; Private Practice; Universal City, TX, USA David Myers, MD; Practicing Physician; SAUSHEC; San Antonio, TX, USA
Phyu Thin Naing, MBBS; New York Presbyterian/Queens; Elmhurst, NY, USA Abdullah Naji, BS; Western University of Health Sciences; Pomona, CA, USA Sara Naji Rad, MD; Attending; Icahn School of Medicine at Mount Sinai; Mineola, NY, USA Japbani Nanda, BS; Renaissance School of Medicine at Stony Brook University; East Meadow, NY, USA George Nassar, MD; NYU Winthrop Hospital / NYU Langone; Mineola, NY, USA Muhammad Nauman, MBBS; SIUT, Karachi, Pakistan; Lahore, Pakistan Gul Nawaz, MD; Resident; Marshfield clinic; Marshfield, WI, USA Cornelia Ndifon, MBBCH, MPH, MResearch; Technical Director; Johns Hopkins Programme; Windhoek, Namibia Todd Needs, DO; San Antonio Military Medical Center; San Antonio, TX, USA Daniel Neelon, MD; Resident; Walter Reed National Military Medical Center; Bethesda, MD, USA
Zeid Nesheiwat, DO; Practicing Physician; Dr; The University of Toledo; Waterville, OH, USA Minhthao Nguyen, DO; University Hospitals Richmond and Bedford Medical Center; Cleveland, OH, USA Alexandra Nguyen, MD; Resident; Arrowhead Regional Medical Center/Kaiser Permanente Fontana; Riverside, CA, USA Sonia Oad, MD; ETSU / Quillen College of Medicine; NA, TN, USA Brian O'Donnell, MDc; Hofstra Northwell School of Medicine; Long Island City, NY, USA Mari Ogino, BS; Creighton University; Omaha, NE, USA Michael Okocha, BSc, MSc, MRCS, MAcadMEd; Associate Tutor; University of Bristol; Bristol, United Kingdom Yetunde Bernice Omotosho, MD; CMS-RFUMS Internal Medicine Residency Program at Northwestern McHenry Hospital; McHenry, IL, USA Michael Ortiz Torres, MD; University of Missouri SoM; Columbia, MO, USA Azeberoje Osueni, MBBS; Brookdale University Hospital Medical Center; Brooklyn, NY, USA
Jenna Otter, MD; Temple University Hospital; Philadelphia, PA, USA Sandeep Padala, MD; Augusta UMC/ Medical College of Georgia; Martinez, GA, USA Inderbir Padda, MD, MPH; Richmond University Medical Center; Surrey, Canada Jaishree Palanisamy, DO; McLaren Oakland; Rochester Hills, MI, USA Fathima Fijula Palot Manzil, MD; Assistant Professor of Radiology; University of Arkansas for Medical Sciences; Little Rock, AR, USA Ravi Paluri, MD, MPH; Assistant Professor; University of Alabama at Birmingham; Vestavia Hills, AL, USA Jyotsna Pandey, MBBS, MD PhD; Professor; MD PhD; Central Michigan University College of Medicine; Mount Pleasant, MI, USA Sudha Pandit, MD; Louisiana State University HSC; Shreveport, LA, USA Sreelakshmi Panginikkod, MD; Practicing Physician; University of Massachusetts; SHREWSBURY, MA, USA
Nirzar Parikh, MBBS; Jaslok Hospital and Research Centre; Mumbai, India Malvinder Parmar, MB, MS, FRCP, FASN; Professor; Medical Director, Internal Medicine, TDH; Timmins, Canada Meghana Parsi, MD; Crozer Chester Medical Center/ Drexel University; Clifton Heights, PA, USA Jayesh Patel, MBBS; Fellow; University of Iowa Hospitals and Clinics; Rochester, NY, USA Dharti Patel, MD; Oak Hill Hospital; Spring Hill, FL, USA Jamie Pawlowski, MD; UT Health San Antonio; San Antonio, TX, USA Basil Peechakara, MBBS; Private Practice; Westlake, OH, USA Joel Pellot, MD; University of Puerto Rico; San Juan, FL, USA Stephanie Prater, MD; Aventura Hospital & Medical Center; Miami, FL, USA Charles Preuss, BSPharm, PhD; Associate Professor; University of South Florida; Tampa, FL, USA
Yana Puckett, MD, MPH, MBA, MSc; Assistant Professor; Complex General Surgical Oncology; West Virginia University School of Medicine; Waunakee, WI, USA Joel Pulickal, MD; Assistant Professor; King Faisal University; Hofuf, Saudi Arabia Jack Pullan, MBChB, BMedSci, MRCS; Surgical Trainee; Stepping Hill Hospital; Manchester, United Kingdom Abdallah Qasim, MD; Creighton University; Omaha, NE, USA Asma Raboudi, MD; Faculty of Medicine of Tunis; Tunis, Tunisia Maitreyee Rai, MD; Crozer Chester Medical Center; Upland, PA, USA Kshitij Raizada, MS; Fellow; Prakash Netra Kendr; Bareilly, India Karan Ramakrishna, MBBS; Chief Resident; SUNY Upstate Medical University; Syracuse, NY, USA Neeraj Ramakrishnan, BA; California Northstate University; Elk Grove, CA, USA Kamleshun Ramphul, MBBS , MD; Shanghai Jiao Tong University; Triolet, Mauritius
Hussain Rangoonwala, MBBS; Resident; Creighton University; Omaha, NE, USA Logan Ranzenberger, DO; Michigan State Un, McLaren Health; Bloomfield Hills, MI, USA Prashanth Rawla, MD; Attending; Sovah Health, Martinsville, VA; Martinsville, VA, USA Appaji Rayi, MD; Staff Neurologist; Charleston Area Medical Center; Charleston, WV, USA Gerardo Rea, MD; Universidad de Guadalajara; Guadalajara, Mexico Alejandro Recio-Boiles, MD; University of Arizona; Tucson, AZ, USA Russell Reeves, MD, MS, MBAc; Thomas Jefferson University; Philadelphia, PA, USA Adriana Rendon, MD; Heidelberg University Hospital,; , Germany, USA Gizem Reyhanoglu, DO; LECOM; Ormond Beach, FL, USA Lucas Rindy, DO; Aventura Hospital and Medical Center; Miami, FL, USA Michael Ritchie, MD; Assitant Professor; Pediatric Anesthesiologist; West Virginia University; Morgantown,
WV, USA Sarah Rizzo, MD; Resident; Hofstra/Northwell, Department of Orthopaedic Surgery, Plainview Hospital; Mineola, NY, USA Arturo Robles-Tenorio, MD, MSc; Universidad de Guadalajara; Guadalajara, Mexico Eleanor Rosario, BSc (Hons.), MBChB (Hons.); Imperial College London Hospitals; London, United Kingdom Veronica Rutt, DO; Philadelphia College of Osteopathic Medicine; Allentown, PA, USA Patrick Saad, MDc; LECOM; Palm Harbor, FL, USA Bhanusivakumar Sabbula, MBBS; Nassau University Medical Center; New Hyde Park, NY, USA Ahmed Saber, MBBCH,MSc,PhD,MRCS,FRCS T&Or,EBOT, SICOT; Calderdale and Huddersfield NHS Foundation Trust; Huddersfield, United Kingdom Ahmed Sabry, MBBS; Resident; Cairo, Egypt Qandeel Sadiq, MBBS; Resident; University of Tennessee Health Science Center; Lakeland, TN, USA Mina Said, MD; Resident; Rochester General Hospital; Rochester, NY, USA
Hussain Sajjad, MBBS; Medical Officer; RMU and Allied Hospitals; Islamabad, Pakistan Shoaib Saleem, MBBS; Resident; King Edward Medical University Lahore; Lahore, Pakistan Tabinda Saleem, MD; UPMC Pinnacle Hospitals; Harrisburg, PA, USA Dahlia Saleh, DO; Sampson Regional Medical Center; Wilmington, NC, USA Sartaj Sandhu, MD; Sewell, NJ, USA Sumina Sapkota, MBBS; Gandaki Medical College, Pokhara; Kohalpur11, Nepal Parul Sarwal, MD; Saint Vincent Hospital; Worcester, MA, USA Ayesha Sarwar, MBBS, MD; Doctor; King Edward Medical University; Lahore, Pakistan Yub Raj Sedhai, MD; Clinical Assistant Professor; Virginia Commonwealth Un School of Med; Northville, MI, USA Marc Seligson, BS; SUNY Upstate Medical University; Syracuse, NY, USA Benjamin Senst, MD; Universitätsklinikum Halle; Halle (Saale), Germany
Karlin Sevensma, DO; Program Director; Metro Health Un of Michigan Health; Wyoming, MI, USA Robert Sexton, MD; American University of the Caribbean; East Meadow, NY, USA Ismat Shafiq, MD; Associate Professor; Uiversity of Rochester; Rochester, NY, USA Muneeb Shah, DO; Penn State Health; Hershey, PA, USA Abdul Basit Shah, MBBS; University of Health Sciences; Islamabad, Pakistan Zainab Shahid, BS; Lake Erie College of Osteopathic Med.; Glen Oaks, NY, USA Younus Shamam, MBCH.B; Other; University of Mosul, College of Medicine , Mosul Iraq; Houston, TX, USA Shafeek Shamsudeen, MBBS, MS, MCh; Consultant; Consultant Surgical Oncologist; MVR Cancer Centre & Research Institute; Kozhikode, India Elena Shanina, MD, PhD; Associate Professor; University of Texas Medical Branch; Houston, TX, USA Aarushi Sharma, PhD; Mayo Clinic; Jacksonville, FL, USA Bashar Sharma, MD; SUNY Upstate University Hospital; Syracuse, NY, USA
Sarah Shaw, MBBS, BSc; King's College London; London, United Kingdom Nafiz Sheikh, MD, BS, AS; Medical College of Georgia at Augusta University; Tifton, GA, USA Muhammad Mubbashir Sheikh, MD; Feinberg School of Medicine; Mechanicsburg, PA, USA Utsav Shrestha, MD; Clinical Instructor; University of Pittsburgh Medical Center; Pittsburgh, PA, USA Ismaeel Siddiqi, DO; University of Cincinnati; Cincinnati, OH, USA Faraz Siddiqui, MD; Private Practice; Staten Island, NY, USA David Sigmon, MD; Fellow; Surgical and Simulation Fellow; University of Pennsylvania; Philadelphia, PA, USA Gunjan Singh, MBBS, MD, MRCOG; Membership of the Royal College of Obstetricians and Gynecologists; Pittsburgh, PA, USA Achint Singh, MD; Associate Professor; Un of Texas Health Sci Ctr San Antonio; San Antonio, TX, USA Daulath Singh, MD; Stormont Vail Healthcare; Lawrence, KS, USA
Jennifer Singleton, MD; Assistant Professor; University of Colorado; Denver, CO, USA Shreya Sinha, MD; SUNY Upstate Medical University; Syracuse, NY, USA Melinda Smith, MD, MA; Assistant Professor; Section Chief; West Virginia University; Morgantown, WV, USA John Sobiesk, MS; Ochsner LSU - Shreveport; Shreveport, LA, USA Gregory Sprowl, MD; Practicing Physician; MD; Brooke Army Medical Center; Fort Sam Houston, TX, USA Vijay Srinivasan, MD; Fellow; Aventura Hospital and Medical Center; Aventura, FL, USA Christopher Stang, DO; Conway Med. Center/Campbell University; Conway, SC, USA Jessica Stempel, MD; Private Practice; Philadelphia, PA, USA Matthew Steward, BMBS; Pleural Fellow; Royal Devon & Exeter NHS Foundation Trust; Exeter, United Kingdom Kristin Sticco, DO, MS; Stony Brook University Hospital; Westbury, NY, USA
Surabhi Subramanian, MD; IWK Health Centre; Halifax, Canada Srinath Sundararajan, MD; Assistant Professor; University of Arizona; Tucson, AZ, USA Dallas Swayzer, BS; California Northstate University COM; Grand Prairie, TX, USA Dawood Tafti, MS, MD; Brooke Army Medical Center; San Antonio, TX, USA Mehran Taherian, MD; University at Buffalo, Department of Pathology; Buffalo, NY, USA Muhammad Taqi, MBBS; King Edward Medical University, Lahore, Pakistan; Lahore, MI, USA Muhammad Ali Tariq, MBBS, MD; Research Fellow; University of Louisville; Louisville, KY, USA Katherine Taylor, MDc; Saint James School of Medicine; Grassie, Canada Steven Tenny, MD, MPH, MBA; University of Nebraska Medical Center; Omaha, NE, USA Dac Teoli, MD; University of California, Riverside; Riverside, CA, USA Bicky Thapa, MD; Cleveland Clinic; Cleveland, OH, USA
Samuel Theis, DO; Captain; Wright-Patterson AFB / Wright State; Beavercreek, OH, USA Jonathan Theriot, DO; HonorHealth Scottsdale Thompson Peak Medical Center; Glendale, AZ, USA Aye Thida, MD; New York Health + Hospitals/Woodhull; Brooklyn, NY, USA Robert Thomson, BS; McGovern Medical School; Houston, TX, USA Sandra Tomlinson-Hansen, MD; Kaweah Delta Health Care District; Visalia, CA, USA Ariel Toomey, DO; Private Practice; East Lansing, MI, USA Craig Tork, MD; San Antonio Military Medical Center; San Antonio, TX, USA Todd Troxell, DO, BA; A.T. Still University; NERMC; Kirksville, MO, USA Gizem Tumer, MD; Assistant Professor; Co-Director; University of Minnesota; Indianapolis, IN, USA Connor Tupper, MDc; Creighton University School of Medicine; Omaha, NE, USA
Nicole Ufkes, BS; Other; Medical University of South Carolina; salt lake city, UT, USA Matthew Uy, DO; Pathology Resident; San Antonio Uniformed Services Health and Education Consortium; San Antonio, TX, USA Tanvi Vaidya, MBBS; Father Muller MC, Rajiv Gandhi Un of HS.; Thane, India Anusha Vakiti, MBBS; Private Practice; Silver Spring, MD, USA Merryl Varghese, MD; Michigan State University; Haslett, MI, USA Priyanka Venkatesh, MBBS; Rajiv Gandhi University ,; Bangalore, India Vipin Villgran, MD, MS; Allegheny General Hospital; WEXFORD, PA, USA Vibhu Krishnan Viswanathan, MBBS, MS; IWK Health Center; Halifax, Canada Lahari Vudayagiri, DO; Western Reserve Hospital; Stow, OH, USA Cynthia Waickus, MD, PhD; Associate Professor; Rush University Medical Center; Chicago, IL, USA
Samuel Weeks, MDc; SUNY Upstate Medical University; Syracuse, NY, USA Grant Williams, MD; San Antonio Uniformed Services Health ED; Schertz, TX, USA Andrew Williamson, BSc, MBChB; Imperial College London; London, Canada Jess Wilson, MD; PGY-3; Resident; UTHSCSA; San Antonio, TX, USA Grant Wintheiser, MD; Creighton University; Omaha, NE, USA Jennifer Witek, BS; LECOM; Belleville, MI, USA Andrew Wyker, DO; SOVAH Health Danville; Danville, VA, USA Siva Naga Yarrarapu, MBBS, MD; Internal Medicine Resident Physician; MD; Monmouth Medical Center; Long Branch, NJ, USA Muhammad Yasir, MBBS; Allama Iqbal Medical College/JHL, Lahore; Multan, Pakistan Rajini Kanth Yatavelli, MD; Louisiana State University HSC; Shreveport, LA, USA
Athina Yoham, MDc; University of Miami; North Miami, FL, USA Michael Young, MD; Practicing Physician; Marian University; Indianapolis, IN, USA Allen Young, MD, MPH; Practicing Physician; Resident; Las Vegas, TX, USA Irini Youssef, MD; SUNY Downstate Medical Center; Ridgewood, NY, USA Bo Yu, MD; Resident; Lincoln Medical Center, Bronx, NY; Monterey Park, CA, USA Seongseok Yun, MD, PhD; University of Arizona; Tucson, AZ, USA Rabia Zafar, MD; East Tennessee State University; Johnson city, TN, USA Syed Zaidi, MBBS; Medical Officer; Dow University of health Sciences; KARACHI, Pakistan Syed Rafay Zaidi, MBBS; Isra University, Al Nafees; Rawalpindi, Pakistan Zarlakhta Zamani, MBBS; King Edward Medical University (Mayo Hospital Lahore); Asadabad, Afghanistan
Edgar Zamora, MD; Montefiore Medical Center; Bronx, NY, USA Levi Zehr, BS; University of Nebraska Medical Center; Omaha, NE, USA Chen Zhang, MD; University of Miami/JFK Medical Center; West Palm Beach, FL, USA Fenghao Zhuo, MD; Einstein Medical Center; Philadelphia, PA, USA Lance Zimmerman, MD; Aventura Hospital and Medical Center; Aventura, FL, USA Patrick Zito, DO, PharmD; Voluntary Assistant Professor; University of Miami; Miller School of Medicine; Fort Lauderdale, FL, USA Muhammad Zubair, MBBS, FCPS; Practicing Physician; Indus Hospital & Health Network; MULTAN, Pakistan Arshad Zubair, MBBS, MS, MRCSEd; National Hospital for Neurology and Neurosurgery, London WC1N 3BG; Kozhikode, India Hassam Zulfiqar, MBBS; Practicing Physician; Rawalpindi Medical University & Allied Hospitals; Islamabad, Pakistan
Grammar and Illustration Editor Gerson Cordero Rubio
Editorial Support Erin Hughes
Table of Contents Please Contribute Learn More Introduction Editors in Chief Contributing Editors Authors Section 1 ( Questions 1 - 100 ) Section 2 ( Questions 101 - 200 ) Section 3 ( Questions 201 - 300 ) Section 4 ( Questions 301 - 400 ) Section 5 ( Questions 401 - 500 ) Section 6 ( Questions 501 - 600 ) Section 7 ( Questions 601 - 700 ) Section 8 ( Questions 701 - 800 ) Section 9 ( Questions 801 - 900 ) Section 10 ( Questions 901 - 1000 ) About StatPearls
Section 1 Question 1:
A 62-year-old male with a history of fatigue presents to his primary care office. A complete blood count was performed and showed a hemoglobin of 7.4 grams/dL, mean corpuscular volume (MCV) of 65.6 femtoliters, and platelet count of 50/microliter. The peripheral blood showed less than 1% circulating blasts. This prompted a bone marrow biopsy, which showed 17% ringed sideroblasts and 12% dysplasia in the erythroid lineage (see image). Based on the clinical information and bone marrow biopsy, what is the most useful molecular test for determining survival?
Choices: 1. 2. 3. 4.
G742D SF3B1 ETV6 PDGFRB
Photo:Image courtesy S Bhimji MD
Answer: 2 - SF3B1 Explanations: Although G742D with rings sideroblasts is found within myelodysplastic syndromes (MDS), it is not the most common nor the most useful molecular test to perform to determine prognosis. However, it is the second most common hotspot mutation along with K666 and H662. SF3B1 is the best answer choice based on the clinical presentation and the bone marrow biopsy. This bone marrow would be most likely defined at MDS-RS-SLD which would benefit from a determination of this genetic mutation. The primary defect is in mitochondrial metabolism, which is determined largely by SF3B1linked to the formation of RS. Normally, this is detected in 64% to 83% of MDS-RS-SLD patients and 57% to 75% in those with metachromatic leukodystrophy (MLD). These patients have clinically better survival than in unmutated cases and run a relatively indolent course. In nearly 1% of patients, there is progression to acute myelogenous leukemia (AML). ETV6 is not found in myelodysplastic syndrome cases and is largely found in its counterpart of AML t(8;21) and variably expressed in familial MDS or AML cases. There are two membrane receptors, PDGFR-a and PDGFR-beta, involved in PDGF signaling. Like other growth factor receptors, both PDGFR-a and PDGFR-beta dimerize and can auto-phosphorylate on several tyrosine residues. Myeloproliferative neoplasms with rearrangements of PDGFRA, PDGFRB, or FGFR1 are very rare. Those resulting from a PDGFRA rearrangement often have a cryptic deletion at 4q12 and are associated formation of a FIP1L1/PDGFRA fusion protein.
Occasionally, a translocation at 4q12 also will result in a PDGFRA fusion protein. PDGFRA rearrangements are more commonly found in chronic eosinophilic leukemia and are generally responsive to imatinib and other tyrosine kinase inhibitors. Go to the next page if you knew the correct answer, or click the link image(s) below to further research the concepts in this question (if desired).
Research Concepts: Refractory Anemia With Ring Sideroblasts
We update eBooks quarterly and Apps daily based on user feedback. Please tap flag to report any questions that need improvement.
Question 2:
A 42-year-old male presented complaining of difficulty urinating at times. He stated that his urine looked dark. He has a history of lung cysts, spontaneous pneumothorax, multiple fibrofolliculomas, trichodiscomas, and acrochordons. Upon exam, there was blood in his urine. Which of the following carcinoma is most likely seen with patient's underlying diagnosis?
Choices: 1. 2. 3. 4.
Hepatocellular carcinoma Squamous cell carcinoma Renal cell carcinoma Basal cell carcinoma
Photo:Contributed by DermNetNZ
Answer: 3 - Renal cell carcinoma Explanations: Birt-Hogg-Dubé syndrome (BHDS) is an autosomal dominant genodermatosis usually manifesting in the third decade of life with multiple fibrofolliculomas, trichodiscomas, and acrochordons. The characteristic skin lesions of Birt-Hogg-Dube syndrome are trichodiscomas, fibrofolliculomas, and acrochordons. Lung cysts and pneumothorax may occur in adult life. Patients with BHDS are at increased risk of developing renal tumors lifelong, with an average age of onset at 50 years. Lifelong renal surveillance should begin at the age of twenty. Some experts recommend initial MRI followed by annual MRI or ultrasound, while others suggest CT scans every 3 to 5 years. Because of the risk of malignancies later in life these patients are best managed by an interprofessional team that includes a geneticist, pulmonologist, thoracic surgeon, dermatologist, urologist, and an internist. Patients with BHDS are at increased risk of developing renal tumors and renal carcinomas lifelong. Go to the next page if you knew the correct answer, or click the link image(s) below to further research the concepts in this question (if desired).
Research Concepts: Birt Hogg Dube Syndrome
We update eBooks quarterly and Apps daily based on user feedback. Please tap flag to report any questions that need improvement.
Question 3:
An 11-year-old male with several months of pain and swelling following minor trauma below the left knee presents to your office. Radiography demonstrates well defined, eccentric, expansile, lytic lesion in the lateral metaphysis of the proximal left tibia. It shows a Codman's triangle laterally and has slight sclerotic rim medially. It does not cross the growth plate. MRI reveals fluid-fluid levels and no involvement of the growth plate. Bone scan shows a rim of peripheral activity with a central drop in activity. What is the nature of this lesion?
Choices: 1. 2. 3. 4.
Benign Moderately aggressive Malignant Infection
Answer: 2 - Moderately aggressive Explanations: An Aneurismal bone cyst (ABC) is a multicystic lytic bone lesion of uncertain origin. The lesion has a fibrous wall and blood as its content. They can arise de novo, due to trauma or secondary to underlying lesions like fibrous dysplasia, chondroblastoma, chondrosarcoma, etc. MRI helps in ruling out secondary causes of ABC. They usually occur before 20 years of age. They present with prolonged pain. A lesion in the spine may show neurological symptoms related to the level of involvement. Sites of involvement in decreasing order: Long bones, spine, small bones of hands /feet, and pelvis. 75% of long bone tumors occur in the metaphysis. ABC is eccentric lytic lesion involving metaphysis of long bones, posterior elements in case of the spine. They cause cortical thinning. MRI shows multicystic mass with or without fluid-fluid levels. Signal intensities vary according to the stages of blood products. CT may reveal subtle periosteal reaction and fluid-fluid levels. The treatment for ABC is curettage and bone grafting. Go to the next page if you knew the correct answer, or click the link image(s) below to further research the concepts in this question (if desired).
Research Concepts: Aneurysmal Bone Cysts
We update eBooks quarterly and Apps daily based on user feedback. Please tap flag to report any questions that need improvement.
Question 4:
A 55-year-old woman with metastatic breast cancer who has gone through multiple chemotherapy regimens is admitted to the hospital with failure to thrive and altered mental status. Work up reveals that the patient now has worsening metastatic lesions in the liver and new intracranial lesions. After a discussion with her and her husband, her oncologist recommends hospice. Her biggest concern is to be a burden on her husband when she goes home. She asks how hospice can help. Which of the following statements about hospice are true?
Choices: 1. Hospice will provide 24-hour access so that she or her husband will be able to call for help with her symptoms or unmet needs 2. Hospice will have skilled nurses, social works and chaplains assessing her, including talking with her husband to assess how he is coping 3. Hospice will provide a 24-hour caregiver to take care of SK covered under insurance 4. Hospice will provide 24-hour access so that she or her husband will be able to call for help with her symptoms or unmet needs. Skilled nurses, social works and chaplains will assess her, including talking with her husband to assess how he is coping
Answer: 4 - Hospice will provide 24-hour access so that she or her husband will be able to call for help with her symptoms or unmet needs. Skilled nurses, social works and chaplains will assess her, including talking with her husband to assess how he is coping
Explanations: The hospice interdisciplinary team comprises of skilled professionals including at its core, a nurse, a social work, and a chaplain. During home visits, the hospice team makes physical and environmental assessments and assesses the patient’s and family’s needs for additional services and assistance. Hospice staff, most often nurses, are on-call 24 hours a day, seven days a week. They attend to calls from patients and families. Hospice does not provide a 24-hour caregiver to patients. If families need caregivers for the patient, they usually hire them privately at their own expense. Go to the next page if you knew the correct answer, or click the link image(s) below to further research the concepts in this question (if desired).
Research Concepts: Hospice Care
We update eBooks quarterly and Apps daily based on user feedback. Please tap flag to report any questions that need improvement.
Question 5:
A 40-year-old woman with a history of hypothyroidism presents with a 1-month history of right epigastric pain. A physical exam is unremarkable. Abdominal ultrasound reveals multiple calculi in the gall bladder. CT of the abdomen revealed a 5 x 4 cm mass in the pancreatic head. Which serum marker is most likely to be elevated in this case?
Choices: 1. 2. 3. 4.
Serotonin Somatostatin Gastrin Vasoactive intestinal polypeptide
Answer: 2 - Somatostatin Explanations: The pancreas is the most common site of origin for somatostatinoma. Pancreatic somatostatinomas are usually secretory and present with elevated serum levels of somatostatin (> 14 mmol/L). Somatostatin inhibits thyrotropin release from the hypothalamus causing central hypothyroidism. Somatostatin suppresses cholecystokinin secretion thus affecting gall bladder contractility and emptying, which leads to cholelithiasis and cholecystitis. Go to the next page if you knew the correct answer, or click the link image(s) below to further research the concepts in this question (if desired).
Research Concepts: Somatostatinoma
We update eBooks quarterly and Apps daily based on user feedback. Please tap flag to report any questions that need improvement.
Question 6:
A 65-year-old male with no significant past medical history presents to with a chief complaint of pruritus and new onset "bumps" all over his back, chest, and abdomen. He admits to some recent unintentional weight loss and fatigue. There is the concern the patient may be displaying the sign of Leser-Trelat, characterized by an abrupt eruption of multiple seborrheic keratoses. What underlying malignancy is the most common in patients with the sign of Leser-Trelat?
Choices: 1. 2. 3. 4.
Prostate cancer Lung cancer Gastric cancer Bladder cancer
Answer: 3 - Gastric cancer Explanations: The sign of Leser-Trelat a fairly rare paraneoplastic cutaneous marker of internal malignancy with the hallmark finding being an abrupt eruption of multiple seborrheic keratoses. There is much debate about the validity of the sign as a true paraneoplastic syndrome This “sign” has been met with a significant amount of skepticism by many considering the commonality of both seborrheic keratoses and malignancy in elderly patients. The most commonly implicated malignancies include gastrointestinal adenocarcinomas (gastric, colon, rectal), with gastric adenocarcinoma being the overall most common malignancy, followed by breast cancer, and lymphoproliferative disorders/lymphoma. Additional malignancies reported to display the sign of Leser-Trelat include melanoma, prostate, lung, kidney, laryngeal, ovarian, mycosis fungoides, hepatocellular carcinoma, bladder cancer, nasopharyngeal carcinoma, and squamous cell carcinoma. Go to the next page if you knew the correct answer, or click the link image(s) below to further research the concepts in this question (if desired).
Research Concepts: Leser Trelat Sign
We update eBooks quarterly and Apps daily based on user feedback. Please tap flag to report any questions that need improvement.
Question 7:
A 52-year-old-male presents to the clinic for follow-up with a recent diagnosis of squamous cell carcinoma of the lung. He is found to be positive for the epidermal growth factor receptor (EGFR) mutation. Which of the following drugs is most appropriate to treat this patient's cancer?
Choices: 1. 2. 3. 4.
Infliximab Osimertinib Bevacizumab Crizotinib
Answer: 2 - Osimertinib Explanations: EGFR mutation identifies if the cancer is sensitive to the tyrosine kinase inhibitors (TKIs), directed towards EGFR, or not. If a patient is EGFR positive, you can use drugs such as osimertinib, erlotinib, afatinib, gefitinib, or dacomitinib. The observed frequency in squamous cell carcinoma is 2% to 3%. The frequency is much higher in adenocarcinoma to 10%. Go to the next page if you knew the correct answer, or click the link image(s) below to further research the concepts in this question (if desired).
Research Concepts: Squamous Cell Lung Cancer
We update eBooks quarterly and Apps daily based on user feedback. Please tap flag to report any questions that need improvement.
Question 8:
A 29-year-old man presents to the clinic with an 8-month history of right cheek swelling. He states the swelling has suddenly been increasing in size over the last several weeks and now causing significant pain, swelling, and deformity of the right side of his face. Upon examining the patient, the swelling appears to be firm, nontender, and immobile. Intra-orally, a 2 cm ulcerated mass is seen involving the cheek mucosa. There is no palpable neck mass. After CT-imaging to determine the extent of the mass showed multilocular and cystic with a “soap bubble” or “honeycomb” appearance, the patient is sent for a biopsy of the mass, which shows peripheral palisading nuclei, polarization, cellular atypia, low mitotic index, and stellate reticulum-like cells. What is the most appropriate treatment strategy for this patient?
Choices: 1. Surgical resection chemoradiotherapy 2. Surgical resection chemoradiotherapy 3. Surgical resection 4. Surgical resection
with 1 to 2 cm margins followed by with 2 to 3 cm margins followed by with 1 to 2 cm margins only with 2 to 3 cm margins only
Answer: 3 - Surgical resection with 1 to 2 cm margins only
Explanations: The likely diagnosis, in this case, is malignant ameloblastoma. The treatment of choice for all ameloblastomas is surgical excision. Chemoradiotherapy is rarely required, except in select cases, with malignant ameloblastomas due to its generally slow-growing nature. Surgical excisional margins are around 1 to 2 cm for malignant ameloblastoma and 2 to 3 cm for ameloblastic carcinoma. Go to the next page if you knew the correct answer, or click the link image(s) below to further research the concepts in this question (if desired).
Research Concepts: Ameloblastoma
We update eBooks quarterly and Apps daily based on user feedback. Please tap flag to report any questions that need improvement.
Question 9:
A 59-year-old male undergoes a partial penectomy for a 2 cm penile lesion that is biopsy-proven squamous cell carcinoma. His physical examination reveals no inguinal lymphadenopathy. Which primary tumor characteristic is associated with inguinal lymph node metastases?
Choices: 1. 2. 3. 4.
Low stage of the primary tumor, T2 or greater Low grade of the primary tumor Lymphovascular invasion is present History of Human papillomavirus
Answer: 3 - Lymphovascular invasion is present Explanations: A multi-institutional review identified a number of factors that are predictive of inguinal node metastases. The most important factors that have been identified include a higher stage of the tumor, T2 or greater, high grade of primary tumor and lymphovascular invasion. Other factors that increase the likelihood of pathologic involvement of the lymph nodes include superficial growth pattern, tumor thickness, involvement of the corporal tissue, and involvement of the urethra. The factor most predictive of lymph node involvement in a multi-institutional review was the presence of lymphatic and vascular invasion seen in the primary tumor. Go to the next page if you knew the correct answer, or click the link image(s) below to further research the concepts in this question (if desired).
Research Concepts: Penile Cancer
We update eBooks quarterly and Apps daily based on user feedback. Please tap flag to report any questions that need improvement.
Question 10:
A 65-year-old female patient with a past medical history of nicotine use, morbid obesity, hypertension, and diabetes mellitus type 2, undergoes an esophagogastroduodenoscopy for further evaluation of dyspepsia. The patient was initially given a trial of proton pump inhibitor (PPI) for eight weeks with minimal symptom improvement. The frequency of PPI was then changed to twice daily without much effect. The endoscopic examination of the duodenum and stomach was unremarkable. She is found to have a 2 cm long abnormal appearing salmon-colored mucosa extending above the esophagogastric junction in a tongue-like fashion. Biopsy confirms intestinal metaplasia with areas of high-grade dysplasia without submucosal invasion. What is the next best step in the management of this patient?
Choices: 1. 2. 3. 4.
Radiofrequency ablation Repeat endoscopy in 6 months Repeat endoscopy in 3 to 5 years Esophagectomy
Answer: 1 - Radiofrequency ablation Explanations: There is a substantial risk of progression of high-grade dysplasia to esophageal adenocarcinoma. Therefore the presence of high-grade dysplasia is an indication for therapy. Endoluminal therapies such as radiofrequency ablation have become the mainstay of therapy of Barrett metaplasia. This can be achieved using the application of heat through laser, radiofrequency, argon plasma coagulation or cold through cryotherapy or photodynamic therapy (PDT) to destroy the abnormal epithelium. Due to comparable efficacy and beneficial side effect profile, radiofrequency ablation is the preferred mode of endoscopic therapy compared to PDT. Esophagectomy was previously the treatment of choice for high-grade dysplasia and intramucosal carcinoma in Barrett disease. It is associated with prolonged hospital stay, short and long-term complications with a poor quality of life in the immediate post-operative period. It, therefore, is no longer the first-line treatment. Go to the next page if you knew the correct answer, or click the link image(s) below to further research the concepts in this question (if desired).
Research Concepts: Barrett Metaplasia
We update eBooks quarterly and Apps daily based on user feedback. Please tap flag to report any questions that need improvement.
Question 11:
A 65-year-old male patient presents to the hospital with hoarseness, pain with swallowing, and earache. On examination, there is a 3 cm node palpable on the left side of the neck. On further investigations, he is found to have a squamous cell carcinoma of the right side of the laryngeal surface of the epiglottis. A PET/CT scan reveals hypermetabolic lesions corresponding to the epiglottic lesion and the palpable node but no others. What is the nodal staging of this tumor?
Choices: 1. 2. 3. 4.
N1 N2 N2a N2c
Answer: 4 - N2c Explanations: According to the tumor, node, metastases classification by the American Joint Committee on Cancer, N2c represents "bilateral or contralateral positive nodes, none more than 6 cm in diameter." Therefore, the neck staging is N2c because the node is on the contralateral side of the tumor. N2a is metastasis in a single ipsilateral lymph node which is more than 3 cm but not more than 6 cm at its greatest dimension. N2b is metastasis in multiple ipsilateral lymph nodes, but none are more than 6 cm at their greatest dimension. Go to the next page if you knew the correct answer, or click the link image(s) below to further research the concepts in this question (if desired).
Research Concepts: Laryngeal Cancer
We update eBooks quarterly and Apps daily based on user feedback. Please tap flag to report any questions that need improvement.
Question 12:
A 33-year-old female was diagnosed recently with metastatic melanoma (BRAF V600E positive) and was started on vemurafenib. She started her targeted therapy three weeks prior, and she is in the office today due to new onset of a lesion in her left arm. On exam, the lesion has a well-demarcated hyperkeratosis with ulceration in the center and without signs of inflammation. Keratoacanthoma is suspected. What is the next step in management?
Choices: 1. Stop vemurafenib and proceed with excision biopsy 2. Continue vemurafenib and proceed with excision biopsy 3. Continue vemurafenib and advice patient to use sunscreen with SPF 30 4. Continue vemurafenib and treat with BCG intralesional injection
Answer: 2 - Continue vemurafenib and proceed with excision biopsy
Explanations: Vemurafenib does not have to be stopped when keratoacanthoma develops Excisional biopsy is the best next step for diagnosis and treatment of keratoacanthoma which is a low-grade skin tumor. Sunscreen will not prevent the progression of keratoacanthoma. BCG injection is used in certain cancers like low-grade bladder cancer and is not used in the treatment of keratoacanthoma. Go to the next page if you knew the correct answer, or click the link image(s) below to further research the concepts in this question (if desired).
Research Concepts: Vemurafenib
We update eBooks quarterly and Apps daily based on user feedback. Please tap flag to report any questions that need improvement.
Question 13:
A 45-year-old male presents with increased tenderness in the epigastric region. He lacks an appetite and has experienced weight loss over the past few months. His vital signs are within in the normal reference range. Initial diagnostic testing reveals a fasting serum gastrin level of more than 1000 picograms/mL. Which of the following would be expected upon secretin administration?
Choices: 1. Elevated gastrin levels 2. Decreased gastrin levels 3. Normal gastrin levels 4. Secretin administration is not necessary to make a diagnosis
Answer: 1 - Elevated gastrin levels Explanations: This patient most likely has Zollinger-Ellison syndrome. Gastrin remains increased after administration of secretin. Fasting serum gastrin levels of more than 1000 picograms/mL is only half of the diagnostic evidence needed to make the diagnosis of Zollinger-Ellison syndrome. Therefore, secretin should be administered to gather appropriate information in terms of the elevation or depression of gastrin levels. Go to the next page if you knew the correct answer, or click the link image(s) below to further research the concepts in this question (if desired).
Research Concepts: Zollinger Ellison Syndrome
We update eBooks quarterly and Apps daily based on user feedback. Please tap flag to report any questions that need improvement.
Question 14:
A 54-year old male presents with a 3month history of worsening cough with recurrent hemoptysis. He has smoked 2 packs of cigarettes a day for the past 30 years. He has had a 10kg(22lb) weight loss during this period. His temperature is 37.3 C (99.1F), blood pressure is 120/76mm Hg, respirations per minute are 20, the pulse is 70/min. A chest x-ray shows a 3-cm mass in his left upper lung lobe with centrally located cavitation. Subsequently, a diagnosis of squamous non-small cell lung cancer is made. Which of the following drugs is approved for use in first-line metastatic squamous NSCLC in combination with paclitaxel and carboplatin?
Choices: 1. 2. 3. 4.
Pembrolizumab Natalizumab Rituximab Eculizumab
Answer: 1 - Pembrolizumab Explanations: Pembrolizumab is approved for use in first-line metastatic squamous NSCLC with paclitaxel (or paclitaxel protein-bound) and carboplatin. In the KEYNOTE-407 study, pembrolizumab showed a median overall survival of 15.9 months in this population as compared to 11.3 months on chemotherapy alone. Additionally, in non-small cell lung cancer, pembrolizumab is also approved for first-line metastatic non-squamous NSCLC without EGFR/ALK tumor abnormalities in combination with platinum chemotherapy and pemetrexed. Additionally, in non-small cell lung cancer, pembrolizumab is also approved as first-line singleagent use for nonresectable or metastatic NSCLC without EGFR/ALK tumor abnormality with tumor PD-L1 expression greater than 1% and single-agent use in metastatic NSCLC with tumor PD-L1 expression greater than 1% who have progressed after platinum-based chemotherapy (EGFR/ALK tumor abnormality patients with progression after targeted therapy for their respective tumor abnormality). Go to the next page if you knew the correct answer, or click the link image(s) below to further research the concepts in this question (if desired).
Research Concepts: Pembrolizumab
We update eBooks quarterly and Apps daily based on user feedback. Please tap flag to report any questions that need improvement.
Question 15:
A 67-year-old female presented with memory loss, confusion, and personality changes. The neurologist found mild cognitive dysfunction during the examination. The neurological history and physical examination are unremarkable. A CT scan of the brain is normal. An MRI of the brain revealed hyperintense lesions in the medial temporal cortex. Which of the following should be the next step in management?
Choices: 1. 2. 3. 4.
Repeat CT scan Consult an oncologist Prescribe aspirin Order a PET scan
Answer: 2 - Consult an oncologist Explanations: This patient has paraneoplastic limbic encephalitis (PLE). Symptoms include memory loss, confusion, personality or behavioral changes, seizures, and sleep disturbances. Because the CT and MRI are negative except for the hyperintense lesions in the medial temporal lobe, the diagnosis should be made after excluding other possible causes. One of the most common causes of PLE is occult cancer of the breast, lung, or ovary, which may be diagnosed after symptom onset. MRI is superior to CT scan in infratentorial lesions and some early manifestations of stroke. Also, brain parenchyma may be evaluated better with MRI. Aspirin should be used in a patient who has suffered or is susceptible to a stroke, such as a previous myocardial infarction or very high LDL cholesterol. A PET scan should be considered to find occult malignancies. This step would be reasonable if the oncologist does not find a tumor or to confirm the diagnosis. Go to the next page if you knew the correct answer, or click the link image(s) below to further research the concepts in this question (if desired).
Research Concepts: Paraneoplastic Limbic Encephalitis
We update eBooks quarterly and Apps daily based on user feedback. Please tap flag to report any questions that need improvement.
Question 16:
A 67-year-old female with a performance status of 0, is brought to the clinic with symptomatic anemia and has required several transfusions over the past 2 to 3 months. Currently, her hemoglobin is 7.5 g/dL, absolute neutrophil count 1500/microL and platelet count is 119,000/microL. She had a bone marrow biopsy performed that reveals myelodysplastic syndrome (MDS) with 6% blasts and cytogenetics positive for 5q deletion. She did not have any mutations or other cytogenetic abnormalities. Which of the following is the best initial course of treatment?
Choices: 1. 2. 3. 4.
An erythropoietin stimulating agent A thalidomide derivative A hypomethylating agent Stem cell transplant
Answer: 2 - A thalidomide derivative Explanations: This is a patient requiring frequent transfusions with a diagnosis of MDS and 5q deletion, which is generally a favorable type of MDS. A patient with symptomatic anemia and 5q deletion is generally a good candidate for lenalidomide, a thalidomide derivative. Stem cell transplant may be a potential treatment down the line, but the initial treatment of lenalidomide is generally recommended. A hypomethylating agent may be used, but initial treatment with lenalidomide in the setting of the patient's cytogenetics should be tried first. Go to the next page if you knew the correct answer, or click the link image(s) below to further research the concepts in this question (if desired).
Research Concepts: Myelodysplastic Syndrome
We update eBooks quarterly and Apps daily based on user feedback. Please tap flag to report any questions that need improvement.
Question 17:
A 72-year-old male with a past medical history of hypertension, hyperlipidemia, and known prostate cancer with metastases to the bone presents to the emergency department after an acute onset of confusion. His son provided most of the history. He states that his father had recently switched from bisphosphonate therapy to denosumab for his uncontrolled bone pain. He reports over the last week or so, his father had been reporting a tingling sensation around his mouth and fingertips as well as an increase in leg swelling. Then this morning, his father woke up and seemed confused, not knowing where he was or his son's name. Physical exam reveals bilateral lower extremity edema and positive Trousseau and Chvostek signs. What laboratory test should have been obtained prior to initiating denosumab?
Choices: 1. 2. 3. 4.
Complete blood count Thyroid-stimulating hormone level Comprehensive metabolic panel Parathyroid level
Answer: 3 - Comprehensive metabolic panel Explanations: Denosumab's mechanism of action involves competitively inhibiting RANKL and therefore preventing the binding of RANKL to RANK. This, in turn, decreases osteoclast maturation and function and therefore decreases bone resorption. The decrease in bone resorption causes a decrease in serum calcium. Hypocalcemia is a common and possibly serious adverse effect of denosumab. The symptoms of hypocalcemia include muscle cramps, leg swelling, confusion, memory loss, perioral and upper extremity tingling and numbness, hallucinations, depression, weak and brittle bones, seizures. Obtaining vitamin D 25 OH and serum calcium levels is highly recommended before initiating denosumab therapy due to the increased risk of hypocalcemia while on denosumab. A serum calcium level can be obtained through a comprehensive metabolic panel. These levels should also be periodically checked throughout the continuation of therapy. Studies show that the administration of 500 mg of calcium and 400 IU of vitamin D daily decreases the risk of developing hypocalcemia while on denosumab by about 40%. Go to the next page if you knew the correct answer, or click the link image(s) below to further research the concepts in this question (if desired).
Research Concepts:
Denosumab
We update eBooks quarterly and Apps daily based on user feedback. Please tap flag to report any questions that need improvement.
Question 18:
A 68-year-old African American female was recently diagnosed with small cell carcinoma of the right lung and is yet to start definitive therapy. Respiratory symptoms are minimal at present except for a cough. For the past month, however, she has been complaining of back pain, and since last week she has had intense pain in her legs. On examination, her knee joints are significantly swollen. A positron emission tomography (PET) scan was done to stage her lung cancer and showed multiple high 2[18F] fluoro-D-glucose (FDG) uptake spots in ribs, thoracic vertebra, and both tibia. The tibial lesions show no cortical destruction or distortion of bone architecture in the bone scan. She wants help with her leg pain so she can walk again. Which of the following is the best initial therapy for this patient after biopsy confirms the diagnosis?
Choices: 1. Localized radiation of thoracic vertebral lesions and tibial lesions 2. Definitive chemoradiation treatment of primary lung malignancy 3. Use of opioid analgesics followed by radiation 4. Combination of propranolol and phenoxybenzamine
Answer: 4 - Combination of propranolol and phenoxybenzamine
Explanations: Even though the patient has a diagnosis of small cell lung cancer, tibial skip metastasis is very rare and is usually not bilaterally symmetrical. The fact that there is no destruction or deformation of bony architecture also points more towards hypertrophic osteoarthropathy than metastatic disease. Secondary hypertrophic osteoarthropathy (HOA) lesions can be FDG avid in PET scans. Adrenergic blockade with phenoxybenzamine and propranolol has been demonstrated to be an effective treatment strategy for HOA from small cell carcinoma. Opioids are usually ineffective for the treatment of secondary hypertrophic osteoarthropathy. Go to the next page if you knew the correct answer, or click the link image(s) below to further research the concepts in this question (if desired).
Research Concepts: Secondary Hypertrophic Osteoarthropathy
We update eBooks quarterly and Apps daily based on user feedback. Please tap flag to report any questions that need improvement.
Question 19:
A 43-year-old single female presented, with a lump in the right breast that had been gradually increasing for 9 months. Two months before the patient sought medical advice, the mass had ulcerated and had started to bleed with severe pain. On breast examination, a giant ulcerating mass (16 cm in diameter) occupied the right breast with areas of necrosis. There were no axillary lymph nodes. An ultrasound-guided percutaneous biopsy (core biopsy) on the breast lesion was performed. Pathologic examination disclosed a biphasic neoplasm with an epithelial and stromal component. The stromal component showed nuclear pleomorphism of stromal cells, stromal overgrowth, and 20 mitoses per 10 high power fields. There were also areas of necrosis. What is the most likely diagnosis?
Choices: 1. 2. 3. 4.
Cellular fibroadenoma Angiosarcoma Borderline phyllodes tumor Malignant phyllodes tumor
Answer: 4 - Malignant phyllodes tumor Explanations: Malignant phyllodes tumors are characterized by marked nuclear pleomorphism of stromal cells, stromal overgrowth, increased mitoses (greater than or equal to 10 per 10 HPF), increased stromal cellularity which is usually diffuse, and infiltrative borders. Malignant phyllodes tumor may be confused with pure sarcomas of the breast. In such cases, diagnosis depends on finding residual epithelial structures. However, the clinical impact of these two entities appears to be similar. The phyllodes tumors are graded according to the areas of highest stromal cellular activity and most florid architectural pattern. Stromal overgrowth has been defined as a stromal proliferation to the point where epithelial elements are absent in at least one low-power field. Malignant phyllodes tumors carry a poor prognosis. Distant metastases occur almost exclusively in malignant phyllodes tumors, the lungs and bones being the most common sites of spread. Therefore, close follow-up of patients with malignant phyllodes tumors is mandatory. Go to the next page if you knew the correct answer, or click the link image(s) below to further research the concepts in this question (if desired).
Research Concepts: Phyllodes Tumor Of The Breast
We update eBooks quarterly and Apps daily based on user feedback. Please tap flag to report any questions that need improvement.
Question 20:
A 54-year-old man with HIV presents with bright red blood per rectum. Digital rectal examination reveals an ulcerated and friable 5.5 cm mass in the anal canal originating 2 cm from the anal verge. A biopsy confirms squamous cell carcinoma. There is no involvement of the rectum or urethra. Positron emission tomographycomputed tomography (PET/CT) demonstrates a 1.2 cm left external iliac lymph node without additional metastases. What is this patient’s TNM stage according to American Joint Committee on Cancer (AJCC) Eighth edition?
Choices: 1. 2. 3. 4.
T2N1bM0 - Stage IIIA T3N1bM0 - Stage IIIC T2N1aM0 - Stage IIIB T3N1aM0 - Stage IIB
Answer: 2 - T3N1bM0 - Stage IIIC Explanations: Primary tumors larger than 5 cm that do not involve adjacent organs are classified as T3. American Joint Committee on Cancer (AJCC) Eighth edition now uses N1a, N1b, and N1c to define nodal staging. External iliac lymph nodes are classified as N1b, whereas metastasis to inguinal, mesorectal, or internal iliac lymph nodes are classified as N1a. Metastasis to external iliac lymph nodes with any N1a nodes is now classified as N1c. Anal canal cancers are initially worked up with a digital rectal examination, anoscopy with biopsy, pelvic CT/MRI, and inguinal lymph node evaluation with biopsy of suspicious nodes. PET/CT should be considered to assist in radiation treatment planning and to evaluate for distant metastatic disease. Clinical presentation of anal canal cancers can include bleeding (45%), pain (30%), pruritis, perianal mass, rectal urgency, or a change in stool caliber. Go to the next page if you knew the correct answer, or click the link image(s) below to further research the concepts in this question (if desired).
Research Concepts: Radiation Therapy For Anal Cancer
We update eBooks quarterly and Apps daily based on user feedback. Please tap flag to report any questions that need improvement.
Question 21:
A 40-year-old male was recently diagnosed with a mediastinal mass when he was evaluated for chest pain. CT scan showed 6 cm anterior mediastinal mass with prominent calcification. Serum alpha-fetoprotein and beta-hCG are within normal limits. The patient underwent surgical excision of the mass. Gross cut section of the mass had chalky mass with sebum like material, hair, and tooth. He was diagnosed with mediastinal nonseminoma. What treatment should be offered to this patient next?
Choices: 1. 4 cycles of adjuvant therapy with bleomycin, etoposide, and cisplatin 2. Radiation therapy 3. Surveillance 4. Combined chemotherapy with 3 cycles of paclitaxel, ifosfamide, and cisplatin with radiation.
Answer: 3 - Surveillance Explanations: The patient has mature mediastinal teratoma (nonseminoma). Surgical excision is the treatment of choice and is curative. After excision, surveillance alone is advised for mature teratoma. Mature teratomas are resistant to chemotherapy, and hence there is no role for adjuvant chemotherapy. Mature teratomas are known to be radiotherapy resistant tumors as well. Go to the next page if you knew the correct answer, or click the link image(s) below to further research the concepts in this question (if desired).
Research Concepts: Mediastinal Nonseminoma
We update eBooks quarterly and Apps daily based on user feedback. Please tap flag to report any questions that need improvement.
Question 22:
A patient presents with weight loss, anemia, and fatigue. Blood work reveals a white blood cell count of 88,000 cells/mm3 and a hemoglobin of 9 grams/dL. He has a mildly enlarged spleen and liver. Cytogenetic testing reveals a chromosomal translocation resulting in a BCR-ABL1 fusion gene. What is the most likely diagnosis?
Choices: 1. 2. 3. 4.
Acute myeloid leukemia (AML) Acute lymphoblastic leukemia (ALL) Chronic myeloid leukemia (CML) Chronic lymphocytic leukemia (CLL)
Answer: 3 - Chronic myeloid leukemia (CML) Explanations: The translocation is t(9;22)(q34;22q11) and results in the Philadelphia chromosome. This results in the BCR (breakpoint cluster region) gene from chromosome 22 fusing with the ABL1 gene of chromosome 9. Abl is from "Abelson," a leukemia virus that has a similar protein. The Philadelphia chromosome is seen in 95% of patients with chronic myeloid leukemia (CML), which is detected by traditional karyotype. The remainder are positive for cryptic BCR-ABL1 fusion gene, which is detected by fluorescence in situ hybridization or reverse transcriptase-polymerase chain reaction. The Philadelphia chromosome also is found in B-cell acute lymphoblastic leukemia (B-ALL), acute myeloid leukemia (AML), and mixed phenotype acute leukemia (MPAL). Go to the next page if you knew the correct answer, or click the link image(s) below to further research the concepts in this question (if desired).
Research Concepts: Chronic Myelogenous Leukemia
We update eBooks quarterly and Apps daily based on user feedback. Please tap flag to report any questions that
need improvement.
Question 23:
A 52-year-old man presents with a threemonth history of a right thigh mass. On exam, there is a firm nonmobile mass in the right anterior thigh with mild discomfort on palpation. An MRI demonstrates a 5.6 cm infiltrative mass with heterogeneous fat signal on T1 sequence in the anterior compartment of the thigh without neurovascular involvement. A core biopsy is performed, and a high-grade malignancy is discovered. A CT of the chest is negative for metastatic disease. What is the most appropriate treatment?
Choices: 1. 2. 3. 4.
Surgical resection followed by adjuvant radiotherapy Preoperative chemotherapy followed by surgical resection Preoperative radiotherapy followed by surgical resection Surgical resection alone
Answer: 3 - Preoperative radiotherapy followed by surgical resection
Explanations: This patient has a high-grade liposarcoma of the thigh, which have a high rate of local recurrence with resection alone. Preoperative radiotherapy followed by surgical resection is the most appropriate treatment modality in these patients, allowing for smaller radiation fields with lower doses resulting in less fibrosis and functional impairment. Sarcomas make up about 1% of all adult malignancies. Liposarcomas are commonly found in the extremities or retroperitoneum. Their MRI characteristics will vary based on the grade of the lesion, with lower grade lesions having higher fat content and less infiltrative and higher grade lesions containing less fat and being more infiltrative. Patients with sarcoma must be treated in a multidisciplinary setting. Surgical planning for sarcoma resection should have the goal of limb preservation in mind, potential for an R0 resection, and neurovascular involvement. Adjuvant radiotherapy can be performed, but it is not preferred as the patient will require a higher dose, a larger radiation field, and higher rates of functional impairment. Preoperative chemotherapy can also be given, but it is typically recommended that the patients also receive adjuvant radiotherapy. Definitive chemoradiation is typically not recommended unless the tumor is unresectable or in the context of a clinical trial.
Go to the next page if you knew the correct answer, or click the link image(s) below to further research the concepts in this question (if desired).
Research Concepts: Liposarcoma
We update eBooks quarterly and Apps daily based on user feedback. Please tap flag to report any questions that need improvement.
Question 24:
A 65-year-old male with history of stage IIIb squamous cell cancer of the lung presents with increasing dyspnea on exertion and orthopnea ongoing for four weeks with gradual worsening. On presentation, he is noted to be 90% on ambient air and is tachypneic to a respiratory rate of 30/minute. Labs are at baseline. A chest x-ray demonstrates large pleural effusion with contralateral mediastinal shift. He undergoes large-volume thoracentesis of 3 liters with immediate relief of dyspnea. The next morning, he develops respiratory distress with new hypoxia requiring supplemental oxygen. What is the most likely cause of the patient's newly developed respiratory distress?
Choices: 1. 2. 3. 4.
Pneumothorax ex vacuo Hemothorax Reexpansion pulmonary edema Pulmonary embolism
Answer: 3 - Reexpansion pulmonary edema Explanations: Reexpansion pulmonary edema occurs after the removal of fluid or air from the pleural space quickly over a short period of time. The mechanism of edema is believed to be increased capillary permeability. Risk factors for this condition include young age, a long duration of lung collapse, and rapid reexpansion. Treatment is largely supportive. Thoracentesis operators should not perform largevolume thoracentesis. No more than 1.5 liters of fluid should be removed in any instance. Go to the next page if you knew the correct answer, or click the link image(s) below to further research the concepts in this question (if desired).
Research Concepts: Malignant Effusion
We update eBooks quarterly and Apps daily based on user feedback. Please tap flag to report any questions that need improvement.
Question 25:
A 45-year-old female presents to the clinic with a very tender lesion under her right second toenail. The patient has trouble wearing shoes and touching the area due to the pain. Physical exam reveals a blueish growth underneath the nail bed. If the lesion is due to a tumor, which of the following tissue is it most likely to originate from?
Choices: 1. 2. 3. 4.
Light touch receptors Vibration and proprioception receptors Thermoregulation smooth muscle Nail matrix formation
Answer: 3 - Thermoregulation smooth muscle Explanations: This neoplasm is consistent with the diagnosis of a Glomus tumor, modified smooth muscle cells of a glomus body. Glomus bodies are encapsulated neurovascular organs found in the dermis of the nail bed. They function by thermoregulation through shunting blood away from the skin surface in cold temperatures. This is done to minimize heat loss through nails. In warmer temperatures, they increase blood flow through nails to help facilitate dissipation of heat. Go to the next page if you knew the correct answer, or click the link image(s) below to further research the concepts in this question (if desired).
Research Concepts: Anatomy, Bony Pelvis and Lower Limb, Toe Nails
We update eBooks quarterly and Apps daily based on user feedback. Please tap flag to report any questions that need improvement.
Question 26:
A 65-year-old male presents with a cough and hemoptysis for four weeks. CT showed a 1.5 cm right upper lobe lesion and normal-sized lymph nodes. PET-CT showed fludeoxyglucose (FDG) uptake in the mass and right hilar lymph node. What is the next step in his care?
Choices: 1. 2. 3. of 4. of
Do preoperative clearance for surgery Refer to oncology Perform convex probe endobronchial ultrasound sampling hilar lymph node Perform convex probe endobronchial ultrasound sampling mediastinal and hilar lymph nodes
Answer: 4 - Perform convex probe endobronchial ultrasound sampling of mediastinal and hilar lymph nodes
Explanations: This is a high-risk patient for lung cancer. According to PET-CT, he has stage IIB disease; therefore, surgery is an option. Before surgery, N2 and N3 diseases should be ruled out. PET showed uptake in only an N1 node, but PET has a significant false-negative rate. As per ACCP guidelines, N1 disease on PET should be confirmed with tissue biopsy. The sampling of a hilar lymph node will only give a diagnosis but not staging. Preoperative clearance is only done after ruling out metastatic and extensive disease. Go to the next page if you knew the correct answer, or click the link image(s) below to further research the concepts in this question (if desired).
Research Concepts: Lung Cancer
We update eBooks quarterly and Apps daily based on user feedback. Please tap flag to report any questions that need improvement.
Question 27:
A 51-year-old Asian female with a history of gastric cancer and peritoneal metastasis was treated two months ago with extensive cytoreductive surgery and intraperitoneal chemotherapy. She had an unremarkable perioperative course in the hospital. She now presents to the clinic with a history of fever with chills for 2 days. Her temperature is 101 F, pulse is 103 bpm, blood pressure is 116/70 mm Hg, and her respiratory rate is 20 breaths per minute. Her O2 saturation on pulse oximetry is 96%. The abdominal exam was unremarkable except for the presence of a postoperative surgical healing incision. The cardiopulmonary exam was unremarkable as well. Which of the following abnormal laboratory finding do you expect to see in this patient?
Choices: 1. 2. 3. 4.
Low hemoglobin Proteinuria Elevated lipase Neutropenia
Answer: 4 - Neutropenia Explanations: Anemia is not expected in this context unless it was present preoperatively or resulted from intraoperative blood loss. The question does not offer this information. Instead, the patient had a normal perioperative recovery and was treated with cytoreductive surgery in combination with intraperitoneal chemotherapy for her locally advanced gastric cancer. Cytoreductive surgery, in combination with intraperitoneal chemotherapy, is the newer successful treatment approach in case of gastric cancer with locoregional peritoneal metastasis. While myelosuppression is a common complication of therapy with the chemotherapeutic agents used, proteinuria from protein-losing nephropathy is not common or expected. The patient’s clinical presentation of fever with chills and a hyperdynamic circulatory state with tachycardia and a slight drop in blood pressure in the absence of localized symptoms suggest systemic infection. An elevated lipase is more commonly associated with pancreatic inflammation, and there is no history of symptoms or clinical signs consistent with the diagnosis of pancreatitis. Cytoreductive surgery, in combination with intraperitoneal chemotherapy, is a contemporary treatment for gastric cancer with loco-regional peritoneal metastasis. Either oxaliplatin or mitomycin are the drugs of choice as a chemotherapeutic agent for hyperthermic intraperitoneal chemotherapy. Both of these drugs could cause neutropenia in 1/3 of patients. This patient’s clinical presentation with signs of sepsis
and systemic infection. Due to the decrease in white blood cell count, this patient is immunocompromised and prone to any type of infection. Go to the next page if you knew the correct answer, or click the link image(s) below to further research the concepts in this question (if desired).
Research Concepts: Peritoneal Metastasis
We update eBooks quarterly and Apps daily based on user feedback. Please tap flag to report any questions that need improvement.
Question 28:
A red 5 mm well-circumscribed lesion is present under a fingernail. It is very painful on palpation. What is the most likely diagnosis?
Choices: 1. 2. 3. 4.
Fibroma Nevus Angioma Glomus tumor
Answer: 4 - Glomus tumor Explanations: A glomus tumor is a benign growth of a normally occurring arteriovenous anastomosis of the finger. It is often located under the nail and is very tender on palpation. Glomus tumor is a very rare lesion of the soft tissues. It usually occurs in young adults. The lesion is typically small and appears dark blue, red, or black. It can be found on the extremities and under the nails. These lesions can be very painful to palpation. In many cases, changes in temperature or pressure can induce paroxysmal pain. Go to the next page if you knew the correct answer, or click the link image(s) below to further research the concepts in this question (if desired).
Research Concepts: Glomus Cancer
We update eBooks quarterly and Apps daily based on user feedback. Please tap flag to report any questions that need improvement.
Question 29:
A 33-year-old female presents with pain in the lower abdomen. She says the pain has been ongoing for the past 2 months, and she feels full in the lower pelvic area. At times, she has had difficulty holding urine and has been constipated. X-rays and bloodwork are unremarkable. Ultrasound shows a 7 cm cyst of the left ovary. Laparoscopy is done, and the fluid has no evidence of malignancy. She most likely has which of the following conditions?
Choices: 1. 2. 3. 4.
Endometrioma Tubo-ovarian abscess Serous cystadenoma Cystic teratoma
Answer: 3 - Serous cystadenoma Explanations: Ovarian cysts are quite common and may be stimulated by sex hormones. Functional cysts are usually benign. Multiple cysts can occur in those with diabetes mellitus and individuals with a hydatidiform mole. Malignant ovarian cysts cause severe pain, abdominal distension, bowel obstruction, wasting, and abnormal uterine bleeding. Neoplastic ovarian cysts can arise from the surface epithelium and may be partially cystic in nature. Benign cysts can cause pain and discomfort and may twist, rupture, or hemorrhage. Teratomas are a type of germ tumor with all three embryonic germ layers. Endometriomas are filled with blood. Go to the next page if you knew the correct answer, or click the link image(s) below to further research the concepts in this question (if desired).
Research Concepts: Ovarian Cystadenoma
We update eBooks quarterly and Apps daily based on user feedback. Please tap flag to report any questions that need improvement.
Question 30:
A 62-year-old male undergoes a partial penectomy for a 2.5 cm, grade 3, squamous cell carcinoma of the penis. Pathology reveals invasion of the corpus cavernosum and negative margins. On physical exam, he has matted, firm 6 cm right inguinal lymph nodes. A computed tomography (CT) scan of the chest, abdomen, and pelvis reveals right-sided inguinal adenopathy but no other lesions. What is the next step in management for this patient?
Choices: 1. Fine needle aspiration of the inguinal lymph nodes 2. Right inguinal lymph node dissection followed by radiation therapy 3. Bilateral inguinal lymph node dissection 4. Neoadjuvant chemotherapy followed by bilateral inguinal lymph node dissection
Answer: 4 - Neoadjuvant chemotherapy followed by bilateral inguinal lymph node dissection
Explanations: Men with penile cancer who present with bulky, fixed inguinal nodes or pelvic lymph nodes have relatively low survival rates when treated with surgery alone and are candidates for multimodal treatment strategies. Neoadjuvant chemotherapy is indicated for patients with difficult to resect matted or fixed inguinal lymph nodes from metastatic penile cancer. The TIP regimen is the standard regimen used and consists of paclitaxel, ifosfamide, and cisplatin. This is given at 3 to 4-week intervals for 4 cycles. Neoadjuvant chemotherapy should reduce the size of the nodes and increase their resectability. Following chemotherapy, the patient should have bilateral inguinal lymph node dissection and pelvic lymph node dissection. Go to the next page if you knew the correct answer, or click the link image(s) below to further research the concepts in this question (if desired).
Research Concepts: Penile Cancer
We update eBooks quarterly and Apps daily based on user feedback. Please tap flag to report any questions that
need improvement.
Question 31:
A 60-year-old male with a history of smoking presents to the emergency department with a cough for the past 6 months. He reports weight loss, hemoptysis, and hoarseness. The patient denies recent travel, recent illness, or a new exercise program. On exam, the patient is afebrile, and his vitals are within normal limits. The provider notes anisocoria with the left pupil at 2 mm and the right pupil at 4 mm. Additionally, the provider notes drooping of the left upper eyelid. During a workup for the most likely diagnosis, what additional finding can be expected?
Choices: 1. 2. 3. 4.
Diffuse end expiratory wheezing Flushing and dryness of the left face Rhonchi and green sputum production Positive anti-acetylcholine receptor antibody
Answer: 2 - Flushing and dryness of the left face Explanations: This patient's presentation is suspicious for a Pancoast tumor, which is a cause of Horner syndrome. Additionally, Pancoast tumors can cause hoarseness secondary to recurrent laryngeal nerve compression. Horner syndrome is characterized by the classic triad of unilateral ptosis (drooping of the eyelid), miosis (constricted pupil), and anhydrosis (flushing and dryness to the face). Horner syndrome is due to the disruption of the sympathetic nervous innervation to the head, including disruption of the pupillary dilation pathway. In subtle presentations of Horner syndrome, cocaine eye drops can be applied to the affected eye. If there is an absence of pupillary dilation, in addition to the presence of ptosis, miosis, and anhydrosis, Horner syndrome can be confirmed. Go to the next page if you knew the correct answer, or click the link image(s) below to further research the concepts in this question (if desired).
Research Concepts: Horner Syndrome
We update eBooks quarterly and Apps daily based on user feedback. Please tap flag to report any questions that
need improvement.
Question 32:
A 6-year-old male develops a rapidly enlarging jaw lesion and increased abdominal girth. A biopsy of the lesion reveals a monomorphic population of intermediate-sized lymphocytes, with a basophilic nucleolus and increased mitotic figures. Derangement of which gene is most likely responsible for this tumor?
Choices: 1. 2. 3. 4.
BCR-ABL DDX1 WT1 c-MYC
Answer: 4 - c-MYC Explanations: Patients with Burkitt lymphoma (BL) often present with a rapidly growing mass, elevated lactate dehydrogenase (LDH), and increased uric acid levels because of the tumor’s rapid doubling time. The primary site of sporadic BL is typically the abdomen, but the head and neck may be affected as well. Endemic BL characteristically presents with an enlarging jaw lesion, periorbital swelling, or genitourinary involvement. Jaw involvement is seen predominantly in children. Burkitt lymphoma is a hematologic malignancy that presents in children often as a jaw or abdominal mass that is rapidly growing. Commonly there is translocation and deregulation of the c-MYC gene on chromosome 8 leading to reduced apoptosis and a high rate of tumor cell proliferation. The BRC-ABL gene is associated with leukemia, the DDX1 gene is associated with rhabdomyosarcoma and the WT1 gene is associated with nephroblastoma. Go to the next page if you knew the correct answer, or click the link image(s) below to further research the concepts in this question (if desired).
Research Concepts: Burkitt Lymphoma
We update eBooks quarterly and Apps daily based on user feedback. Please tap flag to report any questions that need improvement.
Question 33: Choices: 1. 2. 3. 4.
Lower limbs Along the spine Trunk Head and neck
Where is a cylindroma most likely found?
Answer: 4 - Head and neck Explanations: Cylindroma is a rare, benign appendage tumor that presents on the scalp in adulthood. Malignant transformation is very rare. Single lesions are usually seen after the fifth decade of life. Multiple lesions are more likely to be seen in early adulthood and then increase in number progressively. Multiple cylindromas are associated with Brooke Spiegler syndrome. Go to the next page if you knew the correct answer, or click the link image(s) below to further research the concepts in this question (if desired).
Research Concepts: Cylindroma
We update eBooks quarterly and Apps daily based on user feedback. Please tap flag to report any questions that need improvement.
Question 34:
A 67-year-old black male presents to the clinic for his annual diabetic foot examination. He complains of thick, yellowed toenails that rub in his shoes. Lower extremity physical examination reveals decreased sensation in both feet, palpable popliteal and pedal pulses. There is a small black nevus on his right heel that exhibits a parallel ridge dermoscopic pattern. Which of the following types of skin cancer has the lowest 5-year survival rate?
Choices: 1. 2. 3. 4.
Basal cell carcinoma Acral lentiginous melanoma Squamous cell carcinoma Superficial spreading melanoma
Answer: 2 - Acral lentiginous melanoma Explanations: Most benign acral nevi typically exhibit either a parallel furrow, lattice or fibrillar dermoscopic patterns. The parallel ridge pattern predicts acral melanoma in 94 percent of cases. Within the lower extremity, pedal melanoma has a poorer prognosis than melanomas occurring on the thigh. Lesion asymmetry, border notching, color changes, increasing diameter, and evolution or change in the nevus are all suspicious signs of melanoma that can be appreciated on clinical examination. Go to the next page if you knew the correct answer, or click the link image(s) below to further research the concepts in this question (if desired).
Research Concepts: Melanocytic Nevi
We update eBooks quarterly and Apps daily based on user feedback. Please tap flag to report any questions that need improvement.
Question 35:
A 32-year-old multiparous female presented with complaints of repeated spotting for the last 3 months. She had no history of oral contraceptive intake or irregular menstruation. Vaginal examination showed an irregular growth at the lower lip of the cervix. On rectal examination, no adhesion or thickening was palpated on both the left and right parametrium. Contrast computed tomography scan of the pelvis was done, revealing an irregular dense mass of 3.5 × 3 × 1.5 cm occupying the inferior lip of the cervix and posterior fornices. No pelvic extension and no regional lymphadenopathy were noted on the CT scan. A punch biopsy was taken and sent for histopathological examination. Histopathology revealed partly solid and tubulo-cystic growth pattern of neoplastic cells. The tumor cells had abundant clear cytoplasm, round nuclei, and hobnailing appearance in many cells. The nuclei showed hyperchromasia, moderate pleomorphism, and prominent nucleoli. No other organ involvement was noted. What will be the best treatment for this patient?
Choices: 1. 2. 3. 4.
Radical hysterectomy and pelvic lymphadenectomy Vaginal hysterectomy Radiotherapy Chemotherapy and radiotherapy
Answer: 1 - Radical hysterectomy and pelvic lymphadenectomy
Explanations: In this case, the histopathological examination confirmed the diagnosis of clear cell carcinoma of the uterine cervix. Surgery is the treatment of choice for patients with early-stage carcinoma of the uterine cervix. Patients with early-stage cervical carcinoma FIGO ( either stage IB or IIA) could be treated by radical hysterectomy and pelvic lymphadenectomy. In this case, the tumor is classified as stage IB since the tumor is strictly confined to the uterine cervix and does not extend onto the lower third of the vagina or to the pelvic wall. Moreover, no regional lymphadenopathy was noted. For early-stage cervical cancer, fertility-sparing surgery is a safe option. This was confirmed by a recent study that studied oncologic outcomes of fertility-sparing radical trachelectomy versus radical hysterectomy for stage IB1 cervical carcinoma in young adult women and found no statistically significant survival differences. For patients with stage I cervical cancer, metastasis occurs in about 18 percent of the cases. After treatment of clear cell carcinoma of the cervix, long-term follow-up is necessary to detect local and distant relapse. Go to the next page if you knew the correct answer, or click the link image(s) below to further research the concepts in this question (if desired).
Research Concepts:
Cervical Clear Cell Carcinoma
We update eBooks quarterly and Apps daily based on user feedback. Please tap flag to report any questions that need improvement.
Question 36:
A 66-year-old female is admitted for a five days history of fever, bloody diarrhea, and abdominal pain. The patient has acute myeloid leukemia, and the last chemotherapy she received was two weeks ago as an inpatient. While the patient was in the hospital two weeks ago, she had a UTI and was treated with IV antibiotics. On physical examination, her temperature is 38.5 C (101.3 F), blood pressure 95/50 mmHg, pulse rate 103/min, and respiratory rate is 15/min. The abdominal exam shows diffuse tenderness. Imaging showed thickened bowel wall of the cecum and ascending colon, fat stranding, and air under the diaphragm. Her labs show hemoglobin (Hb) 7 mg/dL, white blood cells (WBC) 300/micoL, and platelets 15000/microL. What is the next best step in the management of this patient?
Choices: 1. 2. 3. 4.
Stool analysis and culture Bowel rest and Nasogastric suctioning Evaluation for surgery Oral vancomycin
Answer: 3 - Evaluation for surgery Explanations: The patient presented with symptoms of colitis (fever, abdominal pain, and diarrhea) and in such patient stool analysis and culture is part of the workup. However, the patient has concerning signs, including the hemodynamic instability and the air under diaphragm seen on CT imaging. So the most important next step in management is to be evaluated by surgery for possible bowel perforation. Bowel rest and NG suctioning is part of the management of any patient with colitis, but the most appropriate next step the patient needs is to be evaluated by surgery for early intervention. She might have had neutropenic enterocolitis or colitis due to other infectious etiology. Regardless of the cause, the patient has hemodynamic instability, and CT imaging shows a sign of bowel perforation (air under diaphragm) and surgical evaluation is needed to do intervention as soon as possible to decrease complications and mortality. The patient has colitis of which the cause is still unknown. Clostridium difficile testing should be done as part of the workup, and if positive, she will be given oral vancomycin. However, the patient has complicated colitis that needs to be evaluated by surgery urgently. Go to the next page if you knew the correct answer, or click the link image(s) below to further research the concepts in this question (if desired).
Research Concepts:
Neutropenic Enterocolitis (Typhlitis)
We update eBooks quarterly and Apps daily based on user feedback. Please tap flag to report any questions that need improvement.
Question 37:
A 60-year old farmer presents with several lesions on his head, neck, abdomen, and nose. He says he has had these skin lesions for many years but they do not seem to be going away. He has been a farmer all his life and has no other pertinent medical history. On physical exam, there are two lesions that appear waxy and have a central depression. The edges have a pearly appearance. Excisional biopsy of one of the lesions reveals that it is a malignancy. This type of malignancy has often been linked to intake of what metal?
Choices: 1. 2. 3. 4.
Mercury Arsenic Lead Chromium
Answer: 2 - Arsenic Explanations: While the majority of basal cell cancers are due to sun exposure, they can also occur as a result of ionizing radiation. X-ray therapy was once used to treat acne and resulted in basal cell cancers. Exposure to arsenic has also been linked to basal skin cancer of note it is seen in sun-exposed and non-sun exposed parts of the body. Treatment of chronic arsenic poisoning is possible. British anti-lewisite is prescribed in doses of 5 mg/kg up to 300 mg every 4 hours for the first day, then every 6 hours for the second day, and finally every 8 hours for 8 additional days. Blood, hair, nails, and urine may be tested for arsenic; however, these tests cannot foresee possible health outcomes from the exposure. Long-term exposure and consequent excretion have been linked to bladder and kidney cancer in addition to cancer of the liver, prostate, skin, and lungs. Go to the next page if you knew the correct answer, or click the link image(s) below to further research the concepts in this question (if desired).
Research Concepts: Arsenic Toxicity
We update eBooks quarterly and Apps daily based on user feedback. Please tap flag to report any questions that need improvement.
Question 38:
A 15-year-old female presents with increased snoring and difficulty articulating, which she attributes to her tongue. She states that her tongue has slowly enlarged over the last 5 weeks. Her tongue appears to be enlarged, although it does not protrude from her mouth and has no visually obvious lesions. She denies trauma to the tongue. Given her presentation, which of the following is the most likely etiology for her enlarging tongue?
Choices: 1. 2. 3. 4.
Down syndrome Hyperthyroidism Lymphangioma Lymphoma
Answer: 3 - Lymphangioma Explanations: Lymphangioma is a rare cause of macroglossia, and physical examination clues the provider into its likely presence. There can be diffuse enlargement with or without vesicles. The patient's history is consistent with the presence of lymphangioma but is not detailed enough to necessarily rule out all of the other choices. Intraoral lymphangiomas occur more frequently on the dorsum of the tongue, followed by palate, buccal mucosa, gingiva, and lips. The anterior two-thirds on the dorsal surface of the tongue is the most common site for intraoral lymphangiomas leading to macroglossia. Go to the next page if you knew the correct answer, or click the link image(s) below to further research the concepts in this question (if desired).
Research Concepts: Lymphangioma
We update eBooks quarterly and Apps daily based on user feedback. Please tap flag to report any questions that need improvement.
Question 39:
A 67-year-old female complains of a pruritic rash in her genital region. Physical exam is notable for a 5-centimeter erythematous, well-demarcated plaque involving her labia majora. A biopsy reveals clusters of large, anaplastic cells infiltrating the epidermis with increased cytoplasm separating them from normal cells. The anaplastic cells stain for mucin. Immunohistochemical staining is performed. Which of the following immunohistochemical stains are positive in this condition?
Choices: 1. Cytokeratin 7+, gross cystic disease fluid protein 15 (GCDFP-15), periodic acid–Schiff (PAS), and carcinoembryonic antigen (CEA) 2. S100 , Melan-A , HMB-45 3. CD3+, CD4+ 4. CK5/6+, CK7
Answer: 1 - Cytokeratin 7+, gross cystic disease fluid protein 15 (GCDFP-15), periodic acid–Schiff (PAS), and carcinoembryonic antigen (CEA)
Explanations: Extramammary Paget disease has an important association between another underlying malignancy in at least 10% to 30% of cases, so a thorough investigation to rule out malignancy is critical. With extramammary Paget disease, the histologic findings are similar to anaplastic cells that infiltrate the dermis separated by a clear space from the normal epithelium. The patter is often referred to as the pagetoid spread of neoplastic cells within the epidermis. Immunohistochemical staining is used to differentiate extramammary Paget disease from other disease entities with similar pagetoid distribution on histology. Cytokeratin 7+, CK20-, CEA+, BerEP4+ helps to distinguish from Pagetoid Bowen disease. Pagetoid Bowen disease staining includes CK5/6+, CK7- (usually), BerEP4- (always). Melanoma in situ staining includes S100, Melan-A, HMB-45+++. Mycosis Fungoides staining includes CD3+, CD4+, CD8-. Often has loss of CD5 and CD7. Go to the next page if you knew the correct answer, or click the link image(s) below to further research the concepts in this question (if desired).
Research Concepts: Extramammary Paget Disease
We update eBooks quarterly and Apps daily based on user feedback. Please tap flag to report any questions that need improvement.
Question 40:
A 47-year old male presents to the hospital with melena. Colonoscopy is unremarkable for any mass or lesion. He undergoes laparoscopic appendectomy. On gross examination, no characteristic features are seen after resection. Serial sectioning of appendix revealed an easily bleeding tiny polypoid mucosa. The microscopic diagnosis of florid vascular proliferation (FVP) is established. What typical histopathologic features of Kaposi sarcoma differentiate it from FVP?
Choices: 1. Proliferation of capillaries, veins or lymphatics, usually involving the mucosa or submucosa only 2. Infiltrative, anastomosing vascular channels, with cytologic atypia and brisk mitotic activity 3. Compressed, slit-like vascular channels with extravasated erythrocytes, hemosiderin and plasma cells; mitoses are common but pleomorphism is usually minimal 4. The granulation tissue type reactive proliferation of vasculature endothelial cells involving the entire thickness of bowel wall and showing positivity for CD31 and CD34 and negativity for HHV-8 (LANA1)
Answer: 3 - Compressed, slit-like vascular channels with extravasated erythrocytes, hemosiderin and plasma cells; mitoses are common but pleomorphism is usually minimal
Explanations: Kaposi sarcoma is a malignant vascular neoplasm, associated with human herpesvirus-8 (HHV-8), involving people with immune deficiency states. Microscopically shows compressed, slit-like vascular channels with extravasated erythrocytes, hemosiderin, and plasma cells. Mitoses are common, but pleomorphism is usually minimal. Florid vascular proliferation (FVP) demonstrates lobular granulation tissue-type florid proliferation of small vascular channels, lined by plump endothelial cells extended from the submucosa through the entire thickness of the bowel wall. The endothelial cells have minimal nuclear atypia, and few to no mitotic figures. On immunohistochemistry show positivity for CD31 and CD34 and negativity for HHV-8 (LANA1). Hemangiolymphomas (HLAs), are benign vascular lesions and commonly present as polypoid lesions, ranging from 0.5 cm to 3.5 cm. Histologically HLAs show proliferation of capillaries, veins or lymphatics, usually involving the mucosa or submucosa only and is not transmural. Angiosarcoma more commonly involves the small intestine, having an epithelioid appearance, where the epithelioid cells are arranged in sheets with only subtle vasoformation. Histopathology shows infiltrative, anastomosing vascular channels, with cytologic atypia and brisk mitotic activity.
Go to the next page if you knew the correct answer, or click the link image(s) below to further research the concepts in this question (if desired).
Research Concepts: Benign Florid Vascular Proliferation Mimicking Vascular Neoplasm
We update eBooks quarterly and Apps daily based on user feedback. Please tap flag to report any questions that need improvement.
Question 41:
A 45-year-old G0P0 female presents to the clinic with worsening intermittent pelvic pain over the past three months. There seems to be no alleviating or aggravating factors associated with the pain. She denies any abnormal vaginal discharge, change of bowel movements, or urinary habits. Her last menstrual period was three weeks ago, and she reports that it is getting a bit irregular this year. She is monogamous with her partner and denies any history of sexually transmitted infections. Past medical history is notable for a history of ruptured extrauterine pregnancy that led to resection of the left fallopian tube eight years ago. Physical examination shows normal vital signs and a palpable nontender right pelvic mass. A quick pregnancy test is negative. CT scan reveals a 4 cm cystic ovarian mass, dilation of the appendiceal lumen, and a moderate amount of ascites. The patient then undergoes a diagnostic laparoscopy which discovers a necrotic and detachable mass over the right ovary. There are also multiple gelatinous implants over the peritoneal surface as well as in the pelvis. The right ovary is resected, generous biopsies are performed, and the pathology is still pending. Which of the following is the most common cause of the potential disease?
Choices: 1. 2. 3. 4.
Ovarian mucinous cystadenoma Appendiceal mucocele Endometriosis with myxoid change Krukenberg tumor
Answer: 2 - Appendiceal mucocele Explanations: This is a typical case of pseudomyxoma peritonei (PMP). Females are more frequently affected than males, and the peak age is around 53. The initial presentation can be a newly discovered pelvic mass or atypical pelvic pain, which can be confused with gynecological disorders. PMP most commonly arises from an appendiceal cystadenocarcinoma. Initially, tumor cells originating from the mucinous epithelium of the appendix continuously produce mucus into the appendiceal lumen and form a mucocele, which eventually ruptures. Due to the absence of adhesive properties, tumor cells can spread all over the peritoneal cavity and pelvis, forming multiple gelatinous implants. Diagnostic laparoscopy and generous biopsies are the most reliable methods to confirm the diagnosis. Apart from the appendix, PMP can also arise from small and large bowel, stomach, pancreas, lung, breast, gallbladder, fallopian tubes, and ovaries, but with a much lower frequency. Ovarian cystadenoma can be a cause of PMP, but the ovarian lesion described in this case is more likely originate from other places since it's easily detachable from the ovary. History of extrauterine pregnancy does increase the risk of endometriosis, but the patient does not present with cyclic pelvic pain and the peritoneal implants of endometriosis are often necrotic rather than gelatinous. Krukenberg tumor represents bilateral metastases to ovaries from mucin-secreting gastric
cancer with abundant signet ring cells. The patient does not present with symptoms of end-stage gastric cancer. Go to the next page if you knew the correct answer, or click the link image(s) below to further research the concepts in this question (if desired).
Research Concepts: Pseudomyxoma Peritonei
We update eBooks quarterly and Apps daily based on user feedback. Please tap flag to report any questions that need improvement.
Question 42:
A 38-year-old man with metastatic colon cancer has undergone a bilateral cordotomy for refractory opioid-resistant pain. The patient has no other significant prior medical conditions, and a percutaneous cervical approach for the procedure was chosen. In the clinic, his vital signs are within normal reference ranges. He reports improvement to his somatic pain. However, he describes a new "annoyance" with eating and drinking. Regarding the risk to surrounding structures, given the anatomic location most commonly targeted with this procedure, what deficit is most expected as leading to his new complaint regarding oral intake?
Choices: 1. 2. 3. 4.
A severe tremor in his upper extremities Intractable nausea with oral intake Difficulty with initiating deglutition Ageusia leading to anorexia
Answer: 3 - Difficulty with initiating deglutition Explanations: In the percutaneous cervical cordotomy, the lateral spinothalamic tract anterolateral column is destroyed most often at the level of C1-C2. With the disruption of the lateral spinothalamic tract at C1-C2, the aim is for a contralateral disruption of painful sensations beyond C4. The geniohyoid muscle is innervated by fibers from the first cervical nerve. The geniohyoid muscle moves the hyoid bone up and forward facilitating the initiation of deglutition and also assisting respiration. Go to the next page if you knew the correct answer, or click the link image(s) below to further research the concepts in this question (if desired).
Research Concepts: Cordotomy
We update eBooks quarterly and Apps daily based on user feedback. Please tap flag to report any questions that need improvement.
Question 43:
A 49-year-old woman with Stage IA, T1c N0 M0, grade 2, ER positive, PR negative, HER2/neu negative breast cancer undergoes successful lumpectomy. A 21 gene assay score is 8. What is the most appropriate management?
Choices: 1. Observation 2. Endocrine therapy alone 3. Hypofractionated whole breast radiation therapy and endocrine therapy 4. Hypofractionated whole breast radiation therapy alone
Answer: 3 - Hypofractionated whole breast radiation therapy and endocrine therapy
Explanations: Radiation therapy is recommended after lumpectomy to reduce the risk of in-breast recurrences. Recent long-term data from multiple clinical trials support the safety and efficacy of hypofractionation in most patients regardless of age, tumor grade, left or right-sided breast cancer, prior chemotherapy, receptor status, and breast size. Radiotherapy and endocrine therapy can reduce the risk of local recurrence to a greater extent than when used alone in hormone receptor-positive patients. Endocrine therapy plus radiotherapy is recommended for those with hormone receptor-positive tumors. Older patients with significant comorbidities may not require radiotherapy. Go to the next page if you knew the correct answer, or click the link image(s) below to further research the concepts in this question (if desired).
Research Concepts: Radiation Therapy For Early Stage Breast Cancer
We update eBooks quarterly and Apps daily based on user feedback. Please tap flag to report any questions that need improvement.
Question 44:
A 25-year-old male with no known past medical history presents for an acute visit. He reports right knee pain that is progressive and started "months" ago. He denies any trauma to his knee and doesn't remember when he first noticed it. Physical exam reveals tenderness to palpation on the anteromedial aspect of the distal femur with associated edema, erythema, and soft-tissue expansion. Anterior and posterior drawer tests are both negative. No pain is elicited when the knee is put under valgus or varus stress. An anteroposterior radiograph of the right distal femur revealed an expansile lytic metaphysealepiphyseal lesion. The diagnosis of giant cell tumor was made, and treatment with denosumab was initiated. What is the recommended dosing for this specific patient?
Choices: 1. 120 mg subcutaneous injection every four weeks 2. 120 mg subcutaneous injection every four weeks with additional 120 mg doses on days 8 and 15 of the first month of therapy 3. 240 mg subcutaneous injection every four weeks 4. 240 mg subcutaneous injection every four weeks with additional 120 mg doses on days 8 and 15 of the first month of therapy.
Answer: 2 - 120 mg subcutaneous injection every four weeks with additional 120 mg doses on days 8 and 15 of the first month of therapy
Explanations: Denosumab is administered via subcutaneous injection and should not be administered intravenously or intramuscularly. Injection sites include the upper arm, upper thigh, or abdomen. No dosing adjustments are necessary for hepatic or renal impairment. Correct denosumab dosing for giant cell tumor of the bone is 120 mg injection every four weeks with additional 120 mg doses on days 8 and 15 of the first month of therapy. Correct denosumab dosing for skeletal-related events (e.g., bone pain and fractures) secondary to multiple myeloma and bone metastases from solid tumors is a 120 mg injection every four weeks. Correct denosumab dosing for osteoporosis and bone loss is 60 mg every six months. Go to the next page if you knew the correct answer, or click the link image(s) below to further research the concepts in this question (if desired).
Research Concepts: Denosumab
We update eBooks quarterly and Apps daily based on user feedback. Please tap flag to report any questions that need improvement.
Question 45:
A 25-year-old female patient presented with a 5-month history of irregular vaginal bleeding and mild breathing difficulties for one month. On examination, there was a decrease in the breath sounds in the left lower lobe. On gynecologic examination, there was an exophytic mass measuring 5.5×5.5 cm arising from the cervix involving up to the lower third of the vagina as well as both the parametrium. The histopathological evaluation of the cervical growth showed a papillary configuration with focal areas of necrosis. The papillae were mostly lined by a single layer of tumor cells having abundant clear cytoplasm with a centrally located round to oval nuclei. Computed tomography of abdomen and pelvis showed a 55×55 mm well-defined solid cystic mass in the uterine cervix with bladder wall infiltration. Computed tomography of thorax revealed bilateral pleural effusion. Pleural fluid cytology analysis revealed the presence of adenocarcinomatous cells. What is the FIGO classification of this cervical carcinoma?
Choices: 1. 2. 3. 4.
Stage Stage Stage Stage
IIIA IIIB IVA IVB
Answer: 4 - Stage IVB Explanations: According to the FIGO staging system of cervical cancer, stage IVB corresponds to the spread of cervical cancer to distant organs. In this case, clear cell carcinoma of the uterine cervix spread to the pleura, which corresponds to stage IVB. In this case, a pleural fluid cytology analysis revealed the presence of adenocarcinomatous cells. This confirms the metastatic spread of clear cell carcinoma of the uterine cervix to the pleura. The FIGO (International Federation of Gynecology and Obstetrics) staging system is used most often for cancers of the female reproductive organs, including cervical cancer. Due to the advanced stage, the patient should be planned for paclitaxel at 175 mg/m2 followed by intravenous infusion of carboplatin dosed to an area under the curve of six (maximum dose 600 mg) in day 2. Six cycles of chemotherapy are recommended. Go to the next page if you knew the correct answer, or click the link image(s) below to further research the concepts in this question (if desired).
Research Concepts: Cervical Clear Cell Carcinoma
We update eBooks quarterly and Apps daily based on user feedback. Please tap flag to report any questions that need improvement.
Question 46:
A 70-year-old male presents with a 3month history of dyspnea with a recent occurrence of hemoptysis. He has a past medical history of a 40 pack-year smoking history. His vitals are stable, and his physical examination is unremarkable. A computed tomography (CT) scan of the chest shows a 4.3 cm spiculated mass in the left upper lobe with an enlarged hilar lymph node. The patient undergoes surgical excision which shows that the mass is a poorly differentiated adenocarcinoma with negative margins. Hilar lymph nodes are positive, but mediastinal lymph nodes are negative. What is the most appropriate management?
Choices: 1. 2. 3. 4.
Palliative care Adjuvant chemotherapy Adjuvant chemoradiation Adjuvant radiation
Answer: 2 - Adjuvant chemotherapy Explanations: Treatment options for non-small-cell lung carcinoma (NSCLC) include chemotherapy, radiation, surgical resection, and combinations of these. This patient has options and should not be directed to palliative care at this time. This patient has stage II non-small cell lung cancer (NSCLC) as his mass is less than 5 cm and is localized. With stage II and stage III NSCLC, it is recommended to undergo adjuvant chemotherapy. Studies have not shown a survival benefit with chemoradiation for stage II NSCLC. Chemoradiation can be an option for stage III NSCLC with mediastinal lymphadenopathy. As this patient had negative surgical margins, it would not be recommended that he undergo radiation. Go to the next page if you knew the correct answer, or click the link image(s) below to further research the concepts in this question (if desired).
Research Concepts: Non Small Cell Lung Cancer
We update eBooks quarterly and Apps daily based on user feedback. Please tap flag to report any questions that need improvement.
Question 47:
A 58-year-old female presented with a three-month history of nipple discharge. She had no medical and family history regarding breast cancer. Physical examination disclosed a painless right breast mass in the upper outer quadrant with nipple and skin retraction. The mammogram showed a large high-density soft mass lesion in the upper outer quadrant of the right breast with speculated outline measuring (5.5 X 4.6 mm). Fine-needle aspiration showed invasive carcinoma. The patient underwent radical mastectomy with ipsilateral axillary lymph node dissection. Grossly, the tumor size was (6.5 X 4.5 cm) with no skin involvement. Histopathological examination revealed pleomorphic invasive lobular carcinoma. The immunohistochemical study confirmed the loss of expression of the cell-cell adhesion molecule Ecadherin. Five out of ten lymph nodes were metastatic. ER and PR receptors were positive, and Her2neu was negative. Chest, abdomen, and pelvis CT scan and bone scan were normal. What is the pTNM staging of this tumor?
Choices: 1. 2. 3. 4.
pT2N1cM0 pT3N1aM0 pT3N1bM0 pT3N2aM0
Answer: 4 - pT3N2aM0 Explanations: The largest diameter of the tumor on the surgery specimen was 6.5, which corresponds to T3. Five out of ten lymph nodes were metastatic, which corresponds to N2a. Chest, abdomen, and pelvis CT scan and bone scan were normal, which corresponds to M0. Thus the pump staging of this tumor is pT3N2aM0. The most widely used clinical staging system for invasive lobular breast carcinoma is the one adopted by both the International Union for Cancer Control (UICC) and the American Joint Commission on Cancer (AJCC). Its basis is in the TNM system (T, tumor; N, nodes; M, metastases). Various studies have shown that invasive lobular carcinoma has the same, better, or worse prognosis than invasive ductal carcinoma. Part of this may depend on the time of follow-up and sample size because lobular carcinoma is associated with lower early local recurrence rates and a lower rate of axillary lymph node involvement at the time of diagnosis. The metastatic pattern of invasive lobular carcinoma differs from that of invasive carcinoma of no particular type. A higher frequency of tumor extension to the bone, gastrointestinal tract, uterus, meninges, ovary, and diffuse serosal involvement is observed in invasive lobular carcinoma, while the extension to the lung is more frequent in invasive carcinoma of no particular type. Go to the next page if you knew the correct answer, or click the link image(s) below to further research the concepts in
this question (if desired).
Research Concepts: Lobular Breast Carcinoma
We update eBooks quarterly and Apps daily based on user feedback. Please tap flag to report any questions that need improvement.
Question 48:
A 55-year-old male presents after a 70 Gray course of radiation therapy for squamous cell carcinoma of the oral tongue with a non-healing oral lesion and exposed bone of the anterior lingual cortex of the mandible. He has stopped smoking five months before beginning his radiation therapy and is now in an alcohol treatment program, started at the same time. Which of the following is a significant risk factor for developing this complication?
Choices: 1. 2. 3. 4.
Prophylactic antibiotics were held during radiotherapy Total radiation dose less than 6,000 rads (cGy) A tooth extraction one month before starting radiotherapy Total radiation dose greater than 7,000 rads (cGy)
Answer: 4 - Total radiation dose greater than 7,000 rads (cGy)
Explanations: Of the listed options, a radiation dose greater than 7,000 rads (cGy) is an independent risk factor for the increased incidence of osteoradionecrosis (ORN) of the mandible. Below 6,000 rads total exposure is unlikely to develop ORN. A mechanical injury such as tooth extraction can increase the incidence of ORN if done anytime within two weeks of radiotherapy and indefinitely afterward. ORN is an aseptic disease process. Antibiotics only have a role in established secondary bacterial infections. Go to the next page if you knew the correct answer, or click the link image(s) below to further research the concepts in this question (if desired).
Research Concepts: Mandible Osteoradionecrosis
We update eBooks quarterly and Apps daily based on user feedback. Please tap flag to report any questions that need improvement.
Question 49:
A 26-year-old woman is evaluated for a lump in her neck. She discovered the lump a week ago while showering. The patient feels well and has no associated pain, fever, night sweats, or weight changes. Her medical history is unremarkable, although she has had irregular menses for a year since insertion of an intrauterine device. The patient does not use tobacco, alcohol, or illicit drugs. Her mother died during surgery for thyroid cancer. Blood pressure is 133/80 mm Hg and pulse are 78/min. On examination, there is a palpable 2-cm nodule in the right thyroid lobe. The remainder of the examination, including chest, abdomen, extremities, and skin, is normal. Serum TSH and calcium levels are normal, and calcitonin is elevated. Ultrasound-guided aspiration biopsy reveals malignant cells. Which of the following tests is the best next step in the evaluation of this patient?
Choices: 1. 2. 3. 4.
Serum prolactin assay Plasma fractionated metanephrine assay Insulin-like growth factor-1 The aldosterone-renin activity ratio
Answer: 2 - Plasma fractionated metanephrine assay Explanations: This patient with a malignant thyroid mass elevated serum calcitonin level, and family history of thyroid malignancy likely has an inherited form of medullary thyroid cancer (MTC). MTC is a calcitonin-producing tumor arising from neuroendocrine parafollicular C cells of the thyroid gland. Although most MTC is sporadic, approximately one-third of MTC is inherited as a component of multiple endocrine neoplasias (MEN) types 2A and 2B. Serum prolactin is the screening test for prolactin-secreting pituitary tumors. Pituitary tumors occur in MEN1 but are not a feature of MEN2. MEN2A and MEN2B are associated with pheochromocytoma, which can be asymptomatic at the time of diagnosis but cause life-threatening hypertensive crisis during surgical procedures (e.g., thyroidectomy). In light of this risk, most patients with MTC should undergo RET mutation testing and screen for pheochromocytoma with a plasma fractionated metanephrine assay. If found, pheochromocytoma should be resected before thyroidectomy. Serum insulin-like growth factor-1 is the screening test for growth hormone-secreting pituitary tumors. Pituitary tumors occur in MEN1 but are not a feature of MEN2. The aldosterone-renin activity ratio can help evaluate hyperaldosteronism (e.g., hypertension, hypernatremia, hypokalemia). This patient has no such manifestations. Also, adrenocortical tumors are uncommon in MEN syndromes, with biochemical testing recommended only in patients with manifestations of excessive hormonal production.
Go to the next page if you knew the correct answer, or click the link image(s) below to further research the concepts in this question (if desired).
Research Concepts: Multiple Endocrine Neoplasias Type 2
We update eBooks quarterly and Apps daily based on user feedback. Please tap flag to report any questions that need improvement.
Question 50:
The first-line FDA-approved agent to treat metastatic non-small cell lung cancer is being considered for a 67-year-old patient. It is most appropriate to evaluate this patient for which gene mutation before initiating treatment?
Choices: 1. 2. 3. 4.
KRAS p53 Fragile histidine triad (FHIT) Epidermal growth factor receptor (EGFR)
Answer: 4 - Epidermal growth factor receptor (EGFR) Explanations: Patients with metastatic non-small cell lung cancer (NSCLC) should be checked for epidermal growth factor receptor (EGFR) mutation because targeted therapy with tyrosine kinase inhibitors improves survival in patients with the mutation. Afatinib is a targeted therapy that irreversibly inhibits the ErbB family of tyrosine kinases. The first-line FDA-approved indication is to treat locally advanced or metastatic NSCLC that harbors nonresistant EGFR mutations. There are three known tyrosine kinase inhibitors (EGFR TKIs) widely used as a treatment for advanced NSCLC with proven efficacy: gefitinib, erlotinib, and afatinib. Go to the next page if you knew the correct answer, or click the link image(s) below to further research the concepts in this question (if desired).
Research Concepts: Afatinib
We update eBooks quarterly and Apps daily based on user feedback. Please tap flag to report any questions that need improvement.
Question 51:
A 46-year-old patient presents with persistent cough symptoms and normal lung functions. Further evaluation with computerized tomography ( CT) shows a 2.1 cm nodule with ground glass appearance. Positron emission tomography ( PET CT) showed a standardized uptake value (SUV) of 3.1 in the nodular area of the opacity with no hypermetabolic activity elsewhere. He is then sent for a CT guided transthoracic needle biopsy. Histopathology shows hyperplasia of type 2 pneumocytes with atypical cells replacing normal alveolar epithelium. There is no disruption or invasion of the alveolar wall. What is the best treatment approach for this patient?
Choices: 1. Wedge resection of the nodule only 2. Wedge resection and further decision on lobectomy based on frozen section 3. Wedge resection and lobectomy based on the frozen section along with a sampling of lymph nodes and assessment of pleura 4. Radiation with gamma knife
Answer: 3 - Wedge resection and lobectomy based on the frozen section along with a sampling of lymph nodes and assessment of pleura
Explanations: The histopathology specimen here describes adenocarcinoma in situ in the current classification of adenocarcinoma with lepidic growth. A diagnosis of adenocarcinoma with lepidic growth made by cytology or core biopsy, however, can be found to contain a more invasive component in an excised specimen. A thorough evaluation of lymph nodes and pleura, along with frozen section intraoperatively, helps in making the best treatment decision. There has been an increasing trend of sublobar resection for excision specimens of solitary adenocarcinoma in situ (AIS) with disease-free margins. A tumor that cannot be removed completely by resection should be assessed for epidermal growth factor receptor (EGFR ) mutation than planing for gamma knife therapy only. Go to the next page if you knew the correct answer, or click the link image(s) below to further research the concepts in this question (if desired).
Research Concepts: Bronchoalveolar Cancer
We update eBooks quarterly and Apps daily based on user feedback. Please tap flag to report any questions that need improvement.
Question 52:
A 30-year-old male patient presented with a 3-month history of pruritus. He received 5% permethrin treatment topically with no improvement. Physical examination revealed excoriated and umbilicated papules associated with central plugs on the legs. Histopathological examination of a skin biopsy specimen revealed altered collagen bundles perpendicular to a necrotic epidermis. There were parakeratotic cells, and inflammatory infiltrate. These findings were consistent with the diagnosis of acquired perforating dermatosis. One month later, the patient presented with axillary adenopathy. Lymph node biopsy was consistent with the diagnosis of Hodgkin lymphoma. What is the most appropriate long-term treatment for itching in this patient?
Choices: 1. 2. 3. 4.
Phototherapy Topical corticosteroids Chemotherapy Hemodialysis
Answer: 3 - Chemotherapy Explanations: The diagnosis of acquired perforating dermatosis associated with Hodgkin lymphoma was made in this patient. Itching is the common factor between these pathologies. Treatment of acquired perforating dermatosis is based on the control of itching. This leads to the clearance of the lesions. Therefore, chemotherapy is indicated to treat Hodgkin lymphoma and help control pruritus. Acquired perforating dermatosis is associated with several disorders, especially diabetes mellitus and renal disease. The association of Hodgkin lymphoma and acquired perforating dermatosis is rarely reported but should be suspected. Pathogenesis of acquired perforating dermatosis is not completely elucidated. It is considered as a reaction to traumatic stimulations (scratching) leading to transepidermal elimination of the altered collagen fibers. Go to the next page if you knew the correct answer, or click the link image(s) below to further research the concepts in this question (if desired).
Research Concepts: Acquired Perforating Dermatosis
We update eBooks quarterly and Apps daily based on user feedback. Please tap flag to report any questions that need improvement.
Question 53:
A patient presents with a painful large right toe. On examination, there is a blue discoloration under the nail bed. Local pressure over the discolored area by a pinhead produces excruciating pain. Which of the following tests has 100% specificity in diagnosing the condition observed in this patient?
Choices: 1. 2. 3. 4.
Hildreth's test Cold sensitivity test Transillumination test Love's pin test
Answer: 1 - Hildreth's test Explanations: This patient has a positive Love's pin test, which highly suggests a glomus tumor under his toenail. A glomus tumor is a rare solitary and small neoplasm arising from the glomus body and usually found under the nail, on the fingertip, or in the foot. They account for 1-2% of all soft tissue tumors. The majority are benign, but they can also show malignant features. Hildreth's test is positive if there is a reduction in pain and tenderness following the application of a tourniquet owing to the diminished blood supply to the tumor. It has 100% specificity in diagnosing glomus tumors. Love's pin test has 78% specificity, and the transillumination test is 90% specific. Go to the next page if you knew the correct answer, or click the link image(s) below to further research the concepts in this question (if desired).
Research Concepts: Glomus Cancer
We update eBooks quarterly and Apps daily based on user feedback. Please tap flag to report any questions that need improvement.
Question 54:
A 34-year old patient presents to the clinic with complaints of cutaneous flushing, diarrhea and dry skin. He claims that his skin has been quite itchy lately and he seems to be more forgetful than usual. He did go to his general practitioner a few weeks ago for vague abdominal pain but was told that he had developed irritable bowel syndrome. He was started on vitamin B3 but his symptoms have persisted. All the other routine blood work appears normal. What blood test can you order that can help with diagnosis and prognosis of this patient?
Choices: 1. 2. 3. 4.
Erythrocyte sedimental levels Natriuretic peptide levels Plasma chromogranin A Alpha-1 anti trypsin
Answer: 3 - Plasma chromogranin A Explanations: Recently the use of chromogranin A has been used for both diagnosis and prognosis in patients with gastrointestinal carcinoids. Highly functioning carcinoids tend to have high levels of chromogranin A in blood. Levels of chromogranin A are associated with patient age, sex, and the location of the tumor. Cutaneous flushing and diarrhea tend to be common features in patients with intestinal carcinoid. The flushing can be triggered by alcohol, food or exercise. Asthma may also occur as a result of the elevated levels of bradykinin, serotonin or substance P. Go to the next page if you knew the correct answer, or click the link image(s) below to further research the concepts in this question (if desired).
Research Concepts: Intestinal Carcinoid Cancer
We update eBooks quarterly and Apps daily based on user feedback. Please tap flag to report any questions that need improvement.
Question 55:
A 17-year-old female patient presented with a four-month history of nasal obstruction along with nasal discharge and epistaxis from the left side. On examination of the patient, thick blood-stained foul-smelling discharge was seen coming from the left nasal cavity. On cleaning the discharge, a pinkish-grey mass was seen filling the left nasal cavity. There were proptosis and lateral displacement of left eyeball. Cervical lymphadenopathy was seen. CT scan showed there was a mass involving left nasal cavity, left maxillary sinus, and both side ethmoid sinuses. There was an intraorbital extension of the mass with the erosion of lamina papyracea and medial part of the floor of orbit on the left side. Under general anesthesia, a biopsy was taken from the nasal mass. Histological examination of the biopsy specimen showed sharply demarcated nests, lobules, or sheets of cells, often separated by richly vascular or hyalinized fibrous stroma. Pseudorosettes (Homer-Wright rosettes), with neoplastic cells palisading or cuffed around the central delicate fibrillar neural matrix were seen. The tumor cells were uniform with sparse cytoplasm and round or ovoid nuclei with punctate salt-and-pepper chromatin. Immunohistochemically, the tumor cells were immunoreactive with neuron-specific enolase, synaptophysin, chromogranin A, and CD56. What is the most likely diagnosis?
Choices: 1. 2. 3. 4.
Sinonasal undifferentiated carcinoma Sinonasal neuroendocrine carcinoma Sinonasal malignant melanoma Esthesioneuroblastoma
Answer: 4 - Esthesioneuroblastoma Explanations: Clinically, esthesioneuroblastoma often has a subtle presentation mimicking benign inflammatory or infectious disease, and delay in diagnosis is not uncommon. Nasal obstruction and epistaxis are typical early manifestations; however, other, more specific symptoms can occur depending on the location and extent of the tumor. Anosmia can precede the diagnosis of esthesioneuroblastoma by several years. Other symptoms are related to extension into paranasal sinuses (facial swelling, medial canthus mass), the orbit (orbital pain, proptosis, epiphora, visual field defects), or anterior cranial fossa (headache), and manifestations of inappropriate antidiuretic hormone secretion. Esthesioneuroblastoma is a locally aggressive neoplasm and metastasizes by both hematogenous and lymphatic routes. Metastasis develops in 35% of cases. The cervical lymph nodes are the most common site of spread. Low-grade esthesioneuroblastoma form submucosal, sharply demarcated nests, lobules, or sheets of cells, often separated by richly vascular or hyalinized fibrous stroma. Pseudorosettes (Homer-Wright rosettes), with neoplastic cells palisading or cuffed around the central delicate fibrillar neural matrix, may be seen. The tumor cells are often uniform, with sparse cytoplasm and round or ovoid nuclei with punctate salt-and-pepper chromatin and nucleoli that are either small or absent.
Go to the next page if you knew the correct answer, or click the link image(s) below to further research the concepts in this question (if desired).
Research Concepts: Esthesioneuroblastoma
We update eBooks quarterly and Apps daily based on user feedback. Please tap flag to report any questions that need improvement.
Question 56:
A 49-year-old male with fatigue and easy bruising is found to have terminal deoxynucleotidyl transferase (TdT) positive cells in the blood. He may have which of the following conditions?
Choices: 1. 2. 3. 4.
Acute myelogenous leukemia Hodgkin lymphoma Hairy cell leukemia Acute lymphoblastic leukemia
Answer: 4 - Acute lymphoblastic leukemia Explanations: Terminal deoxynucleotidyl transferase (TdT) is an important marker for diagnosing ALL. Immunochemistry is usually negative for myeloperoxidase but confirmation of the diagnosis is made by demonstration of lymphoid antigens CD3 or CD 19. It is important to know that Tdt is not a specific test for acute lymphoblastic leukemia as it is absent from L3 subtype ALL. More than 95% of cases of ALL test positive for TdT, especially L1 and L2 but TdT is absent in the mature L3 subtype of ALL. Assessment of TdT helps to differentiate acute lymphoblastic leukemia from nonhodgkin lymphoma. Go to the next page if you knew the correct answer, or click the link image(s) below to further research the concepts in this question (if desired).
Research Concepts: Acute Lymphocytic Leukemia
We update eBooks quarterly and Apps daily based on user feedback. Please tap flag to report any questions that need improvement.
Question 57:
A 47-year-old female presents to a health care provider after discovering a lump in her right breast. She has not had any other symptoms. Physical examination reveals a 2.5 cm, firm, fixed, non-tender mass in the upper, outer quadrant of the right breast. Diagnostic mammography shows the presence of suspicious calcifications, and biopsy demonstrates invasive ductal carcinoma. Molecular testing reveals that the tumor is positive for HER2 and negative for estrogen receptor and progesterone receptor. The patient undergoes partial mastectomy and sentinel lymph node biopsy, which indicates the presence of regional nodal metastasis. A postoperative positron emission tomography (PET) scan shows no evidence of distant metastases. In addition to radiation therapy, treatment with which of the following is most appropriate?
Choices: 1. 2. 3. 4.
Anastrozole and doxorubicin Cyclophosphamide alone Tamoxifen alone Trastuzumab and paclitaxel
Answer: 4 - Trastuzumab and paclitaxel Explanations: Trastuzumab is indicated for the neoadjuvant treatment of HER2-positive breast cancer in combination with a taxane like paclitaxel or anthracycline-based chemotherapy. The primary mechanism of action of trastuzumab is the blockade of HER2, a type of epidermal growth factor receptor. Patients receiving trastuzumab should be monitored for the development of cardiotoxicity. Trastuzumab has a similar mechanism of action, though not identical, to pertuzumab, a newer antineoplastic biologic agent. Go to the next page if you knew the correct answer, or click the link image(s) below to further research the concepts in this question (if desired).
Research Concepts: Trastuzumab
We update eBooks quarterly and Apps daily based on user feedback. Please tap flag to report any questions that need improvement.
Question 58:
A 74-year-old man presents to the clinic for a new consultation regarding palliative care services. He was sent by his internist who believes the patient has lung cancer given a recent episode of hemoptysis associated with a distant history of extensive tobacco usage. The internist notes in his letter to the palliative team that in his prior experience, all patients with this presentation were later confirmed to have lung cancer on imaging and that this patient would, therefore, benefit from palliative care assessment. Today, the patient's vital signs are all within the normal range, and he has not undergone any radiologic imaging since suffering an ankle sprain one decade ago. The patient's internist recently attended a palliative care seminar where speakers emphasized the main recognized way in which primary care providers can best improve care outcomes from palliative specialist consultations. Which of the following best captures a potential risk, as demonstrated by the internist, with adopting this emphasized approach?
Choices: 1. 2. 3. 4.
Higher than expected rate of false positives Lower than expected rate of false negatives Decreased life expectancy of their patients Decreased government funding for orphan diseases
Answer: 1 - Higher than expected rate of false positives Explanations: Palliative care is a form of specialized medical care which aims to optimize the quality of life and alleviate the suffering of patients. The main way a patient's primary care physician can improve outcomes and symptom management by the specialized palliative team is by early identification of candidates and timely subsequent referral for care. Given the main way a primary care provider can improve palliative care outcomes is by early identification, there is a definite risk of higher than expected false-positive diagnoses if the referring physician is focused on making the referral too quickly without confirmatory testing (such as in this question, where the internist makes the referral without a confirmed diagnosis). A false-positive is when a result incorrectly indicates the presence of a particular condition. By rushing into a diagnosis without clear evidence to support a condition, the internist is likely to have a higher than expected rate of false-positives in his patients by diagnosing all patients with hemoptysis and smoking history as having lung cancer. While palliative care outcomes can be improved by early identification of patients with conditions which would benefit from palliative services, there is a risk of wasted time and resources if the palliative team must filter off falsepositives. Go to the next page if you knew the correct answer, or click the link image(s) below to further research the concepts in
this question (if desired).
Research Concepts: Palliative Care
We update eBooks quarterly and Apps daily based on user feedback. Please tap flag to report any questions that need improvement.
Question 59:
A 75-year-old man with a history of unresectable prostate cancer presents with diarrhea, fecal urgency, and tenesmus. He denies fever, nausea, or vomiting. He underwent brachytherapy one year ago. He is not sexually active and does not have a history of sexually transmitted infections. The physical exam is remarkable for diffuse lower abdominal tenderness to palpation and mild perianal inflammation. Which of the following is the best initial therapy for this patient?
Choices: 1. 2. 3. 4.
Topical short-chain fatty acid (SCFA) enemas Corticosteroid enemas Sucralfate enemas Topical 5-aminosalicylic acid (5-ASA) enemas
Answer: 3 - Sucralfate enemas Explanations: This patient's history of brachytherapy for prostate cancer more than nine months ago, with the current complaints of diarrhea, fecal urgency, and tenesmus, points to possible chronic radiation proctitis. Topical sucralfate enemas are the best available treatment for chronic radiation proctitis. For patients with intractable symptoms despite intensive therapy or complications including strictures, fistulas, and persistent bleeding, colostomy, ileostomy, proctectomy, or proctocolectomy with ileal pouch–anal anastomosis (IPAA) may be considered. Topical short-chain fatty acid (SCFA) enemas, topical 5aminosalicylic acid (5-ASA), and topical corticosteroids are used for diversion proctitis. Go to the next page if you knew the correct answer, or click the link image(s) below to further research the concepts in this question (if desired).
Research Concepts: Proctitis And Anusitis
We update eBooks quarterly and Apps daily based on user feedback. Please tap flag to report any questions that need improvement.
Question 60:
A 67-year-old woman presents with a 3cm malignant mass in her left breast. The surgeon recommends lumpectomy, axillary node dissection, and postoperative radiation treatment. What is true of axillary node dissection in patients with breast cancer undergoing a lumpectomy and radiation?
Choices: 1. The procedure improves outcomes. 2. The procedure improves survival. 3. The procedure has not been shown to improve outcomes or survival. 4. Axillary node dissection is superior to sentinel lymph node biopsy.
Answer: 3 - The procedure has not been shown to improve outcomes or survival.
Explanations: Studies have shown that in women undergoing breast conservation therapy with lumpectomy and postoperative radiation, axillary lymph node dissection did not significantly affect overall or disease-free survival. The morbidity from axillary lymph node dissection has led researchers to study alternative axillary treatment strategies, such as observation, radiation, and systemic chemotherapy. These methods have demonstrated low axillary failure rates and no significant difference in overall survival. Sentinel lymph node biopsy is associated with less morbidity than axillary lymph node dissection. This procedure can offer excellent regional control in select patients with early-stage breast cancer. Go to the next page if you knew the correct answer, or click the link image(s) below to further research the concepts in this question (if desired).
Research Concepts: Axillary Lymphadenectomy
We update eBooks quarterly and Apps daily based on user feedback. Please tap flag to report any questions that
need improvement.
Question 61:
A 28-year-old female presents with a pleural effusion and undergoes ultrasound-guided thoracentesis. The fluid collected is sent for cytopathologic examination, and a population of atypical cells is identified. Which of the following immunohistochemical markers, if positive, likely indicate the cells are of mesothelial origin?
Choices: 1. 2. 3. 4.
MOC-31 Ber-EP4 Calretinin CDX2
Answer: 3 - Calretinin Explanations: Distinguishing reactive mesothelial cells from malignant/neoplastic populations of cells in pleural effusion analysis can be difficult, and utilizing immunohistochemical (IHC) markers can aid in the diagnosis and differentiation of mesothelial proliferation. Calretinin, D2-40, and cytokeratin 5/6 are among the most important IHC markers to examine when determining if mesothelial cells are present in pleural effusion samples. CDX2 is a marker for gastrointestinal adenocarcinoma and is a useful IHC to differentiate between malignant mesothelioma and metastatic gastrointestinal adenocarcinoma. Ber-EP4, MOC-31, and CEA are markers which if positive likely indicate carcinoma cells in pleural effusion. Go to the next page if you knew the correct answer, or click the link image(s) below to further research the concepts in this question (if desired).
Research Concepts: Benign Mesothelioma
We update eBooks quarterly and Apps daily based on user feedback. Please tap flag to report any questions that need improvement.
Question 62:
A 66-year-old post-menopausal female presents to the clinic with pelvic pain and abnormal vaginal bleeding. An ultrasound demonstrates a 9 cm complex right adnexal mass and heterogenous uterus with a 12 mm endometrial stripe. CA-125 is 42 U/mL, inhibin A and B levels are elevated as well, and endometrial biopsy demonstrates a malignancy. What is the most likely cause of the patient's abnormal vaginal bleeding?
Choices: 1. 2. 3. 4.
Adenomyosis Uterine leiomyoma Endometritis Hormone-secreting ovarian neoplasm
Answer: 4 - Hormone-secreting ovarian neoplasm Explanations: Both a 9 cm adnexal mass and 12 mm endometrial stripe are abnormal in a 54-year-old female. Elevated inhibin levels in a post-menopausal female are indicative of an adult granulosa cell tumor of the ovary. While adenomyosis and leiomyoma can cause abnormal uterine bleeding, it is much more likely that this patient's bleeding is due to estrogen secretion by a granulosa cell tumor of the ovary. The endometrial biopsy showed a malignancy, which almost certainly represents an endometrial carcinoma from estrogen stimulation. Go to the next page if you knew the correct answer, or click the link image(s) below to further research the concepts in this question (if desired).
Research Concepts: Endometrial Cancer
We update eBooks quarterly and Apps daily based on user feedback. Please tap flag to report any questions that need improvement.
Question 63:
A 68-year-old male with a known diagnosis of duodenal ulcers had EGD completed with biopsies which were significant for positive H. pylori. Further testing of H. pylori strain showed expression of CagA protein. Which of the following is most often associated with this strain of H. pylori?
Choices: 1. 2. 3. 4.
Decreased levels of gastrin in the serum Cytotoxicity Increased risk of gastroesophageal reflux Increased risk of gastric cancer
Answer: 4 - Increased risk of gastric cancer Explanations: The CagA producing strain of H.pylori is associated with increased risk of gastric cancer. This strain is associated with elevated gastrin levels both basal and post meals. CagA is not cytotoxic but is antigenic. This strain is associated with a decreased risk of GERD. Go to the next page if you knew the correct answer, or click the link image(s) below to further research the concepts in this question (if desired).
Research Concepts: Duodenal Ulcer
We update eBooks quarterly and Apps daily based on user feedback. Please tap flag to report any questions that need improvement.
Question 64:
A 63-year-old male with a past medical history of surgically treated colorectal cancer was referred to a specialized hospital for elective treatment with cytoreductive surgery in combination with hyperthermic intraperitoneal chemotherapy (HIPEC) for his extensive peritoneal metastasis. His liver and the rest of the gastrointestinal tract have no metastatic lesions on imaging. He had a right hemicolectomy for adenocarcinoma of the colon three years ago. He later developed peritoneal metastasis. His past medical history also includes coronary artery disease, diabetes mellitus, and hyperlipidemia. His preoperative evaluation and workup are otherwise unremarkable. Which of the following HIPEC delivery-related parameters are necessary for effective treatment?
Choices: 1. 2. 3. 4.
Pressure of inflow Low dosage of the chemotherapeutic agent Duration and volume of perfusion Hemoglobin A1c level less than 6.0%
Answer: 3 - Duration and volume of perfusion Explanations: Cytoreductive surgery, in combination with HIPEC, is the treatment of choice in patients with colorectal cancer and peritoneal metastasis. HIPEC produces direct cytotoxicity to the micro-metastasis. Certain parameters related to the drug delivery system are found to affect its impact. The pressure with which the HIPEC is delivered has not been shown to impact the effectiveness of treatment in the context of the stated case. Familiarity with these factors and their standardization allows for better therapeutic delivery of HIPEC for best-anticipated outcomes. Many parameters of HIPEC administration influence the effectiveness of therapy. Treatment duration is one of them. Many parameters of HIPEC administration influence the effectiveness of therapy. These include the type of chemotherapeutic drug used, its concentration, carrier solution, the volume of the perfusate, the temperature of the perfusate, treatment duration, delivery technique, and patient selection. The volume and temperature of the perfusate solution influence the effectiveness of HIPEC delivery. Go to the next page if you knew the correct answer, or click the link image(s) below to further research the concepts in this question (if desired).
Research Concepts: Peritoneal Metastasis
We update eBooks quarterly and Apps daily based on user feedback. Please tap flag to report any questions that need improvement.
Question 65:
A 66-year-old female is being treated for metastatic non-small-cell lung cancer with progression despite initiating a chemotherapy regimen that was started on erlotinib ten weeks ago. She has tolerated the medication well, except for the development of erythematous follicle-based papules and pustules on the central face, which have improved slightly. She now complains of pain in bilateral feet, specifically bilateral great toes. Based on the patient's history, what clinical finding is most likely to be discovered on the exam?
Choices: 1. 2. 3. 4.
Erythematous, friable plaque surrounding the nail Purulent discharge from the proximal nail fold Necrosis and gangrene of distal digits Verrucous papules involving lateral nail fold
Answer: 1 - Erythematous, friable plaque surrounding the nail
Explanations: Periungual pyogenic granulomas occurring during epidermal growth factor receptor inhibitor therapy are a mounting problem for oncology patients, arising after two months of drug exposure. Drugs that inhibit the epidermal growth factor receptor (EGFR) include two therapeutic monoclonal antibodies (cetuximab and panitumumab) and five orally active small molecule EGFR inhibitors (gefitinib, erlotinib, lapatinib, afatinib, and osimertinib). Nailfold paronychia and pyogenic granuloma changes are common, especially involving bilateral great toes. Secondary bacterial infection (often with Staphylococcus aureus) may occur. Clinical judgment and, at times, a culture may be necessary. Treatment options include topical antibiotics, topical corticosteroids, and/or electrodesiccation for larger lesions. Go to the next page if you knew the correct answer, or click the link image(s) below to further research the concepts in this question (if desired).
Research Concepts: Pyogenic Granuloma
We update eBooks quarterly and Apps daily based on user feedback. Please tap flag to report any questions that need improvement.
Question 66:
A 65-year-old man with squamous cell carcinoma of the penis invading the left corpus cavernosum undergoes partial penectomy. After six weeks of cephalexin, a 3.5 cm right inguinal lymph node has decreased in size to 2.0 cm. Pelvic CT scan is normal. What stage is the is this patient?
Choices: 1. 2. 3. 4.
Stage Stage Stage Stage
3A 1 2 4
Answer: 1 - Stage 3A Explanations: The TNM staging system is used for staging carcinoma of the penis. It is currently in its 8th edition and is used to define prognostic stage groups and to develop an appropriate treatment plan. Primary Tumor (T) Tx: Primary tumor cannot be assessed, T0: No evidence of primary tumor, Tis: Carcinoma in situ, Ta: Noninvasive localized squamous cell carcinoma (SCC), T1: Glans: Tumor invades the lamina propria, Foreskin: Tumor invades dermis, lamina propria, or dartos fascia. Shaft: Tumor invades connective tissue between epidermis and corpora, regardless of location, T1a: Tumor is without lymphovascular invasion or perineural invasion and is not high grade, T1b: Tumor exhibits lymphovascular invasion and/or perineural invasion OR is high grade, T2: Tumor invades into corpus spongiosum (either glans or ventral shaft) with or without urethral invasion, T3: Tumor invades into corpa cavernosum (including tunica albuginea) or with or without urethral invasion, T4: Tumor invades into adjacent structures (scrotum, prostate, pubic bone). Regional lymph nodes (N), Clinical N (cN) cNX: Regional lymph nodes cannot be assessed, cN0: No palpable or visibly enlarged inguinal lymph nodes, cN1: Palpable mobile unilateral inguinal lymph node, cN2: Palpable mobile greater than or equal to 2 unilateral inguinal nodes or bilateral inguinal lymph nodes, cN3: Palpable fixed inguinal nodal mass or pelvic lymphadenopathy unilateral or bilateral. Pathological N (pN) pNX: Lymph node metastasis cannot be established, pN0: No lymph node metastasis, pN1: less than or equal to 2 unilateral
inguinal metastasis, no extranodal extension, pN2: Greater than or equal to 3 unilateral inguinal metastases or bilateral metastases, pN3: Extranodal extension of lymph node metastases or pelvic lymph node metastases. Distant Metastasis (M) M0: No distant metastasis, M1L Distant metastasis. Anatomic Stage/Prognostic Groups, Stage 0: TiS, N0, M0; Ta, N0, M0; Stage I: T1a, N0, M0; Stage II: T1b, N0, M0; T2, N0, M0; T3, N0, M0; Stage IIIA: T1-3, N1, M0; Stage IIIB: T1-3, N2, M0; Stage IV: T4, any N, M0; Any T, N3, M0; Any T, Any N, M1. This patient is Stage II with invasion into the corpora. Go to the next page if you knew the correct answer, or click the link image(s) below to further research the concepts in this question (if desired).
Research Concepts: Penile Cancer
We update eBooks quarterly and Apps daily based on user feedback. Please tap flag to report any questions that need improvement.
Question 67:
An 86-year-old male with history of lymphoma, not under any treatment, presents to the hospital for the fifth time this month for a small bowel obstruction secondary to a large mesenteric mass. The patient has nausea and vomiting, is unable to tolerate oral intake, and has moderate abdominal distention causing him pain. How would a PEG tube placement aid his obstruction?
Choices: 1. Primarily increases nutritional support due to chronic illness 2. Enteral feeds stimulate his bowel 3. Decompresses his stomach, relieving some of his symptoms of nausea, vomiting, distention, and pain 4. PEG tube is not indicated in this patient
Answer: 3 - Decompresses his stomach, relieving some of his symptoms of nausea, vomiting, distention, and pain
Explanations: A PEG tube provides enteral feeds in a patient with malnutrition, however, enteral feeds do not aid in relieving his distended stomach and symptoms secondary to his bowel obstruction. Enteral feeds do not provide any bowel stimulation and do not aid in relieving the patient's bowel obstruction. PEG tubes can also be used to decompress the stomach due to direct percutaneous access to the stomach. PEG tube can be used to decompress the stomach in a patient with a gastric obstruction causing a distended stomach. Go to the next page if you knew the correct answer, or click the link image(s) below to further research the concepts in this question (if desired).
Research Concepts: Percutaneous Endoscopic Gastrostomy Tube
We update eBooks quarterly and Apps daily based on user feedback. Please tap flag to report any questions that need improvement.
Question 68:
A 23-year-old pregnant G1P0A0 female presents to her provider because she thinks her stomach is growing too rapidly. Transvaginal ultrasound reveals a snowstorm appearance and patient undergoes suction curettage to remove the mass. The patient is scheduled to follow up with her primary care provider in one week to begin tracking her recovery to ensure all the mass was removed and monitor possible cancer progression. The patient denied taking oral contraceptive pills because she would like to become pregnant. The patient is non-compliant with follow up visits and does not return to the doctor until two months later when she is pregnant again. She complains of also experiencing symptoms of increased heat intolerance, agitation, fever, diarrhea, and tachycardia. What is elevated in this patient and what is causing her to present with these symptoms and which of the following is not a true statement about hormones?
Choices: 1. Thyroid stimulating hormone – the patient has undiagnosed thyroid disease 2. Human chorionic gonadotropin – the patient has choriocarcinoma 3. Tri-iodothyronine – increased peripheral conversion 4. Thyroxine – increased thyroid production secondary to pregnancy
Answer: 2 - Human chorionic gonadotropin – the patient has choriocarcinoma
Explanations: This patient initially presented with a complete hydatidiform mole. When patients are treated for a complete molar pregnancy, they need to have human chorionic gonadotropin (hCG) measured every week to ensure the entire mole was removed and so to monitor choriocarcinoma risk. When the patient does follow she is experiencing symptoms of hyperthyroidism. This is a result of elevated hCG as it shares the same alpha unit as thyroid stimulating hormone (TSH). When hCG is elevated, it can bind to TSH receptors and stimulate it due to the similarity of the two hormones, leading to presenting symptoms. Human chorionic gonadotropin (hCG), is one of four hormones (hCG, TSH, luteinizing hormone, and follicle stimulating hormone) that all share same the same alpha unit. When levels of one of these four hormones are chronically elevated it can lead to stimulation of other receptors due to the shared alpha unit leading to other hormonal and physiologic abnormalities. TSH is synthesized and released from cells within the anterior pituitary gland, known as thyrotrophs. This hormone is composed of two subunits, 1 alpha, and 1 beta. TSH is one of the four hormones which share the same alpha unit, the beta unit is what makes TSH unique and determines its specificity within the human body. Due to the physiologic effects of thyroxine (T4) and triiodothyronine (T3), these two hormones will help tightly control the levels of TSH released into the body. Minute increases in serum T3 and T4 will result in TSH
inhibition, conversely, small decreases in serum T3 and T4 result in increased TSH. T3 and T4 levels will also work to increase/decrease TRH through a negative feedback look, another mechanism for modulating TSH levels. TSH levels will slowly change depending on several factors, such as initial TSH level, the hormone given (T3 or T4), and the dose of hormone given. A higher TSH level will take longer to decrease and will gradually decline over several days. TSH levels will respond faster to T3 than T4, additionally, when a higher dose is given, TSH will respond more rapidly. If high doses of T3 are administrated, TSH levels will begin to decline over the course of several hours in hypothyroid patients. Three additional substances have been shown to also affect TSH levels, somatostatin, dopamine, and glucocorticoids. When a somatostatin analog is given to a patient, TSH levels will decrease, however, not to the point of hypothyroidism. Animal studies have been performed showing there is a correlation between somatostatin and TSH levels, but it is not fully understood and not extensively studied in humans. Dopamine can cause a rapid decrease in TSH levels, and accordingly dopamine antagonists can acutely raise TSH levels. Patients in the intensive care unit (ICU) often have altered TSH levels when receiving dopamine or dopamine antagonists. TSH is an extremely important hormone for the thyroid. It stimulates each step in hormone synthesis within the thyroid, affects the expression of multiple genes and can cause thyroid hyperplasia or hypertrophy. Action begins when TSH binds to a plasma membrane receptor, activating adenylyl cyclase which increases cyclic adenosine monophosphate (cAMP), resulting in activation of several protein kinases. Via the same receptor, TSH stimulates phospholipase C, increasing phosphoinositide
turnover, protein kinase C activity, and intracellular calcium concentration. How the above steps are specifically linked to T3, and T4 synthesis, release and other thyroid metabolic processes are not fully understood. Go to the next page if you knew the correct answer, or click the link image(s) below to further research the concepts in this question (if desired).
Research Concepts: Choriocarcinoma
We update eBooks quarterly and Apps daily based on user feedback. Please tap flag to report any questions that need improvement.
Question 69:
A 66-year-old male presents with severe diarrhea, abdominal pain, hematochezia, fever, and fatigue. He has been on treatment for diabetes for twenty years. Investigations, including coloscopy and immunohistochemistry, identify cytomegalovirus (CMV) in tissue biopsies. Other tests excluded bacterial, parasitic, ischemic, microscopic colitis. No evidence of immunodeficiency is found. Which of the following is the next best step in the management of this patient?
Choices: 1. 2. 3. 4.
Initiate therapy with ganciclovir Exclude ulcerative colitis Initiate therapy with corticosteroids Order CMV stool culture
Answer: 1 - Initiate therapy with ganciclovir Explanations: Patients who are immunocompetent may not need treatment with antiviral medications. However, antiviral treatment in immunocompetent patients with CMV colitis could be used in males aged 55 and above who suffer from severe disease, and have co-morbidities such as diabetes mellitus or chronic renal failure. The side effects of ganciclovir include myelosuppression, hepatotoxicity, nephrotoxicity, and central nervous system disorders. The patient gives no history suggestive of recurrent attaches of diarrhea, blood in stools, arthralgia, fever, and abdominal pain. A colonoscopy does not show evidence of ulcerative colitis. Corticosteroids are not indicated in patients with CMV colitis. A diagnosis should be confirmed first and if CMV colitis is confirmed, the next step should be to decide on treatment strategies of an immunocompetent patient with CMV colitis. Such a decision should be individualized and tailored to the patient's risks e.g., age, sex, co-morbidities such as diabetes mellitus and chronic renal failure. The results from the immunohistochemistry of biopsy tissues are diagnostic of CMV colitis. There is no need for other tests such as CMV stool culture. Go to the next page if you knew the correct answer, or click the link image(s) below to further research the concepts in this question (if desired).
Research Concepts:
Cytomegalovirus Colitis
We update eBooks quarterly and Apps daily based on user feedback. Please tap flag to report any questions that need improvement.
Question 70:
A 30-year-old white female who is diagnosed with acute promyelocytic leukemia was started on all-trans retinoic acid (ATRA). On the third day of the therapy, the patient developed fever, peripheral edema, and shortness of breath. Her vital signs include a temperature of 102.3 F, blood pressure 90/58 mmHg, pulse 112 beats/min, pulse oximetry 84% on 10 L of high flow oxygen, and respiratory rate 24 breaths/min. Pertinent laboratory studies include WBC 31,000 cells/mL, hemoglobin 7 grams/dL, platelet count 50,000/mL, serum creatinine 2.4 mg/dL (baseline 0.9 mg/dL). A chest x-ray suggests pulmonary edema. She was transferred to the intensive care unit for further management. Which of the following is correct regarding the management of this condition?
Choices: 1. Add arsenic trioxide to ATRA 2. Supportive therapy and continue ATRA 3. Continue ATRA and start hydroxyurea 4. Discontinue ATRA and start the patient on IV dexamethasone 10 mg twice daily
Answer: 4 - Discontinue ATRA and start the patient on IV dexamethasone 10 mg twice daily
Explanations: Differentiation syndrome is a cytokine release syndrome seen after initiation of differentiating agents characterized by fever, peripheral edema, pulmonary edema, and multi-organ dysfunction. It can be fatal, if not promptly recognized and treated. High dose systemic corticosteroids are helpful in these cases. All-trans retinoic acid (ATRA) can be continued in mild cases in addition to IV corticosteroids. ATRA must be discontinued in severe cases with multiorgan dysfunction and can be restarted when the differentiation syndrome resolves. Go to the next page if you knew the correct answer, or click the link image(s) below to further research the concepts in this question (if desired).
Research Concepts: Acute Promyelocytic Leukemia
We update eBooks quarterly and Apps daily based on user feedback. Please tap flag to report any questions that need improvement.
Question 71:
A 49-year-old male patient presented with a swelling on the left lower anterior chest wall for the past three years. Initially, the swelling was small in size and then gradually progressed to reach the present size of around 6 cm × 8 cm. Computed tomography scan showed a well-defined lobulated round heterogeneous soft tissue density with hypodense areas. Necrosis and peripheral calcifications were noted in the anterior aspect of the 8th rib. The whole tumor was excised en masse along with the 7th, 8th, and 9th ribs and a large portion of a diaphragm. Histological examination revealed irregularly shaped lobules of cartilage varying in size and shape. These lobules were separated by fibrous bands or permeate bony trabeculae. The chondrocytes were atypical, varying in size and shape with enlarged hyperchromatic nuclei. Binucleation was seen. There were also some myxoid changes. What is the most likely diagnosis in this patient?
Choices: 1. 2. 3. 4.
Chondroblastoma Enchondroma Chondrosarcoma Osteochondroma
Answer: 3 - Chondrosarcoma Explanations: Histologically, chondrosarcomas show abundant bluegrey cartilage matrix-production. Irregularly shaped lobules of cartilage varying in size and shape are present. The chondrocytes are atypical, with variable size and shape and contain enlarged hyperchromatic nuclei. Binucleation is frequently seen. Chondroid matrix liquefaction or myxoid changes are a common feature of chondrosarcomas. Necrosis and mitoses can be seen. Local swelling and/or pain are the most common presenting symptoms of chondrosarcomas. The symptoms are usually of long duration, months to years. The radiographic features of chondrosarcoma are often very characteristic, and a definitive diagnosis can often be made on the basis of imaging examination alone. Computed tomography scan demonstrates a welldefined lobulated round heterogeneous soft tissue density with hypodense areas. In typical forms, MRI shows a lobulated lesion with a high-signal intensity on T2 and a low or intermediate signal on T1-weighted images. Chondrosarcomas are large tumors, usually greater than 4 cm in size. The most common locations of involvement of chondrosarcoma include the bones of the axial skeleton (pelvis, scapula, sternum, and ribs). The primary treatment modality of chondrosarcoma is extensive resection with a sufficient margin. Go to the next page if you knew the correct answer, or click the link image(s) below to further research the concepts in this question (if desired).
Research Concepts: Chondrosarcoma
We update eBooks quarterly and Apps daily based on user feedback. Please tap flag to report any questions that need improvement.
Question 72:
A 45-year-old male known to have chronic myelogenous leukemia (CLL) and HIV developed altered mental status, visual hallucinations, and agitation one week after admission for neutropenic fever and herpes zoster. He received treatment for CLL five years ago, went into remission and then relapsed one year ago. The exam revealed a temperature of 103 F, splenomegaly, and an extensive vesicular dermatomal infiltrate across the midline of his back to the midline of his right abdomen. Similar lesions were noted over the trunk, legs, face, and scalp. IV vancomycin and cefepime were started for the neutropenic fever, IV acyclovir for herpes zoster and due to concern for possible alcoholic hallucinosis, he was placed on withdrawal protocol. The symptoms, however, did not improve and he became more agitated, confused, and he started to have blurred vision and even experienced one episode of seizures. What is the most likely diagnosis?
Choices: 1. 2. 3. 4.
Alcohol withdrawal Central nervous system leukemic infiltrates Brain abscess Meningioma
Answer: 2 - Central nervous system leukemic infiltrates Explanations: Central nervous system (CNS) leukemic infiltration may present in any of the following intracranial forms: meningeal disease as “carcinomatous meningitis,” localized tumor masses, as “myeloid sarcomas,” intravascular tumor aggregates throughout the brain parenchyma as “carcinomatous encephalitis.” The most common effects of CNS leukemic spread: increased intracranial pressure (vomiting, headache, papilledema, lethargy, seizure, coma), cranial nerve palsies, visual disturbances (blurred vision, diplopia, blindness, photophobia), and other (vertigo, ataxia, hallucinations, nystagmus, myelopathy). Especially in a patient with a history hematological disease, previous bone marrow or solid organ transplantation status and compromised immune status, high suspicion must be held for the presence of intracranial sarcoma. Further investigations in the form of CT and/or MRI must be done to further distinguish the intracranial lesions of myeloid sarcoma from other commonly-found intracranial lesions, such as meningioma, lymphoma or brain metastasis. The skin lesions in this patient are likely leukemia cutis. Leukemia cutis has a predilection for sites of herpes zoster, trauma, and surgical sites. The development of leukemia cutis has a poor prognosis and 90% of individuals are likely to have extramedullary spread. In 40% of these individuals, the meninges are likely to be involved.
Go to the next page if you knew the correct answer, or click the link image(s) below to further research the concepts in this question (if desired).
Research Concepts: Leukemia Cutis
We update eBooks quarterly and Apps daily based on user feedback. Please tap flag to report any questions that need improvement.
Question 73:
A 65-year-old male patient was diagnosed with tubercular spine disease and associated Frankel type D neurologic deficit six weeks ago. He was started on first-line anti-tuberculosis treatment (ATT). His image-guided biopsy revealed the growth of tubercle bacilli and good sensitivity to first-line ATT. His ESR and CRP have demonstrated an overall downward trend, and his back pain, as well as neurological status, have considerably improved. However, his MRI has shown a significant increase in the size of his abscess since the inception of chemotherapy. What is the appropriate next line of management?
Choices: 1. Repeat MRI in 6 weeks to assess the need for surgical intervention 2. Continue antitubercular medications; repeat ESR, CRP, liver function tests (LFTs), and MRI at 6 weeks 3. Consider the bacilli as drug-resistant and start secondline ATT 4. Abandon the non-operative line of management and immediately book the patient for surgery
Answer: 2 - Continue antitubercular medications; repeat ESR, CRP, liver function tests (LFTs), and MRI at 6 weeks
Explanations: Plain radiographs have an overall 15% sensitivity. In the early stages, with less than 30% vertebral destruction, they do not play much of a role. In later stages, with more than 30% vertebral destruction, patients can present with disc space reduction, endplate rarefaction, vertebral body destruction, instability, and spinal deformity. Computed tomography has almost 100% sensitivity. It can help in the diagnosis at a much earlier stage than plain x-rays. The types of vertebral destructive lesions based on CT in spinal TB include fragmentary, osteolytic, subperiosteal, and localized sclerosis. CT scan can also aid in image-guided biopsy for establishing the diagnosis. Magnetic resonance imaging has almost 100% sensitivity and 80% specificity. MRI is the most useful modality in the diagnosis of spinal TB. The extent of soft tissue enhancement, the location of the abscess, and spinal canal compromise is best detected by MRI. Gadolinium-enhanced MRI may provide additional information regarding the diagnosis. Screening sequences involving the whole spine can also help in identifying non-contiguous vertebral involvement. MRI can also help in the assessment of response to treatment; however, immediately after starting antitubercular therapy, the MRI scan can reveal a worsening picture; and often lags behind the clinical improvement. At the time of initial evaluation, patients are classified based on clinical examination and detailed radiological
imaging (plain radiographs, CT, and MRI) into 5 stages: 1 - predestructive stage - hyperemia on scintiscan; 2 early destructive stage - disc space reduction and paradisical erosion, knuckle less than 10 degrees, MRI demonstrates marrow edema, and CT shows erosions or cavitations; 3 - mild angular kyphosis, 2-3 vertebrae involved and kyphosis 10 - 30 degrees; 4 - moderate angular kyphosis, 2-3 vertebrae involved and kyphosis 30 - 60 degrees; 5 - severe angular kyphosis, >3 vertebrae involved and kyphosis greater than 60 degrees. The most appropriate investigations at the time of follow-up in patients on treatment for TB include thorough clinical evaluation, blood inflammatory parameters (ESR, CRP), plain radiographs, and liver function tests to evaluate drug-associated complications. In a patient with evident clinical improvement and no specific concerns regarding these other targets at follow-up, no further change in antitubercular therapy or need for surgery is indicated. However, if a patient does not show any improvement clinically or based on inflammatory markers, or if they continue to worsen, all possibilities, including wrong diagnosis, poor immune status and nutrition, the inadequate dosage of drugs, drug resistance, or need for surgical intervention (possibility of underlying abscess which needs drainage) need to be considered. A further line of management needs to be individualized under such circumstances. Go to the next page if you knew the correct answer, or click the link image(s) below to further research the concepts in this question (if desired).
Research Concepts:
Pott Disease
We update eBooks quarterly and Apps daily based on user feedback. Please tap flag to report any questions that need improvement.
Question 74:
A 30-year-old male presented to with peripheral lymphadenopathy. Lymph node biopsy showed large anaplastic cells staining for CD30 in the background of CD3 and CD20 cells. Genetic testing showed t(1;2). What is the most likely diagnosis?
Choices: 1. 2. 3. 4.
Anaplastic large cell lymphoma (ALK-positive) Anaplastic large cell lymphoma (ALK-negative) T-ALL Peripheral T cell lymphoma, NOS
Answer: 1 - Anaplastic large cell lymphoma (ALKpositive)
Explanations: Anaplastic large cell lymphoma (ALCL), ALK-positive is a T cell lymphoma that shows a characteristic large pleomorphic lymphoid cells with abundant cytoplasm and horseshoe-shaped nuclei. Immunohistochemistry (IHC) is usually necessary in confirming ALCL (ALK positive). Positive stains supporting ALCL (ALK positive) diagnosis include: CD30 (cytoplasmic), EMA, CD2, CD4, CD5, TIA1, granzyme B, perforin, CD45, CD45RO, CD61, CD25 (strong), BNH9. Negative stains include: CD15, CD20, CD79a, cytokeratin, bcl2, PAX5 / BSAP, PGM1, EBV (EBER & LMP1). There are different distinct translocations all of which result in upregulation of ALK protein. The most common translocation is t(2;5)(p23;q35). Other translocations/rearrangements include: t(1;2)(q25;p23), inv(2)(p23;q35), t(2;3)(p23;q21), t(2;17)(p23;q23) t(2;X) (p23;q11-12), t(2;19)(p23;p13.1), t(2;22)(p23;q11.2), t(2;17)(p23;q25). Other secondary genetic alterations include: -4, del11q, del13q, +7, +17p, +17q). Go to the next page if you knew the correct answer, or click the link image(s) below to further research the concepts in this question (if desired).
Research Concepts: Anaplastic Large Cell Lymphoma
We update eBooks quarterly and Apps daily based on user feedback. Please tap flag to report any questions that need improvement.
Question 75:
A 30-year-old female with HIV is receiving highly active antiretroviral therapy (HAART). Her CD4 count is 250 cells/microliter. She has a history of an abnormal pap smear 5 years ago, but colposcopy and follow-up Pap smears were normal. The patient has had no other sexually transmitted infections. Which of the following is true about screening for cervical cancer in this patient?
Choices: 1. 2. 3. 4.
The The The The
patient patient patient patient
should should should should
have have have have
Pap smears every 6 months Pap smears every 12 months a total hysterectomy a partial hysterectomy
Answer: 2 - The patient should have Pap smears every 12 months
Explanations: According to the American College of Obstetricians and Gynecologists (ACOG), HIV-infected women should be screened at the time of diagnosis and 6 months thereafter. If the initial results are normal, patients should be rescreened annually if their CD4 count is more than 200 cells/microliter or every 6 months if their CD4 count is less than 200 cells/microliter. According to ACOG, HIV-infected women should be screened using a regular vaginal cuff Pap smear following hysterectomy if they have a history of highgrade CIN or invasive cervical cancer. This is because of the increased risk of vaginal cancer associated with HIV infection. ACOG also recommends that HIV-infected women be screened annually for anal carcinoma using Pap smears. Go to the next page if you knew the correct answer, or click the link image(s) below to further research the concepts in this question (if desired).
Research Concepts: Cervical Screening
We update eBooks quarterly and Apps daily based on user feedback. Please tap flag to report any questions that need improvement.
Question 76:
An investigator plans to use a commercially available complementary DNA (cDNA) microarray to investigate the expression of known differentiation markers in thyroid cancer. Surgical biopsy specimens of cancer are obtained, messenger RNA (mRNA) is extracted, cDNA is generated, fluorescently labeled, and hybridized to the microarray. Subsequently, the microarray is scanned, intensity values are measured, and statistical analysis is performed. What must the investigator do to make sense of this data?
Choices: 1. 2. 3. 4.
Compare it to healthy control tissue Repeat the experiment Reanalyze the data using other data analysis software Rescan the microarray
Answer: 1 - Compare it to healthy control tissue Explanations: The use of new technology does not nullify the importance of sound experimental design. The investigator is studying differentiation markers in thyroid cancer; a suitable control would entail healthy thyroid tissue. The data produced from a microarray experiment typically constitute a long list of measurements of spot intensities and intensity ratios, generated either by pairwise comparison of 2 samples or a comparison of several samples with common control. Every new treatment or diagnostic test must be compared to a healthy population or conventional standard markers. Go to the next page if you knew the correct answer, or click the link image(s) below to further research the concepts in this question (if desired).
Research Concepts: Thyroid Cancer
We update eBooks quarterly and Apps daily based on user feedback. Please tap flag to report any questions that need improvement.
Question 77:
A 17-year-old female presents with significant polyposis of the colon. Her parents are both healthy, but she has an extended family history of earlyonset colon cancer. Genetic testing is positive for heterozygous adenomatous polyposis coli (APC) gene mutation. She is diagnosed with familial adenomatous polyposis and undergoes a total colectomy. What is the most important screening modality for this patient going forward?
Choices: 1. Continued proctoscopies every 3 years 2. Upper endoscopy every 3 years to look for periampullary tumors 3. CT abdomen every 5 years to look for sporadic tumors 4. MRI of the head every year
Answer: 2 - Upper endoscopy every 3 years to look for periampullary tumors
Explanations: Serial colonoscopies are appropriate as screening in a patient who is at high risk for inherited colorectal cancer before a total colectomy is performed. The recent guidelines recommend screening for polyps in children with classic FAP, starting at age 10 with flexible sigmoidoscopy. Those without evidence of polyposis should have the examination every 2 years. Further data to guide decision-making for children of affected individuals who reach adulthood without polyposis and with a negative genetic diagnosis is lacking. However, a colonoscopy might replace flexible sigmoidoscopy in the later teenage years (16) and should be done every 2 years until 20. If there are still no adenomas, surveillance intervals might be gradually extended. Upper endoscopy every 3 years is recommended for patients with familial adenomatous polyposis (FAP) who have had a colectomy. The most common cause of death in patients with FAP who have had a colectomy performed is duodenal cancer. Although Turcot syndrome is multiple cancerpredisposing syndromes, there currently is no guideline recommending generalized imaging with CT scans as a screening tool for tumors. A yearly MRI of the head is not recommended in a patient with no genetic evidence of Turcot syndrome. If there is genetic confirmation of Turcot syndrome, the US Multi-Society Task Force suggests an MRI of the head every 6 months starting at 2 years of age. The evidence is considered low-quality.
Go to the next page if you knew the correct answer, or click the link image(s) below to further research the concepts in this question (if desired).
Research Concepts: Turcot Syndrome
We update eBooks quarterly and Apps daily based on user feedback. Please tap flag to report any questions that need improvement.
Question 78:
A 52-year-old white male with a recent diagnosis of necrolytic migratory erythema presents asking about treatment options for his disease. He is otherwise healthy and has no other major comorbidities. Abdominal CT and selective celiac angiography have demonstrated a mass in the tail of the pancreas as well as metastases to the surrounding peripancreatic lymph nodes. What treatment plan would give this patient the best outcome?
Choices: 1. 2. 3. 4.
Lanreotide only Chemotherapy with 5-fluorouracil and streptozotocin Surgical excision of the mass only Surgical excision of the mass plus lanreotide
Answer: 4 - Surgical excision of the mass plus lanreotide Explanations: Necrolytic migratory erythema is associated with the glucagonoma, most commonly located in the tail of the pancreas. Surgical excision of the primary tumor confers a survival benefit even in the presence of tumor metastases. Somatostatin analogs (e.g., lanreotide) can abolish the effects of glucagon excess and are used to palliate symptoms of hyperglucagonemia. They have also been shown to increase survival. Use of somatostatin analogs in conjunction with excision of the tumor is typically used in the treatment of glucagonoma. Go to the next page if you knew the correct answer, or click the link image(s) below to further research the concepts in this question (if desired).
Research Concepts: Necrolytic Migratory Erythema
We update eBooks quarterly and Apps daily based on user feedback. Please tap flag to report any questions that need improvement.
Question 79:
A 45-year-old male was diagnosed with insulinoma and underwent surgical resection. At his 6-month post-resection follow-up, he was found to have the recurrence of the tumor on imaging studies. He was started on diazoxide but remained symptomatic with hypoglycemic episodes. Diazoxide was then switched to octreotide by his endocrinologist. He now presents again with recurrent symptoms of diaphoresis, palpitations, and blurry vision. His insulinoma size is also growing on repeat imaging. Which of the following is the next best step in the management of this patient?
Choices: 1. 2. 3. 4.
Adjusting the dose of octreotide Surgical resection Everolimus Radiation
Answer: 3 - Everolimus Explanations: This patient already underwent surgical resection and still had recurrent symptoms while on diazoxide or octreotide. He was also found to have increased tumor volume on repeat imaging. For this group of patients, targeted therapy (everolimus or sunitinib) should be offered given its survival benefits. Everolimus is an inhibitor of the mammalian target of rapamycin. Everolimus was found to improve median progressionfree survival in patients with low or intermediate-grade, advanced, progressive pNET and in a subgroup of functional pNET (including but not limited to insulinomas). It also controls hypoglycemia in patients with malignant insulinomas. Go to the next page if you knew the correct answer, or click the link image(s) below to further research the concepts in this question (if desired).
Research Concepts: Insulinoma
We update eBooks quarterly and Apps daily based on user feedback. Please tap flag to report any questions that need improvement.
Question 80:
A 30-year-old female presents to the emergency room with one day of epistaxis and extensive bruising of the skin and reports two weeks of generalized weakness and fatigue. Initial lab studies reveal pancytopenia, elevated PT and PTT, and the low fibrinogen. The peripheral smear reveals large atypical promyelocytes with Auer rods and other myeloid precursors in various stages of development in the peripheral blood. Acute promyelocytic leukemia (APL) is suspected. Which test should be performed next to confirm APL?
Choices: 1. Conventional karyotyping 2. Fluorescent in situ hybridization (FISH) for the fusion of PML/RARA 3. Reverse transcriptase polymerase chain reaction (RT-PCR) for PML-RARA RNA 4. Computed tomography of the chest
Answer: 2 - Fluorescent in situ hybridization (FISH) for the fusion of PML/RARA
Explanations: When acute promyelocytic leukemia (APL) is suspected, evaluation of peripheral blood smear and FISH for the fusion of PML/RARA should be expedited for rapid diagnosis for this time-sensitive disease with high pretreatment mortality. FISH results can be obtained within 24 hours; however, conventional karyotyping takes about 2 days to get results. Conventional karyotyping should also be performed as a part of the initial workup as it detects rare molecular subtypes of APL and other additional coexistent cytogenetic abnormalities. Reverse transcriptase-polymerase chain reaction (RTPCR) for PML-RARA RNA is also used for confirming the diagnosis of APL and can be also be used can for monitoring minimal residual disease. Go to the next page if you knew the correct answer, or click the link image(s) below to further research the concepts in this question (if desired).
Research Concepts: Acute Promyelocytic Leukemia
We update eBooks quarterly and Apps daily based on user feedback. Please tap flag to report any questions that need improvement.
Question 81:
A 65-year-old male patient presents to the hospital with a 3 cm, mobile, group 2 lymph node. Fineneedle aspiration reveals squamous cell carcinoma. The physical examination and radiographic evaluation reveal no evidence of a primary site of lesion and no distant metastases. Which of the following is most likely the primary site?
Choices: 1. 2. 3. 4.
Nasopharynx Floor of mouth Base of tongue Infrahyoid epiglottis
Answer: 3 - Base of tongue Explanations: The primary site is detected in approximately 40% of patients presenting with a metastatic upper cervical lymph node from an unknown primary site. Specifically, in the head and neck of patients who have undergone a modern evaluation with flexible fiber-optic endoscopy, CT or MRI of the head and neck, and directed biopsies. Approximately 40% of detected primary sites are found in the base of the tongue and 40% in the tonsillar fossa. With tonsillar cancer, metastasis to regional lymphatics is a frequent occurrence. Neck metastases are seen in 50% to 65% of patients. Even in patients with a clinically negative physical exam, 30% to 40% will have neck disease. Tonsillar squamous cell cancer also may spread to the retropharyngeal lymph nodes. Go to the next page if you knew the correct answer, or click the link image(s) below to further research the concepts in this question (if desired).
Research Concepts: Tongue Cancer
We update eBooks quarterly and Apps daily based on user feedback. Please tap flag to report any questions that need improvement.
Question 82:
A 39-year-old man presents with a onemonth history of night sweats and unintentional weight loss of 4 kg. On examination, there is waxing and waning limphadenopathy in the cervical and axillary nodes. Blood tests show hemoglobin 9 g/dL (12-15) and lymphocytes 8 x 10^9/L (1-4) showing atypical lymphocytes, and the majority showing deeply cleft nuclei. Flow cytometry shows a kappa restricted CD20+, CD19+, and CD10+ population. What is the most likely origin of the cells causing this disease?
Choices: 1. 2. 3. 4.
Follicular center B cells Follicular center T cells Plasmablastic cells Follicular dendritic cells
Answer: 1 - Follicular center B cells Explanations: Follicular lymphoma (FL) originates from germinal/follicular center B cells. Most cases of FL show a t(14;18)(q32;q21), which leads to an overexpression of BCL2, an anti-apoptotic protein. FL is one of the most common types of non-Hodgkin lymphoma (NHL). Diagnosis of FL needs morphological assessment, flow cytometry, and sometimes genetic testing to confirm the diagnosis. A peripheral smear in FL may show atypical lymphocytes that have scant cytoplasm and a deeply cleft nucleus. Genetic analysis is a useful tool for assessing FL. Most cases will show clonally rearranged immunoglobulin genes. Furthermore, the majority of the cases will show a t(14;18)(q32;q21); the translocation affects the IgH and BCL2. Go to the next page if you knew the correct answer, or click the link image(s) below to further research the concepts in this question (if desired).
Research Concepts: Follicular Lymphoma
We update eBooks quarterly and Apps daily based on user feedback. Please tap flag to report any questions that
need improvement.
Question 83:
A 73-year-old female presents to the clinic with abdominal pain and weight loss. She had a CT abdomen that showed an intraductal papillary mucinous neoplasm (IPMN). MRCP shows a 3 cm cyst at the tail of the pancreas with a solid component but without dilation of the pancreatic duct. There is no involvement of any adjacent structures. EUS with FNA reveals high-grade dysplasia, later followed by distal pancreatectomy. Which of the following is the next best step in the management of this patient?
Choices: 1. 2. 3. 4.
No specific management Chemotherapy Radiation Surveillance with MRI
Answer: 4 - Surveillance with MRI Explanations: The American Gastroenterology Association (AGA) suggests that patients with invasive cancer or dysplasia in a cyst that has been surgically resected should undergo MRI surveillance of any remaining pancreas every 2 years. No further workup is inappropriate at this time. This patient will require regular surveillance with MRIs every 2 years given that she had a distal pancreatectomy and had dysplasia on her cytology. The 5-year survival for intraductal papillary mucinous neoplasm is about 45% to 50%. There is no role for chemotherapy in IPMN without invasive carcinoma or invasion of surrounding structures. The role of radiation in the management of IPMNs is unclear and can only be considered in a patient with invasive carcinoma. Go to the next page if you knew the correct answer, or click the link image(s) below to further research the concepts in this question (if desired).
Research Concepts: Intraductal Papillary Mucinous Cancer Of The Pancreas
We update eBooks quarterly and Apps daily based on user feedback. Please tap flag to report any questions that need improvement.
Question 84:
An older adult patient comes to your family practice with complaints of recurring fever, bone pain, frequent nosebleeds, and easy bruising. The provider explains that it would be best to perform some bloodwork to see if these symptoms stem from leukemia. What is the provider looking for if acute myeloid leukemia is suspected?
Choices: 1. 2. 3. 4.
Succinate Fumarate Lactate 2-hydroxyglutarate
Answer: 4 - 2-hydroxyglutarate Explanations: Isocitrate dehydrogenase mutations are found in 20% of adult patients with AML. Isocitrate dehydrogenase 2, an isoform of IDH, mitigates oxidative cellular damage in the mitochondria. A mutation in isocitrate dehydrogenase 2 results in epigenetic changes due to DNA and histone hypermethylation. 2-hydroxyglutarate, fumarate, and succinate are all used as oncometabolite biomarkers for pediatric cancers--whether for diagnosis or in monitoring recurrence. Go to the next page if you knew the correct answer, or click the link image(s) below to further research the concepts in this question (if desired).
Research Concepts: Biochemistry, Citric Acid Cycle
We update eBooks quarterly and Apps daily based on user feedback. Please tap flag to report any questions that need improvement.
Question 85:
A 54-year-old male presents with a 3week history of pain in the right hip worsening over the last week. He experiences right hip pain throughout the day. Although over the past 3 to 4 days, he notices worsening pain with weight-bearing. There is no history of fever in the last month. He is a chronic smoker and has become aware of increasing breathlessness for the prior 2 months. His radiograph shows a round opacity over the left lung apical region and lytic lesions over the left proximal humerus and right proximal femur. The lytic lesion of the femur is illdefined, measuring 4 cm x 3 cm in size, involving the intertrochanteric region. It involves more than two-thirds of the bone diameter. The proximal humerus lesion is partly lytic and sclerotic, with ill-defined borders, and measures 1 × 2 cm. It involves less than one-third of the bone diameter. This lesion is associated with mild pain. After a complete evaluation, the patient is diagnosed with pulmonary adenocarcinoma with skeletal metastasis. Which of these lesions would require prophylactic fixation?
Choices: 1. 2. 3. 4.
Not needed Both lesions Right proximal femur lesion Left proximal humerus lesion
Answer: 3 - Right proximal femur lesion Explanations: Mirel's classification is a system used to predict the risk of pathological fracture among long bones affected by metastasis. It is based on the site of the lesion, the extent of bone involved, the matrix of the lesion, and the presence of pain. One point is given for lesions in the upper limb, involving less than one-third of bone diameter, blastic matrix, and mild pain. Two points are given each to lesions in the lower limb, involving onethird to two-thirds of bone diameter, mixed lytic/sclerotic matrix, and moderate pain while three points are given each to trochanteric lesions, involving more than two-thirds of bone diameter, lytic matrix, and functional pain. A maximum score is 12, and a minimum score is 4. A score of more than or equal to 9 warrants a prophylactic fixation. In our case, Mirel's score for the proximal femur fracture is 12 and for the proximal humerus fracture is 5. Prophylactic fixation of the proximal femur lesion should be performed. The sensitivity of Mirel's classification is about 90% and the specificity is about 35%. Go to the next page if you knew the correct answer, or click the link image(s) below to further research the concepts in this question (if desired).
Research Concepts: Bone Metastasis
We update eBooks quarterly and Apps daily based on user feedback. Please tap flag to report any questions that need improvement.
Question 86:
A 4-year-old boy presented with spontaneous raccoon eyes. There was no history of trauma or coagulopathy. On examination, he had a non-tender abdominal lump. Which of the following is the strongest predictor of poor prognosis in this child?
Choices: 1. 2. 3. 4.
MYCN amplification 1p deletion 11q deletion ALK amplification
Answer: 1 - MYCN amplification Explanations: Metastatic neuroblastoma (secondary to tumoral obstruction of the palpebral vessels) can present as Raccoon eyes. Neuroblastoma can present as periorbital ecchymosis in 5.4% of cases. MYCN amplification has been reported in around 25% of neuroblastomas. MYCN amplification is generally accepted as the strongest predictor of poor prognosis and rapid tumor progression in neuroblastoma. Other poor prognostic features of neuroblastomas include deletions of 1p (30%) and 11q (45%) and an unbalanced gain of 17q (60%). Amplification of ALK is observed in 1 to 2% of cases of neuroblastomas and ALK is often co-amplified with MYCN. Go to the next page if you knew the correct answer, or click the link image(s) below to further research the concepts in this question (if desired).
Research Concepts: Raccoon Sign
We update eBooks quarterly and Apps daily based on user feedback. Please tap flag to report any questions that need improvement.
Question 87:
An 80-year-old female with no significant medical history presented to the primary care provider with generalized fatigue, weakness, and abdominal discomfort. There is no history of a cough, fever, chills, rash, bleeding, sick contact, or weight loss. She is a nonsmoker and does not drink alcohol. Vitals were within normal limits. Physical examination was positive for pallor and splenomegaly. Initial laboratory workup was significant for hemoglobin of 7 grams/dL, platelet count of 200 x 10^9/L, and a white blood cell count of 80 X 10^9/L with a neutrophil count of 65 X 10^9/L. Chest x-ray was unremarkable. The patient was referred to a hematologist for further evaluation. A genetic workup was negative for BCR-ABL1 and JAK2 mutation. A bone marrow biopsy showed hypercellularity and elevated neutrophil granulocytes. Which of the following genetic mutations is specific for diagnosing the likely diagnosis?
Choices: 1. 2. 3. 4.
Calreticulin (CALR) gene mutation MPL proto-oncogene mutation JAK2 V617F mutation CSF3R gene mutation
Answer: 4 - CSF3R gene mutation Explanations: CSF3R gene mutation is one of the diagnostic criteria for chronic neutrophilic leukemia (CNL), and CSF3RT618I is the most common mutation. Bone marrow biopsy in patients with CNL shows hypercellularity with elevated neutrophil counts and normal maturation of neutrophils. The patient can be diagnosed with CNL in the absence of CSF3R mutation if they have persistent neutrophilia for 3 months; splenomegaly; another etiology of neutrophilia, such as infection; and other myeloid neoplasms have been ruled out. JAK2 mutation is common in CNL but is not required for the diagnosis of CNL as per WHO diagnostic criteria. Go to the next page if you knew the correct answer, or click the link image(s) below to further research the concepts in this question (if desired).
Research Concepts: Myeloproliferative Neoplasms
We update eBooks quarterly and Apps daily based on user feedback. Please tap flag to report any questions that need improvement.
Question 88:
A soft tissue mass of the groin in a 17year-old male is biopsied and reveals a subcutaneous lesion consisting of cellular nodules of epithelioid cells. Some show marked rhabdoid morphology with prominent nucleoli. Immunohistochemical stains are performed showing positivity for cytokeratins, epithelial membrane antigen, vimentin, and CD34. The tumor cells are negative for CD31, S100, SOX-10, desmin, and myogenin. What is the most likely diagnosis?
Choices: 1. 2. 3. 4.
Metastatic carcinoma Rhabdoid melanoma Epithelioid sarcoma Rhabdomyosarcoma
Answer: 3 - Epithelioid sarcoma Explanations: Epithelioid sarcoma has a relatively unique immunohistochemical staining profile, expressing cytokeratins, epithelial membrane antigen, vimentin, and often express CD34 (approximately 50%). CD34 is unlikely to be positive in most carcinomas. Vimentin is positive in only select carcinomas, notably clear cell renal cell and endometrial. S100 and SOX-10 are negative in epithelioid sarcoma and are helpful in ruling out rhabdoid melanoma. Desmin and myogenin are negative in epithelioid sarcoma and are helpful in ruling out rhabdomyosarcoma. Go to the next page if you knew the correct answer, or click the link image(s) below to further research the concepts in this question (if desired).
Research Concepts: Epithelioid Sarcoma
We update eBooks quarterly and Apps daily based on user feedback. Please tap flag to report any questions that need improvement.
Question 89:
A 68-year-old male with a history of prostate cancer presented with new-onset back pain. X-ray of the spine showed a mass in the bilateral T12 pedicles and vertebral body. He has a normal neurological examination. What is the appropriate next step in the management of this patient condition?
Choices: 1. Pain medication and send home with follow up the primary care provider 2. Metastatic workup and spine surgery consultation 3. Biopsy of the T12 vertebral body mass 4. Surgical intervention to stabilize the spine
Answer: 2 - Metastatic workup and spine surgery consultation
Explanations: In patients with a history of cancer and acute back pain, metastasis should be considered as one of differential diagnosis. Metastasis spine tumors are the most common cancer in the spine. X-ray findings for concern for metastasis should be further investigated using CT, PET scan, or MRI. An isolated vertebral mass should be biopsied prior to definitive treatments. Go to the next page if you knew the correct answer, or click the link image(s) below to further research the concepts in this question (if desired).
Research Concepts: Spinal Metastasis
We update eBooks quarterly and Apps daily based on user feedback. Please tap flag to report any questions that need improvement.
Question 90:
A 56-year-old woman is being admitted to the medical floors after presenting to the emergency department for oncology consult and recommendations. The patient has been complaining for about 4 weeks of abdominal pain and on physical examination, is found to be jaundiced. Patient's laboratory results show elevated liver function tests and bilirubin level. Her tumor markers, including carcinoembryonic antigen (CEA) and carbohydrate antigen (CA) 19-9, are pending. On radiological evaluation by triple-phase contrast-enhanced computed tomography (CT), a 2.2-cm cystic lesion with a solid on the head of the pancreas with moderate bile duct dilation, borderline enlarged local lymphadenopathy, and no other neighboring organ involvement. What is the most appropriate step in the care of her disease?
Choices: 1. Perform a gadolinium-contrast magnetic resonance imaging (MRI) 2. Perform immediate upfront surgical resection for pancreatic cancer 3. Perform an endoscopic ultrasound (EUS) with fine-needle aspiration (FNA) 4. Wait for the carcinoembryonic antigen (CEA) and carbohydrate antigen (CA) 19-9 results to aid the next step
Answer: 3 - Perform an endoscopic ultrasound (EUS) with fine-needle aspiration (FNA)
Explanations: EUS-FNA provides high-quality imaging of mixed solid and cystic lesions, vasculature involvement, and the opportunity to biopsy suspicious pancreatic malignant lesions and lymph nodes, which will increase diagnostic accuracy. A contrast-MRI is required to assess major vessel involvement and resectability of pancreatic lesions and could help distinguish pancreatic carcinoma from mucinous cystic pancreatic neoplasms, although on this clinical scenario, the patient had an equivalent to MRI, sensitive and specific triple-phase contrast-enhanced CT for vascular resectability, lymph node assessment and liver metastasis evaluation, MRI will not provide further information. Treatment of pancreatic neoplasms mainly involves surgical resection. Although, pancreatic adenocarcinoma requires tissue biopsy for confirmation and further complete body imaging for final staging. Even though there may be a correlation of tumor marker (CEA and CA 19-9) with malignancy and prognosis, both have limited sensitivity and specificity, and do not discriminate between benign and malignant pancreaticbiliary disease. Therefore a biopsy is a mandate. Go to the next page if you knew the correct answer, or click the link image(s) below to further research the concepts in this question (if desired).
Research Concepts:
Mucinous Cystic Pancreatic Neoplasms
We update eBooks quarterly and Apps daily based on user feedback. Please tap flag to report any questions that need improvement.
Question 91:
What stage is invasive cervical carcinoma by cone biopsy, the upper portion of the vagina involved by tumor, and a CT positive for periaortic and pelvic nodes?
Choices: 1. 2. 3. 4.
lA lB IIIB IIIC2
Answer: 4 - IIIC2 Explanations: Cervical cancer is staged clinically, but this can often miss pelvic and aortic lymph nodes in 10-40% of patients. Thus, surgical staging is recommended. Stage lA disease is preclinical (i.e., microscopic). Stage 1B is a clinically visible lesion. Stage IVB denotes macroscopic disease spread to other parts of the body. Stage III is when the lesion extends to the pelvic wall and/or also involves the lower third of the vagina. There may be associated malfunction of the kidney or hydronephrosis. In stage IIIC, the tumor involves regional lymph nodes. In stage IIIC1, cancer has spread to the lymph nodes in the pelvis. In stage IIIC2, cancer has spread to the paraaortic nodes. Go to the next page if you knew the correct answer, or click the link image(s) below to further research the concepts in this question (if desired).
Research Concepts: Cervical Cancer
We update eBooks quarterly and Apps daily based on user feedback. Please tap flag to report any questions that need improvement.
Question 92:
A patient presents with a raised, skincolored 1 cm skin lesion on the lower abdomen. It is firm, but not ulcerated or irregular. A decision is made to perform a biopsy. The pathology report returns as “fibroepithelial tumor of Pinkus, incompletely excised.” This patient will have a good prognosis if which of the following treatments is employed?
Choices: 1. 2. 3. 4.
Complete excision Excision and radiation Excision followed by 5-FU and cisplatin No further treatment is necessary
Answer: 1 - Complete excision Explanations: The treatment of fibroepithelioma of Pinkus (FEP) involves recognition of the lesion, followed by complete excision. Electrodesiccation and curettage of the lesion may be effective, but since this is a rare and likely under-recognized tumor, evidence for the efficacy of these treatments is not available. Mohs surgery has been performed for this type of tumor and is indicated if clear margins are limited by anatomic location on the body. There is no role for radiation in the treatment of fibroepithelioma of Pinkus. FEP has not been reported to result in patient death. There is no role for chemotherapy in the treatment of fibroepithelioma of Pinkus, as it does not invade or metastasize. Clear margins should be obtained to prevent the recurrence of the lesion. Prognosis is generally excellent when complete excision is performed, and excision is considered curative. Go to the next page if you knew the correct answer, or click the link image(s) below to further research the concepts in this question (if desired).
Research Concepts: Premalignant Fibroepithelial Tumor Of Pinkus
We update eBooks quarterly and Apps daily based on user feedback. Please tap flag to report any questions that need improvement.
Question 93:
A 55-year-old male is presenting for follow up after recently being diagnosed with non-small cell lung cancer. He has no new complaints on this visit. No prior medical or surgical history and does not take any medications. He continues to smoke one pack per day for past 30 years. Staging workup for his cancer reveals a normal MRI of the brain. Contrast-enhanced CT of abdomen and pelvis does not reveal any evidence of distant metastasis but a left renal vein thrombus extending to the inferior vena cava. Both kidneys appear normal in size. Routine labs including complete blood count, renal and liver function panel are unremarkable. INR is 1. Urinalysis and microscopy are unremarkable. Which is the best treatment option for the patient’s renal vein thrombus?
Choices: 1. 2. 3. 4.
Aspirin 81mg and clopidogrel 75 mg daily Enoxaparin 1mg/kg Q12 hours Rivaroxaban No treatment is necessary
Answer: 2 - Enoxaparin 1mg/kg Q12 hours Explanations: Patients with asymptomatic renal vein thrombosis are best treated with anticoagulation. The reason is to prevent progression of the thrombus or development of a new thromboembolic event such as pulmonary embolism. Initial treatment consists of starting unfractionated or low-molecular-weight heparin and then bridged to warfarin, for a goal INR of 2 to 3. Anticoagulation is generally given for 6 to 12 months Novel oral anticoagulants (NOAC) are not approved for the treatment of renal vein thrombosis. Go to the next page if you knew the correct answer, or click the link image(s) below to further research the concepts in this question (if desired).
Research Concepts: Renal Vein Thrombosis
We update eBooks quarterly and Apps daily based on user feedback. Please tap flag to report any questions that need improvement.
Question 94:
A pathology specimen is examined under the microscope, hoping to obtain a diagnosis for the specimen presented. Eosinophilic polygonal cells with eosinophilic stained cytoplasm proliferating in the chondromatous matrix with some myxoid changes are noted. Nuclear irregularity, binucleated cells, bone permeation, and encasement are observed. The mitotic index is 10 mitoses per 10 high power fields. What is the most likely diagnosis?
Choices: 1. 2. 3. 4.
Enchondroma Chordoma Chondromyxoid fibroma Chondrosarcoma
Answer: 4 - Chondrosarcoma Explanations: Chondrosarcomas show abundant blue-grey cartilage matrix-production. Irregularly shaped lobules of cartilage varying in size and shape are present. These lobules are separated by fibrous bands or permeate bony trabeculae. Calcified areas suggesting the presence of a pre-existing enchondroma can often be found. The chondrocytes are atypical, with variable size and shape and contain enlarged hyperchromatic nuclei. Binucleation is frequently seen in chondrosarcomas. Chondroid matrix liquefaction or myxoid changes are a common feature of chondrosarcomas. Necrosis and mitoses can be seen. There is often permeation into the cortical bone and the marrow space with entrapment of bony trabeculae. Radiological and pathological findings are crucial for the differential diagnosis between enchondroma and chondrosarcoma. Comparison of histological and radiological findings is crucial to establish a correct diagnosis. Grading of chondrosarcomas is important and is useful in predicting histological behavior. Chondrosarcomas are divided into three grades based upon their histopathology: Grade I, Grade II, and Grade III. Go to the next page if you knew the correct answer, or click the link image(s) below to further research the concepts in this question (if desired).
Research Concepts:
Chondrosarcoma
We update eBooks quarterly and Apps daily based on user feedback. Please tap flag to report any questions that need improvement.
Question 95:
A 50-year-old male with a history of hairy cell leukemia presents to the clinic with weakness and fatigue for 1 month. He has been in remission for 3 years. He denies fever, shortness of breath, or abdominal discomfort. On examination, the spleen is mildly enlarged. Lab results show hemoglobin 10.5 grams/dL, WBC 4600/mm3 with an absolute neutrophil count of 1550/mm3, and platelet count 120,000/microliter. A peripheral blood smear shows hairy cells. After careful evaluation, diagnosis of relapsed disease is made, and the patient is considered for retreatment. Which of the following is the reason for retreatment?
Choices: 1. WBC 4600/mm3 with an absolute neutrophil count of 1550/mm3 2. Platelet 120,000/microliter 3. Enlarged spleen on examination 4. Constitutional symptoms
Answer: 4 - Constitutional symptoms Explanations: Criteria for retreatment is the same as the indication for initial treatment. Criteria one: Significant cytopenias including an absolute neutrophil count less than 1000/microliter, hemoglobin concentration less than 11 grams/dL, or platelet count less than 100,000/microliter. Criteria two: Symptomatic splenomegaly or symptomatic adenopathy. Asymptomatic splenomegaly is not an indication for retreatment. Criteria three: Constitutional symptoms such as fever, night sweats, fatigue, and weight loss. Go to the next page if you knew the correct answer, or click the link image(s) below to further research the concepts in this question (if desired).
Research Concepts: Hairy Cell Leukemia
We update eBooks quarterly and Apps daily based on user feedback. Please tap flag to report any questions that need improvement.
Question 96:
A 65-year-old male with a history of recurrent pancreatitis presents to the emergency department with severe epigastric pain. Labs show serum lipase 750 IU/L. Given his history of recurrent pancreatitis, he had a CT abdomen that showed a pancreatic cyst. MRCP reveals a 4 cm cyst with 1 mm dilation of the main pancreatic duct with a 2 mm solid component. Which of the following is the next best step in the management of this patient?
Choices: 1. 2. 3. 4.
Surgical resection Monitoring with annual MRI EUS with FNA No further work up is needed
Answer: 1 - Surgical resection Explanations: Given that there is involvement of the pancreatic duct, it is an intraductal papillary mucinous neoplasm (IPMN). American gastroenterology association (AGA) recommends surgical resection for IPMNs in patients with both a solid component and a dilated pancreatic duct and/or concerning features on EUS and FNA. Since this patient a solid component and a dilated main pancreatic duct, this makes his cyst at high risk for malignant transformation and will require surgical resection at this point. The AGA suggests that patients with pancreatic cysts less than 3 cm without a solid component or a dilated pancreatic duct undergo MRI for surveillance in 1 year and then every 2 years for a total of 5 years if there is no change in size or characteristics. This patient has a solid component and dilated main pancreatic duct and will require surgical resection. The AGA suggests that pancreatic cysts with at least two high-risk features, such as size greater than or equal to 3 cm, a dilated main pancreatic duct, or the presence of an associated solid component, should be examined with EUS-FNA. However, since this patient has involvement of the main pancreatic duct with more than 1 mm dilation and a large cyst size and having recurrent pancreatitis that is likely secondary to the cyst, he is at risk of having a malignant cyst and will require surgery. No further work up is inappropriate at this time given that the cyst will need to be resected.
Go to the next page if you knew the correct answer, or click the link image(s) below to further research the concepts in this question (if desired).
Research Concepts: Intraductal Papillary Mucinous Cancer Of The Pancreas
We update eBooks quarterly and Apps daily based on user feedback. Please tap flag to report any questions that need improvement.
Question 97:
A 43-year-old female presents with increasing blurred vision in her left eye for four weeks. A review of systems is positive for floaters in her left eye, decreased visual acuity, and headaches. She has a past medical history of idiopathic pulmonary fibrosis for which she underwent a lung transplant in her 30’s. Ophthalmologic slit-lamp examination reveals a widening of the optic nerve as well as vitreous debris. Pars plana vitrectomy with cytology shows atypical lymphoid cells along with many reactive lymphocytes. What is the next best step in the management of this patient?
Choices: 1. 2. 3. 4.
Methionine PET scan MRI brain with gadolinium contrast CT chest, abdomen and pelvis Fluorescein angiography
Answer: 2 - MRI brain with gadolinium contrast Explanations: Intraocular lymphoma can represent the initial presentation of primary central nervous system lymphoma. Symptoms of intraocular lymphoma are often nonspecific and can include blurry vision, floaters, decreased visual acuity, and visual hallucinations. It can also be asymptomatic. When suspected, slit-lamp examination of both eyes along with cytologic analysis of CSF and vitreous fluid aspirate aids in the diagnosis of intraocular lymphoma. It is of paramount importance that further central nervous system staging with an MRI brain should be done in patients diagnosed with intraocular lymphoma, as CNS disease has been shown to later develop in 80% of these patients. Go to the next page if you knew the correct answer, or click the link image(s) below to further research the concepts in this question (if desired).
Research Concepts: Central Nervous System Lymphoma
We update eBooks quarterly and Apps daily based on user feedback. Please tap flag to report any questions that need improvement.
Question 98:
A 70-year-old female undergoes a hysterectomy, salpingo-oophorectomy, pelvic and aortic lymph node dissection, and omentectomy for a malignant uterine neoplasm. The pathology report demonstrates a primary uterine malignancy with both a high-grade adenocarcinoma and a sarcomatous component. Metastatic disease is noted in several lymph nodes and the omentum. What is the most appropriate next treatment for this patient?
Choices: 1. 2. 3. 4.
Whole pelvic radiation Carboplatin and paclitaxel chemotherapy Carboplatin and ifosfamide chemotherapy Gemcitabine and docetaxel chemotherapy
Answer: 2 - Carboplatin and paclitaxel chemotherapy Explanations: This patient has a primary uterine malignancy. The combination of carcinomatous and sarcomatous elements is consistent with a uterine carcinosarcoma. The presence of metastatic disease in the lymph nodes and omentum constitutes Stage IV disease, necessitating further therapy. Chemotherapy is indicated; whole pelvic radiation alone would not be adequate treatment. A recent study presented at the American Society of Clinical Oncologists annual meeting in 2019 demonstrates that combined carboplatin and paclitaxel is not inferior to paclitaxel and ifosfamide in terms of overall survival and results in a longer progression-free interval for patients with carcinosarcoma. As such, carboplatin and paclitaxel would be favored over paclitaxel and ifosfamide. Gemcitabine and docetaxel is an appropriate regimen for patients with leiomyosarcoma, but this patient has a carcinosarcoma, so carboplatin and paclitaxel is a more appropriate choice. Go to the next page if you knew the correct answer, or click the link image(s) below to further research the concepts in this question (if desired).
Research Concepts: Endometrial Cancer
We update eBooks quarterly and Apps daily based on user feedback. Please tap flag to report any questions that need improvement.
Question 99:
A 67-year-old male is discussed in the head and neck interprofessional meeting. He has presented with a biopsy-proven squamous cell carcinoma (SCC) on the apex of his scalp measuring 5 cm in diameter. It had been growing over the last 6 month prior to him seeking medical help. CT imaging shows a deep lesion but that the underlying skull appears unaffected. What is the best course of action from the options stated below?
Choices: 1. Excise the lesion with appropriate margins and close with a rotational flap 2. Excise the lesion with appropriate margins and examine the bone below; if the bone is uninvolved close with a fullthickness skin graft 3. Excise the lesion with appropriate margins and examine the bone below; if the bone is uninvolved close with a splitthickness skin graft 4. Excise the lesion with appropriate margins and close with a free flap
Answer: 3 - Excise the lesion with appropriate margins and examine the bone below; if the bone is uninvolved close with a split-thickness skin graft
Explanations: Risk factors for SCC of the scalp include long term sun exposure, having fair skin (low Fitzpatrick skin types), chronic lesions, and immunosuppressed patients. The depth of invasion is an important prognostic factor for survival, with dural and brain parenchyma involvement being considered inoperable. Calvarial involvement requires bone to be drilled or removed. Often with large SCC scalp lesions, sentinel lymph node biopsying is done. This will help to decide on adjuvant therapy. Split-thickness skin grafting (STSG) would be ideal with the large defect caused here. The take rate is higher in STSG due to the graft's lower metabolic requirements. As this defect is down near to bone the vascular bed left behind the following excision will be poor, making a fullthickness skin graft more likely to fail. A dermatome is used to take donor skin from the thigh. The patients often require postoperative radiotherapy, but this requires further discussion at the interprofessional team meeting. Go to the next page if you knew the correct answer, or click the link image(s) below to further research the concepts in this question (if desired).
Research Concepts: Scalp Reconstruction
We update eBooks quarterly and Apps daily based on user feedback. Please tap flag to report any questions that need improvement.
Question 100:
A patient comes to the clinic with longstanding nonspecific abdominal pain, unintentional weight loss, jaundice, and fatigue. The CT of the abdomen and pelvis shows a 2 cm hilar irregular-shaped mass with subcentimeter extension to the left main hepatic bile duct. The surgical team considers resection and requests a 3D volumetric evaluation, which predicts future liver remnant volume that is inadequate. Which of the following is the next best step in the management of this patient?
Choices: 1. Chemotherapy and palliative therapy consult since the tumor is extending to both bile ducts and unresectable 2. Radiation therapy with chemoembolization of the tumor to see the response and re-evaluation after treatment 3. Targeted embolization 4. Targeted immunotherapy
Answer: 3 - Targeted embolization Explanations: The described clinical scenario is most consistent with hilar cholangiocarcinoma (Klatskin tumor). The invasion of both bile ducts is not a criterion for non-resectability for cholangiocarcinoma. Although surgery and curative liver transplantation are options for selected patients with perihilar carcinoma, 5year survival rates are 5 to 10% (the best outcome is with transplant and neoadjuvant therapy). For patients requiring hepatic resection who have a predicted future liver remnant volume that is inadequate, most institutions suggest portal vein embolization (PVE) to induce lobar hypertrophy of the unaffected side prior to resection. Targeted therapy options with RAF kinase inhibitors are also considered for intrahepatic cholangiocarcinomas with BRAF V600E mutations. Radiofrequency ablation has low effectiveness in lesions larger than 5 cm. Recurrence rates for intrahepatic cholangiocarcinoma are also quite high after radiofrequency ablation. Go to the next page if you knew the correct answer, or click the link image(s) below to further research the concepts in this question (if desired).
Research Concepts: Biliary Tract Cholangiocarcinoma
We update eBooks quarterly and Apps daily based on user feedback. Please tap flag to report any questions that need improvement.
Section 2 Question 101:
A male patient of 25 years of age presented with chronic abdominal pain and mild rectal bleeding increasing over two years. A colonoscopy revealed over 200 polyps throughout the colon. The pathology report indicates adenomatous polyps, and one of them is adenocarcinoma which invades the submucosa and the musculature. Which of the following is the most appropriate surgical recommendation for this patient?
Choices: 1. Resection of the rectum with a proximal colostomy (Hartmann intervention) 2. Subtotal colectomy with an ileorectal anastomosis 3. Left hemicolectomy with a low anterior resection of the rectum and coloanal anastomosis 4. Coloproctectomy with an ileal reservoir and ileoanal anastomosis
Answer: 4 - Coloproctectomy with an ileal reservoir and ileoanal anastomosis
Explanations: Familial adenomatous polyposis (FAP) is characterized by hundreds or thousands of precancerous adenomatous polyps, which generate hereditary polyposis, colorectal cancer, and other cancers. FAP is related to the germline mutations of the APC gene. It may also occur in extracolonic manifestations (desmoid tumor, osteomas, epidermoid cysts, papillary thyroid carcinoma, pancreatic carcinoma, gastric cancer, duodenal cancer, hepatobiliary, and central nervous system tumors. FAP is classified clinically in two ways: classical FAP and attenuated FAP. Classical FAP involves the entire colon with the appearance of 100 to 1,000 adenomatous polyps. The age of onset of polyps ranges from 10 to 20 years, and it is estimated that colorectal cancer (CRC) diagnosis corresponds with the 30 to 40-year age group. Attenuated FAP frequently affects the right colon with the appearance of 10 to 100 adenomatous polyps, and the appearance of the CRC occurs later. This procedure is indicated in patients with significant rectal involvement (more than 15 polyps or the presence of large flat adenomas or high-grade dysplasia), potentially curable rectal cancer. Total colectomy is recommended due to the risk of recurrence in patients with poor compliance for follow-up. Go to the next page if you knew the correct answer, or click the link image(s) below to further research the concepts in this question (if desired).
Research Concepts: A Review Of Hereditary Colorectal Cancers
We update eBooks quarterly and Apps daily based on user feedback. Please tap flag to report any questions that need improvement.
Question 102:
A 77-year-old patient is being evaluated at a home visit. She has dementia and stage-4 metastatic breast cancer. Her advance directive states no heroic measures are to be performed. End-of-life care is being delivered in the home. Improved pain control is being discussed. Which of the following pain scales is most appropriate to assess this patient’s pain?
Choices: 1. 2. 3. 4.
PQRST PAINAD CHIPPS COLDSPA
Answer: 2 - PAINAD Explanations: PAINAD is used when the patient's verbal responses might be unreliable because of a dementia diagnosis. The scale assesses the patient's breathing, vocalization, facial expression, body language, and ability to be consolability (0-2 each). The scale describes 0 as no pain and 10 as severe pain. PQRST is a pain scale, but because of dementia, the patient would not be able to give reliable answers to these questions. CHIPPS is a pain scale for infants and children. COLDSPA is a pain scale, but because of dementia, the patient would not be able to give reliable answers to these questions. Go to the next page if you knew the correct answer, or click the link image(s) below to further research the concepts in this question (if desired).
Research Concepts: End of Life Care
We update eBooks quarterly and Apps daily based on user feedback. Please tap flag to report any questions that need improvement.
Question 103:
A 54-year-old patient with known metastatic cancer is referred to nuclear medicine for a follow-up bone scan to assess the burden of osseous metastatic disease. She has completed a recent cycle of chemotherapy within the last month. If a bone scan is performed at this time, which of the following may be seen due to the recent administration of chemotherapy?
Choices: 1. 2. 3. 4.
Glove phenomenon Flare phenomenon Superscan Diffuse marrow activation
Answer: 2 - Flare phenomenon Explanations: The glove phenomenon is caused by an arterial injection and is characterized by significant asymmetric uptake extending from the arterial injection site down into the fingers. The flare phenomenon is a pattern of uptake where osseous metastatic disease may appear to worsen in the two to six months following chemotherapy. This likely represents increased bone turnover as the bone heals the area of prior metastatic disease. This worsening will be seen to resolve on follow-up bone scans. A superscan may be possible in this patient, but it would not be a result of recently administered chemotherapy. If the kidneys and bladder are not seen on a bone scan, this should raise suspicion of the so-called “superscan.” A superscan implies diffusely increased osseous uptake due to metastatic disease, most commonly breast or prostate cancer, or metabolic bone disease which may be difficult to detect due to its symmetric appearance. Diffuse marrow activation is a complicating factor for image interpretation following the administration of GCSF, or granulocyte-macrophage-colony-stimulating factor. G-CSF is given to patients who experience chemotherapy-induced myelosuppression. This treatment causes marrow hyperplasia and increased marrow blood flow, resulting in bone scans with symmetrically increased activity around the major joints, particularly the knees.
Go to the next page if you knew the correct answer, or click the link image(s) below to further research the concepts in this question (if desired).
Research Concepts: Bone Scan
We update eBooks quarterly and Apps daily based on user feedback. Please tap flag to report any questions that need improvement.
Question 104:
A 56-year-old euthyroid woman is found to have an asymptomatic thyroid nodule. Fine needle biopsy shows follicles that resemble normal thyroid tissue; however, after excision, the tumor cells show PAX-PPARgamma fusion gene. What is the most likely diagnosis?
Choices: 1. 2. 3. 4.
Follicular thyroid carcinoma Medullary thyroid carcinoma Papillary thyroid carcinoma Anaplastic thyroid carcinoma
Answer: 1 - Follicular thyroid carcinoma Explanations: Follicular thyroid carcinoma can be difficult to distinguish from follicular adenoma with fine needle biopsy. Almost half of patients with follicular carcinoma show RAS point mutations, while one-third may show PAXPPAR-gamma rearrangements, and only 3% show both. Paired box or PAX genes are associated with several cancers, including PAX-2 with optic nerve colobomas, PAX-5 with lymphoplasmacytoid lymphoma, and PAX-6 with Wilms tumor. Papillary thyroid cancer accounts for 80% of thyroid cancers, follicular carcinoma 10%, medullary 5%, and anaplastic 1.6%. Thyroid lymphoma may also occur, usually with a background of Hashimoto thyroiditis. Go to the next page if you knew the correct answer, or click the link image(s) below to further research the concepts in this question (if desired).
Research Concepts: Follicular Thyroid Cancer
We update eBooks quarterly and Apps daily based on user feedback. Please tap flag to report any questions that need improvement.
Question 105:
A 53-year-old female underwent tumorectomy for a breast carcinoma lesion. Histopathological examination of the surgical specimen established the diagnosis of tubular carcinoma. Immunohistochemical study revealed positive immunostaining for estrogen and progesterone receptors, negative immunostaining for HER2, and low index of proliferation: Ki67 5%. What is the molecular classification of this tumor?
Choices: 1. 2. 3. 4.
Luminal A Luminal B Basal-like Normal-like
Answer: 1 - Luminal A Explanations: The current molecular classification divides breast cancer into five groups: luminal A, luminal B, basal, HER2, and normal breast-like. Luminal A breast carcinoma is HER2 negative, hormone-receptor positive (estrogenreceptor and/or progesterone-receptor positive), and has low levels of Ki-67. Tubular carcinoma is nearly always positive for estrogen and progesterone receptors has a low growth fraction. Tubular carcinoma is typically negative for HER2, EGFR, P-cadherin, p53, and high molecular weight keratins. Tubular carcinoma typically displays a luminal A phenotype. It is characterized by high expression of luminal cytokeratins. Luminal A carcinomas are lowgrade, tend to grow slowly and have the best prognosis. Go to the next page if you knew the correct answer, or click the link image(s) below to further research the concepts in this question (if desired).
Research Concepts: Tubular Breast Carcinoma
We update eBooks quarterly and Apps daily based on user feedback. Please tap flag to report any questions that need improvement.
Question 106:
An immunohistochemical stain is performed on a biopsy sample from a large mass found on the back of the lower thyroid pole of a 60-year-old male in hypercalcemic crisis. His calcium level is 14.9 mg/dL, and PTH is elevated at 412 ng/L (normal range: 10-65 ng/L), raising a strong suspicion for parathyroid carcinoma. Which of the following immunohistochemical markers is most likely to be absent in this specimen?
Choices: 1. 2. 3. 4.
Parafibromin Chromogranin-A Keratin 7 Ki-67
Answer: 1 - Parafibromin Explanations: A complete loss of parafibromin expression is strongly associated with parathyroid carcinoma. Chromogranin-A is a neuroendocrine marker expressed in normal parathyroid tissue. The loss of this marker has not been found to be associated with parathyroid carcinoma. Keratin 7 is a marker typically found in normal parathyroid tissue. The loss of this marker has not been found to be associated with parathyroid carcinoma. Expression of Ki-67 is associated with parathyroid carcinoma, not loss of this marker. A Ki-67 proliferation index of greater than 5% is associated with parathyroid carcinoma. Go to the next page if you knew the correct answer, or click the link image(s) below to further research the concepts in this question (if desired).
Research Concepts: Histology, Parathyroid Gland
We update eBooks quarterly and Apps daily based on user feedback. Please tap flag to report any questions that need improvement.
Question 107:
An elderly male who presented with ongoing constipation, abdominal pain, and rectal bleeding for a year was recently diagnosed with rectal cancer. He undergoes radiation for rectal cancer following surgical eradication. Which of the following best describes the most likely initial changes that can be observed 2 to 3 hours following radiation?
Choices: 1. 2. 3. 4.
Inhibition of epithelial apoptosis in the crypts Superficial ulcerations Loss of villi cells Apoptotic fragments
Answer: 1 - Inhibition of epithelial apoptosis in the crypts
Explanations: The initial changes can be observed 2 to 3 hours following radiation and are mainly inhibition of epithelial apoptosis in the crypts. With time, the villi cells are lost; this leads to the absence of infectious and fluid barrier of the intestine. Apoptotic fragments consisting of nuclear or cytoplasm fragments and condensation of cytoplasm may also be visible. The repetitive injuries caused to the intestinal mucosa by ionizing radiation, as well as its complex mechanism of healing, is proposed to cause radiation enteritis. Exposure of normal tissues to radiation leads to the production of reactive ions that combine with intracellular water molecules to form radicals like hydroxyl and other free radicals. Go to the next page if you knew the correct answer, or click the link image(s) below to further research the concepts in this question (if desired).
Research Concepts: Radiation Enteritis
We update eBooks quarterly and Apps daily based on user feedback. Please tap flag to report any questions that
need improvement.
Question 108:
A 65-year-old Asian male with recently diagnosed prostate cancer presents to the clinic with several months' history of a progressive skin rash. Physical examination shows polygonal flat-topped erythematousbrown papules coalescing into sheets but sparing the skin folds. The patient is subsequently diagnosed with Papuloerythroderma of Ofuji. Which of the following would best describe this condition?
Choices: 1. 2. 3. 4.
It It It It
is most commonly seen in Chinese males usually does not require treatment and resolves itself is associated with elevated IgA levels is associated with eosinophilia
Answer: 4 - It is associated with eosinophilia Explanations: Papuloerythroderma of Ofuji is most commonly found in males over the age of 55 and has been associated with underlying malignancy, infectious processes, and various pharmacological agents. The most frequently associated underlying malignancies are gastric carcinomas, hepatocellular carcinoma, adenocarcinoma of the colon, cutaneous T-cell lymphoma, prostate cancer, and chronic lymphocytic leukemia. Peripheral eosinophilia, elevated IgE and lymphocytopenia are minor criteria used in the diagnosis of Papuloerythroderma of Ofuji. Other minor criteria used in the diagnosis of Papuloerythroderma of Ofuji include age greater than 55 and male gender. Major criteria proposed include erythroderma-like eruptions, ‘deck-chair sign,’ pruritus, and exclusion of CTCL or other skin diseases. Go to the next page if you knew the correct answer, or click the link image(s) below to further research the concepts in this question (if desired).
Research Concepts: Papuloerythroderma of Ofuji
We update eBooks quarterly and Apps daily based on user feedback. Please tap flag to report any questions that need improvement.
Question 109:
A 68-year-old man receives radiation therapy to the right face and neck for oropharyngeal cancer. Chemotherapy with paclitaxel is initiated a month later. Five days after the initiation of chemotherapy, he develops a pruritic rash on his right neck with associated edema. He is diagnosed with radiation recall at the site of the radiation therapy. Which of the following best describes radiation recall?
Choices: 1. Radiation recall most commonly affects internal organs rather than skin 2. Only chemotherapeutic agents can trigger radiation recall 3. Radiation recall occurs in just under 50% of patients undergoing both radiation therapy and chemotherapy 4. Corticosteroids are known to have protective effects against radiation recall in treating the symptoms during chemotherapy following radiation therapy
Answer: 4 - Corticosteroids are known to have protective effects against radiation recall in treating the symptoms during chemotherapy following radiation therapy
Explanations: Radiation recall is an acute inflammatory reaction that occurs at a site that has been exposed to ionizing radiation when a chemotherapeutic agent is used within approximately two months after the original radiation therapy. It occurs in up to 6% of patients undergoing radiation treatment. These reactions are drug-specific. Radiation recall usually is associated with commonly used chemotherapeutic agents, including anthracyclines, taxanes, and antimetabolites. Some of the chemotherapeutic agents involved are doxorubicin, docetaxel, paclitaxel, methotrexate, tamoxifen, vinblastine, hydroxyurea, and dactinomycin. Nonchemotherapeutic agents also can trigger radiation recall. Clinically, the most common signs and symptoms of radiation recall include maculopapular eruptions, dry desquamation, pruritus, swelling, and ulcerations at the site of the original radiation therapy. In general, the treatment of radiation recall symptoms is focused on local skin care and alleviation of pruritus and edema. Corticosteroids have been suggested to have some protective effects against radiation recall in treating the external symptoms during chemotherapy following radiation therapy. Go to the next page if you knew the correct answer, or click the link image(s) below to further research the concepts in this question (if desired).
Research Concepts: Radiation Therapy Induced Skin Ulcer
We update eBooks quarterly and Apps daily based on user feedback. Please tap flag to report any questions that need improvement.
Question 110:
An 85-year-old female is brought to the emergency department for evaluation of confusion. Her family reports she has become depressed and is talking to her long-dead husband. She complains of abdominal, back, and hip pain, and she also has new-onset constipation. The neurological exam is non-focal, the cardiopulmonary exam is unremarkable, and a 3-centimeter, fixed breast mass is discovered. What is the most likely ECG finding in this patient?
Choices: 1. 2. 3. 4.
Flattened T waves Prolonged QT interval Shortened QT interval Peaked T waves
Answer: 3 - Shortened QT interval Explanations: Given the fixed breast mass and symptoms, this patient presents with paraneoplastic hypercalcemia secondary to metastatic breast cancer. Hypercalcemia results in a shortened QT interval, and it can also cause an arrhythmia on rare occasions. Hypercalcemia can also cause calcium deposits in the coronary arteries and valves, resulting in increased cardiovascular morbidity. Hypercalcemia can affect the gastrointestinal system, central nervous system, kidneys, and musculoskeletal system. Symptoms may include abdominal pain, constipation, confusion, depression, nephrolithiasis, polyuria, weakness, and myopathy. The mnemonic for hypercalcemia is stones, bones, abdominal groans, and psychiatric overtones. Go to the next page if you knew the correct answer, or click the link image(s) below to further research the concepts in this question (if desired).
Research Concepts: Malignancy-Related Hypercalcemia
We update eBooks quarterly and Apps daily based on user feedback. Please tap flag to report any questions that need improvement.
Question 111:
A 75-year-old man with increasing pain because of mesothelioma and increasing pleural lesions is admitted to the hospital. He has been taking oxycodone controlled-release 30 mg every 12 hours and oxycodone 5 mg every 4 hours as needed for breakthrough pain (four doses a day). He rates his pain as 7/10, and he has been tolerating the opioids well. His calculated total oral morphine equivalents (OME) is 120 mg in 24 hours. Given his pain is not controlled, it is decided to increase his daily OMEs to 180 mg. What is the new prescription for his longacting and short-acting opioid?
Choices: 1. Oxycodone controlled-release 90 mg every 12 oxycodone 10 mg every 4 hours as needed 2. Oxycodone controlled-release 40 mg every 12 oxycodone 5 to 10 mg every 4 hours as needed 3. Oxycodone controlled-release 30 mg every 12 oxycodone 10 mg every 4 hours as needed 4. Oxycodone controlled-release 60 mg every 12 oxycodone 5 mg every 4 hours as needed
hours and hours and hours and hours and
Answer: 2 - Oxycodone controlled-release 40 mg every 12 hours and oxycodone 5 to 10 mg every 4 hours as needed
Explanations: Follow the steps for opioid conversion and calculation. 1. Determine the total daily dose of the present opioid. 2. Calculate the 24-hour oral morphine equivalents (OME). Add all scheduled and breakthrough opioid doses over 24 hours. Using the equianalgesic dose chart, determine the morphine equivalents of the current opioid. This will be the 24-hour equivalent of morphine: 24-hour OME = 24-hour dose of current opioid X 30/Number of equianalgesic units in current opioid. 3. Determine new opioid and route of administration. Using the equianalgesic dose chart, determine the 24-hour dosage of the new opioid. 24-hour dose of new opioid = Total OME X Equianalgesic units of new opioid/30. 4. Decrease the dose of new opioid due to incomplete crosstolerance. Decrease the dose of the new opioid by 50% to reach the final amount of the drug to be given. 5. Develop a pain prescription. 1. This patient's total daily dose of oxycodone is 80 mg. 2. 24 hour OME: 80 X 30/20 = 120 mg. 3. There is no change in opioid or route of administration. 4. There is no need to decrease for cross-tolerance. In step 5, the patient's pain is not well controlled. Accordingly, it makes sense to increase the dose by 50% for severe pain. Using the total oxycodone amount of 80 mg add 50%, making the total amount to be dispensed in the day 120 mg. To calculate the long-acting opioid dose, the amount of oxycodone controlled-release would be 96 mg. A commercially available dose of oxycodone
controlled-release close to that amount is 40 mg dosed every 12 hours. The short-acting formulation is calculated as 10% to 20% of the long-acting opioid, hence the dose of oxycodone for breakthrough pain would be 10 mg, dosed every 4 hours as needed. Go to the next page if you knew the correct answer, or click the link image(s) below to further research the concepts in this question (if desired).
Research Concepts: Opioid Equivalency
We update eBooks quarterly and Apps daily based on user feedback. Please tap flag to report any questions that need improvement.
Question 112:
A 55-year-old female is found to have low hemoglobin and macrocytic hyperchromic anemia during a routine check-up. Further investigations, including gastroscopy and gastric mucosal biopsies, confirm the presence of atrophic gastritis, achlorhydria, raised fasting gastrin, and absent H. pylori antibodies. Which of the following test results is associated with the highest gastric cancer risk?
Choices: 1. 2. 3. 4.
Elevated pepsinogen-1/pepsinogen-2 ratio Decreased pepsinogen-1 Normal iron studies Elevated serum zinc concentration
Answer: 2 - Decreased pepsinogen-1 Explanations: The patient has atrophic autoimmune gastritis in the presence of absent H. pylori antibodies, raised serum gastrin, and achlorhydria. Low level of pepsinogen 1, low pepsinogen-1/pepsinogen-2 ratio, together with high fasting gastrin increase the gastric cancer risk irrespective of the status of H. pylori infection. In patients with atrophic autoimmune gastritis, pepsinogen-1 is low; a decreased pepsinogen-1 is a risk factor for gastric cancer development. Patients with atrophic autoimmune gastritis usually have early iron-deficiency anemia. Iron deficiency is also seen in advanced disease. Achlorhydria and raised gastrin contribute to the pathogenesis of iron deficiency. Patients with atrophic autoimmune gastritis usually have associated failures in the absorption of vitamin B12, iron, calcium, magnesium, and zinc. Go to the next page if you knew the correct answer, or click the link image(s) below to further research the concepts in this question (if desired).
Research Concepts: Gastritis
We update eBooks quarterly and Apps daily based on user feedback. Please tap flag to report any questions that
need improvement.
Question 113:
A 65-year-old male presents with a redbrown, moist papule on the right forearm. Dermoscopy of the lesion shows vessels arranged in a curvilinear pearl necklace pattern. A biopsy of the lesion was performed, and histologic evaluation shows a well-demarcated area of acanthotic epidermis with regular psoriasiform hyperplasia comprised of pale staining keratinocytes that stained positively with periodic acid-Schiff (PAS) stain and there was an abrupt transition to normal appearing non-lesional epidermis. Which of the following is least likely to be associated with this condition?
Choices: 1. 2. 3. 4.
Stasis dermatitis Arthropod bites Obesity Pre-existing psoriatic plaques
Answer: 3 - Obesity Explanations: The exact etiology of clear cell acanthomas is unknown. Some evidence suggests this entity is an inflammatory dermatosis rather than neoplastic. Clear cell acanthomas have been reported to occur in areas of pre-existing inflammatory dermatoses such as psoriatic plaques, stasis dermatitis, and arthropod bites. Clear cell acanthomas have been shown to express cytokeratins expressed in other inflammatory dermatoses, such as psoriasis. Obesity has not been associated with clear cell acanthomas. Inflammatory dermatoses have been associated with their development. Go to the next page if you knew the correct answer, or click the link image(s) below to further research the concepts in this question (if desired).
Research Concepts: Clear Cell Acanthoma
We update eBooks quarterly and Apps daily based on user feedback. Please tap flag to report any questions that need improvement.
Question 114:
A 56-year-old man presents to the emergency department for a “bad rash.” Approximately three weeks ago, he developed painful ulcers in his mouth and on his lips, followed by the diffuse onset of fragile blisters affecting the extremities, chest, and back. Examination of the oral cavity reveals several ulcerations and erosions. Skin examination shows scattered large erosions and a few flaccid bullae. Nikolsky sign is positive. His past medical history is unremarkable except for nonHodgkin lymphoma in remission. Detection of which of the following autoantibodies is most likely to confirm the suspected diagnosis?
Choices: 1. 2. 3. 4.
Alfa chain of collagen VII Laminin 332 Tissue transglutaminase Envoplakin
Answer: 4 - Envoplakin Explanations: Paraneoplastic pemphigus patients can have autoantibodies to various proteins in plakins, which are an intracellular component of desmosomes and hemidesmosomes. Autoantibodies against envoplakin (210-kd), periplakin (190-kd), desmoplakin I (250-kd), desmoplakin II (210-kd), plectin (500-kd), alpha2macroglobulin-like–1 (170-kd), and bullous pemphigoid antigen I (230-kd) have been found. Patients may also exhibit antibodies to antigens typical in pemphigus vulgaris, such as desmoglein 3, or pemphigus foliaceus (desmoglein 1). Autoantibodies to the alfa chain of collagen VII are characteristic of epidermolysis bullosa acquisita. Laminin 332 is the target of an autoantibody in mucous membrane pemphigoid (MMP). However, the target antigens vary. Subsets of patients affected exclusively by oral and ocular mucosal diseases have autoantibodies targeting laminin 332, laminin 311, BP180, a6ß4 integrin, and BP230. Tissue trans-glutaminase auto-antibodies are observed in celiac disease. Dermatological manifestations of celiac disease include dermatitis herpiformis. Go to the next page if you knew the correct answer, or click the link image(s) below to further research the concepts in this question (if desired).
Research Concepts: Paraneoplastic Pemphigus
We update eBooks quarterly and Apps daily based on user feedback. Please tap flag to report any questions that need improvement.
Question 115:
A 76-year-old white male presents to the clinic complaining of a pearly papule with ulceration near his right cheekbone and right eye. The patient has lived in a sunny climate his whole life and is an avid golfer. The patient also complains of having lost his eyelashes on the affected side. Which of the following eyelash changes is most suggestive of underlying malignancy of the eyelid?
Choices: 1. 2. 3. 4.
Poliosis and Trichomegaly Madarosis and Trichomegaly Trichomegaly Poliosis and Madarosis
Answer: 4 - Poliosis and Madarosis Explanations: Poliosis is a graying or color change of the eyelash. This may be associated with conditions of the eye such as uveitis (VKH syndrome), thyroid dysregulation, stress, medication (chloramphenicol, cetuximab), or systemic conditions (Waardenburg syndrome, piebaldism, Tuberous sclerosis). Melanoma of the eyelid may present with poliosis Madarosis is a loss of eyelashes. Causes include eyelid malignancy (Basal cell carcinoma is most common), blepharitis and other dermatological conditions, autoimmune conditions, thyroid abnormalities, medication, trauma, nutritional abnormalities, and others. Trichomegaly is most likely associated with medication (prostaglandin analog eye drops), hair disorders, connective tissue disorders, uveitis, thyroid abnormalities, and inherited conditions (Cornelia de Lange syndrome, Familial trichomegaly, etc.). It is rarely associated with systemic malignancy such as metastatic renal cell carcinoma. Madarosis and poliosis are both associated with local eyelid malignancy. Madarosis is a loss of eyelashes. Causes include eyelid malignancy (Basal cell carcinoma is most common), blepharitis and other dermatological conditions, autoimmune conditions, thyroid abnormalities, medication, trauma, nutritional abnormalities, and others. Poliosis is a graying or color change of the eyelash. This may be associated with conditions of the eye such as uveitis (VKH syndrome), thyroid dysregulation, stress, medication
(chloramphenicol, cetuximab), or systemic conditions (Waardenburg syndrome, piebaldism, Tuberous sclerosis). Melanoma of the eyelid may present with poliosis Go to the next page if you knew the correct answer, or click the link image(s) below to further research the concepts in this question (if desired).
Research Concepts: Diseases of the Eyelashes
We update eBooks quarterly and Apps daily based on user feedback. Please tap flag to report any questions that need improvement.
Question 116:
An adult patient is noted to have multiple bilateral renal tumors composed of round and polygonal cells with abundant eosinophilic cytoplasm and round central nuclei. The tumors stain positive for CD117 and CK14 and negative for CK7 and Hales colloidal iron. This patient should be tested for which of the following?
Choices: 1. 2. 3. 4.
Beckwith-Wiedemann Lynch Birt-Hogg-Dube Tuberous sclerosis
Answer: 3 - Birt-Hogg-Dube Explanations: Birt-Hogg-Dube syndrome is an autosomal dominant syndrome of fibrofolliculomas, multiple renal tumors, and pulmonary cysts. It is associated with 17p12q11 abnormality involving the folliculin protein. Associated renal tumors are usually chromophobe carcinomas or oncocytomas. Patients usually have multiple tumors with a mean of 5.3. If a patient has multiple renal tumors, Birt-Hogg-Dube syndrome should be considered. The staining pattern for renal oncocytomas is CD117+, CK14+, EMA+, Hales colloidal iron-, and CK7-. Tuberous sclerosis is more closely associated with angiomyolipomas of the kidney. Lynch syndrome is most closely associated with colorectal cancers. Beckworth Wiedmann is a syndrome of overgrowth or organ enlargement but not renal tumors. Go to the next page if you knew the correct answer, or click the link image(s) below to further research the concepts in this question (if desired).
Research Concepts: Renal Oncocytoma
We update eBooks quarterly and Apps daily based on user feedback. Please tap flag to report any questions that
need improvement.
Question 117:
A 55-year-old man presents with worsening pruritic skin lesions on his thighs and gluteal region that have become bothersome. He has a past medical history of type 2 diabetes mellitus, coronary artery disease, and chronic kidney disease. His vital signs are within normal range. On examination, the skin lesions are superficial, erythematous plaques and involve more than 50% of his body surface area. A skin biopsy specimen demostrates malignant T-cells with irregular cerebriform nuclei. Given the likely diagnosis, which of the following viruses is associated with this patient's condition?
Choices: 1. 2. 3. 4.
Parvovirus Human immunodeficiency virus Human T-lymphotropic virus Hepatitis B virus
Answer: 3 - Human T-lymphotropic virus Explanations: Human T-lymphotropic virus (HTLV) is the etiological agent of adult T-cell leukemia/lymphoma (ATLL) and is associated with cutaneous T-cell lymphoma (CTCL). A confirmed causative role of HTLV-I in mycosis fungoides (MF) or Sezary syndrome (SS) remains controversial. A few cases have reported an association of SS with HTLV-I. However, retrospective studies done in multiple centers have not shown an association between the two. The role of HTLV-I in the pathogenesis of MF and SS needs to be explored further. No association has been shown with parvovirus, HIV, or hepatitis B virus. Go to the next page if you knew the correct answer, or click the link image(s) below to further research the concepts in this question (if desired).
Research Concepts: Sezary Syndrome
We update eBooks quarterly and Apps daily based on user feedback. Please tap flag to report any questions that need improvement.
Question 118:
The RTOG 98-11 trial randomized patients with locally advanced non-metastatic anal cancer to chemoradiation with 3D conformal radiation and either MMC/5FU or cisplatin/5FU. It showed an improved diseasefree survival, overall survival, and colostomy-free survival in the MMC arm. What is a major criticism of this trial?
Choices: 1. Adherence to radiotherapy was significantly worse in the cisplatin-based group 2. The radiation dose is not considered adequate for treatment of gross disease 3. The patients were not randomized in a balanced manner 4. The cisplatin/5FU arm received induction cisplatin, and 5FU is a potential confounding variable
Answer: 4 - The cisplatin/5FU arm received induction cisplatin, and 5FU is a potential confounding variable
Explanations: The RTOG 98-11 trial randomized 644 patients with T2T4, any N, M0 anal cancer to concurrent 5FU (1000 mg/m2), mitomycin-C (10 mg/m2) and radiation versus induction cisplatin (75 mg/m2) and 5FU (1000 mg/m2) for two cycles followed by concurrent cisplatin and 5FU at same doses. Because there were two variables in the experimental arm (both the induction chemotherapy and experimental concurrent chemotherapy regimen), this resulted in a potential confounding variable. Ben Josef et al. demonstrated that total treatment time correlates with colostomy failure and local failure. Therefore, it is unclear if the difference in colostomy rate in RTOG 98-11 was due to the chemotherapy regimens or the treatment time. The 5-year disease-free survival in RTOG 98-11 was 67.8% vs 57.8% (p = 0.006) for the MMC vs cisplatin arms. The 5-year overall survival rates were 78.3% vs 70.7% (p = 0.026) for the MMC vs cisplatin arms. The 5yr colostomy-free survival rates were 71.9% vs 65% (p = 0.05) for the MMC vs cisplatin arms. Standard of care for treatment of locally advanced nonmetastatic anal cancer is definitive chemoradiation with MMC/5FU. Go to the next page if you knew the correct answer, or click the link image(s) below to further research the concepts in this question (if desired).
Research Concepts:
Radiation Therapy For Anal Cancer
We update eBooks quarterly and Apps daily based on user feedback. Please tap flag to report any questions that need improvement.
Question 119:
A 55-year-old man with colorectal cancer undergoes primary resection of the tumor. He presents to initiate adjuvant chemotherapy with fluorouracilbased therapy. Previous records reveal a normal carcinoembryonic antigen (CEA) level before surgery. Which of the following best describes postoperative CEA testing in this patient?
Choices: 1. CEA surveillance is not recommended as it was normal before surgery 2. CEA testing should be done now and continued for at least 36 months after surgery 3. CEA surveillance should be initiated after adjuvant chemotherapy is completed 4. CEA surveillance is recommended only if combined with circulating tumor DNA testing
Answer: 3 - CEA surveillance should be initiated after adjuvant chemotherapy is completed
Explanations: Carcinoembryonic antigen (CEA) testing lacks sensitivity and specificity for colorectal cancer and is not a useful screening or diagnostic tool for the disease. However, it does have clinical utility in patients with established disease, as a marker of relapse, disease burden, and prognosis. The current recommendation is to obtain a CEA level every 3 to 6 months for at least 3 years after primary resection, even if the preoperative CEA level was normal. In patients undergoing adjuvant chemotherapy with fluorouracil-based regimens, it is advisable to begin CEA surveillance after adjuvant chemotherapy is completed. This is due to the increased number of falsely elevated CEA levels in patients receiving adjuvant chemotherapy with fluorouracil. In the correct clinical scenario, a progressively rising CEA level may indicate relapse and should be followed up with further testing, including imaging. According to expert guideline panels, there is no evidence of clinical utility for circulating tumor DNA testing at this time. Go to the next page if you knew the correct answer, or click the link image(s) below to further research the concepts in this question (if desired).
Research Concepts: Carcinoembryonic Antigen
We update eBooks quarterly and Apps daily based on user feedback. Please tap flag to report any questions that need improvement.
Question 120:
A 59-year-old male presents to the clinic for follow-up after an esophagogastroduodenoscopy (EGD). He has a long-standing history of gastroesophageal reflux disease (GERD) and takes 40 mg daily of esomeprazole. He reports following the recommendations of a dietician to control his symptoms. His EGD revealed Barrett metaplasia endoscopically and subsequent multiple biopsy samples tested negative for dysplasia. What is his overall annual risk for malignant transformation to adenocarcinoma?
Choices: 1. 2. 3. 4.
0.05% 0.5% 5% 50%
Answer: 2 - 0.5% Explanations: The overall risk of malignant transformation from nondysplastic Barrett esophagus is low but still substantial compared to the general population. Earlier estimates ranged between 0.25% to 2.9%, but more recent data puts the number closer to 0.5% per year. The risk of malignant transformation increases with the degree of dysplasia, with the highest risk for high-grade dysplasia, around 6%. The overall risk of malignant transformation from nondysplastic Barrett esophagus is low, close to 0.5% per year. Go to the next page if you knew the correct answer, or click the link image(s) below to further research the concepts in this question (if desired).
Research Concepts: Barrett Metaplasia
We update eBooks quarterly and Apps daily based on user feedback. Please tap flag to report any questions that need improvement.
Question 121:
A 65-year-old African American female has been admitted to the hospital with severe pain and swelling of both knee joints and wrist joints over the past month. She has a low-grade fever but is otherwise hemodynamically stable. Lab work shows a white cell count of 12000/microL with normal liver and renal function. There is evidence of bilateral knee joint tenderness with effusions. She has been very active functionally in the past. Her medical history is significant for small cell carcinoma of the lung treated with chemoradiation three years ago. She had post-radiation fibrosis with secondary cavitation and was diagnosed with aspergilloma for which she has been on voriconazole for the past two months. A bone scan to evaluate the knee joints shows evidence of bilateral tibial periostitis with the involvement of epiphysis and diaphysis. Which of the following is the best initial therapy for this patient?
Choices: 1. 2. 3. 4.
Intravenous bisphosphonates Change antifungal medication Chemoradiation therapy for small cell lung cancer Broad-spectrum antibiotics
Answer: 2 - Change antifungal medication Explanations: Periostitis caused by voriconazole can imitate secondary hypertrophic osteoarthropathy (HOA) and therefore should be considered in the differential. It can involve both epiphysis and diaphysis of long bones, unlike HOA which rarely involves epiphysis in secondary form. The precise mechanism of periostitis in association with voriconazole therapy is unclear; however, recent reports have suggested that fluoride toxicity might be the underlying etiology. It can involve both axial and appendicular skeleton, unlike HOA, which involves axial skeleton only. Go to the next page if you knew the correct answer, or click the link image(s) below to further research the concepts in this question (if desired).
Research Concepts: Secondary Hypertrophic Osteoarthropathy
We update eBooks quarterly and Apps daily based on user feedback. Please tap flag to report any questions that need improvement.
Question 122:
A 66-year-old woman presents to the clinic for her one-year surveillance visit after partial mastectomy and whole breast radiation for left T2N0M0 invasive ductal carcinoma. Her yearly mammogram shows a BIRADS 2 architectural distortion over the previous surgical site. What is the next best step in the management of this patient?
Choices: 1. 2. 3. 4.
Ultrasound-guided biopsy Stereotactic fine needle aspirate Re-excision of positive margins Repeat mammogram in 1 year
Answer: 4 - Repeat mammogram in 1 year Explanations: Architectural distortion is a common finding on surveillance mammogram following breast conservation therapy. BIRADS 2 lesions are considered benign and therefore do not require biopsy. Routine surveillance following breast conservation therapy should consist of a yearly bilateral screening mammogram. The development of new architectural distortion on surveillance imaging in previously clear tissue may require closer monitoring or biopsy-based on the BIRADS category. Go to the next page if you knew the correct answer, or click the link image(s) below to further research the concepts in this question (if desired).
Research Concepts: Breast Cancer Conservation Therapy
We update eBooks quarterly and Apps daily based on user feedback. Please tap flag to report any questions that need improvement.
Question 123:
A 48-year-old female presents for abdominal distension. Vitals and lab work completed is within normal limits. An abdominal CT is completed and demonstrates a 17 cm multiseptated cyst in the liver. Which imaging findings would one expect to see with a biliary cystadenoma (BCA)?
Choices: 1. A peripheral enhancing cystic lesion in segment VI with perilesional edema and associated segmental perfusion abnormality 2. A well defined unilocular hypoattenuating cystic lesion in segment VII without enhancement 3. A homogenous low attenuating lesion in segment VI with peripheral, nodular, discontinuous enhancement 4. A multiloculated cystic lesion in segment IV with thin intrinsic septa, mild peripheral and septal enhancement and mural nodularity
Answer: 4 - A multiloculated cystic lesion in segment IV with thin intrinsic septa, mild peripheral and septal enhancement and mural nodularity
Explanations: BCAs are solitary, multiloculated cystic lesions that are predominantly found in segment IV. Sonographically BCAs appear predominantly anechoic with thin echogenic internal separations. On CT and MRI BCA demonstrate imaging characteristics of typical cysts and may demonstrate subtle capsular and septal enhancement. Imaging features concerning for malignant transformation include internal debris, bile duct dilation, and enhancing nodularity. Go to the next page if you knew the correct answer, or click the link image(s) below to further research the concepts in this question (if desired).
Research Concepts: Hepatic Cystadenoma
We update eBooks quarterly and Apps daily based on user feedback. Please tap flag to report any questions that need improvement.
Question 124:
A 29-year-old woman who was diagnosed with ulcerative colitis eight years ago presents with severe diarrhea, abdominal pain, fever, tiredness, and blood in stools. She also has s history of cytomegalovirus (CMV) complicating her ulcerative colitis. Bloodwork reveals a hemoglobin of 9.5 g/dL and ESR 55 mm/h. Stool microscopy shows red blood cells and inflammatory cells, but no parasites, cysts, or ova. Tissue biopsies taken during colonoscopy are negative for CMV as per hematoxylin and eosin stain. Immunohistochemistry staining is not available. Which of the following is the next best investigation to confirm CMV infection in this patient?
Choices: 1. 2. 3. 4.
Quantification-PCR CMV DNA Stool culture for CMV CMV IgG and IgM CMV viremia
Answer: 1 - Quantification-PCR CMV DNA Explanations: Most current guidelines recommend that tissue PCR or immunohistochemistry for the diagnosis of CMV colitis in ulcerative colitis. Quantification-PCR CMV DNA is an ideal teat that could help in making the diagnosis. Stool culture for CMV test has good sensitivity and specificity for the diagnosis of CMV colitis but it takes a long time to obtain the results and has other limitations. CMV IgG verifies past exposure to CMV. CMV IgM verifies acute infection with CMV or reactivation. Both tests cannot help in making the diagnosis of CMV colitis. CMV viremia may help but the test has lower sensitivity for such diagnosis. Go to the next page if you knew the correct answer, or click the link image(s) below to further research the concepts in this question (if desired).
Research Concepts: Cytomegalovirus Colitis
We update eBooks quarterly and Apps daily based on user feedback. Please tap flag to report any questions that need improvement.
Question 125:
A 70-year-old man presents with a ninemonth history of several hyperpigmented patches on both legs. Initially, he ignored them because they were asymptomatic, but they have begun to bother him, becoming pruritic and enlarging. He denies constitutional symptoms, and his vital signs are within range. A complete skin examination demonstrates multiple hyperpigmented patches on both legs. A skin biopsy is performed, and the pathology is consistent with Mycosis fungoides. Peripheral blood flow cytometry demonstrates Sezary cells. A reduction of which of the following is most likely to be seen in this patient?
Choices: 1. 2. 3. 4.
IL-6 IL-10 IL-12 IL-18
Answer: 3 - IL-12 Explanations: Patients with Sezary syndrome (SS) have suppressed immunity, as the malignant cells produce Th2 cytokines that suppress Th1 immunity by decreasing IL-12 production. IL-12 stimulates the production of interferon-gamma and tumor necrosis factor-alpha, thus protecting against tumorigenesis. Interferon therapy (both IFN-alpha and IFN-gamma) constitute the preferred initial treatment of SS. Recombinant IL-12 can cause a therapeutic response in cutaneous T-cell lymphoma through normalizing interferon production, enhancing cell-mediated cytotoxicity, and augmenting natural killer cell cytotoxicity. However, the agent is currently unavailable. Go to the next page if you knew the correct answer, or click the link image(s) below to further research the concepts in this question (if desired).
Research Concepts: Sezary Syndrome
We update eBooks quarterly and Apps daily based on user feedback. Please tap flag to report any questions that need improvement.
Question 126:
A 65-year-old male with no significant past medical history presents with complaints of a 6-week history of diarrhea and abdominal cramping. He denies nausea, vomiting, or hematochezia, but endorses a 12pound (5 kg) weight loss. The physical examination is remarkable for flushing of the face, bilateral expiratory wheezes, and a right-sided systolic murmur. Twenty-fourhour urine testing reveals elevated levels of 5hydroxyindoleacetic acid. Abdominal CT reveals a mass in the tip of the appendix. What is indicated by the presence of these findings?
Choices: 1. 2. 3. 4.
Presence of tumor in the appendix only Tumor invasion through the serosa of the appendix Metastasis of tumor to the liver Tumor production of dopamine
Answer: 3 - Metastasis of tumor to the liver Explanations: The most common cancer of the appendix is a carcinoid tumor, often 2 to 3 cm in size at the distal tip of the appendix. The neoplasm is generally asymptomatic and found incidentally on abdominal CT or appendectomy. Sometimes, patients present with carcinoid syndrome. Carcinoid tumors are of neuroendocrine origin and produce serotonin and other vasoactive substances. Thus, carcinoid syndrome refers to the excess of serotonin which can cause flushing, wheezing, diarrhea, and right-sided heart murmurs. 5-hydroxyindoleacetic acid is the primary metabolite of serotonin, and urine levels can be used to assess for the presence of a carcinoid tumor. In a patient with a carcinoid tumor in the gastrointestinal tract, symptoms of carcinoid syndrome indicate metastasis, most commonly to the liver. This is because the liver metabolizes 5-hydroxytryptamine before it can reach systemic circulation and cause vasoactive effects. If it has metastasized, excess serotonin spills into the circulation and causes wheezing, flushing, diarrhea, and right-sided heart murmurs. Thus, tumor location determines whether carcinoid syndrome appears. Carcinoid syndrome rarely causes left-sided valvular disease because of pulmonary metabolism of the hormonal substances. Left-sided valvular disease in the context of carcinoid syndrome can occur with a patent foramen ovale.
Go to the next page if you knew the correct answer, or click the link image(s) below to further research the concepts in this question (if desired).
Research Concepts: Intestinal Carcinoid Cancer
We update eBooks quarterly and Apps daily based on user feedback. Please tap flag to report any questions that need improvement.
Question 127:
A 44-year-old woman presents with a history of abnormal vaginal bleeding. She cannot recall the last time she has been to a doctor. Upon physical examination a palpable mass is palpated in her cervix – a biopsy reveals squamous carcinoma of the cervix. She is prescribed pembrolizumab, what is the most appropriate dosing regimen for her?
Choices: 1. 2. 3. 4.
200 mg infusion dosed every three weeks 20 mg/kg infusion dosed weekly 5 mg infusion dosed weekly 1 mg infusion dosed daily
Answer: 1 - 200 mg infusion dosed every three weeks Explanations: The most common administration schedule is a 200 mg infusion given over 30 minutes every 3 weeks. An alternate approved schedule is 2 mg/kg every three weeks for pediatric patients. Most monoclonal antibodies are dosed based on body size, however, studies have shown that pembrolizumab fixed-dosing provides adequate coverage and yields the advantage of reduced dosage errors, convenience, and less waste (for an expensive therapy). While some of the earlier pembrolizumab clinical trials used every 2-week dosing, this is not one of the answer choices. Go to the next page if you knew the correct answer, or click the link image(s) below to further research the concepts in this question (if desired).
Research Concepts: Pembrolizumab
We update eBooks quarterly and Apps daily based on user feedback. Please tap flag to report any questions that need improvement.
Question 128:
A 50-year-old male presents to the clinic with a palpable, central neck mass that has been present for several months. He admits to a history of joint pain and arthritis that has been progressively worsening. He had a kidney stone 2 weeks ago that brought him to the emergency department (ED) because he was in excruciating pain. In the ED the patient was found to have hypercalcemia and a parathyroid hormone level of 10 times the upper limit of normal. What is the likely pathology?
Choices: 1. 2. 3. 4.
Benign primary hyperparathyroidism Parathyroid carcinoma Papillary thyroid carcinoma Medullary thyroid carcinoma
Answer: 2 - Parathyroid carcinoma Explanations: Laboratory evaluation is similar to that of primary hyperparathyroidism, which includes parathyroid hormone (PTH) and serum calcium levels. PTH and calcium levels tend to be higher in parathyroid carcinoma compared with primary hyperparathyroidism. A PTH level three times the upper limit of normal is suspicious, but a PTH level of 10 times the upper limit of normal has a positive predictive value of 84% for parathyroid carcinoma. A palpable neck mass in combination with hyperparathyroidism or hypercalcemia is suggestive of parathyroid carcinoma. Fifty percent of patients with parathyroid carcinoma present with a palpable central neck mass, kidney stones, or osteoporosis. Go to the next page if you knew the correct answer, or click the link image(s) below to further research the concepts in this question (if desired).
Research Concepts: Parathyroid Cancer
We update eBooks quarterly and Apps daily based on user feedback. Please tap flag to report any questions that need improvement.
Question 129:
A 64-year-old male with metastatic melanoma is receiving treatment with ipilimumab. He presents with complaints of fatigue and diarrhea. His symptoms started a week after the second cycle of the drug, and he reports having seven to ten episodes of watery diarrhea for the past five days. He denies any fever, chills, nausea, or vomiting. His 4-year-old grandson was recently sick with stomach flu. On examination, he appears mildly dehydrated, vital signs are within normal limits, and the abdominal exam shows mild, diffuse tenderness. Which of the following is the best management approach for this patient?
Choices: 1. Symptomatic treatment for viral gastroenteritis with fluids and antidiarrheal 2. Check for Clostridium difficile and start empiric metronidazole 3. Hold further doses of ipilimumab and start prednisone 1 to 2 mg/kg daily 4. Hold ipilimumab and start loperamide
Answer: 3 - Hold further doses of ipilimumab and start prednisone 1 to 2 mg/kg daily
Explanations: This patient likely has immune-mediated colitis related to ipilimumab. He has severe diarrhea, rated at grade 3 per the Common Terminology Criteria for Adverse Events (CTCAE). Severe or life-threatening colitis is seen in about 7% of patients treated with ipilimumab. Ruling out other causes of diarrhea would be important. While performing the workup, holding further doses of ipilimumab is crucial for management. Patients with severe colitis need to be treated with highdose corticosteroids (1 to 2 mg/kg of prednisone). They need to be continued until the resolution of diarrhea to grade 1 and slowly tapered. Ipilimumab activates the immune system by targeting CTLA-4. CTLA-4 is a protein receptor that functions as an immune system checkpoint and downregulates immune response. By targeting this receptor, cytotoxic Tlymphocytes are activated and can recognize and attack cancer cells. Go to the next page if you knew the correct answer, or click the link image(s) below to further research the concepts in this question (if desired).
Research Concepts: Ipilimumab
We update eBooks quarterly and Apps daily based on user feedback. Please tap flag to report any questions that need improvement.
Question 130:
A 65-year-old man with squamous cell carcinoma of the penis invading the left corpus cavernosum undergoes partial penectomy. After 6 weeks of cephalexin therapy, a 3.5 cm right inguinal lymph node has decreased in size to 2.0 cm. A pelvic CT scan is normal. What is the next step in the management of this patient?
Choices: 1. 2. 3. 4.
Re-evaluation in 3 months Sentinel lymph node biopsy Fine needle aspiration Bilateral inguinal lymph node dissection
Answer: 4 - Bilateral inguinal lymph node dissection Explanations: This patient has stage II penile cancer with invasion into the corpora. This is associated with a much higher incidence of positive lymph nodes. The patient's right inguinal lymph node is still palpable after 6 weeks of treatment, even though it has decreased in size. It needs to be excised. This patient is at high risk for nodal disease; therefore, a negative fine needle aspiration or sentinel lymph node biopsy should not dissuade the practitioner from performing lymphadenectomy on this patient. Patients found to have unilateral positive groin nodes should undergo bilateral lymphadenectomy due to the high rate of bilateral disease. Go to the next page if you knew the correct answer, or click the link image(s) below to further research the concepts in this question (if desired).
Research Concepts: Penile Cancer
We update eBooks quarterly and Apps daily based on user feedback. Please tap flag to report any questions that need improvement.
Question 131:
A 28-year-old-male presented with a two-year history of cough, hemoptysis, and shortness of breath. Chest x-ray showed a left hilar opacity. Chest computed tomography (CT) scan revealed an oval mass in the left upper lobe obstructing the left upper lobar bronchus with a collapse of the upper lobe segments. The patient underwent a left upper lobectomy with regional lymph node dissection through a left posterolateral thoracotomy. Histopathological examination of the surgical specimen revealed a tumor proliferation arranged in a trabecular pattern with a fine vascular stroma. The tumor cells were uniform with finely granular nuclear chromatin and abundant eosinophilic cytoplasm. There was one mitosis per 2 mm2 and no foci of necrosis. Immunohistochemically, the tumor cells were positive for chromogranin A, synaptophysin, and CD56. The Ki-67 index was about 1%. What is the most likely diagnosis?
Choices: 1. 2. 3. 4.
Small cell carcinoma Typical carcinoid Atypical carcinoid Squamous cell carcinoma
Answer: 2 - Typical carcinoid Explanations: Typical carcinoids are carcinoid tumors with less than 2 mitoses per 2 mm2 without necrosis. Carcinoid tumors are characterized by growth patterns suggesting neuroendocrine differentiation. Organoid and trabecular patterns are most common; however, rosette formation, papillary growth, and follicular growth may also be seen. The tumor cells are usually uniform in appearance with a polygonal shape, finely granular nuclear chromatin, inconspicuous nucleoli, and moderate to abundant eosinophilic cytoplasm. The background stroma is classically highly vascularized. Immunohistochemistry may be required to confirm neuroendocrine and epithelial differentiation. An antibody panel including chromogranin A, synaptophysin, and CD56 is recommended. In carcinoids, Ki67 is low less than 10% to 20%. Go to the next page if you knew the correct answer, or click the link image(s) below to further research the concepts in this question (if desired).
Research Concepts: Lung Carcinoid Tumors
We update eBooks quarterly and Apps daily based on user feedback. Please tap flag to report any questions that need improvement.
Question 132:
A 66-year-old female presents to her dermatologist's office for an annual skin exam. She has white skin, white hair, and eyes that rapidly dart side-toside. She has a history of multiple squamous cell carcinomas requiring Mohs microsurgery. The patient most likely has a defect in which enzyme?
Choices: 1. 2. 3. 4.
Aromatic L-amino acid decarboxylase Tyrosinase Phenylalanine hydroxylase Dopamine hydroxylase
Answer: 2 - Tyrosinase Explanations: Aromatic l-amino acid decarboxylase (AADC) deficiency is a genetic disorder that affects neurotransmitter production. Signs and symptoms include developmental delay, hypotonia, and athetosis. People who suffer from this may have ptosis, miosis, and oculogyric crisis with rotational eye movements, but not nystagmus. They are not at increased risk for squamous cell carcinoma. The patient's presentation is most consistent with oculocutaneous albinism (OCA), type 1a. While white hair is not uncommon in her age group, the presence of white, not just pale, skin and nystagmus, noted as "eyes that rapidly dart from side-to-side," is sufficient to suspect albinism. OCA, type 1a is caused by a tyrosinase gene mutation. Tyrosinase is imperative for melanin production, and individuals with OCA are melanin deficient. Because melanin is the major photoprotective pigment in the skin, hypomelanotic individuals are at increaesed risk for solar-damage and UV-associated malignancies, such as squamous cell carcinoma. While not the most common form of albinism worldwide, it is the most common form in America and has the classic presentation of white-hair and skin. All other forms of oculocutaneous albinism retain some pigmentation. Phenylalanine hydroxylase deficiency is a genetic condition that results in the inability to process phenylalanine, resulting in a systemic accumulation of the amino acid. This condition is called phenylketonuria (PKU). The surplus phenylalanine is neurotoxic and can cause irreversible cognitive delay. PKU is associated with
eczema but not increased risk of squamous cell carcinoma. Dopamine hydroxylase deficiency leads to increased levels of serum dopamine and deficiency of epinephrine and norepinephrine. As the major neurotransmitters of the autonomic nervous system, epinephrine and norepinephrine modulate the sympathetic and parasympathetic responses. Thus, collapse of the autonomic nervous system causes hypotonia, hypothermia, hypoglycemia, and orthostatic hypotension. Sufferers also may have ptosis, nasal congestion, vomiting, and dehydration. There are no skin manifestations or increased risk of malignancy. Go to the next page if you knew the correct answer, or click the link image(s) below to further research the concepts in this question (if desired).
Research Concepts: Albinism
We update eBooks quarterly and Apps daily based on user feedback. Please tap flag to report any questions that need improvement.
Question 133:
A 15-year-old male is found to have medulloblastoma with significant polyposis of the colon. What is the most likely genotype for this presentation?
Choices: 1. 2. 3. 4.
Homozygous mutation of the APC gene Homozygous mutation of the hMLH1 gene Heterozygous mutation of the APC gene Biallelic loss of the hML1a gene
Answer: 1 - Homozygous mutation of the APC gene Explanations: Turcot syndrome is characterized by multiple benign colon polyps occurring in association with a primary brain tumor. A homozygous mutation of the APC gene is associated with Turcot syndrome type 2. Medulloblastoma is the most common brain tumor that arises with this mutation. A homozygous mutation of the hMLH1 gene is identified with Turcot syndrome type 1, in which glioblastoma multiforme is the most common brain tumor. Heterozygous mutation of the APC gene results in familial adenomatous polyposis (FAP). A homozygous mutation of that same gene can result in Turcot syndrome. "Biallelic loss" of a gene is another way of saying "homozygous mutation." This means that both copies of a specific gene are lost. Turcot syndrome type 1 requires a biallelic loss of the MMR genes. Turcot syndrome type 2 requires a biallelic loss of the APC gene. Go to the next page if you knew the correct answer, or click the link image(s) below to further research the concepts in this question (if desired).
Research Concepts: Turcot Syndrome
We update eBooks quarterly and Apps daily based on user feedback. Please tap flag to report any questions that need improvement.
Question 134:
A 56-year-old man with a history of hypertension, tobacco use disorder, and chronic obstructive pulmonary disease presents with loss of appetite, weight loss, imbalance, and a recent change in his speech and vision. Physical examination is remarkable for diminished bilateral lower extremity sensation, proximal leg muscle weakness, an abnormal gait, areflexia, and positive Romberg sign. MRI brain is negative, and CT chest shows a right upper lobe lung mass. A biopsy of the lung mass is positive for small cell lung cancer (SCLC). Which paraneoplastic antibody is most likely to be positive for this patient?
Choices: 1. Anti-recoverin 2. Anti-Tr (delta/notch-like epidermal growth factor-related receptor) 3. Anti-VGCC (voltage-gated calcium channel) 4. Anti-Yo PCA-1(Purkinje cell antibody)
Answer: 3 - Anti-VGCC (voltage-gated calcium channel) Explanations: Anti-VGCC is associated with small cell lung cancer (SCLC). It causes cerebellar degeneration leading to complex clinical manifestations, including proximal muscle weakness (primary symptom), areflexia, and autonomic dysfunction (e.g., dry mouth). Anti-VGCC is also associated with Lambert-Eaton myasthenic syndrome (LEMS). Anti-recoverin autoantibodies have been associated with cancer-associated retinopathy, a paraneoplastic disease. Anti-TR (DNER) is a paraneoplastic antibody associated with Hodgkin lymphoma and clinically manifested with cerebellar signs and symptoms. Anti-Yo PCA-1 is also well recognized paraneoplastic antibody which manifests with cerebellar signs and symptoms but is associated with gynecologic and breast cancers. Go to the next page if you knew the correct answer, or click the link image(s) below to further research the concepts in this question (if desired).
Research Concepts: Paraneoplastic Syndromes
We update eBooks quarterly and Apps daily based on user feedback. Please tap flag to report any questions that
need improvement.
Question 135:
A 59-year old patient presents to the clinic with complaints of diarrhea and flushing. He claims that his skin has been very itchy lately and he seems to be more forgetful than usual. He did go to his general practitioner a few weeks ago for vague abdominal pain but was told that he had developed irritable bowel syndrome. He was started on vitamin B3 but his symptoms have persisted. All of the other routine blood work appears normal. You suspect that the patient has a lesion in his gastrointestinal tract. What is the ideal test for detecting the tumor that this patient may have?
Choices: 1. 2. 3. 4.
Endoscopy Contrast-enhanced triple phase CT Ultrasound MRI
Answer: 2 - Contrast-enhanced triple phase CT Explanations: Contrast-enhanced triple phase CT is recommended for evaluation of all patients with carcinoid. Somatostatin receptor scintigraphy is a useful test for detecting primary and metastatic carcinoids. Upper and lower endoscopy with attention to the terminal ileum should be performed for the evaluation of metastatic carcinoid with an unknown primary site. MRI can be used as an alternative to CT. It has the highest sensitivity to detect liver metastasis. Ultrasound has low sensitivity to detect carcinoid tumors. Go to the next page if you knew the correct answer, or click the link image(s) below to further research the concepts in this question (if desired).
Research Concepts: Intestinal Carcinoid Cancer
We update eBooks quarterly and Apps daily based on user feedback. Please tap flag to report any questions that need improvement.
Question 136:
A 65-year-old darkly pigmented male noticed a discolored lesion on the sole of his foot. He denies any pain or history of comorbidity. Upon examination, the patient has an irregular lesion of 1-cm maximum dimension that is purplish in color and located on his right heel. No other lesions are on the foot or leg. The distal pulses are normal, and there is no pedal edema. Inguinal lymph nodes are not palpable. The patient undergoes a punch biopsy from the edge of the lesion, which is diagnostic of melanoma greater than 1 millimeter in thickness. What is the best therapeutic option for this patient?
Choices: 1. Interferon-alfa chemotherapy 2. Wide excision with 2 cm margin and biopsy of the sentinel lymph node 3. Above-the-knee amputation 4. Below-the-knee amputation and biopsy of the sentinel lymph node
Answer: 2 - Wide excision with 2 cm margin and biopsy of the sentinel lymph node
Explanations: Wide excision with a dissection of the sentinel lymph node is the best option for this patient. The margin for wide local excision depends on its Breslow thickness. Amputation is too radical an approach and is not indicated. Elective lymph node dissection is no longer advised unless biopsy of the sentinel lymph node reveals micrometastases. Interferon-alfa chemotherapy may be an option for treatment if metastatic nodes are found in the biopsy sample. Go to the next page if you knew the correct answer, or click the link image(s) below to further research the concepts in this question (if desired).
Research Concepts: Malignant Melanoma
We update eBooks quarterly and Apps daily based on user feedback. Please tap flag to report any questions that need improvement.
Question 137:
A 55-year-old female presents to the clinic with worsening abdominal distension and right iliac fossa pain for the past two months. She has no significant past medical history. Physical examination shows moderate abdominal ascites and pale conjunctivae. An ultrasound reveals a suspicious 5 cm mass arising from the appendix. Which tumor marker is most specific for this tumor?
Choices: 1. 2. 3. 4.
CA125 CK7 MUC2 MLH1
Answer: 3 - MUC2 Explanations: Pseudomyxoma peritonei most frequently arises from the appendix. The primary site is identified predominantly as mucinous appendiceal adenocarcinoma. Other primary sites are small and large bowel, stomach, pancreas, lung, breast, gallbladder, Fallopian tubes, and ovaries. MUC 2 (mucin 2) over-expression has been suggested as a molecular marker for PMP of appendiceal origin. Mucins are high molecular weight glycoproteins produced by many epithelial tissues, especially the GI system. The protein is secreted and forms an insoluble mucous barrier that protects the gut lumen. Other positive markers of appendiceal origin pseudomyxoma peritonei include CK20, CEA, and CDX2. CA 125 is also found to be high but with ovarian involvement. Cytokeratin 7 (CK7) proteins are found in most ductal, glandular, and transitional epithelium of the urinary tract and bile duct epithelial cells, but it's often negative in appendiceal tumors. Loss of protein expression of the repair genes MLH1 has also been reported in pseudomyxoma peritonei. Go to the next page if you knew the correct answer, or click the link image(s) below to further research the concepts in this question (if desired).
Research Concepts: Pseudomyxoma Peritonei
We update eBooks quarterly and Apps daily based on user feedback. Please tap flag to report any questions that need improvement.
Question 138:
A 70-year-old white female presents to the clinic with a chief complaint of several moles on her legs. Past medical history includes type 2 diabetes and hypertension. Her blood pressure is 127/76 mmHg, while her hemoglobin A1c is 6.5%. She has type I Fitzpatrick skin type. Physical examination reveals three brown to black nevi on both legs. Her skin appears tanned. Each of the nevi is symmetric with smooth borders and measure less than 6 mm in diameter. Dermoscopy examination reveals reticulated melanocytic nevi patterns in all the lesions. Her nails, soles, and toe webs are clear of pigmented lesions. She reports having had two benign moles removed from her back three years ago. She enjoys sitting in the sun with her legs uncovered. Which of the following is the best advice for this patient?
Choices: 1. Punch biopsies are recommended to detect melanoma. 2. The benign appearing moles should be watched for any changes, and sunscreen with a skin protection factor of at least 15 should be applied every day. 3. The moles appear to be benign and do not need periodic clinical monitoring. 4. An excisional biopsy of each lesion should be done.
Answer: 2 - The benign appearing moles should be watched for any changes, and sunscreen with a skin protection factor of at least 15 should be applied every day.
Explanations: Besides sun exposure, independent risk factors for developing melanoma include a history of childhood cancer; Parkinson disease, high birth weight, and inflammatory bowel disease. First use of tanning bed before age 35 years is associated with a 75-fold increase in risk for melanoma. Dermoscopy can be helpful for decision making about performing biopsy but rarely overrides clinical judgment. Asymmetry, notched border, color variations, diameters greater than 6 mm or any changes in a mole over time are all suspicious signs of melanoma. Go to the next page if you knew the correct answer, or click the link image(s) below to further research the concepts in this question (if desired).
Research Concepts: Melanocytic Nevi
We update eBooks quarterly and Apps daily based on user feedback. Please tap flag to report any questions that need improvement.
Question 139:
A 32-year-old woman with a past medical history of acute myelocytic leukemia presents complaining of cough with a small amount of bright red blood. She had a one-week history of cough productive of thick brown sputum, fever, and pleuritic chest pain. A chest x-ray done 5 days ago revealed a right upper lobe infiltrate; she was immediately started on oral antibiotics by her primary care provider. However, the symptoms persist. The patient underwent allogeneic stem cell transplantation for her acute myelocytic leukemia 5 weeks ago, complicated by acute graft-versus-host disease and neutropenia. Temperature is 39.5 C (103. 1 F), blood pressure is 100/62 mmHg, pulse is 110/min, and respiratory rate is 20/min. The physical exam is significant for right-sided crackles. Laboratory results show leukocytes 1500/microL, hematocrit 28%, and platelets 138,000/microL. Chest x-ray shows a right upper lobe infiltrate, increased in size compared to the previous x-ray. Chest CT scan reveals several nodular lesions with surrounding ground-glass opacities in the right upper lobe. Sputum gram stain shows no organisms. What is the best initial therapy for this patient?
Choices: 1. 2. 3. 4.
Voriconazole Voriconazole + caspofungin Caspofungin Embolization
Answer: 1 - Voriconazole Explanations: Caspofungin is effective against invasive aspergillosis in patients who are refractory to or not tolerating voriconazole. The main population at risk of developing invasive aspergillosis is immunocompromised patients. These patients will typically present with a triad of fever, chest pain, and hemoptysis. Chest CT reveals pulmonary nodules with the "halo" sign. Treatment consists of voriconazole with or without caspofungin. Caspofungin is not recommended as first-line therapy for invasive aspergillosis. Patients on caspofungin should be monitored for hepatotoxicity, as it has been associated with increased liver enzymes. Go to the next page if you knew the correct answer, or click the link image(s) below to further research the concepts in this question (if desired).
Research Concepts: Caspofungin
We update eBooks quarterly and Apps daily based on user feedback. Please tap flag to report any questions that need improvement.
Question 140:
A 75-year-old male with stage IIIB adenocarcinoma of the lung is undergoing thoracentesis of a newly-discovered large right pleural effusion for which he was admitted after presenting with increased dyspnea on exertion ongoing for four weeks. Pleural pressures are measured and demonstrate initial pleural pressure of -10 cm H2O; this further decreases to -40 cm H2O after removal of 500 mL of pleural fluid. A CT chest obtained after thoracentesis reveals the right lung has not re-expanded with thoracentesis and small hydropneumothorax. This is treated conservatively with supplemental oxygen, and the patient is discharged home. Cytology from pleural fluid returns demonstrating adenocarcinoma. One week later, the patient returns to the emergency department with respiratory distress and is found to have reaccumulation of large, right pleural effusion. Which of the following treatment options should the patient receive?
Choices: 1. 2. 3. 4.
Indwelling pleural catheter Talc pleurodesis Pleurodesis with bleomycin Placement of pleuro-peritoneal shunt
Answer: 1 - Indwelling pleural catheter Explanations: The patient demonstrates trapped lung physiology as demonstrated by initially negative pleural pressure followed by steep decline in pleural pressure after removal of pleural fluid as well as evidence of no lung re-expansion on follow up imaging. For patients with recurrent malignant effusion and trapped lung physiology, an indwelling pleural catheter is the best palliative treatment available. Indwelling pleural catheters have been shown to result in fewer hospitalization days related to effusion and improvement in dyspnea. In certain patients, indwelling pleural catheters can result in spontaneous pleurodesis and removal thereafter. Go to the next page if you knew the correct answer, or click the link image(s) below to further research the concepts in this question (if desired).
Research Concepts: Malignant Effusion
We update eBooks quarterly and Apps daily based on user feedback. Please tap flag to report any questions that need improvement.
Question 141:
A 37-year-old female patient with no significant past medical history, presented with a left breast lesion detected during the self-breast examination. Mammography showed a mass lesion with central density and spiculated margins measuring 1.2 cm. The patient underwent tumor resection. Histological examination revealed a tumor proliferation composed of more than 90% of angulated tubules lined by a single layer of epithelial cells. These tubules were arranged haphazardly within a cellular desmoplastic stroma. There was little nuclear pleomorphism, inconspicuous nucleoli, and scanty mitotic figures (2 mitoses/10 HPF). What is the most likely diagnosis?
Choices: 1. 2. 3. 4.
Sclerosing adenosis Low-grade invasive carcinoma of no special type Tubular carcinoma Microglandular adenosis
Answer: 3 - Tubular carcinoma Explanations: The characteristic feature of tubular carcinoma is the predominance of open tubules composed of a single layer of epithelial cells enclosing a clear lumen. These should comprise greater than 90% of the tumor. The tubules are generally an admixture of oval or rounded and angulated shapes and are arranged haphazardly. There is a lack of consensus concerning the proportion of tubular structures necessary for a diagnosis of tubular carcinoma, with the requirement being set at between 75% and 100%, but 90% purity offers a practical solution and is recommended. Tumors exhibiting between 50% and 90% tubules admixed with another morphology should be regarded as being of mixed type. No specific clinical feature distinguishes tubular carcinoma from the more common invasive carcinomas of no special type or other types. The diagnosis of tubular carcinoma can only be established based on histopathological examination of the surgical specimen. The mammographic abnormality reported in the majority of patients with tubular carcinomas is a mass lesion with central density, occasionally with microcalcifications. The mass may appear round, oval, or lobulated, with irregular or spiculated margins. Tubular carcinomas cannot be distinguished reliably from invasive duct carcinomas and sometimes from radial scars on imaging studies, as these lesions show similar architectural patterns. Go to the next page if you knew the correct answer, or click the link image(s) below to further research the concepts in
this question (if desired).
Research Concepts: Tubular Breast Carcinoma
We update eBooks quarterly and Apps daily based on user feedback. Please tap flag to report any questions that need improvement.
Question 142:
A 48-year-old female presents complaining of intermittent right frontal headaches that have been progressively worsening over the past two months. She has a past medical history of breast cancer, for which she underwent radiation and chemotherapy. Upon further review, she admits she feels as if her eyesight has slightly worsened, and she has been falling more frequently, usually to her left. Her significant other admits they have noticed rapid mood swings, which is unusual for the patient. The physical exam reveals a body mass index of 26, blood pressure 125/84 mmHg, heart rate of 90 beats/min, and a respiratory rate of 16. The neurologic exam reveals that the cranial nerves, distal sensation, and distal motor are intact. The muscle tone of the right upper, left upper, and right lower extremities are +5/5. The muscle tone of the left lower extremity is +3/5. What is the most probable diagnosis?
Choices: 1. 2. 3. 4.
Vitamin B12 deficiency Metastatic brain tumor Long-term adverse effects of chemotherapy Idiopathic intracranial hypertension
Answer: 2 - Metastatic brain tumor Explanations: Breast cancer is a common cause of metastatic brain tumors. Symptomology of a frontal lobe brain tumor includes progressively worsening headaches, vision changes, and personality or mood changes. A frontal lobe brain tumor is one etiology of subfalcine herniation. The herniation causes the cingulate gyrus on one side under the falx cerebri to compress the ipsilateral anterior cerebral artery, lateral ventricles, and later the contralateral frontal lobe. When the anterior cerebral artery is compressed, contralateral lower extremity weakness develops. Go to the next page if you knew the correct answer, or click the link image(s) below to further research the concepts in this question (if desired).
Research Concepts: Subfalcine Herniation
We update eBooks quarterly and Apps daily based on user feedback. Please tap flag to report any questions that need improvement.
Question 143:
A 62-year-old female from near Chernobyl presents with complaints of a mass in her neck, which she noted while showering a few weeks ago. She has no symptoms and denies any recent infection, travel, or use of any medications. The exam reveals a painless mass that is hard and measures 3 x 2 cm. The mass appears to be fixed and moves with the trachea during swallowing. There is no adenopathy present. Blood work, including thyroid function studies, is normal. While waiting for imaging studies, a core biopsy shows papillary thyroid cancer. Which of the following is most likely correct regarding this particular patient?
Choices: 1. 2. 3. 4.
That the patient is much older in age There is a high rate of rearrangement of RET/PTC There are a high number of mutations in the BRAF gene She is more likely to fail primary radio-iodine treatment
Answer: 2 - There is a high rate of rearrangement of RET/PTC
Explanations: The histology reveals psammoma bodies. The history and physical suggest a thyroid malignancy, and the presence of psammoma bodies indicates papillary thyroid cancer. At least 70% of thyroid cancers are associated with some type of genetic rearrangement. Mutations in the BRAF gene are the most common, and this is also associated with aggressive cancer. These mutations occur in the absence of radiation exposure. In patients exposed to radiation, alterations in RET/PTC rearrangements are very common. Radiation-induced papillary thyroid cancer is known to occur at a younger age, usually in children and teenagers, and within a short time after radiation exposure. The risk of malignancy after radiation exposure is present for life and is much higher than expected in the general population. Go to the next page if you knew the correct answer, or click the link image(s) below to further research the concepts in this question (if desired).
Research Concepts: Papillary Thyroid Carcinoma
We update eBooks quarterly and Apps daily based on user feedback. Please tap flag to report any questions that need improvement.
Question 144:
A 65-year-old African American man comes for a follow-up appointment in the clinic. He has a two-year history of prostate cancer with a Gleason score of 4+3, status-post radical prostatectomy 18 months ago. He has a 5-year history of coronary artery disease, hyperlipidemia, obesity, and type 2 diabetes. He is currently complaining of lower back pain of two months duration, which is unrelieved by NSAIDs. A review of systems is negative except for a 10 pound (4.5 kg) weight loss within the last two months. Vitals signs are within normal limits. An MRI of his lumbar spine reveals a diffuse marrow infiltration replacing most of the L5 vertebra that shows mild enhancement on post-contrast T1 weighted images. A biopsy is arranged for a definitive diagnosis. Which of the following venous plexus allows for a direct spread of his prostate cancer to the vertebral bodies of his spine?
Choices: 1. The posterior spinal venous plexus 2. The radiculomedullary venous plexus 3. The superficial venous plexus of the extrinsic system 4. The epidural (internal vertebral) venous plexus (Batson Plexus)
Answer: 4 - The epidural (internal vertebral) venous plexus (Batson Plexus)
Explanations: The vertebral venous plexus (also called the Batson plexus) comprises the extradural venous system. It consists of the extradural veins within the spinal canal (called the internal vertebral venous plexus), the veins surrounding the vertebral column (called the external vertebral venous plexus), as well as the basivertebral veins that run through the vertebral bodies. The spinal epidural venous plexus is connected to various intracranial and pelvic venous plexus. It is specifically connected to pelvic veins via the ascending lumbar and azygos veins. Veins of the internal vertebral venous plexus do not contain valves. Retrograde flow is possible especially during times of increase intraabdominal and intrathoracic pressures which provide a route for prostate cancer to metastasize to the vertebral body. Go to the next page if you knew the correct answer, or click the link image(s) below to further research the concepts in this question (if desired).
Research Concepts: Neuroanatomy, Spinal Cord Veins
We update eBooks quarterly and Apps daily based on user feedback. Please tap flag to report any questions that need improvement.
Question 145:
A 71-year-old male presents with pleuritic chest pain and left shoulder pain. A palpable left anterior chest wall mass is noted on physical examination. Past medical history is significant for testicular cancer that was treated with radiation approximately five years prior to presentation. CT chest demonstrates a 6.5 cm lobulated soft tissue mass arising from the left anterior rib and involving the costal cartilage. An ultrasound-guided needle biopsy is obtained, and low-grade chondrosarcoma is diagnosed. What is the most important predictor for this disease to recur?
Choices: 1. 2. 3. 4.
History of previous malignancy Family history of malignancy Number of chest wall structures involved Histological grade
Answer: 4 - Histological grade Explanations: The histological grade is the single most important predictor of local recurrence and metastasis. Low-grade chondrosarcomas grow insidiously and rarely metastasize. The number of structures involved is a poor predictor of recurrence and metastasis. The 5-year survival of grade I chondrosarcomas is 83%. Go to the next page if you knew the correct answer, or click the link image(s) below to further research the concepts in this question (if desired).
Research Concepts: Chest Wall Tumors
We update eBooks quarterly and Apps daily based on user feedback. Please tap flag to report any questions that need improvement.
Question 146:
Lymphomatoid papulosis (LyP) is considered a low-grade variant of cutaneous T cell lymphoma. It can be associated with cutaneous or systemic lymphoma. Which of the following conditions is most significantly associated with LyP over the course of the disease?
Choices: 1. 2. 3. 4.
Hodgkin lymphoma Mycosis fungoides Mantel cell lymphoma Primary cutaneous large B-cell lymphoma, leg type
Answer: 2 - Mycosis fungoides Explanations: When associated with another systemic or cutaneous lymphoma, lymphomatoid papulosis (LyP) lesions show identical T-cell clones in LyP lesions and associated lymphoma lesions. LyP is most commonly associated with mycosis fungoides and anaplastic large cell lymphoma (both cutaneous and systemic). There are overlapping clinical and histologic features between LyP and cutaneous anaplastic large cell lymphoma. Clonally rearranged TCR genes are found in 60 to 70% of patients with LyP. LyP was originally regarded as a variant of Mucha– Habermann disease. This hypothesis is no longer considered valid. Go to the next page if you knew the correct answer, or click the link image(s) below to further research the concepts in this question (if desired).
Research Concepts: Lymphomatoid Papulosis
We update eBooks quarterly and Apps daily based on user feedback. Please tap flag to report any questions that need improvement.
Question 147:
A 36-year-old man with colon cancer was treated with a chemotherapy regimen consisting of leucovorin, fluorouracil, oxaliplatin, and a drug with side effects of impaired wound healing, hemorrhage, and gastrointestinal perforation. Which of the following drugs best fits this description?
Choices: 1. 2. 3. 4.
Irinotecan Panitumumab Bevacizumab Trastuzumab
Answer: 3 - Bevacizumab Explanations: Bevacizumab is administered as an intravenous (IV) infusion. U.S. boxed warnings of bevacizumab include gastrointestinal (GI) perforations. GI perforations, some fatal, have been seen in patients treated with bevacizumab. It should be discontinued in patients with GI perforations. Serious or fatal hemorrhage, including GI bleeding, central nervous system hemorrhage, vaginal bleeding, hemoptysis, and epistaxis, occurs up to five times more frequently in patients receiving bevacizumab. The incidence of wound healing and surgical complications, including both serious and fatal complications, is increased in patients treated with bevacizumab. It should not be given for the 28 days prior to an elective surgical procedure or for at least 28 days following surgery, or until the surgical wounds are completely healed. Bevacizumab is a recombinant humanized monoclonal antibody that binds to all known vascular endothelial growth factor A (VEGF-A) isoforms. No mutagenicity or carcinogenicity studies have been conducted. Administration of bevacizumab in monkeys showed adverse effects on general growth and development, fertility, and wound healing. Teratogenicity is suspected based on rabbit studies. Go to the next page if you knew the correct answer, or click the link image(s) below to further research the concepts in this question (if desired).
Research Concepts: Bevacizumab
We update eBooks quarterly and Apps daily based on user feedback. Please tap flag to report any questions that need improvement.
Question 148:
A 65-year-old female undergoes a partial mastectomy with sentinel lymph node biopsy for invasive carcinoma of the right breast. During the procedure, shave biopsies are taken of each margin. Upon return of pathology results, the lumpectomy specimen is found to be positive for tumor cells on the medial margin. However, the additional shave biopsy of the medial margin is negative for ink on tumor. What is the next best step in the management of this patient?
Choices: 1. Return to the operating room to achieve a negative medial margin of 1 cm 2. Return to the operating room to achieve a negative medial margin of 0.5 cm 3. Return to the operating room to achieve negative circumferential margins of 1 cm 4. No further operative intervention
Answer: 4 - No further operative intervention Explanations: Current NCCN guidelines define a negative margin for invasive breast carcinoma as "no ink on the tumor," found during pathologic processing. While the specimen pathology shows a positive medial margin, additional shave biopsy is negative for ink on tumor and therefore requires no further surgical intervention. Many surgeons will take additional shave biopsies of the specimen cavity if there is a concern for positive margins. Multiple studies have shown this decreases the margin positivity rate. Margins of 0.5 cm or 1 cm are not required. Go to the next page if you knew the correct answer, or click the link image(s) below to further research the concepts in this question (if desired).
Research Concepts: Breast Cancer Conservation Therapy
We update eBooks quarterly and Apps daily based on user feedback. Please tap flag to report any questions that need improvement.
Question 149:
A 56-year-old male with a history of smoking and metastatic melanoma comes to the clinic for shortness of breath. The patient started treatment with ipilimumab and nivolumab for melanoma two months ago and has received two cycles. He reports exertional dyspnea for 3 to 4 days with a non-productive cough. He denies any preceding fever. The oxygen saturation on room air was 93%. On examination, he is not in respiratory distress, and bilateral crackles were noted in lung bases. You obtain a stat CT chest without contrast that shows bilateral ground-glass opacities without focal consolidation. How should this patient be managed?
Choices: 1. 2. 3. 4.
Admit the patient to the hospital for atypical pneumonia Start outpatient broad-spectrum antibiotics Start prednisone at 1 mg/kg Refer to pulmonary for outpatient bronchoscopy
Answer: 3 - Start prednisone at 1 mg/kg Explanations: This patient likely has immune-mediated pneumonitis because he is afebrile, a CT scan shows ground-glass opacity without an infiltrate, and he received combination immunotherapy recently. Immunemediated pneumonitis is seen in about 3% of patients treated with single-agent anti-PD 1 agent such as nivolumab and about 6% of patients treated with a combination of nivolumab with ipilimumab. Prednisone 1mg/kg body weight is the initial drug of choice. Once clinical improvement is seen, prednisone can be tapered. Immunosuppressive agents such as mycophenolate have been shown to have benefits in refractory cases. Starting broad-spectrum antibiotics is not warranted since the workup is not suggestive of an infectious etiology. Routine bronchoscopy is unnecessary for diagnosing immune-mediated pneumonitis and can be used when the etiology is uncertain or if the patient does not improve with corticosteroids. Admitting this patient is not indicated since he appears to be oxygenating well. Go to the next page if you knew the correct answer, or click the link image(s) below to further research the concepts in this question (if desired).
Research Concepts: Metastatic Melanoma
We update eBooks quarterly and Apps daily based on user feedback. Please tap flag to report any questions that need improvement.
Question 150:
A 63-year-old female recently underwent a left mastectomy and axillary node dissection for breast cancer. The pathology report comes back stating that she has a 4 cm HER2-positive tumor and no axillary lymph node involvement. In addition to HER-targeted therapy, which of the following is the most appropriate treatment?
Choices: 1. 2. 3. 4.
Radiation Taxane-based regimens Tamoxifen Clomiphene
Answer: 2 - Taxane-based regimens Explanations: Having HER2-positivity usually signals a more aggressive breast cancer. These patients are usually treated with HER2-directed agents like trastuzumab. Taxane-based regimens are the chemotherapy backbone for the majority of HER-2/neu positive patients. Current research also shows that patients who are HER2-positive tend to respond favorably to anthracycline-based regimens. It is believed that there is co-amplification of topoisomerase with HER2. Go to the next page if you knew the correct answer, or click the link image(s) below to further research the concepts in this question (if desired).
Research Concepts: HER2
We update eBooks quarterly and Apps daily based on user feedback. Please tap flag to report any questions that need improvement.
Question 151:
A 55-year-old male presents with a 6month history of a mass growing in the right ankle. Pathological evaluation after surgical excision reveals polygonal fusiform cells with abundant cytoplasm and mitochondria. Karyotype analysis reveals a t(12;22)(q1314;q12) translocation. Based on the most likely diagnosis, which of the following immunohistochemical marker(s) will most likely be positive?
Choices: 1. 2. 3. 4.
Desmin Cytokeratin Epithelial membrane antigen (EMA) HMB-45 and Melan-A
Answer: 4 - HMB-45 and Melan-A Explanations: Immunohistochemistry for clear-cell soft tissue sarcoma (CCSST) commonly shows positivity for HMB-45, MelanA, and S-100. Histologic examination may show abundant and clear cytoplasm with numerous mitochondria and a centrally located round nucleus. Other soft-tissue sarcomas also may show positive immunohistochemistry for HMB-45 and Melan-A, such as angiomyolipomas. Cytoplasm in CCSST stains positive for periodic acid– Schiff and represents intracellular glycogen accumulation. Anaplastic characteristics, such as marked pleomorphism with extremely hyperchromatic nuclei and a nuclear-cytoplasmic ratio approaching 1:1, are non-specific and commonly seen in aggressive malignancies. Go to the next page if you knew the correct answer, or click the link image(s) below to further research the concepts in this question (if desired).
Research Concepts: Soft Tissue Clear Cell Sarcoma
We update eBooks quarterly and Apps daily based on user feedback. Please tap flag to report any questions that
need improvement.
Question 152:
A 61-year-old man is being evaluated in the surgical oncology clinic for a recent diagnosis of colon cancer, identified during a screening colonoscopy. The lesion is located in the sigmoid colon and is biopsy-proven to be well-differentiated adenocarcinoma. The patient is asymptomatic with normal vital signs and no anemia noted on a recent complete blood count evaluation. He is being planned for laparoscopic sigmoidectomy, and a preoperative carcinoembryonic antigen (CEA) level is drawn. It came back elevated at 6.2 ng/L. Staging CT scans return negative for metastatic disease. Which of the following is not true in regards to CEA?
Choices: 1. Well-differentiated tumors secrete more CEA 2. CEA is more elevated in organ-confined tumors than in those involving lymph nodes and distant metastasis 3. Tumors that cause bowel obstruction are associated with higher CEA levels 4. CEA is not to be used as a screening test for colon cancer
Answer: 2 - CEA is more elevated in organ-confined tumors than in those involving lymph nodes and distant metastasis
Explanations: CEA is not reliable for diagnosing cancer nor as a screening test for early detection of cancers. Serum from individuals with colorectal carcinoma often has high levels of CEA. CEA measurement is used as a tumor marker to monitor response to treatment, identify recurrences, or diagnose cancer spread by evaluating biological fluids. CEA levels may be raised in the gastric, pancreatic, lung, breast, and medullary thyroid carcinomas and in ulcerative colitis, pancreatitis, cirrhosis, COPD, Crohn's, hypothyroidism, and smokers. Elevated CEA levels usually return to normal after successful surgical removal of the tumor. Well-differentiated tumors secrete more CEA. CEA is more elevated in tumors with lymph node and distant metastasis than organ-confined tumors. Well-differentiated colon tumors secrete more CEA than poorly differentiated tumors. Left-sided tumors tend to have higher CEA levels than right-sided tumors. Tumors that cause bowel obstruction produce higher CEA levels. Go to the next page if you knew the correct answer, or click the link image(s) below to further research the concepts in this question (if desired).
Research Concepts: Carcinoembryonic Antigen
We update eBooks quarterly and Apps daily based on user feedback. Please tap flag to report any questions that need improvement.
Question 153:
A 61-year-old patient presents to the clinic due to pain and swelling along the right posterior mandibular ridge. The patient is having prostate cancer and is taking bisphosphonates for the last 2 years. CT scan shows osseous changes, and the patient is diagnosed with bisphosphonate-associated osteonecrosis of the jaw. MRI is advised to evaluate the affected region further. Which of the following are most likely MRI findings?
Choices: 1. Non-enhancement in the region of ischemia, high signal intensity in the areas of fibrosis and sclerosis on T-1 weighted images and low signal intensity along the unexposed diseased bone 2. Enhancement in the region of ischemia, high signal intensity in the areas of fibrosis and sclerosis on T-2 weighted images and low signal intensity along the unexposed diseased bone 3. Non-enhancement in the region of ischemia, low signal intensity in the areas of fibrosis and sclerosis on both T-1 and T-2 weighted images and high signal intensity along the unexposed diseased bone 4. Enhancement in the region of ischemia, low signal intensity in the areas of fibrosis and sclerosis on both T-1 and T-2 weighted images and high signal intensity along the unexposed diseased bone
Answer: 3 - Non-enhancement in the region of ischemia, low signal intensity in the areas of fibrosis and sclerosis on both T-1 and T-2 weighted images and high signal intensity along the unexposed diseased bone
Explanations: Signal intensity varies with bone vascularity. Areas of ischemia, fibrosis, and sclerosis have low signal intensity due to compromised vascularity. MRI currently may provide as the method of choice to detect the early bone marrow and soft tissue changes surrounding the osteonecrotic area. Osseous change evaluation by MRI is similar to CT imaging. Go to the next page if you knew the correct answer, or click the link image(s) below to further research the concepts in this question (if desired).
Research Concepts: Bisphosphonate Related Jaw Osteonecrosis
We update eBooks quarterly and Apps daily based on user feedback. Please tap flag to report any questions that need improvement.
Question 154:
A 44-year-old female underwent radical mastectomy for breast cancer. Histological examination of the surgical specimen reveals a tumor proliferation composed of more than 90% of angulated tubules lined by a single layer of epithelial cells. There is little nuclear pleomorphism, inconspicuous nucleoli, and scanty mitotic figures (1 mitosis/ 10 HPF). What is the histological grade of this tumor according to the Nottingham Modification of the Bloom-Richardson system, if applicable?
Choices: 1. 2. 3. 4.
Grade I Grade II Grade III Grading is not applicable to this subtype of tumor
Answer: 1 - Grade I Explanations: A total score is obtained by scores on tubule formation, nuclear pleomorphism, and mitotic count. The total score of this tumor is 1+1+1=3, which corresponds to grade I. A total score of 3–5 corresponds to grade I. A total score of 6–7 corresponds to grade II. A total score of 8–9 corresponds to grade III. Tubule formation: 1 point. Tubule formation constitutes more than 75% of the tumor. In this case, tumor proliferation was composed of more than 90% of tubules. Nuclear pleomorphism: 1 point. Nucleus shape and size difference is mild. In this case, there was little nuclear pleomorphism. Mitotic count: 1 point. In this case, there were very few mitotic figures: 1 mitosis/10 HPF. Go to the next page if you knew the correct answer, or click the link image(s) below to further research the concepts in this question (if desired).
Research Concepts: Tubular Breast Carcinoma
We update eBooks quarterly and Apps daily based on user feedback. Please tap flag to report any questions that need improvement.
Question 155:
A 55-year-old man with no significant past medical history presents with acute onset right epigastric pain in the setting of polyuria, polydipsia, and foul-smelling loose stools for 2 months. His abdomen exam is benign. His initial lab work shows random glucose of 350 mg/dL, AST of 140 IU/L, ALT of 120 IU/L, and a normal pancreatic lipase level. Abdominal ultrasound shows enlargement of gall bladder with gallstones and abnormal echogenicity in the liver, which were subsequently found to be multiple hyperintense lesions in CT abdomen. What is the most likely diagnosis?
Choices: 1. 2. 3. 4.
Pancreatic adenocarcinoma Somatostatinoma VIPoma Gastrinoma
Answer: 2 - Somatostatinoma Explanations: Somatostatinomas located in the pancreas are usually secretory and present with various non-specific systemic symptoms due to the inhibitory effects of somatostatin on a wide range of hormones, including insulin, pancreatic digestive enzymes, and cholecystokinin. In rare instances, somatostatinomas present as a classic triad of diabetes mellitus, cholelithiasis, and diarrhea/steatorrhea referred to as an inhibitory syndrome. Around 75% of cases are metastatic at diagnosis, and the liver is one of the commonest sites of metastasis due to the portal circulation. The most common age of presentation for Somatostatinoma is between 40 and 60 years. Go to the next page if you knew the correct answer, or click the link image(s) below to further research the concepts in this question (if desired).
Research Concepts: Somatostatinoma
We update eBooks quarterly and Apps daily based on user feedback. Please tap flag to report any questions that need improvement.
Question 156:
A 72-year-old woman presents following a mammogram showing a lesion suspicious for malignancy. A tissue biopsy is recommended for a definitive diagnosis. What is the most appropriate laboratory test to determine if she would be a candidate for trastuzumab therapy at some point in the future?
Choices: 1. 2. 3. 4.
Karyotype analysis DNA sequencing Fluorescent in situ hybridization (FISH) Flow cytometry
Answer: 3 - Fluorescent in situ hybridization (FISH) Explanations: Trastuzumab is a monoclonal antibody that targets the RTK ErbB2 (HER2). It works by preventing HER2 homo and heterodimerization. It is indicated in the adjuvant setting in localized HER2+ breast cancer, as well as in metastatic HER2+ breast cancer. Overexpression of HER2 in breast cancer results from gene amplification. FISH is the preferred laboratory test for detecting gene amplification. Strongly HER2+ breast cancers typically have six or more copies of the HER2 gene/nucleus or a HER2/CEP17 ratio>2.2. HER2 is located on chromosome 17, and the ratio adjusts for cancer cells that may have more than one chromosome 17 but no gene amplification. Immunohistochemistry can also detect HER2+ breast cancers. It does not detect gene amplifications like FISH but rather the presence of HER2 on the cell surface. Karyotype analysis is able to detect gene translocations. DNA sequencing is more useful in detecting point mutations or small gene insertions and deletions. Flow cytometry is most commonly used in hematologic malignancies to detect populations of monoclonal cells. Go to the next page if you knew the correct answer, or click the link image(s) below to further research the concepts in this question (if desired).
Research Concepts: Physiology, Tyrosine Kinase Receptors
We update eBooks quarterly and Apps daily based on user feedback. Please tap flag to report any questions that need improvement.
Question 157:
A 42-year-old woman presents to the clinic to assess options for her cancer-related pain. The patient currently works as a museum curator and has a history of von Willebrand disease, hypertension, diabetes mellitus, liver failure with severe coagulopathy, and metastatic breast cancer. Her vital signs are all within normal reference ranges. She rates her pain as 6 out of 10 in her pelvis with a continuous infusion of opioids and oral gabapentin, and the pain in her right leg is making it difficult for her to work. She is interested in definitive, permanent options to palliate her pain. Which of the following is the most appropriate approach to this patient?
Choices: 1. Open bilateral cordotomy; if hypertension can be controlled, proceed with the procedure as the patient wants definitive palliation of her pain 2. Right unilateral percutaneous cervical cordotomy; even if blood glucose cannot be adequately controlled, proceed with the procedure to target the patient's leg pain 3. Midline myelotomy; if the patient's pain remains at level 3 for 2-weeks, proceed with the procedure to address the patient's pelvic pain 4. The patient is not a surgical candidate for the intervention which would meet her needs
Answer: 4 - The patient is not a surgical candidate for the intervention which would meet her needs
Explanations: Cordotomy leads to the nociceptive pathways in the lateral spinothalamic tract anterolateral column being destroyed in the cervical spinal cord at the level of C1C2. Percutaneous cordotomy is contraindicated in patients with a coagulation disorder, such as patients with liver failure. Patients must be at level 3 of pain on the World Health Organization (WHO) cancer ladder with a prognosis of under six months to be a candidate for cordotomy. In otherwise healthy patients without contraindications, the risk of severe complications with unilateral cordotomy is low. Go to the next page if you knew the correct answer, or click the link image(s) below to further research the concepts in this question (if desired).
Research Concepts: Cordotomy
We update eBooks quarterly and Apps daily based on user feedback. Please tap flag to report any questions that need improvement.
Question 158:
A 65-year-old woman with a history of Hashimoto thyroiditis presents with worsening dyspnea. Examination is notable for a significantly enlarged thyroid gland compared to the last office visit two months ago, and it is now firm and fixed to the surrounding structures. Diffuse large B-cell lymphoma (DLBCL) confined to the thyroid gland, stage I E, is diagnosed after all appropriate investigations are completed. What is the most appropriate treatment?
Choices: 1. Radiation therapy only 2. Chemotherapy followed by involved site radiation therapy (ISRT) 3. Combined modality treatment (CMT) with chemotherapy and radiotherapy 4. Surgery
Answer: 3 - Combined modality treatment (CMT) with chemotherapy and radiotherapy
Explanations: Combined modality treatment (CMT) with chemotherapy and radiotherapy is preferred for stage I E and II E diffuse large B-cell thyroid lymphoma (DLBCL). The chemotherapy regimen used is rituximabcyclophosphamide, doxorubicin, vincristine, and prednisolone (R-CHOP). After three cycles of R-CHOP, based on the response noted on interim PET/CT, patients can undergo one additional cycle of R-CHOP or involved site radiation therapy (ISRT). Advanced stage (III E and IV) DLBCL cannot be treated within one irradiation field; hence, chemotherapy alone is used. Surgery is not recommended, except for diagnostic biopsy, due to the potential surgical risks and no additional benefit compared to chemotherapy +/radiotherapy. Obstructive symptoms like dyspnea usually improve within days of R-CHOP initiation due to the steroid (prednisolone) in the regimen. Go to the next page if you knew the correct answer, or click the link image(s) below to further research the concepts in this question (if desired).
Research Concepts: Thyroid Lymphoma
We update eBooks quarterly and Apps daily based on user feedback. Please tap flag to report any questions that need improvement.
Question 159:
A 52-year-old male with a past medical history significant for hypertension, colon carcinoma status post colectomy, insulin dependent diabetes mellitus, and stage II chronic kidney disease presents to the outpatient clinic for evaluation of a “bump” on his cheek. The lesion is biopsied, and the pathology results reveal a sebaceous adenoma. What is the next best step in management?
Choices: 1. Reassure the patient that this is a benign tumor 2. Request genetic testing on the specimen for mismatch repair genes 3. Recommend further surgical excision of the biopsy site 4. Recommend local radiation to the biopsy site
Answer: 2 - Request genetic testing on the specimen for mismatch repair genes
Explanations: Sebaceous adenomas are benign tumors, but can sometimes be a clue to underlying Muir-Torre syndrome. Testing for mismatch repair genes should be performed when a sebaceous adenoma is discovered with biopsy. Positive testing for mismatch repair genes in a sebaceous adenoma does not definitively diagnose MuirTorre syndrome as there are several reports describing mismatch repair gene defects in sebaceous adenomas in patients without the syndrome. If there is a positive test for mismatch repair gene defects in a sebaceous adenoma, the patient should be sent for genetic testing. Go to the next page if you knew the correct answer, or click the link image(s) below to further research the concepts in this question (if desired).
Research Concepts: Muir-Torre Syndrome
We update eBooks quarterly and Apps daily based on user feedback. Please tap flag to report any questions that need improvement.
Question 160:
A patient with bone cancer is being treated with a long-acting morphine formula. He presents with vague complaints of abdominal distension, nausea, hard stools, and straining. The working diagnosis is opioidinduced constipation. The Bristol stool chart is used to characterize his constipation. The chart uses which criteria to assess constipation?
Choices: 1. 2. 3. 4.
Stool Stool Stool Stool
size color form weight
Answer: 3 - Stool form Explanations: The Bristol stool form scale, or Bristol stool chart, can be used to classify the form of feces into seven categories. Type 1: sperate hard lumps, like nuts (hard to pass). Type 2: sausage-shaped but lumpy. Type 1 indicates severe constipation. Type 2 indicates mild constipation. Type 3: like a sausage but with cracks on its surface. chart 4: like a sausage or snake, smooth and soft. Types 3 and 4 indicate normal stools. Type 5: soft blobs with clear-cut edges (passed easily). Type 6: fluffy pieces with ragged edges, a mushy stool. Type 7: water, no solid pieces, entirely liquid. Type 5 indicates a lack of fiber in the diet. Type 6 indicates mild diarrhea. Type 7 indicates severe diarrhea. Go to the next page if you knew the correct answer, or click the link image(s) below to further research the concepts in this question (if desired).
Research Concepts: Opioid Induced Constipation
We update eBooks quarterly and Apps daily based on user feedback. Please tap flag to report any questions that need improvement.
Question 161:
A 45-year-old black male computer programmer presents to the clinic with a nonpainful black streak within the nail of his left index finger. He denies any significant medical history and takes no medications other than an occasional aspirin. He denies a history of skin cancer in himself or his family. His vital signs are within normal limits. He specifically denies any trauma to his fingers. The black streak was first noticed last year and has gotten longer and wider. The solitary band of pigmentation measures 4 mm at its proximal base and narrows to 2 mm wide at the distal free edge. Dermoscopy examination reveals multiple parallel pigmented bands of varying widths that are gray to black in color. No globules are seen. Which of the following is the best course of action?
Choices: 1. Take a nail clip biopsy of the distal nail plate for fungal culture 2. Photograph the lesion and plan follow up examination in six months 3. Subungual nail matrix biopsy of the proximal portion of the streak 4. Plan nail avulsion and shave biopsy of the distal 2/3 of the nail bed streak
Answer: 3 - Subungual nail matrix biopsy of the proximal portion of the streak
Explanations: Single streaks carry a higher risk of melanoma than multiple bilateral streaks. Benign subungual pigmented streaks typically do not leak pigment into the proximal nail fold and maintain less than a 3 mm width throughout their length. Subungual hematomas are more likely to be transverse rather than longitudinal. Longitudinal pigmented nail streaks wider than 3 mm are much more likely to represent subungual melanoma. Go to the next page if you knew the correct answer, or click the link image(s) below to further research the concepts in this question (if desired).
Research Concepts: Melanocytic Nevi
We update eBooks quarterly and Apps daily based on user feedback. Please tap flag to report any questions that need improvement.
Question 162:
A 70-year-old male with a past medical history significant for hypertension, diabetes mellitus type 2, chronic kidney disease stage IV, deep vein thrombosis (DVT), and colon cancer is admitted for an elective, low anterior resection, and colostomy placement. What is the choice and duration of medication for DVT prophylaxis?
Choices: 1. Low molecular weight heparin (LMWH) at a reduced dose for 2-3 weeks after discharge 2. Unfractionated heparin (UFH) until discharge from the hospital 3. Warfarin for 2 weeks 4. UFH for 4 weeks
Answer: 1 - Low molecular weight heparin (LMWH) at a reduced dose for 2-3 weeks after discharge
Explanations: Advanced age, prior history of deep vein thrombosis (DVT), and active cancer make this patient a high-risk candidate for venous thromboembolism postoperatively. Warfarin typically is not used for DVT prophylaxis unless low molecular weight heparin (LMWH) and unfractionated heparin (UFH) are contraindicated. UFH can be used instead of LMWH in patients with renal impairment. LMWH is used at a reduced dose in patients with a glomerular filtration rate of less than 30. Patients with major cancer surgery may benefit from posthospital prophylaxis with LMWH. The level of evidence is II, and the grade of recommendation is C. Although the optimum duration of VTE prophylaxis is unknown, most DVT occurs within the first week or two following the surgery. However, VTE complications can occur beyond that time. Collectively, in cancer-surgery patients, continued prophylaxis for two to three weeks after discharge reduces the incidence of asymptomatic DVT. Go to the next page if you knew the correct answer, or click the link image(s) below to further research the concepts in this question (if desired).
Research Concepts: Deep Venous Thrombosis Prophylaxis
We update eBooks quarterly and Apps daily based on user feedback. Please tap flag to report any questions that need improvement.
Question 163:
A 35-year-old female with multiple insulinomas followed up with endocrinology after surgical resection and was found to be cured (free of disease on repeat imaging and no recurrence of symptoms). When should this patient return to the endocrinology office?
Choices: 1. The patient does not need to follow up with endocrinology anymore 2. In 6 months 3. In 1 year 4. In 2 years
Answer: 3 - In 1 year Explanations: Insulinoma patients without MEN1 should have followups at 3 to 6 months post-resection. Biochemical testing and imaging studies should be repeated at follow-ups. If cured, patients can be followed up as needed if there is a recurrence of symptoms. Patients with persistent symptoms after surgery might have multiple tumors or MEN1 syndrome and should get repeat imaging studies. Patients with multiple insulinomas or MEN1 should be followed yearly even if they continue to be cured postresection. Go to the next page if you knew the correct answer, or click the link image(s) below to further research the concepts in this question (if desired).
Research Concepts: Insulinoma
We update eBooks quarterly and Apps daily based on user feedback. Please tap flag to report any questions that need improvement.
Question 164:
A 42-year-old female with a history of hypertension presents to the clinic with persistent epigastric discomfort for three months. Physical examination is unremarkable. Abdominal ultrasonography (US) reports a heterogeneous semisolid mass with cystic components. Abdominal computerized tomography (CT) reveals a welldefined and heterogeneously circumscribed 4 x 3 x 2.5 cm mass in the tail of pancreas without vascular involvement. Based on imaging characteristics, solid papillary cystic tumor (SPT) is considered to be the preliminary diagnosis. This patient then undergoes surgical resection. Which of the following histopathological characteristics confirms the diagnosis of SPT?
Choices: 1. Solid nest of cohesive cells in pseudopapillary pattern with evidence of degeneration in the form of hemorrhage and calcifications 2. Increased mitotic figures with non-eosinophilic cytoplasm 3. KI 67 staining showing greater than 70 percent of cells positively stained 4. Lymphovascular and perineural invasion are characteristic findings of papillary cystic tumors of the pancreas
Answer: 1 - Solid nest of cohesive cells in pseudopapillary pattern with evidence of degeneration in the form of hemorrhage and calcifications
Explanations: Solid papillary cystic tumors of the pancreas or SPENs are benign tumors. The microscopic description includes solid nests of cells surrounding blood vessels, resulting in a pseudopapillary or trabecular pattern. Associated with findings of degeneration such as hemorrhage, calcification and cholesterol clefts. Tumor cells of SPENs usually have a moderate amount of eosinophilic cytoplasm with large intracytoplasmic hyaline globules and perinuclear vacuoles. SPEN's are indolent tumors and KI 67 is very low. Lymphovascular and perineural invasion are characteristic findings of malignant neoplasm but not SPENs of the pancreas. Go to the next page if you knew the correct answer, or click the link image(s) below to further research the concepts in this question (if desired).
Research Concepts: Solid and Papillary Epithelial Neoplasm
We update eBooks quarterly and Apps daily based on user feedback. Please tap flag to report any questions that need improvement.
Question 165:
A 46-year-old male with a history of small cell lung cancer undergoing chemotherapy admitted to the hospital with unexplained muscle twitches, paresthesias, hallucinations, insomnia, excessive sweating, excessive thirst, and palpitations. The patient was found to have a stocking-type sensory loss, loss of deep tendon reflexes and memory loss. Initial lab work only remarkable for hyponatremia. CT/MRI brain was normal. EMG showed bursts of doublet, triplet, and multiplet continuous muscle fasciculations consistent with neuromyotonic discharges. What is the next best step in the diagnosis?
Choices: 1. 2. 3. 4.
Check CASPR2 and LGI1 antibody titers EEG Lumbar puncture Check acetylcholine receptor antibody
Answer: 1 - Check CASPR2 and LGI1 antibody titers Explanations: This patient is most likely suffering from Morvan syndrome which is a male predominant condition often associated with malignancies such as small cell lung cancer. There is now reasonable evidence that Morvan’s syndrome is an autoimmune phenomenon caused by autoantibodies to proteins such as CASPR2 (contactinassociated protein 2) and LGI1 (leucine-rich glioma inactivated 1) found in voltage-gated potassium channel (VGKC) complexes. Checking the level of CASPR2 and LGI1 antibodies would help establish the diagnosis. This patient did not have seizures. An EEG in Morvan syndrome would either be normal or would show diffuse nonspecific slowing. On the other hand, EEG would be helpful, had the patient had seizures of clinical features on limbic encephalitis. The MRI of the brain also would show temporal lobe structural abnormalities. Lumbar puncture in Morvan syndrome is typically unyielding. It would usually show normal finding or sometimes oligoclonal bands which would not establish a definite diagnosis in this patient. Checking acetylcholine rector antibodies would be helpful if myasthenia is suspected. However, this patient does not have a history or clinical features suggestive of myasthenia gravis. Myasthenia gravis is a neuromuscular disease that causes varying degrees of skeletal muscle weakness involving the muscles of the eyes, face, and swallowing resulting in diplopia, ptosis, dysarthria.
Go to the next page if you knew the correct answer, or click the link image(s) below to further research the concepts in this question (if desired).
Research Concepts: Morvan Syndrome
We update eBooks quarterly and Apps daily based on user feedback. Please tap flag to report any questions that need improvement.
Question 166:
A 16-year-old male presents with pain in his right knee for one month. He has no significant past medical history and takes no medication. His vital signs are within normal limits. On examination, he has tenderness over the right proximal tibia. Blood tests are unremarkable. An MRI reveals a T2/STIR high signal lesion centered within the proximal epiphysis of the right proximal tibia. The patient undergoes extended curettage and bone grafting, and recovery is uneventful. What is the most likely complication associated with the treatment of this patient's lesion?
Choices: 1. Malignant transformation of the primary tumor at the local site 2. Metastasis to the lungs 3. Recurrence of the tumor 4. Pathologic fracture
Answer: 3 - Recurrence of the tumor Explanations: The clinical vignette most likely represents chondroblastoma in the right proximal tibia. The most common complication associated with chondroblastoma is local recurrence after surgery. The risk of recurrence is 10-30%. The epiphyseal location of the tumor increases the risk of recurrence significantly. Long-term clinical and radiologic follow-up of patients is recommended after surgery to evaluate for local recurrence or detect malignant transformation. Malignant transformation of chondroblastoma is very rare and occurs in less than 2% of patients. Pathologic fracture and metastases are not commonly seen in chondroblastoma. Go to the next page if you knew the correct answer, or click the link image(s) below to further research the concepts in this question (if desired).
Research Concepts: Chondroblastoma
We update eBooks quarterly and Apps daily based on user feedback. Please tap flag to report any questions that need improvement.
Question 167:
A 65-year-old female presents to the outpatient department with a large erythematous area on her left arm. Her past medical and surgical history is significant for left breast cancer and radical mastectomy fifteen years ago and diabetes. Vital signs show pulse rate 70/min, blood pressure 128/78 mmHg, respiratory rate 16/min, and the temperature is 98.6 F (37 C). Physical examination reveals a raised purple-red papule on the left arm. Which of the following is most likely to be another finding in this patient?
Choices: 1. 2. 3. 4.
Varicose veins Lymphedema Basal cells Squamous cells
Answer: 2 - Lymphedema Explanations: Stewart-Treves syndrome is a rare and deadly skin angiosarcoma. The syndrome often develops in long-standing chronic lymphedema. Women who have had a radical mastectomy and develop lymphedema may develop this syndrome. Lymphangiosarcomas are very aggressive cancers and have a high recurrence rate. Long-term survival is rare. Go to the next page if you knew the correct answer, or click the link image(s) below to further research the concepts in this question (if desired).
Research Concepts: Stewart-Treves Syndrome
We update eBooks quarterly and Apps daily based on user feedback. Please tap flag to report any questions that need improvement.
Question 168:
A 75-year-old female presents for follow up for multiple episodes of pneumonia. The patient has had a history of non-small cell lung cancer that has been treated with chemotherapy and radiation 15 years ago. Her only complication of treatment at that time was nausea and radiation esophagitis. What would be the next best step in her workup for her pneumonia?
Choices: 1. 2. 3. 4.
Start prophylactic antibiotics Chest x-ray CT scan of the chest and neck Monitor closely, no intervention at this time
Answer: 3 - CT scan of the chest and neck Explanations: Rare complications of radiation esophagitis are tracheoesophageal fistula, stricture, ulceration, and perforations. Treatment is usually supportive measure with diet alteration, nutrition supplementation, prophylactic antifungal therapy, and acid suppression. Risk factors for developing radiation esophagitis include age greater than 70, female sex, white race, low BMI, low initial performance status, gastroesophageal reflux, and current dysphagia. Radiation esophagitis typically presents two to three weeks after initial treatment. Go to the next page if you knew the correct answer, or click the link image(s) below to further research the concepts in this question (if desired).
Research Concepts: Radiation Esophagitis
We update eBooks quarterly and Apps daily based on user feedback. Please tap flag to report any questions that need improvement.
Question 169:
A 68-year old woman being treated for cervical cancer is referred to the hyperbaric oxygen treatment facility by her oncologist. She has hemorrhagic cystitis secondary to radiation treatment and her oncologist feels that hyperbaric oxygen would facilitate healing. Part of her current treatment strategy includes doxorubicin. How soon can she start hyperbaric oxygen therapy?
Choices: 1. Immediately 2. She must wait at least 3 days from her last dose of doxorubicin 3. She must wait 6 months 4. She is no longer a candidate for hyperbaric oxygen therapy due to doxorubicin use
Answer: 2 - She must wait at least 3 days from her last dose of doxorubicin
Explanations: Doxorubicin has a relatively short half-life of about 20 to 48 hours. Current recommendations state that hyperbaric therapy may be used 3 days after the last dose of doxorubicin. Concomitant doxorubicin and hyperbaric oxygen therapy have been shown to increase mortality in rat models. This synergy has not been studied in humans. Out of caution, although not proven to be harmful in humans, the combination of hyperbaric oxygen and doxorubicin is to be avoided. Go to the next page if you knew the correct answer, or click the link image(s) below to further research the concepts in this question (if desired).
Research Concepts: Hyperbaric Contraindicated Chemotherapeutic Agents
We update eBooks quarterly and Apps daily based on user feedback. Please tap flag to report any questions that need improvement.
Question 170:
What skin lesion is relatively specific for Cowden syndrome, especially when multiple lesions are present?
Choices: 1. 2. 3. 4.
Sclerotic fibromas Genital lentigines Fibrofolliculomas Neurofibromas
Answer: 1 - Sclerotic fibromas Explanations: Sclerotic fibromas are a relatively specific finding for Cowden syndrome. Genital lentigines are seen in Bannayan-Riley-Ruvalcaba syndrome. It is another PTEN gene mutation syndrome that shares overlapping features with Cowden syndrome such as cancers of breast, thyroid, or endometrium as well as intestinal polyps. Fibrofolliculomas are seen in Birt-Hogg-Dube syndrome. However, more than 80% of patients with Cowden syndrome will develop skin involvement early on, and usually, lesions on the skin or mucous membranes can be seen by the second to third decade of life. Angiofibromas are seen in MEN1 or tuberous sclerosis. Multiple extra-mucocutaneous manifestations can also occur in Cowden disease. The skeletal system may form a high-arched palate, scoliosis, or macrocephaly. More than 85% of patients may have gastrointestinal involvement with hamartomatous polyps. Go to the next page if you knew the correct answer, or click the link image(s) below to further research the concepts in this question (if desired).
Research Concepts: Cowden Disease
We update eBooks quarterly and Apps daily based on user feedback. Please tap flag to report any questions that need improvement.
Question 171:
A 57-year-old female underwent a radical mastectomy for invasive lobular carcinoma. Histological examination of the surgical specimen revealed a tumor proliferation composed of single-file linear cords that invade the stroma without glandular formation. There is moderate nuclear pleomorphism and scanty mitotic figures (3 mitoses/ 10 high power fields). What is the histological grade of this tumor according to the Nottingham Modification of the Bloom-Richardson system, if applicable?
Choices: 1. Grade I 2. Grade II 3. Grade III 4. Grading is not applicable to this histological type of breast carcinoma
Answer: 2 - Grade II Explanations: In this case, there is no tubule formation (score 3), there is moderate nuclear pleomorphism (score 2), the mitotic count is 3 mitoses/ 10 HPF (score 1). The sum is : 3 + 2 + 1 = 6 which corresponds to grade II (moderately differentiated). Because the mitotic rate is low, except for some pleomorphic invasive lobular carcinomas, few invasive lobular carcinomas are histologic grade III. Histologic grading of invasive lobular carcinomas according to the Nottingham system is recommended, although it is dependent on the nuclear grade. The Nottingham combined histologic grade, also called the modified Scarff-Bloom-Richardson grading system, is recommended for the grading of invasive lobular carcinoma. This grade reflects how rapidly the tumor proliferates and how far the tumor architecture and cytology deviate from normal. Go to the next page if you knew the correct answer, or click the link image(s) below to further research the concepts in this question (if desired).
Research Concepts: Lobular Breast Carcinoma
We update eBooks quarterly and Apps daily based on user feedback. Please tap flag to report any questions that need improvement.
Question 172:
A 49-year-old female notices a red, scaly rash on her left breast, which has been present for the past eight weeks. She denies any symptoms and reports applying a hydrocortisone cream. The exam reveals no mass. However, a biopsy reveals large, clear cells at the dermal-epidermal junction. The mucin staining is positive. Which of the following is true?
Choices: 1. 2. 3. 4.
Similar condition may be located on the vulva or penis Lesions are usually not painful Lesions usually ulcerate Rarely associated with ductal carcinoma
Answer: 1 - Similar condition may be located on the vulva or penis
Explanations: Paget disease of the breast usually appears as an eczematous rash, but it is a cancerous lesion. Extramammary Paget’s disease (extramammary Paget disease) is a rare, slow-growing, noninvasive intraepithelial adenocarcinoma outside the mammary gland and it includes Paget's disease of the vulva and penis. Patients typically complain of itching, burning, and soreness. Presence of vulvar pain, bleeding, and tumor formation is reported to be more common in invasive disease. Skin lesions are often mistaken as eczema, may be itchy or painful. Vulvar Paget disease presents as a velvety, soft, and red or pink with scattered white islands of hyperkeratosis (strawberry and cream appearance). The lesions become erythematous, plaque-like, and desquamating and rarely ulcerate. The borders are irregular, elevated, and demarcated. The visible borders of vulvar Paget disease are misleading as Paget cells may spread along the basal layers of normal-appearing skin with multicentric foci. Involvement may be extensive including the perianal region, genitocrural, and inguinal folds. The histology of a vulvar lesion is the same as for Paget's disease of the breast. Paget's disease of the vulva is rare and may be a primary lesion or associated with adenocarcinoma from local organs such as the Bartholin gland, urethra, or rectum. Patients tend to be postmenopausal. Paget's disease of the penis may be primary or secondary, and is even rarer than vulvar Paget’s
disease. Primary disease is treated with surgical excision. Extramammary Paget's disease is associated with an underlying invasive malignancy about 12% of the time. It is associated with an underlying adnexal malignancy about 24% of the time. Paget's disease of the breast is highly associated with mammary ductal carcinoma. Go to the next page if you knew the correct answer, or click the link image(s) below to further research the concepts in this question (if desired).
Research Concepts: Mammary Paget Disease
We update eBooks quarterly and Apps daily based on user feedback. Please tap flag to report any questions that need improvement.
Question 173:
A 59-year-old postmenopausal woman presents to the hospital with a four weeks history of shortness of breath and right-sided pleuritic chest pain. Associated symptoms are mild cough, orthopnea, and pedal edema. The patient denies fever, chills, abdominal pain, or nausea. She has a 40-pack-year history of smoking. On examination, she looks cachectic, with diminished rightsided breath sounds. Mild abdominal distention is noted. On bimanual examination, a left-sided pelvic adnexal mass is palpated. Laboratory tests show normocytic anemia with a hemoglobin of 10 gm/dL. The white cell count is 10,000 cells per microliter. A complete metabolic panel shows bilirubin 0.8 mg/dL, alkaline phosphatase (ALP) 55 IU/L, aspartate transaminase (AST) 12 IU/L, and alanine transaminase (ALT) 20 IU/L. Serum creatinine is 0.8 mg/dL. A chest X-ray shows a large right-sided pleural effusion. What is the appropriate next step in evaluating this patient?
Choices: 1. 2. 3. 4.
Whole-body PET CT scan Serum CA 125 Levels Echocardiogram CT scan of the abdomen and pelvis
Answer: 4 - CT scan of the abdomen and pelvis Explanations: Important differential diagnoses for an unexplained pleural effusion, ascites, and pelvic adnexal mass include ovarian malignancy and Meigs syndrome. The next step is to evaluate for the presence of ascites and confirm the possible ovarian mass that was palpated. CT scan or a pelvic ultrasound are good imaging modalities to accomplish this. Meigs syndrome is a benign condition. Early detection and treatment will lead to a good prognosis. The pleural effusion and ascites will resolve permanently once the patient’s tumor is resected. Serum CA 125 is elevated in both ovarian cancer and Meigs syndrome. Its real value lies in the surveillance and evaluation of response to cancer treatment. Even though PET CT scan helps determine malignant and metastatic lesions. Its role is limited in Meigs syndrome as the frozen section is essential for the diagnosis. Go to the next page if you knew the correct answer, or click the link image(s) below to further research the concepts in this question (if desired).
Research Concepts: Meigs Syndrome
We update eBooks quarterly and Apps daily based on user feedback. Please tap flag to report any questions that need improvement.
Question 174:
A 17-year-old White male patient presents to the clinic complaining of a scaling rash that seems to migrate. He states the rash has been present for the last three months and has waxed and waned in intensity. He also admits that trauma seems to aggravate the rash. Upon inspection, it is noticed that multiple erythematous lesions with serpiginous borders in various stages of healing are located in and around his axillae and lower extremities. He also admits having diarrhea and fatigue starting around the same time as the rash. What genetic condition should he be screened for?
Choices: 1. 2. 3. 4.
Lynch syndrome Autoimmune polyglandular syndrome type 1 Ataxia-telangiectasia syndrome Multiple endocrine neoplasia type 1
Answer: 4 - Multiple endocrine neoplasia type 1 Explanations: Necrolytic migratory erythema is characteristic of glucagonoma. The rash of necrolytic migratory erythema is often located in areas of increased friction (e.g., intertriginous areas) and is aggravated by trauma. Glucagonomas can in rare cases be seen in association with multiple endocrine neoplasia type 1, often in younger patients. However, most glucagonomas are deemed to be sporadic. Go to the next page if you knew the correct answer, or click the link image(s) below to further research the concepts in this question (if desired).
Research Concepts: Necrolytic Migratory Erythema
We update eBooks quarterly and Apps daily based on user feedback. Please tap flag to report any questions that need improvement.
Question 175:
A 55-year-old female is found to have an incidental pancreatic cyst on MRI. The cyst is 2 cm in size, communicating with a pancreatic duct but without dilation or solid component. Which of the following is the next best step in the management of this patient?
Choices: 1. 2. 3. 4.
Surgical resection Monitoring with annual MRI EUS with FNA No further workup is needed
Answer: 2 - Monitoring with annual MRI Explanations: Given that the cyst is communicating with a pancreatic duct, so it is an intraductal papillary mucinous cancer (IPMN) of the pancreas. The American gastroenterology association (AGA) suggests that pancreatic cysts with at least 2 high-risk features, such as size greater than or equal to 3 cm, a dilated main pancreatic duct, or the presence of an associated solid component, should be examined with EUS-FNA. The AGA suggests that patients with pancreatic cysts less than 3 cm without a solid component or a dilated pancreatic duct undergo MRI for surveillance in 1 year and then every two years for a total of 5 years if there is no change in size or characteristics. AGA recommends surgical resection for IPMNs in patients with both a solid component and a dilated pancreatic duct and/or concerning features on EUS and FNA. Since this patient only has one feature, surgery is not indicated at this moment. No further work is inappropriate at this time given that the cyst will need to be monitored. Go to the next page if you knew the correct answer, or click the link image(s) below to further research the concepts in this question (if desired).
Research Concepts: Intraductal Papillary Mucinous Cancer Of The Pancreas
We update eBooks quarterly and Apps daily based on user feedback. Please tap flag to report any questions that need improvement.
Question 176:
A benign bone tumor is identified in a 15-year-old male. Histopathological analysis reveals a central nidus consisting of highly vascularized bone. Nerve fibers are identified within the nidus and are stimulated by increased prostaglandin levels. What is the current treatment of choice for this patient?
Choices: 1. 2. 3. 4.
Open en bloc excision Acetaminophen Curettage and packing CT guided percutaneous radiofrequency ablation
Answer: 4 - CT guided percutaneous radiofrequency ablation
Explanations: Osteoid osteomas are benign tumors that more commonly affects males in the age range of 5 to 25. Clinically, the pain is worse at night and relieved with NSAIDs. NSAIDs are effective in decreasing pain because histologically, there are increased levels of prostaglandin found within the tumor. Histologically, osteoid osteomas consist of a central nidus of highly vascularized bone. To date, CT guided percutaneous radiofrequency ablation is the preferred treatment. Conservative treatment consists of non-steroidal antiinflammatory medications (NSAIDs). NSAIDs successfully provide pain relief by inhibiting prostaglandin synthesis. Prostaglandin levels are 100 to 1000 times higher than that found in normal bone. Minimally invasive CT guided percutaneous radiofrequency ablation is now the treatment of choice with a documented success rate of 90%. CT guided percutaneous radiofrequency ablation minimizes damage to the surrounding bone. Historically, open en bloc excision was curative but had the potential for complications. CT guided percutaneous radiofrequency ablation is now the treatment of choice. Acetaminophen has no anti-inflammatory properties and therefore not considered as a form of conservative treatment. Go to the next page if you knew the correct answer, or click the link image(s) below to further research the concepts in this question (if desired).
Research Concepts: Osteoid Osteoma
We update eBooks quarterly and Apps daily based on user feedback. Please tap flag to report any questions that need improvement.
Question 177:
A 64-year-old woman was diagnosed with high-grade intraductal papillary mucinous neoplasm (IPMN) after she presented with obstructive jaundice. MRCP confirmed a 4 cm neoplasia involving the head of the pancreas and showed lymph node involvement. Which of the following is the next best step in the management of this patient?
Choices: 1. Whipple procedure (pancreaticoduodenectomy) 2. Whipple procedure (pancreaticoduodenectomy) followed by chemotherapy 3. Chemotherapy 4. Surveillance
Answer: 2 - Whipple procedure (pancreaticoduodenectomy) followed by chemotherapy
Explanations: Patients with large IPMNs that are symptomatic and causing obstructive jaundice should be surgically resected. Studies have shown that patients with invasive carcinoma and/or lymph node involvement have better survival with adjuvant therapy compared to surgery alone. Given her lymph node involvement, this patient will benefit from surgery followed by chemotherapy. Chemotherapy alone would not be enough for the management of her condition. This patient has a malignant cyst and will require surgical resection followed by chemotherapy. Go to the next page if you knew the correct answer, or click the link image(s) below to further research the concepts in this question (if desired).
Research Concepts: Intraductal Papillary Mucinous Cancer Of The Pancreas
We update eBooks quarterly and Apps daily based on user feedback. Please tap flag to report any questions that need improvement.
Question 178:
A woman whose mother was diagnosed with breast cancer in her 50’s undergoes initial screening mammogram and calcifications associated with a single unilateral BI-RADS 4 lesion are identified. She elects to undergo stereotactic biopsy, and the breast biopsy cores are sent to pathology for examination. A solitary focus of atypical ductal hyperplasia with associated calcifications is the only abnormality reported. What would be the most likely recommended subsequent treatment due to this finding?
Choices: 1. 2. 3. 4.
Bilateral radical mastectomy Surveillance only Repeat mammogram in one to three years Excisional lumpectomy
Answer: 4 - Excisional lumpectomy Explanations: Atypical ductal hyperplasia (ADH) is considered a premalignant high-risk lesion and is often found associated with calcifications on breast biopsy. There is a considerable risk of the pathologic upgrade for excision of BI-RADS 4 to 5 lesions with ADH present as the sole abnormality in the biopsy. For some low-risk patients with a solitary focus of ADH on breast biopsy in a BI-RADS 1 to 2 lesion, surveillance may be an acceptable management option. Tamoxifen can be considered for low-risk patients in whom a non-malignant atypical hyperplasia breast lesion is found on biopsy. Go to the next page if you knew the correct answer, or click the link image(s) below to further research the concepts in this question (if desired).
Research Concepts: Atypical Breast Hyperplasia
We update eBooks quarterly and Apps daily based on user feedback. Please tap flag to report any questions that need improvement.
Question 179:
A 65-year-old female presents to the emergency department with new-onset worsening dyspnea of 5 days duration. She has a medical history of heart disease, hypertension, and breast cancer. She takes lisinopril, aspirin, and undergoes chemotherapy for her ongoing breast cancer. Her vitals on presentation are the following: T 99.6, blood pressure 156/97 mmHg, heart rate of 130 bpm, and saturation 70%. She was immediately placed on high-flow nasal cannula. On physical exam, the patient was in respiratory distress using accessory muscle. A CT scan shows diffused irregular interlobular septal thickening with beading not previously seen on prior imaging. What is the most likely diagnosis?
Choices: 1. 2. 3. 4.
Radiation pneumonitis Lymphangitic carcinomatosis Pulmonary embolism Pulmonary edema
Answer: 2 - Lymphangitic carcinomatosis Explanations: Pulmonary lymphangitic carcinomatosis is a metastatic lung disease where an adenocarcinoma infiltrates the lymphatic ducts, causing obstruction. On a CT scan, it shows up as thickening of the interlobular septum. The patient has a history of breast cancer. Breast cancer is known to infiltrate the pulmonary lymphatic system. The imaging pattern described may not be diagnostic but is at least suggestive of lymphangitic spread and with the appropriate clinical scenario makes lymphangitic carcinomatosis more likely. A definitive diagnosis is obtained with a tissue biopsy, but a presumptive diagnosis is acceptable. Prognosis is generally poor, and the only therapy is to address the underlying malignancy. Go to the next page if you knew the correct answer, or click the link image(s) below to further research the concepts in this question (if desired).
Research Concepts: Lymphangitic Carcinomatosis
We update eBooks quarterly and Apps daily based on user feedback. Please tap flag to report any questions that need improvement.
Question 180:
A 66-year-old male is admitted to the hospital for abdominal pain, diarrhea, and fever of 2 days duration. The patient underwent stem cell transplantation three weeks ago. He denies blood in the stool. On physical exam, his temperature was 38.5 C (101.3 F), blood pressure 109/70 mmHg, pulse rate 109/mon, and respiratory rate 16/min. Abdominal exam reveals tenderness in the left lower quadrant. His labs show hemoglobin (Hb) 7 mg/dL, white blood cells (WBCs) 400/micoL, platelets 22000/micoL, and creatinine 1.3 mg/dL. The patient was seronegative for CMV before transplant. Computed tomography (CT) scan shows thickening of the bowel wall of the descending colon. What is the next best step in the management of this patient?
Choices: 1. 2. 3. 4.
Granulocyte infusion Stool culture Chest CT scan Testing for Clostridium difficile
Answer: 4 - Testing for Clostridium difficile Explanations: The patient presented with signs and symptoms concerning for colitis. He had a recent stem cell transplant, and he is immunosuppressed and neutropenic. He needs prompt workup and management to prevent complications. The CT findings confirm the diagnosis of colitis. The cause of colitis still needs to be determined. Given his neutropenia, the patient might have had neutropenic enterocolitis (NE). However, the location of inflammation on CT scan (descending colon) is not typical for NE, but it still can occur anywhere in the colon. However, before diagnosing NE, workup should be done to rule out other causes of colitis that include infectious causes. Stool panel, stool culture, and clostridium difficile testing should be done. Clostridium difficile gives results faster than stool culture, and early testing for Clostridium difficile allows earlier intervention with appropriate antibiotics. Some studies suggest granulocyte infusion for the treatment of neutropenic enterocolitis. However, to diagnose neutropenic enterocolitis, one must rule out other causes of colitis first such as clostridium difficile infection which can give a similar presentation. Stool culture is part of the workup done for a patient presenting with colitis symptoms, especially in the setting of immune suppression such as the patient in the question. However, it takes a long time for the results to come back as compared to Clostridium difficile testing. Early testing for Clostridium difficile allows earlier intervention with appropriate antibiotics. The
main concern in this patient is colitis, which is likely the source of his symptoms and signs. There is no evidence for a lung infection. Go to the next page if you knew the correct answer, or click the link image(s) below to further research the concepts in this question (if desired).
Research Concepts: Neutropenic Enterocolitis (Typhlitis)
We update eBooks quarterly and Apps daily based on user feedback. Please tap flag to report any questions that need improvement.
Question 181:
An 84-year-old male presents with an eyelid lesion that was diagnosed as a seborrheic keratosis. When you ask him if he has any other skin lesions on his body, he tells you that over the past few months he has developed about 50 similar lesions on his abdomen. What is this finding called?
Choices: 1. 2. 3. 4.
Troisier sign Leser-Trelat sign Trousseau sign Levine sign
Answer: 2 - Leser-Trelat sign Explanations: The Leser-Trelat sign is a rare finding. Normally seborrheic keratoses are benign proliferations of epithelial cells. The sudden appearance of multiple seborrheic keratoses in one region of the body could indicate a paraneoplastic process, usually from gastric adenocarcinoma. These lesions often have an inflamed base. This is called Leser-Trelat sign. Seborrheic keratoses classically have a “stuck on” appearance and can have varying degrees of pigmentation. Lesions tend to vary from a pink or flesh color to dark brown. These lesions are well circumscribed and are usually slightly elevated. Seborrheic keratoses are common in middle-aged and elderly individuals. Go to the next page if you knew the correct answer, or click the link image(s) below to further research the concepts in this question (if desired).
Research Concepts: Eyelid Papilloma
We update eBooks quarterly and Apps daily based on user feedback. Please tap flag to report any questions that need improvement.
Question 182:
A 52-year-old female without significant past medical history presents with chronic bouts of nausea. Labwork and vitals are within normal limits. An abdominal CT is then completed and demonstrates a 12 cm multiloculated cyst with multiple thin septa and subtle capsular enhancement in segment IV of the liver. What are the known causes of the most likely diagnosis?
Choices: 1. 2. 3. 4.
Developmental Oral contraceptive use History of bacterial infection History of autosomal dominant polycystic kidney disease
Answer: 1 - Developmental Explanations: The exact cause of biliary cystadenoma development is currently unknown. Current theories suggest biliary cystic adenoma are developmental malformations originating from endodermal precursors of biliary epithelium or implanted mesenchymal cells. BCAs have a predilection for the hepatic segment IV which is suggestive of its presumed implantation origin Autosomal dominant polycystic kidney disease, oral contraceptive pilluse, and history of bacterial infection are associated with simple cyst, hepatic adenoma, and pyogenic abscess formation respectively. Go to the next page if you knew the correct answer, or click the link image(s) below to further research the concepts in this question (if desired).
Research Concepts: Hepatic Cystadenoma
We update eBooks quarterly and Apps daily based on user feedback. Please tap flag to report any questions that need improvement.
Question 183:
A 65-year-old female with multiple myeloma presents with modest renal insufficiency and bone pain. She is treated with thalidomide and dexamethasone. Despite two months of therapy, her urine M-protein level continues to rise, and her creatinine is worsening. Which of the following is the next step in management for this patient?
Choices: 1. 2. 3. 4.
Switch to lenalidomide Add bortezomib Increase the dose of thalidomide Increase fluid intake
Answer: 2 - Add bortezomib Explanations: The patient's multiple myeloma (MM) is not responding to the current regimen, and bortezomib needs to be added. Bortezomib is a proteasome inhibitor. It can be given at a full dose in the presence of severe renal failure and can induce a response in about 35% of patients with MM. Side effects include peripheral neuropathy and myelosuppression. Bortezomib has contributed significantly to the treatment of patients with MM. Go to the next page if you knew the correct answer, or click the link image(s) below to further research the concepts in this question (if desired).
Research Concepts: Myeloma Kidney
We update eBooks quarterly and Apps daily based on user feedback. Please tap flag to report any questions that need improvement.
Question 184:
A 59-year-old female with a history of stage II breast cancer treated with radical mastectomy and radiation therapy to the left breast presents to the clinic to discuss treatment options after her specialist recommended the addition of hyperthermia to the treatment regimen. She is curious about the risks and benefit of hyperthermia. Which of the following is most accurate about the recommended treatment?
Choices: 1. There is no difference in local control between radiation alone versus radiation and hyperthermia 2. Local control rates ranging from 55%-66% have been reported 3. Toxicities include anemia, leukopenia, or myelosuppression 4. Higher doses of radiation are required when combined with hyperthermia compared to radiation alone
Answer: 2 - Local control rates ranging from 55%-66% have been reported
Explanations: The addition of hyperthermia to radiation therapy has been shown to achieve complete response rates ranging from 55% to 66%, much higher than the 16% to 47% seen with chemoradiotherapy. Hyperthermia combined with radiation permits the advantage of lower radiation doses due to its radiosensitizing properties. This is especially important in women who have received prior radiation therapy to the breast or chest wall. Acute toxicities associated with hyperthermia and radiation therapy have been reported as persistent ulceration seen in patients with prior radiation, blisters, second and third-degree burns, and infection. The ability to safely and effectively administer repeat radiation therapy is more difficult due to the toxicity associated with high cumulative radiation doses, as well as a lower likelihood of controlling a tumor that was originally resistant to prior definitive doses of radiation. Go to the next page if you knew the correct answer, or click the link image(s) below to further research the concepts in this question (if desired).
Research Concepts: Hyperthermia for Chest Wall Recurrence
We update eBooks quarterly and Apps daily based on user feedback. Please tap flag to report any questions that need improvement.
Question 185:
A 12-year-old boy presented to the urology service with a history of asymptomatic gross hematuria. Cystoscopy showed a polypoid, papillary lesion approximately 2 cm above and lateral to the left ureteral orifice. The lesion was connected to the wall of the bladder by a thin stalk. The lesion was entirely removed by transurethral resection and sent for pathologic examination. The specimen received in the fresh state consisted of an exophytic, friable, pale yellow mass. The mass exhibited multiple papillary projections as well as a cauterized stalk. Microscopically, the lesion consisted of intramucosal and submucosal papillary and anastomosing islands and cords of urothelium. Associated glandular spaces lined by urothelium and sometimes containing eosinophilic secretion were focally present. In other areas, submucosal glands resembling Brunn nests and submucosal cyst formations were seen. Neither cytologic atypia nor mitosis was seen. A minimal amount of edematous stroma with associated mild and focal inflammation is seen. What is the most likely diagnosis?
Choices: 1. 2. 3. 4.
Urothelial dysplasia Inverted urothelial papilloma Verrucous carcinoma Low-grade papillary urothelial carcinoma
Answer: 2 - Inverted urothelial papilloma Explanations: Most patients with inverted urothelial papilloma are in their fifth or sixth decade of life, with a reported patient age range of 9 to 88 years. This lesion is rare in children, with the first case described in 1979 and the youngest case described in a 9-year-old boy in 1993. On cystoscopy, inverted urothelial papilloma of the bladder appears as a pedunculated or sessile mass with a relatively obvious smooth surface, or as a polypoid/papillary tumor with a smooth surface. The diameter of the mass is less than 3 cm in most cases. Inverted urothelial papilloma is characterized by anastomosing islands and cords of normal urothelium originating from the overlying mucosa and growing downward into the stroma. The urothelial cells may invaginate extensively from the surface into the lamina propria but not into the muscularis propria. The base of the lesion is well circumscribed. Cytologic atypia is minimal to absent. Mitotic figures are rare. A subset of inverted urothelial papillomas contains foci that resemble cystitis cystica or glandularis. Complete transurethral resection is recommended for the treatment of inverted urothelial papilloma of the bladder. Go to the next page if you knew the correct answer, or click the link image(s) below to further research the concepts in this question (if desired).
Research Concepts:
Inverted Urothelial Papilloma
We update eBooks quarterly and Apps daily based on user feedback. Please tap flag to report any questions that need improvement.
Question 186:
A 37-year-old woman presents with a history of weight loss and malaise. She is a lifeguard and also notes a 3 cm irregularly colored mole on her right leg that recently has been painful and itchy, but doesn’t recall it being there last year. A biopsy reveals S-100 tumor cells without a BRAF V600E mutation, and there is metastatic disease. She has prescribed a new antibody directed against PD-1 which initially received FDA accelerated approval for advanced melanoma, which drug was she administered?
Choices: 1. 2. 3. 4.
Durvalumab Avelumab Pembrolizumab Atezolizumab
Answer: 3 - Pembrolizumab Explanations: Permbrolizumab was developed by Merck and initially received FDA accelerated approval for refractory, advanced melanoma in September 2014. Pembrolizumab has been approved for the treatment of many other oncologic conditions and many more are currently in clinical development. Pembrolizumab is an antibody directed against PD-1. Durvalumab, Avelumab and Atezolizumab are antibodies directed against PD-L1. Go to the next page if you knew the correct answer, or click the link image(s) below to further research the concepts in this question (if desired).
Research Concepts: Pembrolizumab
We update eBooks quarterly and Apps daily based on user feedback. Please tap flag to report any questions that need improvement.
Question 187:
A 23-year-old male with an inherited bone marrow failure syndrome is admitted to the hospital for allogeneic bone marrow stem cell transplant. The donor is his younger sibling. He feels well and has no complaints including fevers or spontaneous bleeding. Medications are acyclovir, cyclosporine, and trimethoprim/ sulfamethoxazole. Vital signs are stable. Physical examination reveals a well-developed male with conjunctival pallor, hyperpigmented patches on his back, no thumbs bilaterally, and petechiae over bilateral anterior shins. Labs are notable for hemoglobin 7.3 mg/dL, platelets 21000/microL, reticulocyte 0.5%, and white blood cell count 2100/microL with 25% polymorphonuclear leukocytes (PMNs). Bone marrow biopsy done one month ago was negative for clonal evolution or malignant features. Which of the following is the most appropriate next step in management?
Choices: 1. Conditioning regimen with full-dose cyclophosphamide and total body irradiation at 2 Gy 2. Conditioning regimen with dose-reduced cyclophosphamide and total body irradiation at 4 Gy 3. Conditioning regimen with fludarabine and dose-reduced cyclophosphamide 4. Conditioning regimen with fludarabine, dose-reduced cyclophosphamide, and total body irradiation at 4 Gy
Answer: 3 - Conditioning regimen with fludarabine and dose-reduced cyclophosphamide
Explanations: Bone marrow failure refers to the decreased production of one or more major hematopoietic lineages leading to diminished or absent hematopoietic precursors in the bone marrow and attendant cytopenias. Inherited bone marrow failure syndrome (IBMFS) is bone marrow failure that occurs from germline mutations passed down from parents or arising de novo and is accompanied by unique extra-marrow features. Definitive treatment for marrow disease associated with IBMFS is hematopoietic stem cell transplant (HSCT), ideally before clonal evolution or many blood product transfusions. However, this is associated with increased mortality using conventional regimens. The conditioning regimens for most patients with IBMFS should avoid total body irradiation (TBI) except low dosage in patients at high risk of rejection and use reduced-intensity conditioning (RIC) with decreased amounts of cyclophosphamide while adding fludarabine. Fludarabine is a purine analog that incorporates into DNA to cause DNA strand breaks while preventing DNA repair synthesis by inhibiting several DNA enzymes (ligase I, polymerase, primase) and ribonucleotide reductase. Fludarabine-containing conditioning regimens are recommended for HSCT in patients with Fanconi anemia, Diamond-Blackfan anemia, and congenital amegakaryocytic thrombocytopenia. While there is increased use of fludarabine in similar settings for those with dyskeratosis congenita or Shwachman Diamond
syndrome, there is not enough data available to establish recommendations. Go to the next page if you knew the correct answer, or click the link image(s) below to further research the concepts in this question (if desired).
Research Concepts: Bone Marrow Failure
We update eBooks quarterly and Apps daily based on user feedback. Please tap flag to report any questions that need improvement.
Question 188:
A 56-year-old female has an ultrasound-guided core biopsy of her left breast performed for calcifications identified on screening mammography. Pathologic examination reveals a focal monotonous intraductal cribriform epithelial proliferation that is suggestive of ductal carcinoma in situ (DCIS) but does not quite meet the full criteria for low-grade DCIS. Which of the following is pertinent to the involved mechanistic and or molecular biology alteration involved?
Choices: 1. 2. 3. 4.
Telomerase inactivation P16 activation Suppressed cyclin-D1 expression HOXA inactivation
Answer: 4 - HOXA inactivation Explanations: Atypical ductal hyperplasia (ADH) is defined histologically by an atypical, low-grade, intraductal proliferation of epithelial cells in the breast that does not meet all criteria for low-grade DCIS. Molecular studies have discovered shared molecular characteristics between atypical proliferative lesions such as ADH and low-grade DCIS. This leads many to believe that ADH will continue on a spectrum to develop into low-grade DCIS. Atypical lobular hyperplasia (ALH), on the other hand, is not necessarily considered a precursor for malignancy but a high-risk indicator. When ADH or ALH is identified in a breast biopsy, the risk of developing DCIS or invasive breast cancer increases by four to five times with a 2:1 predilection for the ipsilateral breast versus contralateral. It is unclear if a precursor lesion exists for high-grade DCIS or invasive carcinoma; however, breast cancer relative risk is increased four times when atypical hyperplasia is identified on a breast biopsy. The lifetime risk of developing breast cancer in patients with ADH or ALH on biopsy is 15% to 20%. Molecular studies have identified important agents related to unchecked proliferation, including cyclin D1 overexpression, p16 inactivation, HOXA inactivation, and activation of telomerase. These changes can lead to inhibition of cell death and the development of malignancy. The sequence and interplay of these changes are not well understood, but further studies on their relation to each other and the development of atypical proliferative lesions and breast cancer could
shed light on accurate risk stratification, prognosis, and early predictors of advanced neoplasia. Go to the next page if you knew the correct answer, or click the link image(s) below to further research the concepts in this question (if desired).
Research Concepts: Atypical Breast Hyperplasia
We update eBooks quarterly and Apps daily based on user feedback. Please tap flag to report any questions that need improvement.
Question 189:
A 65-year-old male with known GOLD stage 4 chronic obstructive pulmonary disease was diagnosed with unresectable stage 3b adenocarcinoma of the right lung. He was also diagnosed with hypertrophic pulmonary osteoarthropathy resulting in excruciating pain in both arms and elbow. What will be the medication of choice for symptom relief that has the best outcome?
Choices: 1. 2. 3. 4.
Hydrocodone Bevacizumab Intravenous pamidronate Ibuprofen
Answer: 3 - Intravenous pamidronate Explanations: Intravenous pamidronate and zoledronic acid have the best success rate in symptom suppression in hypertrophic pulmonary osteoarthropathy (HPOA) with bronchogenic carcinoma. Therapeutic response was noted regarding symptom suppression as well as radiographic resolution of periostitis in bone scan. HPOA lesions are typically poorly responsive to opioids. Although NSAIDs are helpful, typically a COX -2 inhibitor like celecoxib has better efficacy. The role of bevacizumab is still experimental. Go to the next page if you knew the correct answer, or click the link image(s) below to further research the concepts in this question (if desired).
Research Concepts: Secondary Hypertrophic Osteoarthropathy
We update eBooks quarterly and Apps daily based on user feedback. Please tap flag to report any questions that need improvement.
Question 190:
A 75-year-old male with hyperlipidemia and coronary artery disease presents to the emergency department with complaints of palpitations, transient wheezing and dyspnea, abdominal pain, and occasional diarrhea. His blood pressure is 142/70 mmHg, heart rate is 102/min, respiratory rate 18/min, and temperature is 37 C (98.6 F). Physical exam reveals an elderly well-nourished male with tachycardia, facial flushing, bilateral expiratory wheezing, and isolated right lower quadrant abdominal pain. An electrocardiogram shows sinus tachycardia, and the chest radiograph appears to be normal. Basic lab work is obtained, which shows a white blood cell count of 14,000/microL, chloride 85 mmol/L, potassium 3.0 mmol/L, creatinine 1.2 mg/dL, lactic acid 2.5 mmol/L, and urinalysis with large esterase and negative for nitrite or bacteria. Computed tomography of the abdomen and pelvis with oral and intravenous contrast is obtained and reveals right-sided liver lesions as well as appendiceal dilation, hypervascularity, and adjacent fat stranding with what appears to be a 2.5 cm mass at its base. What is the most appropriate management of the patient's current condition?
Choices: 1. 2. 3. 4.
Intravenous antibiotics Appendectomy Right hemicolectomy Ileocecectomy
Answer: 3 - Right hemicolectomy Explanations: The patient presents with a conglomerate of symptoms that may at first seem to be disconnected, but after computed tomography reveals an appendiceal mass with hepatic lesions, the constellation of symptoms is most likely secondary to carcinoid syndrome. Carcinoid syndrome often presents with palpitations, facial flushing, wheezing and dyspnea, and occasionally diarrhea. The most common site of carcinoid is in the appendix, and to have carcinoid syndrome, the carcinoid must be metastatic, which often is to the liver. For masses found at the base of the appendix, operative right-sided hemicolectomy is the procedure of choice to give the best chance to remove the associated lymph node basin and effect an oncologic cure. Carcinoid syndrome is caused by the endogenous secretion of mainly serotonin and kallikrein. Laboratory tests to identify carcinoid tumors measure the levels of 5-HIAA in a urine sample collected over 24 hours. When a tumor forms at the base of the appendix, the associated mass effect can lead to obstruction of the ostia of the appendiceal lumen, leading to appendicitis. More commonly in adults, the ostia become obstructed by a fecalith and in children from Peyer's patch hyperplasia. This obstruction leads to bacterial overgrowth and retained mucous formation with increased wall pressure, leading to venous stasis and arterial ischemia. If left untreated, it can develop into appendicitis with gangrene and rupture. The most common tumor that is associated with appendicitis is carcinoid. A simple appendectomy or
ileocecectomy would not remove the lymph node basin in which the appendix drains and, therefore, would be considered not a definitive treatment. Gastroenteropancreatic neuroendocrine tumors (GEPNETs) are the most common histopathological subtypes. They might rarely metastasize to the liver and or lymph nodes. Therefore, in patients with suspicious GEP-NETs (carcinoid tumor), further evaluation of the liver and the ileocolic lymph node basin are essential. Basically, the primary tumor size dictates the demanding surgical steps. Accordingly, in the carcinoid tumors of less than 1-centimeter size, an appendectomy with negative margins is the only requested surgical management. Although in the carcinoid tumor of greater than 2 cm, a right hemicolectomy is indicated, the surgical plan in appendiceal carcinoid lesions of 1 to 2 cm is still equivocal. However, in the presence of mesenteric invasion, and enlarged lymph nodes, and or equivocal surgical margins, right hemicolectomy is recommended. Go to the next page if you knew the correct answer, or click the link image(s) below to further research the concepts in this question (if desired).
Research Concepts: Appendicitis
We update eBooks quarterly and Apps daily based on user feedback. Please tap flag to report any questions that need improvement.
Question 191:
A 45-year-old male with no significant past medical history presents with a 1-day history of renal colic. He takes no medications, and he has no significant family history. On physical exam, pulse is 84/min, BP 120/70 mmHg, and T 36.9 C. A non-contrast CT scan of the abdomen showed a 5 mm left ureteric stone and a 1.5 cm right adrenal mass which has been reported as benign adenoma with a Hounsfield unit less than 10. Urology has planned expectant management for the renal stone. What other investigations are indicated in this case?
Choices: 1. CT scan of the adrenals with contrast 2. MRI scan of the adrenals 3. Dexamethasone suppression test and plasma fractionated metanephrines 4. Fine needle aspiration of the adrenal mass
Answer: 3 - Dexamethasone suppression test and plasma fractionated metanephrines
Explanations: All adrenal adenomas need further work up to clarify the functional status. There should be an investigation for Cushing syndrome and pheochromocytoma, and if the patient has hypertension, then there should be an investigation for aldosterone excess. The majority of adrenal adenomas are non-functioning, but the hormonal workup is considered mandatory especially for safety purposes before any surgical or invasive intervention takes place. Biopsy of the adrenal lesion is indicated in very rare cases when there is a suspicion of metastatic disease from a known or very infrequently unknown primary cancer. Adrenal lesions which exhibit less than 10 HU (Hounsfield Units) on non-contrast CT scan strongly suggest a benign adenoma and there is no need for further imaging. Go to the next page if you knew the correct answer, or click the link image(s) below to further research the concepts in this question (if desired).
Research Concepts: Adrenal Adenoma
We update eBooks quarterly and Apps daily based on user feedback. Please tap flag to report any questions that need improvement.
Question 192:
A 65-year-old male with HIV presents with fevers, night sweats and 20-lb weight loss in the past three months. His physical exam reveals inguinal lymphadenopathy. CT of the chest, abdomen, and pelvis reveals a gastric mass. Biopsy of the gastric mass reveals plasmablasts and immunophenotype is positive for CD 19, CD 38, MUM-1 and MYC gene rearrangement is present. Which of the following is the most likely diagnosis?
Choices: 1. 2. 3. 4.
Multiple myeloma Hodgkin lymphoma T cell anaplastic lymphoma Plasmablastic lymphoma
Answer: 4 - Plasmablastic lymphoma Explanations: Plasmablastic lymphoma (PBL) is a neoplasm that exhibits B cell and plasma cell markers such as CD19 and CD79a which are the B cell markers and CD 138 which is a plasma cell marker. Morphologically, plasmablast is the cell of origin which is a B cell in process of becoming a plasma cell. Plasmablastic lymphoma is very hard to diagnose and treat due to its distinctive pathology and expression of both B and plasma cell markers. There is no established standard of care chemotherapy regimen for the treatment of PBL. R-CHOP (rituximab, cyclophosphamide, adriamycin, vincristine, prednisone) and da-R-EPOCH (dose adjusted rituximab, etoposide, prednisone, vincristine, adriamycin) are some of the regimens used in the clinical practice. Go to the next page if you knew the correct answer, or click the link image(s) below to further research the concepts in this question (if desired).
Research Concepts: Plasmablastic Lymphoma
We update eBooks quarterly and Apps daily based on user feedback. Please tap flag to report any questions that need improvement.
Question 193:
A 67-year-old woman with a history of stage I right breast cancer treated with lumpectomy and radiation therapy presents to discuss treatment options after being diagnosed with recurrence to the right breast. She is concerned about additional radiation to the breast. Her provider recommends concurrent temperature regulation therapy, which may allow the radiation amount to be reduced. Which of the following is most accurate about this therapy?
Choices: 1. Decreased cell permeability preventing the diffusion of nutrients into tumor cells 2. Decreased cell permeability, causing retention of sodium ions in the cell, followed by enhancing water diffusion and cell lysis 3. Increased cell permeability allowing entry of calcium ions into the cell, with the initiation of the apoptosis cascade 4. Increased cell permeability allowing increased exit of potassium ions from the cell interior, leading to cell dehydration
Answer: 3 - Increased cell permeability allowing entry of calcium ions into the cell, with the initiation of the apoptosis cascade
Explanations: Cellular alterations induced by extreme temperatures include increased cellular permeability. The increased cellular permeability induced by supraphysiologic temperatures increases the influx of calcium into the cell. Calcium is a component of the apoptotic cascade. Increased cytosolic calcium induces the release of mitochondrial caspases, activating apoptosis. Go to the next page if you knew the correct answer, or click the link image(s) below to further research the concepts in this question (if desired).
Research Concepts: Hyperthermia for Chest Wall Recurrence
We update eBooks quarterly and Apps daily based on user feedback. Please tap flag to report any questions that need improvement.
Question 194:
An 18-year-old male patient presented with repeated swelling and pain in the right lumbosacral region for approximately five months. Computed tomography and magnetic resonance imaging of the pelvis revealed a segmented, expansive, multiseptate lesion. Resection with wide margins and complete curettage of the tumor were performed. The pathological analysis of the resected tissue revealed that the tumor was composed of woven bone spicules and trabeculae. These spicules were haphazardly arranged and were lined with a single layer of osteoblasts. The vascularity was rich with extravasated erythrocytes. Diffusely scattered osteoclast-type, multinucleated giant cells were present. What is the most likely diagnosis?
Choices: 1. 2. 3. 4.
Telangiectatic osteosarcoma Giant cell tumor Osteoma Osteoblastoma
Answer: 4 - Osteoblastoma Explanations: Osteoblastoma affects mainly adolescents and young adults (mean age, 20 years) with a male predominance. Osteoblastoma can involve any bone, with spine and sacrum accounting for one-third of cases. Osteoblastoma has identical histological features to osteoid osteoma. The tumor is composed of woven bone spicules or trabeculae. These spicules are haphazardly arranged and are lined with a single layer of osteoblasts. The vascularity is rich, often with extravasated erythrocytes. Osteoblasts may have mitoses but they are not atypical. Diffusely scattered osteoclast-type, multinucleated giant cells are often present. Osteoblastomas do not infiltrate or isolate preexisting lamellar bone structures as do osteosarcomas, so special attention should be given to the border between preexisting cortex or marrow trabeculae. Go to the next page if you knew the correct answer, or click the link image(s) below to further research the concepts in this question (if desired).
Research Concepts: Osteoblastoma
We update eBooks quarterly and Apps daily based on user feedback. Please tap flag to report any questions that need improvement.
Question 195:
A patient presents to the clinic and complains of photophobia, has light skin, and silvery hair. He appears to be an albino and complains of suffering frequent bouts of sinusitis, pneumonia, and acne. He was diagnosed with neuropathy at age 16, but no one has ever diagnosed the cause. What syndrome do you suspect might be responsible for his symptoms?
Choices: 1. 2. 3. 4.
Sezary Disease Caplan Syndrome Felty Syndrome Chediak-Higashi Syndrome
Answer: 4 - Chediak-Higashi Syndrome Explanations: Chediak-Higashi Syndrome typically presents with clotting problems, leaving the patient prone to easy bruising. Chediak-Higashi Syndrome causes neuropathy, usually diagnosed when the patient is a teenager. Chediak-Higashi Syndrome causes impaired phagocytosis of bacteria, which impairs bacteriolysis, leading to frequent infections of the skin, mucous membranes, and respiratory tract from bacterial infections. People with Chediak Higashi Syndrome suffer from oculocutaneous albinism. They typically have light pigmentation of the skin, hair, and eyes, and frequently complain of photophobia and sensitivity to light. Go to the next page if you knew the correct answer, or click the link image(s) below to further research the concepts in this question (if desired).
Research Concepts: Lymphoproliferative Disorders
We update eBooks quarterly and Apps daily based on user feedback. Please tap flag to report any questions that need improvement.
Question 196:
A hepatologist encountered a 45-year male, having complaints of boring, persistent, mild intensity pain over right hypochondrium. The patient reports some weight loss with an increase in the severity of the pain on movement. Radiological examination, including a computed tomography scan of the abdomen, excluded the possibility of hepatoma and cirrhosis. Furthermore, a targeted ultrasound-guided biopsy of the lesion revealed the absence of fibrosis, with an abundant cellular component. What is the underlying diagnosis in this patient?
Choices: 1. Hepatoma 2. Diffuse large B cell lymphoma 3. Bening hepatic adenoma 4. Metastatic tumor of the liver with primary epithelial tumor elsewhere
Answer: 2 - Diffuse large B cell lymphoma Explanations: The characteristic tumor cells of primary hepatic lymphoma appear with the abundant mitotically active large and atypical cells with pale to clear cytoplasm and vague cell membranes associated with granular, eosinophilic necrotic debris. The nuclei are large with condensed, or open, chromatin with prominent nucleoli. CD 20 is a pan B cell marker, characteristically present among primary hepatic lymphoma of B cell origin. CD3, Hep Par 1 and Cytokeratin AE1/AE3 are present in progenitor T cell, hepatocellular carcinoma, and tumors of epithelial origin respectively. The primary hepatic lymphoma can occur over the background of cirrhosis, and sometimes can occur in association with hepatitis C and B infection; which could further complicate with the diagnosis of hepatocellular carcinoma. Hence, primary hepatic lymphoma requires the application of characteristic markers; notably the B and T cell markers while excluding the hep par1, to confirm the diagnosis of the lymphoma. Go to the next page if you knew the correct answer, or click the link image(s) below to further research the concepts in this question (if desired).
Research Concepts: Hepatic Lymphoma
We update eBooks quarterly and Apps daily based on user feedback. Please tap flag to report any questions that need improvement.
Question 197:
A 12-year-old male presents to his provider with a 2-month history of pain in his lower right thigh. He describes the pain as "achy" in character, moderate in severity, and persistent in duration. On physical examination, there is a palpable mass in the distal right thigh. He adds that it is tender. He also exhibits a limited range of motion at the knee. Radiographs of the lower extremity reveal an osseous lesion with a radiodense matrix and a "sunburst" appearance. Local soft tissue extension is apparent. Which organ is the most common site of spread of the most likely lesion in this patient with inadequate treatment?
Choices: 1. 2. 3. 4.
Liver Brain Lungs Adrenal glands
Answer: 3 - Lungs Explanations: Osteosarcoma is a lethal pediatric malignancy which most commonly occurs in the long bones of the lower extremity. Physical exam findings may include warmth and tenderness in the affected area, a palpable mass, and a limited range of motion when a joint is involved. The lungs are the most common site of extraosseous metastasis. CT imaging of the chest may reveal pulmonary nodules with density (measured in Hounsfield units) similar to that of bone due to the production of osteoid by the metastatic osteosarcoma cancer cells. The characteristic "sunburst" appearance of osteosarcoma seen on radiography is the result of aggressive periostitis. Extrapulmonary metastasis related to osteosarcoma is exceedingly rare. Go to the next page if you knew the correct answer, or click the link image(s) below to further research the concepts in this question (if desired).
Research Concepts: Osteosarcoma
We update eBooks quarterly and Apps daily based on user feedback. Please tap flag to report any questions that
need improvement.
Question 198:
A 45-year-old female presents with a multinodular mass in the hand. The pathology report showed a multinodular lesion composed of bland spindled to oval cells in an abundant myxoid stroma subdivided by fibrous septae. No mitotic activity was seen. What features are not associated with nerve sheath myxomas?
Choices: 1. 2. 3. 4.
Confusion with cellular neurothekeomas Tendency to be benign Curvilinear vessels and cytologic atypia Positivity for S-100
Answer: 3 - Curvilinear vessels and cytologic atypia Explanations: Nerve sheath myxomas and cellular neurothekeomas were once labeled as "neurothekeomas." However, the former is a nerve sheath tumor, and the latter is a tumor of uncertain differentiation. They can be distinguished from each other by the S-100 staining pattern. Nerve sheath myxomas are benign and generally managed with conservative complete excision. Nerve sheath myxomas are distinguished from lowgrade myxofibrosarcomas by their bland cytology and lack of curvilinear vessels. Nerve sheath myxomas are positive for S-100. Go to the next page if you knew the correct answer, or click the link image(s) below to further research the concepts in this question (if desired).
Research Concepts: Neurothekeoma
We update eBooks quarterly and Apps daily based on user feedback. Please tap flag to report any questions that need improvement.
Question 199:
A fifty-year-old man with a history of diabetes mellitus and cancer treated with bleomycin five years previously is admitted for Fournier gangrene. Consult is requested from a hyperbaric oxygen specialist. Which of the following is true?
Choices: 1. Hyperbaric oxygen therapy is contraindicated due to the bleomycin use in the past 2. Bleomycin confers no risk given the distant exposure 3. Echocardiogram to exclude impaired cardiac function should be done prior to using hyperbaric oxygen 4. Pulmonary fibrosis or impairment would be a contraindication for hyperbaric oxygen therapy
Answer: 4 - Pulmonary fibrosis or impairment would be a contraindication for hyperbaric oxygen therapy
Explanations: Some studies show that with even a remote history of bleomycin, treatment exposure to supplemental oxygen could cause a severe pneumonitis. The consensus is that as long as the patient has no signs of pulmonary compromise from fibrosis and that it has been at least 3 to 4 months since treatment with bleomycin. Exposure to hyperbaric oxygen should not be contraindicated. Four percent of patients with cancer experienced bleomycin-related lung injury following treatment. Many patients previously treated with bleomycin have successfully been treated with hyperbaric oxygen without sequelae. Go to the next page if you knew the correct answer, or click the link image(s) below to further research the concepts in this question (if desired).
Research Concepts: Hyperbaric Contraindicated Chemotherapeutic Agents
We update eBooks quarterly and Apps daily based on user feedback. Please tap flag to report any questions that need improvement.
Question 200:
A 17-year-old male presents with a tender mass in his leg. He has had general malaise for 3 months. A leg x-ray reveals a mass with an "onion layering" pattern. A biopsy shows a small, round, blue cell tumor. What is the best treatment for this patient?
Choices: 1. 2. 3. 4.
Surgical resection alone Surgery followed by radiation Preoperative chemotherapy followed by surgery Chemotherapy alone
Answer: 3 - Preoperative chemotherapy followed by surgery
Explanations: Preoperative chemotherapy followed by surgery has produced the best results for localized Ewing sarcoma. Multidrug chemotherapy always includes vincristine, doxorubicin, ifosfamide, and etoposide and may include cyclophosphamide. The most commonly used form of local control is surgery. When the functional morbidity of surgery is considered too high, radiation therapy is used for local control. Go to the next page if you knew the correct answer, or click the link image(s) below to further research the concepts in this question (if desired).
Research Concepts: Ewing Sarcoma
We update eBooks quarterly and Apps daily based on user feedback. Please tap flag to report any questions that need improvement.
Section 3 Question 201:
A 23-year-old man presented with a three-month history of cough and shortness of breath. Electrocardiogram indicated normal sinus rhythm, and physical examination revealed normal blood pressure and regular pulse. Echocardiography revealed a large mass located in the left ventricular lateral wall. Cardiac magnetic resonance imaging (MRI) indicated a large mass located in the lateral wall of the left ventricle. The mass was regular and well defined, with a size of 8×5×4 cm. The mass was T1 iso-intense compared with cardiac muscle and T2 hypointense compared with cardiac muscle. The patient underwent surgical excision of the tumor under standard cardiopulmonary bypass, and median sternotomy was performed. Grossly, the tumor was well-circumscribed, firm and white. Histopathological examination of the surgical specimen revealed a tumor proliferation composed of monomorphic fibroblasts set in a collagenous matrix. There were no atypia and no mitotic figures. The periphery of the tumor interdigitated with cardiac muscle. What is the most likely diagnosis?
Choices: 1. 2. 3. 4.
Cardiac Cardiac Cardiac Cardiac
papillary fibroelastoma myxoma rhabdomyoma fibroma
Answer: 4 - Cardiac fibroma Explanations: Cardiac fibromas are well-circumscribed, firm, usually solitary white masses, grossly resembling fibromatosis or uterine leiomyomas. Cardiac fibroma is composed of monomorphic fibroblasts that demonstrate little or no atypia. The degree of cellularity often decreases with the age of the patient, and the amount of collagen increases. There can be focally myxoid background. Mitoses are generally present only in tumors occurring in infants. Calcification is a common finding in fibromas from patients of all ages. A variable number of elastic fibers can be present. The presence of tumor at surgical margins has not been shown to increase the risk of recurrence. Cardiac fibromas that can be resected even incompletely have a good prognosis. Go to the next page if you knew the correct answer, or click the link image(s) below to further research the concepts in this question (if desired).
Research Concepts: Cardiac Fibroma
We update eBooks quarterly and Apps daily based on user feedback. Please tap flag to report any questions that
need improvement.
Question 202:
A 45-year-old female with a history of Hashimoto thyroiditis presents with a generalized nonerythematous papular rash. Skin biopsy was positive for malignancy. The abnormal cells were positive for CD45, lysozyme, and myeloperoxidase. Bone marrow aspiration and biopsy are likely to reveal which type of leukemia?
Choices: 1. 2. 3. 4.
Acute myeloid leukemia Acute lymphocytic leukemia Chronic lymphocytic leukemia Adult T-cell leukemia/lymphoma
Answer: 1 - Acute myeloid leukemia Explanations: While CD45 is a pan-leukocyte marker, lysozyme and myeloperoxidase positivity refer to myeloid origin of the cells. Lymphocytic cells and their precursors are negative for lysozyme and myeloperoxidase. Lymphocytic cells and their precursors are negative for lysozyme and myeloperoxidase. Adult T-cell leukemia/lymphoma is lymphocytic leukemia and negative for lysozyme and myeloperoxidase. The cells in ATLL have a characteristic floret shaped morphology that helps in distinction from other entities. Go to the next page if you knew the correct answer, or click the link image(s) below to further research the concepts in this question (if desired).
Research Concepts: Leukemia Cutis
We update eBooks quarterly and Apps daily based on user feedback. Please tap flag to report any questions that need improvement.
Question 203:
A 20-year-old female with inherited bone marrow failure syndrome is undergoing evaluation for stem cell transplant. Her older brother was recently confirmed to be a match. The patient has received transfusions: one unit of packed red blood cells and two units of platelets in the last two years from anonymous donors. Medications are acyclovir and trimethoprim/sulfamethoxazole. Vitals including orthostatic blood pressure is stable. Physical examination reveals a well-developed female with conjunctival pallor, dried blood in nares, and petechiae over bilateral anterior legs. Labs are notable for hemoglobin 7.0 mg/dL, platelets 10000/microL, reticulocyte 0.5%, and white blood cell count 2100/microL with 30% polymorphonuclear leukocytes (PMNs). Cytomegalovirus (CMV) IgG and IgM antibodies are negative. Which of the following is the most appropriate next step in management?
Choices: 1. One unit leukoreduced platelets from a cross-matched unrelated donor 2. One unit leukoreduced CMV-negative platelets from a cross-matched unrelated donor 3. One unit leukoreduced CMV-negative platelets from the patient’s older brother 4. One unit each platelet and red blood cells, both leukoreduced and CMV-negative, from a cross-matched unrelated donor
Answer: 2 - One unit leukoreduced CMV-negative platelets from a cross-matched unrelated donor
Explanations: Bone marrow failure refers to decreased production of one or more major hematopoietic lineages leading to diminished or absent hematopoietic precursors in the bone marrow and attendant cytopenias. Inherited bone marrow failure syndrome (IBMFS) is bone marrow failure that occurs from germline mutations passed down from parents or arising de novo and is accompanied by unique extra-marrow features. The definitive treatment for marrow disease involves hematopoietic stem cell transplant (HSCT). Supportive care includes infection prevention/treatment and transfusions with leukoreduced red blood cells (RBCs) for hemoglobin less than 7 mg/dL or platelets less than 10,000/microL or less than 50,000/microL for active blood loss. Minimize transfusions in patients preparing for HSCT because platelet and RBC antigens can cause allosensitization of the recipient and increase the risk of graft failure. If necessary, choose leukoreduced blood products to decrease exposure to donor white blood cell antigen and cytomegalovirus (CMV)-negative products in patients who are CMV-negative that Never use transfusions from potential HSCT donors. Go to the next page if you knew the correct answer, or click the link image(s) below to further research the concepts in this question (if desired).
Research Concepts: Bone Marrow Failure
We update eBooks quarterly and Apps daily based on user feedback. Please tap flag to report any questions that need improvement.
Question 204:
A 50-year-old male with abdominal fullness and discomfort presents to the clinic for evaluation. On examination, splenomegaly without hepatomegaly is detected. Lab work shows a hemoglobin of 11.5 grams/dL, WBC 4500/mm3, and platelet count 110,000/microliter. The absolute neutrophil count is 1500/mm3. After further investigation, a diagnosis of hairy cell leukemia is made. What is the next step in management?
Choices: 1. 2. 3. 4.
Watch and wait Start with single-agent cladribine Start with single-agent rituximab Give interferon beta
Answer: 2 - Start with single-agent cladribine Explanations: Cladribine is the preferred initial therapy for most patients with previously untreated hairy cell leukemia. For most patients with hairy cell leukemia, five daily infusions of cladribine or 7 days of continuous infusion cladribine are given. A watch and wait strategy is appropriate for asymptomatic patients with hairy cell leukemia. Rituximab is not used as a monotherapy for hairy cell leukemia but may be used in adjunct with cladribine to induce longer remission. Interferon alpha, not beta, is usually considered in pregnant patients or as initial treatment in patients with severe pancytopenia or active infection. Go to the next page if you knew the correct answer, or click the link image(s) below to further research the concepts in this question (if desired).
Research Concepts: Hairy Cell Leukemia
We update eBooks quarterly and Apps daily based on user feedback. Please tap flag to report any questions that need improvement.
Question 205:
A patient has had several episodes of syncope preceded by blurred vision, profuse sweating, and lethargy. Blood tests reveal hypoglycemia, and ultrasound shows a mass in the mid-epigastrium. What is the most likely diagnosis?
Choices: 1. 2. 3. 4.
Metastatic brain cancer Insulinoma Type 1 diabetes mellitus Pheochromocytoma
Answer: 2 - Insulinoma Explanations: About 85% of patients with insulinoma present with diplopia, blurred vision, palpitations, or weakness. Other symptoms of insulinoma include confusion, abnormal behavior, unconsciousness, and amnesia. About 12% of patients have grand mal seizures. Laboratory results will reveal high insulin, low glucose, and high levels of C peptide and proinsulin. In patients with insulinoma, diazoxide is used to decrease insulin secretion, and somatostatin analogs are used to prevent hypoglycemia. Go to the next page if you knew the correct answer, or click the link image(s) below to further research the concepts in this question (if desired).
Research Concepts: Insulinoma
We update eBooks quarterly and Apps daily based on user feedback. Please tap flag to report any questions that need improvement.
Question 206:
A 54-year-old male is seen in the hematology clinic with symptoms of confusion, loss of coordination, headaches, fatigue, and weight loss. He denies any illness except for diet-controlled diabetes mellitus. He does not smoke or drink. Examination reveals several swollen cervical lymph nodes and blood work reveals an elevated erythrocyte sedimentation rate. Serology reveals that he has an excess level of IgM. What is the most likely diagnosis?
Choices: 1. 2. 3. 4.
Multiple myeloma Waldenstrom macroglobulinemia Leukemia Hodgkin lymphoma
Answer: 2 - Waldenstrom macroglobulinemia Explanations: There is proliferation of IgM-producing lymphoid tissue in Waldenstrom macroglobulinemia (WM). The differential diagnosis of Waldenstrom macroglobulinemia includes leukemia, chronic lymphocytic leukemia, small lymphocytic lymphoma, mantle cell lymphoma, marginal zone lymphoma, plasma cell neoplasms, Castleman disease, and reactive plasmacytosis. Plasma cells in WM are CD138 negative, differentiating WM from multiple myeloma. Diagnosis of WM usually is challenging due to the lack of distinct diagnostic features; rendering the diagnosis is usually through exclusion. Go to the next page if you knew the correct answer, or click the link image(s) below to further research the concepts in this question (if desired).
Research Concepts: Lymphoplasmacytic Lymphoma
We update eBooks quarterly and Apps daily based on user feedback. Please tap flag to report any questions that need improvement.
Question 207:
A 70-year-old male presents to the clinic with progressive dysphagia for a month. He has been experiencing difficulty in swallowing both solid and liquid foods. He also developed bad breath, and sometimes he regurgitates swallowed food. His past medical history is significant for laryngeal cancer treated with surgical resection and radiotherapy ten years ago. He has a longterm history of peripheral artery disease, hypertension, and osteoarthritis of both knees. He takes lisinopril, ibuprofen, and simvastatin. He is allergic to penicillin. He never smoked cigarettes, and he does not drink alcohol. He appears malnourished, and his BMI is 19 kg/m2. The rest of the physical examination is unremarkable. He undergoes a barium swallow, which describes high grade narrowing at the mid esophagus and minimal contrast pass beyond the obstructing point. The narrowing measures about 3-4 cm long with a smooth passage. Which of the following is the most likely cause of dysphagia in this patient?
Choices: 1. 2. 3. 4.
Metastatic cancer of the esophagus Radiation-induced esophageal stricture Achalasia Pseudoachalasia
Answer: 2 - Radiation-induced esophageal stricture Explanations: Barium swallow shows a smooth narrowing of the middle part of the esophagus with tapering. This is a classic finding of esophageal stricture. The patient, with his previous exposure to radiation as part of his laryngeal cancer treatment, developed esophageal stricture due to mucosal damage from radiation exposure. Radiation-induced stricture usually occurs in the upper or middle part of the esophagus, and they tend to cause long smooth narrowing. The clinical presentation would be similar to any other stricture involving progressive dysphagia to solid and liquid. The clinical history is important and helps narrow the differential diagnosis and guide further management that often initially includes upper GI endoscopy and biopsy. Esophageal manometry may also be considered to evaluate for neuromuscular disorders of the esophagus that can result in similar symptoms in the absence of stricture. Go to the next page if you knew the correct answer, or click the link image(s) below to further research the concepts in this question (if desired).
Research Concepts: Esophageal Stricture
We update eBooks quarterly and Apps daily based on user feedback. Please tap flag to report any questions that need improvement.
Question 208:
A middle-aged African male presents with a great deal of difficulty swallowing solids that started about six weeks ago. His inability to eat has led to a weight loss of 7 pounds (3.2 kg) in four weeks. His other medical conditions include hypertension and hyperlipidemia, for which he takes nifedipine and rosuvastatin. There is no history of abdominal pain or diarrhea. However, the patient feels lethargic. Work up reveals that he has a granular cell tumor of the upper esophagus. The origin of this tumor is most likely from which of the following cell types?
Choices: 1. 2. 3. 4.
Squamous cells Schwann cells Muscle cells Papillary cells
Answer: 2 - Schwann cells Explanations: Granular cell tumors, also referred to as Abrikossoff tumors, are rare soft-tissue lesions whose origin is most likely neural or (from Schwann cells). The majority of granular cell tumors are benign but they can be locally aggressive and invade the adjacent tissues. Less than 2 percent are malignant, and this diagnosis confirms a poor prognosis since malignant lesions do not respond to chemotherapy or radiation. Most granular cell tumors are solitary and less than 4 cm in size. They tend to occur in the subcutaneous tissue but can sometimes occur in smooth muscle, submucosa, or striated muscle. They are most commonly found in the tongue but may occur in other parts of the aerodigestive tract. Go to the next page if you knew the correct answer, or click the link image(s) below to further research the concepts in this question (if desired).
Research Concepts: Granular Cell Tumor
We update eBooks quarterly and Apps daily based on user feedback. Please tap flag to report any questions that need improvement.
Question 209:
A 55-year-old male construction worker with no significant comorbid conditions presents with severe fatigue and bruising for the past two weeks. Labs show hemoglobin of 7.4 g/dL, white blood cell count of 28000/microL, and platelet count of 56000/microL. Peripheral smear shows numerous blasts with Auer rods. FISH for t(15;17) PML-RARA is positive. EKG shows normal sinus rhythm with normal QTc, and echocardiography shows an ejection fraction of 65% with no abnormalities. What is the most appropriate treatment regimen for this patient?
Choices: 1. Daunorubicin + cytarabine (7+3) 2. Arsenic trioxide (ATO) monotherapy 3. All-trans retinoic acid (ATRA) + ATO + gemtuzumab ozogamicin 4. Azacitidine + venetoclax
Answer: 3 - All-trans retinoic acid (ATRA) + ATO + gemtuzumab ozogamicin
Explanations: This patient is diagnosed with "high risk" acute promyelocytic leukemia (WBC count of more than 10,000/microL). He has normal cardiac function and normal QTc. He is a candidate for aggressive regimens like all-trans retinoic acid (ATRA) + arsenic trioxide (ATO) + gemtuzumab ozogamicin or ATRA + ATO + idarubicin. QTc monitoring and optimizing electrolytes are important for the safe administration of ATO. Go to the next page if you knew the correct answer, or click the link image(s) below to further research the concepts in this question (if desired).
Research Concepts: Acute Promyelocytic Leukemia
We update eBooks quarterly and Apps daily based on user feedback. Please tap flag to report any questions that need improvement.
Question 210:
A 50-year-old woman presents to the clinic for follow-up after having undergone a unilateral cordotomy. The patient currently works as a chef and has a history of metastatic breast cancer. Her vital signs are all within normal reference ranges. The patient states that while her somatic pain has improved, she has noticed other side effects of the procedure. Which of the following is the most appropriate precaution the patient must take given the interventional target of this procedure?
Choices: 1. The patient must be extra cautious when working near an open flame or hot objects 2. The patient must be extra careful when utilizing sharp knives 3. The patient must carefully handle raw seafood and other common sources of Vibrio cholerae 4. The patient must be extra cautious when lifting boxes heavier than 10 pounds (4.5 kg)
Answer: 1 - The patient must be extra cautious when working near an open flame or hot objects
Explanations: The lateral spinothalamic tract is the target of the cordotomy. The lateral spinothalamic tract is responsible for transmitting temperature sensation as well as pain and coarse touch. Depending on the characteristics of pain, unilateral and bilateral cordotomy are both options for consideration. Other cordotomy side effects include dysesthesia, urinary retention, ataxia, paresis, sympathetic dysfunction, sexual sensitivity impaired or lost, and a form of sleep apnea. Go to the next page if you knew the correct answer, or click the link image(s) below to further research the concepts in this question (if desired).
Research Concepts: Cordotomy
We update eBooks quarterly and Apps daily based on user feedback. Please tap flag to report any questions that need improvement.
Question 211:
A 30-year-old male with a history of uncontrolled hypertension presents for follow-up. He was recently admitted to the hospital with a hypertensive emergency and hypokalemia. He had a computed tomography scan of the abdomen which revealed a 2 cm benign-appearing right adrenal mass. His cortisol and urinary vanillylmandelic acid levels were normal, but he had an increased serum aldosterone to renin ratio. What is the next best step in management?
Choices: 1. 2. 3. 4.
Magnetic resonance imaging (MRI) angiography Right adrenalectomy Long-term spironolactone therapy Biopsy of the adrenal mass
Answer: 2 - Right adrenalectomy Explanations: The patient demonstrates hyperaldosteronism with the adrenal nodule. The patient can be directly referred for an adrenalectomy. In patients younger than 40 years of age with an adrenal adenoma and hyperaldosteronism, adrenal vein sampling is not necessary. Although he will require spironolactone in the preoperative period to control blood pressure, it is not recommended alone as a long-term therapy without surgery, especially in younger patients. However, medical management with spironolactone can be used for the long-term in patients who are poor surgical candidates or refuse surgery. A biopsy of the mass will provide no additional diagnostic information and is therefore not recommended. There is no need for angiography. Go to the next page if you knew the correct answer, or click the link image(s) below to further research the concepts in this question (if desired).
Research Concepts: Adrenal Adenoma
We update eBooks quarterly and Apps daily based on user feedback. Please tap flag to report any questions that
need improvement.
Question 212:
A 53-year-old female is diagnosed with stage 2 breast cancer that is positive for HER2 and negative for estrogen receptor (ER) and progesterone receptor (PR). She undergoes successful surgical excision of the primary tumor, and she begins neoadjuvant therapy with cyclophosphamide, 5-fluorouracil, paclitaxel, and trastuzumab. Three months after beginning this regimen, she presents to her oncologist with a complaint of progressive shortness of breath. It is worse with exertion and lying flat; on two occasions, she has awakened in the middle of the night gasping for breath. She has also noticed swelling of her ankles. Which of the following is the most likely cause of her new symptoms?
Choices: 1. 2. 3. 4.
Paclitaxel Trastuzumab 5-Fluorouracil Cyclophosphamide
Answer: 2 - Trastuzumab Explanations: Trastuzumab is associated with a decrease in left ventricular ejection fraction in a significant proportion of patients. This effect is most commonly subclinical and resolves upon cessation of therapy. Two to 7% of patients receiving trastuzumab will develop clinical symptoms of congestive heart failure. In patients receiving trastuzumab, the most significant risk factor for the development of cardiotoxicity is cotreatment with anthracyclines (e.g., doxorubicin, epirubicin). The main indication for discontinuation of trastuzumab is a decrease in left ventricular ejection fraction of 10% or more. Go to the next page if you knew the correct answer, or click the link image(s) below to further research the concepts in this question (if desired).
Research Concepts: Trastuzumab
We update eBooks quarterly and Apps daily based on user feedback. Please tap flag to report any questions that need improvement.
Question 213:
A 65-year-old Hispanic female patient presents to the hospital with several months' history of warmth, erythema, and a burning sensation on the bilateral lower extremities. On examination, she is found to have lymphadenopathy and hepatosplenomegaly. Biopsy of the skin demonstrates arteriolar fibrosis and occlusion with platelet thrombi sparing the venules, capillaries, and nerves. What underlying disease has most commonly been associated with this condition?
Choices: 1. 2. 3. 4.
Hypertension Myeloproliferative disorder Chemotherapy-induced peripheral neuropathy Dermatomyositis
Answer: 2 - Myeloproliferative disorder Explanations: Erythromelalgia is a rarely occurring disease entity characterized by a triad of erythema, warmth, and recurrent burning pain most notably affecting the extremities. Myeloproliferative disorders are most commonly associated with secondary erythromelalgia. Erythromelalgia can present years before the diagnosis of a myeloproliferative disorder is made. Hypertension is not commonly associated with erythromelalgia. Chemotherapy can cause a peripheral neuropathy but is not the cause of erythromelalgia. Dermatomyositis is not associated with erythromelalgia. Go to the next page if you knew the correct answer, or click the link image(s) below to further research the concepts in this question (if desired).
Research Concepts: Erythromelalgia
We update eBooks quarterly and Apps daily based on user feedback. Please tap flag to report any questions that need improvement.
Question 214:
A 70-year-old white male status post radiation therapy (6600 cGy) for prostate cancer 5 years previously has developed rectal bleeding and discomfort thought to be secondary to radiation proctitis. The patient has not responded to sucralfate or sulfasalazine. What is the most appropriate next step in this patient's evaluation and treatment?
Choices: 1. 2. 3. 4.
Colonoscopy Rectal biopsy Hyperbaric oxygen therapy Diverting colostomy
Answer: 3 - Hyperbaric oxygen therapy Explanations: Hyperbaric oxygen therapy promotes angiogenesis and repair of damaged rectal tissue and should be used prior to any surgical intervention. The diagnosis of radiation proctitis is made clinically. Colonoscopy is relatively contraindicated initially as this can result in perforation. Rectal biopsy is relatively contraindicated due to the danger of causing complications such as erosion or fistula. Sucralfate enema can be useful for symptom relief but is not curative. Sulfasalazine can be used orally or per rectum. Go to the next page if you knew the correct answer, or click the link image(s) below to further research the concepts in this question (if desired).
Research Concepts: Hyperbaric Treatment Of Radiation Proctitis
We update eBooks quarterly and Apps daily based on user feedback. Please tap flag to report any questions that need improvement.
Question 215:
A 5-year-old female with a past medical history of atopic dermatitis, asthma, and osteosarcoma that is admitted to the hospital is being evaluated for a fever of unknown origin. The patient is currently undergoing chemotherapy for her osteosarcoma but otherwise has no other complaints. She reports taking an albuterol inhaler as needed for asthma attacks and uses a ceramide lotion for her atopic dermatitis per mother at the bedside. On physical exam, her vitals are within normal limits except a temperature of 102.1 F. Physical exam is otherwise unremarkable. Lab work is ordered. Two days later, the patient develops an asymptomatic, pink papular rash located on her trunk and extremities. However, the fever has resolved. Upon further investigation, it is discovered that the patient had a similar rash three years ago. What is the most likely explanation for the child's rash?
Choices: 1. Contact dermatitis from the ceramide lotion 2. Allergic reaction from the chemotherapy for her osteosarcoma 3. Reactivation of a latent virus in her central nervous system (CNS) 4. Exacerbation of atopic dermatitis
Answer: 3 - Reactivation of a latent virus in her central nervous system (CNS)
Explanations: The rash of roseola infantum is typically asymptomatic and rarely requires the application of topical agents. Roseola infantum presents as multiple, pink papules beginning on the trunk and then spreading to the neck and extremities. The virus causing roseola infantum lies dormant in the CNS after the initial infection. During episodes of immunosuppression, it is capable of reactivating and causing a similar, more severe fever and rash. It is important to obtain a thorough history from the patient and family of any previous skin diseases including rashes as a child. Go to the next page if you knew the correct answer, or click the link image(s) below to further research the concepts in this question (if desired).
Research Concepts: Roseola Infantum
We update eBooks quarterly and Apps daily based on user feedback. Please tap flag to report any questions that need improvement.
Question 216:
A 57-year-old female with an unremarkable past medical history is found to have Bence Jones proteins in her urine through gel electrophoresis when being screened for multiple myeloma. The consulting pathologist recommends a follow-up study to confirm the presence of Bence Jones proteins in this patient’s urine. Which of the following is a drawback that must be considered with this particular follow-up study?
Choices: 1. 2. 3. 4.
Demanding and difficult technique High cost Possibility of light chain ladders Minimal available antigen
Answer: 3 - Possibility of light chain ladders Explanations: Follow-up immunofixation electrophoresis after gel electrophoresis is the most sensitive measure of Bence Jones proteinuria. Polyclonal light chains, usually kappa, can produce characteristic banding proteins called light chain ladders after immunofixation electrophoresis. Bence Jones proteins can co-migrate with the bands in these ladders. These light chain ladders themselves are not Bence Jones proteins. Immunofixation electrophoresis is conducted with antisera to IgG, IgA, IgM, total kappa, and total lambda. The issue of antigen excess can arise with immunofixation electrophoresis. This can be easily overcome through decreased initial loading of antigen. Go to the next page if you knew the correct answer, or click the link image(s) below to further research the concepts in this question (if desired).
Research Concepts: Bence-Jones Protein
We update eBooks quarterly and Apps daily based on user feedback. Please tap flag to report any questions that need improvement.
Question 217:
A 75 year-old-male with a history of diabetes, COPD, and tobacco abuse is diagnosed with leftsided bronchogenic carcinoma. Pulmonary function tests reveal a very poor reserve with predicted postoperative FEV1 and DLCO of 20%. Cardiopulmonary exercise test with measurement of maximal oxygen consumption is done to assess fitness for surgery. What is the cut-off value of maximal oxygen consumption below which other nonsurgical options should be considered?
Choices: 1. 2. 3. 4.
40 30 20 10
ml/kg/min ml/kg/min ml/kg/min ml/kg/min
Answer: 4 - 10 ml/kg/min Explanations: Pulmonary function test is usually done as a part of the preoperative workup in all patients undergoing pneumonectomy. FEV1 and DLCO provide the most accurate predictors of postoperative morbidity and mortality. Patients are usually divided into three categories, those with predicted postoperative FEV1, DLCO more than 60%, between 30-60% and less than 30%. Predicted postoperative FEV1, DLCO less than 30% is considered high risk and formal cardiopulmonary exercise test with measurement of maximal oxygen consumption (VO max) should be done for further risk stratification. VO max more than 20 ml/kg/min is considered acceptable for postoperative complications. Values less than 10 ml/kg/min is considered high risk and nonsurgical modalities should be pursued. Go to the next page if you knew the correct answer, or click the link image(s) below to further research the concepts in this question (if desired).
Research Concepts: Pneumonectomy
We update eBooks quarterly and Apps daily based on user feedback. Please tap flag to report any questions that need improvement.
Question 218:
A 62-year-old female presents to the hospital with right upper quadrant pain and nausea. Her physical examination is significant for hepatomegaly. Vital signs are within normal limits. Labs are significant for ALT 75 IU/L and AST 70 IU/L with ALP 160 U/L and bilirubin levels of 1.5 mg/dL. CT abdomen with contrast shows a 5 cm lesion with enhancement after contrast. Liver biopsy shows evidence of solid spindle cell proliferation without hemorrhage. Immunohistochemistry is positive for CD31 and CD34. PET/CT is negative for distant local or distant metastasis. What is the best next step in the management of this patient?
Choices: 1. 2. 3. 4.
Chemotherapy Liver transplantation Surgical resection Radiation
Answer: 3 - Surgical resection Explanations: This patient likely has angiosarcoma of the liver. Angiosarcoma usually presents with nonspecific symptoms including abdominal pain and ascites or can be asymptomatic and found incidentally on imaging. It is associated with elevations in liver enzymes as well as thrombocytopenia and anemia if the tumor burden is large and disease is extensive. Imaging usually shows a hypodense lesion or multiple lesions on non-enhanced CT with variable enhancement on contrast CT based on the presence of hemorrhage and necrosis. Diagnosis is usually confirmed with histopathology showing spindle cells with possible areas of necrosis, infarction, and hemorrhage. Single or multifocal lesions are usually managed by complete hepatic resection or radical resection of the tumor. Regional or distant metastasis makes resection less likely possible, and chemotherapy can be tried; however, no effective chemotherapy has been established. Liver transplantation is not recommended due to the high rate of recurrence and the rapid progression of the disease. Chemotherapy can be considered in metastatic disease; however, no clear regimen or guidelines are available and has not proven to be effective in treatment. Liver transplantation has not been proved to be effective in the management of liver angiosarcoma. Radiation therapy has limited use in liver angiosarcoma. Go to the next page if you knew the correct answer, or click the link image(s) below to further research the concepts in
this question (if desired).
Research Concepts: Liver Angiosarcoma
We update eBooks quarterly and Apps daily based on user feedback. Please tap flag to report any questions that need improvement.
Question 219:
A 1-year-old girl is brought to the clinic with a hairy plaque of the right thigh. Physical examination shows a well-limited skin-colored plaque of 5 cm, covered with villous hair. What is the most likely diagnosis?
Choices: 1. 2. 3. 4.
Congenital smooth muscle hamartoma Acquired smooth muscle hamartoma Congenital melanocytic nevus Becker hamartoma
Answer: 1 - Congenital smooth muscle hamartoma Explanations: The classic form of the congenital smooth muscle hamartoma presents as a well-limited skin-colored or pigmented plaque covered with villous hair. The diagnosis of congenital smooth muscle hamartoma is usually made at the early infancy. The congenital smooth muscle hamartoma is a common condition and the diagnosis should be considered in front of any congenital hairy plaque. Unlike smooth muscle hamartoma, congenital melanocytic nevus is necessarily a pigmented lesion. Go to the next page if you knew the correct answer, or click the link image(s) below to further research the concepts in this question (if desired).
Research Concepts: Congenital Smooth Muscle Hamartoma
We update eBooks quarterly and Apps daily based on user feedback. Please tap flag to report any questions that need improvement.
Question 220:
A 65-year-old male patient presents to the hospital for routine follow-up of hypertension and to discuss smoking cessation. He has smoked a pack of cigarettes a day since he was in high school. Today, he is complaining of swelling and pain in both wrists. No other joints are affected. He denies injury, fever, morning stiffness, and rashes. He also reports a 20-pound (9 kg) weight loss in the last three months and denies shortness of breath, chest pain, or hemoptysis. He does not have any gastrointestinal or genitourinary symptoms. His blood pressure is well controlled. The physical exam is remarkable for palpable effusion in both wrists without evidence of synovial inflammation, and limited range of motion due to pain. There is no small joints tenderness. He has gross clubbing of the fingers in both hands. Which of the following is the most appropriate next step in the evaluation of this patient?
Choices: 1. 2. 3. 4.
Rheumatoid factor ANA Joint aspiration with synovial fluid analysis Chest X-ray
Answer: 4 - Chest X-ray Explanations: This patient's presentation is most consistent with hypertrophic osteoarthropathy, most likely secondary to a pulmonary malignancy. Given the patient's smoking history and recent weight loss, a chest x-ray would be the best test to screen for pulmonary neoplasm. Hypertrophic osteoarthropathy (HOA), consists of the presence of digital clubbing, increased the periosteal activity of the tubular bones, arthralgias, and joint effusion and is characterized by abnormal proliferation of the skin, soft tissues and osseous tissues in the distal parts of extremities. There are two forms of the disease - primary and secondary. Primary HOA (PHO), also called pachydermoperiostosis, comprises of digital clubbing, periostosis, and pachydermia and is a rare genetic disease, while secondary hypertrophic osteoarthropathy is associated with a variety of pulmonary, cardiac and other conditions. ANA would be the correct answer if SLE were suspected. Given this patient's weight loss, smoking history, and lack of rash and morning stiffness, the more likely diagnosis is secondary hypertrophic osteoarthropathy. This patient has bilateral arthropathy, making gout and septic arthritis less likely. The rheumatoid factor would be more appropriate as a screen for rheumatoid arthritis (RA). While RA may present with wrist effusion and pain, it would be extremely unlikely for RA to be the cause of his distal joint swelling and pain. He also denies morning stiffness, a hallmark of rheumatoid arthritis.
Go to the next page if you knew the correct answer, or click the link image(s) below to further research the concepts in this question (if desired).
Research Concepts: Hypertrophic Osteoarthropathy
We update eBooks quarterly and Apps daily based on user feedback. Please tap flag to report any questions that need improvement.
Question 221:
A 35-year-old male presents to the clinic for a lump below his left axilla. He first noticed it two months ago, but it has gradually been increasing in size. He has no known medical problems and takes no medications. He also denies smoking or using illicit substances. Physical examination reveals a firm growth barely palpable in the left midaxillary line. A CT of the chest reveals a 4 cm rounded growth in the chest wall muscles. Which of the following is the most appropriate next step in the management of this patient?
Choices: 1. 2. 3. 4.
Needle aspiration Incisional biopsy Excisional biopsy Radiotherapy
Answer: 3 - Excisional biopsy Explanations: Lesions measuring less than five centimeters undergo an excisional biopsy. Lesions greater than five centimeters either undergo needle aspiration or incisional biopsy. Radiologic evidence is not sufficient to make a confirmatory diagnosis. Histopathologic diagnosis is necessary to confirm the diagnosis. The size of the lesion determines the type of biopsy. Go to the next page if you knew the correct answer, or click the link image(s) below to further research the concepts in this question (if desired).
Research Concepts: Chest Wall Tumors
We update eBooks quarterly and Apps daily based on user feedback. Please tap flag to report any questions that need improvement.
Question 222:
A 63-year-old male with a past medical history of hyperlipidemia and gastroesophageal reflux disease presents with progressively worsening pancytopenia over the past 2 years. His absolute neutrophil count is 800/microL, hemoglobin is 8.9 g/dL and platelet count is 104,000/microL. A bone marrow biopsy reveals 13% blasts and features consistent with myelodysplastic syndrome. Cytogenetics reveals inversion in chromosome 16. The patient is negative for FLT3, NPM1, CEBPA, and RUNX1 genes. What is the best initial therapy for this patient?
Choices: 1. Observation 2. Erythropoietin +/- G-CSF 3. Azacitidine 4. Induction chemotherapy (7+3) cytarabine and an anthracycline
Answer: 4 - Induction chemotherapy (7+3) cytarabine and an anthracycline
Explanations: The myelodysplastic syndrome (MDS) typically has blasts of less than 20%. Patients are considered to have acute myeloid leukemia (AML) when the blasts are 20% or more. However, in MDS with certain cytogenetics that involves core-binding factors, the patient should be treated similarly to AML with induction chemotherapy. Any patient with MDS and one of the following are considered to have AML regardless of blast count: inversion 16, t(16;16), and t(8;21). The above cytogenetics are considered favorable risk cytogenetics in AML, along with t(15;17), seen in acute promyelocytic leukemia. The patient should be treated with conventional induction chemotherapy (cytarabine + anthracycline such as daunorubicin or idarubicin) and receive consolidation therapy with high dose cytarabine, based on his cytogenetics and good performance status. Erythropoietin +/- G-CSF and azacitidine are standard treatment regimens for MDS without these cytogenetics. Go to the next page if you knew the correct answer, or click the link image(s) below to further research the concepts in this question (if desired).
Research Concepts: Myelodysplastic Syndrome
We update eBooks quarterly and Apps daily based on user feedback. Please tap flag to report any questions that need improvement.
Question 223:
A 72-year-old female presents with a 3month long history of increasing dyspnea on exertion as well cough productive of green sputum. She has also experienced a 15-pound (7 kg) weight loss and recently began to notice specks of blood in her sputum. Social history is pertinent for a 50 pack-year history of smoking. Chest x-ray reveals a mass in the left lower lobe. Plasma osmolarity is 250 mOsm/L, plasma sodium of 127 mEq/L, urine osmolarity is 400 mOsm/L, and urine sodium is 45 mEq/L. Which of the following is most likely to be seen on biopsy of the lung mass?
Choices: 1. Small, dark blue cells that stain positive for chromogranin A 2. Cells arranged in a glandular pattern that stains positive for mucin 3. Pleomorphic giant cells 4. Keratin pearls and intercellular bridges
Answer: 1 - Small, dark blue cells that stain positive for chromogranin A
Explanations: This woman likely has syndrome of inappropriate antidiuretic hormone secretion. SIADH is diagnosed in a patient with euvolemic hyponatremia who has a plasma osmolarity less than 275 mOsm/L, urine osmolarity greater than 100 mOsm/L, and urine sodium greater than 40 mEq/L. SIADH is frequently a paraneoplastic phenomenon. This patient likely has lung cancer given her history of smoking, dyspnea, hemoptysis, and cough as well as chest x-ray findings. SIADH is most commonly associated with small cell lung cancer. This cancer is marked by positive chromogranin A staining as well as the appearance of small, blue cells. Cells arranged in a glandular pattern staining positive for mucin is an adenocarcinoma. Pleomorphic giant cells describe a large cell cancer. Keratin pearls and intercellular bridges are seen in squamous cell cancers. A variety of other things may also cause SIADH. This includes nausea, pain, medications, infections, brain injury, and others. Notable medications include carbamazepine and SSRI class of drugs. Go to the next page if you knew the correct answer, or click the link image(s) below to further research the concepts in this question (if desired).
Research Concepts: Small Cell Lung Cancer
We update eBooks quarterly and Apps daily based on user feedback. Please tap flag to report any questions that need improvement.
Question 224:
A 16-year-old male diagnosed with acute lymphoblastic leukemia presents to the emergency department with cough, brownish sputum, and fever. He has a history of bronchial asthma, which was previously well controlled with regular inhaled bronchodilators. He has never had such an episode before and believes that his cancer is spreading to his lungs now. He is taking oral chemotherapy medications and has shown a good response to therapy but has had prolonged neutropenia. Chest x-ray shows an infiltrate on the right lower lung zone. His vital signs are normal. Which of the following organisms is most likely responsible for the patient's condition?
Choices: 1. 2. 3. 4.
Streptococcus pneumoniae Aspergillus fumigatus Staphylococcus aureus Mycoplasma pneumoniae
Answer: 2 - Aspergillus fumigatus Explanations: Aspergillus species are causative organisms for fungal infection in immunocompromised individuals. Aspergillus fumigatus is the most common ubiquitous airborne fungus responsible for allergic bronchopulmonary aspergillosis (ABPA) in immunocompromised patients. Long duration of neutropenia and high dose corticosteroids are risk factors. Aspergillus fumigatus, a widely distributed sporebearing fungus, causes multiple diseases in humans. These diseases include aspergilloma, different forms of hypersensitivity diseases, including ABPA, allergic bronchial asthma, bronchocentric granulomatosis, extrinsic allergic alveolitis, and invasive pulmonary aspergillosis. A. fumigatus has a very small spore (conidia) size (2-3 microns), which enables the spores to penetrate deeply into the lung. Spores are capable of withstanding extraordinary atmospheric conditions. The pigment of conidia also confers some protection against phagocytosis. In immunocompetent individuals, Aspergillus conidia are easily eliminated from the airway by innate immune system mechanisms, so no pulmonary fungal infection manifests. In immunocompromised individuals, Aspergillus conidia are not eliminated due to host immune defense imbalance, so they colonize airways, germinate into somatic hyphae and stimulate a chronic allergic inflammatory response.
Go to the next page if you knew the correct answer, or click the link image(s) below to further research the concepts in this question (if desired).
Research Concepts: Allergic Bronchopulmonary Aspergillosis
We update eBooks quarterly and Apps daily based on user feedback. Please tap flag to report any questions that need improvement.
Question 225:
A 2-year-old boy is brought to the clinic with a hairy plaque on the left shoulder. Physical examination reveals a well-demarcated pigmented plaque, covered with dark hair. Histological examination of skin biopsy demonstrates badly-defined smooth muscle bundles, which are randomly oriented. These bundles intermingle with collagen fibers. Based on clinical and histological findings, what is the most likely diagnosis?
Choices: 1. 2. 3. 4.
Becker hamartoma Piloleimyoma Congenital smooth muscle hamartoma Nevus pilosus
Answer: 2 - Piloleimyoma Explanations: Piloleimyoma resembles clinically a congenital smooth muscle hamartoma. Both conditions are seen in children Histological examination is necessary to differentiate both conditions. Unlike piloleimyoma, the smooth muscle bundles of the congenital smooth muscle hamartoma are well-defined and are separated from surrounding collagen fibers by a characteristic clear space. Go to the next page if you knew the correct answer, or click the link image(s) below to further research the concepts in this question (if desired).
Research Concepts: Congenital Smooth Muscle Hamartoma
We update eBooks quarterly and Apps daily based on user feedback. Please tap flag to report any questions that need improvement.
Question 226:
A 62-year-old male with no significant past medical history presents to the clinic for fatigue for the past two months. He said he used to be overweight, but now his BMI is below normal. His conjunctivae are yellow. The lower border of the liver is barely palpable in the right hypochondrium. He has no tenderness to palpation. His basic metabolic panel results show a mildly elevated creatinine level. Which of the following is the best next test?
Choices: 1. 2. 3. 4.
noncontrast CT of the abdomen Hepatic enzymes Erythrocyte sedimentation rate Ultrasound of the abdomen
Answer: 4 - Ultrasound of the abdomen Explanations: The patient most likely has a pancreatic mass, occluding the biliary duct leading to jaundice and the portal vein leading to splenomegaly. An abdominal ultrasound is indicated as the initial test to confirm a biliary tract obstruction, which may be followed by a contrastenhanced CT or MRI as a confirmatory test for a diagnosis of cancer. Abdominal ultrasound is the most often recommended best initial test while investigating hepatic, pancreatic, and biliary malignancies. Lack of contrast administration on CT is likely to prevent a confident diagnosis of pancreatic cancer and is not a better screening test than ultrasound for this clinical scenario. Doppler ultrasound is superior to normal ultrasound and contrast-enhanced ultrasound is superior to Doppler ultrasound for the detection of portal vein thrombosis. Possible ultrasound findings include increased portal vein diameter, a hyperechoic shadow in the portal vein and dilated CBD (common bile duct) as seen in this patient due to pancreatic cancer. In a patient with no apparent cause for the thrombus, recommended blood tests should encompass extensive procoagulant factors workup, including antiphospholipid syndrome, protein C, S, antithrombin III levels, factor V and Leiden mutation. Go to the next page if you knew the correct answer, or click the link image(s) below to further research the concepts in this question (if desired).
Research Concepts: Portal Vein Obstruction
We update eBooks quarterly and Apps daily based on user feedback. Please tap flag to report any questions that need improvement.
Question 227:
A 60-year-old man presents with a twoweek history of fever and axillary lymphadenopathy. His past medical history is significant for well-controlled hypertension. His blood pressure is 123/76 mmHg, heart rate 109/min, respiratory rate 18/min, and temperature 100.4 F (38 C). Physical examination demonstrates palpable, bilateral, nontender axillary adenopathy and is negative for hepatosplenomegaly and skin rash. Analysis of an axillary lymph node biopsy shows a sheet-like infiltrate of neoplastic cells, some of which are large with horseshoeshaped nuclei ("hallmark cells"). Immunohistochemistry is positive for CD30, CD4, CD5, CD7, CD43, and negative for ALK, CD15, CD19, CD20. Epstein-Barr encoding region (EBER) in situ hybridization is negative. What is the most likely diagnosis?
Choices: 1. 2. 3. 4.
Anaplastic large B-cell lymphoma Sezary syndrome Large T-cell lymphoma Lymphomatoid papulosis
Answer: 3 - Large T-cell lymphoma Explanations: ALK-negative anaplastic large cell lymphoma (ALKALCL) is a rare subtype of CD30+ large T-cell lymphoma. The characteristic horseshoe-shaped nucleated cells are present in all patients with ALK- ALCL in variable amounts. FISH studies for ALK rearrangement are unnecessary if ALK immunohistochemistry is negative. T-cell lymphomas are negative for CD19 and CD20, in contrast to B-cell lymphomas. Sezary syndrome and lymphomatoid papulosis are usually associated with some form of skin involvement. Go to the next page if you knew the correct answer, or click the link image(s) below to further research the concepts in this question (if desired).
Research Concepts: ALK Negative Anaplastic Large Cell Lymphoma
We update eBooks quarterly and Apps daily based on user feedback. Please tap flag to report any questions that need improvement.
Question 228:
A patient past her reproductive years has carcinoma in situ of the cervix with cone biopsy showing squamous cell carcinoma with invasion 1 mm beyond the basement membrane. There is no lymphovascular space invasion. What is the next best step?
Choices: 1. 2. 3. 4.
Treatment with external beam radiation Implantation of radioactive cesium into the cervical canal Simple hysterectomy Simple hysterectomy with pelvic lymphadenectomy
Answer: 3 - Simple hysterectomy Explanations: Microinvasive carcinoma of the cervix includes lesions within 3 mm of the base of the epithelium, with no confluent tongues or lymphatic or vascular invasion. This is a T1a lesion, and the treatment of choice is surgery. Both hysterectomy and conization are acceptable procedures. Lymph node dissection is not indicated as the depth of invasion is less than 3 mm, and there is no lymphovascular space invasion. For women who would like to maintain fertility, conization is an option with close follow-up. Those patients not suitable for surgery can be offered radiation. Go to the next page if you knew the correct answer, or click the link image(s) below to further research the concepts in this question (if desired).
Research Concepts: Cervical Cancer
We update eBooks quarterly and Apps daily based on user feedback. Please tap flag to report any questions that need improvement.
Question 229:
A 27-year-old female presents to the emergency department with a one-day history of left lower abdominal pain that has moved to the right lower abdomen. She reports a complete loss of appetite and has a fever of 38.3 C (101.0 F). A urine pregnancy test is negative, and ultrasound reveals an enlarged appendix with thick walls and peri-appendiceal fluid. She undergoes laparoscopic appendectomy, and pathology comes back revealing mucinous adenocarcinoma with positive margins. Staging imaging reveals diffuse lesions throughout the peritoneum. What treatment protocol would best minimize tumor burden?
Choices: 1. Surgical debulking alone 2. Systemic chemotherapy alone 3. Combination of surgical debulking and hyperthermic intraperitoneal chemotherapy 4. Combination of surgical debulking and systemic chemotherapy
Answer: 3 - Combination of surgical debulking and hyperthermic intraperitoneal chemotherapy
Explanations: Pseudomyxoma peritonei is characterized by diffuse lesions of mucin on peritoneal surfaces. It can arise from appendiceal mucinous adenocarcinoma. While still being studied, hyperthermic intraperitoneal chemotherapy is showing to be efficacious at reducing tumor burden in diffuse peritoneal disease. Surgical debulking is also used to resect existing mucinous implants present on peritoneal surfaces. Due to the visceral and parietal innervation of the peritoneum, pain in appendicitis can start as vague or diffuse pain or left-sided pain that later localizes to the right as symptoms worsen. That is due to visceral innervation causing vague symptoms and later triggering the parietal peritoneum, which results in localizable symptoms. Appendicitis can present as anorexia, vague abdominal pain that changes in location, and fever. Leukocytosis is commonly seen as well on studies. While it can be a clinical diagnosis, imaging is often used to confirm the diagnosis, and either ultrasound or CT scan can be performed. Imaging results can reveal thickened walls, peri-appendiceal fluid, fat-stranding, abscess formation, or appendiceal perforation. In females, it is important to rule out pregnancy and consider gynecologic causes for lower abdominal pain in the differential diagnosis. Go to the next page if you knew the correct answer, or click the link image(s) below to further research the concepts in this question (if desired).
Research Concepts: Pseudomyxoma Peritonei
We update eBooks quarterly and Apps daily based on user feedback. Please tap flag to report any questions that need improvement.
Question 230:
A 65-year-old female is found to have a 4 cm left breast mass. She undergoes a biopsy, which reveals an invasive ductal carcinoma. She then undergoes a lumpectomy with sentinel node mapping. The pathology reveals that the sentinel node is negative, but the tumor is seen "on ink" at multiple margins. The tumor is ER and PR negative but HER2 positive. Which of the following is the next step?
Choices: 1. Start radiation therapy, followed by trastuzumab 2. Reoperate to obtain clear margins with either breast conservation or mastectomy 3. Recommend radiation therapy only 30 days after surgery 4. Start chemotherapy ASAP and follow up with radiation therapy, no further surgery is needed.
Answer: 2 - Reoperate to obtain clear margins with either breast conservation or mastectomy
Explanations: For all cancers, when the margins are positive or "on ink," the risk of recurrence is very high. Reoperation should be performed to obtain clear margins prior to starting radiation therapy. This may require a mastectomy in many patients if multiple margins are positive, and an adequate cosmetic outcome is unable to be achieved. Radiation may not be needed if mastectomy is performed pending final tumor size and nodal status. Once negative margins have been obtained, adjuvant treatments are appropriate. While radiation may be initiated in those who underwent breast-conserving therapy, usually 3-4 weeks after surgery to allow for the surgical wound to heal, radiation is generally given after adjuvant chemotherapy is completed, which is generally recommended in this case. Delaying radiation therapy increases the risk of death. Go to the next page if you knew the correct answer, or click the link image(s) below to further research the concepts in this question (if desired).
Research Concepts: Breast Cancer Conservation Therapy
We update eBooks quarterly and Apps daily based on user feedback. Please tap flag to report any questions that need improvement.
Question 231:
A 65-year-old male presents with an enlarging, painless thyroid nodule. Bloodwork shows an elevated thyroid stimulating hormone level and is positive for anti-thyroid peroxidase and anti-thyroglobulin antibodies. Histological exam shows dense lymphoplasmacytic infiltrate with the formation of lymphoid follicles. What is this patient at risk for developing?
Choices: 1. 2. 3. 4.
Thyroid storm Hypocalcemia Lymphoma Thyroid cancer
Answer: 3 - Lymphoma Explanations: HLA-DR5 is implicated in Hashimoto thyroiditis. Many patients have a family history of the disease. Different ethnic groups have different genes linked to the disease. It is more common in patients with Down, Klinefelter, and Turner syndromes. Autoantibodies including anti-thyroglobulin, anti-thyroid peroxidase, and anti-TSH receptor are seen in Hashimoto thyroiditis. Non-Hodgkin lymphoma of the thyroid is a possible complication. Go to the next page if you knew the correct answer, or click the link image(s) below to further research the concepts in this question (if desired).
Research Concepts: Hashimoto Thyroiditis
We update eBooks quarterly and Apps daily based on user feedback. Please tap flag to report any questions that need improvement.
Question 232:
A 67-year-old man presents to the otolaryngology outpatient setting with three months of throat discomfort and progressive trismus. He is a current smoker of 40 pack-years and suffers from chronic obstructive pulmonary disease. Physical examination reveals a large lesion within his left tonsil and large bilateral cervical lymphadenopathy. Cross-sectional imaging reveals a large lesion arising from the left tonsil invading the medial pterygoid muscle and several enlarged bilateral cervical lymph nodes, the largest of which measures 5cm. There is no distant metastasis. Fine needles aspiration (FNA) sampling of the neck node is p16 negative. What is the TNM staging according to the American Joint Committee on Cancer 8th edition?
Choices: 1. 2. 3. 4.
T3N2aM0 T3N2cM0 T4aN2aM0 T4aN2cM0
Answer: 4 - T4aN2cM0 Explanations: The revised 8th edition of the TNM classification of oropharyngeal cancer splits the disease into p16 positive and p16 negative disease. This is a p16 negative case and has a tonsil cancer invading the medial pterygoid muscle, making this T4a disease. He also has bilateral cervical nodes measuring less than 6cm and therefore has N2c nodal involvement. Advanced tonsillar p16 negative tonsillar cancer is usually seen in older patients who are smokers and has less favorable outcomes compared to p16 positive disease. Advanced tonsil cancer is typically managed with radiotherapy and either platinum-based chemotherapy cisplatin or monoclonal antibody cetuximab. Surgical intervention for advanced tonsillar cancer is technically very difficult, can cause significant postoperative comorbidity, and often requires postoperative chemoradiotherapy. Go to the next page if you knew the correct answer, or click the link image(s) below to further research the concepts in this question (if desired).
Research Concepts: Tonsil Cancer
We update eBooks quarterly and Apps daily based on user feedback. Please tap flag to report any questions that need improvement.
Question 233:
A 50-year-old male presents to the clinic with a chief complaint of a mole on his right calf that has increased in size over the past year. He works in maintaining swimming pools. His medical history includes hypertension that is controlled with losartan and hydrochlorothiazide. Lower extremity physical examination reveals palpable pedal and popliteal pulses with intact sensation bilaterally. His flexor and extensor muscle strengths are +5/5 bilaterally. His skin and nails are clear except for a 1 cm black mole on the posterior aspect of his right calf. The mole is comma shaped with a notched border but is not bleeding. Which of the following is the next best step in the management of this patient?
Choices: 1. Perform a 3 mm punch biopsy 2. Reassure, photograph and plan follow up examination in one year 3. Perform a superficial shave biopsy of the entire lesion 4. Plan wide excision with a narrow border of normal skin
Answer: 4 - Plan wide excision with a narrow border of normal skin
Explanations: Benign nevi represent collections of melanocytes within the epidermis and or dermis. A Wood’s light lamp is useful to help outline and delineate pigmented lesions. Partial excision of a nevus can lead to lead to misdiagnosis, inaccurate staging, and increased mortality. Dysplastic nevi are histologically rated as mild, moderate, or severe. Go to the next page if you knew the correct answer, or click the link image(s) below to further research the concepts in this question (if desired).
Research Concepts: Melanocytic Nevi
We update eBooks quarterly and Apps daily based on user feedback. Please tap flag to report any questions that need improvement.
Question 234:
A 72-year-old male is admitted to the medicine floors after he presented to the hospital with poor oral intake and a 25-pound (11 kg) unintentional weight loss for the past three months. Social history includes a 40 packyear history, and he was a retired PVC factory worker. Physical examination is remarkable for hepatomegaly, and labs reveal ALT 100 IU/L, AST 110 U/L, ALP 215 U/L, and total bilirubin 3 mg/dL. Triple-phase CT shows multiple hypodense liver lesions, with some lesions becoming hyperdense after contrast is administered. Liver biopsy shows solid spindle cell proliferation and necrosis. What is the most effective treatment of this patient's liver condition?
Choices: 1. 2. 3. 4.
Tumor resection Chemotherapy Radiation therapy Liver transplant
Answer: 1 - Tumor resection Explanations: Resection of the tumor or complete resection is the best way to treat hepatic angiosarcoma (HAS) though only a few achieve complete resection due to the aggressive nature of the tumor. Effective chemotherapeutic agents have not been established for the treatment of HAS. Chemotherapy is tried in unresectable disease. Only a few cases have shown positive results with using 5-FU-carboplatin with doxorubicin or ifosfamide in unresectable HAS. Liver transplant is not recommended due to high recurrence and progression of the malignancy. Go to the next page if you knew the correct answer, or click the link image(s) below to further research the concepts in this question (if desired).
Research Concepts: Liver Angiosarcoma
We update eBooks quarterly and Apps daily based on user feedback. Please tap flag to report any questions that need improvement.
Question 235:
A 71-year-old female with a past medical history of hypertension and presents to the clinic complaining of vague abdominal pain for one week. She denies any nausea, vomiting, or bowel habit changes. Her temperature is 37 degrees C, heart rate 80 bpm, blood pressure 124/86 mm Hg, and respiratory rate 16 breaths per minute. Her abdomen is moderately distended with no tenderness, guarding, or rigidity, but there is dullness to percussion with a positive fluid shift test. Computed tomography scan showed a moderate amount of fluid in the abdomen with multiple small nodules involving the anterior surface of the peritoneum and lateral aspects of the abdominal wall. There is a right ovarian mass. There are no lesions found in the liver, lungs, or any other abdominal organ. Diagnostic paracentesis was performed and showed the following fluid analysis: gross appearance: milky, white fluid; total protein: 31 g/L; serum ascites albumin gradient or gap (SAAG): 0.9 g/dl; CA 125: 78 U/ml; LDH: 525 SU (sigma unit); cytology: Presence of ovarian epithelial cancer cells; and white blood cell: 200 /ul. Which of the following treatment options would provide the greatest benefit in terms of the best 5-year survival prognosis?
Choices: 1. Systemic chemotherapy in combination with continuous ascetic fluid drainage 2. Cytoreductive surgery of all peritoneal lesions in combination with hyperthermic intraperitoneal chemotherapy 3. Radiation therapy 4. Cytoreductive surgery of all visible peritoneal lesions in combination with focused abdominal radiation therapy
Answer: 2 - Cytoreductive surgery of all peritoneal lesions in combination with hyperthermic intraperitoneal chemotherapy
Explanations: Ovarian cancer accounts for 11.6 new cases per 100,000 women per year in the United States. It has a high tendency to invade the peritoneum and surrounding tissue due to the close anatomic proximity of ovaries to peritoneal surfaces and due to the similar mesodermal embryonic origin of cells. About half of the patients with ovarian cancer and peritoneal metastasis presents with ascites in the later stage of the disease, but draining the ascitic fluid alone may provide symptomatic relief benefits but is not curative. Systemic chemotherapy is usually not offered to treat peritoneal carcinomatosis in the absence of cytoreductive surgery in eligible candidates. In the absence of extra-abdominal metastasis, cytoreductive surgery (CRS) involving resection of all visible tumor cells in the abdomen in combination with hyperthermic intraperitoneal chemotherapy (HIPEC) has been found to result in significant improvement in the quality of life in such patients with peritoneal carcinomatosis. There is current evidence of a 5-year survival benefit of 50% in the published literature. CRS with HIPEC has also been shown to prolong disease-free survival compared to standard treatment with systemic chemotherapy. There is no role for radiotherapy here. There is no role for chemoradiotherapy in this context, and at this stage of her disease progression, diagnosis, and treatment.
Go to the next page if you knew the correct answer, or click the link image(s) below to further research the concepts in this question (if desired).
Research Concepts: Peritoneal Metastasis
We update eBooks quarterly and Apps daily based on user feedback. Please tap flag to report any questions that need improvement.
Question 236:
A 52-year-old female presents to the breast imaging center for an annual screening mammogram. A few scattered punctate calcifications are seen in both the breasts. These findings remain stable compared to her mammogram two years ago. Her current medications include atorvastatin and a daily vitamin. She is scheduled for her colonoscopy next month and is really concerned about herself. What BI-RADS assessment is given to this patient; what will you tell her regarding the results of the mammogram?
Choices: 1. BI-RADS 1; findings are benign 2. BI-RADS 2; findings are benign 3. BI-RADS 2; findings are highly suggestive of malignancy 4. BI-RADS 3; another mammogram is needed to comment on malignancy potential
Answer: 2 - BI-RADS 2; findings are benign Explanations: Scattered punctate calcifications are very common benign finding on mammogram. Two-year stability of the calcifications determines that these are benign in nature. Any benign finding on mammogram falls under BI-RADS category 2. Patients with BI-RADS category 2 findings are recommended to return to annual screening mammogram without a need for additional workup. Go to the next page if you knew the correct answer, or click the link image(s) below to further research the concepts in this question (if desired).
Research Concepts: Mammography BI RADS Grading
We update eBooks quarterly and Apps daily based on user feedback. Please tap flag to report any questions that need improvement.
Question 237:
A 65-year-old male presented to the emergency department with complaints of precordial chest pain for the past three weeks. Computed tomography of the chest demonstrated a large 6 cm x 6 cm x 8 cm supradiaphragmatic complex solid mass in the lower right hemithorax, along the anterior chest wall. Surgical excision revealed dedifferentiated liposarcoma with excision of margins. However, eight months later, the patient presented with shortness of breath and right-sided chest pain. Which of the following imaging modalities is preferred to detect recurrence of chest wall tumors?
Choices: 1. 2. 3. 4.
Ultrasonography Positron Emission Tomography (PET) Chest Radiography V/Q Scan
Answer: 2 - Positron Emission Tomography (PET) Explanations: Positron Emission Tomography is involved in the staging of disease, evaluation of treatment response, and detection of recurrent disease. Ultrasonography can be used to guide biopsy and has been shown to depict chest wall invasion by lung cancer more accurately than CT; however, it has a limited role in the evaluation and characterization of superficial chest wall lesions. Chest radiography will be instrumental in detecting the location and possible cortical destruction of bone; however, it is not superior to PET in detecting recurrence of the disease. V/Q scanning has no role in the detection of recurrence of chest wall tumors. Go to the next page if you knew the correct answer, or click the link image(s) below to further research the concepts in this question (if desired).
Research Concepts: Chest Wall Tumors
We update eBooks quarterly and Apps daily based on user feedback. Please tap flag to report any questions that need improvement.
Question 238:
A 68-year-old male presents to the hospital with abdominal pain. He has a past medical history significant for hypertension, hyperlipidemia, and chronic kidney disease. Past surgical history was notable for spinal laminectomy. There is some clinical concern for small bowel obstruction. He is taken to the operating room, and an intussusception is identified within the small bowel that is resected. Grossly, the resection specimen demonstrates a raised lesion, measuring 4 cm located at the lead point for the intussusception. Histologically, the lesion demonstrates transmural vascular proliferation with ulceration. The endothelial cells do not have overt cytologic atypia, and there is no morphologic evidence of malignancy. No other vascular abnormalities are seen. The patient subsequently follows up 14 months postoperatively without recurrence. Which of the following is most often associated with patient's finding?
Choices: 1. Infiltrative, anastomosing vascular channels, with cytologic atypia and brisk mitotic activity 2. The vascular endothelial cell shows positivity for HHV-8 (LANA1) and negativity for CD31 3. Granulation tissue type proliferation of vasculature endothelial cells showing positivity for CD31 and CD34 and negativity for HHV-8 (LANA1) 4. Polypoid lesion demonstrating proliferation of capillaries, veins or lymphatics, usually involving the mucosa or submucosa only
Answer: 3 - Granulation tissue type proliferation of vasculature endothelial cells showing positivity for CD31 and CD34 and negativity for HHV-8 (LANA1)
Explanations: Florid vascular proliferation (FVP) is a benign reactive process, commonly involving colon. Histopathology shows lobular granulation tissue-type proliferation of small vascular channels involving the entire thickness of the bowel wall. The endothelial cells have minimal nuclear atypia and few to no mitotic figures. The overlying mucosa shows ulceration and some features of mucosal prolapse. Immunohistochemical study shows the endothelial cells are positivity for CD31 and CD34 stains, and negativity for HHV-8 stain. Angiosarcoma is a malignancy of vessels, associated with chronic lymphedema and radiation exposure. Histopathologically show atypical vascular channels with brisk mitosis and necrosis. On immunohistochemistry, endothelial cells are positive for factor VIII, CD31, CD34, and are negative for HHV-8. Kaposi sarcoma is a locally aggressive, vascular neoplasm caused by HHV-8. Histopathologically, demonstrate slit-like vessels dissecting through dermis with minimal atypia and brisk mitosis. Extravasated erythrocytes, hemosiderin, and plasma cell are also common findings. Hemangiolymphomas (HLAs) are benign vascular lesions and commonly are polypoid, ranging from 0.5 cm to 3.5 cm. Histologically HLAs show proliferation of capillaries, veins or lymphatics, usually involving the mucosa or submucosa only and is not transmural.
Go to the next page if you knew the correct answer, or click the link image(s) below to further research the concepts in this question (if desired).
Research Concepts: Benign Florid Vascular Proliferation Mimicking Vascular Neoplasm
We update eBooks quarterly and Apps daily based on user feedback. Please tap flag to report any questions that need improvement.
Question 239:
A 48-year-old female with past medical history of B cell lymphoma colitis on pembrolizumab presents with bilateral perianal abscesses. Patients states she was here 1 week ago with the same and now increase in pain and erythema. What is the appropriate treatment for this patient?
Choices: 1. Incision and drainage with a surgeon at the bedside 2. Addition of ciprofloxacin for recurrence of infection. 3. Obtain an endoanal ultrasound 4. Obtain blood cultures, start intravenous antibiotics and admit to the hospital for imaging and possible surgical drainage
Answer: 4 - Obtain blood cultures, start intravenous antibiotics and admit to the hospital for imaging and possible surgical drainage
Explanations: Patients with recurrent or bilateral abscesses should be evaluated by surgery for intraoperative drainage, including cases where internal draining may be necessary. Ciprofloxacin should be combined with metronidazole for appropriate antibiotic coverage. CT should be obtained in patients with significant comorbidities, complex suppurative anorectal conditions, and when surgical consultation is not immediately available. Patients who are immunocompromised with bilateral and recurrent abscesses should be started on intravenous antibiotics after cultures are obtained and have a prompt surgical evaluation for intraoperative drainage. Go to the next page if you knew the correct answer, or click the link image(s) below to further research the concepts in this question (if desired).
Research Concepts: Perirectal Abscess
We update eBooks quarterly and Apps daily based on user feedback. Please tap flag to report any questions that need improvement.
Question 240:
A 52-year-old woman presents to the hospital with chronic diarrhea and vague upper abdominal pain. She has a history of diabetes mellitus 2 and hypertension. CT of her abdomen shows a 3 cm x 2 cm mass in the head of the pancreas. An endoscopic retrograde cholangiopancreatography with biopsy is performed, and the histopathology revealed grade 1 differentiation, and immunohistochemistry is positive for somatostatin, neuronspecific enolase, and vasoactive intestinal polypeptide. PET scan is negative for metastasis. Which of the following is the best initial therapy for this patient?
Choices: 1. 2. 3. 4.
Etoposide Octreotide Cisplatin Everolimus
Answer: 2 - Octreotide Explanations: The presence of diabetes mellitus and diarrhea in the setting of somatostatin positive tumor gives the diagnosis of functional/ secretory somatostatinoma. Somatostatin analogs are one of the first-line medical therapy in functional somatostatinoma. Somatostatin analogs function by inhibiting cellular proliferation and mediating apoptosis by binding to the somatostatin receptors. They also exert indirect antiproliferative mechanisms include inhibition of circulating growth factors such as vascular endothelial growth factor. WHO has classified gastro-entero-pancreatic tumors based on the mitotic index into grade 1 – Ki-67 =2%, grade 2 – Ki-67 3%–20%, grade 3 – Ki-67 >20%, mixed adenocarcinoma, and neuroendocrine carcinoma. Lowgrade tumors generally do not respond to platinumbased therapy. However, they have proven efficacy in advanced grade tumors in conjunction with etoposide/irinotecan. Everolimus, mammalian target of rapamycin (mTOR) inhibitor is effective in patients with refractory, functional neuroendocrine tumors. Currently, it is not recognized as first-line therapy. Go to the next page if you knew the correct answer, or click the link image(s) below to further research the concepts in this question (if desired).
Research Concepts:
Somatostatinoma
We update eBooks quarterly and Apps daily based on user feedback. Please tap flag to report any questions that need improvement.
Question 241:
A skin biopsy from a 55-year-old man, clinically described as "skin neoplasm uncertain nature" shows, by H&E, a circumscribed, low to a moderatelycellular proliferation of bland spindled cells with some larger and more hyperchromatic forms in a mildly-myxoid stroma. Mast cells are evident as well. Which immunohistochemical panel is most useful in confirming the diagnosis?
Choices: 1. 2. 3. 4.
CD34, S-100 CD34, Factor 13a MART-1, SOX10 CD117, CD163
Answer: 1 - CD34, S-100 Explanations: The lesion described is a localized neurofibroma, in which a portion of cells mark with CD34 (often in a "fingerprint" pattern) and a portion will mark with S-100. Neurofibromas are mixed proliferations of peripheral nerve sheath cells and fibroblastic cells. Due to their composition of both nerve sheath and fibroblastic cells, some cells mark with S-100 and some with CD34. The panel CD34 and Factor13a will help distinguish dermatofibroma (CD34 mostly negative, Factor 13a positive) from dermatofibrosarcoma protuberans (CD34 positive, Factor 13a negative). MART-1 and SOX10 are melanocytic markers (SOX10 will also stain peripheral nerve sheath cells). CD117 marks mast cells, and CD163 marks histiocytes. Go to the next page if you knew the correct answer, or click the link image(s) below to further research the concepts in this question (if desired).
Research Concepts: Neurofibroma
We update eBooks quarterly and Apps daily based on user feedback. Please tap flag to report any questions that need improvement.
Question 242:
A 62-year-old female is diagnosed with a Helicobacter pylori infection that is causing peptic ulcer disease. She is appropriately treated with two antibiotics and a proton pump inhibitor, which is continued long term. The patient's symptoms recur. Stool tests and a urea breath test are negative for recurrence of H. pylori, so a repeat endoscopy is done. Persistent and new ulcers are noted. Fasting gastrin and basal acid secretion are elevated. What is the best next step?
Choices: 1. 2. 3. 4.
Secretin-stimulated gastrin levels Gastrin levels after a meal Double the proton pump inhibitor dose Re-treat for H. pylori
Answer: 1 - Secretin-stimulated gastrin levels Explanations: In order to make the diagnosis of Zollinger-Ellison syndrome (ZES), a secretin-stimulated gastrin level should be done. If 2 picograms/kilogram of secretin intravenously increases gastrin levels by more than 200 picograms within 15 minutes, this is more than 90% sensitive and specific for ZES. Endoscopic ultrasound may identify the location of the gastrinoma. A somatostatin radioscan is very sensitive for detecting the primary location of metastatic lesions of ZES. Other tests include upper endoscopy and CT scan. Go to the next page if you knew the correct answer, or click the link image(s) below to further research the concepts in this question (if desired).
Research Concepts: Zollinger Ellison Syndrome
We update eBooks quarterly and Apps daily based on user feedback. Please tap flag to report any questions that need improvement.
Question 243:
A 63-year-old female presents to the emergency department with painful blistering and erosions of her lips and gingiva that have been present for two weeks. A review of systems is notable for an unintentional 10 lb (4.5 kg) weight loss in the last month. A biopsy has been taken, and laboratory studies ordered. Which of the following is most likely to narrow the diagnosis?
Choices: 1. 2. 3. 4.
Immunoblot for detection of autoantibodies Indirect immunofluorescence (IIF) Histopathology findings on H&E Direct immunofluorescence (DIF)
Answer: 1 - Immunoblot for detection of autoantibodies Explanations: Three features demonstrate high specificity (84%-100%) and sensitivity (82-86%) in the diagnosis of paraneoplastic pemphigus. These are (1) presence of an underlying lymphoproliferative disorder, (2) an IIF positive for autoantibodies on rat bladder, and (3) detection of autoantibodies on immunoblot against plakin proteins, including periplakin, envoplakin, and desmoplakin I and II. Additional diagnostic guidelines have been created that classify criteria as either major or minor. PNP diagnosis requires three major criteria, or two major and two minor criteria. The major criteria include polymorphic mucocutaneous eruptions, concomitant internal neoplasm, and serum antibodies with a specific immunoprecipitation pattern. The minor criteria include acantholysis observed histologically, DIF displaying intercellular and basement membrane staining, and IIF staining positive on rat bladder epithelium. Serologic detection of autoantibodies against the 210kD band of envoplakin or the 190-kD band of periplakin is both sensitive and specific for the diagnosis of PNP. However, PNP serum may react with desmoglein 3, desmoglein 1, desmoplakin I (250 kD), desmoplakin II (210kD), bullous pemphigoid antigen I (230kD), and alpha-2-macroglobulin-like-1 (A2ML1), among others. DIF, IIF, and histologic findings may overlap with other pemphigoid diseases and thus are not likely to provide the most diagnostic information. Typical histologic findings of PNP, such as acantholytic cells and interface dermatitis, are relatively nonspecific and nondiagnostic.
Though IIF staining positive on rat bladder epithelium is suggestive of PNP, immunoblot is a more accurate method of diagnosis. Go to the next page if you knew the correct answer, or click the link image(s) below to further research the concepts in this question (if desired).
Research Concepts: Paraneoplastic Pemphigus
We update eBooks quarterly and Apps daily based on user feedback. Please tap flag to report any questions that need improvement.
Question 244:
A 67-year-old male comes in for the evaluation of fatigue, muscle weakness in his extremities, and weight loss over the last five months. He has a history of hypertension that has been under control with diet and denies any other medical history. He has been a long-term smoker for 45 years and smokes two packs of cigarettes daily. He does not drink alcohol or use any illicit drugs. On physical examination, muscle strength of 3/5 is noted in the proximal muscle groups bilaterally and symmetrically. Reflexes are normal, and no sensory abnormality is seen. Erythematous papules are noted on his fingers. He denies having been in contact with any animals recently or having any history of allergies. No other major abnormal findings are seen. A chest X-ray shows a mass in the right lower lobe, and a biopsy is done to search for malignancy. Muscle biopsy shows perimysial inflammation. Which other specific findings can be seen in this patient's muscle biopsy?
Choices: 1. Presence of CD4+ T cells 2. Presence of CD8+ T cells 3. Presence of antibodies against postsynaptic Ach receptors 4. Presence of antibodies preventing the release of Ach from presynaptic junctions
Answer: 1 - Presence of CD4+ T cells Explanations: This patient has a lung mass, weight loss, and longstanding history of smoking. A diagnosis of lung cancer is very likely. There are multiple paraneoplastic conditions associated with lung cancer, including myasthenia gravis, Lambert-Eaton syndrome, and dermatomyositis. Dermatomyositis can be seen in lung cancer and presents as heliotrope or violaceous rash and muscle weakness. Darkening and thickening of fingertips can also be found. Biopsy findings in dermatomyositis include perimysial inflammation with possible atrophy and the presence of CD4+ T cells. Multiple laboratory-based findings in dermatomyositis include a positive ANA, increased CK, positive anti-Jo-1 (histidyl-tRNA synthetase), positive anti-SRP (signal recognition particle), and positive anti-Mi-2 (helicase). Go to the next page if you knew the correct answer, or click the link image(s) below to further research the concepts in this question (if desired).
Research Concepts: Paraneoplastic Syndromes
We update eBooks quarterly and Apps daily based on user feedback. Please tap flag to report any questions that need improvement.
Question 245:
A 20-year-old female presents to the clinic with a slowly growing mass on her left nasal ala. She noticed the lesion 2 years ago and denies drainage, bleeding, pain, or history of trauma. The pathology report shows a lobular pattern of epithelioid cells that stained positive for CD10 and NK1/C3 and stained negative for S100. What is the most likely diagnosis for this lesion?
Choices: 1. 2. 3. 4.
Cellular neurothekeoma Plexiform fibrohistiocytic tumor Melanocytic tumor Nerve sheath myxoma
Answer: 1 - Cellular neurothekeoma Explanations: Cellular neurothekeomas have the given the staining profile of S-100 negative and NK1/C3 positive. Morphologically they are lobules and nests of epithelioid cells with abundant eosinophilic cytoplasm. Plexiform fibrohistiocytic tumors typically are located in the deep soft tissue of the extremities. They are characteristically biphasic with a prominent fibrous component containing scattered histiocytic cell aggregates with multinucleated giant cells. Melanocytic tumors would stain positive for S-100. Nerve sheath myxomas would stain positive for S-100, contain abundant myxoid stroma, and have higher recurrence rates than neurothekeomas. Go to the next page if you knew the correct answer, or click the link image(s) below to further research the concepts in this question (if desired).
Research Concepts: Neurothekeoma
We update eBooks quarterly and Apps daily based on user feedback. Please tap flag to report any questions that need improvement.
Question 246:
A 65-year-old female presents to the office with complaints of fatigue and chest discomfort. The patient has a history of clinical T2N0M0, ypT0N0Mx moderately-differentiated invasive ductal carcinoma in the right breast, which was ER/PR/Her2neu positive, and was treated with right breast mastectomy and sentinel lymph node biopsy revealing complete pathologic response and 0/4 lymph nodes involved. The clinician suspects chest wall recurrence. She wants to discuss the reasons behind her recurrence. Which of the following increased this patient's risk of recurrence?
Choices: 1. 2. 3. 4.
Tumor size of 4 cm Unifocal disease Nipple involvement ER-positive status
Answer: 1 - Tumor size of 4 cm Explanations: Katz et al. found that patients with tumors greater than or equal to 4 cm or at least four involved nodes experience local recurrence rates over 20%. Gross multifocal/multicentric disease is more likely to lead to recurrent disease. Age younger than 40 and genetic predispositions to breast cancer are associated with the risk of recurrent breast cancer. ER-positive hormone status permits the use of selective estrogen receptor modulators, drugs whose benefits in the treatment of breast cancer outweigh their relatively low toxicity. Go to the next page if you knew the correct answer, or click the link image(s) below to further research the concepts in this question (if desired).
Research Concepts: Hyperthermia for Chest Wall Recurrence
We update eBooks quarterly and Apps daily based on user feedback. Please tap flag to report any questions that need improvement.
Question 247:
A 65-year-old male presents to a clinic with complaints of lumbar pain and difficulty with urination. Detailed medical history revealed that he had lost 22 kgs of weight over a period of six months. He also reports that he often feels fatigued. On physical examination, his back is non-tender, and there is no abnormal finding on the neurologic exam. The rectal exam reveals an enlarged, nodular hard prostate with obliteration of the slit and immobile rectal mucosa in the region corresponding to the prostate. His prostate-specific antigen (PSA) is 38 ng/mL. Magnetic resonance imaging and prostate biopsy show a clinically advanced disease and a Gleason score of 4 + 4 = 8. He was started on oral hormonal agents but did not show any response. The blood workup for AR-V7 was positive. What does the presence of AR-V7 (androgen receptor splice variant-7 messenger RNA) in this patient mean?
Choices: 1. 2. 3. 4.
Resistance to enzalutamide Resistance to abiraterone acetate Resistance to both enzalutamide and abiraterone acetate Sensitivity to both enzalutamide and abiraterone acetate
Answer: 3 - Resistance to both enzalutamide and abiraterone acetate
Explanations: The finding of AR-V7 (androgen receptor splice variant-7 messenger RNA) in circulating tumor cells in patients with castrate-resistant prostate cancer indicates expected resistance to both enzalutamide and abiraterone acetate. Both of these drugs target the androgen receptor site. A blood test for AR-V7 is now commercially available and is currently recommended for patients who fail initial treatment with any of these oral hormonal agents. Interestingly, a positive AR-V7 blood test also suggests an enhanced response to chemotherapy. Earlier use of these drugs is encouraged. If there is no clinical response as evidenced by a drop in the prostatespecific antigen over the following 12 weeks, then another therapy should be substituted. Patients with metastatic, hormone-sensitive disease and especially those with high volume disease, benefited most from adding docetaxel chemotherapy initially along with their androgen-deprivation therapy. Go to the next page if you knew the correct answer, or click the link image(s) below to further research the concepts in this question (if desired).
Research Concepts: Prostate Cancer
We update eBooks quarterly and Apps daily based on user feedback. Please tap flag to report any questions that need improvement.
Question 248:
A 25-year-old male presents to the clinician with complains of discomfort in the testis and a concern that both his breasts are getting bigger. Physical examination reveals a well-circumscribed, 3 x 3 cm sized mass in his right testis. The cut surface of this mass shows a distinct golden brown homogenous appearance. Microscopically, the tumor cells are large to polygonal with a round to oval nucleus and deeply acidophilic, abundantly granular cytoplasm. The cytoplasm shows rod-shaped crystals. Which hormone is secreted from the cell types comprising the most likely mass in this patient?
Choices: 1. 2. 3. 4.
Testosterone Luteinizing hormone Prolactin Inhibin
Answer: 1 - Testosterone Explanations: Leydig cell tumors are the most frequently seen testicular sex cord-stromal tumors. Leydig cells are the principal testicular source of androgens and are capable of estrogen production. These tumors may produce either virilizing or feminizing symptoms. In adults, the hormonal secretion can produce gynecomastia, while in children, it may produce precocious puberty. Histology of these tumors reveals Reinke crystals, which are intracytoplasmic, rod-shaped granules. About 90% of these testicular tumors are painless, benign, and carry an excellent prognosis. Go to the next page if you knew the correct answer, or click the link image(s) below to further research the concepts in this question (if desired).
Research Concepts: Leydig Cell Cancer
We update eBooks quarterly and Apps daily based on user feedback. Please tap flag to report any questions that need improvement.
Question 249:
A 55-year-old man presented with vague epigastric pain. He was initially diagnosed with a stomach ulcer and received treatment with proton-pump inhibitors and sucralfate for three months without improvement. A recent upper gastrointestinal endoscopy was normal, but now the patient is also experiencing back pain. Less than six months after the initial presentation, an abdominal ultrasound shows a mass in the pancreas with concern for liver lesions. Which of the following tumor suppressor genes is likely mutated in this patient?
Choices: 1. 2. 3. 4.
PTEN BRAF p16/CDKN2A MEK
Answer: 3 - p16/CDKN2A Explanations: Pancreatic cancer is considered a disease of older people and is unfortunately found late. The peak incidence is between 65 to 76 years of age, with the main risk factor being cigarette smoking. Approximately 5 to 10% of all pancreatic cancers have an inherited component. Familial cancer syndromes associated with pancreatic cancer are Peutz-Jeghers syndrome, hereditary nonpolyposis colorectal cancer syndrome (Lynch syndrome), and familial breast cancer syndrome associated with BRCA mutations. Studies show that early genetic alterations in pre-cancerous pancreatic lesions include telomere shortening and mutations in the KRAS2 oncogene. Often, inactivation of the p16/CDKN2A tumor suppressor gene is also seen early. Later on, the tumor accumulates further mutations in TP53 and SMAD4. About half of patients diagnosed with pancreatic cancer have metastatic disease at presentation, with a life expectancy of only under 6 months without treatments. The only hope for significant survival benefit is surgical resection when detected in earlier stages. Go to the next page if you knew the correct answer, or click the link image(s) below to further research the concepts in this question (if desired).
Research Concepts: Tumor-Suppressor Genes
We update eBooks quarterly and Apps daily based on user feedback. Please tap flag to report any questions that need improvement.
Question 250:
A patient is being evaluated for a nonhealing wound in an irradiated field. He had squamous cell carcinoma of the neck and received 8000 Gy of radiation along with chemotherapy 12 months ago. Which historical chemotherapy agent would be most concerning for the development of pulmonary fibrosis during a course of hyperbaric oxygen therapy?
Choices: 1. 2. 3. 4.
Adriamycin Bleomycin Cis-platinum Doxyrubicin
Answer: 2 - Bleomycin Explanations: Bleomycin has been implicated in the development of pulmonary fibrosis in patients exposed to high oxygen tensions. The incidence of pulmonary fibrosis is lower with lower doses of bleomycin. The exact causal relationship between hyperbaric oxygen and bleomycin causing pulmonary fibrosis has not been established but it is possible that bleomycin exposure lowers the lungs' tolerance to pulmonary oxygen toxicity. When treating a patient with a history of bleomycin exposure with hyperbaric oxygen therapy, it is prudent to establish baseline pulmonary function and monitor closely during the treatment course for any signs of toxicity. Go to the next page if you knew the correct answer, or click the link image(s) below to further research the concepts in this question (if desired).
Research Concepts: Hyperbaric Complications
We update eBooks quarterly and Apps daily based on user feedback. Please tap flag to report any questions that need improvement.
Question 251:
A 69-year-old woman presents with a palpable mass in the left breast. Mammography shows a relatively well-circumscribed lobulated mass in the right upper portion of the left breast. Ultrasonography reveals a well-defined, oval, isoechogenic solid mass with small cystic component measuring approximately 2.8cm in the left breast. An excisional biopsy is performed. Microscopic evaluation reveals nests of cells floating in lakes of mucin partitioned by delicate fibrous septa containing capillary blood vessels. The cell clusters are variable in shape and size with an occasional tubular arrangement. Immunohistochemical studies show positive immunostaining of tumor cells for estrogen and progesterone receptors. HER2 is not amplified. What is the most likely diagnosis?
Choices: 1. 2. 3. 4.
Myxoid fibroadenoma Mucocele-like-lesion Metaplastic carcinoma Mucinous carcinoma
Answer: 4 - Mucinous carcinoma Explanations: Histologically, mucinous carcinoma is characterized by nests of cells floating in lakes of mucin partitioned by delicate fibrous septae containing capillary blood vessels. The cell clusters are variable in shape and size with an occasional tubular arrangement. Nuclear atypia is generally low in classic mucinous carcinoma, but in rare cases, atypia and mitoses may prevail. Typically, mucinous carcinoma is positive for estrogen and progesterone receptors, while androgen receptors are expressed at a low level and HER2 is not amplified. On mammography, circumscribed and lobulated are common findings for a mucinous carcinoma. On ultrasonography, mucinous carcinoma is usually a complex mass with cystic and solid components microlobulation, vascularity, and distal enhancement. Pure mucinous carcinoma of the breast has high levels of internal echoes with accentuated levels of posterior echoes. Go to the next page if you knew the correct answer, or click the link image(s) below to further research the concepts in this question (if desired).
Research Concepts: Mucinous Breast Carcinoma
We update eBooks quarterly and Apps daily based on user feedback. Please tap flag to report any questions that need improvement.
Question 252:
A 48-year-old male patient came to the emergency department complaining of abdominal pain and distention for three days. He reports two episodes of emesis this morning. There is no history of weight loss. His last bowel movement was two days ago, but he is passing flatus. His past medical history is otherwise unremarkable. Abdominal x-rays are unremarkable. Abdominal computed tomography with oral contrast showed a 3 cm intraluminal mass in a proximal small bowel segment with a very small amount of contrast material passing distally to this obstructing lesion. There is also extensive nodular involvement of the surrounding mesentery. He was managed with segmental resection of the bowel with its mesentery laden with local carcinomatosis, and he was concurrently treated with cytoreductive surgery in combination with intraperitoneal hyperthermic chemotherapy. The pathology report described a carcinoid tumor with positive lymph nodes and positive tumor cells in mesenteric lesions. Which of the following is the most significant prognostic factor for long-term survival in this patient?
Choices: 1. 2. 3. 4.
Small size of the primary tumor Hyperthermia related to chemotherapy Completeness of cytoreduction surgery Age
Answer: 3 - Completeness of cytoreduction surgery Explanations: Carcinoid is the most common cancer of the small intestine and often metastasizes to the peritoneum, including that covering the mesentery. Carcinoid is a slow-growing tumor and often presents at an advanced stage with symptoms of bowel obstruction as is the case in this question. At this stage of presentation with the loco-regional spread, tumor size has less of an impact on overall survival. Cytoreductive surgery can positively influence survival when applied along with primary tumor resection in patients with neuroendocrine tumor involving peritoneal metastasis. The temperature of the intraperitoneal chemotherapy is thought to contribute to its effectiveness, but there is insufficient evidence that the temperature alone is an independent factor contributing to significant differences in long-term survival. Completeness of cytoreduction ensures no visible macroscopic lesions in the peritoneum and mesentery. This complete removal of all lesions has been found to have a higher impact on long term survival as an independent factor in the treatment of carcinoid with peritoneal metastasis. The completeness of cytoreduction surgery plays a more significant role in contributing to the effectiveness of hyperthermic intraperitoneal chemotherapy (HIPEC) therapy and its resultant long-term survival outcomes. Go to the next page if you knew the correct answer, or click the link image(s) below to further research the concepts in this question (if desired).
Research Concepts: Peritoneal Metastasis
We update eBooks quarterly and Apps daily based on user feedback. Please tap flag to report any questions that need improvement.
Question 253:
A 70-year-old woman with CharcotMarie-Tooth disease is diagnosed with Epstein-Barr virusassociated plasmablastic lymphoma and scheduled to start chemotherapy with da-R-EPOCH (dose adjusted rituximab, etoposide, prednisone, vincristine, doxorubicin). Which of the following chemotherapy drugs is contraindicated in this patient?
Choices: 1. 2. 3. 4.
Prednisone Vincristine Etoposide Doxorubicin
Answer: 2 - Vincristine Explanations: Neurotoxicity is a very well-known complication of vincristine, a microtubule inhibitor. Vincristine is absolutely contraindicated in patients with the demyelinating variant of Charcot-Marie-Tooth disease. When used, the recommended dose of vincristine is 1.4 mg/m2 per single dose and is capped at 2 mg/m2 per single dose regardless of the body surface area. Dose reduction should be considered in patients who are experiencing moderate symptoms of paresthesias, sensory and motor neuropathy, and limited instrumental activities of daily living. Duloxetine, an antidepressant, can be used to treat neuropathic pain arising due to chemotherapy such as vincristine. Go to the next page if you knew the correct answer, or click the link image(s) below to further research the concepts in this question (if desired).
Research Concepts: Plasmablastic Lymphoma
We update eBooks quarterly and Apps daily based on user feedback. Please tap flag to report any questions that need improvement.
Question 254:
A 55-year-old woman, who recently had allogeneic stem cell transplantation, presents to the emergency department with severe diarrhea, abdominal pain, bleeding per rectum, and fever. Her investigations show raised cytomegalovirus IgG and IgM. Which of the following is the next best step in the management of this patient?
Choices: 1. 2. 3. 4.
Start ganciclovir treatment Arrange for colonoscopy and histological examination Treat for gastrointestinal graft-versus-host disease Increase the dose of immunosuppressants
Answer: 2 - Arrange for colonoscopy and histological examination
Explanations: High CMV IgG and IgM cannot differentiate gastrointestinal graft-versus-host disease from CMV colitis. This differentiation needs to be established before starting ganciclovir treatment. Tests with high sensitivity and specificity such as colonoscopy and histological examination of tissue biopsies from the colon (hematoxylin and eosin [H&E], and immunohistochemistry staining) are important in making such differentiation. Not enough evidence is available from the case scenario to increase the dose of immunosuppressants. Go to the next page if you knew the correct answer, or click the link image(s) below to further research the concepts in this question (if desired).
Research Concepts: Cytomegalovirus Colitis
We update eBooks quarterly and Apps daily based on user feedback. Please tap flag to report any questions that need improvement.
Question 255:
A 55-year-old woman presents with lower abdominal pain, bloating, and pain during sexual intercourse. She has had spotty bleeding for a few months but says she thinks it is just a part of her postmenopausal phase. She undergoes a pelvic ultrasound which reveals a solid ovarian mass with well-defined margins. Bilateral oophorectomy and hysterectomy reveal large cells shaped like a signet ring and mucoid degeneration. What is the most appropriate next step in management?
Choices: 1. 2. 3. 4.
Observation EGD (esophagogastroduodenoscopy) Chemotherapy Radiation
Answer: 2 - EGD (esophagogastroduodenoscopy) Explanations: Krukenberg tumor refers to a signet ring of metastatic cancer to the ovary. The primary objective of treatment is surgical resection with an R0 result. In patients where complete resection is not possible, surgery can be used in an attempt to achieve palliation. Metastatectomy is favored in this disease. Currently, chemotherapy and radiation offer only a modicum of improvement in the overall prognosis. Gastric and colorectal tumors account for almost 90% of the primary sites for this tumor. A biopsy is mandatory to confirm the diagnosis, so an upper endoscopy and colonoscopy are required. In unusual cases, other organs may need to be evaluated, including breasts, appendix, and small intestine, for the primary source of this tumor. Go to the next page if you knew the correct answer, or click the link image(s) below to further research the concepts in this question (if desired).
Research Concepts: Krukenberg Tumor
We update eBooks quarterly and Apps daily based on user feedback. Please tap flag to report any questions that
need improvement.
Question 256:
A middle-aged female has mild pain over her right hypochondrium for 3 months, accompanied with some weight loss. Her physical examination was unremarkable except her liver is palpable below the right costal margin. Initial blood workup, including blood investigation, reported alfa-fetoprotein 5 ng/dl, Ultrasound of the abdomen followed by computed tomography scan abdomen revealed a heterogeneous partially enhancing lesion with no significant washout on Portoveneous phase. Of note, there were absent centripetal filling or centrifugal filling on contrast imaging. Which of the following is the most likely diagnosis?
Choices: 1. 2. 3. 4.
Hepatoma Hepatic adenoma Focal nodular hyperplasia of the liver Hepatic lymphoma
Answer: 4 - Hepatic lymphoma Explanations: A primary hepatic lymphoma is a rare form of NonHodgkin lymphoma primarily involving liver with no involvement of extrahepatic lymph nodes or viscera; notably the spleen and bone marrow. Primary Hepatic lymphoma is rare tumors of the liver, constituting around 0.4% of all extranodal Non-Hodgkin lymphoma (NHL), and 0.016% of all NHL. In contrast to hepatic adenoma; which shows centripetal enhancement, and focal nodular hyperplasia of the liver; which enhances centrifugally, the primary hepatic lymphoma shows differential enhancement. The lesion is often hypodense with some enhancement on the arterial phase with no characteristic washout, a feature of hepatoma. Furthermore, the lesion appears to be heterogeneous secondary to infarction followed by necrosis of the center of the lesion. The hepatic lymphoma requires a high level of suspicion for diagnosis since the lesion often mimics with other space-occupying lesions of the liver; notably the hepatocellular carcinoma, and benign liver tumors; such as hepatic adenoma and focal nodular hyperplasia. Go to the next page if you knew the correct answer, or click the link image(s) below to further research the concepts in this question (if desired).
Research Concepts: Hepatic Lymphoma
We update eBooks quarterly and Apps daily based on user feedback. Please tap flag to report any questions that need improvement.
Question 257:
A 62-year-old white male has a 1centimeter polyp removed from the sigmoid colon during a screening colonoscopy that is determined to have tubulovillous features on pathology. When should a colonoscopy be repeated?
Choices: 1. 2. 3. 4.
1 3 5 7
year years years years
Answer: 2 - 3 years Explanations: Colonoscopy should be repeated in 3 years if at least one adenoma is 10 mm in diameter. Colonoscopy also should be repeated in 3 years if one or more adenomas show villous features of any size or with high-grade dysplasia. Ten percent of colonic polyps are adenomatous and are classified as tubular, tubulovillous, and villous. Higher risk features include size over 1 centimeter, greater than three polyps, villous component, sessile, and high-grade dysplasia. Go to the next page if you knew the correct answer, or click the link image(s) below to further research the concepts in this question (if desired).
Research Concepts: Colon Polyps
We update eBooks quarterly and Apps daily based on user feedback. Please tap flag to report any questions that need improvement.
Question 258:
A 66-year-old female with a history of breast cancer presents with complaints of diffuse abdominal pain, vomiting, and bloating. She has had some mild discomfort with nausea and vomiting for the last two days, but it became much worse today. She had minimal flatus over the previous 24 hours, and her last bowel movement was three days ago. An abdominal x-ray series shows marked dilated small bowel and minimal air in the colon. She is admitted with a diagnosis of small bowel obstruction, and an NG tube is placed. The next day her status deteriorates, she complains of more pain, and her heart rate is now 115 bpm with a temperature of 102.4 F (39.1 C). A new x-ray is obtained, showing free air under the diaphragm, and an exam shows rebound tenderness throughout the abdomen. What is the next step in the management?
Choices: 1. Laparoscopic exploration 2. IV pain medication, aggressive fluid resuscitation, and repeat CBC 3. Abdominal and pelvic CT with IV contrast 4. Sonographically guided drainage
Answer: 1 - Laparoscopic exploration Explanations: Given this history, physical, and imaging findings, it appears that the patient's small bowel obstruction has likely led to perforation. This is a surgical emergency, especially given the patient's impending sepsis. The most likely location of this perforation is the small bowel, given that there appeared to be dilated small intestine with minimal air in the colon on x-ray. A history of prior malignancy makes the possibility of metastatic disease to the abdomen a possibility and a possible cause of the perforation. Other disease processes can cause small bowel obstruction, such as surrounding inflammation from appendicitis and diverticulitis. Regardless, she needs surgery for the acute perforation of any cause. Further diagnostic measures, including CBC and imaging, would not change the plan from an immediate exploration to conservative management. Go to the next page if you knew the correct answer, or click the link image(s) below to further research the concepts in this question (if desired).
Research Concepts: Intestinal Perforation
We update eBooks quarterly and Apps daily based on user feedback. Please tap flag to report any questions that need improvement.
Question 259:
A 42-year-old female complains of a palpable, non-tender mass growing in her anterior lower leg for the past 5 months. The mass presents with no ulceration, surrounding erythema, or edema. It measures approximately 3 x 3 x 2 cm. After surgical excision, immunohistochemical analysis showed positivity for S100. Which of the following additional findings would most likely support the diagnosis of clear cell soft tissue sarcoma?
Choices: 1. t(X;18)(p11;q11) chromosomal translocation 2. EWSR1-ATF1 gene fusion 3. t(11;22)(q21-24;q11-13) chromosomal translocation 4. Activating mutations in the BRAF gene, most commonly V599E
Answer: 2 - EWSR1-ATF1 gene fusion Explanations: Most cases of clear cell sarcoma of soft tissue (CCSST) present with a t(12;22)(q13;12) translocation, between activating transcription factor 1 (ATF1) on chromosome 12 and Ewing sarcoma oncogene R1 (EWSR1) on chromosome 22. The resulting EWSR1-ATF1 gene fusion, which can be detected using RT-PCR or FISH, differentiates these tumors from malignant melanomas. CCSST shares histological and immunohistochemical characteristics with malignant melanomas, particularly S100 and HMB-45 positivity. For this reason, chromosome analysis for specific translocations is useful for the pathologic confirmation of CCSST. The chromosomal translocation t(X;18)(p11;q11) is found in synovial sarcoma. Similarly, t(11;22)(q2124;q11-13) is seen in Ewing sarcoma. Activating mutations in the BRAF gene, most commonly V599E, commonly are seen in malignant melanoma. Go to the next page if you knew the correct answer, or click the link image(s) below to further research the concepts in this question (if desired).
Research Concepts: Soft Tissue Clear Cell Sarcoma
We update eBooks quarterly and Apps daily based on user feedback. Please tap flag to report any questions that need improvement.
Question 260:
A 59-year-old male patient with a history of non-Hodgkins lymphoma treated with cyclophosphamide, adriamycin, vincristine, and prednisone eight years ago presents with a new diagnosis of myelodysplastic syndrome. Bone marrow biopsy reveals 12% blasts and greater than 10% dysplasia in the erythroid and granulocytic lineages. Fluorescence in situ hybridization (FISH) revealed monosomy 7. He asks you about the risk of transformation into acute myeloid leukemia (AML). What is the risk of treatment-related myelodysplastic syndrome transforming into AML?
Choices: 1. 2. 3. 4.
50% of patients will transform into AML 2% of patients will transform into AML 10% of patients will transform into AML There is no risk of transformation into AML
Answer: 1 - 50% of patients will transform into AML Explanations: 50% of patients with treatment-related myelodysplastic syndrome (MDS) will have transformation into acute myeloid leukemia (AML), while only 30% of de novo MDS will transform. Treatment-related, or therapy-related, MDS is associated with prior exposure to chemotherapeutic agents such as alkylating agents. Treatment-related MDS is associated with monosomy 5 or 7. This type of MDS has a poor prognosis compared to other types of MDS. Go to the next page if you knew the correct answer, or click the link image(s) below to further research the concepts in this question (if desired).
Research Concepts: Myelodysplastic Syndrome
We update eBooks quarterly and Apps daily based on user feedback. Please tap flag to report any questions that need improvement.
Question 261:
A 28-year-old female presents to the hospital with tremors, diaphoresis, and near syncope. Fingerstick glucose shows hypoglycemia. She is given IV dextrose, and her symptoms improve. She undergoes biochemical testing, which is suggestive of endogenous hyperinsulinemia. CT abdomen with contrast done for localization of a suspected tumor is negative. Which of the following is the best next step for diagnosing insulinoma in this patient?
Choices: 1. Abdominal ultrasound 2. Abdominal MRI 3. Endoscopic ultrasound 4. Selective arterial calcium stimulation test with hepatic venous sampling
Answer: 3 - Endoscopic ultrasound Explanations: When there is a high suspicion of insulinoma with negative noninvasive studies, invasive tests should be considered. Endoscopic ultrasound (EUS) is the best next step for diagnosing insulinoma in this patient. EUS has a sensitivity of 70-95% and is superior to CT for localization. EUS also enables tissue acquisition which can be very helpful, especially when the tumor is small. Go to the next page if you knew the correct answer, or click the link image(s) below to further research the concepts in this question (if desired).
Research Concepts: Insulinoma
We update eBooks quarterly and Apps daily based on user feedback. Please tap flag to report any questions that need improvement.
Question 262:
A 59-year-old male presents with complaints of bleeding from a wound on his left lower extremity. He has a remote history of third-degree burns to his left lower extremity that healed without intervention. His past medical history consists of rheumatoid arthritis, for which he takes methotrexate. Physical examination reveals a 6-centimeter ulcer with rolled borders overlying the area of his burn scar. There is palpable left inguinal lymphadenopathy. A biopsy reveals well-differentiated squamous cell carcinoma. Which of the following has been shown to prevent malignant transformation of old burn scars?
Choices: 1. 2. 3. 4.
Proper use of sunscreen Allowing the burn wound to heal by secondary intention Vitamin E ointment Early resection of burn wounds
Answer: 4 - Early resection of burn wounds Explanations: A Marjolin ulcer is a cutaneous malignancy that arises in the setting of previously injured skin, longstanding scars, or chronic wounds. Burn scars are the most common inciting condition that leads to the development of Marjolin ulcers. Squamous cell carcinoma is the most frequent malignancy identified on histopathologic examination (80% to 90%). Management should focus on prevention, as unresected burn wounds that heal by secondary intention are at increased risk for malignant transformation. Burn scars should be excised, and the surrounding skin should be reconstructed by a surgeon trained in reconstruction, such as a plastic surgeon, to prevent malignant transformation. Once discovered, no definitive treatment protocol exists for the management of Marjolin ulcers. The most widely accepted treatment options include Mohs surgery, wide local excision with 1 to 2 cm margins, and amputation proximal to the lesion. Go to the next page if you knew the correct answer, or click the link image(s) below to further research the concepts in this question (if desired).
Research Concepts: Marjolin Ulcer
We update eBooks quarterly and Apps daily based on user feedback. Please tap flag to report any questions that need improvement.
Question 263:
A 39-year-old woman presents with an eight-month history of fatigue, intermittent low-grade fevers, malaise, and unintentional weight loss of about 10 lbs. Physical examination demonstrates cervical and supraclavicular lymphadenopathy. Chest x-ray shows a mediastinal mass. Excisional biopsy of an accessible lymph node reveals fibrous bands within the lymph node dividing it into smaller sections. Multilobed nuclei with small nucleoli and abundant clear-cytoplasm-containing cells are evident. A background of eosinophils, macrophages, and neutrophils is seen. Immunohistochemistry reveals CD15 and CD30 positivity. What is the most likely diagnosis?
Choices: 1. 2. 3. 4.
Mixed cellularity Hodgkin lymphoma Lymphocyte-rich Hodgkin lymphoma Nodular sclerosis Hodgkin lymphoma Lymphocyte-depleted Hodgkin lymphoma
Answer: 3 - Nodular sclerosis Hodgkin lymphoma Explanations: Nodular sclerosis classic Hodgkin lymphoma (NSCHL) exhibits fibrous bands within the lymph node that separate it into different nodules. Diagnostic Reed-Sternberg cells are rarely seen in this histologic type. Instead, variant Hodgkin cells known as lacunar cells are seen with a background of eosinophils, macrophages, and neutrophils. Areas of necrosis may also be evident. Lacunar cells are defined as containing multilobed nuclei, small nucleoli, and abundant, pale, "empty," cytoplasm. Immunohistochemistry of Hodgkin lymphoma usually exhibits positive CD30 cells, and most have CD15 positivity. CD45 staining is characteristically absent in Hodgkin lymphoma. The presence of "B symptoms" in patients with Hodgkin lymphoma is associated with an advanced stage of the disease and portends a poorer prognosis. Go to the next page if you knew the correct answer, or click the link image(s) below to further research the concepts in this question (if desired).
Research Concepts: Hodgkin Lymphoma
We update eBooks quarterly and Apps daily based on user feedback. Please tap flag to report any questions that need improvement.
Question 264:
A 67-year-old male presents to his primary care provider after noticing a swelling on the medial aspect of his thigh. According to the patient, the swelling has been increasing in size for approximately 5 months. The patient denies experiencing pain, fever, or night sweats. On exam, the swelling is a 6 cm non-fluctuant, nontender fixed nodule. Following MRI and tissue biopsy, a soft tissue sarcoma is diagnosed and later excised. The patient then undergoes a course of radiation. Seven months later, he develops a non-healing ulcer at the irradiated site. Which of the following would likely resolve the ulcer?
Choices: 1. Hyperbaric oxygen therapy (HBOT) at 2.0 to 2.4 ATA for 90 minutes five times per week 2. HBOT at 3.5 ATA for 2.5 hours five times per week 3. Clindamycin 300-450 mg orally three times daily for 10 days 4. Hospitalization followed by IV vancomycin twice daily for 7 days
Answer: 1 - Hyperbaric oxygen therapy (HBOT) at 2.0 to 2.4 ATA for 90 minutes five times per week
Explanations: Hyperbaric oxygen therapy (HBOT) should be initiated with daily treatments of 100% oxygen at 2.0 to 2.4 ATA for 90 minutes five times per week. It has been shown that approximately 80% of problem wounds resolve with HBOT. The risk of a patient suffering HBOT adverse effects increases if the patient is subjected to treatments lasting for more than 2 hours or to treatments where the pressure exceeds 3 atmospheres. While an infection could be preventing healing of the patient’s ulcer, soft-tissue radionecrosis commonly results from radiation-induced small vessel loss, and the ulcer would benefit from measures that increase angiogenesis. Go to the next page if you knew the correct answer, or click the link image(s) below to further research the concepts in this question (if desired).
Research Concepts: Hyperbaric Oxygen Effects On Angiogenesis
We update eBooks quarterly and Apps daily based on user feedback. Please tap flag to report any questions that need improvement.
Question 265:
A 60-year-old male with no significant past medical history presents for evaluation after a routine CBC showed a hemoglobin of 9 grams/dL and a platelet count of 76,000 platelets/microliter. On a review of his peripheral smear, mononuclear cells with circumferential hair-like cytoplasmic projections were noted. He has had a several month history of vague left upper quadrant pain and early satiety. Which of the following is most likely to be noted on his bone marrow biopsy or aspirate?
Choices: 1. A hypercellular bone marrow with greater than 20% myeloblasts 2. Dysplasia involving all lineages 3. Hypercellular marrow with a severe decrease in megakaryocytes only 4. "Dry tap" on bone marrow biopsy
Answer: 4 - "Dry tap" on bone marrow biopsy Explanations: Blasts in excess of 20% are expected in acute leukemias. This is not seen in hairy cell leukemia. Dysplasia involving any or all of the three hematopoietic lineages can be seen in myelodysplastic syndromes. This would not be expected in hairy cell leukemia. While marrow fibrosis is often encountered in hairy cell leukemia, a selective, severe decrease in megakaryocytes is not seen. A "dry tap" or inability to perform aspirate during bone marrow biopsy is often encountered in hairy cell leukemia. Go to the next page if you knew the correct answer, or click the link image(s) below to further research the concepts in this question (if desired).
Research Concepts: Hairy Cell Leukemia
We update eBooks quarterly and Apps daily based on user feedback. Please tap flag to report any questions that need improvement.
Question 266:
A 55-year old patient presents to the clinic with complaints of diarrhea and dry skin. He claims that his skin has been quite itchy lately and he seems to be more forgetful than usual. He did go to his general practitioner a few weeks ago for vague abdominal pain but was told that he had developed irritable bowel syndrome. He was started on vitamin B3 but his symptoms have persisted. All of the other routine blood work appears normal. You suspect that the patient has a lesion in his gastrointestinal tract. Endoscopic ultrasound will be of little use if this lesion is located in what region?
Choices: 1. 2. 3. 4.
Pancreas Antrum Duodenal bulb Proximal ileum
Answer: 4 - Proximal ileum Explanations: Endoscopic ultrasound is a good technique for identifying lesions in the gastrointestinal tract. However, the technique is unable to visualize lesions of the small bowel beyond the duodenum. The technique can also be used to detect anal lesions. Endoscopic ultrasound can be used to detect pancreatic masses. Go to the next page if you knew the correct answer, or click the link image(s) below to further research the concepts in this question (if desired).
Research Concepts: Intestinal Carcinoid Cancer
We update eBooks quarterly and Apps daily based on user feedback. Please tap flag to report any questions that need improvement.
Question 267:
A 22-year-old Asian female presents to the clinic with left lumbar pain for three months. She describes the pain is dull with no radiation. Further serology workup, including liver function tests and CA 19-9 tumor markers are unremarkable. Ultrasound suggests a cystic mass in the tail of the pancreas. It is presumed to be a benign neoplasm and planned for further evaluation to confirm the diagnosis. Based on the described clinical scenario, what are the findings expected on CT scan of the abdomen?
Choices: 1. Non encapsulated hypoattenuating mass in the head of the pancreas 2. Homogenous mass with vascular invasion in the tail of the pancreas 3. Heterogenous mass with peripheral calcification and traces of hemorrhage 4. Fluid collection consistent with necrotizing pancreatitis
Answer: 3 - Heterogenous mass with peripheral calcification and traces of hemorrhage
Explanations: The clinical vignette describes the signs and symptoms of solid pseudopapillary and epithelial neoplasm of the pancreas (SPEN). It is benign and indolent. Radiologically it is associated with signs of degeneration and described as a heterogeneous mass with peripheral calcification and traces of hemorrhage. The mass is encapsulated and commonly located in the tail of pancreas and not associated with invasive features such as involvement of surrounding vasculature. Go to the next page if you knew the correct answer, or click the link image(s) below to further research the concepts in this question (if desired).
Research Concepts: Solid and Papillary Epithelial Neoplasm
We update eBooks quarterly and Apps daily based on user feedback. Please tap flag to report any questions that need improvement.
Question 268:
A 27-year-old man comes to the clinic for evaluation after being unable to conceive with his wife. His testes are normal in size, shape, and consistency. No gynecomastia and other physical abnormalities are seen on physical examination. His Y chromosome shows microdeletions in the Yq11 region, and the diagnosis of Sertoli cell-only syndrome is established. Which of the following findings will most likely correspond to the patient's diagnosis?
Choices: 1. High luteinizing hormone (LH), normal testosterone, lowlevel inhibin-B, high follicle-stimulating hormone (FSH) 2. Normal luteinizing hormone (LH), low-level testosterone, low-level inhibin-B, low-level follicle-stimulating hormone (FSH) 3. Normal luteinizing hormone (LH), normal testosterone, low-level inhibin-B, normal follicle-stimulating hormone (FSH) 4. Normal luteinizing hormone (LH), normal testosterone, low-level inhibin-B, high follicle-stimulating hormone (FSH)
Answer: 4 - Normal luteinizing hormone (LH), normal testosterone, low-level inhibin-B, high follicle-stimulating hormone (FSH)
Explanations: Sertoli cells are essential for the proper growth and development of spermatogonia. They secrete antiMullerian hormone in males that prevents the development of female reproductive organs. Sertoli cells secrete inhibin-B. It down-regulates the level of follicle-stimulating hormone secretion (FSH). Activin, also released by Sertoli cells, increases the level of FSH. Most patients with Sertoli cell-only syndrome have an initial complaint of infertility and should be further evaluated by conducting a semen analysis. Azoospermia is a common finding, and only a very small percentage of patients may still have a low detectable level of sperm. Physical examination of Sertoli cell-only syndrome patients can show normal or smaller-sized testes with normal consistency and shape. Some patients have been reported to show atrophy of the testes. Go to the next page if you knew the correct answer, or click the link image(s) below to further research the concepts in this question (if desired).
Research Concepts: Sertoli Cell-Only Syndrome
We update eBooks quarterly and Apps daily based on user feedback. Please tap flag to report any questions that need improvement.
Question 269:
A 30-year-old man with no past medical history presents to the emergency department with multiple episodes of a generalized tonic-clonic seizure. His seizure stopped upon arrival. His vital signs are stable. Blood work shows a plasma glucose concentration of 50 mg/dL. His CT head is negative for acute disease. EEG is negative for seizure activity. He has no family history of endocrinopathies. Further, bloodwork shows insulin level 7 microU/mL, C peptide level 1 ng/mL, proinsulin level 8 pmol/L, beta-hydroxybutyrate level 0.3 mmol/L, and a negative sulfonylurea. CT abdomen shows a 1 cm solid mass in the pancreas, which is enhanced after contrast. He undergoes surgical resection of the mass and follows up with endocrinology six months post-resection. He has had no recurrence of the tumor on repeat imaging. He denies any recurrent symptoms. Which of the following is the next best step in managing this patient?
Choices: 1. Follow up with endocrinology again 12 months postresection 2. Follow up with endocrinology again 24 months postresection 3. Undergo genetic testing 4. Have family members screened for insulinomas
Answer: 3 - Undergo genetic testing Explanations: It is recommended that all young patients with insulinoma undergo genetic testing. MEN1 syndrome should be considered in patients with either family or/and a personal history of other endocrinopathies. Insulinoma patients without MEN1 should have followups at 3 to 6 months post-resection. Biochemical testing and imaging studies should be repeated at follow-ups. If cured, patients can be followed up as needed if there is a recurrence of symptoms. Go to the next page if you knew the correct answer, or click the link image(s) below to further research the concepts in this question (if desired).
Research Concepts: Insulinoma
We update eBooks quarterly and Apps daily based on user feedback. Please tap flag to report any questions that need improvement.
Question 270:
Mutated BRAF kinase is present in up to half of cases of metastatic melanoma. The development of inhibitors targeting BRAF, such as vemurafenib and dabrafenib, has been shown to increase median survival. Unfortunately, these therapies have also been frequently associated with the development of verrucal keratoses and cutaneous squamous cell carcinoma, especially within the first 12 to 24 weeks of treatment. Which of the following agents serves as a suitable chemopreventive agent against developing these lesions in patients on BRAF inhibitors?
Choices: 1. 2. 3. 4.
Acitretin Hydroxychloroquine Rifampin Thalidomide
Answer: 1 - Acitretin Explanations: Cutaneous squamous cell carcinoma (SCC) has been reported in 7% to 31% of patients on vemurafenib and dabrafenib, likely through paradoxical activation of the MAP kinase pathway via hyper-activating RAS mutations. Some patients require regular monitoring and multiple surgical interventions, impacting their quality of life and creating a burden for the medical system. Acitretin is a synthetic retinoid that acts on intracellular retinoic acid receptors mediating epidermal growth factor genes. It has been shown to be beneficial in chemoprevention of squamous cell carcinoma in patients on BRAF inhibitors, possibly through targeting of the MAPK pathway leading to the reduction of BRAF and pERK1/2 as seen in mouse models. There are no guidelines for management, but Acitretin should be considered a chemopreventive agent for SCC in patients taking BRAF inhibitors before considering dosage reductions. Go to the next page if you knew the correct answer, or click the link image(s) below to further research the concepts in this question (if desired).
Research Concepts: Acitretin
We update eBooks quarterly and Apps daily based on user feedback. Please tap flag to report any questions that need improvement.
Question 271:
Subcutaneous panniculitis-like T-cell lymphoma is a well-behaved lymphoid neoplasm of the subcutaneous tissue that can usually be successfully treated with systemic corticosteroids or immunosuppressive agents. However, a severe, life-threatening complication arises in approximately 20% of cases. This complication is classified as a hyperinflammatory syndrome that is associated with neoplastic, infectious, autoimmune, or hereditary diseases. What is the name of the syndrome?
Choices: 1. 2. 3. 4.
Sepsis Hemophagocytic syndrome (HPS) Thyroid storm Langerhans cell histiocytosis
Answer: 2 - Hemophagocytic syndrome (HPS) Explanations: Hemophagocytic syndrome (HPS) is also known as hemophagocytic lymphohistiocytosis (HLH). The hemophagocytic syndrome can be acquired, inherited, or immunodeficiency related. The acquired form occurs more commonly in adults, while the inherited and immunodeficiency related forms occur more frequently in infants and children. Clinically, it may present with fever, malaise, hepatosplenomegaly, and CNS symptoms, especially in children. Cytopenias are also common. Hemophagocytosis is not required for diagnosis. Hemophagocytic syndrome carries a 46% 5-year overall survival rate. These patients will require more aggressive treatment. Go to the next page if you knew the correct answer, or click the link image(s) below to further research the concepts in this question (if desired).
Research Concepts: Subcutaneous Panniculitis Like T-cell Lymphoma
We update eBooks quarterly and Apps daily based on user feedback. Please tap flag to report any questions that need improvement.
Question 272:
A 57-year-old female presented with increased abdominal girth and fatigue over the past two months. She admitted to intermittent right lower quadrant dull abdominal pain but otherwise has no apparent gastrointestinal symptoms. No significant past medical history or comorbidities were discovered. A computed tomography (CT) scan revealed a 5 cm appendiceal mass with ascites. A biopsy confirmed the diagnosis of pseudomyxoma peritonei. She then underwent appendicectomy, complete cytoreduction surgery, and hyperthermic intraperitoneal chemotherapy. Pathology was positive for high-grade mucinous carcinoma peritonei. The initial postoperative period was uneventful, but she came back at nine months postoperatively with progressive left lower quadrant abdominal pain and 15 lb (6.8 kg) of unintentional weight loss. CT scan revealed two pelvic masses, one measuring 4.5 cm in diameter, the other 2 cm, as well as a reoccurrence of localized ascites. What is the most appropriate next step in the management of this patient?
Choices: 1. Positron emission tomography(PET)/CT scan 2. Biopsy of the pelvic masses 3. Complete cytoreduction surgery and hyperthermic intraperitoneal chemotherapy 4. Systemic chemotherapy
Answer: 3 - Complete cytoreduction surgery and hyperthermic intraperitoneal chemotherapy
Explanations: Pseudomyxoma peritonei mostly originates from appendiceal mucinous carcinoma. Due to its indolent behavior, it is most frequently discovered at a relatively advanced stage. The current standard treatment is complete cytoreduction surgery and hyperthermic intraperitoneal chemotherapy (HIPEC). However, a higher recurrence rate is still frequently reported especially in those with high-grade lesions. So a baseline CT scan, as well as tumor markers, are recommended to be completed three months postoperatively and then every six months to monitor recurrence. Elective second look surgery is recommended when a recurrence is considered likely for selected patients. This patient has no significant past medical history and comorbidities. The recurrence remains generally localized. So she is a good candidate for second-look surgery. Repeat cytoreductive surgery is recommended for pseudomyxoma recurrences. In this case, the presence of ascites would suggest adding HIPEC. Lymphatic or hematogenic metastases are very rarely seen in pseudomyxoma peritonei, and this patient doesn't present with any symptoms suggesting an extraperitoneal invasion of tumors, so an expensive restaging PET scan is not necessary at this point. A biopsy is not necessary, because generally a CT scan is enough to monitor recurrence after index surgery and the biopsy is not likely to change the recommended therapy. No well-recognized systemic chemotherapy is
currently available for recurrent pseudomyxoma peritonei. Go to the next page if you knew the correct answer, or click the link image(s) below to further research the concepts in this question (if desired).
Research Concepts: Pseudomyxoma Peritonei
We update eBooks quarterly and Apps daily based on user feedback. Please tap flag to report any questions that need improvement.
Question 273:
A 62-year-old man has a syndrome characterized by severe gastric abdominal pain, dyspepsia, recurrent peptic ulcer disease, and diarrhea. The patient's serum gastrin level is 10 times higher than the normal upper limit. Computed tomography shows 1 cm pancreatic mass with the presence of enlarged peri-pancreatic lymph nodes and liver metastases, suggesting malignancy. Which drug should be used as a part of treating this patient's suspected condition?
Choices: 1. 2. 3. 4.
Bismuth subsalicylate Metronidazole Famotidine Omeprazole
Answer: 4 - Omeprazole Explanations: The goal of medical management of gastrinoma is to treat the symptoms and to prevent complications from peptic ulcer disease. The preferred medical therapy is the use of high-dose proton pump inhibitors (PPI) such as omeprazole. PPIs are preferred over H2 receptor blockers due to their higher potency and longer duration of action. Conservative treatment with PPIs also is recommended for patients who are unsuitable for surgery or patients with widespread metastasis. In about 50% of individuals, ulcer healing can occur within 4 weeks, and most people find symptom relief, including resolution of diarrhea. Go to the next page if you knew the correct answer, or click the link image(s) below to further research the concepts in this question (if desired).
Research Concepts: Gastrinoma
We update eBooks quarterly and Apps daily based on user feedback. Please tap flag to report any questions that need improvement.
Question 274:
An 83-year-old female was evaluated in the emergency department for progressive redness, pain, and decreased vision in the eyes bilaterally. The patient reports that she started on a medication four weeks before the onset of these symptoms. The patient does not recall the name of the drug. The patient has a history of gastroesophageal reflux disease and lung adenocarcinoma with metastasis to bone. Clinical examination and imaging were significant for stromal thinning of the inferior part of the cornea with a significant epithelial defect. Which of the following represents the mechanism of action of the offending drug?
Choices: 1. 2. 3. 4.
Reversibly inhibits epidermal growth factor receptor Reversibly inhibits fibroblast growth factor receptor Irreversibly inhibits epidermal growth factor receptor Irreversibly inhibits fibroblast growth factor receptor
Answer: 3 - Irreversibly inhibits epidermal growth factor receptor
Explanations: Given this clinical picture with ocular imaging results, drug-induced ulcerative keratitis is the most probable diagnosis in this patient. Since this patient has a history of lung adenocarcinoma, likely patient was started on tyrosine kinase inhibitors as they are the first line FDA-approved treatment in nonsmall cell lung cancer with EGFR mutation. This patient likely was started on afatinib. Bilateral ulcerative keratitis has been reported as part of its side effects. Recommendations are to carefully monitor for any ocular adverse events during treatment with afatinib. Afatinib, like other tyrosine kinase inhibitors irreversibly inhibit epidermal growth factor receptor. Go to the next page if you knew the correct answer, or click the link image(s) below to further research the concepts in this question (if desired).
Research Concepts: Afatinib
We update eBooks quarterly and Apps daily based on user feedback. Please tap flag to report any questions that need improvement.
Question 275:
A 4-year-old is brought to the emergency department with complaints of fatigue and undocumented fevers for a few days. He has previously been healthy, but his mother is worried that he looks pale. There have been no gastrointestinal symptoms, but he has woken several times in the past week with leg pain. His gums have been bleeding after brushing his teeth. The child is pale, has diffuse lymphadenopathy, splenomegaly, and scattered petechiae. There is no hepatomegaly or other findings. CBC shows WBC of 9,000/microliter, hemoglobin 8.7 g/dL, hematocrit of 24%, and platelets 8,000/microliter. An automated differential shows many atypical lymphocytes. What is the next best step in management?
Choices: 1. 2. 3. 4.
Anti-parvovirus B19 antibody titers Anti-Epstein Barr virus antibody titers Bone marrow biopsy Abdominal ultrasound
Answer: 3 - Bone marrow biopsy Explanations: The child most likely has leukemia, with acute lymphoblastic leukemia being the most common. The severe anemia and thrombocytopenia probably indicate bone marrow failure. The WBC count can be high, low, or normal. Bone marrow biopsy is the procedure of choice. A manual examination of the peripheral smear would reveal that the atypical lymphocytes are blast forms. Go to the next page if you knew the correct answer, or click the link image(s) below to further research the concepts in this question (if desired).
Research Concepts: Acute Lymphocytic Leukemia
We update eBooks quarterly and Apps daily based on user feedback. Please tap flag to report any questions that need improvement.
Question 276:
A 42-year old female presents with a 4month history of a left-sided neck mass, that she ignored being painless. She says she is now worried, as the mass has not disappeared. She has no other symptoms. On exam, there is a 5 cm mass on the left side of the neck just below the thyroid cartilage. The nodule appears fixed to the surrounding tissues and appears to move with the act of swallowing. The clinician performed a fine needle aspiration that reveals atypical cells with intranuclear clearings. The nuclear scan suggests a cold nodule. Mutations of which gene have been linked to the development of this lesion?
Choices: 1. 2. 3. 4.
C-abl RET proto-oncogene HER/Neu Myc
Answer: 2 - RET proto-oncogene Explanations: Papillary thyroid cancer is strongly associated with some specific rearrangements of the RET proto-oncogene. The intranuclear clearings, often referred to as "orphan Annie eye" nuclei, are a hallmark cytologic finding of papillary thyroid carcinoma. If possible, the assessment of the RET proto-oncogene expression should be performed in anyone having a relative with a history of medullary thyroid cancer. The RET proto-oncogene encodes a receptor tyrosine kinase for many cells. Loss of function mutation in the RET proto-oncogene has been linked to Hirschsprung disease. Go to the next page if you knew the correct answer, or click the link image(s) below to further research the concepts in this question (if desired).
Research Concepts: Medullary Thyroid Cancer
We update eBooks quarterly and Apps daily based on user feedback. Please tap flag to report any questions that need improvement.
Question 277:
A 2-year-old boy presents with rightsided abdominal swelling that his mother first noticed in the bathtub 2 months ago. It has been increasing in size, and the boy now has a decreased appetite and mild pain in the area. Ultrasound and magnetic resonance imaging were performed and demonstrated a solitary liver mass. To stage this tumor, what is the best next step?
Choices: 1. 2. 3. 4.
Perform a biopsy Screen for hepatitis Chest CT Perform liver function tests
Answer: 3 - Chest CT Explanations: A chest CT can help detect lung metastases, as up to 20% of hepatoblastoma cases present with metastases. The lung is the most common site of metastases. Ultrasound and either CT or MRI are the imaging modalities used to define the extent of tumor involvement of the liver and aid in pre-surgical planning. The presence or absence of metastases is important in the staging process. Go to the next page if you knew the correct answer, or click the link image(s) below to further research the concepts in this question (if desired).
Research Concepts: Hepatoblastoma
We update eBooks quarterly and Apps daily based on user feedback. Please tap flag to report any questions that need improvement.
Question 278:
A 42-year-old patient presents with increasing sporadic episodes of vomiting. On physical examination, he shows papilledema. Neuroimaging reveals a tumor arising from the ventricular lining of the fourth ventricle; histology reveals papillary structures. Which of the following is the most common treatment for the most likely tumor in this patient?
Choices: 1. 2. 3. 4.
Chemoradiotherapy Complete resection with adjuvant radiotherapy Radiotherapy only Symptomatic treatment and comfort measures only
Answer: 2 - Complete resection with adjuvant radiotherapy
Explanations: Patients who present with vomiting and papilledema on physical examination should raise suspicion for intracranial hypertension. This patient has a tumor that shows papillary structures on histology and arises from the ventricular lining which is most consistent with ependymoma. Treatment of ependymoma most commonly consists of total resection and adjuvant radiation. Chemotherapy has not demonstrated a clear benefit in the treatment of ependymoma. Go to the next page if you knew the correct answer, or click the link image(s) below to further research the concepts in this question (if desired).
Research Concepts: Ependymoma
We update eBooks quarterly and Apps daily based on user feedback. Please tap flag to report any questions that need improvement.
Question 279:
A 60-year-old man is hospitalized with vague upper abdominal pain for the last three months. On further inquiry, he reports decreased appetite and an unintentional 20-pound (>10%) weight loss. Vitals at admission were within the normal range. His examination is significant for bitemporal muscle wasting. Blood tests show hemoglobin 14 gm/dL, white blood cell count 6300/mm3, platelets 230000/mm3, LDH 200 IU/L (100-199 IU/L), beta-2 microglobulin 2.5 mg/L (3 mg/L), and normal renal and thyroid function tests. Following a CT scan showing a 2 cm space-occupying liver lesion, a biopsy is performed that reveals findings consistent with a primary hepatic lymphoma. Which of the following patient factors will best determine his response to the most appropriate treatment?
Choices: 1. 2. 3. 4.
LDH 200 IU/L Beta-2 microglobulin 2.5 mg/L 2 cm tumor mass 20-pound weight loss
Answer: 4 - 20-pound weight loss Explanations: The commonly reported poor predictors of primary hepatic lymphoma's response to combination chemotherapy are the presence of constitutional symptoms (notably fever, drenching night sweats, and weight loss), LDH >10% of the upper limit of normal, beta-2 microglobulin of more than 3 mg/L, tumor mass >7 cm and Ann Arbor stage III or IV disease. The presence of one pretreatment risk factor determines the response to combination chemotherapy. Commonly primary hepatic lymphoma is considered to be a chemosensitive tumor, and tumor response exceeds 80% with combination chemotherapy. The first line of conventional chemotherapy involves cyclophosphamide, doxorubicin, vincristine, and prednisone (CHOP). The presence of one or more pretreatment risk factors requires alternating triple combination chemotherapy rather than CHOP therapy. Alternating triple combination chemotherapy includes doxorubicin, methylprednisolone, cytosine arabinoside, and cisplatin, alternating with methotrexate, bleomycin, cyclophosphamide, doxorubicin, vincristine, and methylprednisolone, alternating with mesna, ifosfamide, mitoxantrone, and etoposide. Go to the next page if you knew the correct answer, or click the link image(s) below to further research the concepts in this question (if desired).
Research Concepts:
Hepatic Lymphoma
We update eBooks quarterly and Apps daily based on user feedback. Please tap flag to report any questions that need improvement.
Question 280:
A 5-year-old male is brought in by his parents with failure to thrive and general irritability. He is adopted, and his parents do not have access to his past medical and family history. On examination, there are firm cervical lymph nodes. Flow cytometry demonstrates a CD10 positive surface light chain restricted B-cell population. Chromosomal analysis reveals a translocation between chromosomes 8 and 14, t(8;14)(q34;q32). What is the most likely immunophenotype of this patient's condition?
Choices: 1. 2. 3. 4.
CD20+, CD45+, CP19+, PAX5+, CD10+, BCL6+, BCL2CD20+, CD45+, CD19+, CD5+, CD79a+, PAX5 + CD20+, TdT+, CD19+, PAX5+, CD10+ CD5+, CD20+, CD45+, CD19+, PAX5+
Answer: 1 - CD20+, CD45+, CP19+, PAX5+, CD10+, BCL6+, BCL2-
Explanations: Burkitt lymphoma is an aggressive B-cell lymphoma comprising a monomorphic population of intermediatesized mature lymphocytes. Immunohistochemistry and cytogenetics play a significant role in the diagnosis and management of Burkitt lymphoma. The malignant B-cells express surface IgM. The cells are positive for B-cell markers, including CD19, CD20, CD79a, and PAX5. They are positive for germinal center markers CD10 and BCL-6 but are negative for BCL-2. The neoplastic cells do not express T-cell markers and do not express immature markers TdT or CD34. Rapid cell turnover is reflected by high Ki-67 positivity nearing 100%, which is a very helpful diagnostic clue. Go to the next page if you knew the correct answer, or click the link image(s) below to further research the concepts in this question (if desired).
Research Concepts: Burkitt Lymphoma
We update eBooks quarterly and Apps daily based on user feedback. Please tap flag to report any questions that
need improvement.
Question 281:
A 16-year-old female presents with a soft tissue mass around her right shoulder for the past three months. She denies any recent increase in the size of the swelling. On exam, the mass is nontender, and a bruit is heard when the mass is auscultated. MRI demonstrates a lytic lesion with low attenuating fluid levels. Pathology reveals empty cystic spaces resembling aneurysmal bone cysts and septa populated by pleomorphic cells. The diagnosis is made, and treatment is started. Which of the following best describes medical management of the suspected diagnosis?
Choices: 1. 2. 3. 4.
No significant response to chemotherapy Moderate response to chemotherapy Better response than the conventional disease Chemotherapy is utilized for recurrence only
Answer: 3 - Better response than the conventional disease
Explanations: Telangiectatic osteosarcoma usually shows a better response to chemotherapy than conventional osteosarcoma. Survival rates with current chemotherapy protocols are within the range of 65% after five years. Chemotherapeutic protocols for telangiectatic osteosarcoma are similar to those of conventional osteosarcoma. Patients will have two to six cycles of chemotherapy administered before surgery. The common chemotherapeutic agents used include cisplatin, carboplatin, methotrexate, doxorubicin, and ifosfamide. Among these, at least two agents need inclusion in a dual-drug therapy regimen. Go to the next page if you knew the correct answer, or click the link image(s) below to further research the concepts in this question (if desired).
Research Concepts: Telangiectatic Osteosarcoma
We update eBooks quarterly and Apps daily based on user feedback. Please tap flag to report any questions that need improvement.
Question 282:
A 69-year-old female presents to the hospital with right upper quadrant pain and nausea. She reports decreased appetite and weight loss of 25 lbs over the past six months. Her physical examination is significant for hepatomegaly. Vital signs are within normal limits. Labs are significant for hemoglobin 10.1 g/dL, platelet count of 89000/microL, ALT 120 IU/L and AST 115 IU/L with ALP 220 IU/L and bilirubin levels of 2.7 mg/dL. CT abdomen with contrast reveals multiple hypodense liver lesions, the largest being a 6 cm; the lesions have areas of hyperdensities after contrast. She also has multiple enhancing lesions in her spleen. She undergoes a CT chest that shows a 3 cm right upper lobe lung nodule. Liver biopsy reveals evidence of solid spindle cell proliferation with areas of hemorrhage and necrosis. Immunohistochemistry is positive for CD31 and CD34. What is the best next step in the management of this patient?
Choices: 1. 2. 3. 4.
Surgical resection Chemotherapy Liver transplantation Referral for palliative care
Answer: 4 - Referral for palliative care Explanations: This patient likely has angiosarcoma of the liver with metastasis to the spleen and lung. Angiosarcoma usually presents with nonspecific symptoms including abdominal pain and ascites or can be asymptomatic and found incidentally on imaging. A physical examination can reveal hepatosplenomegaly. It is associated with elevations in liver enzymes as well as thrombocytopenia and anemia if the tumor burden is large and disease is extensive. Imaging usually shows a hypodense lesion or multiple lesions on non-enhanced CT with variable enhancement on contrast CT based on the presence of hemorrhage and necrosis. Diagnosis is usually confirmed with histopathology showing spindle cells with possible areas of necrosis, infarction, and hemorrhage. CD31, CD34, Ulex europaeus agglutinin I, and factor VIII-related antigen are immunohistochemical markers that can be used to aid diagnosis. It is an aggressive tumor and usually metastasizes to the lungs, hilar lymph nodes, spleen and bones with a poor prognosis and survival of less than one year even after surgical resection. This patient has extensive metastatic disease and would not benefit from surgery or liver transplantation. Chemotherapy has been used in metastatic disease; however, it has not proven to be effective in treatment. This patient will benefit from palliative care at this point. This patient has extensive metastatic disease and would not benefit from surgery. Chemotherapy has been used in metastatic disease; however, it has not proven to be
effective in treatment. This patient has extensive metastatic disease and would not benefit from liver transplantation. Go to the next page if you knew the correct answer, or click the link image(s) below to further research the concepts in this question (if desired).
Research Concepts: Liver Angiosarcoma
We update eBooks quarterly and Apps daily based on user feedback. Please tap flag to report any questions that need improvement.
Question 283:
A 54-year-old housewife discovered a lump in her left breast accidentally when she was showering. Clinically, it is a 2 x 3 cm lump that is mobile and firm. No palpable axillary swellings. Mammography reveals granular microcalcifications with normal architecture. Truecut biopsy shows fibroadenosis with areas of large abnormal nuclei with no basement membrane invasion. What is the appropriate treatment?
Choices: 1. 2. 3. 4.
Modified radical mastectomy Breast-conserving surgery Follow up with tamoxifen Simple mastectomy followed by radiotherapy
Answer: 2 - Breast-conserving surgery Explanations: Ductal carcinoma in situ (DCIS) although noninvasive but it may contain invasive components. It is treated with breast-conserving surgery. If margins are positive, sentinel lymph node biopsy is needed. DCIS may be present in four forms including papillary, cribriform, comedo, and solid form. Papillary and cribriform are low-grade forms of ductal carcinoma in situ, while comedo and solid are considered high grade. Go to the next page if you knew the correct answer, or click the link image(s) below to further research the concepts in this question (if desired).
Research Concepts: Breast Cancer
We update eBooks quarterly and Apps daily based on user feedback. Please tap flag to report any questions that need improvement.
Question 284:
A 62-year-old female presents to the clinic for a follow after she was found to have an incidental 6 cm liver mass on Imaging. She reports right upper quadrant pain. Her physical examination is remarkable for hepatomegaly. Vital signs are within normal limits. Labs are significant for ALT 90 IU/L and AST 79 IU/L with ALP 180 IU/L and bilirubin levels of 2.4 mg/dL. CT liver protocol shows a 6 cm hypodense lesion with enhancement after contrast. Liver biopsy shows evidence of spindle cell proliferation with areas of hemorrhage. What is the most likely site of metastasis of this tumor?
Choices: 1. 2. 3. 4.
Lung Breast Brain Stomach
Answer: 1 - Lung Explanations: This patient likely has angiosarcoma of the liver. Angiosarcoma usually presents with nonspecific symptoms including abdominal pain and ascites or can be asymptomatic and found incidentally on imaging. It is associated with elevations in liver enzymes as well as thrombocytopenia and anemia if the tumor burden is large and disease is extensive. Imaging usually shows a hypodense lesion or multiple lesions on non-enhanced CT with variable enhancement on contrast CT based on the presence of hemorrhage and necrosis. Diagnosis is usually confirmed with histopathology showing spindle cells with possible areas of necrosis, infarction, and hemorrhage. It is an aggressive tumor that usually metastasizes lungs, hilar lymph nodes, spleen, and bones. Breast, brain, and stomach are not common sites of metastasis for liver angiosarcoma. Go to the next page if you knew the correct answer, or click the link image(s) below to further research the concepts in this question (if desired).
Research Concepts: Liver Angiosarcoma
We update eBooks quarterly and Apps daily based on user feedback. Please tap flag to report any questions that need improvement.
Question 285:
A 65-year-old male patient presented with a one-month history of swelling in the midline of the neck which was associated with pain, dysphagia, and hoarseness of voice. Physical examination showed an enlarged thyroid gland and bilateral neck lymph nodes. Ultrasonography revealed an ill-defined large hypoechoic mass in the left lobe of thyroid. Fine needle aspiration cytology was performed. The smears showed neoplastic cells arranged in loose clusters as well as dispersed singly in the necrotic background. The individual cells showed identifiable malignant cytologic features including large, pleomorphic nuclei with irregular nuclear membranes, coarse chromatin, prominent nucleoli, and several mitotic figures. What is the most likely diagnosis?
Choices: 1. 2. 3. 4.
Papillary thyroid carcinoma Follicular carcinoma Medullary carcinoma Anaplastic thyroid carcinoma
Answer: 4 - Anaplastic thyroid carcinoma Explanations: The diagnosis of anaplastic thyroid carcinoma is most commonly suspected on ultrasonography and clinical examination. The first diagnostic modalities in the evaluation of anaplastic thyroid carcinoma are ultrasonography and fine needle aspiration biopsy. Cytologic findings of anaplastic thyroid carcinoma include (1) biphasic population tumor and uninvolved thyroid, (2) highly cellular with single cells and focal clusters composed of remarkable atypical cells, (3) mitotic figures are prominent, and (4) background necrosis and inflammatory cells may be seen. In previous reports, the cytological diagnosis accuracy of anaplastic thyroid carcinoma was 78.7% to 90%. Fine-needle biopsy is a useful diagnostic modality for anaplastic thyroid carcinoma. However, the diagnosis of anaplastic thyroid carcinoma must be established by surgical biopsy or at surgery. Go to the next page if you knew the correct answer, or click the link image(s) below to further research the concepts in this question (if desired).
Research Concepts: Anaplastic Thyroid Cancer
We update eBooks quarterly and Apps daily based on user feedback. Please tap flag to report any questions that need improvement.
Question 286:
A 58-year-old female with a history of cT1cN0M0 invasive ductal carcinoma of the breast, ER/PR+ HER2- treated with mastectomy and sentinel lymph node biopsy with 1/4 nodes involved, followed by adjuvant chest wall irradiation with regional node irradiation, presents to the clinic with chest wall recurrence. Which of the following is the most appropriate rationale for re-irradiation with hyperthermia in this patient?
Choices: 1. A complete pathologic response rates range from 80% to 90% for combination of hyperthermia and radiation 2. An improvement in benefit in patients with recurrent lesions in previously irradiated areas since concurrent hyperthermia enhanced lower doses of repeat radiation therapy 3. A decreased rate of toxicity to the bone marrow 4. A more durable local control rate in patients who only received radiation
Answer: 2 - An improvement in benefit in patients with recurrent lesions in previously irradiated areas since concurrent hyperthermia enhanced lower doses of repeat radiation therapy
Explanations: The addition of hyperthermia to radiation therapy has been shown to achieve complete response rates ranging from 55% to 66%, much higher than the 16% to 47% seen with chemoradiotherapy. Acute toxicities associated with combined radiation therapy and hyperthermia have been reported as persistent ulceration seen in patients with prior radiation, blisters, second and third-degree burns, and infection. The addition of chemotherapy to cancer treatment increases the risk of bone marrow toxicity. Local control was found to be more durable with the addition of hyperthermia to radiation. The most significant benefit of combining hyperthermia to radiation in previously radiated patients is observed in patients with recurrent lesions in previously irradiated areas, where the dose of radiation is lowered due to prior radiation treatment and concern of toxicity to nearby normal tissues, thus showing that the synergy of hyperthermia helps the lower doses of radiation reach a higher therapeutic ratio leading to local control. Go to the next page if you knew the correct answer, or click the link image(s) below to further research the concepts in this question (if desired).
Research Concepts:
Hyperthermia for Chest Wall Recurrence
We update eBooks quarterly and Apps daily based on user feedback. Please tap flag to report any questions that need improvement.
Question 287:
A 21-year-old male presents to the clinic for a regular follow-up. He was treated for acute myeloid leukemia three years ago with chemotherapy, and complete clinical remission was achieved after three courses. Later, however, the patient relapsed within a year, and although he survived, the second remission was challenging to achieve. Which of the following is the best next step in the management of this patient?
Choices: 1. Gene therapy followed by treatment with anti-CD45 monoclonal antibody 2. Supporting immunotherapy with granulocyte colonystimulating factor and platelet transfusion 3. Bone marrow transplantation followed by treatment with methotrexate 4. Treatment with gemtuzumab ozogamicin
Answer: 3 - Bone marrow transplantation followed by treatment with methotrexate
Explanations: Bone marrow transplantation is considered in a young patient with suitable family members to treat acute myeloid leukemia, and methotrexate is used intermittently to prevent graft-versus-host disease (GVHD), which is a usual complication of stem cell transplantations. GVHD after bone marrow transplantation has high morbidity and mortality, and it can be minimized using highly compatible or HLA identical marrow donors. Irradiation of bone marrow cells is done to prevent a GVHD after transplantation. The bone marrow transplantation is achieved by giving cyclophosphamide (120 mg/kg) followed by total body irradiation. After that, an intravenous transfusion of 10^9 unfractionated bone marrow cells per kilograms is given, which is obtained from a related family or HLA-compatible donor. The patient is supported with platelet transfusions and with granulocyte colony-stimulating factor (G-CSF) during the aplasia before the engraftment has occurred. Go to the next page if you knew the correct answer, or click the link image(s) below to further research the concepts in this question (if desired).
Research Concepts: Graft Versus Host Disease
We update eBooks quarterly and Apps daily based on user feedback. Please tap flag to report any questions that need improvement.
Question 288:
A 65-year-old man presents to the clinic complaining of a growing and bleeding lesion on his right lower leg. The lesion appeared two months ago and has been steadily growing and started bleeding a week ago, which prompted the appointment. On physical exam, a 1.2 cm hyperkeratotic nodule with hemorrhagic crust on the right anterior shin is noted along with numerous 3-5 mm annular lesions with a raised border on the bilateral legs. After a biopsy is performed. Which of the following is the most accurate statement regarding the patient's condition?
Choices: 1. The cause is viral and is self-limited, the biopsy is to confirm the diagnosis 2. The patient has a furuncle caused by bacteria and the treatment option is an oral and topical antibiotic for 2 weeks 3. It is an irritated seborrheic keratosis; the biopsy is done to confirm the diagnosis and the spot can be treated with cryotherapy 4. The lesion is most likely a skin cancer arising from premalignant lesions and patient will be referred to Mohs micrographic surgery
Answer: 4 - The lesion is most likely a skin cancer arising from premalignant lesions and patient will be referred to Mohs micrographic surgery
Explanations: The lesion is most likely a squamous cell carcinoma arising from the porokeratosis but it could also have arisen from years of sun exposure. Squamous cell carcinoma on the anterior shin warrants a referral to Mohs micrographic surgery for removal and repair. After surgery, his disseminated superficial actinic porokeratosis (DSAP) can be treated topically to help prevent progression to malignancy. Topical treatments include topical imiquimod, 5-FU, vitamin D analogs, and diclofenac. Go to the next page if you knew the correct answer, or click the link image(s) below to further research the concepts in this question (if desired).
Research Concepts: Disseminated Superficial Actinic Porokeratosis
We update eBooks quarterly and Apps daily based on user feedback. Please tap flag to report any questions that need improvement.
Question 289:
A 40-year-old female with no significant past medical history presents with nonspecific respiratory symptoms. A chest x-ray shows a proximal mass. A CT scan shows a polypoid endobronchial mass measuring 2 cm. Endoscopic resection of the mass is performed. The microscopic examination reveals a polypoid mass covered by squamous epithelium without atypical cells or invasion. What is the most likely diagnosis?
Choices: 1. 2. 3. 4.
Squamous cell carcinoma Squamous cell papilloma Glandular papilloma Mixed papilloma
Answer: 2 - Squamous cell papilloma Explanations: Squamous cell carcinoma is a malignant epithelial tumor characterized by an infiltrative growth with nests of atypical cells and squamoid pearls. Squamous cell papilloma is a benign tumor with a fibrovascular core covered by regular squamous epithelium. Glandular papilloma is a benign tumor with a fibrovascular core covered by respiratory pseudostratified epithelium. Mixed papilloma is a benign tumor that shares the features of squamous cell and glandular papillomas. Go to the next page if you knew the correct answer, or click the link image(s) below to further research the concepts in this question (if desired).
Research Concepts: Pulmonary Papilloma
We update eBooks quarterly and Apps daily based on user feedback. Please tap flag to report any questions that need improvement.
Question 290:
A 43-year-old male recently returned from a trip to South America 2 weeks ago. He has a past medical history of non-insulin dependent type 2 diabetes mellitus, hypertension, and depression. He has always been very compliant with his medications. While talking about his vacation, he casually mentions he continued monitoring his glucose, which was consistently higher than normal, despite taking medications as prescribed. Patient states he attributed it to eating a little more since he was on vacation. On this visit, he is complaining of diarrhea, which is not relieved by over-the-counter antidiarrheal medication. Upon further questioning, he reports some weight loss and stomach pain, which began several months ago, but the pain has not been severe, and the patient was glad to be losing weight. Which of the following is the most likely cause of the patient's symptoms?
Choices: 1. 2. 3. 4.
Worsening diabetes mellitus Giardia lamblia infection Somatostatinoma Peptic ulcer disease
Answer: 3 - Somatostatinoma Explanations: There are several clues in this stem which lead away from an infection and towards other pathology. The patient has several months of abdominal pain and weight loss, glucose intolerance, despite no change in medication and refractory diarrhea. These symptoms are most likely due to a somatostatinoma. Somatostatinoma's most commonly present with weight loss and stomach pain; however, it is possible to have somatostatinoma syndrome. This most commonly presents with diabetes mellitus, glucose intolerance, diarrhea, steatorrhea, and cholelithiasis. Somatostatinomas can have a variable presentation due to the various physiologic effects of the hormone in multiple organs of the body. Understanding the physiology of somatostatin and how it effects organs will help catch some of the more peculiar and seldom seen symptoms of somatostatinoma, such as gallstones, jaundice, bowel obstruction, and achlorhydria. Once somatostatin binds to its receptor, it produces its physiologic effect, which is primarily inhibition of various hormones. Acetylcholine, arginine vasopressin, cholecystokinin, epidermal growth hormone, glucagon, glucose-dependent insulinotropic peptide, gastrin, growth hormone, insulin, motilin, neurotensin, pancreatic polypeptide, secretin, serotonin, substance P, thyrotropin, and vasoactive intestinal polypeptide are all inhibited by the effects of somatostatin. The pancreas will have both endocrine and exocrine secretions inhibited. Finally, within the gastrointestinal system, somatostatin will inhibit salivary amylase, gastric acid,
and gastrointestinal hormone secretions. It also delays gastric emptying, slows motility, inhibits absorption, and decreases splanchnic blood flow. Go to the next page if you knew the correct answer, or click the link image(s) below to further research the concepts in this question (if desired).
Research Concepts: Somatostatinoma
We update eBooks quarterly and Apps daily based on user feedback. Please tap flag to report any questions that need improvement.
Question 291:
A 65-year-old male with a history of diabetes mellitus type 2 and hypertension complains of right shoulder pain that started 10 days ago and lower back pain that started 3 days ago. He says the pain in his shoulder is new, but he has been having lower back pain for 8 months. He adds that the lower back pain has increased in intensity during the past 3 days. He smokes 2 packs of cigarettes daily for the past 20 years and drinks 2 beers every week. Vital signs show blood pressure of 132/83 mmHg, heart rate of 84 beats per minute, respiratory rate of 17 breaths per minute, and oxygen saturation of 97% on room air. A chest x-ray reveals a mass in the apex of the right lung. Which of the following findings would indicate advanced disease and require immediate intervention?
Choices: 1. 2. 3. 4.
Elevated blood calcium levels Asymmetric pupil size Right-hand muscle atrophy, weakness, and paresthesia Right lower limb hyperreflexia
Answer: 4 - Right lower limb hyperreflexia Explanations: This patient presenting with shoulder pain, and a mass in the apex of the lung in the setting of a long history of heavy smoking likely has a Pancoast tumor, also called superior sulcus tumor. The superior sulcus region of the lung has been used in the past to refer to structures in the apex of the lung. A superior sulcus tumor is a lung tumor, usually a subset of non-small cell lung cancers, that arises in the superior sulcus area. The most common presenting symptom of Pancoast tumor is shoulder pain, which is due to the tumor invading the brachial plexus and extending into the parietal pleura. Depending on the behavior of the tumor, other clinical manifestations may be present. If the tumor invades the paravertebral and inferior cervical sympathetic ganglia, the patient will present with Horner syndrome (ptosis, miosis, anhidrosis). If the tumor invades the C8 to T1 brachial plexus roots, the patient will present with weakness and atrophy of the intrinsic muscle of the hand and pain and paresthesia of the 4th and 5th digits, forearm, and medial arm. Supraclavicular lymph node enlargement and weight loss are also commonly seen in patients with Pancoast tumors. Pancoast tumors metastasize to the intervertebral foramina and cause spinal cord compression in about 25% of patients that can manifest as lower extremity hyperreflexia. New onset or worsening back pain in patients with a Pancoast tumor suggests that the tumor may have spread to the spinal cord. Early diagnosis and
appropriate treatment are crucial to preserving the patient's neurological function. Since there is a wide variety of tumors that can arise in the superior sulcus, histologic diagnosis is required before initiating treatment. Hence, a core needle biopsy is required for a definitive diagnosis. The overwhelming majority of Pancoast tumors are non-small cell cancers of the lung. This includes squamous cell carcinoma, adenocarcinoma, and large cell carcinoma. Although most superior sulcus tumors are non-small cell lung cancers, less than 5% of non-small cell lung cancers arise in that area. Go to the next page if you knew the correct answer, or click the link image(s) below to further research the concepts in this question (if desired).
Research Concepts: Lung Pancoast Tumor
We update eBooks quarterly and Apps daily based on user feedback. Please tap flag to report any questions that need improvement.
Question 292:
A 65-year-old female with a past medical history of hypertension, type 2 diabetes mellitus, osteoporosis, and known breast cancer status post-surgical intervention and chemotherapy, presents for an acute visit. She reports having right hip pain for the last few weeks but had an acute increase in this pain when she was walking yesterday. She denies any other trauma to the area and states that anti-inflammatories have not improved her pain. Her current medications include simvastatin, lisinopril, hydrochlorothiazide, metformin, trastuzumab, and denosumab. Physical exam reveals a shortened and externally rotated right lower extremity with tenderness to palpation on the lateral aspect of her hip. A plain x-ray reveals a subtrochanteric transverse fracture. By what mechanism of action has this pathology occurred?
Choices: 1. Increased osteoclast function only 2. Decreased osteoclast function only 3. Increased osteoclast function and decreased osteoblast function 4. Decreased osteoclast function and increased osteoblast function
Answer: 2 - Decreased osteoclast function only Explanations: Denosumab is a competitive RANKL inhibitor and functions to decrease bone resorption by decreasing osteoclast function. Although the incidence is low, atypical femoral fractures have been reported in patients receiving denosumab. The mechanism by which denosumab causes atypical femoral fractures is counterintuitive. Denosumab decreases osteoclast function and therefore decreases bone resorption and increases bone mineral density. This, in turn, should cause a decrease in fractures in those with a high risk of fracture. However, prolonged osteoclast inhibition can cause a decrease in targeted remodeling of bone with microcracks. Microcracks are a part of the normal physiology of bone and are usually resorbed by osteoclasts and replaced with new bone. When osteoclasts are inhibited, microcracks in the bone accumulate, which leads to an atypical fracture. Bone metastases from a solid tumor have the possibility of causing bone pain and fractures. The pathophysiology of these fractures is the increase of both osteoclast and osteoblast function. Therefore if metastases from her primary breast cancer caused the patient's fracture, an increase in both osteoclast and osteoblast function would be the correct answer. Atypical femoral fractures typically have no or minimal trauma associated with them. They are typically located in the subtrochanteric region or on the femoral shaft. The fractures are typically defined as transverse or short oblique fractures.
Go to the next page if you knew the correct answer, or click the link image(s) below to further research the concepts in this question (if desired).
Research Concepts: Denosumab
We update eBooks quarterly and Apps daily based on user feedback. Please tap flag to report any questions that need improvement.
Question 293:
A 53-year-old woman presents to the clinic with a left-breast lump. On physical examination, the patient has a well-demarcated mass located in the upper outer quadrant of her left breast with a palpable unilateral axillary lymph node. Mammography and ultrasonography depict a well-defined heterogeneous lesion measuring 6-cm in diameter without calcifications. Fine-needle aspiration cytology of the lesion reveals highly cellular smears containing large atypical cells arranged in syncytial sheets and intimately admixed with lymphocytes and neutrophils. The nuclear to cytoplasmic ratio was high in tumor cells whose nucleus exhibited coarse chromatin with one or more conspicuous nucleoli. Which of the following histological criteria can be appreciated on the macroscopic and histological examination of the surgical specimen and not on smears to establish the diagnosis of medullary carcinoma?
Choices: 1. 2. 3. 4.
Moderate to marked nuclear pleomorphism High mitotic index Moderate to marked diffuse lymphoplasmacytic infiltrate Complete histologic circumscription
Answer: 4 - Complete histologic circumscription Explanations: Complete histological circumscription of medullary breast carcinoma could only be evaluated on macroscopic and histopathological examination of the surgical specimen. Cytological examination of the smears alone is not sufficient to establish the definitive diagnosis of medullary breast carcinoma. Medullary carcinoma should meet all of the following five morphologic criteria as defined by the WHO (World Health Organization): 1. Syncytial growth pattern in more than 75% of the tumor. 2. No glandular or tubular structures, even as a minor component. 3. Moderate to marked diffuse lymphoplasmacytic infiltrate in the stroma. 4. Moderate to marked nuclear pleomorphism. 5. Complete histologic circumscription. Grossly, medullary carcinoma is well circumscribed and moderately firm. The tumor is often well-defined clinically and on imaging studies. Go to the next page if you knew the correct answer, or click the link image(s) below to further research the concepts in this question (if desired).
Research Concepts: Medullary Breast Carcinoma
We update eBooks quarterly and Apps daily based on user feedback. Please tap flag to report any questions that need improvement.
Question 294:
A 38-year-old patient presents with a history of a total hysterectomy 1 year ago for leiomyomas and menorrhagia. She has no history of abnormal Pap smears or lower genital tract cancers. Which of the following strategies for follow-up is indicated?
Choices: 1. Vaccination against high-risk human papillomavirus (HPV) infection 2. Discontinue screening with Pap tests or HPV screening 3. Vaginal cytology and HPV screening every 5 years 4. Vaginal cytology every 3 years
Answer: 2 - Discontinue screening with Pap tests or HPV screening
Explanations: Vaginal cancer is extremely rare, and this patient's risk of developing it is low. There is no screening test for vaginal cancer. If this patient had a history of cervical intraepithelial neoplasia (CIN) 2 or 3, then continued screening is indicated for 20 years past the point of treatment for the CIN. Vaccination for human papillomavirus (HPV) is indicated for individuals up to 26 years of age. The Advisory Committee on Immunization Practices recommends HPV vaccination based on shared clinical decision-making for individuals ages 27 through 45 years who are not adequately vaccinated. For women who have had a hysterectomy, including removal of the cervix, and have no history of a highgrade precancerous lesion or cervical cancer, the United States Preventive Services Task Force (USPSTF) recommends against screening for cervical cancer. Over-screening a patient can cause undue procedures, anxiety, and increased cost but no benefit to greater disease detection. Go to the next page if you knew the correct answer, or click the link image(s) below to further research the concepts in this question (if desired).
Research Concepts: Cervical Screening
We update eBooks quarterly and Apps daily based on user feedback. Please tap flag to report any questions that need improvement.
Question 295:
A 69-year-old man has increasing fatigue, fever, and fine petechial spots on his gums. His primary care provider notices that his labs show a white count of 35,000/mcL with 89% lymphocytes, decreased hemoglobin, and platelets. Physical exam reveals painful hepatosplenomegaly, oral mucosal petechiae, and a diffuse eczematous rash. He has been advised a peripheral smear that shows small lymphocytes with condensed chromatin and fragile destructed lymphocytes. Flow cytometry reveals CD 5, 19 and 23+ cells. What is the first-line treatment for symptomatic treatment in this patient?
Choices: 1. 2. 3. 4.
Alemtuzumab Fludarabine Cytarabine ATRA (all-trans retinoic acid)
Answer: 2 - Fludarabine Explanations: Fludarabine is a purine analog that shows a 70% response in chronic lymphocytic leukemia (CLL). With a reversal of peripheral blood findings, one can expect the reversal of the leukemic skin changes as well with fludarabine. The best response rate and duration of treatment have been found with fludarabine+cyclophosphamide/ fludarabine+cyclophosphamide and rituximab (anti CD20). Fludarabine comes in both oral and IV preparations. Alemtuzumab is used in CD 52+ CLL. Cladribine and not cytarabine has been used in CLL with a 50% response. ATRA is used in APL or acute promyelocytic leukemia. Go to the next page if you knew the correct answer, or click the link image(s) below to further research the concepts in this question (if desired).
Research Concepts: Leukemia Cutis
We update eBooks quarterly and Apps daily based on user feedback. Please tap flag to report any questions that need improvement.
Question 296:
A 12-year-old is brought in with a twoday history of fever, cough, headache, and nausea. He denies any chest pain. His medical history is positive for sickle cell disease. His mother reveals that his sister also had a fever last week, followed by a red rash on her cheeks. The patient's temperature is 38.4 C (101 F), heart rate 130 bpm, respiratory rate 30/minute, pulse oximetry 96%, and blood pressure 102/70 mmHg. On general physical examination, he is an alert but tired-appearing child with pale conjunctivae. The cardiac exam demonstrates tachycardia and a 2/6 systolic ejection murmur. Lungs are clear to auscultation. The abdomen is soft, and there is no hepatosplenomegaly. What complication has this child most likely developed?
Choices: 1. 2. 3. 4.
Hyperhemolytic crisis Myocarditis Aplastic crisis Acute chest syndrome
Answer: 3 - Aplastic crisis Explanations: Aplastic crisis can occur as a complication in patients with sickle cell disease that are infected with parvovirus B19. The virus suppresses bone marrow erythropoiesis, leading to anemia and reticulocytopenia. Patients having an aplastic crisis are treated with supportive care and may require red blood cell transfusions. Chronic parvovirus B19 infection can cause chronic anemia, particularly in patients with immunosuppression. These patients may require treatment with intravenous immunoglobulin (IVIG). Acute chest syndrome presents with fever, shortness of breath, and pain in the chest, ribs, and/or extremities. Hyperhemolytic crisis is an uncommon complication of sickle cell disease that may occur after blood transfusions or in conjunction with acute vaso-occlusive episodes and splenic sequestration. The above conditions are not typically associated with parvovirus B19 infection. Parvovirus B19 has been associated with myocarditis. Myocarditis does not cause conjunctival pallor, making the diagnosis less likely in this patient. Go to the next page if you knew the correct answer, or click the link image(s) below to further research the concepts in this question (if desired).
Research Concepts: Sickle Cell Crisis
We update eBooks quarterly and Apps daily based on user feedback. Please tap flag to report any questions that need improvement.
Question 297:
A 67-year-old male presents to the clinic with abdominal pain and weight loss. A CT abdomen shows an intraductal papillary mucinous neoplasm (IPMN). MRCP shows a cyst with dilation of the entire pancreatic duct and with a solid component. There is no involvement of any adjacent structures. EUS with FNA reveals invasive carcinoma. Which of the following is the next best step in the management of this patient?
Choices: 1. 2. 3. 4.
Total pancreatectomy Whipple procedure (pancreaticoduodenectomy) Distal pancreatectomy Monitoring
Answer: 1 - Total pancreatectomy Explanations: If the entire main pancreatic duct is affected, then total pancreatectomy is the treatment of choice. If the head of the pancreas is affected, then Whipple procedure (pancreaticoduodenectomy) is the operation of choice. If the tail of the pancreas is affected, then distal pancreatectomy plus or minus splenectomy may be warranted. If the tail of the pancreas is affected, then distal pancreatectomy plus or minus splenectomy may be warranted. Go to the next page if you knew the correct answer, or click the link image(s) below to further research the concepts in this question (if desired).
Research Concepts: Intraductal Papillary Mucinous Cancer Of The Pancreas
We update eBooks quarterly and Apps daily based on user feedback. Please tap flag to report any questions that need improvement.
Question 298:
A 60-year-old female patient with a past medical history of hypertension was diagnosed with hormone-negative locally advanced breast cancer. She is a candidate for neoadjuvant chemotherapy and received four cycles of treatment. Two years after her last chemotherapy session, she presents to her primary care provider with shortness of breath and swelling of both her lower limbs. Echocardiography shows a 20 percentage point reduction in her left ventricular ejection fraction. Which of the following agents could have caused her findings?
Choices: 1. 2. 3. 4.
Antimetabolites Anthracyclines Alkylating agents Plant alkaloids
Answer: 2 - Anthracyclines Explanations: Anthracyclines are drugs extracted from Streptomyces sp. used to treat various types of cancers. The different types available for treatment are daunorubicin, doxorubicin, epirubicin, idarubicin, mitoxantrone and, valrubicin. The use of anthracyclines for hormone negative locally advanced breast cancer is an FDAapproved indication. One explanation for their mechanism of action is the production of reactive oxygen species (ROS). Cardiotoxicity induced by anthracyclines is a major cause of morbidity and mortality. It is clinically defined as new-onset heart failure and/or detection of left ventricular dysfunction in exposed individuals. Left ventricular ejection fraction is the most commonly used measure. Patients at high risk are those receiving any of the following treatments: High dose anthracyclines (e.g., doxorubicin > 250 mg/m^2), high dose radiation therapy (>30 Gy) where the heart is in the radiation field and, lower dose anthracyclines in combination with lower-dose radiation therapy. Other risk factors are age greater than 60 years at cancer diagnosis, compromised cardiac function, multiple cardiovascular risk factors, including smoking, hypertension, hyperlipidemia, and obesity. Dexrazoxane is a free radical scavenger that may protect against cardiotoxicity. Go to the next page if you knew the correct answer, or click the link image(s) below to further research the concepts in this question (if desired).
Research Concepts: Anthracyclines
We update eBooks quarterly and Apps daily based on user feedback. Please tap flag to report any questions that need improvement.
Question 299: A 56-year-old male presents with a chief complaint of hematuria. Past medical history is notable for an undisclosed prior malignancy. A coronal CT image demonstrating a finding (indicated by an arrowhead) is shown. Which of the following chemotherapeutic agents predisposes to the finding shown in the image?
Choices: 1. 2. 3. 4.
Cycloaramide Cyclophosphamide Chlorambucil Cisplatin
Photo:Contributed by Dawood Tafti, MD
Answer: 2 - Cyclophosphamide Explanations: The coronal CT image demonstrates a mass along the lateral wall of the bladder suggestive of a transitional cell carcinoma of the bladder. Small calcifications are also appreciated within this mass. Risk factors for developing transitional cell carcinoma include treatment with cyclophosphamide which predisposes to bladder malignancy in a dose-response pattern. The predominant mechanism behind the development of bladder neoplasm in patients treated with cyclophosphamide is thought to be secondary to a bladder wall inflammation-mediated response. Of note, cyclophosphamide therapy is also known to predispose to hemorrhagic cystitis of the bladder. Additional risk factors include exposure to aromatic amines (in smokers), arylamines, and polycyclic aromatic hydrocarbons. Go to the next page if you knew the correct answer, or click the link image(s) below to further research the concepts in this question (if desired).
Research Concepts: Bladder Cancer
We update eBooks quarterly and Apps daily based on user feedback. Please tap flag to report any questions that need improvement.
Question 300:
A 68-year-old male is noted to have multiple skin lesions over his entire body that are pruritic and plaque-like. Topical steroids are not effective. Biopsy shows atypical mononuclear cells infiltrating the dermis and occupying epidermal spaces. Periodic acid-Schiff (PAS) stain shows positive material in their cytoplasm. Similar cells are seen on the peripheral smear with many of them having a prominent nuclear cleft. What is the cell of origin of the malignant cells?
Choices: 1. 2. 3. 4.
CD3+ T-cells CD4+ T-cells CD7+ natural killer cells CD25+ B-cells
Photo:Contributed by DermNetNZ
Answer: 2 - CD4+ T-cells Explanations: The patient has mycosis fungoides, a type of cutaneous T-cell lymphoma. It is mostly evident in the skin but >70% of patients have spread to the spleen, lymph nodes, lung, and liver. This condition affects males twice as much as females and African Americans 1.6 times as much as whites. The malignant clone show CD4+ antigen of T-helper cells but often lack normal T-cell antigens of CD2, 5, or 7. Go to the next page if you knew the correct answer, or click the link image(s) below to further research the concepts in this question (if desired).
Research Concepts: Mycosis Fungoides
We update eBooks quarterly and Apps daily based on user feedback. Please tap flag to report any questions that need improvement.
Section 4 Question 301:
A 65-year-old man with a past medical history of diabetes mellitus has a lesion on the right forearm that has enlarged over the past six weeks. The clinical examination reveals a fleshy, dome-shaped nodule with an irregular crateriform shape. The biopsy of the lesion has been performed. A photograph has been shown. What immunohistochemistry staining represents the atypical cell type?
Choices: 1. 2. 3. 4.
BerEP4 Cytokeratin S100 CD207
Photo:Contributed by DermNetNZ
Answer: 2 - Cytokeratin Explanations: The image and clinical presentation are that of a keratoacanthoma, which is characterized by initial rapid growth followed by a period of variable tumor stability and spontaneous regression. Although keratoacanthoma may involute, many believe that they are a subtype of squamous cell carcinoma, which is a malignant transformation of keratinocytes. Cytokeratin is a stain used for keratinocytes or squamous cells. BerEP4 is a marker for basal cell carcinoma, S100 is a marker for neural crest cell origin cells, CD207 is a marker for Langerhans cells; all are not the markers for keratoacanthomas. Go to the next page if you knew the correct answer, or click the link image(s) below to further research the concepts in this question (if desired).
Research Concepts: Keratoacanthoma
We update eBooks quarterly and Apps daily based on user feedback. Please tap flag to report any questions that need improvement.
Question 302:
Topical 5 fluorouracil (5-FU) is only recommended for treatment of which type of basal cell cancer?
Choices: 1. 2. 3. 4.
Nodular Pigmented Superficial Morpheaform
Photo:Contributed by DermNetNZ
Answer: 3 - Superficial Explanations: 5-FU is effective when applied twice daily for 2 to 12 weeks for superficial basal cell cancer (BCC). 5-FU is not recommended for other types as it may not penetrate deep enough. The recurrence rate after 5-FU remains high. Imiquimod is another topical therapy for superficial basal cell carcinoma Go to the next page if you knew the correct answer, or click the link image(s) below to further research the concepts in this question (if desired).
Research Concepts: Basal Cell Cancer
We update eBooks quarterly and Apps daily based on user feedback. Please tap flag to report any questions that need improvement.
Question 303:
A 23-year-old male presented with a swelling of the right anterior tibial region of the leg over the past 6 months. MRI demonstrated a T1 low signal cortically based lesion with heterogeneous enhancement seen. Histologically, the cells were oval to spindle with peripheral palisading. Furthermore, there were focal areas of squamous differentiation. Atypia or mitotic activity was not visualized. Epithelial cells showed cytokeratin positivity. Stromal cells showed positivity for vimentin and smooth muscle actin. What is the most common presenting symptom of this lesion?
Choices: 1. 2. 3. 4.
Painful or painless swelling Ulceration of the overlying skin Tracheal metastasis with hemoptysis Pathological bone fracture
Photo:Contributed by Dr.Douglas Byerly.
Answer: 1 - Painful or painless swelling Explanations: The main complaint of adamantinoma is swelling with or without pain. Clinically, adamantinoma often displays a protracted clinical behavior. There is a gradual enlargement in size. Bone deformity and pathological fracture are other features that lead the patient to seek medical attention. Adamantinoma is a locally aggressive neoplasm with the potential to metastasize. Adamantinoma metastasizes in about 15-30% of cases by a hematogenous or lymphatic route. Go to the next page if you knew the correct answer, or click the link image(s) below to further research the concepts in this question (if desired).
Research Concepts: Adamantinoma
We update eBooks quarterly and Apps daily based on user feedback. Please tap flag to report any questions that need improvement.
Question 304:
A 45-year-old female presents with fatigue. She was recently diagnosed with ER-positive, HER2negative breast cancer, for which she is taking tamoxifen. She states that she has felt increasingly tired and weak for the past two weeks. On examination, there are symmetrical, violaceous papules over the dorsal aspect of her proximal and distal interphalangeal joints, as shown in the picture. She noticed some discoloration around the eyes as well a few months back but did not seek medical attention at that time. Which auto-antibodies are most likely to be positive in this patient?
Choices: 1. 2. 3. 4.
Anti-TIF1 and anti-NXP2 Anti-SAE and anti-MDA 5 Anti-Mi2 and anti-Jo1 Anti-PL7 and anti-PL12
Photo:Contributed by DermNetNZ
Answer: 1 - Anti-TIF1 and anti-NXP2 Explanations: Patients with dermatomyositis who are positive for antiTIF1 and anti-NXP2 are at increased risk of malignancy within three years of their diagnosis. Anti-SAE antibody is associated with mild to moderate muscle involvement and classic skin rash in dermatomyositis. Anti-MDA-5 is associated with rapidly progressive interstitial lung disease in patients with dermatomyositis. Anti-Mi2 is associated with marked skin involvement in dermatomyositis and anti-Jo1 is associated with antisynthetase syndrome. Anti-PL7 and Anti-PL12 are autoantibodies seen in antisynthetase syndrome and both are associated with severe lung involvement. Go to the next page if you knew the correct answer, or click the link image(s) below to further research the concepts in this question (if desired).
Research Concepts: Autoimmune Myopathies
We update eBooks quarterly and Apps daily based on user feedback. Please tap flag to report any questions that need improvement.
Question 305:
A 50-year-old female presents with pain in her left shoulder for 4 months. There is no history of trauma or fever, but she has lost 8 kg weight in the last three months. A diffuse mild tender swelling is present around her left shoulder. The distal neurovascular status is intact. A recent radiograph is shown. ESR and CRP levels are within the normal range. The biopsy reveals low-grade biphasic blue cells. Which of the following is the most appropriate management for this patent?
Choices: 1. 2. 3. 4.
Wide margin excision & radiotherapy Wide margin excision & chemotherapy Wide excision & radio-chemotherapy Radical excision & chemotherapy
Photo:Contributed by Dr. Muhammad Taqi
Answer: 2 - Wide margin excision & chemotherapy Explanations: Primary central chondrosarcoma is the third most common primary malignancy of bone after myeloma and osteosarcoma. Chondrosarcoma is a tumor of adulthood and older age. The majority of patients are older than 50 years of age at diagnosis, with a slight male predominance. The most common locations of involvement of chondrosarcoma include the bones of the axial skeleton (pelvis, scapula, sternum, and ribs) followed by the proximal femur and proximal humerus. The radiograph reveals osteolytic lesion and cortical ballooning of proximal humerus. There is a cortical thinning and cortical breach. Intralesional calcifications and periosteal reaction are visible. Pathological fracture is present at the upper 1/3rd of the humerus. This patient has mesenchymal chondrosarcoma that contains low-grade biphasic blue cells. The other two types are de-differentiated chondrosarcoma (high-grade spindle cells) and clear cell chondrosarcoma (large vacuolated cells). Location and histologic grade determine the treatment approaches of chondrosarcoma. The primary treatment modality of chondrosarcoma is surgical excision. Chemotherapy and radiotherapy are generally not efficient in conventional chondrosarcoma. However, chemotherapy may have a role in mesenchymal and dedifferentiated chondrosarcomas. The histological grade is the single most important predictor of local recurrence and metastasis. Low-grade chondrosarcomas, which grow insidiously and rarely metastasize, have a good prognosis. The 5-year survival
of low-grade chondrosarcomas is 83%. High-grade chondrosarcoma and dedifferentiated chondrosarcoma, by comparison, have an inferior prognosis due to the rapid growth of the tumor and the propensity for early metastasis Go to the next page if you knew the correct answer, or click the link image(s) below to further research the concepts in this question (if desired).
Research Concepts: Chondrosarcoma
We update eBooks quarterly and Apps daily based on user feedback. Please tap flag to report any questions that need improvement.
Question 306:
A 65-year-old man presents to the clinic. He has a large pearly plaque on the face around the orbit. Due to the extensiveness, he was started on vismodegib. While on therapy, the patient developed the lesion shown in the image attached. He describes the lesion as rapidly growing over the past 6 weeks. What is the other medication associated with the development of this lesion?
Choices: 1. 2. 3. 4.
Trametinib Vemurafenib Ipilimumab Sorafenib
Photo:Contributed by DermNetNZ
Answer: 4 - Sorafenib Explanations: Vismodegib is a selective hedgehog pathway inhibitor that binds to and competitively inhibits smoothened (SMO) protein. Inhibition of SMO results in transcription factors GLI1 and GLI2 remaining inactive. This prevents the expression of tumor mediating genes within the hedgehog pathway. Vismodegib is associated with a number of adverse effects, most commonly dysgeusia, alopecia, arthralgias, weight loss, and fatigue. It also can cause electrolyte disturbances, including hyponatremia and hypokalemia. The use of hedgehog pathway inhibitors for basal cell carcinoma (BCC) and the use of BRAF inhibitors in patients with melanoma have been implicated as risk factors for developing keratoacanthomas, which is described by the clinical features and image attached. Sorafenib is also known to cause this lesion. Vemurafenib is a BRAF inhibitor used in the treatment of metastatic melanoma. Ipilimumab is a monoclonal antibody targeted against CTLA-4, nivolumab is a programmed cell death protein 1 (PD-1) inhibitor, and trametinib is a MEK inhibitor. All of these medications are used in the treatment of melanoma, but none of these are associated with the formation of keratoacanthomas. Go to the next page if you knew the correct answer, or click the link image(s) below to further research the concepts in this question (if desired).
Research Concepts: Vismodegib
We update eBooks quarterly and Apps daily based on user feedback. Please tap flag to report any questions that need improvement.
Question 307:
A 65-year-old farmer presents to the clinic with a lesion on his face. The patient states he is on chemotherapy for skin cancer. The clinical examination reveals a 1 cm red, slightly raised plaque on his face. The histopathology report has shown cytological atypia and dyskeratosis, limited to the basal layers of the stratum spongiosum, along with hyperkeratosis and parakeratosis. A clinical image of a similar lesion is shown. What agent is most likely the cause of this condition?
Choices: 1. 2. 3. 4.
Ipilimumab Vemurafenib Nivolumab Trametinib
Photo:Contributed by DermNetNZ
Answer: 2 - Vemurafenib Explanations: The clinical and pathological descriptions are suggestive of a keratoacanthoma. Ultraviolet light, trauma, human papillomavirus (HPV), genetic factors, immune status, use of hedgehog pathway inhibitors for basal cell carcinoma (BCC), and use of BRAF inhibitors in patients with melanoma have been implicated as risk factors. The choices in this question are all therapy agents used for the treatment of melanoma. However, the only vemurafenib is the listed BRAF inhibitor. Ipilimumab is a monoclonal antibody against CTLA-4, nivolumab is a programmed cell death protein 1 (PD-1) inhibitor, and trametinib is a MEK inhibitor. All of these medications are used in the treatment of melanoma, but none of them are associated with the formation of keratoacanthomas. Go to the next page if you knew the correct answer, or click the link image(s) below to further research the concepts in this question (if desired).
Research Concepts: Keratoacanthoma
We update eBooks quarterly and Apps daily based on user feedback. Please tap flag to report any questions that
need improvement.
Question 308:
A 43-year-old female presents for a diagnostic mammogram after being called back from screening for indeterminate calcifications. Her medical history is unremarkable for medical comorbidities or surgical admissions. She admits that she has lost her younger sister at the age of 24 due to metastatic colon cancer. The following images are obtained. What is the diagnosis?
Choices: 1. 2. 3. 4.
Milk of calcium Amorphic deep calcifications Dystrophic calcifications Skin calcifications
Photo:Contributed by Brooke Bell, MD
Answer: 4 - Skin calcifications Explanations: Tangential views are not performed for milk-of-calcium calcifications. Milk of calcium is diagnosed by a lateral view demonstrating layering or "tea-cupping" of calcium sediment. Tangential views are not performed for vascular calcifications. In contrast to the skin calcifications displayed, vascular calcifications have a tram-track appearance. Tangential views are not performed for dystrophic calcifications. Tangential views show the calcifications localizing within the skin. Go to the next page if you knew the correct answer, or click the link image(s) below to further research the concepts in this question (if desired).
Research Concepts: Benign Breast Calcifications
We update eBooks quarterly and Apps daily based on user feedback. Please tap flag to report any questions that need improvement.
Question 309:
A 55-year-old man with a past medical history of HIV and Hashimoto thyroiditis presents to the clinic for evaluation. He complains of painless, enlarged lymph nodes, weight loss, fever, and night sweats for the past 3 months. Biopsy of an enlarged lymph node reveals a "starry sky" pattern. Which of the following sets of immunophenotypes is most likely to be seen on flow cytometry in this patient?
Choices: 1. 2. 3. 4.
CD20, CD5 Bcl-2, CD43 CD20, CD22, CD23 CD20, CD22, CD11c, CD25, CD103
Photo:Maecker, H. T., McCoy, J. P., & Nussenblatt, R. (2012). Standardizing immunophenotyping for the Human Immunology Project. Nature reviews. Immunology, 12(3), 191–200. https://doi.org/10.1038/nri3158
Answer: 2 - Bcl-2, CD43 Explanations: Patient history and physical is suggestive of an aggressive B-cell lymphoma, most likely Burkitt lymphoma or diffuse large B-cell lymphoma. Both these malignancies are Bcl-2 and CD43 positive. The patient's history of HIV with B symptoms and lymphadenopathy are suggestive of an aggressive B-cell lymphoma. CD20 is seen in mature leukemias and aggressive B-cell lymphomas. Go to the next page if you knew the correct answer, or click the link image(s) below to further research the concepts in this question (if desired).
Research Concepts: Immunophenotyping
We update eBooks quarterly and Apps daily based on user feedback. Please tap flag to report any questions that need improvement.
Question 310:
A 65-year-old male with a past medical history of hypertension and hyperlipidemia presents with fatigue and abdominal fullness. He states that he had a recent dental procedure with persistent gingival bleeding, but he did not think much of it at the time. On physical examination, he is found to have splenomegaly. Further examination reveals a temperature of 98.8 F (37.1 C), a blood pressure of 100/62 mmHg, a heart rate of 92 beats per minute, and a respiratory rate of 18 breaths per minute. Laboratory workup shows an elevated acid phosphatase that, when oxidized, turned into a purple color. Which of the following cells would characteristically be seen in his blood smear?
Choices: 1. 2. 3. 4.
Hairy cells Smudge cells Myeloblasts with Auer rods Reed-Sternberg cells
Photo:This picture is a contribution of Paulo Henrique Orlandi Mourao and is licensed to be shared under the Creative Commons Attribution-Share Alike 3.0 Unported license (https://creativecommons.org/licenses/by-sa/3.0/deed.en). The picture is obtained from the following link (https://commons.wikimedia.org/wiki/File:Hairy_cell_leukemia_smear_2009-08-20.JPG)
Answer: 1 - Hairy cells Explanations: Tartrate-resistant acid phosphatase is characteristically elevated in hairy cell leukemia. Acid phosphatases with a metallic center turn purple when oxidized. They are also resistant to tartrate, thus they are called the tartrate-resistant acid phosphatases. Hairy cell leukemia is named so because of the characteristic "hairy" appearance of the abnormal lymphocytes. Hairy cell leukemia is a chronic lymphoproliferative disease in which neoplastic B cells infiltrate the bone marrow, spleen, and blood, leading to splenomegaly, anemia, and recurring infections. Go to the next page if you knew the correct answer, or click the link image(s) below to further research the concepts in this question (if desired).
Research Concepts: Acid Phosphatase
We update eBooks quarterly and Apps daily based on user feedback. Please tap flag to report any questions that need improvement.
Question 311:
A 17-year-old male patient presents to the clinic with visibly blood-stained stool for the past 3 weeks. He says that he has also lately been feeling tired and generally unwell. A review of systems reveals mild weight loss, altered bowel habits, and anorexia. The patient says that his father died when he was only 2 years old due to some “stomach problem”. Vital signs show a blood pressure of 95/65 mmHg, a respiratory rate of 16/min, a temperature of 37 C (98.6 F) and a pulse rate of 90/min. Abdominal examination reveals a nontender, non-distended, soft abdomen with no palpable masses. Cardiovascular and respiratory examinations are unremarkable. A complete blood picture shows a hemoglobin of 10.5 g/dL and an MCV of 70 femtoliters/cell. Fecal occult blood testing is positive and colonoscopy shows 100s of adenomatous polyps. What is the preferred treatment of this disease?
Choices: 1. 2. 3. 4.
Hemicolectomy of the region with polyps Total colectomy Radiation therapy Polypectomy
Photo:Contributed by Steve Bhimji, MS, MD, PhD
Answer: 2 - Total colectomy Explanations: This patient has familial adenomatous polyposis. Familial adenomatous polyposis is an autosomal dominant genetic disorder resulting in multiple adenomas that progress to carcinoma. If untreated, the risk of colorectal cancer is 100%. Patients with a family history should undergo screening sigmoidoscopy at twelve years of age. Gastric and duodenal adenomas are also common in this population so upper endoscopy screening is also necessary. Go to the next page if you knew the correct answer, or click the link image(s) below to further research the concepts in this question (if desired).
Research Concepts: Familial Adenomatous Polyposis
We update eBooks quarterly and Apps daily based on user feedback. Please tap flag to report any questions that need improvement.
Question 312:
A 65-year-old male presents for a rapidgrowing nodule of the cheek, appeared 3 weeks ago. It is a solitary, flesh-colored nodule with a central scaly plug. It is painless but a bit itchy, and he is concerned for neoplastic disease. The patient is currently taking cyclosporine and prednisone for a hepatic transplant 3 years ago for hepatocellular carcinoma, and his previous follow-ups were unremarkable. What is the most appropriate next step in the management of this patient?
Choices: 1. 2. 3. 4.
Refer to the hepatology for an urgent revision Biopsy the lesion to decide a proper treatment Topic treatment with corticosteroid and keratolytic cream Complete excision
Photo:Contributed by DermNetNZ
Answer: 2 - Biopsy the lesion to decide a proper treatment
Explanations: Keratoacanthoma is a neoplastic lesion, typically arising on the face (tropical climates) or extremities and dorsal surface of hands. It can involute spontaneously; for that reason, it is also called self-healing squamous cell carcinoma. Lesions may last for years without involuting and may cause local destruction with aesthetic and functional concerns, particularly nose or eyelids. Active treatment is usually preferred in this case. When left for self-healing, they usually leave a crateriform and irregular scar that may compromise eyelid functionality. Keratoacanthomas are associated with burns and excessive sun exposure, immunosuppression, xeroderma pigmentosum. They tend to develop most often in older (sixth and seventh decades) males. Keratoacanthomas have a typical "volcano-like" appearance with a keratin-filled crater, atypical keratinocytes with pale, eosinophilic cytoplasm. It is a well-differentiated carcinoma, with typical keratin pearls and lichenoid infiltrate. Surgery is the treatment of choice for solitary lesions. Go to the next page if you knew the correct answer, or click the link image(s) below to further research the concepts in this question (if desired).
Research Concepts: Hyperkeratosis
We update eBooks quarterly and Apps daily based on user feedback. Please tap flag to report any questions that need improvement.
Question 313:
A 54-year-old woman presents to the clinic with a one-year history of progressive swallowing difficulties. An MRI is obtained showing a T2 fat saturation hyperintense lesion as pictured. A needle biopsy is performed, which shows bubbly cells separated into lobules with fibrous septa and a myxoid stroma. Which of the following is the best initial therapy for this patient?
Choices: 1. 2. 3. 4.
Neoadjuvant radiation therapy Chemotherapy Conventional photon radiation Highly-conformal radiotherapy
Photo:Contributed by Steven Tenny, MD, MPH, MBA and Christopher Gillis, MD.
Answer: 4 - Highly-conformal radiotherapy Explanations: Chordomas are relatively radioresistant which necessitates high-dose radiation including proton beam radiation or radiosurgery (highly conformal radiotherapy). When a needle biopsy is performed, the surgical resection should include the needle tract as chordomas can seed the needle biopsy tract. En bloc resection followed by high-dose radiation provides the best current long-term outcomes for chordoma treatment. Chordomas are slow-growing and chemotherapy is not a generally used treatment strategy. If chemotherapy is given, it is usually given as part of a clinically trial. Go to the next page if you knew the correct answer, or click the link image(s) below to further research the concepts in this question (if desired).
Research Concepts: Chordoma
We update eBooks quarterly and Apps daily based on user feedback. Please tap flag to report any questions that need improvement.
Question 314:
A 76-year-old female with a past medical history of hypertension and COPD presents to the office for her routine annual physical. She is worried about some new lesions that suddenly appeared on her back. Examination reveals multiple uniform, brown, round skin lesions on her back and shoulders that have a verrucous appearance (see picture). What is the sign associated with the sudden appearance of these lesions?
Choices: 1. 2. 3. 4.
Nikolsky Sign Leser-Trelat sign Hutchinson sign Trousseau sign
Photo:Contributed by DermNetNZ
Answer: 2 - Leser-Trelat sign Explanations: Seborrheic keratosis is a common type of epidermal tumor that is prevalent throughout middle-aged and elderly individuals. Generally, seborrheic keratoses are benign and slowgrowing lesions, however, there should be a concern if there is sudden growth or emergence of multiple seborrheic keratoses. The sign of Leser-Trélat refers to the sudden appearance of seborrheic keratosis that is suggestive of internal malignancy, such as gastrointestinal or pulmonary carcinomas. For patients with seborrheic keratosis, it is extremely important to conduct a thorough history and physical, age-appropriate cancer screening, and order any additional labs based on risk factors and patient’s history. Go to the next page if you knew the correct answer, or click the link image(s) below to further research the concepts in this question (if desired).
Research Concepts: Seborrheic Keratosis
We update eBooks quarterly and Apps daily based on user feedback. Please tap flag to report any questions that
need improvement.
Question 315:
A 17-year-old male is found to have a painless testicular mass that does not transilluminate. After surgical removal, it is determined to be malignant and radiation therapy is initiated. The histopathology of the tissue is shown in the figure. Which of the following are the most likely findings seen in this patient?
Choices: 1. Scattered immature neural elements with abnormal tissue from 3 germ layers 2. Tumor cells with clear cytoplasm that are large and distinct cell membranes in a fibrous stroma with numerous lymphocytes 3. Mixed malignant syncytiotrophoblasts and cytotrophoblasts 4. Tumor cells that have abundant granular, eosinophilic cytoplasm with few intracytoplasmic rhomboid crystals
Photo:Contributed by Muhammad Ali Tariq MD
Answer: 2 - Tumor cells with clear cytoplasm that are large and distinct cell membranes in a fibrous stroma with numerous lymphocytes
Explanations: Testicular tumors can be seminomas or nonseminomatous germ cell tumors (NSGCT). Seminomas comprise about one-third of all germ cell tumors. NSGCTs include yolk sac tumors, embryonal carcinomas, teratocarcinoma, immature teratomas, mixed germ cell tumors and choriocarcinomas. Seminomas spread lymphatically. Histologically, cells are large with clear cytoplasm. Seminomas are extremely radiosensitive while NSGCTs are not as sensitive and have a somewhat worse prognosis. Despite this sensitivity, chemotherapy is frequently used for seminomas as it tends to have fewer long-term side effects. Syncytiotrophoblasts, which make beta human chorionic gonadotropin (HCG), and cytotrophoblasts, which make alpha-fetoprotein (AFP), are found together in choriocarcinoma. The presence of any abnormal AFP means that the tumor cannot be a pure seminoma. Go to the next page if you knew the correct answer, or click the link image(s) below to further research the concepts in this question (if desired).
Research Concepts: Testicular Seminoma
We update eBooks quarterly and Apps daily based on user feedback. Please tap flag to report any questions that need improvement.
Question 316:
A 22-year-old woman presents to the outpatient department for the evaluation of a nodular lesion on her leg for the past 6 months that has gradually grown in size and is now approximately 0.9 cm. She mentions that the lesion is hyperpigmented and she experiences no associated pain or itching. She does not use tobacco or alcohol. She has no medical problems and does not take any medications. She is sexually active with her boyfriend and uses condoms regularly. Vitals show a blood pressure of 125/75 mm Hg, a pulse of 77/min, a respiratory rate of 12/min, and a temperature of 37 C (98.6 F). Physical examination reveals a hyperpigmented nodular lesion on the patient’s left leg. It is non-tender on palpation and central dimpling is observed when the lesion is pinched at the edges. Which of the following best describes the course of the patient’s skin condition if no medical or surgical treatment is provided?
Choices: 1. 2. 3. 4.
Rapid infiltration into the underlying tissues Rapid metastasis Spontaneous resolution or no change Superficial infection
Photo:Contributed by DermNetNZ
Answer: 3 - Spontaneous resolution or no change Explanations: The most likely diagnosis is a dermatofibroma, a superficial benign fibrous histiocytoma. Dermatofibromas are slow-growing lesions that can affect any part of the body but have a predilection for the extremities. Clinically these lesions are firm, nontender, nodular with or without overlying skin changes, and a smooth surface. Dermatofibromas are benign lesions with excellent prognosis. Spontaneous resolution or no change in the lesions will likely be observed even if no medical or surgical treatment is provided. Rapid metastasis may be seen with a melanoma. No central dimpling is observed with a melanoma. Go to the next page if you knew the correct answer, or click the link image(s) below to further research the concepts in this question (if desired).
Research Concepts: Dermatofibroma
We update eBooks quarterly and Apps daily based on user feedback. Please tap flag to report any questions that need improvement.
Question 317:
A 78-year-old white man presents with a 0.5 cm erythematous plaque on his arm. Skin biopsy results reveal a nodular basal cell carcinoma. Which of the following is the best treatment option?
Choices: 1. 2. 3. 4.
Curettage Radiation therapy Clinical observation Surgical excision
Photo:Contributed by DermNetNZ
Answer: 4 - Surgical excision Explanations: Not all skin cancers require Mohs micrographic surgery. A small nodular primary basal cell carcinoma on the arm is treated adequately with surgical excision. In almost all cases of basal cell cancer, surgery is the primary modality of treatment. However, the surgical approach depends on the depth, diameter, and location of the tumor. Modalities of treatment include excisional surgery and Mohs surgery. Curettage, electrodesiccation, and cryosurgery are less preferable options. Go to the next page if you knew the correct answer, or click the link image(s) below to further research the concepts in this question (if desired).
Research Concepts: Basal Cell Cancer
We update eBooks quarterly and Apps daily based on user feedback. Please tap flag to report any questions that need improvement.
Question 318:
A 60-year-old man emigrated from china presents with the third episode of bleeding from the nose in the last one month. He has also noticed a neck mass recently. He is a longtime smoker of cigarettes and denies fevers, night sweats, fatigue, or cough. He had multiple episodes of otitis media in the past. On physical examination, there is a 1.5 cm hard, fixed mass arising from the nasal mucosa. Neck examination reveals enlarged cervical lymph nodes. Biopsy of the mass reveals keratin pearls along with atypical mitosis. Which of the following is the most likely cause of this condition?
Choices: 1. 2. 3. 4.
Human herpesvirus Staphylococcus aureus Human papillomavirus Ebstein-Barr virus
Answer: 4 - Ebstein-Barr virus Explanations: The classic triad of symptoms of nasopharyngeal cancer includes a neck mass due to lymph node metastasis, otitis media, and nasal obstruction is rarely seen. Nasopharyngeal cancers (NPC) are associated with the Ebstein-Barr virus (EBV). The most common presenting symptom in nasopharyngeal cancer is repeated episodes of epistaxis in a patient from China. Other regions affected include central, south, and southeast Asia, the Arctic, Middle East, and North Africa. Oropharyngeal cancers are associated with human papillomavirus (HPV) infection. Go to the next page if you knew the correct answer, or click the link image(s) below to further research the concepts in this question (if desired).
Research Concepts: Nasopharyngeal Cancer
We update eBooks quarterly and Apps daily based on user feedback. Please tap flag to report any questions that need improvement.
Question 319:
A 51-years old male presents with complaints of abdominal discomfort and abdominal distention for the last three months. He has lost 4 kg of weight during the same period. Abdominal examination shows a palpable mass of about 4 cm x 3 cm occupying the epigastric region. Computed tomography of the abdomen shows a flat mass with multiple cystic spaces and enhancement of solid areas, well-defined margins, and highly vascularized. It is located in the anterior compartment of the abdomen anterior to the small bowel and transverse colon. Immunohistochemical staining these tumors is negative for CD117, CD99, and positive for CD34. What is the most likely treatment of choice?
Choices: 1. 2. 3. 4.
Surgical resection with post-op follow up for recurrence Resection and chemotherapy Resection and radiotherapy Chemotherapy
Answer: 1 - Surgical resection with post-op follow up for recurrence
Explanations: Primary omental tumors are mostly asymptomatic, but most patients present with abdominal discomfort, palpable abdominal mass, nausea, early satiety, and weight loss. A solitary fibrous tumor (SFT) of omentum is derived from mesenchymal cells. It is mostly composed of spindle-shaped cells and collagen bundles. SFTs are generally benign. On immunohistochemical staining, these tumors can be negative for CD117, CD99, CD68, cytokeratin, calretinin, desmin, epithelial membrane antigen, F8, and S-100, but positive for CD34, BCL-2, a-smooth muscle actin (aSMA) and vimentin (VIM).[12] CD34 is a useful positive biomarker for SFT; few studies have shown a 100% positivity rate. The treatment of choice is surgical resection, but malignant tumors have the potential for recurrence even several years after the surgery. The beneficial role of adjuvant therapy has not been established yet, but some reports suggest adjuvant radiotherapy and show response to chemotherapy, but their effectiveness has not been proved. Go to the next page if you knew the correct answer, or click the link image(s) below to further research the concepts in this question (if desired).
Research Concepts:
Omentum Tumors
We update eBooks quarterly and Apps daily based on user feedback. Please tap flag to report any questions that need improvement.
Question 320:
A 65-year-old male presents to the clinic with complaints of cough, fever, and chest pain, along with bone pains involving the spine and ribs. In addition, he also mentions having abdominal pain, confusion, depression, and generalized weakness. A blood test is done, which shows a hemoglobin of 9 g/dL, mean corpuscular volume (MCV) 85 fL, mean corpuscular hemoglobin (MCH) 29 pg. His renal function tests reveal a creatinine of 2.5 mg/dl. Which of the following findings are most likely to be present in this patient?
Choices: 1. Chest x-ray showing hilar lymphadenopathy 2. Bence Jones proteins 3. Bone marrow biopsy showing lymphocytes 4. Increased white blood cells count and lymphocytes in the blood
Answer: 2 - Bence Jones proteins Explanations: Multiple myeloma is a type of plasma cell cancer characterized by hypercalcemia, renal problems, anemia, and bone pains. Bence Jones proteins are lambda light chains in the urine. These light chain proteins are found in multiple myeloma, which is the diagnosis in this patient, and are used to monitor therapy. Today, beta-2 microglobulin is a strong predictor of multiple myeloma outcomes. Plain x-rays remain the gold standard for imaging newly diagnosed multiple myeloma. Go to the next page if you knew the correct answer, or click the link image(s) below to further research the concepts in this question (if desired).
Research Concepts: Multiple Myeloma
We update eBooks quarterly and Apps daily based on user feedback. Please tap flag to report any questions that need improvement.
Question 321:
A 20-year-old man was recently diagnosed with a subtype of hepatocellular carcinoma (HCC) and local regional metastatic adenopathy. His specialist recommends resection and lymph node dissection as the treatment. What is the patient's most likely 5-year survival rate if he declines any treatment?
Choices: 1. 2. 3. 4.
50% 75% 100% 0%
Answer: 4 - 0% Explanations: Resectability improves survival. Surgical resection is the primary line of care of patients with a newly diagnosed fibrolamellar HCC. Overall 5-year survival in fibrolamellar HCC is approximately 21-45%, which improves with resectability to 63-76%. If the patient declines treatment five-year survival rate is 0%. If the patient were to undergo resection, his five-year survival rate would be 50%. Go to the next page if you knew the correct answer, or click the link image(s) below to further research the concepts in this question (if desired).
Research Concepts: Fibrolamellar Hepatocellular Carcinoma
We update eBooks quarterly and Apps daily based on user feedback. Please tap flag to report any questions that need improvement.
Question 322:
A 49-year-old woman with a past medical history of hypertension presents to the clinic with several months of a grouping of four painless, violaceous plaques of the neck and chest. Four years ago, she successfully underwent radiotherapy for a total of 30Gy on a similar lesion which was noted to be a cutaneous B-cell lymphoma. Today, she denies any other symptoms, such as fevers, chills, night sweats, weight loss, pruritus, or easy bruising or bleeding. Her vital signs are within normal limits, and she has no abnormal findings on physical examination. Complete blood count, comprehensive metabolic panel, and lactate dehydrogenase levels are within normal limits. A CT of the chest, abdomen, and pelvis with contrast does not reveal any lymphadenopathy. Biopsy of one of the plaques is consistent with a recurrence of cutaneous B-cell lymphoma, primary cutaneous follicle center lymphoma. Which of the following is the next best step in the management of this patient?
Choices: 1. Rituximab monotherapy for four cycles 2. R-CHOP (rituximab, cyclophosphamide, doxorubicin, vincristine, prednisone) chemotherapy for four cycles 3. Mohs surgery 4. Repeat radiotherapy
Answer: 4 - Repeat radiotherapy Explanations: Recurrence in primary cutaneous follicle center lymphoma is managed in the same modalities as an initial presentation with similar treatment efficacy rates. Recurrence does not portend a worsened prognosis. Despite the recurrence, rituximab is usually reserved for multiple lesions, not amenable to radiotherapy or excision. The patient is best managed by evaluation of these options first. R-CHOP chemotherapy is reserved for refractory disease; this patient had a response for several years with local therapy, and this can be used again with curative intent. While a viable initial option, relapse rates with surgical excision are higher than with radiotherapy and is not the best next step. Go to the next page if you knew the correct answer, or click the link image(s) below to further research the concepts in this question (if desired).
Research Concepts: Primary Cutaneous Follicle Center Lymphoma
We update eBooks quarterly and Apps daily based on user feedback. Please tap flag to report any questions that need improvement.
Question 323:
A 1-year-old child is brought to the clinic with a whitish appearance in the eye for one month and an altered red reflex seen in his photographs. Examination of the eye reveals a 1 mm hypopyon with no conjunctival or circumciliary congestion. The cornea is clear with no keratic precipitates. The fundal glow was altered with a yellowish reflex. The infant is very much uncooperative for dilated fundus examination. Hence an examination under anesthesia was planned along with the anterior chamber tap for culture, microscopy, and biochemistry evaluation. Which of the following laboratory result confirms the most likely diagnosis?
Choices: 1. Aqueous ascorbic acid levels are higher than serum levels 2. Aqueous total protein levels are lower than serum levels 3. The ratio of aqueous to plasma lactate dehydrogenase (LDH) is more than 1.0 4. The ratio of aqueous to plasma lactate dehydrogenase (LDH) is less than 1.0
Answer: 3 - The ratio of aqueous to plasma lactate dehydrogenase (LDH) is more than 1.0
Explanations: The ratio of aqueous to plasma lactate dehydrogenase (LDH) more than 1.0 is the correct answer and is confirmatory of retinoblastoma. Leukocoria and pseudohypopyon indicate a high probability of retinoblastoma. Aqueous LDH is a histologic tumor marker for retinoblastoma. Aqueous to LDH ratio levels directly correlate with the severity of the disease and are elevated in retinoblastoma types 4 and 5. Aqueous to LDH ratio less than 1.0 is normal in occurrence. Go to the next page if you knew the correct answer, or click the link image(s) below to further research the concepts in this question (if desired).
Research Concepts: Leukocoria
We update eBooks quarterly and Apps daily based on user feedback. Please tap flag to report any questions that need improvement.
Question 324:
A 78-year-old male miner presents to the clinic with a chronic cough and worsening breathlessness for the past 2 months. A chest radiograph is suggestive of a right upper lobe mass. A CT-guided biopsy yields a diagnosis of adenocarcinoma lung. Immunohistochemistry reveals an EGFR L858R mutation. He is given osimertinib. After six months of therapy, he presents to the clinic with lower backache without any signs of bladder involvement. An x-ray of the thoracolumbar spine reveals a compression fracture at the level of L2. His corrected serum calcium level is 8.5 mg/dL. He is scheduled for stereotactic radiotherapy to the spine. Which of the following is the next best step in the management of this patient?
Choices: 1. 2. 3. 4.
3D conformal radiotherapy Intensity-guidance radiotherapy Cone-beam computed tomography Stereotactic body radiotherapy
Answer: 2 - Intensity-guidance radiotherapy Explanations: The incidence of metastatic bone disease ranges from 17%-64% in lung cancer, 65%-75% in breast cancer, 65%-90% in prostate cancer, and 70 to 95% in multiple myeloma. Locoregional approaches for symptomatic palliation of bone metastases include surgery and radiotherapy. Systemic approaches include the usage of inhibitors of bone resorption, radiopharmaceutical targeted agents, endocrine approaches, and chemotherapy. Stereotactic body radiotherapy has emerged as an important treatment modality for the delivery of conformal radiotherapy with a higher biologically effective dose to the spinal vertebrae and surrounding paraspinal tissue. SBRT uses many more multidirectional beams than conventional opposed field radiotherapy. SBRT also allows for a visual dissection of the tumor from the adjacent normal tissues. Using new beam shaping and image guidance technologies have ensured closer proximity to the tumor while allowing greater conformal allegiance to the threedimensional tumor outline. The use of an interactive image-guided approach has been made possible by the use of a multitude of technological advances such as online con- beam computed tomography (CBCT), dynamic multi-leaf collimators (DMLC), and rapid intensity-modulated beam delivery (IMRT). The rapid IMRT approach confines the radiation beam to the target tumor while promoting a rapid dose fall-off to surrounding normal tissues.
Go to the next page if you knew the correct answer, or click the link image(s) below to further research the concepts in this question (if desired).
Research Concepts: Palliative Radiation Therapy For Bone Metastases We update eBooks quarterly and Apps daily based on user feedback. Please tap flag to report any questions that need improvement.
Question 325:
A 65-year-old African American male presented to his primary care provider office with new complaints of difficulty in moving the fourth and fifth fingers of his left hand along with numbness. He has been complaining of left shoulder pain for two weeks now and was treated for a frozen shoulder. Clinical examination is remarkable with ptosis of the left eye and pupillary constriction. There is also increased flushing and sweating of just the left side of his face. A computed tomography (CT) head scan and shoulder X-ray are ordered, and the shoulder X-ray raises concern for an incidental mass lesion in the left upper lobe of the lung. In subsequent workup with CT chest scan and positron emission tomography (PET-CT), a left apical mass is found with PET-positive ipsilateral hilar lymphadenopathy. There is no other evidence of distant metastasis. He is referred for surgical resection but is deemed not to be a surgical candidate. What is his contraindication for surgical resection?
Choices: 1. Acute presentation 2. His age 3. PET-positive left hilar lymph node 4. Inability to flex the fourth and fifth fingers along with paresthesia
Answer: 4 - Inability to flex the fourth and fifth fingers along with paresthesia
Explanations: The inability to flex the fourth and fifth fingers along with paresthesia are indicative of tumor involvement of high brachial plexus at the C8 level. In Pancoast syndrome, surgical resection can be a component of trimodality therapy in addition to neoadjuvant chemoradiation first. It is performed in N0 or N1 disease as well in patients of appropriate performance status. However, the involvement of the brachial plexus above the T1 nerve root is considered a contraindication to surgical resection due to the high likelihood of median nerve injury. Other contraindications to surgery include the presence of distant metastasis, the involvement of ipsilateral or contralateral mediastinal nodes or supraclavicular nodes, the involvement of vertebral bodies more significant than 50%, or involvement of esophagus and/ trachea. Go to the next page if you knew the correct answer, or click the link image(s) below to further research the concepts in this question (if desired).
Research Concepts: Pancoast Syndrome
We update eBooks quarterly and Apps daily based on user feedback. Please tap flag to report any questions that need improvement.
Question 326:
A 68-year-old man presents with a lump on his right forearm that he first noticed three months ago when he accidentally brushed himself against the kitchen sink. Initially, a bruise appeared in the affected area that waxed and waned until about four weeks ago, when it started growing to its current size (4 x 5 cm). He has a history of leiomyosarcoma in the same region that was irradiated, followed by R0 resection nearly ten years ago. He also has grade 2 diabetic peripheral neuropathy. The forearm mass is biopsied, and histopathology shows malignant endothelial cells that stain positively for Ulex europaeus agglutinin 1 and vascular endothelial growth factor (VEGF) and negative for S-100 and HMB-45. The staging workup shows multiple pleural metastases. The patient's overall performance status is between 0 and 1, he does not have any underlying cardiac issues, and an echocardiogram shows a left ventricular ejection fraction of 65%. What is the most appropriate treatment for this patient?
Choices: 1. 2. 3. 4.
Doxorubicin Paclitaxel Bevacizumab Supportive care only
Answer: 1 - Doxorubicin Explanations: Weekly paclitaxel and doxorubicin have never been compared but are recommended first-line treatments for patients with angiosarcoma. Paclitaxel can worsen this patient's neuropathy; therefore, doxorubicin is a better option for this patient with good performance status. Doxorubicin has a response rate of up to 30%. Bevacizumab has been tested in a phase II trial and has not shown a survival benefit. Hospice care is not appropriate for this patient at this point. Go to the next page if you knew the correct answer, or click the link image(s) below to further research the concepts in this question (if desired).
Research Concepts: Angiosarcoma
We update eBooks quarterly and Apps daily based on user feedback. Please tap flag to report any questions that need improvement.
Question 327:
A 35-year-old woman presents to the clinic with pain and swelling in her right knee. She is having pain during the day that is worse with activity. On examination, there is a small mass protruding from her right knee. An extensive workup reveals a giant cell tumor of bone. Which of the following complications is she at the highest risk for if she receives the most appropriate treatment for her disease?
Choices: 1. 2. 3. 4.
Functional compromise Osteonecrosis Malignant transformation Osteoporosis
Answer: 1 - Functional compromise Explanations: Surgery can have significant morbidities resulting in functional compromise. More extensive surgery is required when there is gross damage to the articular surface, an extraosseous extension, a secondary recurrence or a dislocated pathological fracture. When the comorbidities from surgery becomes too high, radiation therapy and denosumab are acceptable alternatives. Functional compromise tends to be higher with a giant cell tumor of bone located in the spine. As such, treatment is centered upon maximizing the benefits while minimizing complications. Go to the next page if you knew the correct answer, or click the link image(s) below to further research the concepts in this question (if desired).
Research Concepts: Osteoclastoma
We update eBooks quarterly and Apps daily based on user feedback. Please tap flag to report any questions that need improvement.
Question 328:
A 58-year-old woman is evaluated for significant fatigue and difficulty standing up. She had a fall from her bed because of dizziness. Her history is significant for stage 4 triple-negative, programmed death ligand-1 (PDL1) positive breast cancer with metastasis diagnosed five months ago. She is currently on atezolizumab, a checkpoint inhibitor, and chemotherapy. She just completed her fourth cycle two weeks ago and reported mild nausea and vomiting during sessions controlled with acetaminophen and ondansetron. The patient also has a history of asthma, managed with a daily albuterol inhaler. Last week, she and her husband were afflicted with two days of a ‘stomach bug’ with self-resolving diarrhea and mild abdominal pain. She does not have any abdominal pain or respiratory distress now. Her vital signs are within normal range; however, she gets dizzy while standing up from a sitting position. Lab workup today reveals a normal complete blood count with serum sodium 131 mEq/L, potassium 3.2 mEq/L, glucose 75 mEq/L, adrenocorticotropic hormone (ACTH): 8 pg/mL (normal range: 10-60 pg/mL), morning cortisol 2 microgram/dL (normal range: 5-25 microgram/dL). Which of the following is the most likely triggering event?
Choices: 1. 2. 3. 4.
Checkpoint therapy Gastroenteritis Chronic adrenal insufficiency Acute adrenal hemorrhage
Answer: 1 - Checkpoint therapy Explanations: Immunotherapy with checkpoint therapy, such as atezolizumab for breast cancer, is associated with acquired endocrinopathies such as adrenal insufficiency due to causing hypophysitis, the autoimmune destruction of the pituitary. Checkpoint therapy for lung, breast, and melanoma cancers may precipitate an adrenal crisis. Some other complications include thyroiditis and type 1 diabetes. The average onset of endocrine-related complications from checkpoint therapy may occur from 6 to 12 weeks after starting therapy. The patient's symptoms and electrolytes are concerning for acute adrenal insufficiency. Her ACTH and cortisol levels are low, suggesting secondary adrenal insufficiency from hypopituitarism. Go to the next page if you knew the correct answer, or click the link image(s) below to further research the concepts in this question (if desired).
Research Concepts: Addisonian Crisis
We update eBooks quarterly and Apps daily based on user feedback. Please tap flag to report any questions that need improvement.
Question 329:
A 45-year-old female with a history of myelodysplastic syndrome presents with hemorrhagic bullae over the bilateral dorsum of the hands, which were associated with pain, for ten days. These were followed by erythematous tender nodules and plaques studded with pseudovesicles. A clinical examination showed a high-grade fever of 38.8 C along with generalized weakness and pallor. Laboratory results reveal decreased counts in all leukocyte lineages, erythrocyte sedimentation rate was 28 mm/hr, and C-reactive protein was raised. The blood smear shows anisocytosis and microcytic anemia, granulated neutrophils were also seen, which are typically described as PseudoPelger Huet anomaly. What is the most appropriate treatment approach in this case?
Choices: 1. 2. 3. 4.
Incision and drainage Administration of systemic corticosteroids Administration of broad-spectrum antibiotics Change in chemotherapeutic regimen
Answer: 2 - Administration of systemic corticosteroids Explanations: The patient is known to have myelodysplastic syndrome. Certain underlying malignancies like myelodysplastic syndrome can predispose an individual to develop Sweet syndrome. Cutaneous examination findings of these patients are suggestive of neutrophil dermatosis of dorsal hands, which is a variant of Sweet syndrome. It is more frequently associated with myelodysplastic syndrome and acute myelogenous leukemia. Systemic corticosteroids are the first line of management of Sweet syndrome. Incision and drainage should be done in cases of infective conditions like cellulitis. Systemic antibiotics are unjustified unless there is a proven infection. In this case, the skin lesions are not likely to be cutaneous adverse drug events due to chemotherapeutic agents. Go to the next page if you knew the correct answer, or click the link image(s) below to further research the concepts in this question (if desired).
Research Concepts: Acute Febrile Neutrophilic Dermatosis
We update eBooks quarterly and Apps daily based on user feedback. Please tap flag to report any questions that
need improvement.
Question 330:
A 2-year-old boy is brought in with a history of fever, cough, and dyspnea. The patient has been treated for recurrent pneumonia several times in the past few months. His temperature is 100.7 F, heart rate is 110/min, respirations are 21/min. The cardiopulmonary examination is unremarkable except for sinus tachycardia. The X-ray of the chest shows rounded opacities in the right lower lung zone. CT chest is done shows 2 cystic masses arising from the right lung pleura. Besides the treatment, this patient should be evaluated for what type of genetic mutation?
Choices: 1. 2. 3. 4.
BMPR-2 gene mutation DICER-1 gene mutation BRCA gene mutation ALK gene mutation
Answer: 2 - DICER-1 gene mutation Explanations: In about 30-40% of children with pleuropulmonary blastoma (PPB), there are other childhood cancers or abnormalities in their immediate or extended family. DICER-1 gene mutation causes an increased risk for PPB and a variety of other conditions such as lung /kidney cysts or tumors, ovarian tumors in children or adults, thyroid nodules or cancers, brain tumors, and tumors in the eye or nose/sinuses. When a DICER-1 gene mutation is found, specific screening studies may be recommended to look for PPB and other conditions known to be related to DICER-1. DICER-1 specific surveillance guidelines are available for anyone with PPB, another DICER1-related condition, or variation in the DICER-1 gene. Go to the next page if you knew the correct answer, or click the link image(s) below to further research the concepts in this question (if desired).
Research Concepts: Pleuropulmonary Blastoma
We update eBooks quarterly and Apps daily based on user feedback. Please tap flag to report any questions that need improvement.
Question 331:
A 65-year-old female presents to the clinic for a health maintenance exam. The patient has a past medical history significant for hypertension and diabetes mellitus. She takes amlodipine, ramipril, and metformin. The patient states that she experiences nausea and epigastric pain after eating food. Ultrasonography reveals a few calculi in the gall bladder and an incidental finding of a mass in the right kidney. On ultrasound, the mass is hyperechoic and located in the renal cortex with posterior acoustic shadowing. Fine-needle aspiration cytology of the renal mass reveals spindle cells with stringy cytoplasm admixed with mature adipose tissue and rare epithelioid cells. Which of the following immunohistochemical stains is most appropriate to confirm the renal diagnosis in this patient?
Choices: 1. 2. 3. 4.
CD10 Cytokeratin 7 HMB45 Epithelial membrane antigen
Answer: 3 - HMB45 Explanations: This patient has a renal angiomyolipoma. Immunohistochemical staining will be positive for HMB45 and MelanA (MART1) but negative for cytokeratin 7 and epithelial membrane antigen which are typically positive in renal cell cancers. Histopathology is characterized by the presence of benign-appearing spindle cells with delicate to stringy cytoplasm. The presence of mature fibro adipose tissue, when admixed with spindle cells, helps differentiate renal angiomyolipoma from a sarcoma or sarcomatoid carcinoma. Renal angiomyolipoma is the most common benign mesenchymal renal neoplasm. It consists of blood vessels, smooth muscle, and different quantities of fat. The tumor originates from the cortex and spreads into the perirenal fat. Go to the next page if you knew the correct answer, or click the link image(s) below to further research the concepts in this question (if desired).
Research Concepts: Renal Angiomyolipoma
We update eBooks quarterly and Apps daily based on user feedback. Please tap flag to report any questions that
need improvement.
Question 332:
A 23-year-old female patient presents to the hospital with an 8-month history of amenorrhea, discharge from her nipples, and decreased libido. She also complains of changes to her vision and headaches. Her blood pressure is 120/82 mmHg, pulse 88/min, respirations 16/min, and temperature 98.0 F (36.7 C). On physical examination, the patient has decreased body hair and bitemporal hemianopsia. Her serum prolactin level is 130 ng/ml (normal 5 to 20 ng/ml), thyroid-stimulating hormone (TSH) 3.4 IU/mL (0.27-4.20), and beta-hCG undetectable. MRI confirms an 8mm microadenoma. What is the most appropriate initial step in the management of this patient?
Choices: 1. 2. 3. 4.
Transsphenoidal surgery Radiation therapy Bromocriptine Estrogen replacement
Answer: 3 - Bromocriptine Explanations: The hormone most often secreted by tumors of the pituitary gland is prolactin. Prolactinomas are asymptomatic. A prolactinoma is classified based on its size. A microprolactinoma is less than 10 millimeters in diameter. A macroprolactinoma is greater than 10 millimeters in diameter. Bromocriptine usually is the treatment of choice for a prolactinoma. The drug has withstood the test of time and is relatively safe. Go to the next page if you knew the correct answer, or click the link image(s) below to further research the concepts in this question (if desired).
Research Concepts: Prolactinoma
We update eBooks quarterly and Apps daily based on user feedback. Please tap flag to report any questions that need improvement.
Question 333:
A 49-year-old commercial fisherman presents to the clinic for follow up. He recently recovered from severe bullous cutaneous burns after exposure to a liquid material that oozed out of an old WWI artillery shell that had been accidentally dredged from the Atlantic Ocean with his catch. His exposure was limited to the skin. Later in life, which of the following malignancies, if any, is most likely to develop in this patient?
Choices: 1. 2. 3. 4.
Pancreatic adenocarcinoma Melanoma Pulmonary adenocarcinoma There is no specific cancer risk after isolated exposure.
Answer: 4 - There is no specific cancer risk after isolated exposure.
Explanations: Japanese factory workers with long-term exposure to low doses of sulfur mustards were at higher risk of developing malignancies later in life. Significant gastrointestinal exposures may predispose to the development of Barrett esophagus and some other malignancies. Isolated cutaneous exposure should not necessarily put the patient at risk for developing malignancies later in life. Derivatives of nitrogen mustards are used currently as chemotherapeutic agents such as cyclophosphamide and ifosfamide. Go to the next page if you knew the correct answer, or click the link image(s) below to further research the concepts in this question (if desired).
Research Concepts: Blister Agents
We update eBooks quarterly and Apps daily based on user feedback. Please tap flag to report any questions that need improvement.
Question 334:
A 65-year-old woman with a history of stage 4 melanoma currently on pembrolizumab and ipilimumab presents to the infusion center for immunotherapy. The nurse cannot draw blood from or infuse drugs into the patient's left subclavian catheter. An ultrasound of the left upper extremity is negative for superficial or deep venous thrombus. What is the best next step in management?
Choices: 1. 2. 3. 4.
Remove the catheter Infuse the catheter with alteplase Start enoxaparin Start rivaroxaban
Answer: 2 - Infuse the catheter with alteplase Explanations: For central catheter-related thrombosis in patients with active malignancy, treatment consists of removing the catheter or anticoagulation if the catheter remains. With no evidence of venous thrombosis in this patient, the best next step is to try and salvage the catheter with fibrinolytic therapy like alteplase. In the presence of deep vein thrombosis, if catheter function is restored with alteplase and the catheter remains, then anticoagulation must be started. If the catheter function is not restored, the catheter must be removed. For anticoagulation in the setting of malignancy, DOACs like rivaroxaban, edoxaban, apixaban, or low molecular weight heparin (LMWH) are preferred options. Go to the next page if you knew the correct answer, or click the link image(s) below to further research the concepts in this question (if desired).
Research Concepts: Cancer-associated Thrombosis
We update eBooks quarterly and Apps daily based on user feedback. Please tap flag to report any questions that need improvement.
Question 335:
A 65-year-old patient is brought with slowly progressive swelling in the submandibular region. The lesion is firm, poorly circumscribed, and unencapsulated. The histology following the excision revealed a basaloid appearance with angulated, hyperchromatic nuclei and scant, clear to eosinophilic cytoplasm. The tumor cells showed a perineural affinity. There were no features of distant metastasis. What is the most relevant prognostic marker in the patient?
Choices: 1. 2. 3. 4.
Brain-derived neurotrophic factor Hematogenous spread Negative tumor margin Mutation
Answer: 3 - Negative tumor margin Explanations: Adenoid cystic cancer is the second most common malignant salivary gland tumor. The tumor also has a characteristic propensity to grow in nerves. The tumor is composed of basaloid cells with small, angulated, and hyperchromatic nuclei and scant cytoplasm arranged into cribriform, tubular, or solid patterns. Tumor grade, stage, lymph node metastasis, invasion of major nerves, and margin status remain the most consistent predictors of prognosis. BDNF is implicated for perineural spread. Adenoid cystic tumors characteristically show lymphatic spread. Go to the next page if you knew the correct answer, or click the link image(s) below to further research the concepts in this question (if desired).
Research Concepts: Adenoid Cystic Cancer
We update eBooks quarterly and Apps daily based on user feedback. Please tap flag to report any questions that need improvement.
Question 336:
A 55-year-old male presents to the clinic for evaluation of dizziness. The patient reports a feeling of dizziness for the past few weeks. He also feels some disturbance in his gait but that is not currently problematic. While talking to the physician, the patient requests the physician to speak closer to his left ear as he has been having difficulty hearing from his left ear. Past medical history is significant for hypertension and diabetes mellitus. The patient regularly takes his medication. His blood pressure is 130/90 mmHg and his last HbA1C was 6.5. Family history is non-contributory. Which of the following is the best diagnostic test for this patient?
Choices: 1. 2. 3. 4.
Computed tomography of the head with contrast Cerebral angiography MRI with and without contrast of the head Brainstem auditory evoked potentials
Answer: 3 - MRI with and without contrast of the head Explanations: This patient most likely has a cerebellopontine angle lesion. Brain MRI not only delineates the extent of the lesion in most cases but the signal characteristics also help narrow the differential. CT of the skull base suffers from hardening artifact. Auditory evoked potentials confirm there is a lesion affecting the cranial nerve but doesn't evaluate which lesion causes the problem. Cerebral angiography plays no role in evaluating the CP angle at this time. Go to the next page if you knew the correct answer, or click the link image(s) below to further research the concepts in this question (if desired).
Research Concepts: Cerebellopontine Angle Cancer
We update eBooks quarterly and Apps daily based on user feedback. Please tap flag to report any questions that need improvement.
Question 337:
Which subtype of basal cell cancer is the most difficult to diagnose?
Choices: 1. 2. 3. 4.
Nodular Morpheaform Cystic Infiltrative
Answer: 2 - Morpheaform Explanations: Morpheaform is a rare type of basal cell cancer and tends to cause collagen deposition that resembles a scar. These lesions appear firm and fibrotic and often get mistaken for a keloid. The morpheaform lesion rarely ulcerates and does not often resemble a typical nodular basal cell cancer. A shave biopsy can diagnose elevated basal cell carcinoma. Most nodular basal cell cancers are diagnosed clinically; however, variants are difficult to distinguish from benign lesions such as intradermal nevus, sebaceomas, fibrous papules, acne scars, and hypertrophic scarring. Basal cell carcinoma is divided into three groups. Superficial basal cell carcinoma is characterized by a superficial proliferation of neoplastic basal cells. This tumor is typically responsive to topical chemotherapy. Infiltrative basal cell carcinoma includes morpheaform and micronodular basal cell cancer and is more difficult to treat, given its tendency to penetrate deeper layers of the skin. Nodular basal cell carcinoma includes most of the remaining categories of basal-cell cancer. Go to the next page if you knew the correct answer, or click the link image(s) below to further research the concepts in this question (if desired).
Research Concepts: Basal Cell Cancer
We update eBooks quarterly and Apps daily based on user feedback. Please tap flag to report any questions that need improvement.
Question 338:
A 71-year-old white man is brought to the emergency department with headache, confusion, and not acting himself since he woke up this morning. The last time known to be normal was the evening before, when he went to sleep. His past medical history is significant of transient ischemic attack 3 years ago and coronary artery disease with one prior stent in his left anterior descending artery (LAD) 5 years ago, gastroesophageal reflux disease (GERD), osteoarthritis, diabetes type 2, and hypertension. His medications include aspirin 81 mg, lisinopril 40 mg daily, lansoprazole 40 mg daily, insulin glargine 15 units at bedtime, metoprolol succinate 100 mg once daily, and paracetamol as needed for joints pain. Vital signs show heart rate 89/min, respiratory rate 23/min, blood pressure 143/90 mmHg, temperature 97.1 F, and oxygen saturation 95% on room air. Upon examination, the patient is drowsy and arousable by painful stimuli only. He is oriented to self only. He does not follow commands. There is no facial droop and speech could not be assessed as the patient is drowsy and non-conversational. The patient is able to move all extremities in response to pain stimuli. Planter reflexes are equivocal bilaterally. The neck is supple with no meningeal signs. Breath sounds are equal bilaterally. Patient value Reference range Hemoglobin 9.1 g/dL 13.2-17.5 g/dL Hematocrit 23.7% 40%-50%% White blood cells 150000/microL 4100-10900/microL 80% (including 30% Neutrophils 54%-62% blasts) Lymphocytes 10% 25%-33% Monocytes 5% 3%-7% Eosinophils 4% 1%-3% Basophils 1% 0.5%-1.0% Platelet count 100000/microL 150000-
400000/microL Patient value Reference range Creatinine 1.5 mg/dL 0.6-1.2 mg/dL BUN 25 mg/dL 5-20 mg/dL Sodium 141 mEq/L 134-144 mEq/L Potassium 3.5 mEq/L 3.6-5.0 mEq/L Glucose 150 mg/dL 65-100 mg/dL Calcium 9.5 mg/dL 8.5-10.2 mg/dL Ethanol level, urine drug screen, and salicylates levels are all negative. Chest x-ray reveals mild cardiomegaly but no infiltrates. Computed tomography of the head reveals no acute intracranial process. Which of the following is the next best step in the management of this patient?
Choices: 1. 2. 3. 4.
tPA administration Cytoreduction and leukapheresis IV vancomycin and gentamicin Mechanical thrombectomy
Answer: 2 - Cytoreduction and leukapheresis Explanations: The patient presents with acute myeloid leukemia (AML) and hyperleukocytosis. This is an oncological emergency that requires urgent cytoreduction, and leukapheresis. Although the patient is presenting with neurological symptoms, his symptoms are nonfocal and are likely attributed to hyperviscosity due to increased blast cells secondary to AML. Blast cells are giant cells that can cause hyperviscosity and reduce organ perfusion. Therefore immediate action with cytoreduction and leukapheresis is necessary to maintain organs perfusion. Infection with meningitis/encephalitis is less likely in this case. The neck is supple with no meningeal signs. The complete blood count with high leukocyte count and 30% blast cells is more consistent with the diagnosis of acute myelocytic anemia and hyperleukocytosis. Go to the next page if you knew the correct answer, or click the link image(s) below to further research the concepts in this question (if desired).
Research Concepts: Neutrophilia
We update eBooks quarterly and Apps daily based on user feedback. Please tap flag to report any questions that
need improvement.
Question 339:
A 19-year-old man is being evaluated for a painless mass in the right testes for the last 1 month. His past surgical history is significant for right-sided orchidopexy done at 7 months old. Besides that, he has no significant past medical or surgical history. His vital signs are within normal limits. He doesn’t smoke but drinks 2-3 glasses of beer every weekend. Ultrasound of the right testes is done, which shows homogeneous hypoechoic intratesticular mass. Orchidectomy is done, and the histopathologic result of the specimen shows cells characteristically have pale to clear cytoplasm, with crisp cytoplasmic membranes and polygonal nuclei with finely granular chromatin and frequently flattened edges. CT scan shows no abnormal masses in the regional or distant nodes and spread of the abnormal cells in the abdomen and thorax. Regarding the follow-up of this patient, what should be done for the next 1 year?
Choices: 1. No followup required for the next 1 year 2. History and physical examination every month 3. Chest x-ray every 3 months 4. History and physical examination every 3-6 months, CT scan of the abdomen at 3, 6, and 12 months
Answer: 4 - History and physical examination every 3-6 months, CT scan of the abdomen at 3, 6, and 12 months
Explanations: Histopathologic and radiologic evidence shows that the patient has stage I seminoma. Orchidectomy is generally curative for stage I seminoma. Following orchidectomy for stage I seminoma, history and physical examination every 3-6 months, and CT scan of the abdomen at 3, 6, and 12 months should be scheduled. Stage I seminoma has a high prognosis rate of around 98-100 %. Go to the next page if you knew the correct answer, or click the link image(s) below to further research the concepts in this question (if desired).
Research Concepts: Seminoma
We update eBooks quarterly and Apps daily based on user feedback. Please tap flag to report any questions that need improvement.
Question 340:
A 65-year-old farmer presents with a two-year history of small scaly lesions on his face. Skin biopsy shows abnormal keratinocytes in the basal layer and increased cellular polarity with the presence of atypical cells. The epidermis is notable for hyperkeratosis and parakeratosis. The skin lesions are confirmed as premalignant, and the photosensitizing agent methyl 5aminolevulinic acid is to be used for daylight photodynamic therapy. What is the recommended incubation time by the FDA before administering the light therapy?
Choices: 1. 2. 3. 4.
2 to 4 minutes 10 to 30 minutes 12 to 18 hours 24 to 48 hours
Answer: 3 - 12 to 18 hours Explanations: Methyl 5-aminolevulinic acid has been FDA-approved, and the incubation time is 14 to 18 hours. Photodynamic therapy utilizes a light-sensitive compound that preferentially accumulates in the actinic keratosis cells. The light therapy is then delivered. With exposure to light, the aminolevulinic acid is converted to protoporphyrin and generates free radicals, which kill the cancer cells. Patients can feel pain with this therapy, and early results show that one treatment may be as effective as topical 5-fluorouracil therapy. Recent studies indicate that shorter incubation times of 1-3 hours may also work. Go to the next page if you knew the correct answer, or click the link image(s) below to further research the concepts in this question (if desired).
Research Concepts: Photodynamic Therapy for Bladder, Esophagus, and Skin We update eBooks quarterly and Apps daily based on user feedback. Please tap flag to report any questions that need improvement.
Question 341:
A 53-year-old woman presents complaining of lower abdominal pain for one month. Her past medical and surgical histories are free. The physical exam reveals a non-distended abdomen with no tenderness. A mass can be palpated in the right lower quadrant. Upon investigations, her CA 125 is 1060.5 U/ml (normal 35 U/ml), other tumor markers are within normal limits. CT scan of the abdomen and pelvis shows enlarged para-aortic lymph nodes and a right ovarian mass. After admission, she develops abdominal pain, rebound tenderness and rigidity, tachycardia, and fever. Her white cell count is 12.150 cells/mm3. A subsequent contrast-enhanced CT scan reveals massive free fluid and gas in the abdomen. What is the most probable cause of her new symptoms?
Choices: 1. 2. 3. 4.
Metastasis to the stomach by peritoneum Disseminated intravascular coagulation Hospital-acquired pseudomembranous colitis Contrast-induced hypersensitivity reaction
Answer: 1 - Metastasis to the stomach by peritoneum Explanations: Dissemination of ovarian cancer occurs commonly to the peritoneum, in form of carcinoma peritonei, which may contribute to ascites and recurrent intestinal obstruction due to infiltration of the bowel wall by malignant cells, also, it may metastasize to the lymph nodes, or to the surrounding structures by direct extension. The stomach is considered a target because it has a rich blood supply. There are some well known extra-intestinal tumors that may metastasize to the stomach like melanoma, lung cancer, and breast cancer. Serum marker CA-125 is a poor screening test, as it can lead to false positive, as it can be elevated in other benign conditions. Abdominal fluid collection and free gas are not associated with disseminated intravascular coagulation, pseudomembranous colitis, or contrast-induced hypersensitivity reactions. Go to the next page if you knew the correct answer, or click the link image(s) below to further research the concepts in this question (if desired).
Research Concepts: Ovarian Cancer
We update eBooks quarterly and Apps daily based on user feedback. Please tap flag to report any questions that need improvement.
Question 342:
An 81-year-old male patient comes into the clinic for evaluation. He has a known diagnosis for prostate cancer. His past medical history is noted for smoking, hypertension managed with medication, and osteoarthritis. His past surgical history was noted for bilateral knee replacements five years ago and transurethral resection of the prostate two months ago. His vitals are stable. He complains of intermittent blood in the urine. He undergoes brachytherapy and then shows up to the clinic one week later with a new symptom. What is the most likely symptom with his presentation?
Choices: 1. 2. 3. 4.
Abdominal distention Rectal incontinence Urinary obstruction Orchitis
Answer: 3 - Urinary obstruction Explanations: The concern of brachytherapy after transurethral resection of the prostate is the increased risk for urethral injury. The radiation from brachytherapy can damage the urethra in the setting of the urethra, already receiving thermal injury from the transurethral resection of the prostate. There is an increased risk for urinary retention for patients who have undergone transurethral resection of the prostate prior to receiving brachytherapy. The range of this occurrence increased from 0-22% to 0-85%, depending on the study. A prior transurethral resection of the prostate (TURP) within the past 3 months is a contraindication for brachytherapy. A TURP done more remotely is not a definite contraindication, and some studies show success with brachytherapy in post-transurethral resection of the prostate patients. Orchitis is not a usual symptom that patients present within the setting of brachytherapy and recent transurethral resection of the prostate. Go to the next page if you knew the correct answer, or click the link image(s) below to further research the concepts in this question (if desired).
Research Concepts: Brachytherapy
We update eBooks quarterly and Apps daily based on user feedback. Please tap flag to report any questions that need improvement.
Question 343:
A 49-year-old woman presents to the clinic for regular follow-up. Her sister recently got diagnosed with breast cancer at the age of 43 years. She was also found to have the BRCA1 mutation. The patient requests genetic testing, which later turns out to be negative. Which of the following is the next best step in the management of this patient?
Choices: 1. 2. 3. 4.
MRI breast Mammograms every 6 months Prophylactic bilateral mastectomy Mammogram every year
Answer: 4 - Mammogram every year Explanations: The patient has family history of BRCA1 breast cancer. Patients with a family history of breast cancer should be counseled about the benefits and limitations of genetic testing. If a BRCA mutation is found, further management could include bilateral mastectomy and oophorectomy. If no mutation is found, routine surveillance with yearly mammograms is recommended. An MRI could have been considered if the patient did not wish to undergo genetic testing. Prophylactic mastectomy would not be recommended if there is no genetic predisposition of the patient to develop breast cancer. Go to the next page if you knew the correct answer, or click the link image(s) below to further research the concepts in this question (if desired).
Research Concepts: Breast Cancer Screening
We update eBooks quarterly and Apps daily based on user feedback. Please tap flag to report any questions that need improvement.
Question 344:
A 65-year-old male presents with complaints of fatigue, generalized weakness, polyuria, and polydipsia for the past seven days. A detailed medical history reveals that he has recently been diagnosed with hypertension and is on multiple medications but without blood pressure control. His vitals show a blood pressure of 190/100 mmHg, pulse rate of 90/min, and respiratory rate of 18/min. Physical examination is unremarkable. A CT scan of the abdomen reveals a mass in the right adrenal gland. What laboratory findings most likely will be found in this patient?
Choices: 1. Increased serum potassium, increased urinary potassium, and increased extracellular fluid volume 2. Decreased serum potassium, increased urinary potassium, and increased extracellular fluid volume 3. Increased serum potassium, decreased urinary potassium, and decreased extracellular fluid volume 4. Decreased serum potassium, increased urinary potassium, and decreased extracellular fluid volume
Answer: 2 - Decreased serum potassium, increased urinary potassium, and increased extracellular fluid volume
Explanations: The clinical scenario suggests the diagnosis of Conn syndrome. Conn syndrome refers to primary hyperaldosteronism caused by excess aldosterone production, in most cases from a benign adrenal tumor. Laboratory investigations relate to the increased production of aldosterone. Serum levels of potassium are decreased, and the urinary excretion of potassium is increased. Decreased serum potassium levels result in the symptoms of polyuria and polydipsia due to hypokalemia-induced nephrogenic diabetes insipidus. As a result of increased reabsorption of sodium due to aldosterone excess, more water is retained. the retained water causes extracellular fluid volume expansion, which is the mechanism behind the persistent hypertension. Unilateral adenomas causing increased aldosterone and hypertension can be resected surgically by laparoscopy. Spironolactone is the first-line medical treatment for Conn syndrome caused by bilateral hyperplasia of the adrenals. Go to the next page if you knew the correct answer, or click the link image(s) below to further research the concepts in this question (if desired).
Research Concepts: Conn Syndrome
We update eBooks quarterly and Apps daily based on user feedback. Please tap flag to report any questions that need improvement.
Question 345:
Which is a possible side effect of leuprolide (GnRH analogue) therapy for prostate cancer?
Choices: 1. 2. 3. 4.
Testicular tenderness Urinary hesitancy Increased libido Gynecomastia
Answer: 4 - Gynecomastia Explanations: A possible side effect is gynecomastia which is breast enlargement in males. Other side effects include gastrointestinal upset, dizziness, decreased libido, emotional lability, hot flashes. GnRH analogs act as inhibitors of gonadotropin release and are palliative treatments for advanced prostate cancer. Leuprolide is an FDA-approved gonadotropin-releasing hormone agonist that is used for the management of endometriosis, uterine leiomyomata (also known as uterine fibroids), treatment of central precocious puberty in children, and advanced prostate cancer. Offlabel uses include management of breast cancer, hormone therapy for male to female transgender patients, premenopausal ovarian suppression and management of paraphilia and hypersexuality. Go to the next page if you knew the correct answer, or click the link image(s) below to further research the concepts in this question (if desired).
Research Concepts: Leuprolide
We update eBooks quarterly and Apps daily based on user feedback. Please tap flag to report any questions that need improvement.
Question 346:
A 17-year-old G1P0 girl presents to her provider for her first prenatal care. She is at 18 weeks age of gestation and reports no symptoms. Prenatal ultrasound is done, showing posterior midline multicystic lesions with internal septations. Amniocentesis reveals a fetal karyotype of 45 XO, and the alpha-fetoprotein level is high. What is the underlying pathology in the fetus?
Choices: 1. 2. 3. 4.
Failure of lymphatic sacs to develop connections Atrophy of lymphatic channels Neoplastic growth of normal lymphatics Inflammation of lymphatic ducts
Answer: 1 - Failure of lymphatic sacs to develop connections
Explanations: Endothelial buds sprout from the primordial lymphatic system. These buds become the peripheral lymphatic plexus. If one of the lymphatic sacs of the internal jugular lymphatics fails to develop proper connections with the lymphatics and eventually venous systems, focal lymph cysts are formed. These are called cystic hygromas or cavernous lymphangiomas. Cystic hygroma is an important marker of aneuploidy, generally including trisomies, but is particularly common in Turner syndrome. The presence of cystic hygroma on ultrasound is attributable to Turner syndrome in 30 to 70% of cases. Go to the next page if you knew the correct answer, or click the link image(s) below to further research the concepts in this question (if desired).
Research Concepts: Cystic Hygroma
We update eBooks quarterly and Apps daily based on user feedback. Please tap flag to report any questions that need improvement.
Question 347:
A 48-year-old woman presents to the clinic with a persistent lump in her left neck. She denies weight loss, fever, night sweats, or palpitations. Her past medical and family history are unremarkable, and her vitals are within normal limits. On physical exam, there is a fullness on the right side of the neck at the level of the hyoid. A mass is palpated and movable horizontally but not vertically. A CT of the neck with contrast demonstrates a 3.8 cm hypervascular mass at the junction of the right external and internal carotid artery causing splaying of the two vessels. A follow-up MRI shows a salt and pepper appearance on T1 and T2 sequence. What is the most appropriate next step in the evaluation of this patient?
Choices: 1. 2. 3. 4.
CT guided biopsy Urinary catecholamines PET/CT Genetic testing
Answer: 2 - Urinary catecholamines Explanations: Carotid body paragangliomas are the most common type of paragangliomas and will typically present as an asymptomatic painless mass. They are most often benign but may be malignant and/or catecholamine secreting in 5-10% of patients. Cranial neuropathies may also occur if the tumor is sufficiently large. Although carotid body paragangliomas are rarely associated with catecholamine secretion, if catecholamines are detected, a further workup for pheochromocytoma is required. If confirmed, it should be removed prior to addressing the carotid body tumor. Surgery is the mainstay of treatment. Radiographically the tumors appear hyper-enhancing on CT with contrast due to their hypervascular nature. If they are sufficiently large, they may cause “lyre sign,” which is a splaying of the external and internal carotid artery. On MRI, they have a characteristic “salt and pepper” appearance. Paragangliomas may be associated with certain genetic syndromes, including von Hippel-Lindau, multiple endocrine neoplasia types IIA and IIB, and Carneys syndrome, although the majority are sporadic. Radiographic appearance and location are usually sufficient to make a diagnosis. Biopsies are invasive, and there is a risk of bleeding and cranial nerve injury. They would only be indicated if the diagnosis is unclear from the imaging. The majority of these lesions do not metastasize. Therefore, a PET/CT is not indicated or recommended for imaging this tumor. Genetic testing would be appropriate if the patient had a family or
personal history of certain tumor types such as medullary thyroid cancer, hemangioblastomas, or pheochromocytomas. Go to the next page if you knew the correct answer, or click the link image(s) below to further research the concepts in this question (if desired).
Research Concepts: Paraganglioma
We update eBooks quarterly and Apps daily based on user feedback. Please tap flag to report any questions that need improvement.
Question 348:
A 36-year-old male patient with a past medical history of hypertension and Hodgkin lymphoma currently on chemotherapy presented to the clinic with a chief complaint of bilateral hand pain and increased sensitivity to cold. Which of the following is the preferred next step in the management of this patient?
Choices: 1. Prescribe topical analgesics 2. Electromyography and nerve conduction studies of bilateral upper extremities 3. Prescribe gabapentin 4. Review chemotherapeutic regimen
Answer: 4 - Review chemotherapeutic regimen Explanations: This patient is most likely experiencing chemotherapyinduced peripheral neuropathy. Vincristine is a common chemotherapeutic used in the treatment of hematological malignancies and is a well-known neurotoxin. Vincristine neurotoxicity affects both the central and peripheral nervous systems (CNS) and (PNS). Peripheral neuropathies usually manifested as sensory abnormalities with numbness or tingling and hyperesthesias, such as cold intolerance. Sensory abnormalities are commonly accompanied by loss of deep tendon reflexes. Motor abnormalities are less common and can occur with and without sensory abnormalities. Centrally, it can cause autonomic neuropathies leading to constipation and orthostatic hypotension. Vincristine neurotoxicity proceeds in a dose-dependent manner and is often the dose-limiting factor. Go to the next page if you knew the correct answer, or click the link image(s) below to further research the concepts in this question (if desired).
Research Concepts: Hyperesthesia
We update eBooks quarterly and Apps daily based on user feedback. Please tap flag to report any questions that need improvement.
Question 349:
A 4-year-old boy presents with intermittent headache and decreased vision for the past month. Examination is notable for a relative afferent pupillary defect in the right eye. MRI of the orbits reveals fusiform enlargement and expansion of the intercanalicular portion of the optic nerve. Further evaluation is negative for neurofibroma, Lisch nodule, café au-lait spots, axillary or inguinal freckling, or skeletal abnormalities. Also, there is no family history of neurofibromatosis type-1. Given the likely diagnosis, which of the following is the most common genetic aberration associated with this patient's condition?
Choices: 1. 2. 3. 4.
Genetic silencing of both NF1 alleles BRAFv600 mutation BRAF-KIAA1549 fusion Gene that encodes p53
Answer: 3 - BRAF-KIAA1549 fusion Explanations: This patient has an optic pathway glioma without clinical signs or a family history of neurofibromatosis type-1. Although genetic testing should be done to confirm, this patient likely has a sporadic optic nerve glioma. The most common genetic aberration in a sporadic optic pathway glioma is a BRAF-KIAA1549 fusion. This activates the MEK/MAPK/ERK pathway causing proliferation, survival, and tumorigenesis. A less common mutation in these tumors is the BRAF-V600 mutation. The most common presenting symptom of optic pathway glioma is headache. Other symptoms include visual acuity deficit, convulsions, nausea, dizziness, squinting, developmental regression, and growth retardation. NF1-associated optic pathway gliomas show genetic silencing of both NF1 alleles. Go to the next page if you knew the correct answer, or click the link image(s) below to further research the concepts in this question (if desired).
Research Concepts: Optic Nerve Glioma
We update eBooks quarterly and Apps daily based on user feedback. Please tap flag to report any questions that need improvement.
Question 350:
A 58-year-old man presents with intermittent diarrhea, constipation, and flatulence for the past four weeks. He undergoes an initial screening colonoscopy, which reveals three sessile structures with finger-like projections in the colon. A biopsy is performed, which shows an adenoma with villous features and sessile serrated polyps with dysplasia. Both were totally resected. What colorectal screening test is most appropriate in this patient?
Choices: 1. 2. 3. 4.
Repeat sigmoidoscopy in 1 year Repeat colonoscopy in 3 years Repeat colonoscopy in 5 to 10 years Contrast abdominal CT
Answer: 2 - Repeat colonoscopy in 3 years Explanations: Repeat colonoscopy three years after initial screening colonoscopy for an average risk individual is the current recommendation for a patient found to have 3 to 10 adenomas, or a large (at least 1 cm) adenoma, or any adenomas with high-grade dysplasia or villous features. Adenomatous polyps can be villous, tubulovillous, and tubular. Tubular polyps are the most common ones. However, villous adenomas are associated with larger adenomas and a more severe degree of dysplasia occurring more commonly in the rectum and the rectosigmoid. Although they may occur in any part of the colon, they usually appear as sessile structures with velvety or cauliflower-like projections where the columnar epithelium is predominant, and immature goblet cells may also be observed. The most common presenting symptom is hematochezia with anemia, most likely microcytic. Other non-specific symptoms include diarrhea, constipation, and change of stool caliber. The patient may have a positive family history of colon polyps. Endoscopy is the most sensitive test, and a biopsy can also be taken along with it. Fecal occult blood testing is also done, and 20 to 40 percent of patients with adenomas have positive findings. If possible, the polyp should be removed during endoscopy. Surgical resection of a polyp may be required if it is larger than 2 to 3 cm and is sessile, as in the case of villous adenoma. Go to the next page if you knew the correct answer, or click the link image(s) below to further research the concepts in
this question (if desired).
Research Concepts: Villous Adenoma
We update eBooks quarterly and Apps daily based on user feedback. Please tap flag to report any questions that need improvement.
Question 351:
A 62-year-old man presents with a three-day history of a new-onset frontal headache that progressed to a generalized headache. He describes the pain as a nine out of ten, throbbing, and does not improve with over the counter medications. He denies any changes in vision, vomiting, seizures, limb weakness, behavioral changes, or weight loss. He has a past medical history of hypertension, type 1 diabetes mellitus, and a 37 pack-year smoking history. However, he is not compliant with his medications. He denies any family history of a cerebral hemorrhage. His vital signs show oxygen saturation 98% on room air, respiratory rate 26 per minute, heart rate 79 bpm, blood pressure 162/96 mmHg, and temperature 98 F. The physical examination is unremarkable. A CT scan of the brain reveals an isolated hyperdense right temporal occipital lobe lesion measuring 1.7 cm in its longest axis. An MRI shows a large right temporal occipital lesion, with several smaller lesions bilaterally with surrounding edema. What is the most appropriate initial step in the management of this patient's condition?
Choices: 1. 2. 3. 4.
Administer intravenous dexamethasone Surgical resection Whole-brain radiotherapy Stereotactic radiosurgery
Answer: 1 - Administer intravenous dexamethasone Explanations: The first step in the management of newly diagnosed brain metastases is the treatment of intracranial edema. Oral or intravenous steroids, such as dexamethasone, are commonly used. A loading dose of 10 mg intravenous (IV) dexamethasone followed by 4 mg IV every six hours is one dosing regimen. After the initial clinical response, which can occur rapidly, the dose may be tapered to avoid many of the long-term high-dose steroid administration's adverse effects. Following the initiation of steroids, definitive management may be initiated. Treatment options include surgical resection (for limited brain metastases in patients with good performance status and surgically accessible lesions), whole-brain radiotherapy, and stereotactic radiosurgery. Go to the next page if you knew the correct answer, or click the link image(s) below to further research the concepts in this question (if desired).
Research Concepts: Brain Metastasis
We update eBooks quarterly and Apps daily based on user feedback. Please tap flag to report any questions that
need improvement.
Question 352:
A 58-year-old woman is brought to the hospital with drenching night sweats and fever. On examination, she has palpable non-tender cervical and axillary adenopathy. Labs show evidence of lymphocytosis with an absolute lymphocyte count of 30,000/microL with hemoglobin of 12 g/dL and platelets of 110,000/microL. CT scan shows cervical, axillary, and mediastinal lymph node enlargement. Flow cytometry on peripheral blood showed a kappa light chain restricted cell population that expresses CD19, dim CD20, CD5, and CD23 and more strongly expresses BCL2. The FISH panel for CLL/SLL shows a loss of 17p with no other abnormality in other FISH probes. Which of the following is the strongest indication for treatment in this patient?
Choices: 1. 2. 3. 4.
Fever and drenching night sweats 17p deletion Absolute lymphocyte count of 30,000/microL Strong expression of BCL-2
Answer: 1 - Fever and drenching night sweats Explanations: Indication for the treatment of chronic lymphocytic leukemia is based on symptoms and not based on cytogenetics. The fact that the patient presented with fever and drenching night sweats would justify the need for treatment. CLL is a chronic lymphoid malignancy where the treatment is based on the symptoms. As such, the treatment has not been shown to change the natural history of the disease. As a result, we do not recommend the treatment of asymptomatic patients. 17p deletion predicts the unfavorable outcome. However, as such, it does not justify the need for treatment. Given the fact this patient had symptoms, that would justify the need for treatment. Absolute lymphocyte count is not a requisite for treatment. However, rapid lymphocyte doubling time is important, and no such information was provided in the stem. The IgHV mutation is not required for treatment and, as such, actually predicts a favorable outcome. Go to the next page if you knew the correct answer, or click the link image(s) below to further research the concepts in this question (if desired).
Research Concepts: Chronic Lymphocytic Leukemia With Variant Genetics
We update eBooks quarterly and Apps daily based on user feedback. Please tap flag to report any questions that need improvement.
Question 353:
A 27-year-old male patient presented with a five-month history of low back pain and bilateral sciatica. Magnetic resonance imaging of the lumbar region demonstrated an intradural tumor with L4 to S2 extension. The patient underwent L3–S3 laminoplasty. An intradural tumor highly adherent to the lumbosacral nerve roots was found and was partially removed. Histologically, the tumor was composed of low-grade ependymal cells surrounding blood vessels, producing the papillary appearance and perivascular myxoid material between blood vessels and ependymal cells, creating the myxopapillary appearance. Nuclear pleomorphism and mitotic figures were absent. There was no necrosis. The tumor cells showed positive immunostaining for vimentin and glial fibrillary acidic protein (GFAP) but were negative for epithelial membrane antigen (EMA). What is the usual macroscopic aspect of this tumor?
Choices: 1. Reddish to purplish in color, sausage-shaped, and soft with sometimes hemorrhagic or mucinous degeneration 2. Grey-white in color, dumbbell-shaped, and firm with extensive necrosis 3. White in color rounded, and firm with occasional necrosis 4. Yellowish in color, ovoid, and firm with extensive mucinous degeneration
Answer: 1 - Reddish to purplish in color, sausageshaped, and soft with sometimes hemorrhagic or mucinous degeneration
Explanations: Macroscopically, myxopapillary ependymomas are often encapsulated and soft. They have a reddish to purplish color and are sausage-shaped. They may demonstrate hemorrhagic or mucinous degeneration. The macroscopic and radiological findings are not pathognomonic of myxopapillary ependymoma. The diagnosis relies on the histopathological examination of the surgical specimen. The gross features of the tumor do not influence the prognosis of myxopapillary ependymoma. Clinical symptoms of myxopapillary ependymoma are related to size, local extent, and site of the tumor. In the present case, the tumor was adherent to the lumbosacral nerve and was partially removed. Partial removal is associated with a higher risk of recurrence; therefore, the patient should receive adjuvant radiotherapy. Go to the next page if you knew the correct answer, or click the link image(s) below to further research the concepts in this question (if desired).
Research Concepts: Myxopapillary Ependymoma
We update eBooks quarterly and Apps daily based on user feedback. Please tap flag to report any questions that need improvement.
Question 354:
What is the estimated 5-year survival for a stage one lung cancer patient?
Choices: 1. 2. 3. 4.
5% 25% 50% 75%
Answer: 4 - 75% Explanations: The estimated 5-year survival for a patient with stage one lung cancer averages about 75%. Stage two survival is about 50%. Stage three survival drops to 30% or less. Stage four lung cancer has less than a 5% survival rate. Go to the next page if you knew the correct answer, or click the link image(s) below to further research the concepts in this question (if desired).
Research Concepts: Lung Cancer
We update eBooks quarterly and Apps daily based on user feedback. Please tap flag to report any questions that need improvement.
Question 355:
A 71-year-old man with a history of hypertension and diabetes presents to the clinic with abdominal discomfort and weight loss and further evaluation is suggestive of a periampullary mass. He undergoes Whipple surgery, and the pathological assessment confirms ampullary cancer. Pathologically, he is staged as T1N1M0 as 3 out of 12 lymph nodes are positive and called R1 resection. Which of the following is the most appropriate adjuvant treatment strategy for this patient?
Choices: 1. 2. 3. 4.
Chemotherapy alone Radiation alone Chemotherapy and radiation No adjuvant treatment
Answer: 3 - Chemotherapy and radiation Explanations: The treatment intent for this patient is a cure. While chemotherapy alone for 6 months is a reasonable option in the adjuvant setting for the node-positive disease, he also had margin positive resection. Therefore, radiation could be beneficial in terms of local control. Given node positivity, he will benefit from systemic chemotherapy terms of distant control and he also needs radiation for local control due to margin positive resection. Radiation alone has no role for this patient as the patient already has needs systemic control in combination with the radiation. No postoperative treatment is not a standard of care for node-positive patients. Go to the next page if you knew the correct answer, or click the link image(s) below to further research the concepts in this question (if desired).
Research Concepts: Ampullary Cancer
We update eBooks quarterly and Apps daily based on user feedback. Please tap flag to report any questions that need improvement.
Question 356:
A 40-year-old male, post recent kidney transplant, presents with a new 1.5 cm nonhealing ulcer on his soft palate. The ulcer is painful and is affecting his ability to eat. His physical exam reveals some mild cervical lymphadenopathy but is otherwise unremarkable. Biopsy of this lesion reveals the infiltration of EBER positive atypical B-cells. Further workup is negative for systemic involvement. Which of the following will help distinguish this entity from diffuse large B-cell lymphoma?
Choices: 1. The localized nature of this disease process and the absence of a mass lesion. 2. The presence of sharp circumscription and a band of large B-cells at the base of the ulcer. 3. The presence of CD30 and CD15 positive Reed-Sternberg like cells on immunohistochemistry. 4. EBER positivity in only the small atypical cells highly suggests a diagnosis of diffuse large B-cell lymphoma.
Answer: 1 - The localized nature of this disease process and the absence of a mass lesion.
Explanations: The clinical features of EBV mucocutaneous ulcer, specifically the localized nature and absence of a mass lesion, are integral in distinguishing it from diffuse large B-cell lymphoma. The presence of sharp circumscription and a band of small T cells at the base of the ulcer can differentiate EBV positive mucocutaneous ulcer from the more infiltrative patterns of diffuse large B cell lymphoma. Finding EBV positivity in a variety of cell sizes favors EBV positive mucocutaneous ulcers. When diffuse large B-cell lymphoma or classical Hodgkin lymphoma is EBV related, typically only the large cells are Epstein-Barr virus (EBV) encoded small nuclear RNAs (EBER) positive. EBV positive mucocutaneous ulcer also shares morphologic and phenotypic characteristics with classical Hodgkin lymphoma, specifically the presence of CD30/CD15 positive Reed-Sternberg like cells. Go to the next page if you knew the correct answer, or click the link image(s) below to further research the concepts in this question (if desired).
Research Concepts: EBV Positive Mucocutaneous Ulcer
We update eBooks quarterly and Apps daily based on user feedback. Please tap flag to report any questions that need improvement.
Question 357:
A 52-year-old African-American male with no significant past medical history presented with abdominal discomfort for the past 2 weeks. The patient denies loss of weight, change in appetite flushing, or diarrhea. Vital signs and exam are normal. Evaluation with ultrasound is notable for a lesion in the tail of the pancreas. The patient underwent multiphase MRI that showed low signal intensity on T1-weighted images and high signal intensity on T2-weighted images. Pancreatic cancer is suspected. Tumor markers including CA 19-9 and CEA are within normal limits. What is the next step in terms of diagnostic intervention?
Choices: 1. 2. 3. 4.
CT scan PET scan Endoscopic ultrasound Laproscopic biopsy
Answer: 3 - Endoscopic ultrasound Explanations: Based on the clinical signs the patient likely has pancreatic neoplasm. The MRI findings were suggestive of possible neuroendocrine neoplasms. The neuroendocrine tumors are not as aggressive as pancreatic adenocarcinomas and may present indolently. The next best diagnostic intervention is endoscopic ultrasound to further characterization of the pancreatic lesion and involvement of surrounding vasculature. This will also facilitate histological confirmation by biopsy. Once the patient is diagnosed with a neuroendocrine tumor, specific pet imaging such as gallium dotatate is suggested to evaluate occult disease. Laparoscopic biopsies are not needed as it is invasive. Go to the next page if you knew the correct answer, or click the link image(s) below to further research the concepts in this question (if desired).
Research Concepts: Neuroendocrine Tumor Lu-177-Dotatate Therapy We update eBooks quarterly and Apps daily based on user feedback. Please tap flag to report any questions that need improvement.
Question 358:
A 46-year-old patient with Hodgkin lymphoma was given treatment for his lymphoma with medication on a high dose. After 6 months of treatment, the patient developed signs and symptoms of congestive heart failure. Even during the early stages of treatment with this medication, the patient showed signs of cardiotoxicity in the form of minor EKG changes. What is the description of the liposomal technology used with this drug?
Choices: 1. 2. 3. 4.
Stealth liposomes Targeted liposome Virosome Gene-based liposome
Answer: 1 - Stealth liposomes Explanations: Liposomes are phospholipid spheres enclosing an aqueous inner vesicle. Liposomes can be used to target drug delivery, increase bioavailability, and protect the bioactive contents from gastric destruction. Newer liposomal technologies, such as the stealth system, are designed to extend half-lives, reduce toxicity, and better control drug release. Targeted and gene-based liposomes are not used with doxorubicin. Go to the next page if you knew the correct answer, or click the link image(s) below to further research the concepts in this question (if desired).
Research Concepts: Doxorubicin
We update eBooks quarterly and Apps daily based on user feedback. Please tap flag to report any questions that need improvement.
Question 359:
A 65-year-old male with gastroesophageal reflux history presented with epigastric pain, epigastric fullness, dysphagia, anorexia, and weight loss. Esophagogastroscopy revealed an ulceroproliferative, intraluminal mass in the distal esophagus obstructing the esophageal lumen. Biopsy showed small cell esophageal carcinoma. Contrast-enhanced chest and abdominal computed tomography demonstrated a large tumor of the distal third of the esophagus without any lymphadenopathy or distant metastasis. What is the course of treatment for this patient?
Choices: 1. Preoperative chemotherapy for 3 months followed by en bloc esophagectomy with two-field lymph node dissection, proximal gastrectomy, cervical esophagogastric anastomosis, and 3 months of postoperative chemotherapy with the same agents as before. 2. Preoperative chemotherapy for 6 months followed by en bloc esophagectomy with two-field lymph node dissection, proximal gastrectomy, cervical esophagogastric anastomosis, and 6 months of postoperative chemotherapy with the same agents as before. 3. En bloc esophagectomy with two-field lymph node dissection, proximal gastrectomy, and cervical esophagogastric anastomosis, and 6 months of postoperative chemotherapy for the complete eradication of the tumor. 4. En bloc esophagectomy with two-field lymph node dissection, proximal gastrectomy, cervical esophagogastric anastomosis, and 3 months of postoperative chemotherapy for the complete eradication of the tumor.
Answer: 1 - Preoperative chemotherapy for 3 months followed by en bloc esophagectomy with two-field lymph node dissection, proximal gastrectomy, cervical esophagogastric anastomosis, and 3 months of postoperative chemotherapy with the same agents as before.
Explanations: Preoperative chemotherapy with cisplatin and etoposide for 3 months results in a significant reduction of the tumor. After that en bloc esophagectomy with two-field lymph node dissection, proximal gastrectomy, cervical esophagogastric anastomosis, and 3 months of postoperative chemotherapy is recommended for the complete eradication of the tumor. Endoscopic eradication therapy (EET) for Barrett dysplasia includes radiofrequency ablation therapy, cryotherapy, endoscopic mucosal ablation. Endoscopic mucosal resection is an acceptable alternative to surgical resection for high-grade dysplasia nodular lesion associated with Barrett's esophagus. Go to the next page if you knew the correct answer, or click the link image(s) below to further research the concepts in this question (if desired).
Research Concepts: Barrett Esophagus
We update eBooks quarterly and Apps daily based on user feedback. Please tap flag to report any questions that need improvement.
Question 360:
A 47-year-old woman with a history of prior fibrocystic disease presents to the office with a palpable right-sided breast mass she noticed last week. The patient denies tenderness and states that she has no family history of breast cancer. On exam, the mass is firm, nontender, mobile within the breast. The patient gets a mammogram as well as ultrasound evaluating the mass, which shows a type 3 mass per Berg et al. criteria. Which of the following is the most appropriate biopsy option for this patient?
Choices: 1. 2. 3. 4.
Core needle biopsy Fine needle aspiration Stereostatic biopsy Excisional biopsy
Answer: 1 - Core needle biopsy Explanations: By Berg et al. criteria, a type 3 breast mass has both mixed cystic and solid components. Due to the solid components of this mass, a core needle biopsy would be most appropriate in order to diagnose and evaluate the solid components of the mass. This is also less invasive than an excisional biopsy. FNAC will not be able to properly evaluate the solid components of the mass. Stereostatic biopsy is usually performed for masses that show microcalcifications. An excisional biopsy may eventually be required depending on the results of the CNB. Go to the next page if you knew the correct answer, or click the link image(s) below to further research the concepts in this question (if desired).
Research Concepts: Breast Cyst
We update eBooks quarterly and Apps daily based on user feedback. Please tap flag to report any questions that need improvement.
Question 361:
A 16-year-old boy with Down syndrome was brought to the clinic because of fever, fatigue, and generalized malaise for the past few days. On examination, his conjunctiva is pale, and gum hypertrophy is present. Several bruises and petechiae are present on his lower limbs. Abdominal examination reveals an enlarged spleen. Full blood count shows Hb 8.6 g/dL, platelet count 66 × 10^9/L, WBC count 120 × 10^9/L, and absolute neutrophil count (ANC) 480 cells/µL. Peripheral smear shows blast cells in abundance. Bone marrow aspirate: 30% blasts with red needle-like cytoplasmic inclusions. What is the most likely diagnosis?
Choices: 1. 2. 3. 4.
Acute lymphoblastic leukemia (ALL) Chronic lymphocytic leukemia (CLL) Acute myeloblastic leukemia (AML) Chronic myeloid leukemia (CML)
Answer: 3 - Acute myeloblastic leukemia (AML) Explanations: Children with Down syndrome and other inherited conditions are predisposed to the development of myeloid proliferation and are at an increased risk of developing leukemia. The patients with AML present with the symptoms of cytopenias, including pallor, fever, lethargy, petechiae, easy bruising, and recurrent infections. Gum hypertrophy and bleeding may occur from the infiltration of blast cells. White blood cell counts vary, but the ANC is often 500 cells/µL. Myeloblasts may be seen in peripheral smears. A bone marrow aspirate with a biopsy that shows 20% myeloblasts is diagnostic for AML. The presence of Auer rods (red needle-like structures composed of azurophilic granular material in the cytoplasm of myeloblasts) is another diagnostic feature seen in AML. Go to the next page if you knew the correct answer, or click the link image(s) below to further research the concepts in this question (if desired).
Research Concepts: Acute Myeloid Leukemia
We update eBooks quarterly and Apps daily based on user feedback. Please tap flag to report any questions that need improvement.
Question 362:
A 56-year-old African-American male comes to the clinic complaining of difficulty starting and maintaining a steady stream of urine, increased frequency, and urgency of urination for the past two months. He also admits noticing streaks of blood on three occasions in his urine. His father died of prostate cancer at the age of 70. The patient refuses to be evaluated for prostate cancer and asks for symptomatic treatment. What is the next step of management?
Choices: 1. Treat him symptomatically 2. Explain that he is at high risk and will have to undergo an evaluation 3. Proposing a thorough discussion before making a firm decision regarding the refusal of screening 4. Discussing the patient's condition with his family members
Answer: 3 - Proposing a thorough discussion before making a firm decision regarding the refusal of screening
Explanations: According to U.S. Preventive Services Task Force guidelines, in men aged 55 to 69 years old, the decision to undergo periodic prostate-specific antigen (PSA) screening should be an individual one after a thorough upfront discussion between the provider and patient regarding risks, benefits, and limitations of such screening incorporating patient's values and preferences in decision making. It is important to ensure that the patient makes an informed decision after explaining the benefits and risks associated with further interventions and management. If the patient denies being evaluated after this discussion, he should not have further evaluation. African-American men have a high risk of developing prostate cancer. A family history of having a first-degree relative with prostate cancer increases the patient's risk of developing prostate cancer. Go to the next page if you knew the correct answer, or click the link image(s) below to further research the concepts in this question (if desired).
Research Concepts: Prostate Cancer Screening
We update eBooks quarterly and Apps daily based on user feedback. Please tap flag to report any questions that need improvement.
Question 363:
A 60-year-old female with a keratotic papule on her trunk presents to the dermatologist. The lesion is biopsied, and the histopathology shows a column of tightly fitted parakeratotic cells overlying the epidermis with an absent granular layer. If this lesion had not been removed, progression to which malignancy would be most likely to have occurred?
Choices: 1. 2. 3. 4.
Melanoma Basal cell carcinoma Keratoacanthoma Squamous cell carcinoma
Answer: 4 - Squamous cell carcinoma Explanations: Porokeratosis can undergo malignant transformation in 6.9% to 30% of cases. Porokeratosis most frequently undergoes malignant transformation to squamous cell carcinoma. Porokeratosis can undergo malignant transformation to basal cell carcinoma, but this is less common. The distinctive histopathologic feature of porokeratosis is a cornoid lamella, which is a column of tightly fitted parakeratotic cells. The column of parakeratosis will occur over the epidermis with an absent granular layer and dyskeratotic cells in the upper spinous zone. Go to the next page if you knew the correct answer, or click the link image(s) below to further research the concepts in this question (if desired).
Research Concepts: Porokeratosis
We update eBooks quarterly and Apps daily based on user feedback. Please tap flag to report any questions that need improvement.
Question 364:
A 70-year-old man presents to the clinic with worsening back pain and numbness in the hands and legs that have been going on for the past 3 months. On examination, he has a pulse rate of 62/min and blood pressure 125/85 mmHg. Blood results show hemoglobin of 10 g/dL, platelet count 120,000/microL, and urine protein electrophoresis reveals Bence Jones proteins. His past medical history includes an inguinal hernia repair, asthma as a child, multiple myeloma, and left elbow bursitis. It is decided that the patient will be put on dexamethasone and lenalidomide, as well as ixazomib for treatment. Which of the following best describes the mechanism of action of ixazomib useful for this patient?
Choices: 1. 2. 3. 4.
HMG-CoA reductase inhibitor Non-selective inhibitor of cyclooxygenase Proteasome inhibitor TNF-alpha inhibitor
Answer: 3 - Proteasome inhibitor Explanations: Ixazomib is an oral proteasome inhibitor that causes apoptosis of multiple myeloma cells. Ixazomib is prescribed to patients with multiple myeloma who have had at least one prior treatment for multiple myeloma. Ixazomib is the first oral proteasome inhibitor. Ixazomib has been demonstrated to have a progressionfree survival advantage. Go to the next page if you knew the correct answer, or click the link image(s) below to further research the concepts in this question (if desired).
Research Concepts: Ixazomib
We update eBooks quarterly and Apps daily based on user feedback. Please tap flag to report any questions that need improvement.
Question 365:
An elderly, febrile patient suffering from weight loss and fatigue has small, firm violaceous nodules, which are shown on biopsy to contain myelogenous blasts. These findings suggest what hematologic condition?
Choices: 1. 2. 3. 4.
Acute myelocytic leukemia Hodgkin lymphoma Non-Hodgkin lymphoma Chronic myeloid leukemia
Answer: 1 - Acute myelocytic leukemia Explanations: The nodules are a manifestation of leukemia cutis. They are most frequently associated with acute monocytic leukemia. Other physical signs of AML include pallor, cardiac murmur, fever and marked weight loss. Patients with thrombocytopenia often have petechiae and hemorrhagic rashes. Infiltration of leukemic cells may also cause hepatosplenomegaly and lymphadenopathy. In rare cases, one may see chloromata or a bony lesion that precedes full blown AML. Go to the next page if you knew the correct answer, or click the link image(s) below to further research the concepts in this question (if desired).
Research Concepts: Acute Myeloid Leukemia
We update eBooks quarterly and Apps daily based on user feedback. Please tap flag to report any questions that need improvement.
Question 366:
A 59-year-old female presents to the clinic with a one-month history of proximal muscle weakness, central obesity, and edema. Additionally, she has noticed, over the past month, a persistent cough, hoarseness, fatigue, and shortness of breath. The patient has smoked one pack per day for the past 30 years. Her blood pressure is 134/82 mmHg, her heart rate is 88 beats per minute, her respiratory rate is 14 breaths per minute, and temperature is 98.0°F (36.7°C). Her breath sounds are absent over the right upper lobe. A computed tomography (CT) scan is performed and confirms a 5.8 cm mass in the right upper lobe, which is located centrally. Serum studies show a serum chloride of 100 mEq/L, sodium of 130 mEq/L, and potassium of 4.0 mEq/L. Additionally, the patient has a fasting glucose of 122 mg/dL. Which of the following is most likely to be present in this patient?
Choices: 1. 2. 3. 4.
Ectopic corticotropin production Elevated parathyroid hormone-related protein Elevated urine 5-HIAA Elevated beta-hCG
Answer: 1 - Ectopic corticotropin production Explanations: This patient presents with symptoms of proximal muscle weakness, central obesity, and edema, in addition to a persistent cough, hoarseness, fatigue, hemoptysis, and shortness of breath. A CT scan confirms a mass in the right upper lobe, and along with hyponatremia, points towards the diagnosis of an antidiuretic hormone (ADH) producing small-cell carcinoma of the lung. Ectopic Cushing syndrome occurs in 2% to 5% of smallcell carcinoma patients. Production of ectopic corticotropin (ACTH) results in elevated free cortisol levels. Additionally, it can present with edema, proximal myopathy, and hypokalemic alkalosis. Small cell cancer generally presents with hilar/mediastinal adenopathy. It may occasionally present with a solitary lung mass. The chest and the hilum are the first sites of recurrence. In addition to ectopic corticotropin production, small cell carcinoma is also associated with the paraneoplastic syndromes causing the syndrome of inappropriate ADH secretion and Lambert Eaton syndrome. However, symptoms of these syndromes are not present in this patient. Go to the next page if you knew the correct answer, or click the link image(s) below to further research the concepts in this question (if desired).
Research Concepts: Small Cell Lung Cancer
We update eBooks quarterly and Apps daily based on user feedback. Please tap flag to report any questions that need improvement.
Question 367:
A 50-year-old man with a history of follicular lymphoma on surveillance for the past 4 years presents to the clinic with increasing swelling over the left axillary region associated with fever, night sweats and unintentional weight loss of one-week duration. Repeat imaging reveals a 6 cm left axillary lymph node, which was previously 3 cm in size along with unchanged lymph nodes adjacent to the left pectoralis muscle. On examination, his temperature is 38 C, heart rate 95/ min, blood pressure 130/75 mmHg, and SpO2 98% on the room air. Physical examination reveals an enlarged left axillary lymph node without any signs of infection. Laboratory data is significant for LDH of 1600 U/L. What is the next best step in the management of this patient?
Choices: 1. R-CHOP (rituximab, cyclophosphamide, doxorubicin, vincristine, and prednisone) x 6 cycles 2. Repeat the biopsy 3. Initiate treatment with bendamustine and rituximab along with radiation 4. Observation and repeat imaging in 2 months
Answer: 2 - Repeat the biopsy Explanations: Suspect transformation, when there is a development of new B symptoms or extranodal disease or the acute rise of LDH or change in the characteristics of a lymph node compared to the other nodes. The diagnosis needs to be confirmed before the treatment can be initiated. Transformation should be suspected and treatment should be based on the type of lymphoma. Observation is not an option, as the patient has new symptoms that need to be worked up and treated appropriately. Go to the next page if you knew the correct answer, or click the link image(s) below to further research the concepts in this question (if desired).
Research Concepts: Diffuse Large B Cell Lymphoma
We update eBooks quarterly and Apps daily based on user feedback. Please tap flag to report any questions that need improvement.
Question 368:
Involvement of the trunk in acrokeratosis paraneoplastica is seen in which of the following stages?
Choices: 1. 2. 3. 4.
Stage Stage Stage Stage
1 2 3 4
Answer: 3 - Stage 3 Explanations: In his original description, Bazex explained three stages of the acrokeratosis paraneoplastica. During the first stage, mainly earlobes, the ear helix, the tip of the nose, distal fingertips, or nails are affected. If the underlying tumor is progressing, erythema can spread to the trunk as well in stage three. In stage two, palms and soles are frequently involved. The skin lesions spread with the involvement of the cheeks, forehead, elbows, tights, and knees. If the underlying tumor is progressing, erythema can spread to the trunk as well in stage three. Go to the next page if you knew the correct answer, or click the link image(s) below to further research the concepts in this question (if desired).
Research Concepts: Acrokeratosis Paraneoplastica
We update eBooks quarterly and Apps daily based on user feedback. Please tap flag to report any questions that need improvement.
Question 369:
A 55-year-old woman presents to the office with an abnormal mammogram. Scintimammography is decided for the next step in management. Which of the following associated factors is most likely to result in abnormal positive results with the correct radiotracer of choice?
Choices: 1. 2. 3. 4.
Equivocal initial breast imaging findings Breast implants with a suspicion of breast neoplasm Family history of breast cancer Lactating patient with a suspicion of breast neoplasm
Answer: 4 - Lactating patient with a suspicion of breast neoplasm
Explanations: The most common reason for obtaining scintimammography is for increasing positive predictive values of a breast cancer diagnosis with imaging after equivocal initial breast imaging. Monitoring neoadjuvant chemotherapy with scintimammography has been a promising use for scintimammography in recently published literature. There are several contraindications for scintimammography, with one of them being lactating or pregnant patients. Scintimammography uses 99mTc-sestamibi or other less common radiopharmaceuticals to detect areas of increased uptake in the breast, which could be suggestive of breast cancer in patients who have not recently undergone breast procedures. Go to the next page if you knew the correct answer, or click the link image(s) below to further research the concepts in this question (if desired).
Research Concepts: Mammoscintigraphy
We update eBooks quarterly and Apps daily based on user feedback. Please tap flag to report any questions that
need improvement.
Question 370:
A 45-year-old woman with small cell carcinoma lung with multiple intracranial metastases and biochemical features of hyponatremia is planned for stereotactic brain radiosurgery. Which of the following is the central criticism against the use of stereotactic radiosurgery without WBRT in the setting of multiple brain metastasis?
Choices: 1. 2. 3. 4.
Neurocognitive decline Radiation-induced necrosis Radiation-induced leukoencephalopathy Recurrence due to micrometastatic disease
Answer: 4 - Recurrence due to micrometastatic disease Explanations: The inability to treat multiple tumors, which might require the use of salvage stereotactic radiosurgery or targeted or intra-CSF treatment at a later date, is the central criticism that has been leveled against the use of stereotactic radiosurgery, without concomitant wholebrain radiotherapy. A delay of 6 months in the development of brain metastases has been shown in patients who have received whole-brain radiotherapy. This might prove to be particularly deleterious in individuals with a life expectancy of close to one year. The rates of leptomeningeal dissemination in those receiving stereotactic radiosurgery in addition to wholebrain radiotherapy and a comparator group receiving Stereotactic radiosurgery were found to be similar at the end of 12 months in patients with multiple tumors. The reduction in cognitive abilities has been pointed out to be a challenge that has necessitated the introduction of hippocampus avoidance strategies and an approach supporting the combination of HA-WBRT with memantine. Go to the next page if you knew the correct answer, or click the link image(s) below to further research the concepts in this question (if desired).
Research Concepts: Palliative Radiation Therapy For Brain Metastases
We update eBooks quarterly and Apps daily based on user feedback. Please tap flag to report any questions that need improvement.
Question 371:
A 16-year-old male presents to the clinic with increasing pain in his upper left arm for four months. On examination, there is a small mass protruding from his left shoulder. MRI demonstrates a fluid-filled lytic lesion with osteoid matrix mineralization in the left proximal humerus. The histopathology report reveals the presence of vascular sinusoids surrounded by pleomorphic cells. The diagnosis is made, and the treatment is started. Which of the following is considered a good response to treatment?
Choices: 1. 2. 3. 4.
Necrosis Necrosis Necrosis Necrosis
in in in in
more more more more
than than than than
65% 75% 85% 95%
of of of of
the the the the
tumor tumor tumor tumor
cells cells cells cells
Answer: 4 - Necrosis in more than 95% of the tumor cells
Explanations: Telangiectatic osteosarcoma is a rare high-grade malignant neoplasm, accounting for less than 4% of all osteosarcoma cases. It is characterized by distinctive radiographic, gross, and microscopic features. It is a bone-forming tumor that mimics radiologically and microscopically aneurysmal bone cysts. In telangiectatic osteosarcoma, the resected specimen should be mapped to assess the amount of viable tumor after chemotherapy. Subsequent management depends on this assessment. Tumor necrosis of more than 95% of the specimen is considered a good response to treatment. Additionally, uninvolved neurovascular bundles and soft tissue and bone resection margins are essential for a favorable prognosis. Go to the next page if you knew the correct answer, or click the link image(s) below to further research the concepts in this question (if desired).
Research Concepts: Telangiectatic Osteosarcoma
We update eBooks quarterly and Apps daily based on user feedback. Please tap flag to report any questions that
need improvement.
Question 372:
A 45-year-old female presented to the clinic due to progressive difficulty in walking. The symptoms first began one month ago. The patient mentions she tends to fall towards her right side. She had a mild febrile illness 2 weeks ago. Her past medical history is unremarkable. Her family history is positive for breast cancer in her mother. On examination, she was noted to have nystagmus, unsteady gait, and poor coordination. Her motor and sensory examinations were normal. A brain MRI of this patient would most likely reveal which of the following findings?
Choices: 1. 2. 3. 4.
Cerebellar atrophy Ischemic Infarct of the posterior cerebral artery territory Parasagittal meningioma No abnormalities
Answer: 4 - No abnormalities Explanations: This patient likely has paraneoplastic cerebellar degeneration. Patients with paraneoplastic cerebellar degeneration often present with an unsteady gait, frequent falls, fine motor deficits, and nystagmus. A brain MRI performed in the early stages of the disease is usually normal without any cerebellar changes. As the disease progresses and patients reach the burnout stage, a brain MRI would show severe cerebellar atrophy. At this stage, the patient is completely disabled. This patient's symptoms have only been going on for 1 to 2 months, and it is more likely that her MRI would be normal. Patients with paraneoplastic cerebellar degeneration may experience flu-like symptoms before the onset of their cerebellar symptoms. MRI of the brain is an important first step in the diagnosis of paraneoplastic cerebellar degeneration. It is essential to rule out vascular insults and spaceoccupying lesions. This patient is young and does not have any risk factors for an infarct. Her symptoms were also insidious in onset and progressive. Meningiomas are slow-growing tumors and are often asymptomatic. A large parasagittal meningioma would cause headaches, visual changes, motor, and sensory deficits. Go to the next page if you knew the correct answer, or click the link image(s) below to further research the concepts in this question (if desired).
Research Concepts:
Paraneoplastic Cerebellar Degeneration
We update eBooks quarterly and Apps daily based on user feedback. Please tap flag to report any questions that need improvement.
Question 373:
A 65-year-old man undergoes prostatespecific antigen (PSA) testing per his request. His PSA level is 8 ng/mL (reference range 4 ng/mL). Digital rectal examination reveals an enlarged prostate gland. Transrectal ultrasound reveals a focal solid appearing 5 mm hypoechoic lesion with minimal vascularity in the peripheral zone. What is the most likely diagnosis?
Choices: 1. 2. 3. 4.
Benign prostatic hyperplasia (BPH) Prostatic abscess Prostate carcinoma Metastatic lesion
Answer: 3 - Prostate carcinoma Explanations: Prostate malignancy is more common in the peripheral zone. Prostatic carcinoma may appear as a hypoechoic lesion in the peripheral zone. Raised PSA is usually associated with prostate malignancy. BPH appears as an enlargement of the gland. The prostatic abscess is hypoechoic, avascular collection, and usually associated with urinary tract infection. Metastatic cancer is less likely, especially in the context of elevated PSA. Go to the next page if you knew the correct answer, or click the link image(s) below to further research the concepts in this question (if desired).
Research Concepts: Prostate Imaging
We update eBooks quarterly and Apps daily based on user feedback. Please tap flag to report any questions that need improvement.
Question 374:
A 43-year-old man presents to the clinic for follow-up. He was recently diagnosed with T1, N0 rightsided breast ductal carcinoma. He has a BMI of 35. He is otherwise healthy and takes no medications. What is the most appropriate management strategy for this patient?
Choices: 1. Anastrozole only 2. Simple mastectomy with sentinel node biopsy 3. Modified radical mastectomy with removal of the pectoralis muscle 4. Modified radical mastectomy with anastrozole
Answer: 2 - Simple mastectomy with sentinel node biopsy
Explanations: For T1-T2, N0-N1 disease, mastectomies that do not involve the additional removal of the pectoralis muscle have been found to be the preferred choice of resection. Treatment often follows the same rationale of breast cancer in females, so for this patient with early-stage disease, simple mastectomies as retrospective studies have not shown a mortality or reoccurrence benefit with radical approaches of resections. Lymph node sampling should be done. Due to the relatively small volume of breast tissue in men, breast-conserving approaches are rarely entertained. Tamoxifen is the common choice of treatment, although aromatase inhibitors have been identified as a potential therapy; however, its role as isolated adjuvant therapy is limited. Go to the next page if you knew the correct answer, or click the link image(s) below to further research the concepts in this question (if desired).
Research Concepts: Male Breast Cancer
We update eBooks quarterly and Apps daily based on user feedback. Please tap flag to report any questions that need improvement.
Question 375:
A 65-year-old female presents to the emergency department with complaints of nausea, vomiting, and anorexia. She has a history of breast carcinoma and was given chemotherapy with paclitaxel and doxorubicin about one week back. Her laboratory investigations one week back were all normal. Now her hemoglobin is 10.5 mg/dl, the white cell count is 3500 cells/MCL, serum uric acid is 14 mg/dl, serum potassium 6.5 mEq/L, phosphate 12 mg/dl, creatinine 2.6 mg/dl and blood urea nitrogen is 86 mg/dl. What is the most appropriate step in her management?
Choices: 1. 2. 3. 4.
IV hydration IV antibiotics Repeat chemotherapy Blood transfusion
Answer: 1 - IV hydration Explanations: Tumor lysis syndrome (TLS) is characterized by hyperuricemia, hyperkalemia, hyperphosphatemia, and hypocalcemia. It can be a complication while treating malignancies. Clinicians should be aware of the clinical signs and laboratory parameters of TLS. It is an oncologic emergency that is caused by massive tumor cell lysis. Management consists of aggressive IV hydration prophylactically and the use of hypouricemic agents such as allopurinol and rasburicase. Hypocalcemia, not hypercalcemia is a feature of TLS. If a patient progresses to TLS despite prophylactic measures, then treatment consists of correcting the underlying electrolyte abnormalities and management of the patient's symptoms. Go to the next page if you knew the correct answer, or click the link image(s) below to further research the concepts in this question (if desired).
Research Concepts: Tumor Lysis Syndrome
We update eBooks quarterly and Apps daily based on user feedback. Please tap flag to report any questions that need improvement.
Question 376:
A 62-year-old man presents for a dental check-up. He has a past medical history of type 2 diabetes mellitus and a 40 pack-year smoking history. On hard tissue examination, there are no caries. On soft tissue examination, a 0.8 mm ulcer with elevated and indurated borders is found in the lateral aspect of his tongue. There are no traumatic factors. A brush biopsy is positive. Which of the following is the most common carcinoma of this mucosa?
Choices: 1. 2. 3. 4.
Polymorphous low-grade adenocarcinoma Mucoepidermoid carcinoma Squamous cell carcinoma Verrucous carcinoma
Answer: 3 - Squamous cell carcinoma Explanations: Squamous cell carcinoma is the most common type of oral mucosal cancer. Non-squamous cell cancers of the oral cavity are rare. Oral malignancies are asymptomatic initially, highlighting the need for oral screening. Dental professionals should carefully examine the oral mucosa and oropharynx during routine care and may do a brush biopsy if an abnormal lesion is seen. Oral malignant lesions can appear as areas of erythroplakia or leukoplakia and may be exophytic or ulcerated. Cancers are usually indurated and firm with elevated borders. Symptoms may include pain, dysarthria, and dysphagia. Go to the next page if you knew the correct answer, or click the link image(s) below to further research the concepts in this question (if desired).
Research Concepts: Oral Mucosa Cancer
We update eBooks quarterly and Apps daily based on user feedback. Please tap flag to report any questions that need improvement.
Question 377:
A 65-year-old man presented to the healthcare provider with an eight-month history of progressively increasing abdominal pain and 20 pounds (9 kg) of unintentional weight loss. CT-abdomen revealed a right-sided 16 cm kidney tumor, with retroperitoneal lymphadenopathy, which is suspicious for angiomyolipoma. Image-guided core biopsy was done, which showed a malignant tumor composed of fascicles of smooth muscles and staining positive for myogenin. What is the best treatment option for this patient?
Choices: 1. 2. 3. 4.
Neoadjuvant chemotherapy Neoadjuvant radiotherapy Radical nephrectomy Radical nephrectomy with neoadjuvant treatment
Answer: 3 - Radical nephrectomy Explanations: The currently recommended treatment is complete surgical excision of the tumor with negative margins. Radical nephrectomy is considered the treatment of choice as renal leiomyosarcoma has a high risk of local recurrence. Demir et al. described nephron-sparing surgery with no sign of recurrence, but the tumor was low grade with no change in size for three years. There is no consensus on the role of neoadjuvant or adjuvant radio/chemotherapy in the management of renal leiomyosarcoma. Neoadjuvant radiotherapy is generally administered, but it is not indicated postoperatively in R0 (tumor resection with negative macro and microscopic margins) and R1 tumors (with negative macro but positive microscopic margins). Adjuvant chemotherapy is recommended in cases of R2 resections (tumors with positive macro and microscopic resection margins) as well as in unresectable and metastatic disease. Go to the next page if you knew the correct answer, or click the link image(s) below to further research the concepts in this question (if desired).
Research Concepts: Renal Leiomyosarcoma
We update eBooks quarterly and Apps daily based on user feedback. Please tap flag to report any questions that need improvement.
Question 378:
A 40-year-old man of Chinese ancestry presents with a three-week history of fatigue, persistent nasal congestion, and a sore throat. He also reports three episodes of otitis media in the last four months. He has a 20-pack-year smoking history but no history of alcohol consumption or illicit drug use. Physical examination is negative for a neck mass; however, an in-office flexible fiberoptic nasopharyngolaryngoscopy is performed, and a mass is seen behind the right Eustachian tube. Analysis of a biopsy speicmen confirms the diagnosis. A complete metastatic workup with an MRI of the head and neck, as well as a PET/CT, shows no evidence of nodal or extranodal metastasis. The appropriate treatment is initiated. He presents to the clinic two weeks later with an inability to eat due to severe pain while swallowing, and he has lost 10 lbs (4.5 kg) since the beginning of treatment. His weekly labs reveal blood glucose of 160 mg/dL, WBC count of 6000/µL (60% neutrophils), ESR of 60 mm/h, creatinine level of 1.2 mg/dL, and total bilirubin of 1 mg/dL. What is the most likely cause of this patient's current symptoms?
Choices: 1. 2. 3. 4.
Secondary infection Primary tumor progression Adverse effect of treatment Gastroesophageal reflux disease (GERD)
Answer: 3 - Adverse effect of treatment Explanations: This vignette depicts a man with nasopharyngeal carcinoma, which is treated with radiation and chemotherapy. His symptoms within two weeks of treatment onset are likely due to radiation-induced mucositis. Most patients develop mucositis from radiation, and many foods irritate the mucosa, resulting in pain and dysphagia. A dietitian consult is needed as these patients may require a soft, bland diet. All acidic and spicy foods should be avoided. In some patients, there may be a need for a gastrostomy feeding tube to allow for hydration and adequate caloric intake. A strong association exists with Epstein-Barr virus (EBV) infections, such that EBV titers may be used to follow a patient's response to treatment. The incidence of nasopharyngeal carcinoma has a bimodal distribution in the teen years and between ages 45 and 55. Risk factors for nasopharyngeal carcinoma include environment, ethnicity, and tobacco use. There is an unusually high incidence of nasopharyngeal cancer in southern China, Africa, and Alaska and in Greenland Eskimos. Secondary infection in a patient on chemotherapy is likely in patients with neutropenia. This patient has a normal WBC count with an appropriate number of neutrophils. Extension of primary pathology in an untreated patient is likely, but progression amid treatment is unlikely.
Go to the next page if you knew the correct answer, or click the link image(s) below to further research the concepts in this question (if desired).
Research Concepts: Nasopharyngeal Cancer
We update eBooks quarterly and Apps daily based on user feedback. Please tap flag to report any questions that need improvement.
Question 379:
A 48-year-old male with a history of tuberous sclerosis presents with bilateral flank discomfort for the past six weeks. Other than the discomfort, the patient does not have any other symptoms. His physical examination is within normal limits. A thorough blood workup including a complete blood count, a comprehensive metabolic panel, and a urinalysis were performed which were all within normal limits. Renal ultrasound showed a hyperechoic mass. Computed tomography (CT) scan of the abdomen was ordered, which showed a hypoattenuating area in the kidney. The patient was scheduled for a percutaneous renal biopsy, which confirmed the diagnosis. Which of the following is required for diagnosing the most likely tumor in this patient?
Choices: 1. This requires thin sectioning reading of less than 10 2. This requires thin sectioning reading of less than 0 3. This requires thin sectioning reading of more than -10 4. This requires thin sectioning reading of -10 or less
along with a CT density (HU) along with a CT density (HU) along with a CT density (HU) along with a CT density (HU)
Answer: 4 - This requires thin sectioning along with a CT density (HU) reading of -10 or less
Explanations: A negative Hounsfield unit (HU) less than 0 does not diagnose fat. The statistical spread of density readings of lesions follows a Bell curve. A lesion is called "water density" if it falls between 0 and +10. Fat is diagnosed only if the HU is -10 or less. Thin sections are recommended to get a more accurate density reading on the CT scan to guard against volume averaging artifact. With MRI, macroscopic fat is diagnosed by a bright signal on T1 and loss of signal on fat-saturation sequences. If the CT clearly shows a fat-rich lesion, there would not normally be a need for a percutaneous biopsy for an obvious angiomyolipoma, especially in a tuberous sclerosis patient. Go to the next page if you knew the correct answer, or click the link image(s) below to further research the concepts in this question (if desired).
Research Concepts: Renal Angiomyolipoma
We update eBooks quarterly and Apps daily based on user feedback. Please tap flag to report any questions that need improvement.
Question 380:
A 49-year-old female has undergone physical examination and mammography for a left-sided 2 cm x 3 cm breast mass. The mammography report states her BIRADS (breast imaging reporting and database system) score as 4b. Which of the following is the next most appropriate step in her management?
Choices: 1. No further investigation is necessary as the lesion is likely to be benign 2. Ultrasound scan 3. Core biopsy 4. Excision biopsy of the mass
Answer: 3 - Core biopsy Explanations: Core biopsy is more invasive than fine-needle aspiration cytology and requires more expertise for its analysis. Either of these would be appropriate next steps in this patient. A core biopsy provides information about the architectural structure of tissues and the invasiveness of a tumor. Fine-needle aspirate cytology can only give information about the cells themselves. Tissue or cytological analysis of a likely malignant lesion must be carried out before excision to guide the surgeon in terms of procedure choice and appropriate margins. Imaging reports are standardized using a tool called BIRADS – Breast Imaging Reporting and Data System (fifth edition). This standard allows breast imaging to be described according to a certain structure as follows: density of breast tissue, presence and location of a mass or masses, calcifications, asymmetry, and any associated features. This classification system divides patients into categories 0 to 6, depending on the likelihood of malignancy in the obtained images: BIRADS 0 – insufficient or incomplete study, BIRADS 1 – normal study, BIRADS 2 – benign features, BIRADS 3 – probably benign (2% risk of malignancy), BIRADS 4 – suspicious features (divided into categories 4a, 4b, and 4c depending on the likelihood of malignancy), BIRADS 5 – probably malignant (>95% chance of malignancy), BIRADS 6 – malignant (proven malignant on tissue biopsy) The BIRADS system includes different classifications for masses depending on the imaging modality in question.
In mammography, to be considered a mass, the lesion must be visible in two different projections, must have convex outer borders, and must be denser in the center than on the periphery. Go to the next page if you knew the correct answer, or click the link image(s) below to further research the concepts in this question (if desired).
Research Concepts: New Breast Mass
We update eBooks quarterly and Apps daily based on user feedback. Please tap flag to report any questions that need improvement.
Question 381:
A 54-year-old woman with a history of squamous cell carcinoma of the lung presents the emergency department. She was treated with radiation therapy for her lung cancer. The patient states that it started after her radiation treatment. Her odynophagia has been progressive to the point where it is difficult to tolerate oral intake. During her hospital stay, she was evaluated by a healthcare provider who performed an esophagogastroduodenoscopy (EGD) that noted an erythematous and friable esophageal lining consistent with radiation esophagitis. What medication class is given prophylactically for patients suffering from radiation esophagitis?
Choices: 1. 2. 3. 4.
Antimicrobials Antifungals Antivirals Antiemetics
Answer: 2 - Antifungals Explanations: Prophylactic antifungal therapy is indicated due to the increased risk of thrush. Dietary modification in the form of soft, bland diets is typically used with the removal of irritant foods such as alcohol, spicy foods, and very hot or cold food. Amifostine, an organic triphosphate, has been studied and shown to act as a radioprotector has been shown to improve swallowing dysfunction but had no effect on reducing rates of esophagitis in severe cases. NSAIDs and other non-steroidal have been studied but have not been proven effective. Go to the next page if you knew the correct answer, or click the link image(s) below to further research the concepts in this question (if desired).
Research Concepts: Radiation Esophagitis
We update eBooks quarterly and Apps daily based on user feedback. Please tap flag to report any questions that need improvement.
Question 382:
A 54-year-old woman presents to the office for her postoperative visit following left breast lumpectomy and sentinel lymph node biopsy two weeks ago. The pathology report indicates the final tumor size was 1.4 cm, and the closest margin was the anterior margin at 3 mm. The tumor is found to be both estrogen and progesterone-receptor positive. Which of the following is the most appropriate next step in the management of this patient’s breast cancer?
Choices: 1. Return to the operating room for re-excision lumpectomy 2. Referral to medical oncology for chemotherapy recommendations 3. Referral to medical oncology for hormone therapy recommendations and radiation oncology for radiation recommendations 4. Referral to medical oncology for hormone therapy recommendations only
Answer: 3 - Referral to medical oncology for hormone therapy recommendations and radiation oncology for radiation recommendations
Explanations: The patient has early-stage breast cancer, as indicated by the final pathology. The closest margin was the anterior margin at 3 mm. Current recommendations for invasive breast cancer would suggest that this should be treated as negative margins, as there is "no tumor on ink," and the patient should proceed with adjuvant therapy as indicated. The tumor is estrogen and progesterone-receptor positive and, therefore, should be recommended for hormone therapy. The patient underwent breast-conserving surgery. As recurrence rates are unacceptably high without adjuvant radiation, she should be recommended for adjuvant radiation. Chemotherapy is not necessarily indicated, particularly in early-stage invasive breast cancer, which is estrogen and progesterone-receptor positive. Go to the next page if you knew the correct answer, or click the link image(s) below to further research the concepts in this question (if desired).
Research Concepts: Breast Cancer Surgery
We update eBooks quarterly and Apps daily based on user feedback. Please tap flag to report any questions that need improvement.
Question 383:
A 73-year-old patient male presents with a complaint of a right-sided lesion over zygoma. On examination, the patient has a blood pressure of 125/78mmHg, a heart rate of 82 beats per minute and a respiratory rate of 18 breaths/min. On local examination of the wound, an indurated pearly lesion with local bleeding is observed. What gene is the most commonly mutated in this lesion?
Choices: 1. 2. 3. 4.
BRAF C-kit PTCH Keratin 17
Answer: 3 - PTCH Explanations: The gene most often altered in basal cell carcinomas (BCCs) is the PTCH1 gene. PTCH1 gene mutations occur in 70% of people with sporadic BCC. 10% to 20% of people with sporadic BCC have smoothened (SMO) mutations. Literature suggests that a sufficiently elevated expression level of Gli, by activating mutations of SMO or by homozygous inactivation of PTCH1 in a responding cell is sufficient to drive the formation of BCC. PTCH is the most common mutation seen in basal cell carcinomas. The second most common mutation found in BCCs are in the P53 gene. Go to the next page if you knew the correct answer, or click the link image(s) below to further research the concepts in this question (if desired).
Research Concepts: Basal Cell Carcinoma
We update eBooks quarterly and Apps daily based on user feedback. Please tap flag to report any questions that need improvement.
Question 384:
A 70-year-old man has been watching an asymptomatic dark spot on his chest slowly increase in size over the last 3 years. His wife has brought him to the office for the lesion to be examined. Cutaneous examination shows a black 3 x 3 cm nodule on his right chest. He has no other raised pigmented lesions. Partial biopsy of the lesion showed Dopa and Fontana-Masson silver stains highlighting melanin and melanocytes. Melanocytes were unevenly scattered throughout the entire epidermis. What is the preferred next management?
Choices: 1. No additional treatment is necessary. 2. The residual tumor should be treated with radiation therapy. 3. A conservative compete excision of the residual tumor should be performed. 4. The patient should receive systemic chemotherapy with vemurafenib.
Answer: 3 - A conservative compete excision of the residual tumor should be performed.
Explanations: Conservative complete excision of a cutaneous melanoacanthoma is the treatment of choice. Therefore, a cutaneous melanoacanthoma that has been completely removed during the biopsy procedure does not require any additional treatment. Complete removal of a cutaneous melanoacanthoma often occurs during biopsy of the lesion. However, incomplete removal of a cutaneous melanoacanthoma can result in persistence and continued growth or recurrence of the lesion. Systemic antineoplastic agents have not been used in the management of cutaneous melanoacanthoma. However, topical 5-fluorouracil 5% cream was used on some of the cutaneous melanoacanthomas in a woman who had multiple tumors; it caused a burning sensation at the application sites without any significant improvement and was discontinued after 4 weeks of treatment. Vemurafenib is an oral agent approved for the treatment of BRAF V600E mutation-positive metastatic malignant melanoma. It binds selectively to and inhibits the activity of the ATP-binding site of BRAF V600E kinase. In melanomas that express BRAF V600E, tumor cell proliferation is reduced. Cutaneous melanoacanthoma is a benign skin tumor. Radiation therapy is a treatment modality that can be used for patients with non-operative cutaneous malignancies such as basal cell carcinoma and squamous cell carcinoma. It can also be used as adjuvant treatment in the postoperative setting of non-
melanoma skin cancers, which demonstrate perineural invasion. Radiotherapy has not been used in the management of benign cutaneous melanoacanthoma. Go to the next page if you knew the correct answer, or click the link image(s) below to further research the concepts in this question (if desired).
Research Concepts: Cutaneous Melanoacanthoma
We update eBooks quarterly and Apps daily based on user feedback. Please tap flag to report any questions that need improvement.
Question 385:
A 24-year-old woman with a history of stage III breast cancer is scheduled for the induction phase of chemotherapy with docetaxel. She is taking an oral contraceptive for pregnancy prevention. She will be administered fosaprepitant IV, dexamethasone IV, and ondansetron IV before infusion for chemotherapy-induced nausea and vomiting prophylaxis (CINV). What is the most appropriate post-infusion advice for this patient?
Choices: 1. Avoid food for the next 12 hours 2. Get renal function tests done 24 hours later 3. Employ a backup means of contraception if the patient engages in sexual activity 4. Not operate a motorized vehicle for 24 hours
Answer: 3 - Employ a backup means of contraception if the patient engages in sexual activity
Explanations: Fosaprepitant can decrease the effectiveness of oral contraceptive medications. The patient should be made aware that they need to use a backup means of contraception. Fosaprepitant exerts its antiemetic effects for up to 48 hours. In contrast to promethazine or droperidol, the combination of dexamethasone, ondansetron, and fosaprepitant leads to little or no effect on alertness. The combination of dexamethasone, ondansetron, and fosaprepitant requires no lab monitoring. Go to the next page if you knew the correct answer, or click the link image(s) below to further research the concepts in this question (if desired).
Research Concepts: Aprepitant
We update eBooks quarterly and Apps daily based on user feedback. Please tap flag to report any questions that need improvement.
Question 386:
A 56-year-old male patient presents with shortness of breath for the last 3 months. The radiological measures are remarkable for a solid mass, and further tissue diagnosis shows a 3-cm differentiated adenocarcinoma in the left upper lobe. A head computed tomography (CT) scan reveals a 2-cm mass in the parietal lobe. Additional workup for metastatic disease is negative. What is the management of this patient?
Choices: 1. 2. 3. 4.
Simultaneous radiation to the lung and cranial lesion Craniotomy followed by a lobectomy Lobectomy followed by a craniotomy Chemotherapy and radiation therapy
Answer: 2 - Craniotomy followed by a lobectomy Explanations: This is one scenario where the metastatic disease is resected. Trials have shown that patients with adenocarcinoma and a single brain metastatic lesion can undergo craniotomy followed by a lobectomy. When a brain lesion is detected first, and a search for a primary lesion is negative, resection of the cranial lesion is the therapy of choice. When a brain lesion presents subsequent to a lung resection for cancer and there is no recurrence, resection of the cranial lesion is performed. When the brain and lung lesions are detected simultaneously and both are resectable, a craniotomy is performed first followed by a thoracotomy. If either the lung or cranial lesion is suspected to be unresectable, surgical therapy is directed first to the resectable lesion. Otherwise, therapy by non-surgical means is recommended. This has prolonged survival. Go to the next page if you knew the correct answer, or click the link image(s) below to further research the concepts in this question (if desired).
Research Concepts: Adenocarcinoma
We update eBooks quarterly and Apps daily based on user feedback. Please tap flag to report any questions that need improvement.
Question 387:
A 45-year-old woman presents to the emergency department for an episode of confusion, nervousness, sweating, and palpitations that occurred one hour ago at home while she was preparing dinner. She has had five similar episodes over the past three weeks. According to the patient, each episode lasts three to five minutes and is relieved by ingesting solid food or fruit juice. Her blood pressure is 141/84 mmHg, her pulse is 86 beats/minute, and her respiratory rate is 18 breaths/minute. The examination is within normal limits. Blood work shows a random blood glucose level of 131 mg/dl, HbA1c of 5.6%, and a TSH level of 1 mIU/L. While in the examination room, the patient suddenly becomes tremulous and complains of palpitations and blurred vision. A cardiac monitor is urgently attached to the patient, revealing a pulse of 123 beats/minute. Blood glucose is measured via the finger-stick and indicates hypoglycemia. The patient is quickly given a bolus of 25% dextrose orally, which rapidly improves her symptoms. The patient is admitted to the hospital and is further tested with a 72-hour fasting test, which is stopped after 36 hours because of another episode of sweating and palpitations that lasts 10 minutes. A plasma glucose level of 55 mg/dl is recorded during the episode. What is the best next step in managing this patient's suspected condition?
Choices: 1. Measure blood levels of insulin, proinsulin, and C-peptide during symptomatic episodes 2. Measure plasma fractionated metanephrines levels when the patient is stable 3. MRI of the pituitary gland 4. CT scan of the pancreas
Answer: 1 - Measure blood levels of insulin, proinsulin, and C-peptide during symptomatic episodes
Explanations: Whipple triad consists of low blood glucose, symptoms of hypoglycemia (e.g., lethargy, syncope, diplopia), and resolution of symptoms after normalizing plasma glucose levels. It is seen in patients with insulinoma. If hypoglycemia is confirmed by measuring blood glucose levels when the patient is symptomatic, it should be followed by a 72-hour fasting test. If hypoglycemia is established during the 72-hour fasting test, blood levels of insulin, proinsulin, and C-peptide are measured to distinguish insulinoma from factitious disorder. In patients with insulinoma, levels of insulin, proinsulin, and C-peptide will be high. Surgical excision of the tumor is recommended for initial therapy of patients with benign, solitary insulinoma. Go to the next page if you knew the correct answer, or click the link image(s) below to further research the concepts in this question (if desired).
Research Concepts: Insulinoma
We update eBooks quarterly and Apps daily based on user feedback. Please tap flag to report any questions that
need improvement.
Question 388:
An 8-year-old female is brought to the clinic by her mother with complaints of double vision, unsteady gait, weakness, and problems swallowing for 3 weeks. An MRI of the brain is ordered, and it shows a nonenhancing, low-intensity lesion at the pons. What is the next best and ideal step in the management of this patient at an experienced center?
Choices: 1. 2. 3. 4.
Stereotactic biopsy Chemotherapy Radiotherapy Shunt placement
Answer: 1 - Stereotactic biopsy Explanations: The patient most likely has a diffuse intrinsic pontine glioma. This is usually high grade and has a very poor prognosis. The current management recommendations are tissue diagnosis with a stereotactic biopsy at an experienced center, followed by conventional radiotherapy at doses from 54-60 Gy. The treatment efficacy in children has not advanced in the past several years. Go to the next page if you knew the correct answer, or click the link image(s) below to further research the concepts in this question (if desired).
Research Concepts: Diffuse Intrinsic Pontine Glioma
We update eBooks quarterly and Apps daily based on user feedback. Please tap flag to report any questions that need improvement.
Question 389:
A 59-year-old male with a history of stage 3 relapsed refractory diffuse large B-cell lymphoma arising from follicular lymphoma was scheduled for chimeric antigen receptor (CAR) T-cell therapy. He was admitted to the hospital for conditioning with fludarabine and cyclophosphamide and subsequent infusion of CAR T-cells. No infusion related reactions were observed. On day 2, the patient became febrile and hypoxic with an oxygen saturation of 88%, which improved with supplemental oxygen. Infection workup was performed, including respiratory viral panel and urine analysis. Blood cultures were taken, and the patient was started empirically on vancomycin and cefepime. The chest imaging was clear. On day 3, the patient's temperature was 101.4 F, heart rate of 110/min, blood pressure of 74/39 mmHg, and oxygen saturation of 89% on 1 L/min oxygen via nasal cannula. Liver and kidney function tests were normal. He was given 2 L of normal saline with improvement in blood pressure. What is the next best step in the management of this patient?
Choices: 1. 2. 3. 4.
Dexamethasone Methylprednisolone Tocilizumab and dexamethasone Tocilizumab
Answer: 4 - Tocilizumab Explanations: Cytokine release syndrome (CRS) is the most common toxicity in patients receiving CAR T-cells. Treatment should be started immediately. In this clinical vignette, the patient has fever, hypoxia, and hypotension. Fluids improved the hypotension, and he did not require a vasopressor. Fever with hypotension not requiring vasopressor is classified as grade 2 CRS, and tocilizumab 8 mg/kg IV over 1 hour is indicated. If there is no improvement with the first dose of tocilizumab, treatment should be repeated in 8 hours with no more than 3 doses in 24 hours. Dexamethasone should be considered if symptoms are refractory to tocilizumab. Other causes of CRS should be ruled out, and empiric antibiotic therapy is needed for neutropenic patients. Go to the next page if you knew the correct answer, or click the link image(s) below to further research the concepts in this question (if desired).
Research Concepts: Chimeric Antigen Receptor T-Cell Therapy
We update eBooks quarterly and Apps daily based on user feedback. Please tap flag to report any questions that need improvement.
Question 390:
A 57-year-old man presents to the office for evaluation. He had an abnormal ultrasound two weeks ago, which showed a single 1.8 cm fluid-filled mass has irregular edges and internal septations within the left hepatic lobe. The patient reports a long history of dull right upper quadrant pain and early satiety associated with nausea and he tried antacids with no benefits. For the past 4 months, he noticed a 12-pound (5 kg) weight loss unintentionally. On physical exam, the patient appears pale. He explains a little discomfort on palpation on the right upper abdomen. Murphy's sign is negative, and there is no organomegaly on deep palpation. Which of the following is the next best step in the management of this patient?
Choices: 1. 2. 3. 4.
Enucleation Surgical resection CT scan every 6-9 months Sirolimus therapy
Answer: 2 - Surgical resection Explanations: The patient most likely has an invasive carcinoma, probably a malignant transformation of biliary cystadenoma into biliary cystadenocarcinoma. The treatment should consist of surgical resection. Neoplastic cysts are typically intrahepatic, solitary, slowgrowing, multiloculated cystic tumors filled with a clear mucinous fluid. Biliary cystadenoma usually does not communicate with the biliary system. These lesions demonstrate mild capsular and septal enhancement on postcontrast imaging at CT and MRI. The existence of internal debris, bile duct dilation, and enhancing mural nodularity is concerning for biliary cystadenocarcinoma. Biliary cystadenoma (BCA) and biliary cystadenocarcinoma (BCAC) have a similar clinical presentation. The differentiation between them can be made through certain clinical variables in terms of demographic criteria, liver enzymes, tumor size, and the presence of mural nodules. Higher enzyme levels in BCAC occur due to the invasion of the adjacent biliary tree, while BCA is a slow-growing tumor and causes late biliary compression. Enucleation is usually not performed since it may be associated with an increased risk of recurrence. Sirolimus has been shown to stop the disordered, unregulated proliferative response of polycystic cells in patients with polycystic kidney disease. Go to the next page if you knew the correct answer, or click the link image(s) below to further research the concepts in this question (if desired).
Research Concepts: Liver Cystic Disease
We update eBooks quarterly and Apps daily based on user feedback. Please tap flag to report any questions that need improvement.
Question 391:
A 70-year-old woman presents to the emergency department (ED) complaining of lightheadedness, drowsiness, and blurry vision. In the past two weeks, she had episodes of fainting at home. In the ED, her vital signs show blood pressure 125/73 mmHg, heart rate 70/min, respiratory rate 18/min, and SpO2 97% on room air. The physical exam is benign, except for mild splenomegaly. A CT head without contrast does not show any relevant pathology. In her blood work, hemoglobin is 10 g/dL, white blood cell count 7500/microL, and platelets 155000/microL. The peripheral blood smear shows rouleaux formation. Further evaluation reveals elevated globulins with an IgM spike. What is the best initial step in the management of her condition?
Choices: 1. A bolus of IV fluids 2. Chemotherapy (dexamethasone, fludarabine, bortezomib, and cyclophosphamide) 3. Aspirin 4. Plasmapheresis
Answer: 4 - Plasmapheresis Explanations: This patient came with hyperviscosity syndrome that is related to Waldenstrom macroglobulinemia (WM). The most common symptoms associated with hyperviscosity include recurrent spontaneous nosebleeds, headaches, blurred vision, tinnitus, vertigo, and slow mentation. Plasmapheresis should be initiated promptly in patients with WM with signs and symptoms of hyperviscosity. Therapeutic plasmapheresis (plasma exchange) is a procedure involving the separation of plasma from circulating blood cells to remove a disease substance, in this case, remove IgM protein. Plasmapheresis, however, should be used only temporarily for symptoms of hyperviscosity, until definitive therapy for WM is instituted. Go to the next page if you knew the correct answer, or click the link image(s) below to further research the concepts in this question (if desired).
Research Concepts: Lymphoplasmacytic Lymphoma
We update eBooks quarterly and Apps daily based on user feedback. Please tap flag to report any questions that need improvement.
Question 392:
A 55-year-old male previously diagnosed with metastatic lung cancer is admitted with uncontrolled pain. The workup of this patient reveals a tumor invading his rib. He has been taking hydrocodone/acetaminophen at home before coming to the hospital. Since admission, he has been prescribed IV hydromorphone 1 mg every 2 hours as needed. He has needed eight doses/24 hours. His pain is now better controlled. What was his opioid requirement in oral morphine equivalents (OME) in the last 24 hours?
Choices: 1. 2. 3. 4.
100 128 160 480
mg mg mg mg
Answer: 3 - 160 mg Explanations: Hydromorphone is one of the most potent opioids, especially when administered intravenously. The equianalgesic ratio for IV hydromorphone and oral morphine is 1:20. This patient required eight doses of IV hydromorphone 1 mg in the last 24 hours; therefore, 1 mg x 8 doses x 20 = 160 mg. It is important to add up all opioid doses administered during the previous day to get an accurate assessment of opioid use. Among opioids, do not favor small numbers, rather be aware of the potency of opioids relative to morphine. Go to the next page if you knew the correct answer, or click the link image(s) below to further research the concepts in this question (if desired).
Research Concepts: Opioid Equivalency
We update eBooks quarterly and Apps daily based on user feedback. Please tap flag to report any questions that need improvement.
Question 393:
A 43-year-old female is admitted to the hospital for uncontrolled diabetes. Her blood work show fasting blood sugar of 253 mg/dl ( 65-99), HbA1c of 11.3% ( 4.8- 56). Other pertinent history is significant for depression, which is being treated by her primary care physician. Her other complaints include painful rash over her face, buttocks, and extremities for which she has been using over-the-counter creams. She does complain of diarrhea over the past six months and has lost 30 pounds. Her physical examination reveals blood pressure 142/ 90 mm hg, pulse rate 102 beats per minute. On physical exam, she is anxious.You note an erythematous papular rash around her face and extremities. A biopsy of this lesion is consistent with " necrolytic migratory erythema." Which of the following test should be ordered to confirm the clinical diagnosis?
Choices: 1. 2. 3. 4.
Serum glucagon level Vitamin B12 level Serum cortisol level Serum lipase level
Answer: 1 - Serum glucagon level Explanations: Her clinical presentation is consistent with glucagonoma syndrome, which usually presents with necrolytic migratory erythema in 90% of the patients. A fasting serum glucagon level should be drawn to confirm the diagnosis. Diabetes mellitus is present in about 80% of patients with glucagonoma syndrome. Depression can be seen in about 50% of patients with glucagonoma syndrome. Other conditions with necrolytic migratory erythema can include chronic liver disease, inflammatory bowel disease, pancreatitis, malignancy, etc. Go to the next page if you knew the correct answer, or click the link image(s) below to further research the concepts in this question (if desired).
Research Concepts: Glucagonoma Syndrome
We update eBooks quarterly and Apps daily based on user feedback. Please tap flag to report any questions that need improvement.
Question 394:
A 65-year-old woman comes to the physician because of a 9-mm papule that has changed considerably in size and color over the past couple of months. There is no personal or family history of skin cancer. Physical examination reveals a flat, heterogeneous, brown-to-black lesion with irregular borders on the dorsum of the hand. Which of the following would be the most appropriate next step in management for this patient?
Choices: 1. 2. 3. 4.
Observe the lesion over the next 3 months Perform a full-body examination during this visit Computed tomography (CT) of the chest Obtain a CA-125 level
Answer: 2 - Perform a full-body examination during this visit
Explanations: The description of an evolving pigmented lesion with color variegation and irregular borders is concerning for melanoma. When melanoma is suspected, a full body examination of the skin is crucial during the first encounter. Features that are concerning for melanoma are remembered with mnemonic ABCDE: Asymmetry, Border irregularity, Color that is not uniform, Diameter > 6mm, and Evolution. Skin lesions that are clinically concerning for melanoma should be biopsied or excised. For small lesions, complex excision is preferred over a biopsy. The tissue must be oriented and sent to the pathologist to evaluate for clear margins. Go to the next page if you knew the correct answer, or click the link image(s) below to further research the concepts in this question (if desired).
Research Concepts: Skin Cancer
We update eBooks quarterly and Apps daily based on user feedback. Please tap flag to report any questions that need improvement.
Question 395:
A 61-year old female with T3N0M0 squamous cell carcinoma of the anal canal completed definitive chemoradiation 6 weeks ago. She has a persistent ulcerated lesion at the location of the original bulky primary tumor. What should be recommended?
Choices: 1. 2. 3. 4.
Biopsy the lesion Abdominoperineal resection Nivolumab Follow up in 4 weeks
Answer: 4 - Follow up in 4 weeks Explanations: Following definitive chemoradiation for anal squamous cell carcinoma, the median time for tumor regression is 3 months, but tumors can continue to regress for up to 26 weeks based on the ACT-II study. Nivolumab is reserved for second-line immunotherapy for metastatic disease. Patients that have not experienced a complete clinical response following definitive chemoradiation may be followed up every 4 weeks up to 6 months. As long as there is no disease progression, surveillance is the most appropriate option. Anal biopsy after definitive chemoradiation can have high morbidity to include fistula formation and should be reserved only for progressive lesions. Go to the next page if you knew the correct answer, or click the link image(s) below to further research the concepts in this question (if desired).
Research Concepts: Radiation Therapy For Anal Cancer
We update eBooks quarterly and Apps daily based on user feedback. Please tap flag to report any questions that need improvement.
Question 396:
A 71-year-old man comes to the office for evaluation of a pigmented lesion on his right shoulder blade. His wife first noticed the lesion two months ago and said it occasionally itches and makes his uncomfortable going to sleep. The patient has numerous freckles on his shoulder and arms. Medical history includes hypertension and diabetes mellitus type 2. He has never smoked or drunk alcohol. On examination, there is a 6-mm, dark brown lesion at the dorsal surface of his right shoulder blade. It has a smooth border and a small, eccentric nodule. Also, multiple scattered, flat, light brown lesions are present on the upper and lower back. Which of the following is the most appropriate next step in the management of this patient's back lesion?
Choices: 1. Excisional biopsy of the main lesion 2. Excisional biopsy with initial margins of 3 mm of normal tissue 3. Shave biopsy of the main lesion 4. Shave biopsy with initial margins of 3 mm of normal tissue
Answer: 2 - Excisional biopsy with initial margins of 3 mm of normal tissue
Explanations: A mole may represent melanoma if it appears substantially different from others ("ugly duckling sign"), itches or bleeds, or develops new nodularity. If melanoma is suspected, an excisional biopsy with initial margins of 1-3 mm of normal tissue should be obtained. Melanomas may develop in or near a previously existing precursor lesion or in healthy-appearing skin. A malignant melanoma developing in healthy skin is said to arise de novo, without evidence of a precursor lesion. Solar irradiation induces many of these melanomas. Melanoma also may occur in unexposed areas of the skin, including the palms, soles, and perineum. Perform excisional biopsy on suggestive lesions so that a pathologist can confirm the diagnosis. Shave biopsies and electrodesiccation are inadequate; a full thickness of the skin is essential for proper histologic diagnosis and classification. The most important prognostic indicator for stage I and II tumors is thickness; obtain a full-thickness biopsy specimen for adequate pathologic interpretation. Biopsy results ultimately determine the margins of resection and which patients are candidates for sentinel lymph node biopsy and other adjuvant treatment. Surgery such as wide local excision with sentinel lymph node biopsy, elective node dissection, or both is the definitive treatment for early-stage melanoma. When performing the wide local excision, first consider the surgical margins. If the primary closure is not feasible, skin grafting or tissue transfers may be needed. Medical
management is reserved for adjuvant therapy of patients with advanced melanoma. Go to the next page if you knew the correct answer, or click the link image(s) below to further research the concepts in this question (if desired).
Research Concepts: Malignant Melanoma
We update eBooks quarterly and Apps daily based on user feedback. Please tap flag to report any questions that need improvement.
Question 397: A 62-year-old man with a history of hypertension, gout, depression, and multiple myeloma presents to the clinic for follow up. His medications include lisinopril, acetaminophen, sertraline, and ixaomib is included. The patient had liver function tests checked two weeks ago. Which of the following additional labs is most appropriate to be ordered for this patient?
Choices: 1. 2. 3. 4.
Blood cholesterol Platelet count Hba1c C-reactive protein (CRP)
Answer: 2 - Platelet count Explanations: Both platelets and neutrophils should be monitored at least monthly due to the risk of thrombocytopenia and neutropenia. Common adverse effects of Ixazomib include diarrhea, constipation, and thrombocytopenia. Ixazomib has also been seen to cause peripheral neuropathy in some patients. Ixazomib is the first oral proteasome inhibitor used in the treatment of multiple myeloma. Go to the next page if you knew the correct answer, or click the link image(s) below to further research the concepts in this question (if desired).
Research Concepts: Ixazomib
We update eBooks quarterly and Apps daily based on user feedback. Please tap flag to report any questions that need improvement.
Question 398:
A 17-year old male patient has undergone an open appendectomy. The surgery was uneventful and the patient is discharged the next morning. Seven days later the pathologist reports that there was a carcinoid measuring 1 cm confined to the mucosal tip. What is the next step?
Choices: 1. 2. 3. 4.
Post-operative radiation Right hemicolectomy Observation Offer patient streptozotocin
Answer: 3 - Observation Explanations: If the carcinoid is only located at the tip of the appendix, some experts recommend only an appendectomy. If the carcinoid is found at the base of the appendix, experts recommend a right hemicolectomy. If the tumor is more than 2 cm and located anywhere but the tip, always consider a more aggressive procedure. Treat it like a colonic malignancy. When there is invasion of the mesoappendix this does not alter the long term prognosis but if there is cecal involvement, then extended surgery or a right hemicolectomy is needed. Go to the next page if you knew the correct answer, or click the link image(s) below to further research the concepts in this question (if desired).
Research Concepts: Intestinal Carcinoid Cancer
We update eBooks quarterly and Apps daily based on user feedback. Please tap flag to report any questions that need improvement.
Question 399:
A 37-year-old woman with a past medical history of migraines and polycystic ovarian syndrome is evaluated for a change in her usual headaches. The headaches are occurring more frequently and respond poorly to her previously effective medications. She is otherwise healthy and exercises three times per week. She wears contact lenses. She discontinued oral contraceptive pills eight years ago due to intolerance. She currently only uses ibuprofen and sumatriptan. On physical examination, her vital signs are normal. Her body mass index is 26 kg/mm2. The rest of her physical examination, including funduscopic examination, is normal. An initial CT brain was nondiagnostic. MRI with gadolinium reveals a 0.7 cm pituitary adenoma with a suprasellar extension but no compression or encroachment of the optic chiasm or optic nerves. There are no other abnormalities. Her chemistry panel is within normal limits. A urine pregnancy test is negative. Laboratory studies show: cortisol (8am) - 17 mcg/dL (normal: 5 - 25 mcg/dL), thyroid-stimulating hormone (TSH) - 3.6 mcU/mL (Normal: 0.5 - 5.0 mcU/mL), and thyroxine (T4) - 1.5 ng/dL (Normal: 0.9 - 2.4 ng/dL). Which of the following would be the best next course of action?
Choices: 1. Visual field testing 2. Referral to neurosurgery 3. Measurements of prolactin and insulin-like growth factor 1 (IGF-1) 4. Abdominal CT scan to check for pancreatic masses
Answer: 3 - Measurements of prolactin and insulin-like growth factor 1 (IGF-1)
Explanations: The establishment of a microadenoma versus a macroadenoma occurs with the size seen on an MRI, and the workup is dictated by this classification. All microadenomas require the complete pituitary hormonal panel to rule out hypersecretory tumors and must include insulin-like growth factor 1 (IGF-1) (for growth hormone) and prolactin levels. On imaging or gross pathology, they can be classified according to their size as macroadenomas when greater than 1 centimeter or microadenoma when less than 1 centimeter. Early morning cortisol levels are reserved for patients with symptomatic manifestations of Cushing's syndrome. Referral to a surgeon for transsphenoidal resection is required when adenomas are refractory to medical therapy, are specific secretory tumors, or cause visual field defects. Visual field testing is indicated in all tumors that are large in size, macroadenomas, or are abutting the optic chiasm or nerves on imaging. This microadenoma that the patient has no signs of abutting the optic chiasm. The patient also does not have any changes in her vision. Repeating an MRI and labs at 6 months is a reasonable option once a functioning pituitary tumor has been ruled out. Go to the next page if you knew the correct answer, or click the link image(s) below to further research the concepts in this question (if desired).
Research Concepts: Pituitary Cancer
We update eBooks quarterly and Apps daily based on user feedback. Please tap flag to report any questions that need improvement.
Question 400:
A 67-year male presented today with symptoms of dry cough, fever, fatigue, exertional dyspnea. He has a history of adult T cell leukemia six months ago and began treatment with a multi-agent chemotherapy regimen. His temperature is 37.8 C( 100F ), blood pressure is 110/60 mmHg, the pulse is 100/min, and respirations are 23/min. Complete blood count reveals leukocytes 12000/mm3 with 92% neutrophils. Arterial blood gases show pH 7.45, PaO2 45 mm Hg, and PaCO2 29mmHg. Chest X-ray shows bilateral diffuse interstitial infiltrates. Which of the following would have prevented the following symptoms of the patient?
Choices: 1. 2. 3. 4.
Azithromycin Doxycycline Trimethoprim-sulfamethoxazole Acyclovir
Answer: 3 - Trimethoprim-sulfamethoxazole Explanations: This patient has pneumocystis jirovecii infection. Patients with adult T-cell lymphoma are immunocompromised and are therefore at risk for lethal opportunistic infections with organisms such as Pneumocystis jirovecii. Other opportunistic infections are commonly seen in such patients are Candida, cytomegalovirus, disseminated Cryptococcus, and Strongyloides stercoralis. Most patients have a fever, dyspnea, and a dry, nonproductive cough that evolves over several weeks (HIV infection) or over several days (other causes of compromised cell-mediated immunity). Dyspnea is common. Chest x-ray characteristically shows diffuse, bilateral perihilar infiltrates. Pulse oximetry and ABG have to be measured. Histopathologic demonstration of the organism is needed for confirmation of the diagnosis. Methenamine silver, Giemsa, and Wright-Giemsa stains are used. Sputum specimens are usually obtained by induced sputum or bronchoscopy with bronchoalveolar lavage. Oral trimethoprim-sulfamethoxazole (TMP-SMX) for Pneumocystis jirovecii pneumonia (PCP) is commonly used for prophylaxis. In addition, antifungals are administered to all patients receiving chemotherapy for ATL. Anti-strongyloides agents are also given to patients with a past and/or present exposure to the parasite. The dose of trimethoprim-sulfamethoxazole for patients with normal kidney function is 15 to 20 mg/kg intravenously or orally daily in three or four divided doses. Patients should receive intravenous therapy until
they are clinically stable (e.g., PaO2 =60 mmHg, respiratory rate 25). For patients with allergy to trimethoprimsulfamethoxazole, desensitization should be performed since trimethoprim-sulfamethoxazole is the most effective regimen. However, if the patient has a history of a severe allergy, trimethoprim-sulfamethoxazole should be avoided, and desensitization is not performed. Alternative agents like atovaquone, pentamidine are used. Go to the next page if you knew the correct answer, or click the link image(s) below to further research the concepts in this question (if desired).
Research Concepts: Adult T Cell Leukemia
We update eBooks quarterly and Apps daily based on user feedback. Please tap flag to report any questions that need improvement.
Section 5 Question 401:
A 55-year-old male is brought to the hospital with complaints of painless jaundice that developed during the last week. He reports diffused mild itching but no rash, weight loss, or fever. The past medical history is remarkable for hypertension and hyperlipidemia, for which he takes thiazide, simvastatin, and lisinopril. He does not use alcohol or cigarette. Currently, the patient is afebrile and hemodynamically stable. Physical examination is positive for mild jaundice. The lab report includes total bilirubin of 4.5 mg/dl, direct bilirubin of 0.9 mg/dl, alanine aminotransferase (ALT) of 30 U/L, aspartate transaminase (AST) of 14 U/L, alkaline phosphatase of 121 U, and negative for anti-mitochondrial antibodies. CT abdomen reveals extrahepatic biliary dilation and a mass at the biliary bifurcation. Which of the following is the most likely diagnosis?
Choices: 1. 2. 3. 4.
Primary biliary cholangitis Primary sclerosing cholangitis Cholangiocarcinoma Hepatocellular carcinoma
Answer: 3 - Cholangiocarcinoma Explanations: In a patient, who has painless jaundice and CT scan showing extrahepatic biliary dilation and presence of a mass at the biliary bifurcation, cholangiocarcinoma is the most likely diagnosis. Intrahepatic cholestasis can be from several diseases, including any form of acute hepatitis (viral, drug, alcohol), drug-induced liver injury, primary biliary cholangitis, primary sclerosing cholangitis, and infiltrative diseases (sarcoidosis, tumors, abscess, and cysts). The extrahepatic biliary obstruction causalities include choledocholithiasis, choledochal cysts, Mirizzi's syndrome, benign stricture disease (primary sclerosing cholangitis (PSC), iatrogenic), neoplastic stricturing disease (cholangiocarcinoma, pancreatic head cancer, ampullary carcinomal or adenoma) and infectious diseases (parasitic cholangiopathy), inflammatory and autoimmune disease (AIDS cholangiopathy, autoimmune cholangiopathy) The prognosis of biliary obstruction is highly dependent on the etiology. If left untreated, it can lead to lifethreatening infection. If the obstruction is persistent and chronic, it is usually due to a chronic liver pathology, which commonly has a poor prognosis. Most acute cases can be successfully managed with medical, surgical, and or endoscopic treatment with full recovery. Obstruction caused by chronic liver disease and carcinoma usually has a less favorable prognosis.
Go to the next page if you knew the correct answer, or click the link image(s) below to further research the concepts in this question (if desired).
Research Concepts: Biliary Obstruction
We update eBooks quarterly and Apps daily based on user feedback. Please tap flag to report any questions that need improvement.
Question 402:
A 44-year-old male presented to his health care provider with a right thyroid nodule discovered six months ago. It measured 2 cm in the greatest dimension and has been stable in size. He had no history of ionizing radiation exposure or a family history of endocrine malignancy. The patient had a fine needle aspiration (FNA) biopsy, and the calcitonin level in the washout of the FNA biopsy needle was reported to be three times higher than the normal values. Which of the following findings is predicted?
Choices: 1. 2. 3. 4.
Low calcitonin level following pentagastrin stimulation Presence of uniform cells on FNA Positive history of diarrhea and flushing Distinguished follicular development
Answer: 3 - Positive history of diarrhea and flushing Explanations: Parafollicular, or C, cells are the cells of origin for medullary thyroid carcinoma (MTC) and are of neural crest origin. Normally, C cells are seen with follicular cells, either in small groups or isolated. In both normal patients and patients with medullary cancer, they manufacture calcitonin. Calcitonin acts on the kidney, bone, and gastrointestinal tract to decrease serum calcium. Amyloid deposits and elevated carcinoembryonic antigen also are seen with medullary carcinoma. Routine measurement of serum calcitonin as a part of the evaluation protocol of patients with thyroid nodules is not recommended. The high frequency of erroneously high serum calcitonin values and, more importantly, the inability to confirm the elevated calcitonin by pentagastrin stimulation would recommend against the routine check of calcitonin level in thyroid nodules. If the clinical scenario of the patient is highly suspicious for MTC, including a positive history of diarrhea, flushing, and the presence of a thyroid nodule, calcitonin might be measured in the washout of the FNA biopsy needle. Go to the next page if you knew the correct answer, or click the link image(s) below to further research the concepts in this question (if desired).
Research Concepts: Medullary Thyroid Cancer
We update eBooks quarterly and Apps daily based on user feedback. Please tap flag to report any questions that need improvement.
Question 403:
A 75-year-old man presents a 1-week history of fever, arthralgia, and malaise. He has also noticed several painful bumps and rashes on his arms that started a few days ago. He reports unintentional weight loss of 15 pounds in the last month. His past medical history is significant for hypertension, for which he was started on hydrochlorothiazide 6 months ago. Vitals signs show respiratory rate 18/minute, heart rate 98 beats per minute, blood pressure 136/88 mmHg, and temperature 101 F. On examination, the patient has multiple tender erythematous plaques and nodules on upper extremities and chest. Lab results show hemoglobin 8.6 mg/dL (13-17 g/dL), platelets 48 x 10^9/L (150-400 x 10^9/L), WBC 2.0 x 10^9/L (4-10 x 10^9/L) and ESR is 88 mm/hr ( less than 35 mm/hr). Kidney function and liver function are normal. Further testing shows that ANA is positive 1:80 in a homogeneous pattern. All other serologies are negative. Skin biopsy shows diffuse dense neutrophilic infiltrate. Corticosteroids are started. What is the most appropriate further step in the management of this patient?
Choices: 1. 2. 3. 4.
Stop hydrochlorothiazide Start hydroxychloroquine Bone marrow biopsy Repeat ANA
Answer: 3 - Bone marrow biopsy Explanations: This patient has Sweet syndrome with a typical rash alongside symptoms of fever and arthralgia, accompanied by a biopsy demonstrating dense dermal neutrophilic infiltrate. Sweet syndrome can be associated with malignancy, especially in elderly patients. The presence of pancytopenia raises strong suspicion of myelodysplasia in this elderly patient with new-onset Sweet syndrome in addition to weight loss. This patient should be started on corticosteroids, and further workup pursued to determine the etiology of Sweet syndrome. The best way to do this, in this case, will be bone marrow biopsy. Drug-induced lupus from hydrochlorothiazide can cause fever, rash, and cytopenias. However, the rash is typically subacute cutaneous lupus rash with pathology showing vacuolar interface dermatitis with lymphohistiocytic infiltration of the upper dermis in a perivascular pattern. ANA is a nonspecific test, and with negative, more specific serologies, the ANA alone in this low titer is unlikely to be of any clinical significance. Go to the next page if you knew the correct answer, or click the link image(s) below to further research the concepts in this question (if desired).
Research Concepts: Sweet Syndrome
We update eBooks quarterly and Apps daily based on user feedback. Please tap flag to report any questions that need improvement.
Question 404:
A 43-year-old female patient presents to the department for evaluation of an incidental left thyroid nodule seen on computed tomography (CT) neck. The radiology report for the CT scan notes that the thyroid nodule measures 0.5 centimeters. Which one of the following transducers should be selected in the evaluation of this patient's thyroid gland?
Choices: 1. 2. 3. 4.
A linear transducer probe A curvilinear transducer probe Phase array probe Endocavitary probe
Answer: 1 - A linear transducer probe Explanations: A linear transducer probe is most often used in the evaluation of a thyroid nodule. As a high-frequency probe that produces high-resolution images, a linear transducer probe has the ability to identify microcalcifications that are important in the workup of a thyroid nodule. A curvilinear probe is used for deep abdominal structures and transvaginal ultrasounds. As a lowfrequency probe, a curvilinear probe allows for the assessment of intraabdominal organs due to its wide depth of field. A phased array probe has a small footprint and is often used during the brain and cardiac examinations. It has the ability to assess deep structures when a sonographer or operator is only offered a small acoustic window such as ribs. An endocavitary probe is a high-frequency probe that is most optimally shaped for a transvaginal ultrasound. Although possibly used in the evaluation of a thyroid nodule, a regular linear transducer probe would be appropriate for the evaluation of thyroid nodules. Go to the next page if you knew the correct answer, or click the link image(s) below to further research the concepts in this question (if desired).
Research Concepts: Thyroid Nodule Biopsy
We update eBooks quarterly and Apps daily based on user feedback. Please tap flag to report any questions that need improvement.
Question 405:
A 54-year-old male with a history of HIV was recently diagnosed with Kaposi sarcoma and initiated on intravenous chemotherapy with daunorubicin. During his first chemotherapy session, he started to complain of severe pain at the infusion site. On examination, there is significant redness and induration of the surrounding area. Which of the following could have prevented this complication of therapy?
Choices: 1. Intravenous fluids 2. Slow intravenous infusion rate 3. Intravenous infusion through a rapidly flowing pump 4. Prophylactic administration of antihistamines and corticosteroids
Answer: 3 - Intravenous infusion through a rapidly flowing pump
Explanations: Daunorubicin is known to cause local skin inflammation. It should only be administered intravenously through rapidly flowing infusions to prevent the extravasation of the drug into the surrounding tissue causing a local skin reaction. In the event of a local reaction, the infusion should be stopped immediately, and cold compress should be applied over the extravasation site. Intramuscular or subcutaneous administration should also be avoided. Concomitant use of intravenous fluids is recommended with amphotericin B and other nephrotoxic agents administration to avoid acute kidney injuries. The slow infusion will further exacerbate the local extravasation of the drug. There is no role of antihistamines or corticosteroids, as this is not a hypersensitivity reaction. Go to the next page if you knew the correct answer, or click the link image(s) below to further research the concepts in this question (if desired).
Research Concepts: Daunorubicin
We update eBooks quarterly and Apps daily based on user feedback. Please tap flag to report any questions that need improvement.
Question 406:
A 55-year-old male is referred to the thoracic surgery department after a 1.3 cm solid nodule in the left upper lobe of the lung discovered on a chest CT scan. After a complete evaluation, the nodule came out to be positive on PET scan without any evidence of mediastinal adenopathy. The patient was stratified as low-risk postoperative dyspnoea. What surgical procedure should be done in this case?
Choices: 1. 2. 3. 4.
Open wedge resection VATS lobectomy with lymph node sampling Open lobectomy with lymph node sampling VATS lobectomy
Answer: 2 - VATS lobectomy with lymph node sampling Explanations: Wedge resection is considered in ground glass opacifications (GGO) nodules 1cm or in patients who cannot tolerate a complete anatomical lung resection in early-stage lung cancer. However, in the last scenario, the recurrence risk is higher. The National Comprehensive Cancer Network (NCCN) and the International Association for the Study of Lung Cancer (IASLC) recommend Video-assisted thoracoscopic surgery (VATS) lobectomy as the first option in early-stage lung cancer with lymph node sampling or resection. In this case, there is no contraindication for performing a VATS lobectomy. However, if there is a contraindication, the open surgery is indicated. VTAS is contraindicated in tumors >5 cm. Regardless of the negative result in the nodal mediastinal evaluation (Early-stage lung cancer), The National Comprehensive Cancer Network (NCCN) and the International Association for the Study of Lung Cancer (IASLC) recommend lobectomy plus lymph node sampling or dissection. Go to the next page if you knew the correct answer, or click the link image(s) below to further research the concepts in this question (if desired).
Research Concepts: Lobectomy
We update eBooks quarterly and Apps daily based on user feedback. Please tap flag to report any questions that need improvement.
Question 407:
A 4-year-old girl is brought to the clinic with headaches, vomiting, and ataxia for two weeks. Initial workup reveals a midline cerebellar large, demarcated, solid mass. Surgery is performed without complications, and pathology shows a non-medulloblastoma, embryonal tumor with multilayered rosettes. Which is the most likely molecular driver for this patient's tumor?
Choices: 1. 2. 3. 4.
1p/19q codeletion Amplification of C19MC region on chromosome 19 Nf-kB pathway activation via C11orf95-RELA fusion K27 mutations in the histone 3 gene H3F3A
Answer: 2 - Amplification of C19MC region on chromosome 19
Explanations: Embryonal tumors are classified depending on if they have on chromosome 19 an amplification of the C19MC region (19q13.42). If they have the amplification, they are called embryonal tumors with multilayered rosettes, C19MC-altered (WHO 9478/3*). Approximately 20% of pediatric brain tumors are composed of CNS embryonal tumors. This number includes all the medulloblastomas and all other embryonal tumors. Those embryonal tumors with rhabdoid features that do not have alterations of either INI1 or BRG1 are classified as (WHO 9508/3). Those embryonal tumors that have no amplification of the C19MC, no alterations of INI1 or BRG1, or no rosettes identified, are called embryonal tumors, NOS (WHO 9473/3). This last group is a category for the tumors previously called PNET, which cannot be classified under a genetic/molecular group. Medulloblastomas are embryonal tumors but have their own histologically defined groups (classic, desmoplastic/nodular, medulloblastoma with extensive nodularity, and large cell/anaplastic) and genetic/molecular defined groups (SHH-activated TP53mutant, SHH-activated TP53-wildtype, WNT-activated, non-SHH/WNT group three, and group four). Go to the next page if you knew the correct answer, or click the link image(s) below to further research the concepts in this question (if desired).
Research Concepts: Primitive Neuroectodermal Tumor
We update eBooks quarterly and Apps daily based on user feedback. Please tap flag to report any questions that need improvement.
Question 408:
A 46-year-old woman presents to the clinic for a regular checkup for her stage 4 pancreatic cancer. She states that she feels well and has not noticed the side effects of her gemcitabine/ erlotinib therapy. What is the best-known mechanism of action of the nonchemotherapy drug in this patient’s treatment?
Choices: 1. 2. 3. 4.
Tyrosine kinase inhibitor Alkylation of DNA Topoisomerase inhibition Mustard formation
Answer: 1 - Tyrosine kinase inhibitor Explanations: Erlotinib is a tyrosine kinase inhibitor acting on the epidermal growth factor receptor. Erlotinib and gemcitabine, in combination, have been shown to be more effective than gemcitabine alone in advanced pancreatic cancer. The EGFR tyrosine kinase receptor forms homodimers or heterodimers when activated, leading to cell proliferation with downstream signaling. This signaling pathway has been identified in many human carcinomas. Alkylating agents, topoisomerase inhibitors, and mustard forming drugs are all examples of chemotherapy agents. Go to the next page if you knew the correct answer, or click the link image(s) below to further research the concepts in this question (if desired).
Research Concepts: Erlotinib
We update eBooks quarterly and Apps daily based on user feedback. Please tap flag to report any questions that need improvement.
Question 409:
An 80-year-old woman with confirmed AML M4 with unfavorable cytogenetics was started on the hypomethylating agent azacytidine. After eight cycles of therapy, her counts have gradually improved, and her pRBC transfusion needs have decreased. Her platelets have improved as well but not quite to normal levels. However, some new skin lesions appeared on her trunk. On examination, multiple violaceous, erythematous papules are noted. A review of her medications reveals no identifiable culprit. Histopathologic examination of the skin lesions shows blastic infiltration with large pleomorphic nuclei staining positive with CD 34, CD117, CD 33, and MPO positive. She is therefore diagnosed with leukemia cutis with AML relapse in the bone marrow synchronously. Nextgeneration sequencing reveals an FLT3 ITD mutation. Given her relapse less than 12 months from initiation of therapy, which of the following is her most appropriate treatment option?
Choices: 1. 2. 3. 4.
A hypomethylating agent with venetoclax Switch to a different hypomethylating agent Switch to treatment with omacetaxine mepesuccinate Palliative and supportive care only
Answer: 1 - A hypomethylating agent with venetoclax Explanations: Relapse less than 12 months from initiation of therapy portends a poor prognosis. Approximately 25-30% of patients with AML may have a mutation in the FLT3 gene. These mutations are associated with a particularly aggressive disease with a higher risk of relapse. FLT3 mutation allows leukemic cells to grow uncontrolled. Based on the Phase III ADMIRAL trial that randomized relapsed AML patients to gilteritinib or standard chemotherapy, 21% of the patients were able to achieve a complete remission with a fairly low toxicity drug. This trial is still currently ongoing. Other treatment options for her include the use of a hypomethylating agent in combination with venetoclax which also included poor-risk cytogenetics in 49% of enrolled patients. Although less favored and as she is CD33 positive, single agent Gemtuzumab, if she is deemed to be fit for more intensive chemotherapy, has been seen to have an estimated response rate of 26% based on two phase III clinical trials. It is a CD33 directed antibody-drug conjugate that recognizes CD33 antigen covalently bound to a small molecule of N acetyl gamma calicheamicin linker that when deployed leads to cell cycle arrest and apoptotic cell death. Side effects of gemtuzumab include infusion reaction, myelosuppression, hepatotoxicity, and venoocclusive disease. Omacetaxine mepesuccinate for injection, for subcutaneous use, is used to treat adult patients with chronic or accelerated phase chronic myeloid leukemia
(CML) who are no longer responding to, and/or who could not tolerate, two or more tyrosine kinase inhibitors (TKI). Go to the next page if you knew the correct answer, or click the link image(s) below to further research the concepts in this question (if desired).
Research Concepts: Leukemia Cutis
We update eBooks quarterly and Apps daily based on user feedback. Please tap flag to report any questions that need improvement.
Question 410:
A 22-year-old female presents for a diagnostic mammogram after a suspicious lesion was palpated on a routine exam. The mammogram identifies a mass concerning for malignancy. Biopsy reveals invasive ductal carcinoma. On physical examination, there is also the presence of a butterfly rash, prominent nose, malar hypoplasia, and short stature. In addition to surgical resection, what is the best treatment for this patient?
Choices: 1. 2. 3. 4.
Proton beam therapy Radiation therapy 5-fluorouracil therapy Cyclophosphamide therapy
Answer: 1 - Proton beam therapy Explanations: This patient most likely has Bloom syndrome. Due to the increased risk of chromosomal breakage, radiation should be avoided in patients with Bloom. Proton beam therapy uses protons to deliver high energy beams to tumor cells instead of x-rays. Studies have shown higher efficacy and a better safety profile compared to traditional radiation in Bloom syndrome patients. If needed, lower-dosed chemotherapy may be used to treat malignancy in patients with Bloom syndrome. However, there is a higher risk of bone marrow suppression and toxicity compared to proton beam therapy. 5-Fluorouracil has been shown to induce higher levels of DNA fragmentation when used in Bloom syndrome patients. Go to the next page if you knew the correct answer, or click the link image(s) below to further research the concepts in this question (if desired).
Research Concepts: Bloom Syndrome
We update eBooks quarterly and Apps daily based on user feedback. Please tap flag to report any questions that
need improvement.
Question 411:
A 74-year-old woman with end-stage pancreatic cancer is being transferred from the inpatient floor to an inpatient hospice unit. During the course of her illness, she had a port, but it recently became infected and was removed. She has very small veins, and it has been very difficult to maintain intravenous (IV) access to administer medications. Given the recent progression of her illness, she is unable to tolerate any pills or liquids by mouth. She is in severe pain at this time, but her family wishes to avoid any further needle sticks or additional discomfort. Morphine is the medication of choice at this time for pain control. What is the most appropriate route of administration for this patient?
Choices: 1. 2. 3. 4.
Intrathecal Intramuscular Oral Sublingual
Answer: 4 - Sublingual Explanations: Sublingual morphine is readily absorbed and is an effective medication to treat pain. This route of administration of morphine is popular in hospice and palliative care treatments. In these scenarios, this medication can also provide relief from shortness of breath called air hunger. In hospice situations, opioids should be used regularly to manage pain. Over-the-counter medications, while a useful adjunct or for treating mild pain, are not appropriate as monotherapy for severe pain related to end-stage cancer. Go to the next page if you knew the correct answer, or click the link image(s) below to further research the concepts in this question (if desired).
Research Concepts: Morphine
We update eBooks quarterly and Apps daily based on user feedback. Please tap flag to report any questions that need improvement.
Question 412:
A 39-year-old female undergoes a thyroid biopsy for a mass. Pathology exam reveals a solid mass that stains positive for calcitonin and amyloid. She mentions that one of her brothers also had a similar neck problem. Before surgery, the patient should be worked up for which of the following conditions?
Choices: 1. 2. 3. 4.
Recurrent medullary cancer Presence of pheochromocytoma Hypocalcemia Multiple myeloma
Answer: 2 - Presence of pheochromocytoma Explanations: Medullary cancer may be sporadic or may occur in a familial fashion. Medullary cancer is associated with multiple endocrine neoplasia type 2, which includes parathyroid hyperplasia and pheochromocytoma. These individuals with multiple endocrine neoplasia type 2 have a mutation in the RET proto-oncogene. Screening for pheochromocytoma involves looking for metanephrines in the urine. Go to the next page if you knew the correct answer, or click the link image(s) below to further research the concepts in this question (if desired).
Research Concepts: Medullary Thyroid Cancer
We update eBooks quarterly and Apps daily based on user feedback. Please tap flag to report any questions that need improvement.
Question 413:
A 67-year-old man with stage 3 prostate cancer is referred for cerebral spinal fluid (CSF) testing. Previous investigations identified a lesion on the lower cervical cord at C6 to C7 and multiple lumbar lesions from L2 to L5. The lesions seem intradural, and diagnostic CSF has been requested because the primary tumor was prostrate. A lumbar puncture is contraindicated due to the tumor invasion. Where is the best place to obtain a CSF sample for this patient?
Choices: 1. 2. 3. 4.
Above the arch of C1 Between C2 and C3 Upper thoracic area, between T1 and T2 Lower thoracic area, between T11 and T12
Answer: 1 - Above the arch of C1 Explanations: Suboccipital puncture is performed between occiput and C1 with a midline or lateral approach. Suboccipital puncture is indicated when there is a complete obstruction documented or for the definition of the superior margin in a complete block. Suboccipital puncture is contraindicated in patients with craniocervical deformities. Below C2, this procedure is too risky because of the anatomy of the cervical and thoracic spinal cord, which occupies most of the intradural compartment. Go to the next page if you knew the correct answer, or click the link image(s) below to further research the concepts in this question (if desired).
Research Concepts: Suboccipital Puncture
We update eBooks quarterly and Apps daily based on user feedback. Please tap flag to report any questions that need improvement.
Question 414:
A 76-year-old man presents to the clinic with a history of an asymptomatic, enlarging 3-month-old bump over his upper chest. The patient had previously received surgery and radiotherapy due to thyroid carcinoma, currently in remission. He currently takes levothyroxine 100 mcg daily. On physical examination, there is a 10 cm x 6 cm x 6 cm subcutaneous mass, with no apparent skin changes, at the suprasternal notch level. An incisional biopsy shows a spindle-cell tumor with numerous mitoses, and nuclear atypia, adopting a storiform pattern over a fibrous stroma that extends to the deep dermis. Immunohistochemistry is negative for keratin 14, S100, smooth-muscle actin, and MDM2, while p53 and LN2 are positive. Which of the following cells is primarily implicated in the pathogenesis of this condition?
Choices: 1. 2. 3. 4.
Histiocytes Pericytes Mesenchymal stem cells Thyroid cells
Answer: 3 - Mesenchymal stem cells Explanations: Undifferentiated pleomorphic sarcoma, formerly known as malignant fibrous histiocytoma, is one of the most common soft-tissue sarcomas in the adult population. Previous radiation therapy may be a risk factor in some cases. The precise etiology of undifferentiated pleomorphic sarcoma remains unknown, but histiocytes are no longer considered culprits. Advancements in cytogenetics and immunohistochemistry show that tumorigenic mesenchymal stem cells are able to recapitulate tumor formation in animal models. These mutated mesenchymal stem cells possess complex genetic aberrations that allow self-renewal, proliferation, and growth through alterations of different signaling pathways. Go to the next page if you knew the correct answer, or click the link image(s) below to further research the concepts in this question (if desired).
Research Concepts: Undifferentiated Pleomorphic Sarcoma
We update eBooks quarterly and Apps daily based on user feedback. Please tap flag to report any questions that
need improvement.
Question 415:
A 42-year-old man is diagnosed with gastric adenocarcinoma "intestinal subtype" without vascular or lymphatic invasion. The 10 mm non-ulcerated flat lesion is staged "T1AN0M0" after an endoscopy ultrasound and a positron emission tomography combined with computerized tomography imaging. What is the best recommendation for treatment?
Choices: 1. Start neoadjuvant therapy prior gastrectomy and lymphadenectomy 2. Recommend upfront gastrectomy and lymphadenectomy with adjuvant therapy subsequently 3. Refer to Comprehensive Cancer Center with Endoscopic Center of Excellence for endoscopic submucosal resection 4. Perform epidermal growth factor receptor 2 (HER2) testing for targeted therapy
Answer: 3 - Refer to Comprehensive Cancer Center with Endoscopic Center of Excellence for endoscopic submucosal resection
Explanations: Endoscopic resection either by endoscopic mucosal resection or endoscopic submucosal resection is offered to selected patients with early gastric cancer with negative lymph nodes who meet selection criteria at centers of expertise. A 10% of mucosal and 20% of submucosal lesions will have lymph node metastasis and should be carefully investigated by PET/CT. Standard selection criteria are high probability of en bloc resection, intestinal type adenocarcinoma confined to the mucosa/submucosal, and absent venous or lymphatic invasion and tumors with diameters less than 20mm without ulceration or 10mm nonpolypoid flat or depressed lesions. Expanded criteria are under active investigation. Successful endoscopic resection may offer a 5-year overall survival of 96% compared to gastrectomy survival rates up to 98%, but no randomized trials have compared both. Neoadjuvant is recommended for muscularis propia invasion (equal or higher than T2) and adjuvant therapy for those with pathological resection with invasion of subserosa (pT3) or positive lymph node (pN1). HER2 testing is recommended for locally advanced unresectable and advanced metastatic disease. Go to the next page if you knew the correct answer, or click the link image(s) below to further research the concepts in this question (if desired).
Research Concepts: Gastric Cancer
We update eBooks quarterly and Apps daily based on user feedback. Please tap flag to report any questions that need improvement.
Question 416:
A 70-year-old woman presents to the hospital with a 6-month history of worsening cough with recurrent hemoptysis. She has smoked at least one pack of cigarettes a day for the past 55 years. She has had a 15 kg (33 lbs) weight loss during this period. She has psoriasis vulgaris, arterial hypertension, and hypercholesterinemia. Her vital signs are within normal limits. Initial blood tests reveal a GFR of 50 mL/min. A chest X-ray shows a 2 cm cavitated mass in the upper lobe of her right lung. Subsequently, a diagnosis of squamous non-small cell lung cancer is made. The tumor expresses no EGFR or ALK sensitizing translocations. Which of the following is the strongest possible contraindication for therapy with nivolumab in this patient?
Choices: 1. 2. 3. 4.
Moderate renal failure Psoriasis vulgaris Hypersensitivity to nivolumab BRAF V600E mutation
Answer: 3 - Hypersensitivity to nivolumab Explanations: Nivolumab is a humanized monoclonal IgG4 kappa antibody directed to block human cell surface PD-1. When PD-L1 engages with PD-1, the T-cell function is inhibited. Nivolumab, a checkpoint inhibitor, blocks the PD-1: PDL-1 complex formation allowing improved T-cell mediated killing. Nivolumab is approved for use in first-line metastatic squamous NSCLC combined with ipilimumab and platinum-based chemotherapy. Hypersensitivity to nivolumab or the excipients in the IV formulation is a contraindication for therapy. No dose adjustment is needed in patients with mild or moderate renal impairment; data on severe renal impairment are limited. Psoriasis vulgaris is not a contraindication; however, a flare-up may be induced by immune checkpoint inhibitors. A BRAF V600E mutation is not a contraindication for immune checkpoint inhibitors, and these patients might profit from BRAF/MEK inhibitors. Go to the next page if you knew the correct answer, or click the link image(s) below to further research the concepts in this question (if desired).
Research Concepts: Nivolumab
We update eBooks quarterly and Apps daily based on user feedback. Please tap flag to report any questions that need improvement.
Question 417:
A 65-year-old male patient underwent excision for a high-grade glioma followed by six months of radiation therapy. Follow-up magnetic resonance imaging (MRI) three months later showed evidence of a single fluidattenuated inverse recovery (FLAIR), suppressed and enhancing 1.5 cm lesion within the operated site. The MR spectroscopy showed low N-acetyl aspartate (NAA)/choline peak. What is the ideal management plan for the patient?
Choices: 1. 2. 3. 4.
Surgical excision Radiation therapy Chemotherapy Conservative management
Answer: 4 - Conservative management Explanations: One of the complications of radiation therapy includes radiation necrosis within the irradiated site that can masquerade as tumor recurrence. Tumor necrosis will be seen as a fluid-attenuated inverse recovery (FLAIR) suppressed and enhancing lesion within the irradiated zone. It will have a characteristic low NAA/choline ratio in MR spectroscopy. They are benign and do not require any intervention. Repeat radiation or chemotherapy is justified only in cases of tumor recurrence. Tumor recurrence will have a choline peak in MR spectroscopy. Go to the next page if you knew the correct answer, or click the link image(s) below to further research the concepts in this question (if desired).
Research Concepts: Adverse Effects Of Radiation Therapy
We update eBooks quarterly and Apps daily based on user feedback. Please tap flag to report any questions that need improvement.
Question 418:
A 59-year-old male patient presents to the emergency department with complaints of headaches of several months' duration. CT head revealed a mass in the left temporal lobe. MRI revealed heterogeneous enhancement and central necrosis. Diagnosis of glioblastoma multiforme (GBM) is made post-surgical resection. The patient is scheduled for concomitant chemoradiotherapy. Which of the following medications is recommended for him along with the therapy he is most likely to receive?
Choices: 1. 2. 3. 4.
Doxycycline Clindamycin Trimethoprim/sulfamethoxazole Azithromycin
Answer: 3 - Trimethoprim/sulfamethoxazole Explanations: Temozolomide is given orally daily during radiation therapy. Adjuvant treatment starts four weeks after radiotherapy, and it is given for six cycles daily for five days in a 28-day cycle. Side effects include leucopenia, thrombocytopenia, hepatotoxicity, nausea, constipation, fatigue, etc. Pneumocystis jirovecii pneumonia prophylaxis is required during treatment with temozolomide, and concomitant radiation therapy as temozolomide typically causes CD4 T cell depletion. It is recommended even if patients are not on steroids. The medications for PCP prophylaxis include trimethoprim/sulfamethoxazole, atovaquone, and dapsone. Empirical treatment for other infections is not recommended. Go to the next page if you knew the correct answer, or click the link image(s) below to further research the concepts in this question (if desired).
Research Concepts: Glioblastoma Multiforme
We update eBooks quarterly and Apps daily based on user feedback. Please tap flag to report any questions that
need improvement.
Question 419:
A 35-year-old man presents to the clinic with a complaint of 4 episodes of focal seizures within the past 5 months. He has also started to experience difficulty in maintaining balance for 1 month. On further history, he reports chronic early morning headache for the past 2 years, associated with intermittent nausea. Which of the following is the most likely diagnosis?
Choices: 1. 2. 3. 4.
Low-grade glioma Acoustic neuroma Meningioma Cerebrovascular accident
Answer: 1 - Low-grade glioma Explanations: Seizures occur in 90% of patients with low-grade glioma. Hence a high likelihood of the disease is there. Also, the age of the patient is consistent with low-grade glioma along with chronic history of headaches & nausea. Usually, malignancies can present with focal symptoms like seizures, visual disturbances. High-grade malignancies usually present in older ages. Acoustic neuromas may also present with gait and balance disturbances, but the possibility of focal seizures is quite less, along with headaches. Go to the next page if you knew the correct answer, or click the link image(s) below to further research the concepts in this question (if desired).
Research Concepts: Astrocytoma
We update eBooks quarterly and Apps daily based on user feedback. Please tap flag to report any questions that need improvement.
Question 420:
A 73-year-old African American male presents with a 6-month history of nearly 30-pound (13.6 kg) weight loss, night sweats, low back pain, and difficulty initiating his urinary stream. His past medical history is significant for hypertension, hyperlipidemia, and coronary artery disease. Initial blood results demonstrate microcytic anemia and an alkaline phosphatase level that is five times the expected value. The serum prostate-specific antigen (PSA) level is 101 ng/mL. What is the relative likelihood of the presence of metastatic disease in this patient?
Choices: 1. Almost none, as PSA values greater than 100 ng/ml usually indicate significant prostate hypertrophy and not cancer. 2. The same likelihood as any elevated serum PSA result, as higher values do not correlate with a higher probability of the presence of metastatic disease. 3. It is likely but not certain that this patient has metastatic disease, as there is often a direct correlation with increased PSA values and more advanced cancer stage. 4. This patient almost certainly has metastatic disease, as PSA values greater than 10 ng/ml almost always indicate prostate cancer of significant duration and severity.
Answer: 3 - It is likely but not certain that this patient has metastatic disease, as there is often a direct correlation with increased PSA values and more advanced cancer stage.
Explanations: Multiple large studies have demonstrated that men with prostate-specific antigen (PSA) levels greater than 100 ng/ml are more likely to have metastatic disease. However, in this same population, there are about 20%-25% of individuals that do not have prostate cancer. A key takeaway is that no PSA value can guarantee the presence or absence of prostate cancer. The same trials show that despite similarly elevated PSA values of greater than 100 ng/ml, the men with prostate cancer but without metastases had a 5-year survival that was 2-3-fold greater than individuals with metastatic disease on initial presentation. It should, therefore, not be assumed that any man with an initial PSA over 100 ng/ml at the time of diagnosis has metastatic disease. PSA levels alone are insufficient to diagnose metastatic disease even when substantially elevated. However, levels at or above 20 ng/ml are sufficient to justify a bone scan to check for bony metastases. On initial diagnosis, as many as 14% of all prostate cancers demonstrate metastatic disease. The decision to investigate for metastasis should be made on an individual basis with consideration for the presentation, physical exam, lab values, and imaging, not on a specific PSA threshold Go to the next page if you knew the correct answer, or click the link image(s) below to further research the concepts in
this question (if desired).
Research Concepts: Prostate Specific Antigen
We update eBooks quarterly and Apps daily based on user feedback. Please tap flag to report any questions that need improvement.
Question 421:
A 59-year-old man with a 30 pack-year smoking history, as well as hypothyroidism, morbid obesity, and esophageal reflux is diagnosed with isolated squamouscell carcinoma of the right upper lobe. He subsequently undergoes successful resection via right upper lobectomy. His recovery is uneventful until six months later when he starts developing severe pain in the right thoracic region near his thoracotomy incision. Which of the following is the next best step in the management of this patient?
Choices: 1. 2. 3. 4.
Intercostal nerve block Gabapentin CT of the chest Pulmonary function testing (PFT)
Answer: 3 - CT of the chest Explanations: In the setting of prior cancer and delayed onset of pain after resection, the possibility of cancer recurrence should be promptly evaluated. CT imaging is the most appropriate next step as it will evaluate for cancer recurrence and may show if any other anatomic abnormalities are present. Visceral and parietal pleura receive abundant innervation. Pulmonary neoplastic process may often go undetected due to the absence of pain. Yet, once the neoplasm distends or starts invading pleura, the pain is often very pronounced and has pleuritic characteristics. A multi-modal pain management medication regimen is certainly a good approach to pain, both acute and chronic, and should be utilized in an attempt to control this patient's pain. Yet the most appropriate next step in this situation is to rule out cancer recurrence. Although PFTs may provide some insight into pulmonary mechanics and functionality, they are not the most appropriate next step. Pulmonary function testing would have been more appropriate for the preoperative workup to evaluate whether the patient with a history of smoking and a lung mass had the pulmonary reserve to tolerate partial lung resection. Go to the next page if you knew the correct answer, or click the link image(s) below to further research the concepts in this question (if desired).
Research Concepts:
Intercostal Neuralgia
We update eBooks quarterly and Apps daily based on user feedback. Please tap flag to report any questions that need improvement.
Question 422:
A female presents with a neck mass but has no other symptoms. Examination reveals a 2 x 2 cm hard mass. A nuclear scan reveals a cold mass. What is the most likely diagnosis?
Choices: 1. 2. 3. 4.
Hashimoto thyroiditis Adenoma Nodular goiter Thyroglossal duct cyst
Answer: 2 - Adenoma Explanations: Eighty percent of cold thyroid nodules are benign. Only 20% of cold nodules are malignant or adenomas. Almost all thyroid adenomas are follicular adenomas. Follicular adenomas are "cold", "warm," or "hot" depending on their level of function. Histopathologically, follicular adenomas can be classified according to their cellular architecture, cellularity, and colloid into six types: (1) fetal (microfollicular) which has small, closely packed follicles lined with epithelium; (2) colloid (macrofollicular) which does not have the potential for microinvasion; (3) embryonal (atypical) which has the potential for microinvasion; (4) Hurthle cell adenoma (oxyphil or oncocytic tumor) which has the potential for microinvasion; (5) hyalinizing trabecular adenoma; and (6) papillary adenomas which are very rare. A thyroid adenoma may be silent ("cold" or "warm") or it may be a functional tumor, producing thyroid hormone ("hot"). In this case, it may result in symptomatic hyperthyroidism, and it is a known as a toxic thyroid adenoma. Most patients with thyroid adenoma can be managed by watchful monitoring for changes in nodule size and symptoms and repeat ultrasonography or needle aspiration biopsy. Go to the next page if you knew the correct answer, or click the link image(s) below to further research the concepts in this question (if desired).
Research Concepts:
Thyroid Adenoma
We update eBooks quarterly and Apps daily based on user feedback. Please tap flag to report any questions that need improvement.
Question 423:
A 60-year-old male with a past medical history of HIV was seen by the primary care physician after he presented with anal pain for 6 months and associated bleeding per rectum. He is otherwise in very good health and compliant with his antiretroviral therapy. On the anal exam, there is a concern for a mass. The patient was further evaluated by anoscopy and found to have a 2.5 cm mass in the anal canal, the biopsy of which turned out to be a squamous cell carcinoma, which is poorly differentiated. Staging studies did not show any lymph node involvement or other metastasis. Which of the following is an appropriate therapy recommended for him?
Choices: 1. 2. 3. 4.
Surgery alone Local radiation and followed by surgery 5-FU + mitomycin with concurrent local radiation 5-FU + mitomycin alone
Answer: 3 - 5-FU + mitomycin with concurrent local radiation
Explanations: 5-FU + mitomycin with concurrent local radiation is the recommended regimen for T1-T4 and N+ anal cancer. Concurrent chemoradiation with local radiation decreased the risk of local failure by 46% and has a better overall survival at 5 years compared to other regimens. Surgery alone may be an option for patients with T1 tumors less than 1 cm in size. But this approach was not compared with RT and chemoradiotherapy, which is the standard of care. Chemotherapy without radiation is not an appropriate therapy in localized anal cancer. Go to the next page if you knew the correct answer, or click the link image(s) below to further research the concepts in this question (if desired).
Research Concepts: Anal Cancer
We update eBooks quarterly and Apps daily based on user feedback. Please tap flag to report any questions that need improvement.
Question 424:
A 23-year-old woman reports visual disturbances, stating that she can no longer see well peripherally on both the right and the left. She thinks this may be the cause of depressive symptoms as she is lethargic due to weight gain, has insomnia, and can't concentrate. She denies any change in her caloric intake. On physical examination, she points to numerous changes in her skin such as easy bruising, acne problems, and purple-red striae on her trunk. MRI of her brain shows a tumor. What area of the brain is the tumor most likely found?
Choices: 1. 2. 3. 4.
Occipital lobe Pituitary Caudate nucleus Putamen
Answer: 2 - Pituitary Explanations: Cushing disease, unlike Cushing syndrome, refers to one specific cause, a pituitary adenoma. Due to the overproduction of adrenocorticotropic hormone (ACTH), these tumors cause a variety of endocrine and psychiatric disturbances. Diagnostic tests used are salivary and blood serum cortisol, 24-hour urinary free cortisol, dexamethasone suppression test, and bilateral inferior petrosal sinus sampling. Multiple tests should be used. First-line treatment is surgical resection of the ACTHsecreting pituitary adenoma. Pituitary radiation or bilateral adrenalectomy may be needed for postoperative persisting hypercortisolemia. Go to the next page if you knew the correct answer, or click the link image(s) below to further research the concepts in this question (if desired).
Research Concepts: Cushing Disease
We update eBooks quarterly and Apps daily based on user feedback. Please tap flag to report any questions that need improvement.
Question 425:
Histopathological analysis of a specimen of an intraventricular lesion shows cells increased in density and arranged in papillary fronds. Nuclear pleomorphism and necrosis are absent. What is the most likely diagnosis?
Choices: 1. 2. 3. 4.
Epidermoid Choroid plexus carcinoma Choroid plexus papilloma Ependymoma
Answer: 3 - Choroid plexus papilloma Explanations: Choroid plexus tumors retain the normal architecture of choroid plexus, i.e., frond-like arrangement. However, the cell density will be increased. They do not show nuclear pleomorphism or necrosis on microscopy. They show less than 2 mitoses per 10 high power fields. Choroid plexus carcinomas have more mitoses, show nuclear pleomorphism, and necrosis. Go to the next page if you knew the correct answer, or click the link image(s) below to further research the concepts in this question (if desired).
Research Concepts: Choroid Plexus Papilloma
We update eBooks quarterly and Apps daily based on user feedback. Please tap flag to report any questions that need improvement.
Question 426:
A 65-year-old male patient presents to the hospital with unilateral hearing loss. He works at the airport. He noted symptoms to be progressive for the past three months. He initially thought that the right side of his earphones was not working properly, but noticed that when answering the phone, he can only hear garbled words in his right ear. He has no known medical illnesses. He smokes half a pack of cigarettes a day and drinks occasionally. On examination, vital signs are normal. The cranial nerve exam is unremarkable except that Weber exam lateralizes to the left and bone conduction (BC) is better than air conduction (AC) on the right. The motor and sensory examinations are normal. A head MRI with contrast reveals a 1.1 cm enhancing lesion on the right vestibular nerve and a 0.5 cm enhancing lesion on the left vestibular nerve. Which of the following chromosomes is likely to be involved in this patient?
Choices: 1. 2. 3. 4.
Chromosome Chromosome Chromosome Chromosome
9 and 12 translocation 17 duplication 22q12.2 15q11-q13
Answer: 3 - Chromosome 22q12.2 Explanations: One disorder commonly associated with bilateral acoustic tumors in neurofibromatosis type II. It is inherited through an autosomal dominant pattern. It is caused by mutation at chromosome 22q12.2 inactivating the NF2 gene. Most patients with neurofibromatosis type II present late with acoustic neuromas. These slow-growing tumors typically appear in the fifth to seventh decade of life. The diagnosis of an acoustic neuroma is made with a contrast magnetic resonance imaging (MRI) or a computed tomogram scan. Contrast is essential; otherwise, the non-enhanced scan can miss small tumors. Go to the next page if you knew the correct answer, or click the link image(s) below to further research the concepts in this question (if desired).
Research Concepts: Acoustic Neuroma
We update eBooks quarterly and Apps daily based on user feedback. Please tap flag to report any questions that need improvement.
Question 427:
A 36-year-old man with no significant past medical history presents to the clinic for loss of vision in his right eye for the last 6 months. Fundoscopy shows a mass in the uveal tract. There is no indication of metastasis on CT abdomen/pelvis/chest. The visual acuity is 7/10 and the intraocular pressure (IOP) is 30 mmHg. A lobulated amelanotic vascularized and nodular (tapioca-like) iris mass with a 180 degrees extent is seen in the right eye. Incisional biopsy of the mass reveals iris melanoma. Brachytherapy with the ruthenium-106 plaque results in complete regression of the tumor. Which of the following is the strongest risk factor associated with this tumor?
Choices: 1. 2. 3. 4.
Dark hair color Light eye color Dark skin color Iris nevi
Answer: 2 - Light eye color Explanations: Common risk factors that have been linked to ocular melanoma include light skin color, atypical cutaneous nevi, light eye color, iris nevi, and freckles. Occupational sun exposure, tanning, outdoor activities, and welding have also been associated with ocular melanoma. There has also has been a genetic correlation with uveal and cutaneous melanoma at the CLPTM1L locus. Different genetic mutations have been associated with uveal melanoma. There have been studies showing the mutations encoding for the G-protein a-subunit. Go to the next page if you knew the correct answer, or click the link image(s) below to further research the concepts in this question (if desired).
Research Concepts: Ocular Melanoma
We update eBooks quarterly and Apps daily based on user feedback. Please tap flag to report any questions that need improvement.
Question 428:
A 21-year-old male with a history significant for smoking and alcohol presented with complaints of swelling in the neck for the past three months, which was insidious in onset gradually progressive in nature associated with dysphagia and otalgia. On examination, the patient had a firm mobile 7x 7 cm ellipsoidal swelling situated in the left side of the neck extending from the lower border of the mandible to 2.5cm above clavicle with the lower border made out. Medially the swelling extended 3cm from midline into the posterior triangle deep to sternocleidomastoid with the carotid pulsations felt superficially over the swelling. The examination of the eyes showed the ptosis of the eyelid and ipsilateral miosis. The rest of the physical examination was within normal limits. Magnetic resonance imaging (MRI) confirmed the diagnosis of cervical sympathetic chain ganglion schwannoma. Which of the following feature on MRI helped in making the diagnosis?
Choices: 1. 2. 3. 4.
Splaying of carotid bifurcation with hypervascularity Splaying of carotid bifurcation without hypervascularity. Absence of splaying Splaying of internal jugular vein and vagus nerve
Answer: 2 - Splaying of carotid bifurcation without hypervascularity.
Explanations: Schwannomas arising from the cervical sympathetic chain are usually difficult to differentiate from a vagal schwannoma and a carotid body tumor during the initial workup. MRI is necessary to differentiate between them and reaching a final diagnosis. Cervical sympathetic chain ganglion splays carotid bifurcation without hypervascularity is a hallmark for schwannoma. After confirmation, surgical resection is recommended for cervical sympathetic chain ganglion. It gives excellent results with no local recurrence. Splaying of carotid bifurcation with hypervascularity usually determines a carotid body tumor. On the other hand, a vagal schwannoma doesn’t make any splaying in the vessels, including the internal and external carotid arteries similar to a cervical sympathetic chain ganglion schwannoma. Go to the next page if you knew the correct answer, or click the link image(s) below to further research the concepts in this question (if desired).
Research Concepts: Schwannoma
We update eBooks quarterly and Apps daily based on user feedback. Please tap flag to report any questions that need improvement.
Question 429:
A 17-year-old male presents with complaints of a recent decrease in his peripheral vision. He developed polyuria and polydipsia six months ago and is currently receiving treatment for diabetes insipidus. On neuro-ophthalmologic examination, the patient is alert and oriented with intact extraocular movement. However, there is bitemporal hemianopsia and decreased visual acuity. Computed tomography (CT) scan of the head with and without contrast revealed a sellar tumor with suprasellar extension. Histological examination demonstrated a biphasic cellular population composed of large malignant epithelioid cells and small lymphocytes. Laboratory evaluation of tumor biomarkers showed an alpha-fetoprotein level of 1 ng/L and a beta-human chorionic gonadotropin (hCG) of 0.001 IU/L. Which of the following is the most important factor in the staging of the tumor?
Choices: 1. Evaluation of surgical resection margins 2. Genetic analysis 3. Molecular profiling 4. Neuraxis magnetic resonance imaging (MRI) and cerebrospinal fluid (CSF) cytology
Answer: 4 - Neuraxis magnetic resonance imaging (MRI) and cerebrospinal fluid (CSF) cytology
Explanations: It is critical to determine the extent of disease before initiating treatment because the approach is different for localized versus disseminated germinoma. Neuroimaging with MRI and lumbar puncture for CSF cytology is important for tumor staging. The presence of tumor cells on CSF cytology is an indication of disseminated disease (M+), and patients are treated more aggressively with craniospinal irradiation for better outcomes. There are no recommended TNM-based staging guidelines for intracranial tumors. However, the modified Chang system used for medulloblastoma can be employed in germinoma. Seminomas are highly radiosensitive and chemosensitive with an excellent prognosis. The role of surgical resection in germinoma is not well established. Go to the next page if you knew the correct answer, or click the link image(s) below to further research the concepts in this question (if desired).
Research Concepts: Germ Cell Seminoma
We update eBooks quarterly and Apps daily based on user feedback. Please tap flag to report any questions that need improvement.
Question 430:
A 27-year-old male surfer presents to the clinic with a history of an ulcerated pigmented skin lesion of the right shoulder. He had not noticed any skin changes until the lesion began to bleed some months ago. The ulcerated lesion is excised with 2 cm wide margins. An 18F-FDG PET scan shows two 3 to 4 cm pulmonary lesions on the upper lobe of the right lung. A computed tomography-guided lung biopsy reveals S-100 positive tumor cells. He is prescribed nivolumab. After three cycles, he develops a fever and cough. A thorax computed tomography shows no pulmonary infiltrates and no new metastases. The pre-existing pulmonary metastases appear to have increased about 0.5 cm each. Which of the following is the next best step in managing this patient?
Choices: 1. 2. 3. 4.
Add dacarbazine Switch therapy to atezolizumab Increase the dose of nivolumab Continue current treatment
Answer: 4 - Continue current treatment Explanations: An initial increase in tumor size followed by tumor shrinkage has been observed in the first months of treatment with nivolumab. It is thought to be related to an influx of immune cells into the tumor. Therefore, it is recommended to continue treatment in a clinically stable patient until disease progression is confirmed. Nivolumab was developed by Bristol-Myers Squibb and initially received FDA accelerated approval for refractory, advanced melanoma in December 2014. Nivolumab has been approved to treat many other oncologic conditions, including different kinds of lung, colorectal, hepatocellular, and renal cancer, among others. Nivolumab has a delayed onset of therapeutic effect. This should be considered in patients with rapidly progressing disease. Chemotherapy, in addition to nivolumab, is not approved for melanoma. Atezolizumab is a PD-L1 monoclonal antibody approved for BRAF V600E mutated melanoma combined with cobimetinib and vemurafenib. The monotherapy dose for nivolumab is a fixed dose of 240 mg every two weeks or 480 mg every four weeks. Go to the next page if you knew the correct answer, or click the link image(s) below to further research the concepts in this question (if desired).
Research Concepts: Nivolumab
We update eBooks quarterly and Apps daily based on user feedback. Please tap flag to report any questions that need improvement.
Question 431:
A 63-year-old female presented with a palpable mass in her right breast. The diagnostic mammogram showed a spiculated mass measuring 1.7 cm. Ultrasound demonstrated a hypoechoic mass measuring 1.8 cm x 1.1 cm x 0.9 cm. Biopsy revealed a well-differentiated mucinous carcinoma that was ER/PR positive and HER2/neu negative. Lumpectomy and sentinel node biopsy revealed 1.8 cm mucinous carcinoma with widely negative margins. No associated ductal carcinoma in situ or lymphovascular invasion was noted. Two sentinel nodes were negative. The Oncotype score was 3. What is the treatment recommendation?
Choices: 1. Radiation therapy to the breast and systemic hormonal therapy 2. Radiation therapy to the breast and regional nodes along with systemic hormonal therapy 3. Neo-adjuvant chemotherapy followed by radiation therapy to the axilla 4. Radical mastectomy followed by systemic hormonal therapy
Answer: 1 - Radiation therapy to the breast and systemic hormonal therapy
Explanations: Mucinous carcinoma (MC) is the pathological diagnosis of up to 4% of all invasive breast cancers. MCs are more common in perimenopausal and postmenopausal women. It has a better prognosis compared to ductal or lobular invasive breast cancers. Radiation therapy is indicated following the surgery to limit the risk of local recurrence. Radiation therapy is especially indicated in patients younger than 70 years of age. Whole breast radiotherapy can be delivered in a conventional fractionation schedule (over 6-7 weeks) and hypofractionation schedule (over 3-4 weeks). Hypofractionation is an option for low risk, nodenegative patients with breast cancer who are 50 years or older, not getting chemotherapy, and have a separation of 25 cm or less. Hormonal therapy, such as anastrozole, is recommended in patients with hormone receptorpositive disease. Radical mastectomy is not an indication in this particular case. Go to the next page if you knew the correct answer, or click the link image(s) below to further research the concepts in this question (if desired).
Research Concepts: Mucinous Breast Carcinoma
We update eBooks quarterly and Apps daily based on user feedback. Please tap flag to report any questions that need improvement.
Question 432:
A 20-year-old male presents to the emergency department for altered mental status. Per the patient’s mother, the patient suddenly started sweating and became unresponsive. She reports that he does not have any medical history, surgical history or any significant family history. He does not take any medications. The patient has a temperature of 98.9 degrees Fahrenheit, the pulse is 110 beats per minute, his blood pressure is 110/60 mm hg, blood oxygen saturation is 100% and he is breathing 14 times a minute. On physical examination, he appears lethargic. His heart, lung, and abdominal examination is normal. His skin is moist. His blood glucose level is found to be 20 mg/dL. He is given a dextrose injection intravenously. Within a few minutes, he regains his consciousness with no residual deficits. He denies any complaints. His EKG and troponins are within normal limits. His urine sulphonyl-urea level is zero. The c-peptide levels are 4 ng/ml. The CT scan of his brain was done and was negative for any abnormalities. What is the most likely cause of the patient’s symptoms?
Choices: 1. 2. 3. 4.
Ischemic Stroke Insulinoma Administration of sulphonylurea drugs Myocardial infarction
Answer: 2 - Insulinoma Explanations: In patients with insulinoma, there is excessive endogenous production of insulin from the insulinsecreting beta cells of the pancreas. Patients with insulinoma can present with sweating, palpitations, tremors, confusion, altered mental status, behavioral changes, seizures, syncope as well as coma. C peptide levels are typically elevated in patients with insulinoma. The use of exogenous insulin is contraindicated in this setting. Rapid reversal of symptoms with the administration of glucose goes against ischemic stroke as a cause of the symptoms. No chest pains with normal EKG and troponins rule out myocardial infarction. Since the patient does not take any medications and the urine sulphonylurea level is zero, it rules out the administration of sulphonylurea as a cause of the patient's symptoms. Go to the next page if you knew the correct answer, or click the link image(s) below to further research the concepts in this question (if desired).
Research Concepts: Human Insulin
We update eBooks quarterly and Apps daily based on user feedback. Please tap flag to report any questions that need improvement.
Question 433:
A 67-year-old man presents for a sixmonth history of nonproductive cough and streaky pattern hemoptysis. He discloses a significant weight loss of 20 pounds (9 kg) over the same period. Chest CT scan shows an isolated subcarinal neoplastic lymphadenopathy with a 2.7 × 2.4 cm left lower lobe lung mass. No distant metastatic disease is found. What is the most appropriate next step in management?
Choices: 1. 2. 3. 4.
Radiation therapy Neoadjuvant chemoradiotherapy Left lower lobectomy Radiofrequency ablation
Answer: 3 - Left lower lobectomy Explanations: Involvement of subcarinal lymph node(s) or ipsilateral mediastinal lymph node(s) corresponds to N2 nodal disease. This, together with a 2 cm peripheral lesion and absence of other findings, indicates stage III-A. N2 nodal involvement with tumor staging of T4 upgrades neoplastic disease to stage III-B. If concomitant distant metastases, pericardial dissemination, contralateral lung lesions then stage IV. In non-small cell lung carcinoma, T staging describes the size and location. T1 lesion: tumor size of smaller than 3 cm without the involvement of the main bronchus. T2 lesion: 3-7 cm tumor or evidence of invasion to the visceral pleura, main bronchus, or cause atelectasis extending to the hilum. T3 lesion: larger than 7 cm tumors or involvement of chest wall, phrenic nerve, parietal pericardium. T4 lesion: invasion of nonresectable structures (carina, esophagus, cardiac structures, diaphragm, or recurrent laryngeal nerve). Although the initial treatment of choice of lung adenocarcinoma of stage III might not be surgical resection, those with an isolated lymph node involvement with less than 3 cm size are considered acceptable candidates for surgical resection. N staging involves location: N1 involves ipsilateral peribronchial or hilar nodes. N2 involves ipsilateral mediastinal or subcarinal nodes. N3 involves contralateral nodes as well as scalene or supraclavicular nodes.
Go to the next page if you knew the correct answer, or click the link image(s) below to further research the concepts in this question (if desired).
Research Concepts: Lung Adenocarcinoma
We update eBooks quarterly and Apps daily based on user feedback. Please tap flag to report any questions that need improvement.
Question 434:
An 80-year-old woman with stage IV NYHA class congestive heart failure is found to have a 1.5 cm spiculated nodule in the left upper lobe on chest CT. Biopsy reveals squamous cell carcinoma. PET-CT demonstrates no evidence of metastatic disease. MRI of the brain is negative for intracranial metastases. What is the most appropriate management strategy for this patient?
Choices: 1. 2. 3. 4.
Stereotactic ablative body radiation Chemotherapy Chemoradiation Wedge resection
Answer: 1 - Stereotactic ablative body radiation Explanations: Stereotactic ablative body radiation (SABR) or stereotactic body radiation therapy (SBRT) is recommended for patients with stage I lung cancer who are not surgical candidates or refuse surgery. SABR delivers high doses of radiation therapy to the lung tumor with precision and accuracy. SABR can be delivered in 3 to 5 fractions for peripheral lung tumors. Studies have shown that the 3-year local control rate is 85 to 95%. Go to the next page if you knew the correct answer, or click the link image(s) below to further research the concepts in this question (if desired).
Research Concepts: Radiation Therapy For Early Stage Non-Small Cell Lung Cancer
We update eBooks quarterly and Apps daily based on user feedback. Please tap flag to report any questions that need improvement.
Question 435:
A 58-year-old woman is brought to the hospital with drenching night sweats and fever. On examination, she has palpable non-tender cervical and axillary adenopathy. Labs show evidence of lymphocytosis with an absolute lymphocyte count of 30000/microL, hemoglobin of 12 g/dL, and platelets of 110000/microL. CT scan shows cervical, axillary, and mediastinal lymph node enlargement. Flow cytometry on peripheral blood shows a kappa light chain restricted cell population that expressed CD19, dim CD20, CD5, and CD23 and more strongly expressed BCL2. The FISH panel for CLL/SLL shows a loss of 17p with no abnormality in other FISH probes. Given the likely diagnosis, which patient factor is the strongest indication for treatment in this patient?
Choices: 1. 2. 3. 4.
Her presenting symptoms The 17p deletion An absolute lymphocyte count of 30000/microL The strong expression of BCL-2
Answer: 1 - Her presenting symptoms Explanations: Treatment for (CLL) is indicated in patients with significant clinical symptoms or the presence of laboratory abnormalities such as significant cytopenia and hypogammaglobulinemia. Absolute lymphocyte count is not used as a requisite for treatment. However, rapid lymphocyte doubling time is important, which is not provided in this clinical vignette. Indication for treatment of chronic lymphocytic leukemia is not based on cytogenetics. Although 17p deletion predicts an unfavorable outcome, it does not justify the need for treatment. Go to the next page if you knew the correct answer, or click the link image(s) below to further research the concepts in this question (if desired).
Research Concepts: Chronic Lymphocytic Leukemia With Variant Genetics
We update eBooks quarterly and Apps daily based on user feedback. Please tap flag to report any questions that need improvement.
Question 436:
A 65-year-old female patient presents to the hospital with complaints of severe headaches associated with nausea and vomiting on awakening for the past few months. She also reports loss of weight and loss of appetite. She recalls having radiation therapy ten years ago for some growth, but can not precisely remember the diagnosis. On examination, she is found to have a heart rate of 100 beats/min, a blood pressure of 100/70 mmHg, respiratory rate of 20/min, and 95 % oxygen saturation on room air. Lab values, including CBC and metabolic profile, are within the normal range. CT head shows an irregular mass-like lesion, which was not present in her previous imaging four years ago. MRI revealed a 3 cm irregular mass in the left temporal lobe. Information regarding which of the following is most likely to help in establishing a diagnosis in such patients?
Choices: 1. 2. 3. 4.
Previous radiation Prior surgeries Family history Smoking and alcohol history
Answer: 1 - Previous radiation Explanations: Glioblastoma multiforme is the most common aggressive primary cranial nervous system tumor. Usually more common in older men and African American race. Studies have shown that a previous history of irradiation in early life is a risk factor for GBM. Although, in most cases, risk factors are not identified. It is important to take a careful history, including a prior history of irradiation, to assess for risk factors. No convincing evidence exists regarding associations of GBM with family history, smoking, or alcohol history. Go to the next page if you knew the correct answer, or click the link image(s) below to further research the concepts in this question (if desired).
Research Concepts: Glioblastoma Multiforme
We update eBooks quarterly and Apps daily based on user feedback. Please tap flag to report any questions that need improvement.
Question 437:
A 6-year-old girl is brought in by her parents with a history of weight loss, cough, and shortness of breath. The mother mentions that her daughter was recently prescribed antibiotics for pneumonia. On cardiopulmonary examination, there is sinus tachycardia. An x-ray of the chest shows a mass-like opacity in the right upper lung area. A CT scan of the chest shows multiple masses arising from the right lung pleura. On further inquiry, she has a family history of DICER-1 gene mutation. For which of the following pathologies should this patient be screened?
Choices: 1. 2. 3. 4.
Cardiomegaly Liver cirrhosis Ovarian tumor Melanoma
Answer: 3 - Ovarian tumor Explanations: The DICER-1 gene mutation causes an increased risk for pleuropulmonary blastoma (PPB) and a variety of other conditions such as lung and kidney cysts or tumors, ovarian tumors in children or adults, thyroid nodules, or cancers, brain tumors, and tumors in the eye or nose and sinuses. When a DICER-1 gene mutation is present, specific screening studies are recommended to look for PPB and other conditions known to be related to DICER-1. When found early, the ovarian tumors associated with DICER-1 may be removed with surgery alone and are associated with a favorable prognosis. DICER-1 specific surveillance guidelines are available for anyone with PPB, another DICER1-related condition, or variation in the DICER-1 gene. Go to the next page if you knew the correct answer, or click the link image(s) below to further research the concepts in this question (if desired).
Research Concepts: Pleuropulmonary Blastoma
We update eBooks quarterly and Apps daily based on user feedback. Please tap flag to report any questions that need improvement.
Question 438:
A 65-year-old female patient presents to the hospital with a 2-month history of facial flushing, nausea, abdominal pain, diarrhea, and wheezing. The patient has no significant medical history and takes no medications. The patient’s blood pressure is 124/70 mmHg, pulse 60/min, respirations 12/min, and temperature 98.0 F (36.7 C). Chest auscultation shows wheezing bilaterally. A 24-hour urine collection shows elevated levels of 5Hydroxyindoleacetic acid (5-HIAA). The best treatment for this patient’s condition is also used in the treatment of which of the following conditions?
Choices: 1. 2. 3. 4.
Acromegaly Rheumatoid arthritis Breast cancer Short stature
Answer: 1 - Acromegaly Explanations: Octreotide has been shown to decrease complications of carcinoid. It is also used to treat acromegaly. Octreotide is available as short-acting subcutaneous injection as well as depot form intramuscular injection (Sandostatin LAR) which can be administered monthly. Patients should start with 20 mg to 30 mg IM every four weeks, and a gradual dose increase may be necessary. Shortacting octreotide can be started for a patient with severe or refractory symptoms. For medical management, there are two somatostatin analogs available, Octreotide and Lanreotide. Somatostatin is an amino acid peptide which is an inhibitory hormone, which is synthesized by paracrine cells located ubiquitously throughout the gastrointestinal tract. It inhibits the release of most of the gastrointestinal, endocrine hormones. About 80% of neuroendocrine tumors have somatostatin receptors. Using somatostatin analog inhibits the release of biogenic amines which leads to control of symptoms such as flushing and diarrhea. Carcinoid syndrome is diagnosed with elevated 24-hour urine 5-HIAA. Treatment options include octreotide or lanreotide. The most common side effects associated with somatostatin analogs are nausea, abdominal bloating, and steatorrhea, which is due to pancreatic malabsorption. Supplementing pancreatic enzymes usually helps to alleviate adverse symptoms.
Go to the next page if you knew the correct answer, or click the link image(s) below to further research the concepts in this question (if desired).
Research Concepts: Carcinoid Syndrome
We update eBooks quarterly and Apps daily based on user feedback. Please tap flag to report any questions that need improvement.
Question 439:
A 53-year-old woman is diagnosed with a triple-negative, node-positive invasive ductal carcinoma of the right breast. She undergoes neoadjuvant chemotherapy followed by a modified radical mastectomy. She then undergoes post-mastectomy radiation to the right chest wall and regional lymphatics. Four months following treatment, she notices the development of a small draining superficial ulcerated area on the right chest wall. Despite conservative measures for several months, the wound fails to heal and starts to enlarge. What is the most appropriate treatment?
Choices: 1. 2. 3. 4.
Pentoxifylline and vitamin E Hyperbaric oxygen therapy Application of growth factor to the wound Myocutaneous flap
Answer: 4 - Myocutaneous flap Explanations: Skin injuries ranging from erythema to full chronic wounds occur in about 95% of patients who undergo radiation therapy. Radiation-induced, nonhealing skin ulcers can be difficult to treat medically. Mild chest wall ulcers often can be successfully treated with conservative therapy. However, moderate to severe lesions usually require debridement followed by repair with myocutaneous flaps or well-vascularized local axial-pattern flaps. Go to the next page if you knew the correct answer, or click the link image(s) below to further research the concepts in this question (if desired).
Research Concepts: Radiation Therapy Induced Skin Ulcer
We update eBooks quarterly and Apps daily based on user feedback. Please tap flag to report any questions that need improvement.
Question 440:
A 50-year-old male patient presents in the outpatient clinic with complaints of generalized malaise, muscle aches, and significant weight loss during the last six months. He works as a manager in a bank, and he often finds it hard to get up from his chair in the office for the last couple of months. There is no history of cough, shortness of breath, or abdominal discomfort. There is no history of previous diseases or drug abuse. He had a mild sore throat one week ago. On examination, multiple lymphadenopathies, both above and below the diaphragm, where noted, and the bulk and the power were also reduced in pelvic girdle muscles. Blood labs are pending. Lymph node biopsy is also performed; results are awaited. At this stage, the patient was diagnosed with polymyositis to explain his muscle aches and malaise. Based on the presentation, what could be the most probable reason in this patient for the development of polymyositis?
Choices: 1. 2. 3. 4.
Viral infection Old age A lymphoma Iatrogenic
Answer: 3 - A lymphoma Explanations: Polymyositis (PM) is a rare and chronic autoimmune disorder. Polymyositis (PM) is one of the inflammatory myopathies and disorders characterized pathologically by inflammatory infiltrates in striated muscle. PM develops due to abnormal activation of cytotoxic T lymphocytes (CD8 Cells) and macrophages against the unknown muscular antigens and the strong extrafusal muscular expression of major histocompatibility complex 1, causing damage to the endomysium of skeletal muscles. An important etiological factor for polymyositis-induced rhabdomyolysis is the underlying malignancy, e.g., lymphomas, especially the non-Hodgkin lymphoma. The presence of multiple lymphadenopathies (LAD) and the absence of other etiological factors make lymphoma the most probable cause for polymyositis in this patient. Go to the next page if you knew the correct answer, or click the link image(s) below to further research the concepts in this question (if desired).
Research Concepts: Polymyositis
We update eBooks quarterly and Apps daily based on user feedback. Please tap flag to report any questions that
need improvement.
Question 441:
A 65-year-old female presented to the clinic with a mass in her right breast. Examination revealed a hard, immobile 5 by 2 cm mass in the left upper quadrant of the breast. On ultrasound, the mass was hypoechoic with irregular borders and septations. An excisional biopsy was done, and the sample was found to be ER+. The patient underwent a radical mastectomy bilaterally. Which of the following is most appropriate for adjuvant therapy in this patient?
Choices: 1. 2. 3. 4.
Finasteride Fadrozole Clomiphene citrate Letrozole
Answer: 4 - Letrozole Explanations: Adjuvant chemotherapy prevents breast cancer recurrence and is indicated in postmenopausal women with hormone receptive breast cancer with a mass of >1 cm at diagnosis or evidence of metastasis. An increase in cells that express ER has been shown to be present in a majority of breast cancers. Therefore, a variety of pharmacologic agents have been developed to decrease estrogen production and activity. Selective Estrogen Receptor Modulators (like Tamoxifen) and Aromatase Inhibitors (like Anastrozole) are two drug classes approved for their use in adjuvant chemotherapy for breast cancer because of their mechanisms related to decreasing estrogen levels. Letrozole and anastrozole are both third-generation aromatase inhibitors that are the most specific towards targeting aromatase and are the most appropriate drugs of choice when choosing aromatase Inhibitors. Fadrozole is a second-generation aromatase inhibitor that exhibited less specificity for the enzyme aromatase and has an increase of adverse side effects such as nausea, vomiting, rashes, etc in comparison to thirdgeneration aromatase inhibitors. Clomiphene citrate is a drug that induced ovulation in patients with infertility or PCOS. Go to the next page if you knew the correct answer, or click the link image(s) below to further research the concepts in this question (if desired).
Research Concepts:
Aromatase Inhibitors
We update eBooks quarterly and Apps daily based on user feedback. Please tap flag to report any questions that need improvement.
Question 442:
A 16-year-old male is diagnosed with a germ cell tumor of the testis and is started on a chemotherapy combination containing ifosfamide. The patient develops gross hematuria a week after completion of chemotherapy and reports urinary urgency, frequency, difficulty emptying his bladder, and mild suprapubic pain. Examination demonstrates suprapubic fullness and discomfort with palpation. Urinalysis confirms hematuria. Ultrasonography of the bladder shows several blood clots. Which of the following measures would most likely have prevented this patient's suspected condition?
Choices: 1. Slow administration of ifosfamide 2. Administration of sodium bicarbonate before ifosfamide administration 3. Administration of mesna before, during, and after ifosfamide administration 4. Aggressive hydration before ifosfamide administration
Answer: 3 - Administration of mesna before, during, and after ifosfamide administration
Explanations: This clinical vignette depicts a patient with hemorrhagic cystitis secondary to ifosfamide treatment. Cyclophosphamide and methotrexate can also cause hemorrhagic cystitis. Before the availability of mesna, the main adverse effect of ifosfamide was hemorrhagic cystitis. Sodium 2mercaptoethane sulfonate (mesna) was developed to help with this adverse effect. Mesna combines with the urotoxic metabolites from the liver (acrolein) at the level of the urothelium, leading to a non-toxic product for excretion in the urine. It is always important to attempt to prevent the development of cystitis rather than to treat it. Mesna should be administered before, during, and after ifosfamide, along with aggressive hydration during administration (at least 2 liters oral or IV) to prevent urotoxicity. Go to the next page if you knew the correct answer, or click the link image(s) below to further research the concepts in this question (if desired).
Research Concepts: Ifosfamide
We update eBooks quarterly and Apps daily based on user feedback. Please tap flag to report any questions that need improvement.
Question 443:
A 38-year-old woman patient presents with seizures. She has been complaining of progressive headaches for the past month, accompanied by nausea. For the past two days, she was drowsy and has decreased appetite. She denies any fever or infection. She has a past medical history of breast cancer diagnosed two years ago and had a radical mastectomy, chemotherapy, and radiation therapy. Vital signs show oxygen saturation 98% on room air, respiratory rate 18 per minute, heart rate 96 beats per minute, blood pressure 140/90 mmHg, and temperature 98.5 F. On examination, she opens her eyes to pain, has an incomprehensible verbal response, and localizes to pain. There are intermittent episodes of preferential gaze to the right with tonic-clonic movements of the right upper extremity with secondary generalization. Babinski sign is positive bilaterally. A head CT is normal. Which of the following are the most likely findings on the cerebrospinal fluid (CSF) examination?
Choices: 1. Elevated opening pressure, WBC 3 cells/mm3, 100% lymphocytes, glucose 55 mg/dL, protein 150 mg/dL 2. Elevated opening pressure, WBC 2500 cells/mm3, 90% neutrophils, glucose 23 mg/dL, protein 250 mg/dL 3. Normal opening pressure, WBC 68 cells/mm3, 100% lymphocytes, glucose 55 mg/dL, protein 65 mg/dL 4. Normal opening pressure, WBC 5 cells/mm3, 100% lymphocytes, glucose 65 mg/dL, protein 40 mg/dL
Answer: 1 - Elevated opening pressure, WBC 3 cells/mm3, 100% lymphocytes, glucose 55 mg/dL, protein 150 mg/dL
Explanations: Malignancy from the breast, lung, melanoma, or even a primary brain cancer may cause carcinomatous meningitis. The survival rate is only a few weeks at best. The most common manifestations are headaches and seizures. Lumbar puncture is the standard diagnostic procedure. Common findings include elevated opening pressure and elevated protein levels. Malignant cells are seen in 60% to 70% of cases after three attempts. The contrast-enhanced MRI is the imaging of choice as a CT scan may not be sensitive enough to pick up leptomeningeal enhancement. Go to the next page if you knew the correct answer, or click the link image(s) below to further research the concepts in this question (if desired).
Research Concepts: Carcinomatous Meningitis
We update eBooks quarterly and Apps daily based on user feedback. Please tap flag to report any questions that need improvement.
Question 444:
A 65-year-old man presents to the clinic with a complaint of gross bloody urine for the past 4 days. He also complains of night sweats, mild fever, and unintentional weight loss of 5 pounds (2 kg) for the past 1 month. The patient has no medical condition and does not take any medicine. He has a 40-pack-year smoking history but quit smoking 6 months ago. The patient's vitals and physical examination are normal. The patient's urinalysis reports gross hematuria with many normal-appearing red blood cells. No dysmorphic red blood cells or red cell casts are observed. Blood urea nitrogen is 19 mg/dl, and creatinine is 0.8 mg/dl. Which of the following is the best next step to rule out any malignancy of the bladder?
Choices: 1. 2. 3. 4.
CT without contrast Cystoscopy Urine cytology Bladder ultrasound
Answer: 2 - Cystoscopy Explanations: Bladder cancer is the most common cause of gross hematuria in patients aged >35 years with a smoking history. The hematuria is often gross and painless but maybe only microscopic. Other symptoms of bladder cancer include dysuria and urinary urgency and frequency. Suspected patients should undergo contrast CT scan and cystoscopy to rule out malignancy in the bladder and urethra. Bladder cancer is most closely associated with smoking due to carcinogenic agents from cigarette smoke being removed from the general circulation by the kidneys and then being excreted via the urine, which is temporarily stored in the urinary bladder. This exposes the bladder mucosal lining to concentrated levels of highly carcinogenic agents, resulting in bladder cancer. Go to the next page if you knew the correct answer, or click the link image(s) below to further research the concepts in this question (if desired).
Research Concepts: Bladder Cancer
We update eBooks quarterly and Apps daily based on user feedback. Please tap flag to report any questions that
need improvement.
Question 445:
A 58-year-old male presents with a history of a dry cough and hemoptysis. Vital signs are stable and the patient is afebrile. Physical exam reveals mild wheezing. A chest x-ray reveals a central mass without distant metastases. What is the most likely cause of the patient's condition?
Choices: 1. 2. 3. 4.
Squamous cell carcinoma Adenocarcinoma Small cell carcinoma Large cell carcinoma
Answer: 1 - Squamous cell carcinoma Explanations: Squamous cell lung cancer is most likely to present as a large, central mass without distant metastases. About 30% of all lung cancers are squamous cell carcinoma (SCC) and centrally located. They may be associated with hypercalcemia. Centrally located tumors may present with a dry cough, hemoptysis, wheezing, or post-obstructive pneumonia. Bronchoscopy is invaluable in the diagnosis of SCC. SCC tends to be locally invasive and positive lymph nodes are common. Hence, a CT scan of the chest is mandatory during work up to assess for mediastinal nodes. Go to the next page if you knew the correct answer, or click the link image(s) below to further research the concepts in this question (if desired).
Research Concepts: Squamous Cell Lung Cancer
We update eBooks quarterly and Apps daily based on user feedback. Please tap flag to report any questions that need improvement.
Question 446:
A 65-year-old patient has end-stage uterine cancer. Which of the following organisms is most likely to be the cause of a urinary tract infection in this patient?
Choices: 1. 2. 3. 4.
Streptococcus agalactiae Hemophilus influenza Staphylococcus epidermidis Pseudomonas aeruginosa
Answer: 4 - Pseudomonas aeruginosa Explanations: Pseudomonas aeruginosa is responsible for most urinary tract infections in immunocompromised patients. Most UTIs by Pseudomonas are nosocomial and associated with surgery, instrumentation, and catheterization. Pseudomonas infections of the urinary tract usually require IV antibiotics. Quinolones and aminoglycosides can be used. Long-term treatment is required in septic patients, especially those with indwelling catheters. Go to the next page if you knew the correct answer, or click the link image(s) below to further research the concepts in this question (if desired).
Research Concepts: Urinary Tract Infection
We update eBooks quarterly and Apps daily based on user feedback. Please tap flag to report any questions that need improvement.
Question 447:
Which of the following would be recommended for an asymptomatic patient in whom a small tubular adenoma was found and completely removed on colonoscopy?
Choices: 1. 2. 3. 4.
Colonoscopy in 1 year Repeat sigmoidoscopy in 5 years Colonoscopy in 5 years Contrast abdominal CT
Answer: 3 - Colonoscopy in 5 years Explanations: Tubular adenomas are precancerous lesions. Larger tubular adenomas have an increased likelihood of malignant degeneration and colonic invasion. About a third of patients have cancerous lesions proximally in the colon, so colonoscopy should be performed. The diagnosis of one small tubular adenoma mandates repeat colonoscopy at least within 5 years. Go to the next page if you knew the correct answer, or click the link image(s) below to further research the concepts in this question (if desired).
Research Concepts: Tubular Adenoma
We update eBooks quarterly and Apps daily based on user feedback. Please tap flag to report any questions that need improvement.
Question 448:
A 54-year-old female patient currently undergoing radiation and chemotherapy for stage 3 breast cancer presents to the clinic for a follow-up examination. She complains of a one-week history of nausea and vomiting. Her blood pressure is 120/80 mmHg, pulse 80/min, respiratory rate 14/min, and temperature 98.0 F (36.7 C). Her provider recommends a strong antiemetic that is considered a cornerstone in the management of chemotherapy-induced nausea and vomiting. This medication acts on a neurotransmitter produced in which location within the central nervous system?
Choices: 1. 2. 3. 4.
Locus ceruleus Raphe nuclei Adrenal medulla Substantia nigra
Answer: 2 - Raphe nuclei Explanations: Ondansetron antagonizes 5-HT3, a subtype of serotonin receptor. Ondansetron is used for the prevention of postoperative and chemotherapy-induced nausea and vomiting in adults and the prevention of chemotherapy-induced nausea and vomiting in children. Ondansetron prevents serotonin from binding in the gastrointestinal tract, preventing nausea and vomiting, and increasing extracellular serotonin levels. Ondansetron is a 5-HT3 serotonin antagonist. Serotonin is produced primarily by the raphe nuclei, a cluster of nuclei in the brainstem, composing the medial portion of the reticular formation. Go to the next page if you knew the correct answer, or click the link image(s) below to further research the concepts in this question (if desired).
Research Concepts: Antiemetic Serotonin-5-HT3 Receptor Blockers
We update eBooks quarterly and Apps daily based on user feedback. Please tap flag to report any questions that need improvement.
Question 449:
A 28-year-old female presents to the clinic with complaints of 15 mucocutaneous lesions on various parts of her body in addition to the tongue and lips. She also has an anterior neck swelling. The biopsy report reveals follicular carcinoma of the thyroid. Physical examination reveals skin tags in the armpits and neck and enlarged size of the head in addition to diffuse, painless, moveable swellings on her upper back and the shoulder regions. The patient's father states that her mental condition is worsening day by day, and she is unable to perform daily household tasks properly. Which of the following carcinomas is this patient most likely at risk of developing?
Choices: 1. 2. 3. 4.
Lymphoma Leukemia Spine cancer Breast cancer
Answer: 4 - Breast cancer Explanations: The presence of greater than six mucocutaneous lesions, lipomas, thyroid cancer, and mental retardation in this patient are suggestive of Cowden disease. The majority of patients affected with the disease go on to develop a malignant neoplasm of the thyroid, endometrium, or breast. Females with Cowden disease are at a much higher risk of breast carcinoma, with about 85 percent of females with Cowden syndrome developing breast carcinoma at some time in their life. Interestingly, breast carcinoma has also been reported in men. Benign fibrocystic disease and fibroadenomas are also commonly seen in females. Go to the next page if you knew the correct answer, or click the link image(s) below to further research the concepts in this question (if desired).
Research Concepts: Cowden Disease
We update eBooks quarterly and Apps daily based on user feedback. Please tap flag to report any questions that need improvement.
Question 450:
A 70-year-old female presents with an infection of her right index finger. She states she was a house cleaner for years but has been retired for a few years now. Over the past year, she has noticed an area of redness and swelling around her fingernail and that the nail plate has become thickened and discolored. Which of the following is the most likely diagnosis?
Choices: 1. 2. 3. 4.
Onychomycosis Chronic paronychia Squamous cell carcinoma Medication side effect
Answer: 2 - Chronic paronychia Explanations: Streptococci and Pseudomonas are more common in chronic infections. In chronic paronychia, the nail fold may be red and swollen, but fluctuance is rare. The nail fold may appear boggy, and the nail plate can become thickened and discolored. Occupation and working environment are critical history findings; homemakers, bartenders, and dishwashers seem predisposed to developing chronic paronychia. If a chronic paronychia is not improving with the usual treatment options, further testing should be done to look for other serious causes, such as squamous cell carcinoma. Go to the next page if you knew the correct answer, or click the link image(s) below to further research the concepts in this question (if desired).
Research Concepts: Paronychia
We update eBooks quarterly and Apps daily based on user feedback. Please tap flag to report any questions that need improvement.
Question 451:
A 65-year-old male came to the clinic with complaints of backbone pain, fever, fatigue and weight loss for the past 3 months. He has a 20-pack-year smoking history. His past medical problems include benign prostatic hyperplasia and hypertension. His medications include amlodipine and prazosin. He does not drink alcohol. His temperature is 99.8 F (37.6 C), blood pressure is 145/80 mmHg, the pulse is 88 bpm and respirations are 21/min. CBC shows decreased Hb, increased ESR, and mild elevation of creatinine. X-ray of the lumbar spine shows lytic lesions. He is started on combination therapy of bortezomib, dexamethasone, and cyclophosphamide. He reported improvement of symptoms 2 months later. On follow up 8 months later he comes with recurrence of symptoms. The Xray findings have worsened and he has had 3 UTI episodes. Which of the following is the most appropriate regimen at this stage?
Choices: 1. 2. 3. 4.
Tositumomab Leflunomide Bevacizumab Bortezomib and dexamethasone
Answer: 4 - Bortezomib and dexamethasone Explanations: The patient who is given in this vignette likely has multiple myeloma as shown by his symptoms, age, radiographic and laboratory findings. Multiple myelomas (MM) are a clonal plasma cell proliferative disorder. Its peak prevalence is ages in between 50-70 years. Chemotherapy is the initial mode of treatment in this patient. He was given the appropriate treatment. He showed improvement but soon relapsed after 6 months. The next step would be to start bortezomib and dexamethasone as they are FDA-approved for patients who initially responded to bortezomib or relapsed after 6 months. Bortezomib is a proteasome inhibitor. It is also used for the treatment of relapsed or refractory mantle cell lymphoma and Waldenstrom macroglobulinemia. Bortezomib reversibly binds to the chymotrypsin-like subunit of the 26S resulting in inhibition. of proteasomes. Side-effects include peripheral neuropathy, diarrhea, nausea, thrombocytopenia, hypotension, and dyspnea. Bevacizumab binds VEGF and inhibits endothelial growth and proliferation. It is used in a variety of cancers. Leflunomide is an immunosuppressive agent used as a DMARD in rheumatoid arthritis. Go to the next page if you knew the correct answer, or click the link image(s) below to further research the concepts in this question (if desired).
Research Concepts:
Bortezomib
We update eBooks quarterly and Apps daily based on user feedback. Please tap flag to report any questions that need improvement.
Question 452:
A 29-year-old man with a recently diagnosed non-seminomatous germ cell tumor of the left testis presents to the clinic for follow-up. His clinical stage based on evaluation is CS II-A with normal tumor markers. He is given four cycles of BEP (bleomycin, etoposide, and cisplatin) chemotherapy. Post chemotherapy evaluation reveals a 2.5 cm mass lesion in para-aortic nodes just below the renal hilum. Which of the following is the next best step in the management of this patient?
Choices: 1. 2. 3. 4.
Radical radiotherapy Surveillance Salvage chemotherapy Retroperitoneal lymph node dissection (RPLND)
Answer: 4 - Retroperitoneal lymph node dissection (RPLND)
Explanations: Patients with CS II-A who undergo chemotherapy are classified into the following categories based on their response to chemotherapy: 1. Complete response (CR), defined by normalization of serum tumor markers and resolution of radiographic disease (usually defined as residual masses =1 cm). 2. Normalization of serum tumor markers with persistent radiographic tumor (partial remission–marker negative). 3. Partial remission–marker positive. 4. Disease progression. Approximately 5 to 15% of patients fall into categories 3 and 4 and are typically managed with second-line (also termed salvage) chemotherapy. Between 38 and 68% of patients have residual masses larger than 1 cm after first-line chemotherapy, and there is a clear consensus that they should undergo postchemotherapy surgery (PCS). Post chemotherapy surgery (RPLND) is a standard treatment in the United States for NSGCT. Go to the next page if you knew the correct answer, or click the link image(s) below to further research the concepts in this question (if desired).
Research Concepts: Nonseminomatous Testicular Tumors
We update eBooks quarterly and Apps daily based on user feedback. Please tap flag to report any questions that need improvement.
Question 453:
A 65-year-old man with a past medical history of type 2 diabetes mellitus and hypertension presents to the emergency department with confusion, headache, and fatigue. The patient has a history of mild shortness of breath for the past few weeks and a persistent cough that has been ongoing for many years. He states that he has been smoking 2 packs of cigarettes per day since age 14. His physical examination reveals paradoxical chest wall motion when the patient takes a deep breath in and out. His investigations reveal a serum sodium of 120 mEq/l. What is the most likely etiology of his condition?
Choices: 1. Phrenic nerve compression due to small cell lung cancer 2. Phrenic nerve compression due to diffuse lung adenocarcinoma 3. Vagus nerve compression due to small cell lung cancer 4. Vagus nerve compression due to diffuse lung adenocarcinoma
Answer: 1 - Phrenic nerve compression due to small cell lung cancer
Explanations: He has small cell lung carcinoma. The diaphragm is mainly innervated by the phrenic nerve. The phrenic nerve is composed of fibers from C3-C5. The most common causes of phrenic nerve injury include open-heart surgery and trauma. A relatively less common cause of phrenic nerve injury includes compression by a lung tumor. The most common lung tumor that can cause hyponatremia in a smoker is small cell lung cancer. Go to the next page if you knew the correct answer, or click the link image(s) below to further research the concepts in this question (if desired).
Research Concepts: Anatomy, Abdomen and Pelvis, Diaphragm
We update eBooks quarterly and Apps daily based on user feedback. Please tap flag to report any questions that need improvement.
Question 454:
A 67-year-old man is brought to the hospital with headaches, confusion, and cherry red skin. He was rescued out of a burning building. His wife states, he was recently diagnosed with multiple myeloma and started on doxorubicin. After several weeks into starting medical therapy for multiple myeloma, the patient began to experience orthopnea, dyspnea upon exertion, and fatigue. Upon physical examination, the clinician appreciates an S3 heart sound, new systolic regurgitant murmur, and jugular venous distention, and diffuse first-degree burns. Laboratory and diagnostic studies include reduced ejection fraction, dilated ventricles, and a widened QRS complex. Which one of the following therapeutic options is most strongly contraindicated to use in this patient's clinical status?
Choices: 1. 2. 3. 4.
100% oxygen therapy Continuous positive airway pressure Methylene blue Hyperbaric oxygen therapy
Answer: 4 - Hyperbaric oxygen therapy Explanations: With a recent clinical diagnosis of multiple myeloma, this patient is most likely taking the FDA-approved drug doxorubicin. Doxorubicin is frequently listed as a contraindication to hyperbaric oxygen therapy (HBO). A group of rats who received HBO after doxorubicin injection experienced an 87% mortality rate, which researchers attributed to the formation of free radicals by both HBO and doxorubicin. While the results of this single study suggest that the concurrent administration of HBO and doxorubicin may be associated with increased mortality. Dexrazoxane, an iron chelator, may be co-administered with doxorubicin to reduce the cardiotoxicity of the drug. The liposomal formulation of doxorubicin is FDAapproved for the treatment of multiple myeloma, ovarian cancer in patients who have failed platinumbased chemotherapy, and AIDS-related Kaposi sarcoma. Go to the next page if you knew the correct answer, or click the link image(s) below to further research the concepts in this question (if desired).
Research Concepts: Doxorubicin
We update eBooks quarterly and Apps daily based on user feedback. Please tap flag to report any questions that need improvement.
Question 455:
A 66-year-old woman with metastatic breast cancer presents to the clinic with a 5-month history of severe pain in her back, upper arms, and legs. She has been prescribed increasing doses of modified-release oral morphine and paracetamol, and while this combination provides her some relief, she is troubled by drowsiness, pruritus, and constipation. At times she feels this is more distressing than her initial pain. Additionally, she takes warfarin for atrial fibrillation. Which of the following is the next best step in the management of this patient?
Choices: 1. 2. 3. 4.
Add diclofenac Add naloxone Reduce the dose of morphine Opioid rotation
Answer: 3 - Reduce the dose of morphine Explanations: Pain is a common symptom amongst those suffering from malignant disease, and its management can be challenging. The etiology of pain in cancer is multifactorial. Opioids are very effective in the management of cancerrelated pain in general. To begin, most patients are prescribed longer-acting oral opioids to treat background pain, supplemented with additional doses to treat breakthrough pain. Long-term opioid use is associated with significant unwanted effects. These include dependence, tolerance, drowsiness, constipation, nausea, vomiting, pruritus, urinary retention, and respiratory depression. If the adverse effects of opioids cause the patient notable distress or discomfort, they warrant specific treatment. In the first instance, the dose of opioids should be reduced. Sometimes, patients may report a paradoxical worsening of their pain. This is a phenomenon known as opioid-induced hyperalgesia. Unwanted side effects will usually settle as tolerance develops. However, at times they may be more distressing than the pain itself, as in this scenario. Go to the next page if you knew the correct answer, or click the link image(s) below to further research the concepts in this question (if desired).
Research Concepts:
Opioid Analgesics
We update eBooks quarterly and Apps daily based on user feedback. Please tap flag to report any questions that need improvement.
Question 456:
A 17-year-old female patient presents to the emergency room with progressively worsening headaches, nausea, and vomiting for the past week. The patient is alert and oriented. She has difficulty looking up, the pupils are unreactive to light, and he has nystagmus with upward gaze. He is unable to close eyelids completely, and the eyes have a “sun-setting” appearance. There are no other motor or sensory deficits. Heart rate is 74/minute, and blood pressure is 116/ 78 mm Hg. What is the next step in the management of this patient?
Choices: 1. Ophthalmology consult 2. High dose steroids 3. Brain magnetic resonance imaging with and without contrast 4. Ventriculostomy placement
Answer: 3 - Brain magnetic resonance imaging with and without contrast
Explanations: Magnetic Resonance Imaging with intravenous contrast is the gold standard for evaluation of pineal region tumors Head Computed Tomography (CT) scan can show a pineal region mass but does not give enough radiological detail as compared to brain Magnetic Resonance Imaging (MRI). A patient with upward gaze palsy is caused by a lesion in the rostral interstitial nucleus of medial longitudinal fasciculus (riMLF). This is a common finding in patients with pineal region tumors. Proper diagnostic imaging studies are necessary before treating or consulting a hemodynamically stable patient. Go to the next page if you knew the correct answer, or click the link image(s) below to further research the concepts in this question (if desired).
Research Concepts: Pineal Gland Cancer
We update eBooks quarterly and Apps daily based on user feedback. Please tap flag to report any questions that need improvement.
Question 457:
A 35-year-old man presents to the clinic with complaints of fatigue, lethargy, and weight loss. The patient states his symptoms have been present for the past 6 months. On examination, there is the presence of an enlarged lymph node in the anterior cervical chain. A fineneedle aspiration biopsy shows variable-sized closely packed follicles containing small cleaved cells without nucleoli and larger noncleaved cells with moderate cytoplasm, open chromatin, and multiple nucleoli. Minimal apoptotic cells and tingible body macrophages are also present. The specimen is sent for flow cytometry. Which of the following sets of findings is most likely to be seen in this patient?
Choices: 1. 2. 3. 4.
Kappa Kappa Kappa Kappa
light light light light
chain chain chain chain
restriction restriction restriction restriction
and and and and
CD10 positive CD3 positive CD5 positive CD5 positive
Answer: 1 - Kappa light chain restriction and CD10 positive
Explanations: The histology findings described in the vignette suggest that this patient has follicular lymphoma (FL). Flow cytometry in follicular lymphoma shows restriction of kappa light chain. Positive stains supporting FL include CD10, CD19, CD20 (strong), CD79a, BCL2 within follicles, and BCL6. Immunoglobulin light chain (LC) restriction is detected in the majority of B-cell non-Hodgkin lymphoma. Go to the next page if you knew the correct answer, or click the link image(s) below to further research the concepts in this question (if desired).
Research Concepts: Follicular Lymphoma
We update eBooks quarterly and Apps daily based on user feedback. Please tap flag to report any questions that need improvement.
Question 458:
A 65-year-old man with metastatic melanoma is prescribed combination therapy with nivolumab and a CTLA-4 inhibitor for four doses, followed by a nivolumab monotherapy. Which of the following checkpoint inhibitors is most appropriate to be administered simultaneously with nivolumab in this patient?
Choices: 1. 2. 3. 4.
Pembrolizumab Ipilimumab Dupilumab Cemiplimab
Answer: 2 - Ipilimumab Explanations: Nivolumab is FDA approved for metastatic, inoperable melanoma (and renal cell cancer) alone or combined with ipilimumab. Nivolumab and ipilimumab have a similar mechanism of action as checkpoint inhibitors but act on different targets. Whereas nivolumab inhibits PD-1, ipilimumab targets CTLA-4 expressed in lymphocytes. Ipilimumab is the only other monoclonal antibody approved for use in combination with nivolumab. In nivolumab and ipilimumab combination therapy, the infusions are given on the same day separately. First, nivolumab is given, followed by ipilimumab. Separate infusion bags and filters are to be used for each infusion. The dosage for the combination therapy is Nivolumab 1 mg/kg over 30 minutes and 3 mg/kg over 90 minutes every three weeks for a total of 4 doses. The treatment then continues as a fix-dose nivolumab monotherapy. Pembrolizumab and cemiplimab are also monoclonal antibodies that target PD-1. Dupilumab is a monoclonal antibody that targets IL-4/IL-13 receptors. Go to the next page if you knew the correct answer, or click the link image(s) below to further research the concepts in this question (if desired).
Research Concepts: Nivolumab
We update eBooks quarterly and Apps daily based on user feedback. Please tap flag to report any questions that need improvement.
Question 459:
A 22-year-old man with a prior diagnosis of cancer is now found to have metastatic lesions in the lung, peritoneum, and right adrenal gland. The patient has a history of a large left hepatic mass for which he underwent resection without chemotherapy. In what percentage of patients do distant metastases occur in this type of primary liver cancer?
Choices: 1. 2. 3. 4.
20-30% 33-100% 50-60% 5-10%
Answer: 1 - 20-30% Explanations: The most common sites of distant metastases in fibrolamellar hepatocellular carcinoma (HCC) are lung, peritoneum and adrenal glands which occur at 20-30%. Disease recurrence is high after hepatic resection of fibrolamellar HCC with a recurrence rate of 33-100%. Local nodal metastases are most commonly seen near the hepatic hilum and near the hepatoduodenal ligament. The incidence can be up to 50 to 60% of cases. Disease recurrence does happen in patients with resectable FL-HCC. Recurrence is more commonly extrahepatic. In patients that have received treatment, the 5-year survival rates can be as high as 80%, and survival rates after 5-years can range from 18 to 50%. In patients, without surgery, the 5-year survival rate is 0. Go to the next page if you knew the correct answer, or click the link image(s) below to further research the concepts in this question (if desired).
Research Concepts: Fibrolamellar Hepatocellular Carcinoma
We update eBooks quarterly and Apps daily based on user feedback. Please tap flag to report any questions that
need improvement.
Question 460:
An older female has routine labs drawn at the time of her physical. She has a history of hypertension and is taking hydrochlorothiazide, calcium, and vitamin D. Her physical is normal. All her labs are normal except her alkaline phosphatase is three times normal. Her Gamma-glutamyl transpeptidase is normal. Which of the following should receive an x-ray first to diagnose the patient's conditions?
Choices: 1. 2. 3. 4.
Spine Ribs Tibia Scapula
Answer: 1 - Spine Explanations: Paget is often multifocal with a predilection for the axial skeleton. It commonly affects the spine, followed by the pelvis, femur, and skull. Once the disease has started, Paget disease does not spread from bone to bone. However, the bony disease is progressive and debilitating. Lower vertebral involvement in patients with Paget disease can be associated with cauda equina syndrome and nerve root compression. Go to the next page if you knew the correct answer, or click the link image(s) below to further research the concepts in this question (if desired).
Research Concepts: Paget Disease
We update eBooks quarterly and Apps daily based on user feedback. Please tap flag to report any questions that need improvement.
Question 461:
A 35-year-old nulliparous woman presents for a check-up. She currently has no symptoms. She does not smoke or ingest illicit drugs. She drinks one glass of wine after dinner every night. Her vitals are within the normal range. She is sexually active with her husband and uses contraceptive pills. Menarche was at 14. Menses are regular, with normal flow and mild cramps. Her past medical problems include obesity. Her BMI is 30 kg/m2. Family history is significant for breast cancer in the paternal grandmother and ovarian cancer in the mother. She is up to date with her screenings and vaccinations. After her family history, which of the following is the most important risk factor for the development of ovarian cancer in this patient?
Choices: 1. 2. 3. 4.
Early menopause Alcohol use Nulliparity Gestational diabetes
Answer: 3 - Nulliparity Explanations: While the most significant risk is family history, nulliparity is a risk factor for developing ovarian cancer. Breastfeeding and pregnancy protect against ovarian cancer as it reduces ovulatory cycles. One of the most important risk factors for ovarian cancer is genetic. BRCA1 and BRCA2 gene mutations are likely causes of ovarian and breast cancer. 10 to 15% of ovarian cancers have a genetic predisposition listed as the main risk factor. This patient should be considered for genetic counseling and testing. Oral contraceptive pills suppress ovulation, causing a protective effect for ovarian cancer. Early menarche increases the risk of developing ovarian cancer because it increases the number of years a woman ovulates. Alcohol consumption is not associated with ovarian cancer. Go to the next page if you knew the correct answer, or click the link image(s) below to further research the concepts in this question (if desired).
Research Concepts: Ovarian Cancer
We update eBooks quarterly and Apps daily based on user feedback. Please tap flag to report any questions that
need improvement.
Question 462:
A 32-year-old male patient presents to the emergency department with nausea, emesis, and diarrhea. He previously underwent a subtotal colectomy as a child due to a genetic condition resulting in hundreds of polyps within the colon. He is diagnosed with gastroenteritis after undergoing a computed tomography (CT) scan of the abdomen. The CT scan demonstrates a large intraabdominal mass with well-circumscribed borders and similar attenuation as muscle. What is the best treatment?
Choices: 1. 2. 3. 4.
Sulindac Radiation therapy Folinic acid, fluorouracil, and oxaliplatin (FOLFOX) therapy 5-Fluorouracil alone
Answer: 1 - Sulindac Explanations: This patient had a subtotal colectomy for familial adenomatous polyposis (FAP). Without prophylactic resection, he would have eventually developed colon cancer. FAP is also associated with desmoid tumors. Approximately 10% to 15% of FAP patients will develop desmoid tumors, most commonly in the abdomen. Desmoid tumors can be locally invasive and difficult to completely resect. Sulindac, celecoxib, and tamoxifen have demonstrated stabilization or regression of desmoid tumors. Radiation therapy is not effective. FOLFOX therapy would be useful for a colorectal adenocarcinoma, not a desmoid tumor. Go to the next page if you knew the correct answer, or click the link image(s) below to further research the concepts in this question (if desired).
Research Concepts: Familial Adenomatous Polyposis
We update eBooks quarterly and Apps daily based on user feedback. Please tap flag to report any questions that need improvement.
Question 463:
A 69-year-old woman presents to the clinic for follow up. She was recently diagnosed with invasive vulvar squamous cell carcinoma after a biopsy. The depth of invasion was noted to be 0.8 mm and no lymph node metastasis was noted. Which of the following is the most appropriate surgical plan for this patient?
Choices: 1. Wide local excision with 1-2 cm surgical margins 2. Simple vulvectomy with 1-2 cm margins and sentinel lymph node biopsy 3. Radical vulvectomy with 2 cm margins 4. Radical vulvectomy with 2 cm margins and lymphadenectomy
Answer: 1 - Wide local excision with 1-2 cm surgical margins
Explanations: This patient has stage IA disease confirmed by the depth of invasion 1 mm and no evidence of lymph node involvement. The recommended treatment for this stage of disease is wide local excision with 1-2 cm surgical margins. Sentinel lymph node biopsy is not recommended for this stage of disease with such shallow depth of invasion. Radical vulvectomy would be too extensive of a procedure for this patient with stage IA disease, and lymphadenectomy would not be necessary, given there is no evidence of lymph node involvement. Go to the next page if you knew the correct answer, or click the link image(s) below to further research the concepts in this question (if desired).
Research Concepts: Vulva Cancer
We update eBooks quarterly and Apps daily based on user feedback. Please tap flag to report any questions that need improvement.
Question 464:
A 48-year-old man presents to the clinic with difficulty in swallowing for 3 months- initially to solids and later to liquids. He reports occasional heartburns for which he takes antacids with mild symptomatic relief. His medical history is significant for hypertension and gastroesophageal reflux disorder. He is a nonsmoker and does not drink alcohol. His home medications include amlodipine and aspirin. Vital signs show blood pressure 120/67 mmHg, pulse 110/min, and respiratory rate 14/min. Abdominal examination shows mild epigastric tenderness. Chest x-ray is unremarkable. Upper gastrointestinal endoscopy shows a rounded, large 8 cm esophageal mass. Histology shows a circumscribed lesions composed of intersecting fascicles of bland spindle cells with abundant cytoplasm. Which of the following is the next best step in the management of this patient?
Choices: 1. 2. 3. 4.
Omeprazole CT scan of the chest Surgical resection MRI of the chest and abdomen
Answer: 3 - Surgical resection Explanations: Esophageal leiomyomas are slow-growing tumors with low malignant potential. The most common site of occurrence of this tumor is the lower two-thirds of the esophagus, and this corresponds with the relative amount of smooth muscle cells present along the wall of the esophagus. Upper endoscopy reveals a submucosal tumor, protruding into the lumen of the esophagus, with normal-looking mucosa. Surgical resection is indicated for large tumors. Go to the next page if you knew the correct answer, or click the link image(s) below to further research the concepts in this question (if desired).
Research Concepts: Esophageal Leiomyoma
We update eBooks quarterly and Apps daily based on user feedback. Please tap flag to report any questions that need improvement.
Question 465:
A 65-year-old male is brought in with a headache and a behavioral change. The patient was alright until three months ago when he began having frequent headaches. Eventually, the headaches worsened, and he started feeling clumsy. He has a 30-pack-year smoking history, but no history of alcohol or illicit drug use. He has a history of hypercholesterolemia for which he takes statins for ten years and of mitral stenosis four years back, for which he was operated and is being medicated for atrial fibrillation. He underwent an age-appropriate cancer screen last year, which was negative. Examination reveals an agitated man with intact palmar reflex and upgoing plantar bilaterally. An ocular examination reveals papilledema. He has also had a weight loss of 8 kg (17.6 lbs) over the past three months. An MRI is ordered. What is the expected finding on the MRI scan?
Choices: 1. 2. 3. 4.
Hockey stick sign in transverse view Hummingbird sign in sagittal view Complex cystic mass with absent dual rim sign Enlarged ventricles
Answer: 3 - Complex cystic mass with absent dual rim sign
Explanations: This patient with headache, behavioral changes, complaints of clumsiness, papilledema, significant weight loss, normal age-appropriate cancer screening, and intact palmar reflex is likely a case primary brain tumor, most likely a frontal lobe tumor. Headaches are the most common presenting symptom of patients with glioma. Depending on the location of the tumor in the brain, the mass effect leads to signs of brain tumors. For example, frontal lobe tumors can present with behavioral changes, while dominant temporal lobe tumors can present with receptive speech problems. Other symptoms related to mass effects include nausea, vomiting, and change in vision, Seizures are the second most common symptom of presentation. Other presenting symptoms are tingling sensations, weakness, difficulty ambulation, and in rare cases, patients can present in a comatose state due to hemorrhage within the tumor, which leads to an acute herniation syndrome. Primary brain tumors are most likely to be gliomas. Gliomas are of 3 types. astrocytoma, oligodendrogliomas, and ependymomas. Astrocytomas are further divided into four grades, which will determine the level of malignancy. Grade 3 and 4 are highly malignant. The hockey stick sign is present in Creutzfeldt-Jacob's disease, which also presents with behavioral changes. Although this too can present with headache, it would not be the primary symptom. Significant weight loss
over three months points to an alternate diagnosis. Hummingbird sign is found in progressive supranuclear palsy, which presents with dementia and frequent falls. However, other parkinsonian features would also be present. Enlarged ventricles can be a physiological finding in aging or a pathological finding in normal pressure hydrocephalus. The absence of urinary symptoms, presence of weight loss, and papilledema, makes this diagnosis less likely. Go to the next page if you knew the correct answer, or click the link image(s) below to further research the concepts in this question (if desired).
Research Concepts: Gliomas
We update eBooks quarterly and Apps daily based on user feedback. Please tap flag to report any questions that need improvement.
Question 466:
A 78-year-old woman presents to the clinic with ongoing pain in her jaw for the last 3 weeks. She has a history of metastatic colon cancer, receiving best supportive care. She is an ex-smoker, stopped 10 years ago, she smoked for 25 years. She takes lisinopril and atenolol for hypertension and atorvastatin for hypercholesteremia. She also takes acetaminophen and codeine for her pain with little benefit. She further receives IV zoledronic acid infusion every 3 months. Her vital signs show temperature 37.6 C, blood pressure 98/65 mmHg, pulse 107/min, respiratory rate 17/min, and SpO2 97% on room air. Oral examination shows poor dentition with evidence of inflamed gum and exposed bone. Which of the following is the best initial step in the management of this patient?
Choices: 1. 2. 3. 4.
Early surgical debridement of the affected tissue Discontinuation of bisphosphonates IV ampicillin Hyperbaric oxygen therapy
Answer: 2 - Discontinuation of bisphosphonates Explanations: Prevention and early identification of high-risk patients is a key strategy in the management of osteonecrosis of the jaw. Conservative management is still considered first-line. Surgery can be considered where conservative management has failed, or significant exposed bone is present from the start. A drug holiday is a concept that has been used to prevent the incidence of osteonecrosis of the jaw. It simply means the temporary withdrawal of bisphosphonates before a planned dental surgery. Hyperbaric oxygen therapy and the use of mesenchymal cells to recreate bone tissue are promising adjunctive modalities that can be considered along with conservative management. Go to the next page if you knew the correct answer, or click the link image(s) below to further research the concepts in this question (if desired).
Research Concepts: Bisphosphonate Toxicity
We update eBooks quarterly and Apps daily based on user feedback. Please tap flag to report any questions that need improvement.
Question 467:
A 65-year-old man presents to the office for evaluation of right upper quadrant abdominal pain, fatigue, and abdominal distention. The patient states he has also lost 6 kg of weight in the past 4 months. On examination, yellowish discoloration of the sclera is noted. The abdomen is distended, and shifting dullness is positive. Ultrasonography shows a liver mass with varying echogenicity due to areas of necrosis and hemorrhage. On contrast-enhanced CT scan (CECT), the mass appears hypodense with multiple patterns of enhancement. Postcontrast, the mass becomes completely isodense in comparison with the hepatic tissue. An image-guided biopsy is performed, and histopathology reveals spindle-shaped and polyhedral cells, demonstrating various patterns of vascular channels. Which of the following immunohistochemical markers is most likely to confirm the diagnosis in this patient?
Choices: 1. 2. 3. 4.
vWF CD30 CD34 S100
Answer: 3 - CD34 Explanations: This patient most likely has a hepatic angiosarcoma. Angiosarcoma is described as a high-grade, malignant neoplasm of endothelial cells of blood or lymphatic vessels. CD34 will show positive staining on immunohistochemistry staining. CD31, CD34, Ulex europaeus agglutinin I, and factor VIIIrelated antigen are immunohistochemical markers that can be used to aid diagnosis. Histologically, the malignancy is composed of spindleshaped and polyhedral cells, demonstrating various patterns of vascular channels. Solid areas that resemble fibrosarcoma and polynuclear giant cells may also be seen. Go to the next page if you knew the correct answer, or click the link image(s) below to further research the concepts in this question (if desired).
Research Concepts: Liver Angiosarcoma
We update eBooks quarterly and Apps daily based on user feedback. Please tap flag to report any questions that need improvement.
Question 468:
A 65-year-old man with a history of colorectal cancer presents to the clinic for follow-up. He previously had radical resection of the tumor, followed by chemotherapy with 5-fluorouracil and leucovorin. His latest imaging revealed the systemic spread of cancer. Oxaliplatin is added to his chemotherapy regimen to work synergistically with 5-fluorouracil to control the spread of cancer. Which of the following best describes the most appropriate laboratory workup needed prior to administering oxaliplatin in this patient?
Choices: 1. Peripheral blood smear, cerebrospinal fluid analysis, hepatitis viral panel 2. Complete blood count with differential, basic metabolic panel, liver function tests, renal function tests, ECG 3. Complete blood count with differential, basic metabolic panel, immunological panel 4. Complete blood count with differential, basic metabolic panel, immunological panel, hepatitis viral panel
Answer: 2 - Complete blood count with differential, basic metabolic panel, liver function tests, renal function tests, ECG
Explanations: Oxaliplatin has hematological toxicity: anemia, thrombocytopenia, and leukopenia. Oxaliplatin initially causes elevated serum aspartate aminotransferase (AST), alanine aminotransferase (ALT), and bilirubin; it is important to know the patient's baseline liver enzymes prior to initiating treatment. Oxaliplatin is moderately emetogenic. To prevent chemotherapy-induced nausea and vomiting (CINV), pretreatment with antiemetics is strongly recommended. It is essential to order a basic metabolic panel to assess sodium, potassium, calcium, and magnesium before initiating treatment. Oxaliplatin is contraindicated in patients with severe renal insufficiency; creatinine clearance 20mL/min. Oxaliplatin increases the risk of QT prolongation. Go to the next page if you knew the correct answer, or click the link image(s) below to further research the concepts in this question (if desired).
Research Concepts: Oxaliplatin
We update eBooks quarterly and Apps daily based on user feedback. Please tap flag to report any questions that need improvement.
Question 469:
A 45-year-old man presents to the office with complaints of pain and swelling in his right leg. The patient works as an accountant and usually spends most of his time on the desk. His past medical history is significant for hypertension and diabetes mellitus. On examination, a tender diffuse swelling is noted below the right knee joint. An x-ray reveals a lytic bone lesion. A biopsy is performed, and the findings are suggestive of a well-differentiated cartilage forming lesion. Histopathology shows islands of cartilage permeating viable, lamellar bone. What is the most likely diagnosis?
Choices: 1. 2. 3. 4.
Grade Grade Grade Grade
I chondrosarcoma II chondrosarcoma III chondrosarcoma IV chondrosarcoma
Answer: 1 - Grade I chondrosarcoma Explanations: Chondrosarcomas are malignant cartilaginous neoplasms with diverse morphological features and clinical behavior. They account for about 20% of all primary malignant tumors of the bone. Chondrosarcomas show abundant blue-grey cartilage matrix-production. Irregularly shaped lobules of cartilage varying in size and shape are present. Fibrous bands separate these lobules or permeate bony trabeculae. Grade I tumors are considered to be low-grade (locally aggressive), also called atypical cartilaginous tumors. Grade I lesions will often closely resemble normal cartilage or the benign enchondroma. In this instance, the distinction between benign and malignant often depends on demonstrating a “chondrosarcoma permeation pattern” where the tumor infiltrates through the marrow cavity instead of being confined by the native architecture. Grade I chondrosarcoma is moderately cellular and contain hyperchromatic, plump nuclei of uniform size. Go to the next page if you knew the correct answer, or click the link image(s) below to further research the concepts in this question (if desired).
Research Concepts: Chondrosarcoma
We update eBooks quarterly and Apps daily based on user feedback. Please tap flag to report any questions that need improvement.
Question 470:
A 56-year-old man with a past medical history of diabetes is seen in the clinic for follow-up on treatment with a selective hedgehog inhibitor medication used to treat a nonresectable cutaneous carcinoma. The patient does not recall the precise type of cancer he is being treated for. Which of the following histopathological findings is most consistent with the patient's most likely diagnosis?
Choices: 1. Atypical keratinocytes with acantholysis 2. Peripheral palisading basaloid cells staining positive for BerEP4 3. Pagetoid spread cells that stain positive for s100 4. Pagetoid spread cells that stain positive for CD4
Answer: 2 - Peripheral palisading basaloid cells staining positive for BerEP4
Explanations: Vismodegib is an oral selective hedgehog inhibitor medication used in the treatment of metastatic or locally nonresectable basal cell carcinoma. Basal cell carcinoma is a cutaneous carcinoma of basaloid cells in the epidermis. Basal cell carcinoma features peripheral palisading of malignant cells with retraction and fibrosis seen. Basal cell carcinoma stains positive with BerEP4. Pagetoid spread is a feature of squamous cell carcinoma, melanoma, and mycosis fungoides. Atypical keratinocytes with acantholysis is a feature of squamous cell carcinoma. Pagetoid spread of s100 staining cells is a feature of melanoma. Pagetoid spread of CD4 positive cells is a feature of mycosis fungoides, not basal cell carcinoma. Go to the next page if you knew the correct answer, or click the link image(s) below to further research the concepts in this question (if desired).
Research Concepts: Vismodegib
We update eBooks quarterly and Apps daily based on user feedback. Please tap flag to report any questions that
need improvement.
Question 471:
A 65-year-old woman with a history of follicular lymphoma diagnosed 10 years ago presents to the clinic for follow up. She has been on observation since. The left cervical mass has been rapidly growing recently. On physical exam, the patient is afebrile, pulse 85/min, blood pressure 135/75 mmHg, and SpO2 97%. Neck exam shows a 6x6 cm mass, firm in consistency, adherent to the surrounding structures. The rest of the physical exam is within normal limits. CT scan of the neck, thorax, abdomen, and pelvis shows an enlarged 6x5 cm level 2 left cervical lymph node and multiple scattered mildly enlarged lymph nodes in the cervical, axillary, and retroperitoneal groups. The patient undergoes biopsy of the cervical lymph node, which identifies diffuse large B-cell lymphoma, consistent with Richter transformation. The patient is deemed a candidate to start R-CHOP (rituximab, cyclophosphamide, doxorubicin, vincristine, and prednisone). Assuming the patient goes into clinical remission, which of the following is the next best step in the management of this patient?
Choices: 1. 2. 3. 4.
Autologous stem cell transplantation Allogeneic stem cell transplantation Continue rituximab for 2 years No further treatment
Answer: 1 - Autologous stem cell transplantation Explanations: Studies have shown that chemotherapy followed by autologous stem cell transplantation in cases of recurrent lymphomas (HL and NHL) that do not respond to initial conventional chemotherapy have better outcomes. Richter transformation is a high-risk transformation; these patients have improved overall survival with RCHOP chemotherapy to induce clinical remission then proceeding with autologous stem cell transplantation. Allogeneic bone marrow transplantation is indicated for patients with AML, ALL, MDS, and myelofibrosis. In these cases, the patient would benefit from the graft-versusleukemia effect of allogeneic transplantation. Rituximab is an anti-CD20. It is usually added to the regimen of patients with DLBCL as their cells are CD20+. Go to the next page if you knew the correct answer, or click the link image(s) below to further research the concepts in this question (if desired).
Research Concepts: Hematopoietic Stem Cell Transplantation
We update eBooks quarterly and Apps daily based on user feedback. Please tap flag to report any questions that
need improvement.
Question 472:
A 70-year-old woman presents with a severe headache. Her symptoms started one day ago, and this was accompanied by nausea and vomiting. Her past medical history includes migraines, and she has lost 10 kg in the past two months. On examination, she is awake, alert, able to follow commands. Her pupils are 2 to 3 mm and reactive to light with full extraocular muscle movement bilaterally. There is no facial asymmetry or dysarthria. The visual confrontation test reveals homonymous hemianopia on the right. There are no motor or sensory deficits. There is positive nuchal rigidity, and Babinski sign is positive bilaterally. A CT scan shows a hypodense lesion with a mixed hyperdensity extending to the subarachnoid space, and with surrounding vasogenic edema with mass effect. What is the most likely diagnosis?
Choices: 1. 2. 3. 4.
Hypertensive intracerebral hemorrhage Head trauma Arteriovenous malformation Metastatic melanoma
Answer: 4 - Metastatic melanoma Explanations: Melanoma is most commonly associated with brain metastases, which often presents with hemorrhage. Intracranial metastases are usually surrounded by massive vasogenic edema on imaging studies. Brain metastases from melanoma, lung, breast, and kidney are common. The most usual source of brain metastases is lung cancer in men and breast cancer in women. However, since there is a higher incidence of lung cancer with metastases, these lesions are the most common source of hemorrhagic lesions overall. Go to the next page if you knew the correct answer, or click the link image(s) below to further research the concepts in this question (if desired).
Research Concepts: Brain Metastasis
We update eBooks quarterly and Apps daily based on user feedback. Please tap flag to report any questions that need improvement.
Question 473:
A 65-year-old man presented to the clinic with multiple swellings in his right groin that have been present for a long time. He experiences pain while walking and moving his legs. General examination revealed no other site involved in lymph node enlargement, and he has no edema, anemia, ascites, or hepatosplenomegaly. He has no history of drug abuse. He has a monogamous relationship with his wife and has three kids. On blood tests, his hemoglobin was 13.5 g/dl, and the white blood cell count (WBC) count was 12000 micro/L, and ESR was 12mm/h (normal is 15mm/h). A biopsy was performed on the lymph node. Which of the following is most likely to be seen on histology?
Choices: 1. 2. 3. 4.
Reed Sternberg cells Tangible body macrophages Starry sky appearance Lollipop sign
Answer: 4 - Lollipop sign Explanations: Unicentric Castleman disease only involves enlargement of a single group of lymph nodes without any other systemic manifestations. It is asymptomatic and therefore is hard to diagnose. The etiology of unicentric Castleman disease is not yet known. Multicentric Castleman disease can be caused by chronic inflammatory disorders, abnormal production of IL-6 and cytokines, and can also be associated with HIV and Kaposi sarcoma. All lab investigations are normal and definitive diagnoses can only be made based on histopathological screening. The characteristic morphological feature for Unicentric Castleman Disease is a lollipop sign (onion skin-like feature of the follicle with hyalinized blood vessels penetrating it- giving it a lollipop appearance.) Reed Sternberg cells are seen in Hodgkin lymphoma, starry sky appearance is seen in Burkitt lymphoma, and tangible body macrophages are seen in follicular hyperplasia. Go to the next page if you knew the correct answer, or click the link image(s) below to further research the concepts in this question (if desired).
Research Concepts: Castleman Disease
We update eBooks quarterly and Apps daily based on user feedback. Please tap flag to report any questions that need improvement.
Question 474:
A 45-year-old white male presents to the office for a routine physical. During the physical, he states his dad's brother was recently diagnosed with prostate cancer at the age of 74 years. He states that he is anxious since he read online that prostate cancer can run in families and wants to be tested. What is the most appropriate recommendation for this patient?
Choices: 1. Digital rectal examination (DRE) alone 2. Reassurance 3. Shared decision-making for prostate-specific antigen (PSA) testing 4. PSA level alone
Answer: 2 - Reassurance Explanations: Screening for prostate cancer is not generally indicated per guidelines for asymptomatic males under 50 years of age unless they are at high risk. However, there are small numbers of patients in their 40s who will develop prostate cancer and when that happens it is quite aggressive. This is why some groups and experts suggest a baseline PSA for men in their 40s. Shared decision-making for testing is reasonable for males aged 40-50 years if they're at high risk (e.g., African-American males/first-degree relative with prostate cancer at age 65 years). DRE alone is not indicated for prostate screening. Go to the next page if you knew the correct answer, or click the link image(s) below to further research the concepts in this question (if desired).
Research Concepts: Localized Prostate Cancer
We update eBooks quarterly and Apps daily based on user feedback. Please tap flag to report any questions that need improvement.
Question 475:
A patient with small cell carcinoma presents to you with fatigue. On exam, the patient has a buffalo hump, moon facies, striae, and hypertension. What is the most common electrolyte abnormality seen in these patients?
Choices: 1. Hyperkalemia 2. Metabolic acidosis with hyponatremia 3. Hypokalemia with metabolic alkalosis 4. Respiratory acidosis with compensated metabolic alkalosis
Answer: 3 - Hypokalemia with metabolic alkalosis Explanations: Remember, patients with malignancy present with various types of syndromes, such as antidiuretic hormone secretion and hypercalcemia. One should always be aware of paraneoplastic syndromes and how to treat them. Ectopic ACTH syndrome is a feature of some lung cancers, including carcinoids and small cell tumors. It is important to recognize ectopic ACTH secretion as it carries a high morbidity. Most patients will present with hypertension and severe hypokalemic metabolic alkalosis. Go to the next page if you knew the correct answer, or click the link image(s) below to further research the concepts in this question (if desired).
Research Concepts: Cushing Syndrome
We update eBooks quarterly and Apps daily based on user feedback. Please tap flag to report any questions that need improvement.
Question 476:
A 55-year-old man presents to the clinic with a sking rash. Physical exam shows violaceous, psoriasiform plaques over his fingers, ears, and nose. A skin biopsy reveals spongiosis, interface dermatitis, and mild dyskeratosis. Detailed evaluation shows that the patient has squamous cell carcinoma of the larynx. Changes simulating psoriasis are most often seen in which of the following in this condition?
Choices: 1. 2. 3. 4.
Nail Mucosa Groin Hair
Answer: 1 - Nail Explanations: The morphology of the skin lesions, location, and the presence of associated squamous cell carcinoma of the larynx strongly suggests the possibility of acrokeratosis paraneoplastica (AP). The histopathology – showing spongiosis, interface dermatitis, and dyskeratosis, although not specific, is strongly suggestive of acrokeratosis paraneoplastica. Acrokeratosis paraneoplastica, also called acrokeratosis neoplastica or Bazex syndrome, is characterized by psoriasiform lesions on the acral areas of the body, associated with internal malignancy. Acrokeratosis paraneoplastica can be associated with nail changes similar to those seen in psoriasis and the nail changes might precede the onset of the skin lesions. Go to the next page if you knew the correct answer, or click the link image(s) below to further research the concepts in this question (if desired).
Research Concepts: Acrokeratosis Paraneoplastica
We update eBooks quarterly and Apps daily based on user feedback. Please tap flag to report any questions that need improvement.
Question 477:
A 62-year-old male with a history of hypertension, diabetes mellitus, and stage III chronic kidney disease presents with a six-month history of facial swelling and flushing, which is worse at the end of the day. He reports night sweats and unintentional weight loss of 10 pounds. Vital signs reveal a blood pressure of 146/82 mmHg, heart rate of 92 beats per minute, respiratory rate of 16 breaths per minute, oxygen saturation of 98% on room air, and temperature of 99.1 F (37.2 C). The physical exam is notable for facial edema and flushing, as well as proptosis. A lung exam is significant for bibasilar crackles. Labs are remarkable for hemoglobin of 10.2 g/dl and a creatinine of 1.9 mg/dl. The rest of the labs are unremarkable. Chest radiograph shows a widened mediastinum with bilateral pleural effusions. Which of the following is the next preferred step in the management?
Choices: 1. Chest computed tomography (CT) without IV contrast 2. Chest CT with IV contrast 3. Magnetic resonance imaging (MRI) chest without IV contrast 4. MRI chest with IV contrast
Answer: 3 - Magnetic resonance imaging (MRI) chest without IV contrast
Explanations: CT chest with IV contrast is the imaging modality of choice for evaluation of anterior mediastinal masses; however, it would be contraindicated in this patient with a creatinine of 1.9 mg/dl. IV contrast is not needed with MRI for evaluation of anterior mediastinal masses; therefore, MRI without contrast is the imaging modality of choice when there is a contraindication to IV contrast. Anterior mediastinal masses can present with compression symptoms, such as superior vena cava syndrome, which presents with facial swelling, flushing, and proptosis. B-symptoms and pleural effusions in this 62-year-old male suggest a possible diagnosis of Hodgkin lymphoma, which has a bimodal distribution of incidence. Go to the next page if you knew the correct answer, or click the link image(s) below to further research the concepts in this question (if desired).
Research Concepts: Anterior Mediastinal Mass
We update eBooks quarterly and Apps daily based on user feedback. Please tap flag to report any questions that need improvement.
Question 478:
A 35-year-old male with a history of gastrointestinal reflux disease presents with hematochezia. The patient has a family history significant for colorectal cancer in his father at age 50 and endometrial cancer in his sister at age 40. His labs are significant for microcytic anemia. Hemoccult is positive on the rectal exam. No palpable abdominal masses or lymphadenopathy are detected. Computed tomography (CT) of the abdomen shows sigmoid thickening. The patient undergoes colonoscopy showing a non-obstructive non-circumferential mass in the sigmoid colon. He undergoes a left hemicolectomy. Pathology shows a 2 x 2 cm colonic adenocarcinoma invading the muscularis propria with 0 of 15 lymph nodes positive. No distant disease is present on chest imaging. Which of the following is the most important next step to be taken in this patient?
Choices: 1. 2. 3. 4.
Magnetic resonance imaging (MRI) scan Repeat CT Microsatellite instability testing Genetic testing and genetic counseling
Answer: 4 - Genetic testing and genetic counseling Explanations: Lynch syndrome involves heritable defects in the mismatch repair genes. This patient meets the Bethesda criteria for Lynch syndrome. The criteria are age less than 50 years at diagnosis, one or more first-degree relatives with a Lynch syndrome-related cancer with at least one diagnosed before age 50, or two or more firstdegree relatives with Lynch-syndrome-related cancers regardless of age. Genetic testing for Lynch syndrome is recommended for patients who meet: Amsterdam II criteria, revised Bethesda guidelines, endometrial cancer diagnosed before age 50, or known Lynch syndrome in the family. Testing should also be considered in patients with at least a 5% risk of Lynch syndrome on prediction models. Genetic testing should be preceded by genetic counseling due to the complexities of test selection. Microsatellites are DNA strands with a repetitive nucleotide sequence that are prone to developing errors when the mismatch repair gene function is defective. Cancer cells in these patients with abnormal mismatch repair genes, as in Lynch syndrome, will show a variable number of microsatellite repeat sequences when compared to normal tissue from the same individual. The degree and number of such inconsistencies constitute the degree of microsatellite instability. Microsatellite instability is closely associated with Lynch syndrome but is not specific as most tumors with this defect occur sporadically. This patient with T3N0M0 disease has stage IIA colorectal cancer. There is no need for either a repeat
CT scan or an MRI. Microsatellite instability testing can be used to help identify Lynch syndrome but is not necessary here as the diagnosis is relatively clear. The definitive diagnosis of Lynch syndrome is based on clinical factors and history as well as the combination of microsatellite instability testing and mismatch repair gene immunohistochemistry. Go to the next page if you knew the correct answer, or click the link image(s) below to further research the concepts in this question (if desired).
Research Concepts: Lynch Syndrome
We update eBooks quarterly and Apps daily based on user feedback. Please tap flag to report any questions that need improvement.
Question 479:
A 42-year-old man presents to the office for follow up. He has a past medical history significant for a kidney transplant and is taking immunosuppressive therapy. He has a history of significant sun exposure and has had multiple basal cell skin carcinomas excisions. Which of the following is the strongest indication to prescribe acitretin in this patient?
Choices: 1. Chemoprevention measure for solid organ cancers in patients' with solid organ transplants 2. Chemoprevention measure for melanoma skin cancers in patients' with solid organ transplants 3. Treatment of existing nonmelanoma skin cancers in patients' with solid organ transplants 4. Chemoprevention measure for nonmelanoma skin cancers in patients' with solid organ transplants
Answer: 4 - Chemoprevention measure for nonmelanoma skin cancers in patients' with solid organ transplants
Explanations: Acitretin is FDA-approved for psoriasis (severe plaquetype psoriasis, pustular psoriasis generalized, pustular psoriasis localized, combination therapy with ultraviolet B (UVB) or psoralen ultraviolet A (PUVA), combination therapy with cyclosporine, combination therapy with biologic therapies. Acitretin is the only systemic retinoid that is FDAapproved for psoriasis and effective as monotherapy. Acitretin has been used off-label in dermatology for other uses. In those with solid organ transplants, acitretin has been used as a chemoprevention measure for nonmelanoma skin cancers. Acitretin has also been used for Darier disease, pityriasis rubra pilaris (PRP), and ichthyoses such as lamellar ichthyosis. Additionally, acitretin has been used in Grover disease (transient acantholytic dermatosis), lichen planus, and lupus erythematosus. Go to the next page if you knew the correct answer, or click the link image(s) below to further research the concepts in this question (if desired).
Research Concepts: Acitretin
We update eBooks quarterly and Apps daily based on user feedback. Please tap flag to report any questions that need improvement.
Question 480:
A 71-year-old woman is being evaluated in the intensive care unit. She was diagnosed with advanced-stage lung cancer with bony and brain metastasis six months ago. The patient was treated with aggressive chemotherapy and radiotherapy; however, she did not respond to the treatment. The surgeon decides not to operate on the patient as the surgery will not alleviate the suffering. The physician recommends terminating care and allowing the patient to die. The patient has a GCS of 8/15. Which of the following takes precedence in determining her care?
Choices: 1. 2. 3. 4.
Ethics committee Power of attorney Physician Patient
Answer: 2 - Power of attorney Explanations: When symptoms are no longer manageable or patient suffering is too great, termination of care must be considered. Termination of care is not decided on a whim but is part of the initial end-of-life discussions. When the treatment the healthcare team members provide is medically futile or prolongs patient suffering, terminating the care and allowing the patient to die is necessary. If the patient can participate in these decisions, it should be the patient’s decision when they want to terminate care. If the patient is developmentally or cognitively incapable of making such decisions, this responsibility falls to a power of attorney. Go to the next page if you knew the correct answer, or click the link image(s) below to further research the concepts in this question (if desired).
Research Concepts: End of Life Care
We update eBooks quarterly and Apps daily based on user feedback. Please tap flag to report any questions that need improvement.
Question 481:
A 65-year-old man presents to the clinic with concerns about a lesion on his arm. He has a history of moderate sun exposure and multiple actinic keratoses treated with cryotherapy. On examination, a pink plaque with a diameter of 1 cm is noted on the right arm. A biopsy is performed, which reveals a superficial basal cell carcinoma. The patient does not wish to undergo surgery. The clinician prescribes a topical imidazoquinolinamine immunomodulator. What is the side effect profile of this medication?
Choices: 1. Application site reactions are often mild, and systemic adverse effects occur less frequently than application site reactions. 2. Systemic adverse effects are often severe and occur frequently. 3. Application site reactions occur less often than systemic adverse effects. 4. Systemic adverse effects are common due to a high degree of percutaneous absorption of the medication.
Answer: 1 - Application site reactions are often mild, and systemic adverse effects occur less frequently than application site reactions.
Explanations: Topical imiquimod cream can be utilized to treat superficial basal cell carcinoma that is not on a high-risk anatomic site. Application site reactions due to topical imiquimod use are often mild. Possible side effects of topical imiquimod include application site pain, burning, or pruritus. Topical imiquimod may be a useful treatment option for superficial basal cell carcinoma in patients who cannot undergo surgery or do not wish to undergo surgery. Go to the next page if you knew the correct answer, or click the link image(s) below to further research the concepts in this question (if desired).
Research Concepts: Imiquimod
We update eBooks quarterly and Apps daily based on user feedback. Please tap flag to report any questions that need improvement.
Question 482:
A 53-year-old woman with a past medical history of breast cancer status-post right mastectomy and persistently elevated calcium levels presents to the office for a follow-up visit. She complains of intermittent nausea, controlled with as-needed sublingual disintegrating ondansetron tablets, loss of appetite, fatigue, frequent urination, and arthralgia. Her most recent labs showed total serum calcium of 11.5 mg/dL (reference range: 9–10.5 mg/dL). Attempts to keep her calcium level under control have been unsuccessful despite previous treatment with calcitonin and bisphosphonates. She expresses concern about her abnormal labs and is eager to know what other treatment modalities will help control her calcium levels. Her physical examination and vital signs are unremarkable. EKG in the office shows sinus rhythm with a ventricular rate of 58/min and PR interval of 0.20 seconds, with no significant ST/T wave changes. Which of the following is the next best step in the management of this patient?
Choices: 1. 2. 3. 4.
Bilateral total mastectomy Oral furosemide Denosumab Encourage increased fluid intake
Answer: 3 - Denosumab Explanations: Denosumab is a monoclonal antibody to RANK-L that decreases osteoclast action and bone resorption. It is highly effective in hypercalcemia refractory to bisphosphonates. The typical dose used is 120 mg subcutaneous injection. It can be safely used in renal failure patients. Hypercalcemia can occur secondary to underlying malignancies from the secretion of PTHrP. However, bilateral total mastectomy will not definitely normalize the serum calcium levels in this case. Surgery is only curative in cases of hypercalcemia as a result of parathyroid adenomas. Loop diuretics such as furosemide are efficient in lowering calcium levels when used adjunctively with adequate hydration to increase calcium excretion. Monitoring alone is inappropriate in severe hypercalcemia, as the patient certainly requires optimal management of her calcium levels. Go to the next page if you knew the correct answer, or click the link image(s) below to further research the concepts in this question (if desired).
Research Concepts: Resistant Hypercalcemia
We update eBooks quarterly and Apps daily based on user feedback. Please tap flag to report any questions that need improvement.
Question 483:
A 48-year-old African-American woman presents to the clinic with a palpable breast mass. A biopsy is concerning for infiltrating ductal carcinoma. Which of the following additional features is most likely to be seen in her tumor compared to a Non-Hispanic white woman of a similar age?
Choices: 1. A size of less than 2 cm in diameter at the time of diagnosis 2. Estrogen-receptor and progesterone-receptor negative 3. Well-differentiated with a lower rate of nuclear atypia 4. Human epidermal growth factor receptor 2 positive
Answer: 2 - Estrogen-receptor and progesteronereceptor negative
Explanations: Black women have a higher mortality rate, greater incidence of late-stage diagnosis, and less favorable outcomes from breast cancer than Non-Hispanic white women. The five-year breast cancer survival rate for Black women is 81% versus 92% in Non-Hispanic white women. The higher incidence of late-stage diagnosis and mortality in black women is multifactorial. Barriers to early screening, lack of insurance, diet, genetics, medical mistrust, and bias within the medical community all contribute to the cultural, environmental, biological, and systems-based issues that lead to health disparity in breast cancer. Breast cancer can be subtyped by the expression of different receptors. Many treatments have been developed to target the expression of estrogen receptor (ER), progesterone receptor (PR), and human epidermal growth factor receptor 2 (HER2) positive cancers. The triple-negative tumor subtype is associated with a poorer prognosis given more limited treatment options. Black and Hispanic women have a higher incidence of triple-negative disease than Non-Hispanic White women. Go to the next page if you knew the correct answer, or click the link image(s) below to further research the concepts in this question (if desired).
Research Concepts:
Disparity In Early Detection Of Breast Cancer
We update eBooks quarterly and Apps daily based on user feedback. Please tap flag to report any questions that need improvement.
Question 484:
A 55-year-old man presents to the clinic with a painful left knee joint for 2 months. The pain is partially relieved on medication. On examination, mild tenderness is present over the distal femur with intact distal neurovascular status. The radiograph reveals a mottled destructive lesion in the distal femur. On biopsy, there is a storiform appearance of spindle cells. What is the most likely diagnosis?
Choices: 1. 2. 3. 4.
Metastatic bone disease Undifferentiated pleomorphic sarcoma Non-Hodgkin lymphoma Plasma cell tumor
Answer: 2 - Undifferentiated pleomorphic sarcoma Explanations: In the above vignette, the patient has features of undifferentiated pleomorphic sarcoma. On a radiograph, it reveals both osteolytic and osteogenic lesions. Histology is characterized by the storiform appearance of pleomorphic spindle cells. Wide local excision, chemotherapy, and radiotherapy are the treatment of choice. Plasma cells and non-Hodgkin lymphoma are B-cell tumors. The storiform appearance of pleomorphic spindle cells is a characteristic feature of undifferentiated pleomorphic sarcoma. Go to the next page if you knew the correct answer, or click the link image(s) below to further research the concepts in this question (if desired).
Research Concepts: Soft Tissue Clear Cell Sarcoma
We update eBooks quarterly and Apps daily based on user feedback. Please tap flag to report any questions that need improvement.
Question 485:
A 66-year-old female patient follows up at a breast care clinic. She had been having recurrent nausea and vomiting after the initiation of chemotherapy and was prescribed dronabinol. She reports significant improvement in her symptoms. Today her vitals are HR 88, RR 17, BP 122/76, and 99% O2 saturation. Her physical exam is unremarkable. Which of the following receptor is the mechanism of her therapy?
Choices: 1. 2. 3. 4.
Post-synaptic endocannabinoid release Interneuron mu-opioid receptor activation Pre-synaptic calcium channel inactivation Post-synaptic cannabinoid receptor activation
Answer: 3 - Pre-synaptic calcium channel inactivation Explanations: The patient has been prescribed dronabinol, a pharmaceutical equivalent of Tetrahydrocannabinol (THC), for treatment of chemotherapy-induced nausea and vomiting. Dronabinol exerts this function by binding primarily to CB1 in the chemoreceptor trigger zone in the medulla. The CB1 receptors are primarily located on presynaptic neurons and function by modulating calcium release and concentration in the presynaptic neuron. This modulation of calcium channels reduces their activity and consequently reduces the release of neurotransmitters as well. Go to the next page if you knew the correct answer, or click the link image(s) below to further research the concepts in this question (if desired).
Research Concepts: Cannabinoids
We update eBooks quarterly and Apps daily based on user feedback. Please tap flag to report any questions that need improvement.
Question 486:
A 55-year-old woman of Asian descent who does not smoke is evaluated in the pulmonary clinic for a cough of 6 months duration that has not resolved despite trying various antitussives. She denies hemoptysis but notes a recent increase in the production of clear sputum associated with the cough. CT of the chest is completed to better assess a faint opacity seen on chest x-ray in the left upper lobe (LUL). CT reveals a 4.1 cm mixed solid and ground glass mass along with mildly enlarged pretracheal and paratracheal adenopathy. CT-guided core needle biopsy of the LUL mass is completed. Histopathology confirms adenocarcinoma with lepidic growth pattern and absent mucin. Immunostaining is strongly positive for TTF-1 (thyroid transcription factor-1) and CK-7 (cytokeratin -7). Kras mutation is negative. Epidermal growth factor receptor (EGFR) exon 19 deletion is positive. PET scan confirms stage II lung cancer with no increased uptake in the mediastinal nodes. This is further confirmed on endobronchial ultrasound (EBUS) guided staging. She undergoes left upper lobectomy with mediastinal lymph node sampling, and the final pathologic stage is pT2bN0, stage IIA. Which of the following is approved for adjuvant therapy in treating this patient's lung cancer?
Choices: 1. 2. 3. 4.
Crizotinib Bevacizumab Erlotinib Osimertinib
Answer: 4 - Osimertinib Explanations: The final size of the tumor being >4 cm (pT2b), is characterized as a high-risk factor; therefore, this patient qualifies for adjuvant therapy. Epidermal growth factor receptor (EGFR) mutations are more frequently associated with the lepidic growth pattern. The most up-to-date adjuvant systemic therapy option for stage IIa, high-risk, EGFR mutated lung adenocarcinoma includes adjuvant cisplatin-based chemotherapy followed by adjuvant osimertinib for 3 years. ADAURA trial investigated the role of adjuvant osimertinib in early-stage EGFR mutated lung cancers and demonstrated significantly improved disease-freesurvival benefit in stage IB to IIIA EGFR mutationpositive non-small cell lung cancers. Go to the next page if you knew the correct answer, or click the link image(s) below to further research the concepts in this question (if desired).
Research Concepts: Bronchoalveolar Cancer
We update eBooks quarterly and Apps daily based on user feedback. Please tap flag to report any questions that
need improvement.
Question 487:
A 65-year-old farmer comes to the physician's office due to a lesion on the tip of his nose. He first noticed the spot a couple of years ago, but it has since enlarged. He has years of unprotected sun exposure. He does not visit the doctor often and has not been sick lately. Biopsy of the lesion confirms malignant melanoma. Based on the diagnosis and location of the lesion, what is the most likely location to find the first evidence of metastasis?
Choices: 1. 2. 3. 4.
Upper superficial cervical lymph nodes Submandibular lymph nodes Retropharyngeal lymph nodes Upper deep cervical lymph nodes
Answer: 2 - Submandibular lymph nodes Explanations: The anterior nasal cavity and external nose drain anteriorly to the face that then makes their way to the submandibular lymph nodes. Submandibular lymph nodes are located in level 1B. The specific levels and location of lymph nodes help determine if cancer has spread. The upper cervical lymph nodes and retropharyngeal lymph nodes drain the posterior nasal cavity and paranasal sinuses. Go to the next page if you knew the correct answer, or click the link image(s) below to further research the concepts in this question (if desired).
Research Concepts: Anatomy, Head and Neck, Nasal Cavity
We update eBooks quarterly and Apps daily based on user feedback. Please tap flag to report any questions that need improvement.
Question 488:
A 65-year-old male with metastatic colorectal cancer presents with complaints of paresthesia, numbness, and tingling in his extremities. The patient has been treated with chemotherapy for the past 6 months. The clinician suspects the patient's symptoms are due to an adverse effect of a chemotherapeutic drug. Which of the following drugs is most likely responsible for the patient's symptoms?
Choices: 1. 2. 3. 4.
Oxaliplatin Isoniazid Cyclophosphamide Trastuzumab
Answer: 1 - Oxaliplatin Explanations: The patient's symptoms are suggestive of peripheral chemotherapy-induced peripheral neuropathy. Among the options listed, oxaliplatin is the drug that is most likely to cause peripheral neuropathy. Oxaliplatin is a medication used in the management and treatment of metastatic colorectal cancer. It is in the platinum-based chemotherapeutic class of drugs. Isoniazid also results in neuropathy, however, it is not a chemotherapeutic drug. Cyclophosphamide and trastuzumab may cause neuropathy but that is not very likely. Go to the next page if you knew the correct answer, or click the link image(s) below to further research the concepts in this question (if desired).
Research Concepts: Neuropathy
We update eBooks quarterly and Apps daily based on user feedback. Please tap flag to report any questions that need improvement.
Question 489:
A 32-year-old female patient started 3 weeks ago with mild perianal numbness but did not seek medical attention. This has significantly progressed during the last week, and since last night, she complains of numbness in the saddle area distribution, has urinary retention, and absent bilateral motor function below the knee. There is lax sphincteric tone on the rectal exam. Emergency magnetic resonance imaging shows a very large intraspinal lesion occupying all the spinal canal at the level of L3. Which is the best time-frame to operate this patient?
Choices: 1. Within the first 48 hours after the presentation of symptoms 2. Within the first 96 hours after the presentation of symptoms 3. Within the first week after the presentation of symptoms 4. Any time as the patient has already bladder dysfunction and absent rectal tone and surgery time will not improve these deficits
Answer: 1 - Within the first 48 hours after the presentation of symptoms
Explanations: The timing of surgical decompression is a significant factor in the prognosis and outcome, the best results are obtained when surgery is done within the first 48 hours after the presentation of symptoms. Some reports give the benefits until the 72-hour mark. Bladder dysfunction has a negative prognostic factor for poor outcome. The return of bladder function is estimated to occur in approximately 50% of patients with cauda equina syndrome. Steroids can be useful in patients with tumors, but surgery should be performed before the 72-hour mark. Go to the next page if you knew the correct answer, or click the link image(s) below to further research the concepts in this question (if desired).
Research Concepts: Conus And Cauda Equina Tumors
We update eBooks quarterly and Apps daily based on user feedback. Please tap flag to report any questions that need improvement.
Question 490:
A 35-year-old female G2P1 in her second trimester presents to the clinic with complaints of fatigue and weakness along with abdominal distension in the right hypochondrium region. Physical examination reveals mild to moderate splenomegaly. A blood test is done, which shows hemoglobin 9 g/dL, platelets 90,000/mcL, and ANC levels of 600/mcL. The flow cytometry markers CD11c, CD25, CD103, are present. Which of the following should be considered as first-line treatment in this patient?
Choices: 1. 2. 3. 4.
Cladribine Pentostatin Interferon Alpha Vemurafenib
Answer: 3 - Interferon Alpha Explanations: Hairy cell leukemia (HCL) is a relatively rare chronic Bcell malignancy that involves the bone marrow, spleen, and peripheral blood. The complete blood count may reveal pancytopenia, including monocytopenia. The median age at diagnosis is approximately 55. Affected patients often have non-specific symptoms, including fatigue and weakness, as well as symptoms related to cytopenias and splenomegaly. Eighty percent of patients will have significant cytopenias on presentation, with severe pancytopenia in less than 10%. While splenomegaly is a predominant feature, massive, symptomatic splenomegaly is less frequent, perhaps due to earlier detection on routine complete blood count (CBC). Diagnosis is achieved by studies on peripheral blood, including flow cytometry and review of the peripheral smear, along with bone marrow biopsy. A “dry tap” or inability to perform bone marrow aspiration is frequently encountered with hairy cell leukemia, although not with HCL-V. The hairy cell has characteristic-appearing mononuclear cells, which are typically abundant with circumferential hair-like cytoplasmic projections and around, well-defined nucleus. Cladribine and pentostatin, two purine analogs, are the first-line treatment for hairy cell leukemia. They are equally effective in inducing and maintaining remission. Nonetheless, cladribine is regarded as first-line chemotherapy among most hematologists due to its favorable toxicity profile. A single course of cladribine causes a complete response in approximately 90% of
individuals after one cycle (5 to 7 days). It is important to note that these patients will be immunosuppressed for many months following therapy, and attention to possible infectious complications is critical. A recent meta-analysis conducted by Andrasiak et al. showed that the most effective treatment in treatment-naive patients and the first relapse was cladribine with rituximab maintenance. Vemurafenib was also shown to be similarly efficacious; however, it is usually used for refractory or progressive cases. Interferon-alpha is the treatment of choice during pregnancy when treatment is warranted. It may be preferred for the initial treatment in patients with severe pancytopenia and active infection to improve blood counts and allow for subsequent therapy with purine analogs. However, patients with HCL who express the CD5 antigen appear to respond poorly to IFN-a. Go to the next page if you knew the correct answer, or click the link image(s) below to further research the concepts in this question (if desired).
Research Concepts: Hairy Cell Leukemia
We update eBooks quarterly and Apps daily based on user feedback. Please tap flag to report any questions that need improvement.
Question 491:
Subcutaneous panniculitis-like T-cell lymphoma (SPTCL) is a rare primary cutaneous lymphoma that has a good prognosis. The differential includes lupus erythematosus, primary cutaneous gamma delta T-cell lymphoma, and extranodal natural killer/T-cell lymphoma, nasal type. Which of these was considered to be the same entity as SPTCL prior to the 2008 revision of the World Health Organization (WHO) Classification of Tumours of Hematopoietic and Lymphoid Tissues?
Choices: 1. 2. 3. 4.
Lupus erythematosus Extranodal natural killer/T-cell lymphoma, nasal type Primary cutaneous gamma delta T-cell lymphoma MALToma
Answer: 3 - Primary cutaneous gamma delta T-cell lymphoma
Explanations: Autoimmune diseases occur in approximately 20% of cases, and lupus erythematosus panniculitis (LEP) is usually part of the differential diagnosis due to similar clinical and histologic features. Clinically, LEP presents more commonly on the face and proximal extremities, while SPTCL usually presents on the lower extremities, upper extremities, and trunk. Findings such as fever and hepatosplenomegaly may favor SPTCL. However, they can also sometimes be observed in LEP. The subcutaneous pattern of primary cutaneous gamma delta T-cell lymphoma can appear similar to the lobular panniculitis seen in SPTCL. However, rimming of the adipocytes is less prominent. By immunohistochemistry, the neoplastic cells of primary cutaneous gamma delta T-cell lymphoma are positive for CD56, gamma delta TCR, TIA1, granzyme B, and perforin and are typically negative for CD8, beta F1, and TCR-alfa and beta. By immunohistochemistry, these neoplastic T-cells of SPTCL express CD8, beta F1 and other cytotoxic T-cell markers including granzyme B, perforin, and TIA1. Go to the next page if you knew the correct answer, or click the link image(s) below to further research the concepts in this question (if desired).
Research Concepts: Subcutaneous Panniculitis Like T-cell Lymphoma
We update eBooks quarterly and Apps daily based on user feedback. Please tap flag to report any questions that need improvement.
Question 492:
A 66-year-old man undergoing hospice care after being diagnosed with lung cancer reports increased dyspnea and chest pain in addition to the blood in the sputum. He also reports a significant decrease in weight during the last few months. Which of the following best explains the patient's deteriorating condition?
Choices: 1. 2. 3. 4.
Spread of cancer to the opposite lung Metastasis to the esophagus Saddle clot Tuberculosis
Answer: 1 - Spread of cancer to the opposite lung Explanations: The most commonly involved sites for lung cancer metastasis are the opposite lung, adrenal gland, bone, brain, and liver. Thus, when a patient experiences new-onset symptoms of lung cancer, such as cough/blood in sputum, pain in the chest, fatigue, loss of weight, difficulty in breathing, or hoarseness, the symptoms are most likely related to metastasis to the opposite lung. To improve outcomes, an interprofessional team approach with close communication between the members may perhaps lead to earlier diagnosis and treatment. Lung cancer symptoms occur due to local effects of the tumor, such as cough due to bronchial compression by the tumor, due to distant metastasis, stroke-like symptoms secondary to brain metastasis, paraneoplastic syndrome, and kidney stones due to persistent hypercalcemia. Go to the next page if you knew the correct answer, or click the link image(s) below to further research the concepts in this question (if desired).
Research Concepts: Lung Metastasis
We update eBooks quarterly and Apps daily based on user feedback. Please tap flag to report any questions that need improvement.
Question 493:
A 71-year-old man presents to the hospital with fatigue, fever, and numbness in his hands and feet. His vital signs reveal blood pressure 135/82 mmHg, temperature 99.8 F, pulse 86/minute, and respiratory rate of 26/minute. The patient’s laboratory values reveal sodium of 140 mEq/L, potassium of 4.1 mEq/L, magnesium of 1.0 mEq/L, creatinine 4.6 mg/dL, BUN 77 mg/dL, and glucose 81 mg/dL. Further labs show an IgG level of 305 mg/dL and an IgM level of 455 mg/dL. Physical exam shows lymphadenopathy, splenomegaly, and peripheral neuropathy in hands and feet. A medication with which of the following mechanisms of action is the most effective pharmacotherapy for this patient?
Choices: 1. 2. 3. 4.
Proteosome inhibitor DNA intercalation Dihydrofolate reductase inhibitor Free radical production
Answer: 1 - Proteosome inhibitor Explanations: The clinical vignette is most consistent with a diagnosis of multiple myeloma. Bortezomib is FDA approved as first-line therapy for multiple myeloma. Bortezomib works by inhibiting proteasomes which leads to cell-cycle arrest, and apoptosis. Bortezomib is also used in the FDA off-label treatment of Waldenstrom macroglobulinemia. Bortezomib is FDA approved for relapsed or refractory mantle-cell lymphoma. Go to the next page if you knew the correct answer, or click the link image(s) below to further research the concepts in this question (if desired).
Research Concepts: Bortezomib
We update eBooks quarterly and Apps daily based on user feedback. Please tap flag to report any questions that need improvement.
Question 494:
A 10-year-old male with Down syndrome is suspected of having acute lymphoblastic leukemia after receiving initial physical exam and laboratory workup. Histologic evaluation for definitive diagnosis and staging is planned. Which of the following correctly identifies the preferred testing site for this diagnostic procedure?
Choices: 1. 2. 3. 4.
Iliac crest Sternum and bilateral iliac crest Sternum Lumbar spine
Answer: 1 - Iliac crest Explanations: The iliac crest is the most common site for both aspiration and biopsy of the bone marrow. The iliac crest is safe and easily accessible, with minimal risk of hitting nerves or vessels. The sternum can be aspirated in patients over 12 if the iliac crest biopsy is not possible, but this is not the preferred site. Biopsy of the sternum carries a high risk of sternal perforation, cardiac tamponade, and death. Go to the next page if you knew the correct answer, or click the link image(s) below to further research the concepts in this question (if desired).
Research Concepts: Bone Marrow Aspiration And Biopsy
We update eBooks quarterly and Apps daily based on user feedback. Please tap flag to report any questions that need improvement.
Question 495:
A 59-year-old male patient presents with a white lesion in his mouth. The patient had previously been diagnosed with oral squamous cell carcinoma three years ago and was treated with 2000 cGY of radiation to his head and neck. The previous cancer lesion appeared to resolve, and the patient is being asked to be evaluated for hyperbaric treatments to treat his new lesion. The patient does not have a fever or lymphadenopathy and denies any purulent discharge. What is the best option at this time?
Choices: 1. 2. 3. 4.
Treatment with hyperbaric oxygen on Marx protocol Ask to have the lesion biopsied Refer for additional radiation Start antibiotics immediately
Answer: 2 - Ask to have the lesion biopsied Explanations: When evaluating a problematic wound in previously radiated tissue, it is crucial to determine that the problem is from soft tissue radionecrosis and not from another cause. Typically, for soft tissue radionecrosis lesions to develop, the accumulative dose of radiation has been over 3000 cGy, and more commonly over 5000 cGy. Lesions caused by lower levels of radiation therapy would be doubtful to develop a future skin ulcer from the delayed effects of radiation therapy. More likely, the skin ulcer may represent a recurrence of skin cancer, and biopsy should be done. Initiating hyperbaric oxygen therapy without confirming the diagnosis may delay appropriate treatment. Alternatively, blindly treating with radiation could cause further unnecessary damage to a soft tissue radionecrosis lesion. There is no evidence that an infection is present. Therefore, antibiotics would be of little to no value at this time but could be appropriate if signs of infection develop, which can be expected in soft tissue radionecrosis lesions. Go to the next page if you knew the correct answer, or click the link image(s) below to further research the concepts in this question (if desired).
Research Concepts: Hyperbaric Soft Tissue Radionecrosis
We update eBooks quarterly and Apps daily based on user feedback. Please tap flag to report any questions that need improvement.
Question 496:
A 17-year-old female is being evaluated for upward gaze palsy. A computed tomography (CT) of the head revealed the presence of a pineal region tumor. The endoscopic third ventriculostomy (ETV) guided biopsy revealed large, epithelioid cells with abundant periodic acid– Schiff positive (PAS+) cytoplasm mixed with mature lymphocytes. Cerebrospinal fluid cytology is negative. Which of the following is the best treatment strategy for this patient?
Choices: 1. 2. 3. 4.
Surgical resection Craniospinal irradiation Localized radiotherapy Chemotherapy combined with ventricular radiotherapy
Answer: 4 - Chemotherapy combined with ventricular radiotherapy
Explanations: The presence of large, epithelioid cells with abundant PAS+ cytoplasm mixed with lymphocytes is characteristic of a germinoma. Germinoma accounts for more than 50% of lesions in the pineal region. Germinomas can be localized or disseminated. Localized pure germinoma is best treated with chemotherapy combined with reduced dose ventricular radiotherapy. Craniospinal irradiation is no longer recommended for localized disease. Germinomas typically have a very good response to radiation therapy and chemotherapy. They are prognostically better than non-germinomatous lesions. Surgery is not indicated for germinoma. Localized radiotherapy is associated with a higher risk of relapse. Go to the next page if you knew the correct answer, or click the link image(s) below to further research the concepts in this question (if desired).
Research Concepts: Germ Cell Seminoma
We update eBooks quarterly and Apps daily based on user feedback. Please tap flag to report any questions that need improvement.
Question 497:
A 68-year-old man with recently diagnosed adenocarcinoma of the pancreatic head presents to the clinic for routine follow-up. He reports that over the last two months, his wife has noticed his skin and sclera have become yellowed; he also notes increased fatigue and generalized pruritis. Physical examination reveals eyelid xanthomas, jaundice, vague abdominal pain, and generalized weakness, with normal vital signs. Laboratory studies obtained are as follows: Reference Test Result Range 135 - 145 Sodium, serum 121 mEq/L mEq/L 3.5 - 5.0 Potassium, serum 3.2 mEq/L mEq/L 98 - 106 Chloride, serum 94 mEq/L mEq/L 23 - 28 Bicarbonate 24 mEq/l mEq/L 0.7-1.3 Creatinine, serum 1.1 mg/dL mg/dL BUN, serum 21 mg/dL 8-20 mg/dL 70-100 Glucose, serum 110 mg/dL mg/dL Alkaline 2100 U/L 36-92 U/L phosphatase Aspartate 276 U/L 0-35 U/L aminotransferase Alanine 197 U/L 0-35 U/L aminotransferase Osmolality, serum 287 280 - 300 mOsm/kg mOsm/kg
H20 H20 1231 140 - 200 Total cholesterol mg/dL mg/dL 1209 50 - 130 LDL cholesterol mg/dL mg/dL What is the most appropriate next step in the management of this patient’s hyponatremia?
Choices: 1. 2. 3. 4.
Obtain a lipoprotein metabolism profile analysis Measure urine sodium level Obtain direct ion-selective electrode potentiometry Administer hypertonic saline
Answer: 1 - Obtain a lipoprotein metabolism profile analysis
Explanations: The correct answer is to obtain a lipoprotein metabolism profile analysis. The patient in the case above has pseudohyponatremia and is displaying signs and symptoms consistent with cholestatic obstruction, likely at the level of the pancreatic head mass. Additionally, liver chemistry enzymes are consistent with a cholestatic pattern of injury. Pseudohyponatremia secondary to cholestasis is a rare disorder and is often associated with increased serum levels of lipoprotein X, an abnormal lipoprotein with the same density of LDL cholesterol. Lipoprotein metabolism profile analysis organizes cholesterols into their separate constituents and can aid in diagnosis. Once serum isotonicity has been confirmed, the measurement of urine sodium level does not provide any additional diagnostic information in the workup of suspected pseudohyponatremia. Direct ion-selective electrode potentiometry (DISE) is indicated for cases of pseudohyponatremia in which there are multiple confounding variables. In most cases, including this one, the likely etiology is clear, and DISE would not provide additional benefit. Hypertonic (3%) saline is indicated only in cases of severe, symptomatic hypotonic hyponatremia. It has no role in the treatment of pseudohyponatremia. Go to the next page if you knew the correct answer, or click the link image(s) below to further research the concepts in
this question (if desired).
Research Concepts: Pseudohyponatremia
We update eBooks quarterly and Apps daily based on user feedback. Please tap flag to report any questions that need improvement.
Question 498:
A 65-year-old smoker presents with a 4month history of recurrent epistaxis from the left nostril associated with blockage of the same side. On rhinoscopy, there was an irregular fleshy granular mass arising from the roof of the nasal cavity. He undergoes a computed tomogram scan, which shows a mass sized 5x5 cm extending from the nasal cavity into the adjacent maxillary sinus. Nuclear medicine study shows that the lesion is MIBGavid. He does not have clinically significant cervical lymphadenopathy. How is this tumor best treated?
Choices: 1. 2. 3. 4.
Chemotherapy Transnasal endoscopic resection Craniofacial resection Radiotherapy
Answer: 2 - Transnasal endoscopic resection Explanations: The patient is diagnosed with esthesioneuroblastoma, as evidenced by the clinical picture and MIBG-avid nuclear medicine scan. The Kadish staging system is an effective means to predict disease-free survival and is widely accepted for esthesioneuroblastoma. Group B tumors extend from the nasal cavity into the paranasal sinus. These are treated by transnasal endoscopic resection or lateral rhinotomy. Craniofacial resection is usually reserved for Kadish C and D tumors. Go to the next page if you knew the correct answer, or click the link image(s) below to further research the concepts in this question (if desired).
Research Concepts: Esthesioneuroblastoma
We update eBooks quarterly and Apps daily based on user feedback. Please tap flag to report any questions that need improvement.
Question 499:
A 63-year-old male who smokes presents with hemoptysis and weight loss. A chest CT shows a 4 x 4 cm central lung mass, with irregular margins and central cavitation. Histology from a biopsy of the mass confirms squamous cell carcinoma of the lung. CT also revealed a 4 cm and a 2 cm heterogeneous left and right adrenal mass, respectively, with high density (14 HU). A positron emission tomography (PET) scan shows metabolically active lung and adrenal masses with a max standardized uptake value (SUV) of 2.2 without other evidence of metastatic spread. What is the best next step in management?
Choices: 1. Surgical resection of the lung mass without further evaluation of the adrenal mass 2. Fine-needle aspiration cytology of the adrenal mass 3. Plasma and urinary metanephrines 4. 24-hour urinary 5-HIAA
Answer: 3 - Plasma and urinary metanephrines Explanations: It is of utmost importance that pheochromocytoma is excluded by biochemical testing of plasma or urinary metanephrines before adrenal biopsy to avoid the potential risk of hypertensive crisis and life-threatening complications. CT-guided fine-needle aspiration (FNA) may be useful when imaging is equivocal; it has not been conclusively characterized as benign, the lesion is hormonally inactive, and most importantly, management of the patient will be altered by information from histology. Therefore, FNA cytology is not useful in the setting of widespread metastatic disease but may have utility in the setting of evaluation for an oligometastatic disease to the adrenal gland. Although FNA cannot differentiate between benign adenoma and adrenocortical carcinoma, it is very useful in confirming the presence of metastatic disease in adrenal lesions with a sensitivity of 80-90% and a positive predictive value of 100%. FNA has the potential for non-diagnostic results (0% to 28%) and complications (2.5% to 13%). Most complications are self-limiting but can include adrenal hemorrhage, pain, pancreatitis, pneumothorax, and hematuria. Urine 5-HIAA is a useful test for secretory neuroendocrine tumors. Go to the next page if you knew the correct answer, or click the link image(s) below to further research the concepts in this question (if desired).
Research Concepts: Adrenal Metastasis
We update eBooks quarterly and Apps daily based on user feedback. Please tap flag to report any questions that need improvement.
Question 500:
A 55-year-old male patient is brought by his sister to the emergency department with a persistent cough, dyspnea, progressive facial and neck swelling. She explains that he has been smoking a pack of cigarettes a day for the last 40 years. Vital signs are blood pressure: 140/80 mmHg, pulse rate: 120 beats per minute, respiratory rate: 27 breaths per minute, and temperature: 37.2 C (98.9 F). Physical examination reveals edema and dilated vessels in the neck and shoulders. Collateral veins are also visible on chest examination. Doppler sonography indicates thrombosis within the superior vena cava. CT scan is remarkable for a mediastinal mass with compression effect over the SVC, and thrombosis. What is the preferred next step management?
Choices: 1. 2. 3. 4.
Bypass grafting with spiral saphenous vein CT-guided biopsy and systemic chemotherapy Airway management, and endovascular recanalization Emergent radiation therapy
Answer: 3 - Airway management, and endovascular recanalization
Explanations: Following a clinical diagnosis, supportive therapy and medical management are commonly initiated. This involves elevation of the patient’s head as a simple maneuver to decrease venous pressure. Further management is guided by the patient’s underlying SVC syndrome etiology. However, this is now reserved for cases in which recanalization through endovascular repair is either not possible or has previously failed. For patients with thrombus related to an indwelling intravascular device, removal should be considered along with anticoagulation therapy and catheterdirected thrombolysis. Interprofessional treatment planning for those with obstruction due to malignancy is important as tumor type, and staging can help to guide appropriate chemotherapy or radiation therapy. Open surgical repair through bypass grafting with spiral saphenous vein, femoral vein, polytetrafluoroethylene (PTFE) graft, or Dacron graft have traditionally been considered to overcome SVC obstruction. With expanding treatment options for both benign and malignant etiology, endovascular therapy is now widely considered the first-line treatment for SVC syndrome. Less invasive endovascular management can offer patients immediate relief of symptoms. Acute or subacute thrombus can be managed with catheterbased thrombolysis or thrombectomy prior to venoplasty and stent placement. Radiation therapy before obtaining a biopsy would obscure the results and is no longer recommended in
the management of acute life-threatening SVC syndrome. Go to the next page if you knew the correct answer, or click the link image(s) below to further research the concepts in this question (if desired).
Research Concepts: Superior Vena Cava Syndrome
We update eBooks quarterly and Apps daily based on user feedback. Please tap flag to report any questions that need improvement.
Section 6 Question 501:
A 72-year-old female patient presented to the emergency department for progressive shortness of breath. Her vital signs are blood pressure of 106/60 mm Hg, heart rate of 88 beats per minute, respiratory rate of 27 breaths per minute, and temperature of 98.6 F (37 C). Her medical history is remarkable for right breast cancer for which she underwent right modified radical mastectomy three years ago. The anterior-posterior chest x-ray is remarkable for bilaterally blunted costophrenic angles. Placement of indwelling pleural catheters was performed. A couple of months later, she presents with painful nodules near the catheter insertion site. What is the preferred management?
Choices: 1. 2. 3. 4.
Epidural catheter placement for pain management Surgical excision for those with greater than 2 cm size Analgesia and radiotherapy Radiofrequency ablation
Answer: 3 - Analgesia and radiotherapy Explanations: Intrapleural catheters (IPCs) are indicated for the palliative treatment and improvement of dyspnea due to a chronic pleural effusion. Malignant pleural effusions (MPE) are the main indication for IPC placement only if they are associated with the respiratory compromise with documentation of improvement of symptoms postpleural fluid removal at least on 1 prior occasion. IPCs are not indicated for the treatment of pleural effusion from a likely curable cause. Most lymphomas respond to therapy and hence do not warrant IPC placement. However, if the patient fails treatment and requires palliative management then an IPC can be considered. It is not uncommon for patients treated with IPC for MPEs to develop painful nodules near the insertion site or entry point into the parietal pleura. Treatment is analgesia and radiotherapy. Mesothelioma is notorious for causing catheter tract metastasis (CTM). The cause of this complication remains unknown. IPC is only for palliative treatment and not therapeutic. Intrapleural chemotherapy is not part of the mesothelioma treatment regimen at this moment. Go to the next page if you knew the correct answer, or click the link image(s) below to further research the concepts in this question (if desired).
Research Concepts: Intrapleural Catheter
We update eBooks quarterly and Apps daily based on user feedback. Please tap flag to report any questions that need improvement.
Question 502:
A 60-year-old woman is diagnosed with ER-positive metastatic breast cancer. She has a past medical history of hypertension, dyslipidemia, and deep venous thrombosis. Her medications include lisinopril, simvastatin, and warfarin. The clinician would like to prescribe tamoxifen for metastatic breast cancer treatment. Which of the following changes in laboratory parameters will most likely occur with tamoxifen therapy in this patient?
Choices: 1. 2. 3. 4.
Increased Increased Increased Increased
LDL-cholesterol serum potassium INR serum creatinine
Answer: 3 - Increased INR Explanations: Tamoxifen is an estrogen receptor antagonist used to treat metastatic breast cancer. Tamoxifen is a potent inhibitor of the CYP2C9 enzyme. Warfarin is primarily metabolized by CYP2C9; thus, tamoxifen increases the risk of bleeding with warfarin. Therefore, the combination of tamoxifen and warfarin is contraindicated when tamoxifen is used as prophylaxis in women at high risk of breast cancer. Tamoxifen may increase the concentration of warfarin requiring warfarin dosage reduction by up to 60% if taken concomitantly. Tamoxifen may decrease LDL levels and does not affect potassium levels. Tamoxifen does not significantly increase serum creatinine and does not require renal dosage adjustment. The change in laboratory parameters that will most likely occur with tamoxifen therapy in this patient is increased INR. Go to the next page if you knew the correct answer, or click the link image(s) below to further research the concepts in this question (if desired).
Research Concepts: Tamoxifen
We update eBooks quarterly and Apps daily based on user feedback. Please tap flag to report any questions that
need improvement.
Question 503:
A 58-year-old female is admitted to the hospital with deep vein thrombosis of the left leg. On further questioning, she admitted to a weight loss of about 25 pounds (11.3 kg) over the past six months. She has noted a painful and pruritic rash around her genitalia. She has not seen her primary care physician over the past several years. Her blood work in the hospital shows fasting blood sugar of 156 mg/dL ( 65 - 95), HbA1c 7.9% ( 4.8- 5.6%), hemoglobin 10.1 g/ dL, creatinine 0.8 mg/dL ( 0.76-1.27). Her physical examination is remarkable for swelling of the left lower extremity. She is noted to have dermatitis around her mouth and perigenital regions. Her fasting glucagon levels are elevated at 848 pg/ ml ( normal 150). A clinical diagnosis of glucagonoma syndrome is made. Which of the following imaging should be ordered next in her case?
Choices: 1. Contrast-enhanced MRI of the abdomen 2. Helical multiphasic contrast-enhanced CT scan of abdomen 3. Somatostatin receptor scintigraphy ( Octreoscan) 4. Functional PET scan with DOTA peptides
Answer: 2 - Helical multiphasic contrast-enhanced CT scan of abdomen
Explanations: Tumor localization is usually started with a helical multiphasic contrast-enhanced CT scan. The sensitivity of a multiphasic CT scan is high at greater than 80% for detecting intrapancreatic neuroendocrine tumors. Contrast-enhanced MRI is performed in the case of indeterminate lesions. They have higher sensitivity for detecting liver metastasis. Somatostatin receptor scintigraphy ( Octreoscan), as well as functional PET imaging, are helpful in the staging of the tumor. Most patients with glucagonoma present in the fifth to the sixth decade of life with similar incidence in males and females. Go to the next page if you knew the correct answer, or click the link image(s) below to further research the concepts in this question (if desired).
Research Concepts: Glucagonoma Syndrome
We update eBooks quarterly and Apps daily based on user feedback. Please tap flag to report any questions that need improvement.
Question 504:
A 55-year-old woman presents to the clinic for a follow-up. She was recently diagnosed with advanced colorectal carcinoma, Eastern cooperative oncology group score 1 with solitary brain metastasis. She has no involvement of the hippocampus and surrounding region. She has a past medical history of ulcerative colitis, taking oral and rectal mesalazine, with a history of multiple exacerbations in the past 2 months. What is the most appropriate management strategy for this patient?
Choices: 1. Stereotactic radiosurgery plus whole-brain radiotherapy 2. Hippocampal avoidance whole-brain radiotherapy 3. Hippocampal avoidance whole brain radiotherapy plus memantine 4. Best supportive care
Answer: 4 - Best supportive care Explanations: Active ulcerative colitis is considered a contraindication to radiotherapy. In addition to idiopathic inflammatory bowel disease, collagen vascular diseases such as systemic lupus erythematosus and Sjogren disease are also conditions that may preclude the use of radiotherapy. Stereotactic radiosurgery without WBRT has been shown to be non-inferior in patients with five to ten metastases. A minimal degree of invasiveness and fewer side effects, when compared to neurocognitive function has been positioned as the major advantages of the procedure. Strategies that have been used to preserve the neurocognitive function include the use of hippocampal avoidance strategies and a combination of HA-WBRT with memantine or donepezil. A randomized control trial has shown preservation of neurocognitive outcomes along with a reduced incidence of adverse effects with the use of hippocampal avoidance whole-brain radiotherapy in combination with memantine without an adverse impact on overall or progression-free survival. The use of intensity-modulated radiotherapy with conformal avoidance of the hippocampus and the parahippocampal region has been proposed to avoid irradiation to the stem cell niche and prevent the development of negative neurocognitive outcomes. The quantum of radiation exposure to the hippocampal stem cell niche is reduced by seven times.
Go to the next page if you knew the correct answer, or click the link image(s) below to further research the concepts in this question (if desired).
Research Concepts: Palliative Radiation Therapy For Brain Metastases
We update eBooks quarterly and Apps daily based on user feedback. Please tap flag to report any questions that need improvement.
Question 505:
A 55-year-old presents to the emergency room for acute bleeding in gums and easy bruising in the last 2 weeks. He was evaluated with labs, which show significantly low fibrinogen with increased coagulation parameters. Peripheral smear was obtained showing > 20% myeloid blasts with significant Auer rods. The patient subsequently underwent a bone marrow biopsy. Cytogenetics report is currently pending. What is the most likely finding on the cytogenetic report?
Choices: 1. 2. 3. 4.
t(8;21) t(15;17) t(6;9) t(9;11)
Answer: 2 - t(15;17) Explanations: From the presentation, the patient likely has DIC from acute promyelocytic leukemia with the presence of worsening coagulopathy and myeloid blasts on smear with significant Auer rods. The hallmark of APL is the presence of t(15;17), a reciprocal translocation between the two chromosomes resulting in the PML-RARA fusion gene. Patients presenting with such a likely APL diagnosis should not wait untll confirmatory genetic testing results are obtained, and treatment with ATRA (All trans-retinoic acid) needs to be initiated immediately. Immediate correction of coagulopathy in bleeding patients with agents like cryoprecipitate should be a priority. Go to the next page if you knew the correct answer, or click the link image(s) below to further research the concepts in this question (if desired).
Research Concepts: Leukemia
We update eBooks quarterly and Apps daily based on user feedback. Please tap flag to report any questions that need improvement.
Question 506:
A 42-year-old woman presents after physical contact with a person with shingles. Three days ago, the patient visited her grandmother, held her hand, and hugged her. Later that evening, she learned that her grandmother had active shingles. The patient’s past medical history is significant for immunosuppression due to chemotherapy and radiotherapy for advanced malignancy. She reports a history of having chickenpox as a child. She is currently asymptomatic. What is the best next step for this patient?
Choices: 1. Passive immunization with varicella-zoster immune globulin 2. Active immunization with varicella vaccine 3. Observation 4. Oral acyclovir
Answer: 1 - Passive immunization with varicella-zoster immune globulin
Explanations: Varicella-zoster immune globulin (VZIG) is indicated for immunocompromised patients who have had close contact with active lesions of shingles. Herpes zoster (shingles) represents the reactivation of the varicella-zoster virus; therefore, it is infectious. Immunocompromised individuals are especially susceptible to this virus. Immediate prophylaxis is recommended. Active immunization with a live vaccine is contraindicated in immunocompromised patients. While prior infection with varicella usually provides lifelong immunity, individuals who are immunocompromised are at higher risk of reinfection. Go to the next page if you knew the correct answer, or click the link image(s) below to further research the concepts in this question (if desired).
Research Concepts: Varicella Zoster (Chickenpox)
We update eBooks quarterly and Apps daily based on user feedback. Please tap flag to report any questions that need improvement.
Question 507:
A 55-year-old woman developed right axillary node enlargement, which has been progressively worsening over the last two months. It is currently the size of a small lemon. A mammogram was completed and was negative with the ultrasound of axilla demonstration pathologic lymphadenopathy in the right axilla, measuring 2.5 cm in size. Positron emission tomography/computed tomography showed increased uptake in the right axillary node along with subpectoral nodes. The bone marrow biopsy was negative. Serum LDH was elevated. Excisional biopsy of right axillary lymph node showed anaplastic lymphoma kinase-negative anaplastic large cell lymphoma, CD 30 positive. She was treated with cyclophosphamide, hydroxydaunorubicin, oncovin, and prednisone (CHOP) regimen, and after four cycles, positron emission tomography/computed tomography showed worsening of lymphadenopathy. What is the next best approach?
Choices: 1. Continue with CHOP regimen for 2 more cycles followed by restaging 2. Proceed to autologous stem cell transplantation 3. Switch therapy to brentuximab vedotin (BV) or BV with CHOP regimen 4. Proceed to allogeneic stem cell transplant
Answer: 3 - Switch therapy to brentuximab vedotin (BV) or BV with CHOP regimen
Explanations: Given that patient has demonstrated refractoriness to cyclophosphamide, hydroxydaunorubicin, oncovin, and prednisone, and the presence of CD 30 positivity, brentuximab vedotin alone or brentuximab vedotin containing chemotherapy is considered one of the standards of care in the second-line setting. Other options in the second-line setting include belinostat, pralatrexate, romidepsin, dihydroxyacetone phosphate, etoposide/ methylprednisolone/cisplatin/cytarabine(ESHAP), gemcitabine/oxaliplatin, ICE, bendamustine, gemcitabine, and lenalidomide. After achieving complete or partial response on secondline therapy, consolidation with autologous stem cell transplant is recommended. Allogeneic stem cell transplant is indicated in the management of patients who relapse after autologous stem cell transplant. Go to the next page if you knew the correct answer, or click the link image(s) below to further research the concepts in this question (if desired).
Research Concepts: ALK Negative Anaplastic Large Cell Lymphoma
We update eBooks quarterly and Apps daily based on user feedback. Please tap flag to report any questions that need improvement.
Question 508:
What is the optimal treatment for a
chordoma?
Choices: 1. 2. 3. 4.
Chemotherapy Radiation therapy Surgery and chemotherapy Surgery and radiation therapy
Answer: 4 - Surgery and radiation therapy Explanations: Chordomas are slow-growing tumors with a high local recurrence rate thus necessitating post-surgical radiation therapy. The slow-growing nature of chordomas makes them relatively resistant to radiation therapy requiring highdose conformal radiation therapy such as proton beam therapy for post-resection treatment. Chemotherapy is uncommonly used for the treatment of chordomas as they are resistant to most current conventional chemotherapeutic agents. Complete en bloc resection of a chordoma with clean margins can increase 5-year survival to 65% from 50% 5-year survival for resection with positive margins. Go to the next page if you knew the correct answer, or click the link image(s) below to further research the concepts in this question (if desired).
Research Concepts: Chordoma
We update eBooks quarterly and Apps daily based on user feedback. Please tap flag to report any questions that need improvement.
Question 509:
A 66-year-old male patient was admitted to the emergency department with sudden onset abdominal pain, which woke him from his sleep. His past medical history is significant for hypertension which is well controlled with hydrochlorothiazide, and peptic ulcer disease, which is well controlled with omeprazole. He has chronic mesenteric ischemia involving both the celiac and mesenteric arteries for which he underwent endovascular stenting two years ago, and T2 N0 M0 gastric cancer was diagnosed six months ago. The patient has been undergoing chemotherapy for his gastric cancer. He rates the pain as 10/10 and states that it radiates to his back. It is accompanied by nausea, although he has not had any emesis. He is tachycardic to 115 beats per minute, has a blood pressure of 90/50 mmHg, and his oxygen saturation is 98% on room air. His abdomen is rigid with rebound tenderness and guarding. His laboratory test results are still pending. An abdominal x-ray reveals free air under the diaphragm. What is the likely etiology of the patient's stomach perforation?
Choices: 1. 2. 3. 4.
Dislodgement of the patient's celiac trunk stent Exacerbation of the patient's peptic ulcer disease The continued growth of the tumor Tumor necrosis due to the chemotherapy
Answer: 4 - Tumor necrosis due to the chemotherapy Explanations: The patient had a successful stenting procedure several years ago. While malignancy increases the risk of a thromboembolic event, this should not be the top diagnosis on the differential list. This patient likely has a perforation secondary to the tumor necrosis due to the chemotherapy, given the sudden onset of his symptoms concerning him starting his treatment. The patient's peptic ulcer disease has been well controlled with omeprazole. He does not report any symptoms such as abdominal pain associated with meals that would raise concern for the recurrence of his ulcers. Given the close temporal association of starting chemotherapy followed by sudden onset abdominal pain and peritoneal signs, the diagnosis highest on the differential currently should be gastric perforation secondary to necrosis of the tumor. T2 N0 M0 is gastric cancer that has penetrated to the level of muscularis propria. While a perforation secondary to continued growth is possible, cancer would have had to grow through the remainder of the muscular propria, the subserosa, and the serosa. The more likely explanation is that tumor necrosis secondary to the chemotherapy has led to a perforation of the patient's stomach leading to his peritonitis and hypotension. Chemotherapy is intended to kill tumor cells. However, when the tumor is within a hollow viscus organ, such as the stomach, the necrosis of the tumor may lead to perforation. In this patient, whose symptoms occurred shortly after starting chemotherapy, the diagnosis of
perforation of the stomach secondary to tumor necrosis must be the assumed diagnosis until proven otherwise. Go to the next page if you knew the correct answer, or click the link image(s) below to further research the concepts in this question (if desired).
Research Concepts: Gastric Perforation
We update eBooks quarterly and Apps daily based on user feedback. Please tap flag to report any questions that need improvement.
Question 510:
A 44-year-old female who was diagnosed with breast cancer three months ago presents to the emergency department with three weeks of right hip pain and difficulty bearing weight. On further questioning, she says she had a lumpectomy with negative margins and has never been diagnosed with any metastasis. She denies trauma. Imaging shows the right pathologic femoral neck fracture. What is the best next step?
Choices: 1. 2. 3. 4.
Right Right Right Right
hip hemiarthroplasty total hip arthroplasty femur open reduction and internal fixation femoral neck biopsy
Answer: 4 - Right femoral neck biopsy Explanations: A biopsy must be performed prior to any surgical intervention to rule out a primary tumor. Failure to do so may lead to mismanagement and contamination of the surgical wound with the tumor leading to unnecessary surgery and possibly amputation. A history of cancer does not preclude one from obtaining a biopsy. While the risk of having two separate cancers is rare, the possibility is still present; therefore, a biopsy is indicated. When performing a biopsy, all principles of a proper biopsy should be followed to avoid contamination. The other choices describe reasonable surgical options, which are determined based on surgeon preference, but only after a tissue diagnosis is obtained. A biopsy may be performed by an interventional radiologist or a surgeon. Go to the next page if you knew the correct answer, or click the link image(s) below to further research the concepts in this question (if desired).
Research Concepts: Pathologic Fractures
We update eBooks quarterly and Apps daily based on user feedback. Please tap flag to report any questions that
need improvement.
Question 511:
A 55-year-old Asian man presents to the hospital with a history of nasal obstruction for two months. He denies fever, weight loss, or night sweats. On initial examination, there is a large anterior septal defect, with nasal crusting and malodorous discharge. Examination under anesthesia reveals exposed bone at the right frontal process of the maxilla and posterior nasal floor. A biopsy is taken, which demonstrates diffuse lymphomatous infiltrates, angiocentric, and angiodestructive growth pattern. Immunophenotyping reveals tumor cells positive in CD56, EBER-ISH, and TIA-1. What virus is mostly associated with this malignancy?
Choices: 1. 2. 3. 4.
Adenovirus Epstein-Barr virus Herpes simplex virus-1 Varicella-zoster virus
Answer: 2 - Epstein-Barr virus Explanations: Irrespective of the ethnic origin of patients, there is a very strong association of Epstein-Barr virus (EBV) infection in almost all cases of extranodal NK/T-cell lymphoma, nasal type, suggesting an important role of the virus in the etiology of this disease. EBV downregulates human non-homologous end-joining factor 1 (NHEJ1), an important DNA repair factor for the NHEJ pathway that mediates repair of double-stranded breaks, resulting in genome-wide instability with the onset of this disease. By expressing EBNA1, LMP1, LMP2A, and LMP2B (latency phase 2), it modulates cell signaling, and forms barriers to apoptotic signals to escape from T-cellmediated immune response. The EBV BART RNAs, which are highly transcribed in extranodal NK/T-cell lymphoma, are thought to play an important role in driving the disease and immune evasion. Go to the next page if you knew the correct answer, or click the link image(s) below to further research the concepts in this question (if desired).
Research Concepts: Extranodal NK-Cell Lymphoma
We update eBooks quarterly and Apps daily based on user feedback. Please tap flag to report any questions that need improvement.
Question 512:
A 16-year-old boy is brought in by his mother with a history of an enlarging mass in the right arm. The mass is currently measuring 6 x 5 cm and is hard and not tender to touch. The patient is referred to a surgeon and a biopsy is obtained. The pathology shows a mesenchymal tumor which is positive for t(X;18)(p11.2;q11.2) translocation. Staging studies are negative for metastatic disease. What is the next best step in management?
Choices: 1. Neoadjuvant chemotherapy, preoperative radiation, and then limb-sparing surgery 2. Preoperative radiation followed by limb-sparing surgery 3. Limb sparing surgery followed by adjuvant chemotherapy 4. Radiotherapy alone
Answer: 1 - Neoadjuvant chemotherapy, preoperative radiation, and then limb-sparing surgery
Explanations: This is a pediatric age group patient with a diagnosis of localized synovial sarcoma. The role of neoadjuvant chemotherapy or adjuvant chemotherapy for synovial sarcoma is very well established in the pediatric age group. However, chemotherapy alone cannot substitute for surgical resection. Preoperative radiation followed by limb-sparing surgery is an integral part of treatment. In pediatric patients with synovial sarcoma, this is usually combined with neoadjuvant or adjuvant chemotherapy. t(X;18)(p11.2;q11.2) translocation is characteristic of synovial sarcoma where the SS18 gene fuses with the SSX gene on X-chromosome. Other options listed here are not standard of care. Go to the next page if you knew the correct answer, or click the link image(s) below to further research the concepts in this question (if desired).
Research Concepts: Synovial Cell Sarcoma We update eBooks quarterly and Apps daily based on user feedback. Please tap flag to report any questions that need improvement.
Question 513:
A 31-year-old man presents to the clinic for follow up. He has a history of acromegaly, for which he takes octreotide. He is currently complaining of skin rash and flushing of the face. An octreotide scan is planned. Which of the following is the next best step in the management of this patient?
Choices: 1. 2. 3. 4.
Perform the scan now Skip octreotide on the day of the scan Stop octreotide for one week before the scan Stop octreotide for 4-5 months before the scan
Answer: 4 - Stop octreotide for 4-5 months before the scan
Explanations: Patients on octreotide therapy or somatostatin must stop treatment. Long-acting octreotide should be stopped 4-6 months before the scan. These medications may have an effect on the test results. Taking octreotide is not a contraindication for the scan. Go to the next page if you knew the correct answer, or click the link image(s) below to further research the concepts in this question (if desired).
Research Concepts: Octreotide Scan
We update eBooks quarterly and Apps daily based on user feedback. Please tap flag to report any questions that need improvement.
Question 514:
A patient is referred to after cholecystectomy as the pathological report showed the incidental finding of gallbladder cancer. The full evaluation shows T2N1M0. Which of the following is not considered an adjuvant option by the current National Comprehensive Cancer Network Clinical Practice Guidelines in Oncology?
Choices: 1. Chemotherapy alone for six months with single-agent gemcitabine or leucovorin-modulated 5- fluorouracil (5-FU) 2. Chemoradiation for four cycles of capecitabine plus gemcitabine followed by concurrent radiation therapy plus oral capecitabine 3. Concurrent radiotherapy plus infusional 5-FU 4. Observation
Answer: 4 - Observation Explanations: Published in the ESPAC-3 Trial with median overall survival for chemotherapy with gemcitabine or leucovorin-modulated 5- fluorouracil being 43.1 months versus 35.2 months in the observation group. This is not statistically significant. It was published in a single-arm SWOG-S0809 Trial with an expected mean overall survival of 35 months without a comparison group. This is based on adjuvant chemoradiation therapy from other gastrointestinal malignancy extrapolation data. Adjuvant therapy should be considered for patients with resected high-risk gallbladder cancer stage T2 or higher, lymph node, and/or margin-positive benefit confirmed median overall survival extension on meta-analysis. Go to the next page if you knew the correct answer, or click the link image(s) below to further research the concepts in this question (if desired).
Research Concepts: Gallbladder Cancer
We update eBooks quarterly and Apps daily based on user feedback. Please tap flag to report any questions that need improvement.
Question 515:
A 54-year-old man with a history of metastatic melanoma presents to the clinic for follow up. After excision of the lesion, he is treated with vemurafenib. What is the most likely amino acid substitution that caused this patient's melanoma?
Choices: 1. 2. 3. 4.
Valine to glutamic acid Glutamic acid to valine Lysine to valine Valine to lysine
Answer: 1 - Valine to glutamic acid Explanations: Vemurafenib is used to treat melanomas with BRAF V600E positive mutations. BRAF V600E is a mutation that causes over-activation of BRAF, which leads to uninhibited growth of the cell. The BRAF V600E is a substitution of glutamic acid for valine. The substitution of lysine for valine is a less common mutation BRAF V600K. Go to the next page if you knew the correct answer, or click the link image(s) below to further research the concepts in this question (if desired).
Research Concepts: Vemurafenib
We update eBooks quarterly and Apps daily based on user feedback. Please tap flag to report any questions that need improvement.
Question 516:
A 65-year-old male presents with a 3month history of generalized body pain in his limbs and back and chronic cough. He is lethargic and has lost 5 kilograms (11 pounds) in the past 4 months. His past medical history is significant for osteosarcoma of his proximal tibia, which was diagnosed 10 years ago, for which he was treated with neoadjuvant chemotherapy, surgical resection, and adjuvant chemotherapy. Physical examination does not reveal any palpable masses or localized tenderness, and neurological examination is unremarkable. Which of the following is pertinent to the further findings?
Choices: 1. Macrometastasis at the time of diagnosis is common and might be missed in this patient 2. The risk for micrometastasis is lower in his age group 3. Bony metastasis in the mid-tibia does not affect the overall survival 4. Chest computed tomography (CT) is indicated
Answer: 3 - Bony metastasis in the mid-tibia does not affect the overall survival
Explanations: Osteosarcoma is a highly malignant osteogenic tumor that can develop in any bone. It tends to develop near the metaphysis of long bones in young patients. The most common sites are the distal femur, proximal tibia, and proximal humerus, with high bone turnover. In adults, the axial skeleton is more common, where previous irradiation or metabolic disease of the bone is often associated. Common genetic changes are not present to explain the growth of this tumor type; however, 70% of cases demonstrate some level of chromosomal abnormality. Alterations in p53, Rb1, and DNA repair/surveillance genes are present in patients with Li-Fraumeni, Bloom, and Rothmund-Thomson syndrome, all linked to increased rates of osteosarcoma. The lungs following bones are the most common sites for metastasis. Macrometastasis occurs in up to 20% of patients with osteosarcoma at the time of diagnosis. Adults older than 60 years of age are at higher risk for metastasis. Bony metastases in the same bone or adjacent to the primary tumor site are associated with a worse prognosis. Go to the next page if you knew the correct answer, or click the link image(s) below to further research the concepts in this question (if desired).
Research Concepts:
Primary Bone Cancer
We update eBooks quarterly and Apps daily based on user feedback. Please tap flag to report any questions that need improvement.
Question 517:
A 25-year-old man is brought to the emergency department with acute onset headache and confusion. An MRI brain with and without contrast shows a large contrast-enhancing mass in the cerebellar region and enlarged lateral ventricles with hydrocephalus. The patient undergoes resection of the mass, and the pathology is consistent with a primitive neuroectodermal tumor. At a follow-up visit, the patient reports a family history of colon cancer in his father and his older brother. A colonoscopy is ordered, and the patient is found to benign lesions. A mutation in which of the following genes is most likely responsible for the patient’s condition?
Choices: 1. 2. 3. 4.
CDH1 APC PTCH1 CDKN2A
Answer: 2 - APC Explanations: APC gene is a tumor suppressor gene, which is associated with the development of colon polyps and colon cancer along with brain tumors, especially medulloblastoma. The negatively regulated the WNT signaling pathway that is involved in the regulation and co-ordination of cell-cell adhesion and gene transcription. The APC protein is also involved in apoptosis. CDH1 encodes for E-cadherin that regulates contact inhibition by binding to a key component of the WNT signaling pathway, beta-catenin. Germline loss-offunction mutation of the CDH1 gene is associated with developing familial gastric carcinoma. PTCH1 tumor suppressor gene encodes protein patched homolog 1 that negatively regulated the hedgehog signaling pathway. Germline loss-of-function mutation of the PTCH1 gene is associated with developing Gorlin syndrome and a high risk of developing basal cell carcinoma and medulloblastoma. CDKN2A encodes for two tumor suppressor proteins, p16/INK4a, and ARF, which augments RB function and stabilizes p53, respectively. Loss-of-function germline mutations in this gene are seen in autosomal dominant familial melanoma. Go to the next page if you knew the correct answer, or click the link image(s) below to further research the concepts in this question (if desired).
Research Concepts:
Tumor-Suppressor Genes
We update eBooks quarterly and Apps daily based on user feedback. Please tap flag to report any questions that need improvement.
Question 518:
A 67-year-old man with a diagnosis of malignant melanoma with BRAF V600E mutation, with metastasis to the brain who is currently on dabrafenib and trametinib. He had good control of his disease until recently when he developed a worsening headache. An MRI of the brain showed one enlarging lesion. All his other lesions were stable. His case was discussed at the multidisciplinary tumor board and recommended he receive stereotactic radiosurgery (SRS) to the brain. Which of the following would be the best next step regarding his dabrafenib and trametinib?
Choices: 1. Continue dabrafenib and trametinib during his stereotactic radiosurgery (SRS) 2. Stop dabrafenib with or without trametinib 3 days prior to stereotactic radiosurgery (SRS) and start them back at least a day after SRS 3. Continue dabrafenib, but stop trametinib 4. Change the therapy with dabrafenib and trametinib to a different BRAF and MEK inhibitor combination
Answer: 2 - Stop dabrafenib with or without trametinib 3 days prior to stereotactic radiosurgery (SRS) and start them back at least a day after SRS
Explanations: BRAF inhibitors are known to cause radiosensitization of normal tissues and thus an increased risk of side effects. Hold treatment with a BRAF with or without a MEK inhibitor for one to three days before and one day after SRS, which appears to minimize the risk of significant toxicity. Continuing BRAF inhibitors during SRS therapy can increase the side effect, especially of the skin and mucosa, as well as other organs. Changing to a different BRAF and MEK inhibitor doesn't decrease the associated toxicity with SRS. Go to the next page if you knew the correct answer, or click the link image(s) below to further research the concepts in this question (if desired).
Research Concepts: Malignant Melanoma Metastatic To The Central Nervous System
We update eBooks quarterly and Apps daily based on user feedback. Please tap flag to report any questions that need improvement.
Question 519:
A 38-year-old woman presents to the clinic with a complaint of left shoulder pain for six months. She denies trauma or heavy lifting. She has a past medical history of pituitary tumor, and her father died of pancreatic cancer at age 45. Shoulder XR revealed distal clavicular thinning. On laboratory analysis, serum calcium level was 11.6 mg/dl, serum albumin level was 5.1 g/dl, serum alkaline phosphatase level was 860 IU/l, and serum vitamin D level was 35 ng/mL. Which one of the following is the pathogenesis of the patient's bone condition?
Choices: 1. Thyroid hormone stimulates bone resorption 2. Excess cortisol level causes decreased intestinal calcium absorption, decreased bone formation, increased bone resorption, and decreased renal calcium reabsorption 3. Parathyroid hormone activates osteoclast cells result in bone resorption with the destruction 4. Estrogen deficiency causes bone resorption
Answer: 3 - Parathyroid hormone activates osteoclast cells result in bone resorption with the destruction
Explanations: Osteitis fibrosa cystica (OFC), a skeletal disorder caused by an overproduction of parathyroid hormone (PTH) from the overactive parathyroid gland(s), is characterized by the occurrence of bone pain associated with the finding of specific radiographic changes such as increased subperiosteal bone resorption in the distal third of the radius and middle phalanges, distal clavicular thinning, “salt and pepper” skull, bone cysts, and brown tumors in long bones and generalized osteopenia are seen in 5% of cases. PTH stimulates the activity of osteoclast cells that breakdown bone. The overactivity of the parathyroid glands called primary hyperparathyroidism (PHP) can be triggered by a parathyroid adenoma, hereditary factors, parathyroid carcinoma, or renal osteodystrophy. Hereditary diseases that caused OFC include familial hyperparathyroidism, multiple endocrine neoplasia type 1 (MEN Type 1) and type 2a ( MEN Type 2a), and hyperparathyroidism-jaw tumor syndrome. MEN Type 1 and type 2a are an autosomal dominant disorder and the most common hereditary form of hyperparathyroidism, affecting about 95% of genetic cases of OFC, and also tends to affect younger patients than other forms. MEN type 1 = Parathyroid tumors, pancreatic tumors, and pituitary tumors. MEN type 2a = Medullary thyroid cancers (MTC), pheochromocytoma, and parathyroid tumors.
Go to the next page if you knew the correct answer, or click the link image(s) below to further research the concepts in this question (if desired).
Research Concepts: Osteitis Fibrosa Cystica
We update eBooks quarterly and Apps daily based on user feedback. Please tap flag to report any questions that need improvement.
Question 520:
A 65-year-old male presents for follow up. For the past four months, he has been complaining of lower limb weakness, most prominent when standing up from a sitting position and when climbing stairs. This is accompanied by a dull ache in his hip and legs. He has a history of coronary artery disease and a 45-pack year history of smoking. On examination, he is awake, alert, and oriented to time, place, and person. Pupillary light reflex is normal, with full and equal extraocular muscle movements. The peripheral muscles have normal bulk and tone, manual muscle testing showed weakness in flexion and extension of the proximal leg muscles, with good strength on knee flexion and extension on both sides. Muscle strength improves with repeated testing. Knee jerk and Achilles reflex are absent. What is the most likely underlying mechanism of this patient's presentation?
Choices: 1. Inhibition of acetylcholine release at the presynaptic clefts located at the myoneural junction 2. Reduction of acetylcholine receptors at the postsynaptic nerve terminal due to antibody-mediated destruction of receptors 3. Reduced acetylcholine release from the presynaptic nerve terminals due to antibodies to presynaptic voltage-gated calcium channels 4. Decreased release of neurotransmitter from nerve endings by preventing the fusion of the synaptic vesicle with the neuronal plasma membrane
Answer: 3 - Reduced acetylcholine release from the presynaptic nerve terminals due to antibodies to presynaptic voltage-gated calcium channels
Explanations: Lambert-Eaton myasthenic syndrome (LES) resembles myasthenia gravis but is it does not involve the eyes or extraocular muscles. Lambert-Eaton myasthenic syndrome is a disorder of reduced acetylcholine release from the presynaptic nerve terminals due to antibodies to voltage-gated calcium channels in the presynaptic neuronal cell membrane. 50% to 70% of patients have identifiable neoplasia, most commonly small cell lung cancer (SCLC). Treatment involves addressing the underlying malignancy, intravenous immunoglobulin (IVIG), plasmapheresis, and 3,4-diaminopyridine. Go to the next page if you knew the correct answer, or click the link image(s) below to further research the concepts in this question (if desired).
Research Concepts: Lambert Eaton Myasthenic Syndrome
We update eBooks quarterly and Apps daily based on user feedback. Please tap flag to report any questions that need improvement.
Question 521:
A 16-year-old male undergoes a bone marrow transplant for a hematologic malignancy. Ten days after the transplant, he presents with a pruritic and painful maculopapular rash involving the nape of the neck, palms, soles, ears, and shoulders. He also reports fever, fatigue, nausea, vomiting, abdominal pain, severe diarrhea, and anorexia. Labs demonstrate transaminitis, rising serum bilirubin, and cytopenias. Stool is negative for Clostridium difficile. Histology of the rash reveals exocytosed lymphocytes, dyskeratotic epidermal keratinocytes, follicular involvement, satellite lymphocytes adjacent to or surrounding dyskeratotic epidermal keratinocytes, and dermal perivascular lymphocytic infiltration. The patient moderately improves with prednisone. What is the most appropriate drug to initiate for this patient?
Choices: 1. 2. 3. 4.
Rituximab Tacrolimus Pentostatin Ecromeximab
Answer: 2 - Tacrolimus Explanations: Clinically significant acute graft-versus-host disease (GVHD) occurs in patients who have received an allogeneic hematopoietic cell transplant (HCT). Classic acute GVHD manifests within 100 days of the HCT. It can occur despite intense prophylaxis with immunosuppressants. The exact incidence can vary between 10 to 50 percent. Risk factors for GVHD include the degree of HLA disparity, donor and recipient sex disparity, intensity of the transplant conditioning regimen, type of the prophylaxis used, source of the graft, etc. Tacrolimus is used for the prevention and management of GVHD. It blocks T-cell proliferation by inhibiting its key signaling phosphatase calcineurin, hence DNA synthesis inhibition. For prevention, tacrolimus 0.03 mg/kg/day lean body weight is administered as a continuous infusion. Prophylaxis should begin atleast 24 hours prior to stem cell infusion and continued until oral medication is tolerated. Calcineurin inhibitors, both cyclosporine and tacrolimus, are used to treat GVHD in combination with prednisone. Common regimens include prednisone 1 mg/kg/day plus cyclosporine 6 mg/kg twice daily every other day or prednisone 1 mg/kg/day plus tacrolimus 0.1 mg/kg/day. Monitoring for hypomagnesemia, hypertension, and nephrotoxicity is very important. Cases of thrombotic microangiopathy and neurotoxicity have been reported. Caution should be exercised in combining these drugs with other potentially nephrotoxic drugs. Close monitoring of serum drug levels is recommended.
Go to the next page if you knew the correct answer, or click the link image(s) below to further research the concepts in this question (if desired).
Research Concepts: Calcineurin Inhibitors
We update eBooks quarterly and Apps daily based on user feedback. Please tap flag to report any questions that need improvement.
Question 522:
A 76-year-old man with nasopharyngeal carcinoma presents to the office with complaints of progressive shortness of breath and difficulty swallowing his food. He has refused chemotherapy/radiation therapy in the past due to the belief that it “will only kill him faster.” He is willing to undergo additional procedures or treatments if it helps improve his quality of life, as he wants to feel comfortable enough to enjoy life with his new grandson. He had already been evaluated by an outpatient pulmonologist for shortness of breath and was found to have tracheal stenosis on recent bronchoscopy. Which of the following is the next best step in the management of this patient?
Choices: 1. 2. 3. 4.
Recommend palliative chemotherapy/radiation again Tracheal stent placement Corticosteroids Hospice evaluation
Answer: 2 - Tracheal stent placement Explanations: Treatment options for laryngotracheal stenosis (LTS) include endoscopic dilatation, surgery, stent placement, laser therapy, or immunosuppression to restore airway patency, depending on the underlying etiology as well as degree and complexity of stenosis. In this patient with stenosis, possibly due to malignancy, palliative radiation therapy, immunotherapy, or chemotherapy could be options. However, if he has symptomatic stenosis, stenting would still first be required to maintain patent airway during such treatment. Tracheal stenting is a palliative option for patients with advanced and unresectable cancer-causing airway obstruction. Stenting is a challenging and risky approach that requires a risk-benefit conversation with the patient. Higher performance status prior to surgery has been linked to better patient outcomes. A Japanese study by Matsuo et al. aimed to clarify indications for stent placement, which include: 1) severe central airway obstruction with dyspnea and flow limitation on a flowvolume curve, 2) prognosis will be prolonged by stent placement, and 3) peripheral airways and lungs are intact. Go to the next page if you knew the correct answer, or click the link image(s) below to further research the concepts in this question (if desired).
Research Concepts:
Laryngotracheal Stenosis
We update eBooks quarterly and Apps daily based on user feedback. Please tap flag to report any questions that need improvement.
Question 523:
A 6-year-old boy is brought to the clinic because of a lump in his neck. The mother describes this lump as slowly growing over the past few months. She says that she noticed her son was tired recently and complained of feeling hot. She also states that his clothes are drenched at times. An ultrasound shows an enlarged lymph node in the neck measuring 3 x 2.5 cm with loss of internal architecture. A few additional enlarged lymph nodes are noted adjacent to the neck lump. A biopsy of the lump is scheduled. Which of the following is the next best step in the management of this patient?
Choices: 1. 2. 3. 4.
FDG PET scan MRI of the neck Repeat ultrasound of the neck CT scan of the neck, chest, abdomen, and pelvis
Answer: 1 - FDG PET scan Explanations: FDG PET scans are the standard of care imaging modality to stage and assess response to treatment in adult and pediatric lymphoma cases. FDG PET scans are very sensitive and provide information on the metabolic activity of neoplastic tissue. This allows the assessment of lesions beyond the anatomical size criteria. FDG PET scans allow a whole-body assessment of tumor burden and accurate staging. The metabolic activity of different lesions has been associated with prognosis. A repeat ultrasound is not helpful, as well as an MRI of the neck. Whole-body MRI is used in some centers but is inferior to FDG PET scans. A CT scan of the neck, chest, abdomen, and pelvis may stage the patient but is also inferior to an FDG PET scan which is the standard of care for Lymphoma staging and restaging. Go to the next page if you knew the correct answer, or click the link image(s) below to further research the concepts in this question (if desired).
Research Concepts: Nuclear Medicine Pediatric Assessment, Protocols, And Interpretation
We update eBooks quarterly and Apps daily based on user feedback. Please tap flag to report any questions that need improvement.
Question 524:
A 56-year-old patient who previously presented to the emergency department with abdominal pain, nausea, vomiting, indigestion, weakness, anorexia, loss of appetite, and weight loss. Physical examination demonstrates jaundice, right upper quadrant pain, and Courvoisier sign. A blood test is done which shows elevated levels of carbohydrate antigen 19-9. After the first cycle of gemcitabine plus cisplatin protocol, there is worsening acute on chronic unbearable abdominal pain despite oral oxycodone. The patient is admitted for pain control. What next step should be recommended for this patient's treatment?
Choices: 1. Switch chemotherapy as obviously this is considered a progression of the disease 2. Refer to radiation oncology for palliative external beam radiation therapy with a radiosensitizer 3. Add a third chemotherapy drug 4. Discharge home with an increased dose of oxycodone
Answer: 2 - Refer to radiation oncology for palliative external beam radiation therapy with a radiosensitizer
Explanations: The patient has the best evidence standard of care gemcitabine plus cisplatin by the ABC-02 study, and tumor response was assessed by RECIST criteria with imaging at 12 weeks. External beam radiation therapy (EBRT) with a sensitizer (5-FU) is an appropriate palliative intervention but rarely will achieve tumor control. PEGF regimen has similar median overall survival of 12.1 months to EBRT but with significantly more toxicities. Pain management by opioid should be achieved but will not affect his pain source. A celiac blockade is offered for those who can not achieve pain control with opioids and have failed palliative radiation therapy. Go to the next page if you knew the correct answer, or click the link image(s) below to further research the concepts in this question (if desired).
Research Concepts: Gallbladder Cancer
We update eBooks quarterly and Apps daily based on user feedback. Please tap flag to report any questions that need improvement.
Question 525:
A 57-year-old postmenopausal woman presents with fullness in her pelvis but denies any associated symptoms. Upon examination, there is a large mass on the left wall of the vagina. On palpation, the mass is noted to extend to the pelvic sidewall. Biopsy shows squamous cell carcinoma. Which additional therapy has been shown to enhance patient response to treatment?
Choices: 1. 2. 3. 4.
Cisplatin Methotrexate Granulocyte colony-stimulating factor Doxorubicin
Answer: 1 - Cisplatin Explanations: Concomitant chemoradiation has shown promise in the treatment of stage III and IV vaginal cancer. The above patient has stage III vaginal cancer due to extension in the pelvic sidewall. Cisplatin has been one of the agents studied to enhance the effectiveness of radiation therapy. Other agents, such as 5-FU and mitomycin, have also been shown to be radiosensitizing chemotherapeutic agents. Cisplatin interferes with DNA replication by interfering with cross-linking with purine bases. Certain chemotherapy agents sensitize cancerous cells to the damaging effects of radiation. Cisplatin is given intravenously. Cisplatin is one of the most common chemotherapeutic drugs used in gynecologic cancers. Side effects include nephrotoxicity, neurotoxicity, nausea and vomiting, ototoxicity, and electrolyte disturbances. Go to the next page if you knew the correct answer, or click the link image(s) below to further research the concepts in this question (if desired).
Research Concepts: Vaginal Cancer
We update eBooks quarterly and Apps daily based on user feedback. Please tap flag to report any questions that need improvement.
Question 526:
Which of the following drugs are most effective in the treatment of acrokeratosis paraneoplastica?
Choices: 1. 2. 3. 4.
Cyclosporine Methotrexate Systemic retinoids Secukinumab
Answer: 3 - Systemic retinoids Explanations: Management of acrokeratosis paraneoplastica mainly involves treatment of the primary malignancy. Among the drug treatments for the conditions, one of the options considered being effective is systemic retinoids. Other systemic drugs reported being effective include systemic corticosteroids. Topical treatment options include topical steroids, vitamin D derivatives, salicylic acid, emollients, and antifungals. Go to the next page if you knew the correct answer, or click the link image(s) below to further research the concepts in this question (if desired).
Research Concepts: Acrokeratosis Paraneoplastica
We update eBooks quarterly and Apps daily based on user feedback. Please tap flag to report any questions that need improvement.
Question 527:
A 54-year-old non-smoker female has recently been diagnosed with a tumor of the hard palate. She undergoes a CT scan of the head and neck prior to surgery for staging purposes, which reveals the spread of the tumor to the pterygopalatine fossa. A subsequent excisional biopsy of the tumor shows small cells of secretory nature with hyperchromatic nuclei and minimal cytoplasm arranged in a cylindrical pattern within a hyaline stroma interspersed with cyst-like spaces. Which of the following is the most common mechanism by which the tumor spreads the pterygopalatine fossa?
Choices: 1. Perineural spread through the greater and lesser palatine branches of the maxillary nerve 2. Hematological spread via the greater palatine branch of the maxillary artery 3. Perineural spread via the glossopharyngeal nerve 4. Hematological spread via the axillary vein
Answer: 1 - Perineural spread through the greater and lesser palatine branches of the maxillary nerve
Explanations: The pterygopalatine fossa is known as the neuromuscular crossroad containing many important neurovascular structures like the maxillary nerve (V2), pterygopalatine ganglion, terminal part of the maxillary artery, veins as well as their associated branches. Adenoid cystic carcinoma is known for its key feature of invasion through perineural tissues. Adenoid cystic carcinoma is known for its neurotropism and very rarely spreads through lymphatics and vascular structures. The greater palatine and lesser palatine nerves are branches of the maxillary nerve which innervates posterior palate. It is via perineural spread along these nerves by which adenoid cystic carcinoma of the hard palate most commonly spread to the pterygopalatine fossa. Go to the next page if you knew the correct answer, or click the link image(s) below to further research the concepts in this question (if desired).
Research Concepts: Adenoid Cystic Cancer
We update eBooks quarterly and Apps daily based on user feedback. Please tap flag to report any questions that need improvement.
Question 528:
A 51-year-old male is admitted to the hospital for his next chemotherapy dose for a recently diagnosed non-Hodgkin lymphoma. The patient has no other significant past medical history. On his 4th day of hospitalization, the patient suddenly develops difficulty breathing, muscle pain, and a significant worsening of his lower limb edema without any fevers or chills. The patient had a blood pressure of 155/94 mmHg 2 hours ago. When the nurse inflates the blood pressure cuff to check the blood pressure, the patient develops muscle spasms in the same upper limb. He is in mild respiratory distress, and crackles can be auscultated throughout the lung fields. The patient has a respiratory rate of 24 breaths per minute and his neck veins seem to be distended. His heart rate is 112 beats per minute with a regular rhythm. Bilateral 2+ edema in the lower limbs is also noted. Lab reports include a serum creatinine of 3.4 mg/dl, a leukocyte count of 12,000 cells/mm3, a hemoglobin of 9 g/dl, platelet count of 20,000 cells/mm3, sodium of 131 mEq/L, potassium of 6.3 mEq/L, bicarbonate of 15 mEq/L, blood urea nitrogen of 77 mg/dl, calcium of 6.8 mg/dl, phosphorus of 8.6 mg/dl, uric acid of 19 mg/dl and albumin of 4 g/dl. Urinalysis reports trace blood and 2+ protein levels. Urine microscopy revealed two red blood cells per high power field, 4-10 white blood cells per high power field, uric acid crystals, and granular casts. In addition to hemodialysis, which of the following is the next most appropriate next step in management?
Choices: 1. A drug that is a competitive inhibitor of xanthine oxidase 2. A drug which is a recombinant uricase catalyzing uric acid to allantoin
3. A drug that binds and stabilizes tubulin to inhibit microtubule polymerization, impairing neutrophil chemotaxis and degranulation 4. A drug that inhibits the reabsorption of uric acid in the distal convoluted tubule
Answer: 2 - A drug which is a recombinant uricase catalyzing uric acid to allantoin
Explanations: In a patient with hyperuricemia due to tumor lysis syndrome, the most appropriate way to lower the uric acid levels rapidly is the use of rasburicase, which is a drug that is a recombinant uricase catalyzing uric acid to allantoin which is more water-soluble than uric acid. Because rasburicase also leads to the formation of hydrogen peroxide, treated patients are at risk for methemoglobinemia and hemolytic anemia. Therefore, patients should be checked for G6PD deficiency. Allopurinol and febuxostat work by limiting uric acid formation. Neither of these drugs metabolizes previously generated uric acid. Thus these drugs can be used only for the prevention of uric acid formation. Conditions of accelerated cell breakdown or turnover such as rhabdomyolysis, hemolysis, and tumor lysis can also be a purine source and thus, increase uric acid production. Uric acid is filtered in the glomerulus and is mostly reabsorbed in the proximal convoluted tubule. The distal convoluted tubule is mostly impermeable to uric acid. Go to the next page if you knew the correct answer, or click the link image(s) below to further research the concepts in this question (if desired).
Research Concepts: Hyperuricemia
We update eBooks quarterly and Apps daily based on user feedback. Please tap flag to report any questions that need improvement.
Question 529:
Which of the following chromosomal rearrangements are not found in ALK-negative anaplastic large cell lymphomas?
Choices: 1. 2. 3. 4.
IRF4 TNFRSF8 BATF3 TMOD1
Answer: 1 - IRF4 Explanations: IRF4 is most suggestive of cutaneous anaplastic large cell lymphoma (ALCL). The presence of TNFRSF8 together with BATF3 and TMOD1 helps differentiate ALK-negative ALCL from peripheral T cell lymphomas. The presence of BATF3 together with TNFRF8 and TMOD1 helps differentiate ALK-negative ALCL from peripheral T cell lymphomas. The presence of TMOD1 together with TNFRF8 and BATF3 helps differentiate ALK-negative ALCL from peripheral T cell lymphomas. Go to the next page if you knew the correct answer, or click the link image(s) below to further research the concepts in this question (if desired).
Research Concepts: ALK Negative Anaplastic Large Cell Lymphoma
We update eBooks quarterly and Apps daily based on user feedback. Please tap flag to report any questions that need improvement.
Question 530:
A 40-year-old male presents to the clinician with progressive dyspnea on exertion for the past 4 weeks. He also says that he has left-sided chest wall pain. The patient is a nonsmoker and has no significant past medical history. The physical examination reveals no abnormality. A chest roentgenogram revealed a large lobulated mass in the left side of the chest and compression of the left lung medially. Computed tomography (CT) of the chest showed a well-circumscribed large mass in the right pleural cavity and inhomogeneous soft tissue density, as well as areas of fat and calcification. A biopsy of the mass is performed that shows intercellular mucoid material, arborizing capillaries, and lipoblasts. Generally, which of the following presentations helps distinguish such tumors from their benign counterpart?
Choices: 1. Liposarcomas are usually less than 10 cm in size 2. Liposarcomas are routinely seen in young adults 3. Thick internal septations more than 2 mm and nodular non-adipose components favor sarcoma 4. A negative biopsy confirms the benign nature
Answer: 3 - Thick internal septations more than 2 mm and nodular non-adipose components favor sarcoma
Explanations: Pleural liposarcomas are typically greater than 10 cm in size. Pleural liposarcomas are a disease of older patients, usually more than 60 years. A negative biopsy does not exclude malignancy, especially if not imagingguided to a focal non-adipose region. Benign adipocytic tumors can arise from any part within the body and can cause symptoms primarily through mass effect. They are usually encapsulated, homogenous without evidence of nodularity or septations, and may contain calcifications or hemorrhage as a result of trauma. Treatment consists of excision beyond the capsule of the tumor. There is some degree of clinical overlap with the potentially malignant form and most common soft tissue sarcoma, the liposarcoma. Liposarcomas are defined as those tumors which are greater than 10 cm in size, have thick internal septations, and those lesions that are generally less than 75% adipose tissue. They represent 45% of retroperitoneal sarcomas. Treatment for liposarcoma is surgical resection with wide margins. Local recurrence is common. Go to the next page if you knew the correct answer, or click the link image(s) below to further research the concepts in this question (if desired).
Research Concepts:
Sarcoma
We update eBooks quarterly and Apps daily based on user feedback. Please tap flag to report any questions that need improvement.
Question 531:
A patient is treated with systemic chemotherapy for breast cancer. Treatment involves a drug that binds and stabilizes the tubule to confer cytotoxicity. Which of the following adverse events is most likely to be life-threatening in this patient?
Choices: 1. 2. 3. 4.
Ototoxicity Dilated cardiomyopathy Pancreatitis Febrile neutropenia
Answer: 4 - Febrile neutropenia Explanations: This patient with breast cancer is most likely receiving treatment with docetaxel, an adjunct chemotherapy that is proven to improve survival in high-risk cases. Docetaxel is a second-generation taxane, which functions by binding and stabilizing beta-tubulin, enhancing its proliferation, and preventing the normal function of the mitotic spindle. This causes cells to be unable to complete the M phase of their life cycle, leading to cell death. The significant adverse effects of docetaxel include diarrhea, skin reactions, and myelosuppression. A more serious manifestation of myelosuppression is febrile neutropenia. To prevent this potentially life-threatening adverse event, patients should be monitored for neutropenia, fever, and signs of infection throughout treatment with docetaxel. Go to the next page if you knew the correct answer, or click the link image(s) below to further research the concepts in this question (if desired).
Research Concepts: Docetaxel
We update eBooks quarterly and Apps daily based on user feedback. Please tap flag to report any questions that need improvement.
Question 532:
A 56-year-old woman with pancreatic cancer is undergoing chemotherapy with FOLFIRINOX (5fluorouracil, leucovorin, irinotecan, and oxaliplatin). To prevent a common oral complication of FOLFIRINOX treatment, the patient is instructed to keep ice chips in her mouth for half an hour during chemotherapy administration. Which of the following best describes the rationale for using ice chips in this patient?
Choices: 1. They reduce oral pain by inducing vasodilation of the oral mucosa vessels. 2. They provide temporary anesthesia by cooling the oral mucosal surfaces. 3. They prevent the delivery of cytotoxic drugs to the oral tissues. 4. They improve the blood circulation and clearance of the cytotoxic drugs.
Answer: 3 - They prevent the delivery of cytotoxic drugs to the oral tissues.
Explanations: Oral mucositis is a severely debilitating condition characterized by erythema, edema, and ulcerations of the oral mucosa. It is a complication of radiation therapy (RT) to the head and neck, chemotherapy, chemoradiotherapy, and hematopoeitic stem cell transplantation (HSCT). Cryotherapy refers to placing a source of cold in the oral cavity, usually ice or cold water, during the delivery of a cytotoxic agent. Cooling the oral mucosa induces vasoconstriction, preventing the delivery of the chemotherapy drugs to the oral tissues and, therefore, preventing oral mucositis. Low temperatures may also decrease the metabolism of the basal epithelial layer. Cooling of the oral mucosa is only temporary; therefore, this measure is indicated in conjunction with short chemotherapy protocols or when short half-life cytotoxic agents are used. The MASCC/ISOO guidelines recommend 30 minutes of oral cryotherapy during the administration of bolus 5-fluorouracil. Go to the next page if you knew the correct answer, or click the link image(s) below to further research the concepts in this question (if desired).
Research Concepts: Oral Mucositis
We update eBooks quarterly and Apps daily based on user feedback. Please tap flag to report any questions that need improvement.
Question 533:
A 55-year-old man presents to the clinic for a follow-up. He has a history of H. pylori-associated MALT lymphoma of the stomach for the past five years. He was treated with antibiotics and was in remission since then. He reports a three-week history of fatigue, night sweats, weight loss, and also reports an enlarging mass on the right side of the neck, which he first noticed 4 weeks ago. Blood work shows elevated lactate dehydrogenase. Which of the following is the most likely cause of the patient's new symptoms?
Choices: 1. 2. 3. 4.
MALT lymphoma progression Viral infection Histological transformation to a high-grade lymphoma Reinfection with H.Pylori
Answer: 3 - Histological transformation to a high-grade lymphoma
Explanations: A clinician should suspect a large cell transformation in patients with prior indolent lymphoma with a rapid change in clinical course and associated B symptoms. A concern for large cell transformation should be further evaluated by a biopsy and PET scan for appropriate staging and treatment considerations. Patients with the histological transformation to a highgrade lymphoma have lower five-year survival rates. Progression of MALT lymphoma doesn't cause rapid progression/worsening of symptoms as it is an indolent lymphoma. Go to the next page if you knew the correct answer, or click the link image(s) below to further research the concepts in this question (if desired).
Research Concepts: Mucosa-associated Lymphoma Tissue
We update eBooks quarterly and Apps daily based on user feedback. Please tap flag to report any questions that need improvement.
Question 534:
A 32-year-old male patient presents to the clinic for evaluation of hearing difficulties. The patient started noticing progressive hearing deficits about six months ago. Audiometric examination reveals an asymmetric, sensorineural hearing deficit. Non-contrast CT scan reveals bilateral cerebellopontine angle masses. MRI and genetic testing further confirm the diagnosis. The patient's family history is positive for intracranial tumors. Past medical history is insignificant. Which of the following genes is mutated in this patient?
Choices: 1. 2. 3. 4.
NF2 EGFR ALK KRAS
Answer: 1 - NF2 Explanations: NF2 gene mutation leads to neurofibromatosis type 2 characterized by bilateral acoustic neuromas. This is an autosomal dominant mutation. Patients with NF2 are at an increased risk of other intracranial tumors like meningiomas and neurofibromas. Patients with neurofibromatosis type 2 present early with a bilateral sensorineural hearing deficit, tinnitus, vertigo, or other non-specific symptoms. Cerebellopontine tumors are usually benign, slowgrowing tumors. The treatment outcome depends on the tumor's size, location, and consistency. Delayed treatment is associated with poor outcomes, and therefore early detection and treatment are important. Post-operative imaging should be carefully reviewed to differentiate normal post-surgical changes from recurrence. Go to the next page if you knew the correct answer, or click the link image(s) below to further research the concepts in this question (if desired).
Research Concepts: Cerebellopontine Angle Cancer
We update eBooks quarterly and Apps daily based on user feedback. Please tap flag to report any questions that need improvement.
Question 535:
A 16-year-old male presents with a large superficial mass on his back which looks like a bag of worms. The mass started developing on his back when he was 3 years old. It has gradually increased in size. On physical examination, the mass is well-circumscribed, grey in color, and ovoid in shape. There is the presence of golden-brown dome-shaped masses on the iris of both eyes. Which of the following has an increased risk of malignant transformation in this patient?
Choices: 1. 2. 3. 4.
Peripheral nerve sheath tumor Melanoma Oligodendroglioma Meningioma
Answer: 1 - Peripheral nerve sheath tumor Explanations: A plexiform neurofibroma is essentially diagnostic for neurofibromatosis type 1. Patients with neurofibromatosis type 1 are at increased risk of a malignant transformation. Malignant peripheral nerve sheath tumors arise de novo, or as a result of a malignant transformation in patients with neurofibromatosis type 1. Malignant peripheral nerve sheath tumor is an aggressive neurogenic neoplasm with a high rate of recurrence and metastasis. Go to the next page if you knew the correct answer, or click the link image(s) below to further research the concepts in this question (if desired).
Research Concepts: Neurofibroma
We update eBooks quarterly and Apps daily based on user feedback. Please tap flag to report any questions that need improvement.
Question 536:
A 65-year-old male presents to the clinic with a complaint of bone pain in his thighs and hips. He has a history of hypertension and small cell lung cancer and has been on myelosuppressive chemotherapy. His other medications include filgrastim and metoprolol. He denies any recent falls or trauma to the region. PET scan done a few months ago was unremarkable for radioactive bone uptake. What is the most likely cause of his pain?
Choices: 1. 2. 3. 4.
Fracture Metastases Medication-induced Thrombosis
Answer: 3 - Medication-induced Explanations: This patient's bone pain is most likely due to filgrastim. Bone pain is one of the most common side effects of filgrastim use. Filgrastim is a granulocyte colony-stimulating factor is often used alongside chemotherapy to stimulate neutrophil production. Other side effects include allergic reactions, headaches, nausea, vomiting, anemia, thrombocytopenia, etc. While the other options can also result in bone pain, they are less likely in this case. Go to the next page if you knew the correct answer, or click the link image(s) below to further research the concepts in this question (if desired).
Research Concepts: Filgrastim
We update eBooks quarterly and Apps daily based on user feedback. Please tap flag to report any questions that need improvement.
Question 537:
A 36-year-old female presents to the office with complaints of neck swelling and hoarseness for one month. The swelling has been present for one year, but she recently started having symptoms of difficulty swallowing and hoarseness. She recently migrated from India. She denies any fever, cough, or trauma. On examination, there is a 3 cm, non-tender mass that moves with swallowing. Labwork shows decreased TSH. The ultrasound of the neck reveals a 3 cm hypoechoic thyroid nodule with calcifications. The biopsy of the nodule reveals Orphan Annie nuclei appearance. Which of the following are the most likely site and mode of metastasis in this patient?
Choices: 1. 2. 3. 4.
Via lymphatics to the lung Hematogenously to the liver Via lymphatics to the kidney Via lymphatics to the spleen
Answer: 1 - Via lymphatics to the lung Explanations: This female with a neck mass with decreased TSH and biopsy showing orphan Annie nuclei is most likely suggestive of papillary carcinoma of the thyroid. The lungs are the most common site of distant metastasis for papillary thyroid carcinoma. Other sites of metastasis include the skull, brain, mediastinum, axilla, and long bones. The liver, spleen, and kidneys are rarely involved. Papillary thyroid carcinoma spreads via the lymphatics system. Go to the next page if you knew the correct answer, or click the link image(s) below to further research the concepts in this question (if desired).
Research Concepts: Thyroid Cancer
We update eBooks quarterly and Apps daily based on user feedback. Please tap flag to report any questions that need improvement.
Question 538:
A 40-year old male presents to the clinic for follow up. He has a history of small cell lung cancer being treated with myelosuppressive chemotherapy. His most recent follow up 12 days ago revealed neutropenia, and the clinician decided to prescribe filgrastim. The patient has no complaints at this visit. His vital signs include blood pressure 128/79 mmHg, pulse 84/minute, respiratory rate 13/minute, and temperature 37.3 C (99.1 F). The physical exam is noncontributory. His lab values today show an absolute neutrophil count of 11000/microliters. What is the best next step in the management of this patient?
Choices: 1. 2. 3. 4.
Discontinue filgrastim Increase dose of myelosuppressive medication Continue current treatment with no changes Increase the dosage of filgrastim
Answer: 1 - Discontinue filgrastim Explanations: Filgrastim is a granulocyte colony-stimulating factor that stimulates the production, proliferation, and maturation of neutrophil progenitors. It, therefore, increases neutrophil counts as a result. This patient's absolute neutrophil count is >10000/microliters. It is recommended that filgrastim be discontinued when the count is >10000/microliters to avoid the potential risks of leukocytosis and other adverse effects. Some adverse effects of filgrastim include bone pain, nausea, headaches, thrombocytopenia, etc. Other options are less likely to be the recommended options. Go to the next page if you knew the correct answer, or click the link image(s) below to further research the concepts in this question (if desired).
Research Concepts: Filgrastim
We update eBooks quarterly and Apps daily based on user feedback. Please tap flag to report any questions that need improvement.
Question 539:
A 45-year-old female presents after screening mammography to the office. A mass is seen in the lower inner quadrant of the left breast. She reports age at menarche to be 9 years, her first child-birth at age 28 years age. She never took any oral contraceptive pills but is currently on hormone replacement therapy. Her father was diagnosed with breast cancer at age 60 with positive genetic testing for breast cancer. Diagnostic mammogram and stereotactic biopsy confirm invasive ductal carcinoma. Which of the following most adversely affected the lifetime risk of this patient to develop breast cancer?
Choices: 1. 2. 3. 4.
Early menarche Hormone replacement therapy Family history Age at first childbirth
Answer: 3 - Family history Explanations: This patient has a strong family history of breast cancer. Male breast cancer in a first-degree relative is strongly suggestive of BRCA mutation, specifically BRCA 2. Her father was tested for genetic mutation and was positive. This patient's relative risk for breast cancer increases to >4 given BRCA mutation. Early menarche can increase the risk of breast cancer. Other cancers in the family such as prostate, pancreatic cancers, or ovarian cancers in first degree relatives should also raise suspicion for BRCA mutation and prompt for genetic testing. Nulliparity can increase risk but not to the magnitude of possessing the BRCA1 gene. Late age at first childbirth(not early as in this case) could increase the risk for breast cancer. Go to the next page if you knew the correct answer, or click the link image(s) below to further research the concepts in this question (if desired).
Research Concepts: BRCA 1 and 2
We update eBooks quarterly and Apps daily based on user feedback. Please tap flag to report any questions that need improvement.
Question 540:
A 39-year old female presents to the outpatient complaining from a bloody discharge from her right nipple upon exerting mild pressure that she had realized 2 days ago. The patient says her aunt had died from breast cancer. Upon examination, a small, smooth, regular 2 cm lump is felt near her right areola. There were no lumps in her axilla and no other clinical findings. Her bleeding profile showed her INR to be 1.03. Breast ultrasonography was performed and showed a regular round homogenous lump around the ducts with a bi-rads score of 2. What is the most probable suitable management in this case?
Choices: 1. 2. 3. 4.
Biopsy and mastectomy Microdochectomy and follow-up MRI Vitamin K and pro-coagulant injections
Answer: 2 - Microdochectomy and follow-up Explanations: The most common cause of bleeding from the nipple is an intraductal papilloma. Intraductal papillomas usually affect middle-aged females, and breast ultrasonography is usually more accurate than mammography. A scoring system called the bi-rads scoring system is used to calculate the risk of the lump being a malignant tumor. It is always important to check the patients bleeding profile when they present with bleeding. It's also important to ask for their drug history and if there is bleeding from any other sites in the body. A biopsy is done when the bi-rads scoring system and clinical examination is inconclusive, but in this case, the findings were typical of intraductal papilloma. Go to the next page if you knew the correct answer, or click the link image(s) below to further research the concepts in this question (if desired).
Research Concepts: Papilloma
We update eBooks quarterly and Apps daily based on user feedback. Please tap flag to report any questions that need improvement.
Question 541:
A 26-year-old female presents to the clinic with blurry vision for the past month. She also complains of intermittent mild to moderate frontal headaches over the past three months. On examination, she has difficulty abducting the left eye and has papilledema. Visual field charting shows bitemporal hemianopia. On detailed questioning, she reveals that she has had milk discharge from both nipples and amenorrhea over the past six months. Her serum prolactin level is 89 ng/ml. Brain imaging revealed a solid and cystic sellar mass measuring 3 cm with calcification. She subsequently undergoes transsphenoidal surgery. The pathology showed dense nodules and trabeculae of squamous epithelium bordered by a palisade of columnar epithelium and yellow-brown cholesterol-rich fluid. What is the most likely diagnosis?
Choices: 1. 2. 3. 4.
Papillary craniopharyngioma Adamantinomatous craniopharyngioma Pituitary adenoma Prolactinoma
Answer: 2 - Adamantinomatous craniopharyngioma Explanations: There are two histologic subtypes of craniopharyngiomas: adamantinomatous and papillary. Adamantinomatous craniopharyngioma is primarily seen in pediatric cases but can be seen in adults as well. They have solid and cystic parts. The solid part is characterized by dense nodules and trabeculae of squamous epithelium bordered by a palisade of columnar epithelium, sometimes referred to as a "picket fence." The cystic part has a yellow-brown, cholesterolrich fluid. Papillary craniopharyngioma is commonly seen in adults. They are mostly solid without calcification. They are characterized as well-differentiated squamous epithelium lacking surface maturation, with occasional goblet cells and ciliated epithelium Mild hyperprolactinemia is due to the large size of the sellar mass resulting in the "stalk effect." Go to the next page if you knew the correct answer, or click the link image(s) below to further research the concepts in this question (if desired).
Research Concepts: Craniopharyngioma
We update eBooks quarterly and Apps daily based on user feedback. Please tap flag to report any questions that need improvement.
Question 542:
A 56-year-old male presents with a subcutaneous nodule that is intermittently painful and has recently grown in size. On physical examination, lesions are strikingly painful, typically small, and have a bluish-purple hue. Histopathological evaluation is consistent with large, sharply demarcated basophilic nodules of the dermis with ovoid nuclei. Which of the following is most likely to be the best treatment in this patient?
Choices: 1. 2. 3. 4.
Radiation therapy Chemotherapy Surgical excision Carbon dioxide laser
Answer: 3 - Surgical excision Explanations: Spiradenomas are well-differentiated, benign, dermal neoplasms originating from the sweat glands. Most spiradenomas occur between the ages of 15 to 35 years. Typically, they present as small solitary nodules that can grow to several centimeters, often with a blue, gray, or purple hue. They are strikingly painful. Spiradenomas usually arise on the head, neck, and trunk; however, cases in other areas such as the breast have occurred. Spiradenomas typically present as dermal or subcutaneous nodules on any area, though more commonly are seen on the head, neck, and trunk, and less commonly on the arms and legs. Several spiradenomas have been reported on the ears and eyelids, as well as papular lesions of the proximal nail fold. On physical examination, lesions are typically small, though they can grow to several centimeters in diameter, and have a blue, gray, or purple hue. Spiradenomas can be strikingly painful, though most are asymptomatic. Although the typical presentation is a solitary lesion, there can be linear, blaschkoid, or grouped spiradenomas. Lesions should be excised for histopathologic examination. On histopathology, spiradenomas show one or more large, sharply demarcated basophilic nodules of the dermis. Two types of cells are seen in the nodules. These cells are either small, dark, basaloid cells with hyperchromatic nuclei, or larger, pale, and with ovoid nuclei. Malignant spiradenomas demonstrate increased mitotic rates, necrosis, nuclear atypia,
pleomorphism, hyperchromasia, loss of nested growth patterns, and loss of the dual cell population. Although spiradenoma is considered a benign adnexal neoplasm with a low recurrence rate, malignant transformation can occur in long-standing lesions, particularly in patients older than the age of 50. The rate of metastasis for malignant spiradenocarcinoma is about 50%. Conservative surgical excision of spiradenomas is therefore recommended. In cases of multiple spiradenomas, adjunctive treatment with carbon dioxide laser may be employed after surgical debulking. Go to the next page if you knew the correct answer, or click the link image(s) below to further research the concepts in this question (if desired).
Research Concepts: Spiradenoma
We update eBooks quarterly and Apps daily based on user feedback. Please tap flag to report any questions that need improvement.
Question 543:
A 17-year-old male comes to the clinic with complaints of frontal headache for five months. The headache is dull and continuous, lasting for hours and not relieved by NSAIDs. He denies any nausea, vomiting, fever, trauma, photophobia/phonophobia, lacrimation, or abnormal movement of the body parts. On examination, there is a 1 x 1 cm soft and minimal tender swelling on the right frontal region. His vitals are stable. Complete blood count with differential is normal. X-ray of the skull shows a single punched-out lesion. CT-scan shows a lytic lesion without soft tissue involvement. A comprehensive skeletal survey is done, which reveals no similar lesion of any bone. Biopsy of the lesion and staining with CD1a and langerin is done, which is positive. Electron microscopy of the lesion shows tennis racquet like structure. Which of the following is the best treatment for this patient?
Choices: 1. 2. 3. 4.
Beta-blocker Sumatriptan Curettage followed by bone grafting Amitriptyline
Answer: 3 - Curettage followed by bone grafting Explanations: This patient has a diagnosis of eosinophilic granuloma of the frontal bone according to the presenting complaints and electron microscopic findings. Solitary skull lesions that are diagnosed with eosinophilic granuloma are treated with curettage followed by bone grafting. High-risk patients like mature skeletal patients, polyostotic EG, CNS-risk bones involvement (sphenoid, ethmoid, orbital, or temporal), and multisystem involvement should be treated with systemic chemotherapy i.e., vinblastine and prednisone or cytarabine for 12 months. Beta-blocker is given for migraine headaches, sumatriptan for cluster headache, and amitriptyline for tension headache. Go to the next page if you knew the correct answer, or click the link image(s) below to further research the concepts in this question (if desired).
Research Concepts: Eosinophilic Granuloma
We update eBooks quarterly and Apps daily based on user feedback. Please tap flag to report any questions that need improvement.
Question 544:
A 49-year-old male who was recently diagnosed with unilateral tonsillar mass, and further evaluation was concerning for a malignancy. He underwent a biopsy of the tonsil, and the pathology showed a tumor, which is p16 positive. Which of the following characteristics of a patient and cancer has the best prognosis?
Choices: 1. Squamous cell carcinoma (SCC) with p16 positive nonsmoker 2. SCC with p16 positive smoker 3. SCC with p16 negative nonsmoker 4. SCC with p16 negative smoker
Answer: 1 - Squamous cell carcinoma (SCC) with p16 positive nonsmoker
Explanations: Squamous cell carcinoma (SCC) of the tonsil with p16 positive is an excellent surrogate marker for HPV positivity. SCC of the tonsil associated with HPV has a better prognosis. SCC of tonsil patients with HPV positive and less than 10 years of smoking history has a better prognosis. p16 -ve tumors have a worse prognosis compared to p16 positive tumors. Go to the next page if you knew the correct answer, or click the link image(s) below to further research the concepts in this question (if desired).
Research Concepts: Tonsil Cancer
We update eBooks quarterly and Apps daily based on user feedback. Please tap flag to report any questions that need improvement.
Question 545:
A 60-year-old man with a 40 pack-year history of smoking presents to the clinic for a tongue mass. Physical exam shows a 2 cm exophytic mass on the right lateral tongue. Biopsy shows nests of epithelial cells, keratin deposits beyond the basement membrane, stromal fibrosis, an inflammatory infiltrate, and regions of patchy necrosis. Which of the following is the next best step in the management of this patient?
Choices: 1. 2. 3. 4.
Clindamycin Surgical excision Neck computed tomography scan with contrast Follow up in four weeks
Answer: 3 - Neck computed tomography scan with contrast
Explanations: The patient has invasive squamous cell carcinoma of the tongue. An inflammatory infiltrate and regions of necrosis frequently coexist with malignant squamous cell carcinoma. Tongue squamous cell carcinoma frequently metastasizes via lymphatics to regional lymph nodes in the neck. CT scan with intravenous contrast will help clinically stage the disease and reveal any lymph nodes with worrisome characteristics. Malignant tongue lesions are frequently mistreated as infections with antibiotics, which would be inappropriate in this case given the pathologic diagnosis of carcinoma. Although surgery may be offered as a treatment alternative, neck imaging (as well as a thorough neck examination) is the best next step in the management of this patient, as this may impact both staging/prognosis and the type of surgery that will be offered. Observation for malignant tongue cancer in this patient is inappropriate. Given the pathologic diagnosis, the patient should be offered therapy. Go to the next page if you knew the correct answer, or click the link image(s) below to further research the concepts in this question (if desired).
Research Concepts: Tongue Cancer
We update eBooks quarterly and Apps daily based on user feedback. Please tap flag to report any questions that need improvement.
Question 546:
A 65-year-old female is diagnosed with a bulky stage III grade 2 follicular lymphoma. She received 6 cycles of bendamustine and rituximab and achieved complete remission. She is followed with routine surveillance imaging as well as physical exams. Three years after completing treatment, she develops an enlarging nontender node in the left axilla. A CT scan shows several nodes above and below the diaphragm, including a 4 cm node in the left retroperitoneum. What is the next step in management?
Choices: 1. 2. 3. 4.
Biopsy Start R-CHOP chemotherapy Rituximab as a single agent Rechallenge with bendamustine
Answer: 1 - Biopsy Explanations: Several salvage treatment options exist for patients with relapse following frontline therapy for follicular lymphoma. However, prior to initiating therapy, aggressive transformation must be ruled out. So biopsy is required in all patients who relapse after attaining initial remission, especially who attain initial remission greater than 24 months. Without a biopsy to confirm relapsed follicular lymphoma, proceeding with salvage therapy is not recommended. Aggressive transformation occurs in approximately 3% of follicular lymphoma patients annually, with a lifetime risk of over 30%. Rituximab as a single agent is suggested for patients with follicular lymphoma with low tumor burden or elderly/frail patients. Patients with prolonged progression-free survival following first therapy for follicular lymphoma may be rechallenged with bendamustine. However, in patients with relapsed lymphoma, a repeat biopsy is required to confirm the diagnosis and rule out aggressive histology. Go to the next page if you knew the correct answer, or click the link image(s) below to further research the concepts in this question (if desired).
Research Concepts: Relapsed And Refractory Follicular Lymphoma
We update eBooks quarterly and Apps daily based on user feedback. Please tap flag to report any questions that need improvement.
Question 547:
A 60-year-old patient presents to the clinic for multiple new skin lesions. The patient states that they grew rapidly over the past 4 weeks and that he just had a sebaceous carcinoma removed from his upper eyelid less than 6 months ago. On examination, multiple fleshcolored exophytic papules with telangiectasias and central crusts are observed. The attending clinician believes the patient has developed multiple keratoacanthomas. What is the most common type of internal malignancy this patient is at risk of developing?
Choices: 1. 2. 3. 4.
Small cell carcinoma of the lung Squamous cell carcinoma of the head and neck Adenocarcinoma of the colon Hodgkin lymphoma of the neck
Answer: 3 - Adenocarcinoma of the colon Explanations: Muir-Torre syndrome shows autosomal dominant inheritance. Muir-Torre syndrome commonly manifests during the fifth or sixth decade of life. Muir-Torre syndrome is associated with cutaneous skin findings that include multiple keratoacanthomas and sebaceous neoplasms (sebaceous adenomas, sebaceous epitheliomas, and sebaceous carcinoma). The most common internal malignancy associated with Muir-Torre syndrome is adenocarcinoma of the colon. Muir-Torre syndrome is also associated with other GI, genitourinary, lung, breast, and hematologic malignancies. Go to the next page if you knew the correct answer, or click the link image(s) below to further research the concepts in this question (if desired).
Research Concepts: Muir-Torre Syndrome
We update eBooks quarterly and Apps daily based on user feedback. Please tap flag to report any questions that need improvement.
Question 548:
A 16-year-old boy with a history of hypertension for a few months is brought to the hospital for palpitations, tremors, flushing, and diarrhea. He is found to have an elevated calcium level. On examination, there are palpable anterior cervical lymph nodes. Ultrasound abdomen reveals abdominal lymphadenopathy. Which of the following is the most likely diagnosis?
Choices: 1. 2. 3. 4.
Essential hypertension Multiple endocrine neoplasia type 2 Familial hypocalciuric hypercalcemia Williams syndrome
Answer: 2 - Multiple endocrine neoplasia type 2 Explanations: MEN 2 is a subtype of multiple endocrine neoplasias (MEN). It has subtypes of its own. MEN 2 is commonly an autosomal dominant inherited disorder associated with several tumors of the endocrine system. MEN 2 involves medullary thyroid carcinoma, pheochromocytoma, and parathyroid adenoma. Von Hippel-Lindau disease is also associated with MEN 2. Most of the time, patients present with symptoms caused by medullary thyroid cancer (MTC). MTC accounts for 5% of all thyroid tumors. It is often malignant and is the most common cause of death in patients with MEN 2. The overall survival of patients with MTC is 86% at 5 years and 65% at 10 years. Its metastases cause lymphadenopathy in the neck and the mediastinum. It spreads to the liver, lung, and bone. Go to the next page if you knew the correct answer, or click the link image(s) below to further research the concepts in this question (if desired).
Research Concepts: Multiple Endocrine Neoplasias Type 2
We update eBooks quarterly and Apps daily based on user feedback. Please tap flag to report any questions that
need improvement.
Question 549:
A 55-year-old male with a recent diagnosis of prostate cancer presents to the office for prostate brachytherapy. Radioactive seeds will be placed inside the prostate gland to target the tumor. The concept of brachytherapy was explained to the patient and how it would be most beneficial for this type of cancer. How is radiation intensity related to distance?
Choices: 1. 2. 3. 4.
Directly proportional to distance Inversely proportional to the square of the distance Exponential Inversely exponential
Answer: 2 - Inversely proportional to the square of the distance
Explanations: Radiation intensity decreases with the square of the distance. Increasing distance decreases radiation, an inverse relationship. This is known as the "inverse square law". Brachytherapy is another form of radiation therapy where radioactive material is temporarily or permanently implanted near or inside the target object. It is a type of internal radiation and is used to deliver higher doses of radiation in a more specific and smaller area that cannot be possible with external beam radiation. This has important implications in radiation safety. The most effective method to decrease one's exposure to radiation is to step back from the source. This law applies to radiation from all sources, both from x-rays and from nuclear medicine radiopharmaceuticals. The inverse square law states that the energy received at a given distance from the source is inversely proportional to the square of the distance from the source, denoted by 1/r^2. Go to the next page if you knew the correct answer, or click the link image(s) below to further research the concepts in this question (if desired).
Research Concepts: Radiation Physics
We update eBooks quarterly and Apps daily based on user feedback. Please tap flag to report any questions that need improvement.
Question 550:
A 65-year-old woman presented to the healthcare provider with vague asymmetric density on a mammogram. An ultrasound revealed a hypoechoic mass with irregular margins measuring 1.4 x 1.5 x 1.3 cm. An ultrasound-guided biopsy of the left breast mass was performed, which showed non-cohesive tumor cells arranged in single files. They had a prominent apocrine differentiation with focal signet ring cell-like features. The cells were large with abundant eosinophilic cytoplasm, enlarged nuclei with vesicular chromatin and prominent nucleoli. Mitotic count was three mitoses per high power field. These were also foci of the classic variant of lobular carcinoma in situ. The immunohistochemical study showed negative immunostaining of tumor cells with E-cadherin. What is the most likely diagnosis?
Choices: 1. 2. 3. 4.
Tubulolobular variant of invasive lobular carcinoma Alveolar variant of invasive lobular carcinoma Invasive carcinoma of no special type Pleomorphic lobular carcinoma
Answer: 4 - Pleomorphic lobular carcinoma Explanations: The tumor cells of the pleomorphic variant of invasive lobular carcinoma are larger, have abundant eosinophilic cytoplasm with large pleomorphic hyperchromatic nuclei that show prominent nucleoli, which can be irregular in some cases, thus making it difficult to differentiate from a high-grade invasive ductal carcinoma. Duct formation is distinctly absent in most cases described. Very often, the cells appear plasmacytoid or have a predominant signet ring cell or apocrine morphology. One of the most consistent molecular alterations in invasive lobular carcinoma and its variants is the loss of expression of the cell-cell adhesion molecule E-cadherin that contributes to the characteristic non-cohesive nature of lobular cells. A pleomorphic variant of lobular carcinoma is an aggressive form of breast carcinoma. Unlike the usual scenario of a positive ER/PR and overexpression of Her2/neu, a pleomorphic variant of lobular carcinoma can be triple-negative, thereby adding to the challenge of planning the treatment strategy of this aggressive tumor. The pleomorphic variant of lobular carcinoma is a variant of invasive lobular carcinoma. It accounts for less than 1% of all breast epithelial malignancies. Go to the next page if you knew the correct answer, or click the link image(s) below to further research the concepts in this question (if desired).
Research Concepts: Lobular Breast Carcinoma
We update eBooks quarterly and Apps daily based on user feedback. Please tap flag to report any questions that need improvement.
Question 551:
A 35-year-old man presents to the clinic with pain in the right leg for 6 months. There is no history of trauma and fever. On examination, the patient has palpable mass 2x2 cm and mild tenderness over the midshaft of the tibia. The radiograph reveals a soap bubble appearance. Biopsy shows epithelial and fibrous mesenchymal cells. What is the most likely diagnosis?
Choices: 1. 2. 3. 4.
Osteoid osteoma Adamantinoma Osteochondroma Osteosarcoma
Answer: 2 - Adamantinoma Explanations: This patient has adamantinoma, which is a common tumor in the shaft of the tibia, presenting as a lump or bony deformity. It consists of biphasic cells, epithelial, and fibrous mesenchymal cells. Keratin stains are positive. Wide surgical excision is the treatment of choice. Radiotherapy and chemotherapy do not have a major role. Osteosarcoma is an osteoid forming tumor, while osteoid osteoma has immature osteoid trabeculae. Go to the next page if you knew the correct answer, or click the link image(s) below to further research the concepts in this question (if desired).
Research Concepts: Adamantinoma
We update eBooks quarterly and Apps daily based on user feedback. Please tap flag to report any questions that need improvement.
Question 552:
A 65-year-old African-American male presents with complaints of multiple episodes of blood in his urine, frequent urination, and fatigue despite minimal activity for the past six months. He has a history of hypertension for more than 20 years, and for the past few years, his blood pressure has been poorly controlled in spite of taking medications. Currently, his blood pressure reading is 150/100 mmHg, his pulse rate is 62 beats per minute, his temperature is 98.7 F, respiratory rate is 18 breaths per minute, and his body mass index is 31 kg/m2. There are no significant findings on his physical examination. Initial tests are done, and his serum electrolytes show a calcium level of 12.5 mg/dl. An abdominal ultrasound is performed, and a 5 cm solid left renal mass is detected off the lower pole of the kidney. No similar findings are observed in the right kidney. Computed tomography (CT) scan confirmed the ultrasound findings but demonstrates no abnormalities in the rest of the abdomen. Assuming that all of the following are possible, what is the most appropriate treatment for this patient?
Choices: 1. 2. 3. 4.
Partial nephrectomy Radical nephrectomy Radiofrequency ablation Chemotherapy
Answer: 1 - Partial nephrectomy Explanations: Renal cell carcinoma, the commonest cancer of the kidneys in adults, occurs more frequently in men aged 50-70. Risk factors include advanced age, African American race, hypertension, renal pathology, and obesity. Although 25% are asymptomatic, a patient may present with hematuria, fatigue, weight loss, flank pain, and fever. The patient may also present with the symptoms of hypercalcemia, such as increased thirst, frequent urination, and body aches. The initial radiographic test to be performed is often an ultrasound. CT scans can then be used to differentiate cystic from solid masses and is helpful in staging since they can evaluate lymph nodes and renal vein/inferior vena cava involvement. A contrast CT scan is the ideal test of choice for making a diagnosis of renal cell cancer. In advanced stages, a whole-body bone scan is carried out for the detection of bony and other metastases. Treatment depends on the staging of the tumor. Tumors limited to the kidney and having a size less than 7 cm are stage I, and thus, partial nephrectomy is the treatment of choice if possible. The tumor is too large for radiofrequency or cryoablation therapy except in very high surgical risk cases. Go to the next page if you knew the correct answer, or click the link image(s) below to further research the concepts in this question (if desired).
Research Concepts: Renal Cell Cancer
We update eBooks quarterly and Apps daily based on user feedback. Please tap flag to report any questions that need improvement.
Question 553:
An adult 40-year-old female presents to the breast clinic with complaints of a breast mass in the right upper quadrant of the right breast. A mammogram is done after an initial assessment for the further evaluation of the breast mass, which is reported as breast imaging reporting and data system (BI-RADS) 2. Which of the following is the next best step regarding this patient?
Choices: 1. Continue routine screening because the lesion is benign 2. Do additional diagnostic testing because the testing is incomplete 3. Perform a biopsy because the lesion is highly suggestive of breast cancer 4. Repeat mammography in 6 months because the lesion is probably benign
Answer: 1 - Continue routine screening because the lesion is benign
Explanations: BI-RADS is a collaborative effort by many groups but is published and trademarked by the American College of Radiology. BI-RADS 2 indicates a benign finding. BI-RADS 1 indicates a negative study. BI-RADS 3 indicates a probable benign finding. The distinction between BIRADS 1 and 2 is to some degree, a matter of individual radiologist preference, but this is not problematic since management for BIRADS 1 and 2 is identical. The clinical management recommendation for BI-RADS 1 and 2 is to continue annual routine screening. Go to the next page if you knew the correct answer, or click the link image(s) below to further research the concepts in this question (if desired).
Research Concepts: Breast Imaging Reporting and Data System
We update eBooks quarterly and Apps daily based on user feedback. Please tap flag to report any questions that need improvement.
Question 554:
A 58-year-old man with a Past medical history liver cirrhosis secondary to hepatitis C is being evaluated in the emergency department for early satiety and nausea. He reports a weight loss of 30 lbs over the last month and feels tired all the time. He is afebrile with a temperature of 37 C, a blood pressure 98/70 mmHg, pulse 78/min, respiratory rate 15 breaths/min, and SpO2 90% on room air. Liver function tests reveal total bilirubin 9mg/dL and direct bilirubin 6 mg/dL. Right upper quadrant Ultrasound does not reveal any lesion. CT abdomen and pelvis reveals a filling defect in the distal common bile duct. What is the next best step in the management of this patient?
Choices: 1. Endoscopic retrograde cholangiopancreatography with brushing of the common bile duct 2. Fibrosure scan 3. Proceed with Percutaneous transhepatic cholangiogram 4. Symptomatic management with IV fluids and pain parenteral nutrition
Answer: 1 - Endoscopic retrograde cholangiopancreatography with brushing of the common bile duct
Explanations: Endoscopic retrograde cholangiopancreatography(ERCP) is a technique that uses endoscopy to manage biliary or pancreatic ductal pathologies An ERCP can detect any abnormality in the biliary system including cancer in about 90% of the patients who are being worked-up for possible adenocarcinoma. ERCP is an important treatment modality for patients with cholangiocarcinoma, as it allows internal drainage of blocked bile ducts by using either a plastic or metal stents A sample obtained with brushing performed with ERCP can be sent to the pathology lab for histological diagnoses, given the clinical signs and symptoms and imaging findings, the first disease on the differential would be cholangiocarcinoma Go to the next page if you knew the correct answer, or click the link image(s) below to further research the concepts in this question (if desired).
Research Concepts: Hepatobiliary Tract Cancer
We update eBooks quarterly and Apps daily based on user feedback. Please tap flag to report any questions that need improvement.
Question 555:
A 65-year-old female known AIDS patient is undergoing intravenous therapy for cytomegalovirus infection. She failed a trial of ganciclovir due to resistance and has been started on a pyrophosphate analog. She was infused with the drug at a rate of 1mg/kg/min, with an induction dose of 60mg/kg/dose over an hour. Once maintenance therapy was initiated, she was left to be monitored over a few hours to assess her progress. Her urinalysis showed signs of muddy, granular casts on following up labs. Which of the following steps could have prevented the results that were seen on her urinalysis?
Choices: 1. 2. 3. 4.
1000mL hydration with 3% hypertonic saline Renal biopsy before administering the drug 800mL hydration with 0.9% normal saline Allergy testing before drug administration
Answer: 3 - 800mL hydration with 0.9% normal saline Explanations: Since renal toxicity is one of the major concerns around administering foscarnet, it can be reduced by concurrent 0.9% normal saline administration (about 0.5L to 1.0L). A fast infusion rate can induce renal toxicity as well as other adverse effects, including severe nausea. Slowing the infusion rate and hydration with normal saline will reduce the risks of both of these side effects. Foscarnet can also cause other nonspecific symptoms like bowel changes and headaches. 0.9 % normal saline is the best choice for hydration compared to other types of fluid. Hypertonic saline should be reserved for patients with hyponatremia. Go to the next page if you knew the correct answer, or click the link image(s) below to further research the concepts in this question (if desired).
Research Concepts: Foscarnet
We update eBooks quarterly and Apps daily based on user feedback. Please tap flag to report any questions that need improvement.
Question 556:
A 79-year-old man is found to have a 2.5 cm nodule in the right upper lobe on chest CT. CT guided biopsy demonstrates moderately differentiated adenocarcinoma. The lung nodule is PET-avid, with an SUV of 22. PET-CT is negative for distant metastatic disease. Pulmonary function test reveals an FEV1 of 50% predicted and DLCO of 30% predicted. What is the most appropriate management strategy for this patient?
Choices: 1. 2. 3. 4.
Lobectomy Chemotherapy Radiation therapy with concurrent chemotherapy Stereotactic body radiation therapy
Answer: 4 - Stereotactic body radiation therapy Explanations: Stereotactic body radiation therapy (SBRT) or stereotactic ablative body radiation (SABR) is an alternative option for patients who are not surgical candidates due to high operative risk from poor cardiopulmonary function, comorbid conditions, advanced age, or if patients refuse surgery. The 5-year survival for stage IA (T1N0M0) is 60-80%. Radiation therapy with concurrent chemotherapy is recommended for Stage IIIA-B non-small cell lung cancer. Radiation therapy is delivered over 6-7 weeks to a total dose of 60-70 Gy. If the patient is a surgical candidate, lobectomy is preferred over pneumonectomy, if anatomically feasible. Wedge resection is not considered oncologic surgery and is only considered if the patient is physiologically compromised. Go to the next page if you knew the correct answer, or click the link image(s) below to further research the concepts in this question (if desired).
Research Concepts: Radiation Therapy For Early Stage Non-Small Cell Lung Cancer
We update eBooks quarterly and Apps daily based on user feedback. Please tap flag to report any questions that need improvement.
Question 557:
A 65-year-old male with non-small cell lung cancer undergoes a lobectomy. Postoperatively, he becomes hypotensive, hypothermic, and unresponsive to a fluid bolus and the usual pressor drugs. Which of the following is best for the immediate treatment of this patient?
Choices: 1. 2. 3. 4.
500 ml of 5% albumin 100 mg hydrocortisone intravenously 2 grams of magnesium sulfate Intra-aortic balloon pump
Answer: 2 - 100 mg hydrocortisone intravenously Explanations: In a patient with non-small cell lung cancer, adrenal insufficiency should be considered. These lung cancers can secrete endocrine hormones, and patients may present with adrenal insufficiency when the tumor is removed. Adrenal insufficiency should be immediately treated with corticosteroids, usually hydrocortisone. Dexamethasone is another initial choice for the management of these patients. In a patient who is hypotensive, hypothermic, febrile, and unresponsive to pressor agents, adrenal insufficiency should be suspected. Go to the next page if you knew the correct answer, or click the link image(s) below to further research the concepts in this question (if desired).
Research Concepts: Adrenal Insufficiency
We update eBooks quarterly and Apps daily based on user feedback. Please tap flag to report any questions that need improvement.
Question 558:
A 68-year-old man with a history of prostate cancer managed with leuprolide is brought to the emergency department by his daughter. She describes that her father has become increasingly confused, somnolent, dizzy, and complained of nausea over the past two days. He is currently taking a transdermal fentanyl patch for his chronic pain associated with his prostate cancer, and he is taking oral transmucosal fentanyl for breakthrough pain episodes. The daughter shares that her father had acute otitis media of the right ear and started taking clarithromycin for the past 8 days. His temperature is 37.2 C (99 F), blood pressure 95/55 mmHg, pulse 77/min, and respiratory rate 12/minute. Lower and upper extremity muscle strength, bicep and patellar tendon reflexes, and sensations are normal. Which of the following is the most likely cause of the patient's signs and symptoms?
Choices: 1. Overdose from oral transmucosal fentanyl 2. The additive effect of oral transmucosal fentanyl and transdermal fentanyl patch 3. Drug interaction 4. Metastasis of prostate cancer to the brain stem
Answer: 3 - Drug interaction Explanations: The patient is experiencing signs and symptoms of higher than therapeutic level fentanyl plasma levels: nausea, dizziness, somnolence, confusion, decreased respiratory rate, hypotension. Clarithromycin is a CYP3A4 inhibitor, which is an enzyme used to metabolize fentanyl. Thus, the concomitant intake of clarithromycin with oral and transdermal fentanyl resulted in increased plasma levels of fentanyl, which contributed to the patient's signs and symptoms. Physicians and healthcare workers must always inquire about which medications a patient is taking before prescribing new medications. Physicians must be vigilant about drug-drug interactions, especially with opioids, since higher plasma levels can lead to fatal respiratory depression. This patient's condition could worsen if no intervention is initiated as it can lead to respiratory depression secondary to toxic levels of plasma fentanyl. The physician must immediately adjust the patient's oral transmucosal fentanyl dose or consider switching the patient to a different class of antibiotic that does not affect CYP3A4 activity. Go to the next page if you knew the correct answer, or click the link image(s) below to further research the concepts in this question (if desired).
Research Concepts: Oral Transmucosal Fentanyl
We update eBooks quarterly and Apps daily based on user feedback. Please tap flag to report any questions that need improvement.
Question 559:
Which of the following chemotherapy agents is used for the treatment of chronic myelogenous leukemia?
Choices: 1. 2. 3. 4.
Vinblastine Imatinib Rituximab Anastrozole
Answer: 2 - Imatinib Explanations: Imatinib works by inhibiting the BCR-ABL gene that has tyrosine kinase activity. This mutant gene is implicated as the cause of chronic myelogenous leukemia. Imatinib is known to inhibit major liver enzymes and may cause drug-drug interactions. Rituximab is a monoclonal antibody for CD-20 B lymphocytes. Go to the next page if you knew the correct answer, or click the link image(s) below to further research the concepts in this question (if desired).
Research Concepts: Imatinib
We update eBooks quarterly and Apps daily based on user feedback. Please tap flag to report any questions that need improvement.
Question 560:
A 10-year-old boy is brought to the clinic with painless enlargement of the right testis. The mother denies any previous pathologies or familiar diseases. Testicular palpation reveals a well-defined mass, which is monolateral. Ultrasound examination reveals a heterogeneous component, with a hyperecogenous cyst containing hypoechoic material. Serological tests do not reveal any specific alteration. What is the most appropriate management strategy for this patient?
Choices: 1. 2. 3. 4.
Observation Chemotherapy Surgical excision Orchiopexy
Answer: 3 - Surgical excision Explanations: Testicular teratomas traditionally have been treated by simple or radical orchiectomy, although more recently testis-sparing enucleation has been advocated in prepubertal teratomas. If a conservative approach is taken, a frozen section could be examined, and if areas of testicular maturity are revealed, orchiectomy should be performed. The more aggressive approach is warranted in the context of pubertal changes secondary to the unpredictable prognosis and much higher risk of metastasis in mature tissue. In contrast, clinical experience has substantiated that testicular teratomas in children are universally benign (pre-pubertal type teratoma). Go to the next page if you knew the correct answer, or click the link image(s) below to further research the concepts in this question (if desired).
Research Concepts: Testicular Teratoma
We update eBooks quarterly and Apps daily based on user feedback. Please tap flag to report any questions that need improvement.
Question 561:
A 65-year-old male patient presents to the office with complaints of fatigue, abdominal pain, and two episodes of bleeding per rectum. The patient states that the bleeding was painless, and there was a streak of blood in the stools. He also reports cough, low-grade fever, and undocumented weight loss in the last couple of months. The clinician performs a proctoscopy, which shows an abnormal mass in the rectum. Laboratory analysis reveals raised liver function tests. CT scan of the chest and liver reveals multiple metastatic lesions. What is the nature of the most specific marker that is most likely raised in this patient's suspected diagnosis?
Choices: 1. 2. 3. 4.
Glycoprotein Lipoprotein Metalloprotein Phosphoprotein
Answer: 1 - Glycoprotein Explanations: This patient has most likely developed metastatic colorectal carcinoma (CRC). Patients with CRC usually have raised levels of carcinoembryonic antigen (CEA). Human carcinoembryonic antigen (CEA) is a glycoprotein present in fetal blood and a minimal amount in adults. CEA contains carbohydrate structures containing Nacetyl-glucosamine, mannose, galactose, fucose, and sialic acid. High levels of carcinoembryonic antigen suggest a variety of cancers, especially colon or pancreatic cancer. Carcinoembryonic antigen is also often used to monitor patients' response to cancer treatment. Go to the next page if you knew the correct answer, or click the link image(s) below to further research the concepts in this question (if desired).
Research Concepts: Carcinoembryonic Antigen
We update eBooks quarterly and Apps daily based on user feedback. Please tap flag to report any questions that need improvement.
Question 562:
A 65-year-old male presents with complaints of nausea, vomiting, and right upper quadrant pain for the past 2 weeks. His past medical history is significant for hypertension, diabetes mellitus type 2, cholecystectomy, and recently diagnosed renal cell carcinoma. Vital signs include blood pressure 120/85 mmHg, heart rate 120/min, respiratory rate 14/min, and temperature 37 C (98.6 F). Physical examination shows right upper quadrant tenderness, distended abdomen with shifting dullness, distended abdominal blood vessels, and jaundice. The liver is palpable 2 cm under the costal margin. Laboratory tests include hemoglobin 18 g/dL, leukocytes 8,000/microL, and platelets 450,000/microL. The acute hepatitis viral panel is negative. What is the most likely complication of this condition?
Choices: 1. 2. 3. 4.
Intra-abdominal adhesions Abscess formation Cirrhosis Right-sided heart failure
Answer: 3 - Cirrhosis Explanations: In a patient with malignancy, there is an increased incidence of venous thrombosis. Given the patient's history of renal cell carcinoma and acute onset, thrombosis of the hepatic veins, that is, Budd-Chiari syndrome is likely. Acute Budd-Chiari syndrome often presents as severe right upper quadrant pain that develops over a few weeks. Variceal bleeding may be present in Budd-Chiari syndrome but usually develops later. Budd-Chiari syndrome has no association with intraabdominal adhesions. Right heart failure can cause hepatic vein thrombosis but is not labeled as a complication of Budd-Chiari syndrome. This syndrome is not associated with infections; hence an abscess is unlikely. Go to the next page if you knew the correct answer, or click the link image(s) below to further research the concepts in this question (if desired).
Research Concepts: Budd Chiari Syndrome
We update eBooks quarterly and Apps daily based on user feedback. Please tap flag to report any questions that
need improvement.
Question 563:
A 52-year-old man was recently diagnosed with a mediastinal germ cell tumor of embryonal carcinoma type that is infiltrating into his right lung. He has a 30-pack-year history of smoking with a diagnosis of mild chronic obstructive pulmonary disease. He also has a history of chronic pancreatitis. His performance status is Eastern Cooperative Oncology Group 1. His pulmonary function tests show an FEV1/FVC ratio of 63%. What is the most appropriate treatment?
Choices: 1. Four cycles of cisplatin, bleomycin, and etoposide (BEP) alone 2. Four cycles of cisplatin, ifosfamide, and etoposide (VIP) alone 3. Four cycles of BEP followed by resection of the residual tumor with lobectomy, if indicated 4. Four cycles of VIP followed by resection of the residual tumor with lobectomy, if indicated
Answer: 4 - Four cycles of VIP followed by resection of the residual tumor with lobectomy, if indicated
Explanations: Patients with mediastinal germ cell tumors often have residual disease post-chemotherapy. Surgical resection of residual tumor improves overall survival and is indicated. Although this patient has chronic obstructive pulmonary disease, his lung reserve and performance status are adequate to undergo lobectomy. Therefore, chemotherapy alone would not be appropriate for this patient. Bleomycin is associated with pulmonary toxicity, especially in patients with preexisting lung disease. Postoperative respiratory failure and death have been reported in patients with mediastinal germ cell tumors who underwent lung surgery after cisplatin, bleomycin, and etoposide (BEP). Four cycles of cisplatin, ifosfamide, and etoposide (VIP) is an appropriate alternative with similar efficacy to BEP. Go to the next page if you knew the correct answer, or click the link image(s) below to further research the concepts in this question (if desired).
Research Concepts: Mediastinal Nonseminoma
We update eBooks quarterly and Apps daily based on user feedback. Please tap flag to report any questions that need improvement.
Question 564:
A 67-year-old woman presents to the hospital with complaints of excessive fatiguability and gradual weight loss for the past 8 months. The patient has no fever, cough, or night sweats. On examination, the temperature is 37 C (98.6 F), pulse rate is 78 beats/minute, and blood pressure is 130/80 mmHg. An abdominal examination reveals splenomegaly. On investigation, hemoglobin is 10 g/dL, leukocyte count 68,000 cells/mm3, platelets 100,000 cells/mm3, and peripheral blood smear shows smudge cells. The drug used to treat this disease most likely inhibits the receptor activated by which of the following molecules?
Choices: 1. 2. 3. 4.
Thyroid-stimulating hormone Epidermal growth factor Vasopressin Histamine
Answer: 2 - Epidermal growth factor Explanations: The patient most likely has chronic myeloid leukemia (CML). Imatinib, an inhibitor of the tyrosine kinase receptor, is used for the treatment of CML due to the Philadelphia chromosome. Epidermal growth factor receptor is a glycoprotein with a carboxy-terminal region and a tyrosine kinase domain. Monoclonal antibodies, ligand-binding cytotoxic agents, and immunotoxins all target the carboxy-terminal region. Another target is the intracellular tyrosine kinase region of the same receptor. There is interference with ATP binding to the receptor that causes autophosphorylation. This would otherwise trigger a signaling network that results in tumor cell proliferation, angiogenesis, motility, metastasis, and protection from apoptosis. Go to the next page if you knew the correct answer, or click the link image(s) below to further research the concepts in this question (if desired).
Research Concepts: Biochemistry, Epidermal Growth Factor Receptor
We update eBooks quarterly and Apps daily based on user feedback. Please tap flag to report any questions that
need improvement.
Question 565:
A 75-year-old White male presents for a growing, erythematous papule on his forehead. Biopsy reveals a fairly-well circumscribed dermal proliferation that stains positive for CD10 and negative for S100, SMA, and CK5/6. Which of the following is a known risk factor for the development of this lesion?
Choices: 1. 2. 3. 4.
Asian race Female sex Li Fraumeni syndrome Presence of >100 melanocytic nevi
Answer: 3 - Li Fraumeni syndrome Explanations: Epidemiologic studies have shown that men are more likely to develop atypical fibroxanthoma than women. Acquired or inherited forms of immunosuppression, such as Li Fraumeni syndrome (with germline p53 mutations), are a risk factor for the development of atypical fibroxanthoma. While no direct association has been made, researchers hypothesize that arsenic exposure may predispose patients to atypical fibroxanthoma. Trauma to the skin and radiation therapy may also be risk factors for the development of atypical fibroxanthoma. Go to the next page if you knew the correct answer, or click the link image(s) below to further research the concepts in this question (if desired).
Research Concepts: Atypical Fibroxanthoma
We update eBooks quarterly and Apps daily based on user feedback. Please tap flag to report any questions that need improvement.
Question 566:
A 60-year-old woman with a history of leukemia is brought to the emergency department with altered mentation. On physical examination, her blood pressure is 80/40 mmHg, her pulse rate is 110/min, and she has a low-grade temperature. She is disoriented and lethargic. She has recently been prescribed sunitinib for leukemia treatment. An initial set of investigations is shown below. Reference Patient results range Hemoglobin 9 g/dL 11.5-16 g/dL 4100WBC count 9840/microL 10900/microL 150000Platelets 297000/microL 400000/microL Creatinine 1.09 mg/dL 0.8-1.4 mg/dL 134-144 Sodium 126 mEq/L mEq/L Potassium 5.2 mEq/L 3.6-5.0 mEq/L Chloride 93 mEq/L 98-107 mEq/L Bicarbonate 21 mEq/L 21-28 mEq/L Urinalysis showed cloudy urine, with a trace of blood, increased WBC, and leukocyte esterase consistent with a urinary tract infection. A chest radiography, CT of the abdomen and pelvis, and CT brain do not reveal any abnormalities. Blood and urine cultures are sent. The patient is given 3 liters of intravenous fluid resuscitation along with broad-spectrum antibiotics. However, her hypotension persists, so vasopressors are then initiated. Which of the following cause is contributing to the persistent hypotension?
Choices: 1. 2. 3. 4.
Severe sepsis from urinary tract infection Sunitinib induced pituitary failure Sunitinib induced adrenal insufficiency Anemia
Answer: 3 - Sunitinib induced adrenal insufficiency Explanations: The patient has an adrenal crisis precipitated by sunitinib use. Patients on tyrosine kinase inhibitors should be educated that they can precipitate adrenal insufficiency. The patient has hypotension with tachycardia, hyponatremia, anemia, and hyperkalemia with confusion, confirming the diagnosis of adrenal crisis. Emergency treatment of suspected adrenal crisis is immediate administration of iv hydrocortisone, aggressive IV fluid resuscitation, and appropriate antibiotics. The dose of intravenous hydrocortisone should be 100 mg (stress dose ) intravenously or intramuscularly (IV/IM) as an initial bolus followed by 100 to 300 mg daily after that for another 2-3 days of boluses every 6 hours or as continuous infusion until complete recovery. These individuals also require aggressive fluid and vasopressors. A thorough search should be done for the cause, and empiric antibiotics are recommended. Close monitoring in the intensive care unit is required. At discharge, the patient should be given a medical bracelet that alerts medical personnel about adrenal insufficiency. Adrenal crises can lead to severe refractory shock and death if not promptly recognized. Even though the patient has severe sepsis from urinary tract infection, treating underlying sunitinib-induced adrenal insufficiency with intravenous steroids, along with IV fluid resuscitation, and antibiotics will help improve hypotension. Sunitinib does not cause pituitary failure.
Go to the next page if you knew the correct answer, or click the link image(s) below to further research the concepts in this question (if desired).
Research Concepts: Adrenal Crisis
We update eBooks quarterly and Apps daily based on user feedback. Please tap flag to report any questions that need improvement.
Question 567:
A 65-year-old woman is suffering from intractable pain due to stage IV ovarian cancer. She has tried multiple opioids via oral and IV routes, and she has even tried intrathecal hydromorphone. Pregabalin, nonsteroidal anti-inflammatory drugs (NSAIDs), acetaminophen, and dexamethasone have been ineffective as adjuvant agents. The pain is causing significant emotional distress for both the patient and her family. Her spouse asks about something that he found on the internet called palliative sedation, and he is wondering whether it might help. What the patient and her family should be told about the procedure?
Choices: 1. It includes the withdrawal of care except for sedative measures 2. The aim is to hasten death as a means to relieve a patient's symptoms 3. It requires an interprofessional approach to treat symptoms and address multiple forms of suffering 4. It is inappropriate for anyone under 75 years of age
Answer: 3 - It requires an interprofessional approach to treat symptoms and address multiple forms of suffering
Explanations: Palliative sedation is part of the palliative care measures. It aims to provide relief from pain and other distressing symptoms when multiple other efforts have failed to do so. It is accepted that providers can administer medications to relieve pain or other sufferings even if doing so poses some increased risk of death. Under the principle of double effect, the risk of death is acceptable as long as the medication is used in a clinically reasonable effort to treat the patient’s pain, but death must not be intended. Palliative sedation, as part of palliative care, requires an interprofessional approach to treat symptoms. It is used in any near-to-death, generally less than twoweek prognosis, in a patient of any age with refractory symptoms. In patients with this short of a life expectancy, palliative sedation is not found to hasten death. Go to the next page if you knew the correct answer, or click the link image(s) below to further research the concepts in this question (if desired).
Research Concepts: Palliative Sedation In Patients With Terminal Illness
We update eBooks quarterly and Apps daily based on user feedback. Please tap flag to report any questions that need improvement.
Question 568:
A 54-year-old male patient noticed decreased hearing in the right ear since 4 months ago. He can not distinguish words adequately when using the phone on the right side. He was diagnosed with moderate sensorineural damage. Magnetic resonance imaging was performed and showed a brain tumor in the cerebellopontine angle. Which one of the following conditions would be most likely to cause injury to the cranial nerve VIII?
Choices: 1. 2. 3. 4.
Pilocytic astrocytoma Epidermoid cyst Cerebellopontine angle meningioma Vestibular schwannoma
Answer: 4 - Vestibular schwannoma Explanations: Vestibular schwannoma is the most common tumor at the cerebellopontine angle causing sensorineural damage. Vestibular schwannomas and meningiomas are the two most common tumors of the cerebellopontine angle, followed by epidermoids. Cerebellopontine angle meningioma grows slowly and may develop damage to the VIII when it grows inside the internal acoustic meatus. Epidermoid tumor rarely causes cranial nerve VIII injury as it grows around the cranial nerve but does not compress the nerve. Surgery may injure the nerve. Go to the next page if you knew the correct answer, or click the link image(s) below to further research the concepts in this question (if desired).
Research Concepts: Epidermoid Cancer
We update eBooks quarterly and Apps daily based on user feedback. Please tap flag to report any questions that need improvement.
Question 569:
A 24-year-old male presented in the emergency with the complaint of one episode of hematemesis three hours back. Previously, he had been having complaints of intermittent epigastric pain, early satiety, and constipation for the last six months. On examination, there was epigastric tenderness on deep palpation. Esophagogastroduodenoscopy was done, which showed esophageal and gastric ulcers along the lesser curvature. A mass was found in the gastric antrum. He was admitted, and colonoscopy was done, which revealed a stricture in the middle of the transverse colon and ileocecal valve. Biopsies were taken and sent for histopathology. CT scan of the abdomen with contrast demonstrated a 4 cm well-enhanced lobulated mass located in the gastric antrum and others in the duodenum loop, and pancreatic head. Diffuse lymphadenopathy was seen. Biopsy results showed nests of poorly differentiated cells that stained positive for desmin and neuron-specific enolase. Which of the following will be the best management for this tumor?
Choices: 1. Cytoreductive surgery + hyperthermic intraperitoneal chemotherapy (P6 protocol) + radiotherapy 2. Neoadjuvant chemotherapy (P6 protocol) + aggressive surgery + radiotherapy 3. Whole abdominal radiotherapy alone 4. Palliative chemotherapy alone (P6 protocol)
Answer: 2 - Neoadjuvant chemotherapy (P6 protocol) + aggressive surgery + radiotherapy
Explanations: Primary peritoneal cancer (PMC) is subdivided into extra ovarian, primary peritoneal carcinoma, malignant mesothelioma (MPM), multicystic mesothelioma, leiomyosarcomas, leiomyomatosis peritonealis disseminata, and desmoplastic small round cells tumor. Desmoplastic round small cell tumor occurs in adolescents (median age 19 years) and 85% in Caucasians. Gerald and Rosai first described it in 1989. It is caused by t(11;22)(p13;q12) translocation. It belongs to the Ewing's sarcoma tumor family. The presenting symptoms are diverse and vague. Most present with non-specific symptoms like abdominal pain, distension, early satiety, anorexia, nausea, weight loss, and constipation. Imaging studies demonstrate an abdominal mass with peritoneal implants. Lymphadenopathy is common. Liver metastasis can also be present at the initial diagnosis. This tumor has aggressive features with a dismal prognosis. On biopsy, it presents as a small round blue cell tumor and consists of clumps, nests or sheets of small round to ovoid cells with extensive mitotic figures. It stains with three different markers delineating its multilineage origin; epithelial cytokeratin, mesenchymal desmin and vimentin, and neural neuron-specific enolase. In DSRCT, neo-adjuvant chemotherapy is the main approach for management. Systemic chemotherapy comprising cyclophosphamide, ifosfamide, vincristine, etoposide, doxorubicin, and mesna (P6 protocol) is done, followed by aggressive surgical excision. HIPEC is
regarded as adjunctive therapy. Consolidative whole abdominal radiotherapy in the pediatric population in particular and in adults generally is essential to improve the outcome. Palliative chemotherapy is reserved for recurrence. Go to the next page if you knew the correct answer, or click the link image(s) below to further research the concepts in this question (if desired).
Research Concepts: Peritoneal Cancer
We update eBooks quarterly and Apps daily based on user feedback. Please tap flag to report any questions that need improvement.
Question 570:
A 53-year-old man with a history of chronic Hepatitis B liver disease presents with a hepatic mass that was found on computed tomography (CT) imaging for work-up of colon cancer. The physical exam was unremarkable. Liver function tests, fetal alpha protein (AFP), and carcinoembryonic antigen (CEA) all within normal limits. Magnetic resonance imaging (MRI) of the liver shows several 1 cm cystic masses in the right liver lobe with hypointensity on T1-weighted imaging. Which of the following is the next best step in the management of this patient?
Choices: 1. 2. 3. 4.
Ultrasound-guided core biopsy Ultrasound-guided fine-needle aspiration and cytology Triple phase-contrast CT abdomen No further diagnostic test required
Answer: 1 - Ultrasound-guided core biopsy Explanations: US-guided core biopsy is the gold standard for the diagnosis of biliary hamartoma. In cases of known primary malignancy, the core or wedge biopsy is required to diagnose biliary hamartoma. If the lesions are too tiny, and biopsy is non-yielding, long-term follow-up imaging may be performed. Ultrasound-guided fine-needle aspiration is not useful for the diagnosis of biliary hamartoma. CT imaging can diagnose the incidental findings of multiple biliary hamartomas. MRI is the best modality for characterization. In the absence of known primary malignancy and with typical MR findings of biliary hamartoma, no further diagnostic test is necessary. However, long-term imaging follow-up is required as there is a known, but rare, the association of multiple biliary hamartomas and subsequent cholangiocarcinoma. Go to the next page if you knew the correct answer, or click the link image(s) below to further research the concepts in this question (if desired).
Research Concepts: Biliary Duct Hamartoma
We update eBooks quarterly and Apps daily based on user feedback. Please tap flag to report any questions that need improvement.
Question 571:
A 56-year-old male presents to the clinic with swelling on his left cheek for one month. Vitals are within normal limits. He denies any pain or cutaneous changes. He is a nonsmoker. Physical exam reveals a nonmobile mass over the left parotid gland and normal neurological exam. Imaging shows a solitary mass. What are the most likely diagnosis and the course of treatment?
Choices: 1. Mucoepidermoid cancer, surgical excision for likely malignancy 2. Pleomorphic adenoma, surgical excision for possible malignant transformation/malignancy 3. Squamous cell carcinoma, radiation 4. Warthin tumor, a benign lesion requiring observation
Answer: 2 - Pleomorphic adenoma, surgical excision for possible malignant transformation/malignancy
Explanations: 80% of salivary gland neoplasias are in the parotid gland, while 10% to 15% are from submandibular glands. 80% of parotid gland tumors are benign, while smaller salivary gland neoplasia is more likely malignant. 60% of parotid gland tumors are pleomorphic adenomas, 8% are mucoepidermoid cancer, and 7% are Warthin tumors. Surgery is necessary for pleomorphic adenoma because of the risk of malignant transformation. Only 2% of parotid gland neoplasias are secondary to squamous cell carcinoma. Go to the next page if you knew the correct answer, or click the link image(s) below to further research the concepts in this question (if desired).
Research Concepts: Parotid Cancer
We update eBooks quarterly and Apps daily based on user feedback. Please tap flag to report any questions that need improvement.
Question 572:
A 50-year-old man presents to the hospital with nausea and vomiting. He has a history of nonresectable cholangiocarcinoma, causing obstructive jaundice. He undergoes percutaneous transhepatic biliary drainage (PTBD) to relieve jaundice. Following the procedure, he starts to have increasing shortness of breath. Which of the following is most likely responsible for the patient's condition?
Choices: 1. 2. 3. 4.
Cholangitis Biliary peritonitis Pneumonia Pneumothorax
Answer: 4 - Pneumothorax Explanations: PTBD is done as an interventional radiology procedure. Bile leakage and pneumothorax are complications that may happen with PTBD specifically. Other complications of PTBD include stent dislodgment, patient discomfort, and electrolyte abnormalities secondary to loss of bile and its contents. Pneumothorax is one of the complications of PTBD that would manifest as shortness of breath. Go to the next page if you knew the correct answer, or click the link image(s) below to further research the concepts in this question (if desired).
Research Concepts: Biliary Stenting
We update eBooks quarterly and Apps daily based on user feedback. Please tap flag to report any questions that need improvement.
Question 573:
A 50-year-old female with core biopsyproven left breast cancer is found to have a positive fineneedle aspiration of a left axillary node. She is scheduled to undergo left breast conservation surgery. Which of the following is the next best step in the management of this patient?
Choices: 1. Perform a left axillary node dissection 2. Perform a left sentinel lymph node biopsy 3. Do not perform a lymph node dissection and have the patient undergo postoperative radiation 4. Do not perform a lymph node dissection and have the patient undergo postoperative chemotherapy
Answer: 1 - Perform a left axillary node dissection Explanations: Axillary lymph node dissection is indicated if the patient is found to have metastasis to the supraclavicular and/or internal mammary nodes. A sentinel lymph node biopsy is not appropriate and will likely miss positive nodes and leave disease-ridden nodes in the patient. Postoperative radiation may be required or considered as an alternative, ut an axillary lymph node dissection remains standard of care. Postoperative chemotherapy may be required, but an axillary lymph node dissection should be pursued during surgery. Go to the next page if you knew the correct answer, or click the link image(s) below to further research the concepts in this question (if desired).
Research Concepts: Breast Lymphatics
We update eBooks quarterly and Apps daily based on user feedback. Please tap flag to report any questions that need improvement.
Question 574:
A 65-year-old male with small cell lung cancer undergoing chemotherapy is admitted to the hospital with unexplained muscle twitches, paresthesias, hallucinations, insomnia, excessive sweating, excessive thirst, and palpitations. The patient was found to have a stocking-type sensory loss, loss of deep tendon reflexes, and memory loss. Initial lab work is only remarkable for hyponatremia. CT and MRI scan of the brain is normal. Electromyography (EMG) shows bursts of a doublet, triplet, and multiplet continuous muscle fasciculations consistent with neuromyotonic discharges. What is the most likely pathophysiology responsible for the patient's condition?
Choices: 1. Antibodies antibodies 2. Antibodies antibodies 3. Antibodies antibodies 4. Antibodies antibodies
against voltage-gated chloride channel against voltage-gated sodium channel against voltage-gated calcium channel against voltage-gated potassium channel
Answer: 4 - Antibodies against voltage-gated potassium channel antibodies
Explanations: Morvan syndrome is an autoimmune phenomenon with unknown triggering factors. Thymectomy, chemotherapy, prostate surgery, and scrotal drainage have been reported to trigger Morvan’s syndrome. There is overwhelming evidence that Morvan syndrome is an autoimmune phenomenon where a strong association with autoantibodies to voltage-gated potassium channel complex (VGKCs) proteins, namely CASPR2 and LGI1, are implicated. Elevated titers of these antibodies help diagnose and monitor disease progression. Morvan's syndrome is a quite rare disease with a very low incidence rate. Go to the next page if you knew the correct answer, or click the link image(s) below to further research the concepts in this question (if desired).
Research Concepts: Morvan Syndrome
We update eBooks quarterly and Apps daily based on user feedback. Please tap flag to report any questions that need improvement.
Question 575:
A 35-year-old man presents to the clinic with complaints of cough, dyspnea, chest pain, fever, night sweats, and weight loss. Physical examination reveals facial plethora and prominent neck veins. Laboratory testing shows elevated levels of alpha-fetoprotein (AFP), betahuman chorionic gonadotropin (b-HCG), and lactate dehydrogenase (LDH). Further evaluation reveals a mediastinal tumor that eventually responds well to chemotherapy. Which of the following long-term complications is most likely to develop in this patient?
Choices: 1. 2. 3. 4.
Lung cancer Testicular cancer Bone cancer Hematological malignancy
Answer: 4 - Hematological malignancy Explanations: The sign and symptoms in this patient are most suggestive of mediastinal non-seminomatous germ cell tumors. Germ cell tumors are cancers that arise from the reproductive cells of the testis or ovaries. Sometimes, the germ cells may arise in the mediastinum, chiefly in young males. Of these, nearly 70% are non-seminomas, and the rest are seminomas. A complete physical exam, including a genital exam, is crucial. Alpha-fetoprotein (AFP), beta-human chorionic gonadotropin (beta-HCG), and lactate dehydrogenase (LDH) are the tumor markers that can be elevated in these patients. Mediastinal germ cell tumors are noted as an anterior mediastinal mass in an x-ray. Teratomas can manifest with derivatives of more than one germ layer that can be seen in an x-ray, including in bones, calcification, and teeth. Although chest x-rays can identify a larger mediastinal mass, a computed tomography (CT) or magnetic resonance imaging (MRI) of the chest is required to define the size, borders, and the location of the mass accurately. Mediastinal non-seminomatous GCT (MNSGCT) patients have increased risk for developing secondary hematological disorders and malignancies such as acute myeloid leukemia, mast cell leukemia, myelodysplastic syndrome, and malignant histiocytosis. Go to the next page if you knew the correct answer, or click the link image(s) below to further research the concepts in this question (if desired).
Research Concepts: Mediastinal Nonseminoma
We update eBooks quarterly and Apps daily based on user feedback. Please tap flag to report any questions that need improvement.
Question 576:
A 54-year-old female consults with a radiation oncologist after a recent diagnosis of invasive ductal carcinoma of the right breast. She comes into the room with an assistance dog and a white cane. She has discussed treatment options with her medical oncologist, which include a total mastectomy or the possibility of preserving part of the breast (partial mastectomy/lumpectomy) if she also received radiation therapy (RT) to the breast tissue. On physical examination, the clinician finds she underwent bilateral enucleation of both eyes, which she refers occurred at some point during her childhood but does not know the exact reason. Considering that RT can increase the risk of second treatment-associated malignancies. What additional testing would be most useful before treatment initiation given this patient’s clinical picture?
Choices: 1. She does not require any additional testing and should be scheduled for RT simulation 2. Mutational panel for TP53 gene 3. Mutational panel for Rb gene 4. Cyclin-dependent kinase levels in peripheral blood
Answer: 3 - Mutational panel for Rb gene Explanations: This patient is presenting a status post bilateral enucleation as a child. Besides traumatic injury. There are very few indications for bilateral enucleation in pediatrics, one being bilateral retinoblastoma, which is associated with germline mutations in the Rb gene. There are very few indications for bilateral enucleation in pediatrics, one being bilateral retinoblastoma, which is associated with germline mutations in the Rb gene. RT in a patient with Rb mutation would be highly discouraged if another option, such as total mastectomy, is possible; due to the increased risk of treatment-associated malignancies with RT such as chest-wall sarcoma. There would be no current role for measuring CDK levels in peripheral blood; CDK inhibitors are now therapeutic agents used in metastatic breast cancer. However, peripheral levels of CDK are not of clinical utility and were not mentioned in this article. Though mutations in TP53 are associated with Li-Fraumeni Syndrome and an increase in the risk of breast cancer, the most useful test should rule out a genetic syndrome, which also explains the patient’s bilateral enucleation. There are findings on PE that should lead you to think about a concomitant genetic disorder, which could lead to a higher-than acceptable risk of second primaries if treatment with RT is initiated. Go to the next page if you knew the correct answer, or click the link image(s) below to further research the concepts in this question (if desired).
Research Concepts: Genetics, Cancer Cell Cycle Phases
We update eBooks quarterly and Apps daily based on user feedback. Please tap flag to report any questions that need improvement.
Question 577:
A 51-years old male presents with chief complaints of abdominal discomfort and abdominal distention for the last four months. The discomfort is exacerbated while lying supine and relieved when standing upright. He lost 5 kg during the same period. Abdominal examination shows a palpable mass of about 5 cm x 6 cm situated in the left flank, which is laterally ballotable. Computed tomography of the abdomen shows a flat mass with multiple cystic spaces and enhancement of solid areas. It is located in the anterior compartment of the abdomen anterior to the small bowel and transverse colon. Histopathology and immunohistochemistry report shows a well-differentiated epithelial pattern; tumor cells are positive for calretinin, cytokeratin, and negative for S-100 protein, Leu-M1. What is the diagnosis?
Choices: 1. 2. 3. 4.
Leiomyosarcoma of omentum Primary omental mesothelioma Solitary fibrous tumor of the omentum Gastrointesinal stromal tumors of omentum
Answer: 2 - Primary omental mesothelioma Explanations: Histological examination of mesothelioma of omentum may show an epithelial, sarcomatoid, or biphasic pattern. Epithelial type tumors make up 75% of cases and vary from well-differentiated with tubulopapillary pattern to solid sheets of round or polygonal cells. Such tumors may mimic carcinomas, and it can be difficult to distinguish epithelioid mesothelioma from a metastatic carcinoma, particularly adenocarcinoma. The sarcomatoid type tumors may be indistinguishable from fibrosarcomas on histology alone. Immunohistochemistry can be helpful in differentiating sarcoma and adenocarcinoma. These tumor cells are positive for calretinin, cytokeratin and epithelial membrane antigen, and negative for S-100 protein, LeuM1, CEA, thrombomodulin, and placental alkaline phosphatase, positive immunoreactivity for calretinin greatly increase the accuracy of the diagnosis. Extragastrointestinal stromal tumors like gastrointestinal stromal tumors are typically positive for KIT, alpha-smooth muscle actin, and negative for desmin and s-100 protein. CD34 is a useful positive biomarker for solitary fibrous tumor (SFT); few studies have shown a 100% positivity rate. Leiomyosarcoma is positive for desmin and smooth muscle actin and strongly negative for S100 and CD34. Go to the next page if you knew the correct answer, or click the link image(s) below to further research the concepts in this question (if desired).
Research Concepts: Omentum Tumors
We update eBooks quarterly and Apps daily based on user feedback. Please tap flag to report any questions that need improvement.
Question 578:
A 45-year-old male with no significant past medical history presents to the clinic for evaluation of progressive fatigue, abdominal pain, nausea, and poor appetite for the last several months. He reports unintentional weight loss of 10 pounds (4.5 kg) during this period. His laboratory workup reveals a hemoglobin of 10.2 g/dL, a mean corpuscular volume (MCV) of 115 fL (normal: 80-100 fL), and normal renal function tests. Serum lactate dehydrogenase level is elevated. Serum folate and vitamin B12 levels are within normal limits. Which of the following is the most appropriate next test for this patient?
Choices: 1. 2. 3. 4.
Red blood cell folate level Antigastric parietal cell antibody testing Serum homocysteine level Serum methylmalonic acid level
Answer: 4 - Serum methylmalonic acid level Explanations: This patient likely has megaloblastic anemia. His relatively young age, unintentional weight loss, and abdominal symptoms are suggestive of pernicious anemia with possible gastric malignancy as a complication of the disease. In patients with pernicious anemia and intrinsic factor antibodies, serum vitamin B12 levels can be falsely low. Further testing with methylmalonic acid levels will help identify this deficiency. The patient needs vitamin B12 supplementation to prevent potentially irreversible neurologic complications of vitamin B12 deficiency. Methylmalonic acid levels are elevated in vitamin B12 deficiency but not in folate deficiency. Falsely elevated levels of methylmalonic acid are seen in patients with renal insufficiency, making the assay unreliable in these patients. Homocysteine is elevated in both vitamin B12 and folate deficiencies. This patient will eventually need serologic testing for anti-intrinsic factor antibody and an esophagogastroduodenoscopy (EGD) to diagnose pernicious anemia and evaluate for suspected gastric malignancy. Antigastric parietal cell antibody testing is unreliable and not recommended, even if serologic testing for intrinsic factor antibody returns negative. Go to the next page if you knew the correct answer, or click the link image(s) below to further research the concepts in this question (if desired).
Research Concepts:
Megaloblastic Anemia
We update eBooks quarterly and Apps daily based on user feedback. Please tap flag to report any questions that need improvement.
Question 579:
A 24-year-old man with a p53 mutation on genetic analysis presents to the office for routine physical. His family history is significant for Li-Fraumeni syndrome in his father, and his father had colon cancer diagnosed when he was 40 years old. What is the earliest age to recommend a colonoscopy for this patient?
Choices: 1. 2. 3. 4.
25 35 40 45
years years years years
old old old old
Answer: 1 - 25 years old Explanations: Patients with LFS have to undergo frequent check-ups, whole-body MRI, abdominal ultrasound. Patients with LFS have frequent testing of biochemical markers for adrenocortical functions. Blood test every 4 months: complete blood count, lactate dehydrogenase, erythrocyte sedimentation rate. Colonoscopy every 2 years, beginning at age 25 to 30 or 10 years before the earliest known colon cancer in the family. For breast cancer, women should undergo breast cancer monitoring, with annual breast MRI and twice-annual clinical breast examination beginning at age 20-25 years. Patients with LFS have to undergo frequent check-ups. Colonoscopy every 2 years, beginning at age 25 to 30 or 10 years before the earliest known colon cancer in the family. Go to the next page if you knew the correct answer, or click the link image(s) below to further research the concepts in this question (if desired).
Research Concepts: Li-Fraumeni Syndrome
We update eBooks quarterly and Apps daily based on user feedback. Please tap flag to report any questions that
need improvement.
Question 580:
A 58-years-old male presents with frequent fatigue, headache, and poor appetite for the last 2 months. He smokes 1 pack of cigarettes every day for the past 10 years. He drinks alcohol occasionally. His temperature is afebrile, blood pressure is 145/90 mmHg, the pulse is 78 bpm and respirations are 19/min. His past medical problems include hypertension, stable angina, and allergic rhinitis. His medications include nifedipine, aspirin, metoprolol, and cetirizine. Family history is significant for stroke in mother and pulmonary embolism in paternal grandfather. He wonders why he has poor exercise endurance despite being told he has a "high blood count" when he donated blood. His current Hb is 19.5 mg/dl, WBC 9500/mm3, HCT 55%, RBC count is 6.1 million/mm3, and MCV is 82 fl. Genetic testing confirms the diagnosis. He has no history of blood clots. Which of the following is the most likely treatment for this patient?
Choices: 1. 2. 3. 4.
Phlebotomy Hydroxyurea Hydroxyurea and phlebotomy Iron infusions
Answer: 1 - Phlebotomy Explanations: The patient in the given case likely has a hematological malignancy. His symptoms of fatigue, weight loss, headache, high blood pressure, and high Hb and RBC count points towards polycythemia vera. Polycythemia is an increase in the absolute red blood cell (RBC) mass. In polycythemia vera with low erythropoietin levels, the primary defect in nearly 95% of cases is an acquired mutation in JAK 2 kinase. Patients under the age of 60 AND without a history of thrombosis are considered lowrisk PV. Patients under the age of 60 and without a history of thrombosis are considered low-risk PV. Low-risk polycythemia vera patients are treated with daily lowdose aspirin and phlebotomy for goal Hct 45% Symptoms include fatigue, headache, dizziness, transient blurring of vision or TIA episodes. Facial ruddiness may be present along with clubbing on physical examination. Serum ferritin, vitamin B12, folate levels, serum creatinine, BUN, and LFTs must be evaluated. Iron infusion is contraindicated in PV because the goal is to create an iron deficiency anemia through phlebotomy. Hydroxyurea is added to the regimen if the patient is over age 60, has had a thrombotic event, or both. Go to the next page if you knew the correct answer, or click the link image(s) below to further research the concepts in this question (if desired).
Research Concepts:
Polycythemia Vera
We update eBooks quarterly and Apps daily based on user feedback. Please tap flag to report any questions that need improvement.
Question 581:
A 45-year-old man is scheduled for a staging endobronchial ultrasound (EBUS) of suspected lung cancer. Positron emission tomography (PET) has confirmed a 2 cm peripheral lesion in the left upper lobe. He has partially avid nodules at stations 11L, 4L, and 7. He has no distant metastases. Staging EBUS confirms malignant cells at 11L consistent with a non-small cell cancer. 4L and 7 are sampled, and nodal material is obtained, but no malignant cells are seen. Which of the following best identifies the nodal stage for this patient’s disease?
Choices: 1. 2. 3. 4.
T1aM0N0 T1aM0N1 T1aM0N2 T1aM0N3
Answer: 2 - T1aM0N1 Explanations: EBUS staging of lung cancer has a sensitivity of around 90%. A positive biopsy sample has a low false-positive rate and should be used to stage patients and guide oncological treatment planning accurately. Lung cancer staging by EBUS is based on the International Association for the Study of Lung Cancer's 8th edition TNM staging. N1 disease is where there is evidence of ipsilateral peribronchial and/or hilar lymph nodes and intrapulmonary lesions. N2 disease is where there is evidence of ipsilateral mediastinal and/or subcarinal lymph nodes. N3 disease is where there is evidence of contralateral mediastinal or hilar lymph nodes or the presence of scalene/supraclavicular nodes. Sampling of all PET avid lesions is essential as, in this case, staging as N1 disease allows for consideration of surgical resection and, therefore, curative intent. For non-small cell lung cancer patients with N1 disease on staging, overall 5-year survival remains low at around 40%. If there is a single N1 site involved, survival is around 48% at 5 years, falling to around 30% with multiple N1 sites involved. Go to the next page if you knew the correct answer, or click the link image(s) below to further research the concepts in this question (if desired).
Research Concepts: Sonography Endobronchial Assessment, Protocols, And Interpretation
We update eBooks quarterly and Apps daily based on user feedback. Please tap flag to report any questions that need improvement.
Question 582:
A 91-year-old man presents to the clinic with moderate lower back pain that is worse with movement and is limiting his daily activity for the past 2 weeks. The patient also experiences nocturnal pain episodes that awake him at night. The patient has a past medical history of severe asthma and currently has renal cell carcinoma with metastasis to the spine. He received palliative radiation therapy for lumbar spine metastases a month ago. He is using a heating pad for this back pain with no resolution. He has never taken opioids before. Physical examination demonstrates full muscle strength, normal patella reflexes, and normal sensation. Which of the following analgesic medications is most appropriate for this patient's lower back pain?
Choices: 1. 2. 3. 4.
Oral transmucosal fentanyl Nonsteroidal anti-inflammatory drugs Transdermal fentanyl patch Intravenous morphine
Answer: 2 - Nonsteroidal anti-inflammatory drugs Explanations: Renal cell carcinoma can metastases to bone such as the lumbar spine. This patient has bone metastases from renal cell carcinoma that is symptomatic despite the use of radiation therapy. Opioids should be avoided in the elderly due to the risk of falls and respiratory depression. Opioids should be avoided in severe asthmatic patients due to the risk of partial or complete airway obstruction which can lead to respiratory depression. Cancer-related pain that is moderate in severity can be treated with nonsteroidal anti-inflammatory drugs. Go to the next page if you knew the correct answer, or click the link image(s) below to further research the concepts in this question (if desired).
Research Concepts: Oral Transmucosal Fentanyl
We update eBooks quarterly and Apps daily based on user feedback. Please tap flag to report any questions that need improvement.
Question 583:
An 80-year-old Native American man presents to the clinic for a follow-up appointment with his son. The patient was recently diagnosed with metastatic pancreatic cancer after the computed tomography scan showed he had a 3 cm mass in the pancreas and several liver lesions, one of which was biopsied. During the last visit, oncologists had a very detailed discussion about the diagnosis, treatment, and prognosis. The patient had declined systemic chemotherapy at that time. The family had also mentioned that they were worried about the patient getting uncomfortable from pain, and pain medications were offered during that visit. After a thorough discussion with the oncologist, the patient had refused to use any kind of pain medication, including opioids, on multiple occasions. The patient has a history of hypertension, for which he takes amlodipine. The patient denies any abdominal pain at the current visit, but he grimaces during the abdominal examination. Meanwhile, his son presents herbal medicine to the oncologist to ask his opinion about using it for pain control. Which of the following is the next best step in the management of this patient?
Choices: 1. 2. 3. 4.
Gather more information about the herb Allow the use of the herb for pain control Allow the use of herb but prescribe opioid Recommend hospice care
Answer: 1 - Gather more information about the herb Explanations: It is not uncommon in certain cultures for patients to be very stoic regarding pain. Patients could deny having any pain and express only privately to family or friends. Clinicians should understand the cultural values of their patients and families and respect them whenever possible. In this case, the clinician should gather more information about the herb and make sure the active ingredient in the herb does not interact with the patient’s other medications. The patient has already decided not to use opioids, so option 3 is not correct. Go to the next page if you knew the correct answer, or click the link image(s) below to further research the concepts in this question (if desired).
Research Concepts: The Importance Of Cultural Competence in Pain and Palliative Care
We update eBooks quarterly and Apps daily based on user feedback. Please tap flag to report any questions that need improvement.
Question 584:
A 35-year-old female presents with a very tender, slow-growing mass on the flexor surface of her arm. MRI imaging is shown below. A biopsy is taken, which shows a biphasic appearance with patterns of spindle cells arranged in intersecting bundles and other patterns with less cellularity and loosely arranged cells. Appropriate treatment would include which of the following?
Choices: 1. 2. 3. 4.
Limb salvage procedure Marginal resection Marginal resection and radiation therapy Wide resection, chemotherapy, and radiation therapy
Answer: 2 - Marginal resection Explanations: This patient presents with symptomatic Schwannoma (neurilemmoma) of the forearm. Treatment is usually observational, but symptomatic masses may be treated with marginal resection. Schwannomas are benign soft tissue tumors that are derived from Schwann cells. They usually arise from the periphery of nerves, known as the epineurium. They most commonly originate in large peripheral nerves of the head, neck, and flexor surfaces of the upper extremities. Schwannomas appear isotonic to muscle on T1weighted imaging and hyperintense on T2-weighted imaging. There is a low risk of malignant transformation in benign peripheral nerve sheath tumors. Go to the next page if you knew the correct answer, or click the link image(s) below to further research the concepts in this question (if desired).
Research Concepts: Neurilemmoma
We update eBooks quarterly and Apps daily based on user feedback. Please tap flag to report any questions that need improvement.
Question 585:
A 70-year-old woman presents to the office for evaluation. Though she has never had breast cancer, she was recently diagnosed with a BRCA 2 mutation. Which of the following best describes the primary benefit of a bilateral prophylactic mastectomy for this patient?
Choices: 1. Improved survival 2. Decreased risk of developing breast cancer in the future 3. Decreased risk of developing ovarian cancer in the future 4. Decreased need for chemotherapy if she develops breast cancer
Answer: 2 - Decreased risk of developing breast cancer in the future
Explanations: Patients with BRCA1 and BRCA2 mutations experience worse breast cancer-specific survival when compared to BRCA-negative patients. BRCA1 carriers have worse overall survival than BRCA2 patients. Studies do show that women who carry BRCA mutations are more likely to develop secondary cancer - either in the same breast or the contralateral breast. For these women, prophylactic bilateral mastectomy is recommended. More important, studies show that women who do undergo surgery are less likely to die from breast cancer compared to women who were treated with unilateral mastectomy. The chance of developing breast cancer is greatly reduced with bilateral mastectomies; this risk is not zero, however. Bilateral prophylactic mastectomy will not increase survival for a 70-year-old woman who is undergoing breast cancer screening. Bilateral mastectomy will not alter the risk for ovarian cancer, though bilateral oophorectomy, in general, will decrease the risk for developing breast cancer. There is no evidence to support a decreased need for chemotherapy in patients who undergo bilateral prophylactic mastectomies. Go to the next page if you knew the correct answer, or click the link image(s) below to further research the concepts in this question (if desired).
Research Concepts:
BRCA 1 and 2
We update eBooks quarterly and Apps daily based on user feedback. Please tap flag to report any questions that need improvement.
Question 586:
A patient presents with metastatic but hormonally sensitive prostate cancer. What is the expected median benefit in the survival of a combination of 6 cycles of docetaxel with androgen deprivation therapy compared to treating with androgen deprivation therapy alone?
Choices: 1. No significant survival benefit as docetaxel can be added later 2. 3 to 4 months survival benefit 3. 7 to 8 months survival benefit 4. 13 to 14 months benefit
Answer: 4 - 13 to 14 months benefit Explanations: The benefit of docetaxel was most apparent in patients with higher volume disease, where combination therapy provided 17 months of increased survival. The magnitude of the benefit from the addition of docetaxel is clinically very significant. Chemohormonal therapy has become standard in previously untreated patients with metastatic but hormonally sensitive prostate cancer, especially with high volume or advanced disease. Adding docetaxel later did not provide the same survival benefit. Go to the next page if you knew the correct answer, or click the link image(s) below to further research the concepts in this question (if desired).
Research Concepts: Docetaxel
We update eBooks quarterly and Apps daily based on user feedback. Please tap flag to report any questions that need improvement.
Question 587:
An 80-year-old man presents for a 2week history of decreased urine output and fatigue. He reports taking acetaminophen for lower back pain that started 10 days ago. His physical examination reveals a tender thoracic spine and pedal edema. Laboratory studies show a hemoglobin concentration of 10 g/dL, serum creatinine of 1.9 mg/dL, and serum calcium of 11.0 mg/dL. Which of the following is the most likely cause of this patient's symptoms?
Choices: 1. 2. 3. 4.
Anti-Smith antibodies Immunoglobulin light chain deposits Anti-double-stranded DNA antibodies Hypersensitivity reaction
Answer: 2 - Immunoglobulin light chain deposits Explanations: Back pain, kidney damage, and increased serum calcium of this patient favor the diagnosis of multiple myeloma. Myeloma cells can produce immunoglobulins that can deposit light chains in the kidneys. These deposits can cause kidney damage and urinary problems. Immunoglobulin light chains are detected in the serum and urine of patients with multiple myeloma. Immunoglobin light chain monitoring is important in assessing for disease progression. Go to the next page if you knew the correct answer, or click the link image(s) below to further research the concepts in this question (if desired).
Research Concepts: Plasmacytoma
We update eBooks quarterly and Apps daily based on user feedback. Please tap flag to report any questions that need improvement.
Question 588:
A patient presents with unilateral hearing loss. The symptoms were noted two months before the consultation and had been progressively getting worse. This is accompanied by tinnitus and a sense of imbalance. The patient has no known medical comorbidities. On examination, the patient is awake, alert, able to follow commands. The cranial nerve examination is within normal limits, except for sensorineural hearing loss on the left. The motor and sensory examinations were also within normal limits. The imaging studies reveal a small, wellcircumscribed enhancing lesion on the left vestibular nerve. What is the expected hearing preservation rate with fractionated stereotactic radiation in this patient if he had an excellent hearing before treatment?
Choices: 1. 2. 3. 4.
Approximately Approximately Approximately Approximately
1% 25% 33% 75%
Answer: 4 - Approximately 75% Explanations: Most patients have hearing preservation of approximately 75% with fractionated stereotactic radiation treatment of a small acoustic neuroma. Hearing preservation is more likely if fractionated therapy stereotactic radiotherapy is used instead of a single dose. Low dose therapy likely prevents further growth of the tumor by causing obliterative endarteritis of the vessels that supply the tumor. Hydrocephalus is a potential complication of fractionated stereotactic radiation. Go to the next page if you knew the correct answer, or click the link image(s) below to further research the concepts in this question (if desired).
Research Concepts: Acoustic Neuroma
We update eBooks quarterly and Apps daily based on user feedback. Please tap flag to report any questions that need improvement.
Question 589:
A 55-year-old African American man presents to the clinic for a routine visit. He has a 40 packyear smoking history. He has no other comorbidities and is currently is in good health. He continues to smoke and has no intention to quit despite extensive counseling about the hazards of smoking. What is the most appropriate recommendation for lung cancer screening for this patient?
Choices: 1. 2. 3. 4.
Annual low dose chest CT starting now Low dose chest CT every 5 years Chest CT now and every 10 years Low dose chest CT starting from the age of 60 years
Answer: 1 - Annual low dose chest CT starting now Explanations: As per USPSTF, adults age 55-80 years who are currently smoking or have 30 pack-year history of smoking or have quit in the last 15 years are recommended to undergo annual low dose lung computed tomography (LDCT) as screening for lung cancer. The decision to screen should be an informed/ shared decision-making process, which includes information about the potential benefits, harms, and limitations of screening with LDCT and have access to a superiorquality lung carcinoma screening and treatment center. Screening is discontinued once the person has not smoked for 15 years or has limited life expectancy or inability or unwillingness to perform curative lung surgery due to health problems. ACS (American Cancer Society) also endorses similar recommendations as USPSTF. Go to the next page if you knew the correct answer, or click the link image(s) below to further research the concepts in this question (if desired).
Research Concepts: Cancer Screening
We update eBooks quarterly and Apps daily based on user feedback. Please tap flag to report any questions that need improvement.
Question 590:
A 52-year-old woman undergoes a core needle biopsy for calcifications found on a screening mammogram. The findings on the pathology slide demonstrate cytonuclear and architectural atypia with partial involvement of the ducts and small size of 2 mm and ER positivity. What is the next best step in the management of this patient?
Choices: 1. Clinical exams every 6-12 months and mammography annually 2. Routine screening 3. Excisional biopsy 4. Mastectomy
Answer: 3 - Excisional biopsy Explanations: Atypical ductal hyperplasia (ADH) found on core needle biopsy will require additional tissue - you need to get an excisional biopsy for more tissue to rule out carcinoma in situ or invasive carcinoma of the breast. ADH found on excisional biopsy, however, does NOT require additional surgery, and you are surgically complete. But it will require additional screening, including clinical exams and mammography yearly. Given the increased risk of future breast cancer in women diagnosed with ADH, follow-up should consist of clinical examinations every 6 to 12 months and annual mammography, in addition to additional tissue biopsy. ADH is a marker of increased risk of breast cancer and not a cancerous or precancerous lesion. Therefore mastectomy is not indicated if only ADH is found. Go to the next page if you knew the correct answer, or click the link image(s) below to further research the concepts in this question (if desired).
Research Concepts: Atypical Ductal Hyperplasia
We update eBooks quarterly and Apps daily based on user feedback. Please tap flag to report any questions that need improvement.
Question 591:
What malignancies to a majority of patients with Cowden disease go on to develop?
Choices: 1. 2. 3. 4.
Spine, ribs, and pelvis Skin, eyes, and brain Thyroid, endometrium, and breast Leukemia, lymphoma, and soft tissue
Answer: 3 - Thyroid, endometrium, and breast Explanations: Uterine leiomyomas and ovarian cysts can occur in females and may produce menstrual abnormalities along with a possible 20% to 30% increased risk of endometrial carcinoma. Females with Cowden disease are also at a much higher risk of breast carcinoma, with about 85% of females with Cowden syndrome developing breast carcinoma at some time in their life. Interestingly, breast carcinoma has also been reported in men. Benign fibrocystic disease and fibroadenomas are also commonly seen in females. The majority of patients affected with the disease go on to develop a malignant neoplasm of the thyroid, endometrium, or breast. Thyroid abnormalities such as goiter, thyroglossal duct cysts, and adenomas are also commonly seen. Thyroid carcinoma risk is also increased by up to 30%. More than 85% of patients may have gastrointestinal involvement with hamartomatous polyps. The risk of colon cancer in patients is slightly elevated. Go to the next page if you knew the correct answer, or click the link image(s) below to further research the concepts in this question (if desired).
Research Concepts: Cowden Disease
We update eBooks quarterly and Apps daily based on user feedback. Please tap flag to report any questions that need improvement.
Question 592:
A 65-year-old presents with a slowly growing mass on the left thigh. The mass is approximately 9 x 4 cm, with hard growth on the left femur which is slightly mobile within the muscles. The CT scan shows the presence of a large mass with patchy areas of necrosis. He is sent to the musculoskeletal team for a biopsy and the pathology reads monophasic synovial sarcoma. Staging CT scans are negative for malignancy. Which of the following is the best next step in his management?
Choices: 1. 2. 3. 4.
Preoperative radiation followed by surgical resection Neoadjuvant chemotherapy followed by surgical resection Preoperative radiotherapy followed by chemotherapy Surgical resection followed by adjuvant chemotherapy.
Answer: 1 - Preoperative radiation followed by surgical resection
Explanations: Localized synovial sarcoma in adults is best treated with preoperative radiation followed by surgical resection. Adjuvant chemotherapy is controversial in adults. It is more commonly used in the pediatric age group and in those who belong to adolescent young adults. Surgical resection without perioperative radiotherapy is associated with poor outcomes, especially for localized synovial sarcoma. Substituting chemotherapy for radiotherapy does not improve survival. In patients with synovial sarcoma presenting with a localized disease of the extremity, not performing surgical resection leads to suboptimal outcomes. Go to the next page if you knew the correct answer, or click the link image(s) below to further research the concepts in this question (if desired).
Research Concepts: Synovial Cell Sarcoma We update eBooks quarterly and Apps daily based on user feedback. Please tap flag to report any questions that need improvement.
Question 593:
A 66-year-old male patient presents to the outpatient department with swelling of the left cheek. CT scan revealed a solid mass measuring 3.5 x 2.5 cm located in the left parotid gland with a cystic component. His past medical history is unremarkable. Fine-needle aspiration cytology (FNAC) showed oncocytic epithelial cells without atypia admixed with polymorphous lymphocytes and cellular debris. His blood pressure is 120/80 mmHg, the temperature is afebrile, and the pulse is 89 bpm. What is the most appropriate management for this patient?
Choices: 1. 2. 3. 4.
Radiotherapy followed by tumor enucleation No treatment is required, a close follow-up is sufficient Left parotidectomy with lymph node dissection Left parotidectomy
Answer: 4 - Left parotidectomy Explanations: According to the FNAC, this patient presents with a warthin tumor of the parotid gland. In the case of warthin tumor, smears characteristically show oncocytic epithelial cells without atypia admixed with polymorphous lymphocytes and cellular debris, as it is the case for this patient. The confirmation of diagnosis with certainty relies on the histopathological examination of the surgical specimen. The optimal treatment modality of the warthin tumor relies on surgical removal, which can be easily realized due to the superficial location of the tumor. Warthin tumor is a benign salivary gland neoplasm, and the likelihood of malignant transformation is only 0.3%, so left parotidectomy is the best treatment modality for this patient. Lymph node dissection is not required in this case. Some surgeons prefer the superficial parotidectomy to avoid the rupture of the tumor capsule, whereas others chose local resection with the surrounding tissue. No neoadjuvant treatment is required. Go to the next page if you knew the correct answer, or click the link image(s) below to further research the concepts in this question (if desired).
Research Concepts: Warthin Tumor
We update eBooks quarterly and Apps daily based on user feedback. Please tap flag to report any questions that need improvement.
Question 594:
A 36-year-old female patient with a history of metastatic melanoma presents to the emergency department with 10 episodes of diarrhea per day, blood in the stool, and abdominal pain. She was started on pembrolizumab 6 months ago and gets it every 3 weeks. The most recent treatment with pembrolizumab was 1 week ago. The patient denied any fever or recent antibiotic use. Colonoscopy with biopsy showed marked active colitis. What is a common toxicity associated with treatment with these agents?
Choices: 1. 2. 3. 4.
CMV colitis Giardiasis Clostridioides difficile diarrhea Immune-mediated colitis
Answer: 4 - Immune-mediated colitis Explanations: Immune-mediated colitis is a common side effect of the class of immune checkpoint inhibitors (ICIs). GI toxicities include colitis, graded based on the frequency of stool and output compared to baseline. For example, this patient has grade 3 colitis when stool frequency is seven or more stools per day over baseline. Permanently discontinue ICIs and start steroids such as IV methylprednisolone (2 mg/kg/day). If there is no response in 2 days, continue steroids and consider adding infliximab 5 to 10 mg/kg. If infliximabrefractory consider vedolizumab. Go to the next page if you knew the correct answer, or click the link image(s) below to further research the concepts in this question (if desired).
Research Concepts: Pembrolizumab
We update eBooks quarterly and Apps daily based on user feedback. Please tap flag to report any questions that need improvement.
Question 595:
A 65-year-old woman presents with features of newfound back pain and abdominal pain. Complete blood work revealed pancytopenia, hypercalcemia, and renal failure. The patient underwent a bone marrow biopsy, which revealed an 80% monoclonal plasma cell population. Serum protein electrophoresis showed a serum M-protein of 5 grams. Which of the following treatment options is most appropriate in this setting?
Choices: 1. Allogeneic stem cell transplantation 2. Bendamustine and rituximab 3. Ibrutinib 4. Induction chemotherapy followed by an early autologous stem cell transplant
Answer: 4 - Induction chemotherapy followed by an early autologous stem cell transplant
Explanations: This patient demonstrates evidence of multiple myeloma as exhibited by end-organ damage and a monoclonal plasma cell population over 10%. While these patients do benefit from more durable remissions post-transplantation, allogeneic stem cell transplantation is not the conventional first-line option for these patients beyond the experimental setting. While this is a treatment option in a variety of indolent and aggressive non-Hodgkin lymphomas, it is not the appropriate induction treatment option in multiple myeloma. Ibrutinib, a relatively newly approved Bruton kinase inhibitor in managing various non-Hodgkin lymphomas, is not used in multiple myeloma. This patient's multiple myeloma is best managed with induction chemotherapy, followed by early autologous stem cell transplant. Survival analysis in such patients has been correlated with more durable versus later transplantation and recommended once patients achieve very good partial remission post-primary induction. Go to the next page if you knew the correct answer, or click the link image(s) below to further research the concepts in this question (if desired).
Research Concepts: Multiple Myeloma
We update eBooks quarterly and Apps daily based on user feedback. Please tap flag to report any questions that need improvement.
Question 596:
A 65-year-old man presents to the clinic with recent-onset thickening of the fingertips with itching. This was followed by the progression of the thickening of the skin to the palms and soles within the last 2 months. A dermatological examination reveals palmoplantar hyperkeratosis with hyperkeratosis involving the nasal tip and the helix of the ears associated with a reddish violaceous hue. The nails reveal longitudinal streaks, onycholysis, and onychomadesis. Which of the following clinical findings best differentiates this condition from psoriasis?
Choices: 1. 2. 3. 4.
Hyperkeratosis of the helix of ears Itching in fingertips Longitudinal streaks on nails Onychomadesis
Answer: 1 - Hyperkeratosis of the helix of ears Explanations: Bazex syndrome (acrokeratosis paraneoplastica) is a rare psoriasiform dermatosis associated with squamous cell malignancies of the upper aerodigestive tract. It presents with hyperkeratosis of the fingertips, nasal tip, helices of ears, palms, soles, and later the forehead, trunk cheeks, elbows, and knees. The involvement of hyperkeratosis in the nasal tip and the helix of the ears should raise a strong suspicion of Bazex syndrome. Intensive violaceous erythema, erosions, and yellowish crusts may be present over the lesions. Some cases are associated with vesiculation resembling eczema. It is usually asymptomatic but may some times be intensely itchy. Go to the next page if you knew the correct answer, or click the link image(s) below to further research the concepts in this question (if desired).
Research Concepts: Acrokeratosis Paraneoplastica
We update eBooks quarterly and Apps daily based on user feedback. Please tap flag to report any questions that need improvement.
Question 597:
A 49-year-old male presented with right iliac pain which was diagnosed with acute appendicitis. He underwent appendectomy operation without complications. Two weeks later, the pathology report of the tissue removed showed a carcinoid tumor more than 2 cm in diameter. What is the next preferred step in management?
Choices: 1. 2. 3. 4.
No more management required Right hemicolectomy Peptide receptor radiotherapy (PRRT) Octreotide
Answer: 2 - Right hemicolectomy Explanations: Surgical resection is the potentially curative treatment for localized and locally advanced neuroendocrine tumor (NET)s. Tumors localized in or near the ileocecal valve and small bowel tumors greater than 2 cm, right hemicolectomy, and small intestinal resection usually performed respectively. For midgut NETs, surgery still the preferred management modality even in patients with small, asymptomatic tumors detected incidentally. When a surgical approach to the tumor is not possible due to tumor size, a debulking surgery is recommended. Appendiceal tumors less than 1 cm, simple appendectomy is usually curative, but for small intestinal NETs, less than 1 cm typical small intestinal resection is required. Systemic therapy aims to control symptoms caused by hormonal secretion and decrease tumor proliferation and progression, while surgery is the potentially curative treatment modality of choice for NETs. Go to the next page if you knew the correct answer, or click the link image(s) below to further research the concepts in this question (if desired).
Research Concepts: APUDoma
We update eBooks quarterly and Apps daily based on user feedback. Please tap flag to report any questions that need improvement.
Question 598:
A 75-year-old female of Indian origin, with a longstanding history of gallstones, presents to the office with complaints of right upper quadrant pain for the past one month. She states that she has lost 10 lb of weight over the last two months and has lost her appetite. Her medical history is positive for hypertension and hyperlipidemia. On examination, the patient appears to be cachectic. There is a pear-shaped irregular hard mass palpable in the right upper quadrant just below the liver edge. Which of the following is the best imaging test of choice for this patient?
Choices: 1. 2. 3. 4.
Ultrasound of the abdomen Endoscopic retrograde cholangiopancreatography CT of the abdomen and pelvis with IV contrast HIDA scan
Answer: 3 - CT of the abdomen and pelvis with IV contrast
Explanations: This patient has a high likelihood of gallbladder cancer. CT of the abdomen and pelvis with IV contrast is the imaging test of choice in cases suspected for gallbladder cancer. HIDA scan can diagnose cholecystitis; however, it is a poor test of choice in case of gallbladder cancer. Ultrasound of the abdomen may be used as an initial test in cases of right upper quadrant pain; however, it is not an imaging test of choice in cases suspected for gallbladder cancer. Go to the next page if you knew the correct answer, or click the link image(s) below to further research the concepts in this question (if desired).
Research Concepts: Gallbladder Imaging
We update eBooks quarterly and Apps daily based on user feedback. Please tap flag to report any questions that need improvement.
Question 599:
A 65-year-old man presents with progressive exertional dyspnea and generalized fatigue. Eight years ago, he was diagnosed with non-small cell carcinoma of the left lung and underwent definitive chemoradiation therapy. The patient has a 25-pack-year history of smoking and has never traveled out of the United States. His blood pressure is 122/79 mmHg, and his pulse is 105/min. Examination demonstrates a cardiac friction rub, clear lungs on auscultation, distended abdomen, and moderate pitting edema of the lower extremities. An electrocardiogram shows a low-voltage QRS complex, and an echocardiogram shows a left ventricular ejection fraction (LVEF) of 60% with enlarged atria. Jugular venous pressure is estimated at 18 cmH2O with prominent X and Y descent on tracing. What is the most likely cause of this patient's presentation?
Choices: 1. 2. 3. 4.
Left-sided heart failure Superior vena cava syndrome Tuberculosis Radiotherapy eight years ago
Answer: 4 - Radiotherapy eight years ago Explanations: This patient has a typical presentation of constrictive pericarditis due to pre and post-operative high-dose radiotherapy. Constrictive pericarditis etiologies also include viral pericarditis, heart surgery, tuberculous pericarditis, and idiopathetic. Clinically this patient has right-sided heart failure due to constrictive pericarditis. Typically rightsided heart failure presents with elevated jugular venous pressure and lower extremity edema with a distended abdomen due to hepatosplenomegaly and ascites. Electrocardiogram in constrictive pericarditis is usually nonspecific, such as low voltage QRS complex, tachycardia, or atrial fibrillation. An echocardiogram may show calcified thickening of the pericardium and enlargement of the atria. Jugular venous pressure tracing shows prominent X and Y descent. This patient has right-sided heart failure due to constrictive pericarditis. His lungs are clear on auscultation, which rules out the possibility of left-sided heart failure. The patient has never traveled outside the United States, which makes tuberculous pericarditis unlikely. Superior vena cava syndrome (SVCS) usually presents with facial plethora and prominent facial and neck veins, mostly due to compression by an apical lung tumor. This patient has right-sided heart failure signs such as lower extremity edema and abdominal distention, which are usually absent in SVCS.
Go to the next page if you knew the correct answer, or click the link image(s) below to further research the concepts in this question (if desired).
Research Concepts: Radiation Therapy Induced Cardiac Toxicity
We update eBooks quarterly and Apps daily based on user feedback. Please tap flag to report any questions that need improvement.
Question 600:
A 54-year-old woman with a history of sarcoma of the mouth undergoes radiation therapy. Six months later, she presented to the emergency department with complaints of pain, drainage, and swelling of the jaw. After evaluation, she is given IV penicillin. The biopsy report states that there is no persistent malignancy now. Which of the following is the next best step in the management of this patient?
Choices: 1. 2. 3. 4.
Supraphysiologic supplementation of vitamin C MRI of the head and neck Biopsy of maxilla Hyperbaric oxygen therapy
Answer: 4 - Hyperbaric oxygen therapy Explanations: The patient has most likely suffered from osteoradionecrosis, a common complication following radiation to the head and neck, commonly occurring in the jaw. Hyperbaric oxygen therapy and IV antibiotics are the mainstays of treatment for osteoradionecrosis, but persistent malignancy must be first ruled out before initiating hyperbaric oxygen therapy. The addition of hyperbaric oxygen treatment improves the tissue response to surgical wounding and improves the patient's chances of healing after surgical resection or reconstruction of the mandible. Perioperative prophylactic hyperbaric oxygen treatment is recommended for patients with previous head and neck irradiation undergoing dental extractions or complete exodontia procedures again to increase the blood flow, oxygenation, and penetration of bone by antibiotics. Studies have shown a reduction from 30% to 5% incidence of osteoradionecrosis using prophylactic hyperbaric oxygen treatment. Go to the next page if you knew the correct answer, or click the link image(s) below to further research the concepts in this question (if desired).
Research Concepts: Osteoradionecrosis
We update eBooks quarterly and Apps daily based on user feedback. Please tap flag to report any questions that need improvement.
Section 7 Question 601:
A 65-year old male presents with numerous red serpiginous patches on his trunk and upper extremities. The lesions have been present for over a year and are growing in number. He reports intense pruritus due to the lesions which is his most pressing concern. He reports dry scaly skin with intermittent patches in the past. He was seen by dermatology for these lesions 10 years ago but the biopsy was nondiagnostic. Given the most likely diagnosis, which of the following is true about the condition of the patient?
Choices: 1. Tumor cells are typically CD8+ 2. It is a type of T-cell lymphoma, typically with a long indolent course 3. The cause is an atypical fungus, commonly carried by farm animals 4. It is a type of Hodgkin's lymphoma, most common in elderly males
Answer: 2 - It is a type of T-cell lymphoma, typically with a long indolent course
Explanations: Mycosis fungoides (MF) is a cutaneous T-cell lymphoma (CTCL) and typically has a long indolent course. It accounts for approximately 50% of all primary cutaneous lymphomas. MF onset is typically in the sixth or seventh decade, but it can occur in younger patients. Mycosis fungoides can progress through a patch, plaque, and tumor stages. MF's typical phenotype is CD3+, CD4+, CD8- mature Tlymphocytes. Go to the next page if you knew the correct answer, or click the link image(s) below to further research the concepts in this question (if desired).
Research Concepts: Mycosis Fungoides
We update eBooks quarterly and Apps daily based on user feedback. Please tap flag to report any questions that need improvement.
Question 602:
A 2-year-old girl was operated for a supratentorial frontal paraventricular tumor in 2015 with subtotal resection. The final histopathological diagnosis was a supratentorial primitive neuroectodermal tumor. She received adjuvant chemotherapy after the surgery. The typical milestones were achieved, and now she is seven years old. Yearly brain magnetic resonance imaging has not shown progression until the one performed last month. She now has tumor growth and is scheduled for a reoperation this week. Which is the most probable histopathology that will be reported?
Choices: 1. 2. 3. 4.
Medulloblastoma Choroid plexus carcinoma Recurrent primitive neuroectodermal tumor High-grade glioma
Answer: 4 - High-grade glioma Explanations: After 2016, a supratentorial lesion has an increased probability of being a high-grade glioma or ependymoma if they were previously called a supratentorial primitive neuroectodermal tumor. DNA methylation profile will define the molecular diagnosis. Their molecular profile defines embryonal tumors in addition to the histological findings. A study investigating previously histological supratentorial primitive neuroectodermal tumors, which were reanalyzed using molecular diagnosis, found that in 71% of them, the diagnosis changed. In 2016, the World Health Organization revised the classification of central nervous system tumors using molecular parameters. In this classification, some tumors previously recognized in the 2007 classification had now been renamed or eliminated. The primitive neuroectodermal tumor is no longer known as such, and those tumors are now under the classification of embryonal tumors. Go to the next page if you knew the correct answer, or click the link image(s) below to further research the concepts in this question (if desired).
Research Concepts: Primitive Neuroectodermal Tumor
We update eBooks quarterly and Apps daily based on user feedback. Please tap flag to report any questions that need improvement.
Question 603:
What are the risks of monoclonal gammopathy of undetermined significance (MGUS) and smoldering multiple myeloma (SMM) progressing to multiple myeloma (MM) per year?
Choices: 1. 2. 3. 4.
1 and 10 percent 5 and 15 percent 10 and 5 percent 15 and 10 percent
Answer: 1 - 1 and 10 percent Explanations: MGUS has a progression risk to MM about 1 percent per year, whereas SMM has a much higher risk of 10 percent per year. The diagnosis of MM requires the presence of 1 or more myeloma defining events (e.g., end-organ damage, clonal plasma cell greater than 60 percent on bone marrow biopsy, more than 1 focal lesion on MRI) in addition to 10 or more percent of clonal plasmas cells on the bone marrow or biopsy-proven plasmacytoma. . Diagnosis for Non-IgM MGUS requires clonal plasma cells less than 10 percent, serum monoclonal protein less than 3 g/dL, and absence of myeloma defining events. Diagnosis for SMM requires clonal plasma cells 10-60% and/or serum monoclonal protein 3 g/dL or more, or urinary monoclonal protein 500 mg or more per 24 hours, and absence of myeloma defining events. Go to the next page if you knew the correct answer, or click the link image(s) below to further research the concepts in this question (if desired).
Research Concepts: Monoclonal Gammopathy Of Undetermined Significance
We update eBooks quarterly and Apps daily based on user feedback. Please tap flag to report any questions that need improvement.
Question 604:
A 10-year-old boy with intermediaterisk acute lymphoblastic leukemia is started on augmented BFM protocol consisting of vincristine, L-asparaginase, daunomycin, cytosine arabinoside, and methotrexate. He also has a history of chronic kidney disease on maintenance hemodialysis. Despite the best anti-hyperkalemic measures, his serum potassium remains 7.8 mEq/L. Which of the following is the most appropriate management strategy for the patient's hyperkalemia?
Choices: 1. Hemodialysis with the cessation of vincristine and dose modification of methotrexate 2. Hemodialysis with dose modification of vincristine and methotrexate 3. Hemodialysis with dose modification of methotrexate 4. Hemodialysis with the addition of febuxostat
Answer: 3 - Hemodialysis with dose modification of methotrexate
Explanations: Vinca alkaloids do not have significant renal excretion. No dose adjustment is required for vinca alkaloids in renal impairment. Vinorelbine has a hepatic excretion amounting to 50% and requires a dose modification of 50% in those undergoing hemodialysis due to an increased risk of toxicity. Despite there being no data available for the other agents, it has been postulated that the same recommendation holds true with respect to dose modification for patients undergoing hemodialysis. The use of vinca alkaloids in peritoneal dialysis has not been studied. Other indications for dose modification of vincristine include neuropathy and severe hepatic dysfunction. Dose modifications for methotrexate are required in patients with renal failure, effusions, and in the setting of acute cerebellar dysfunction (especially when intrathecal methotrexate is being used for intrathecal chemoprophylaxis). Go to the next page if you knew the correct answer, or click the link image(s) below to further research the concepts in this question (if desired).
Research Concepts: Vinca Alkaloid Toxicity
We update eBooks quarterly and Apps daily based on user feedback. Please tap flag to report any questions that need improvement.
Question 605:
A 66-year-old man presents with a sixmonth history of fecal urgency and intermittent frank hematochezia. He reports generally feeling fatigued and having a reduced appetite. Digital rectal examination reveals a 4.5 cm friable mass in the anal canal, a biopsy of which reveals adenocarcinoma. PET/CT imaging reveals bilateral inguinal node involvement. Which of the following best describes the chances of nodal control by radiotherapy and chemoradiotherapy in this patient?
Choices: 1. 50% with radiotherapy alone and chemoradiotherapy 2. 60% with radiotherapy alone and from combined chemoradiotherapy 3. 70% with radiotherapy alone and chemoradiotherapy 4. 80% with radiotherapy alone and chemoradiotherapy
70% with no additional benefit 90% with 95% with
Answer: 3 - 70% with radiotherapy alone and 90% with chemoradiotherapy
Explanations: Lymph node positivity portends a poor prognosis in anal cancer and correlates with worse survival and colostomy rates. Radiation alone controls 70% of involved inguinal nodes, whereas chemoradiation controls 90% of involved inguinal nodes. Therefore, it is important to include mesenteric, iliac, and inguinal lymph nodes within the radiation fields. Despite the effectiveness of chemoradiation as definitive management, reports exist of locoregional failure rates of 10-30%. Progressive or recurrent disease after definitive chemoradiation requires abdominoperineal resection for salvage. Go to the next page if you knew the correct answer, or click the link image(s) below to further research the concepts in this question (if desired).
Research Concepts: Radiation Therapy For Anal Cancer
We update eBooks quarterly and Apps daily based on user feedback. Please tap flag to report any questions that need improvement.
Question 606:
A 50-year-old woman comes because of progressive shortness of breath over the last four months. She also complains of chest pain. Past medical history is significant for hypertension for ten years and takes the antihypertensive drug. She was diagnosed with non-Hodgkin lymphoma five years ago, and she was treated with chemotherapy and radiotherapy. Her blood pressure is 120/70mmhg. Physical examination reveals jugular venous distension (3cm). On cardiac auscultation, the third heart sound is heard. Echocardiogram shows left ventricular enlargement and left ventricular ejection fraction of 30%. What is the cause of her symptoms?
Choices: 1. 2. 3. 4.
Heart Heart Heart Heart
failure failure failure failure
due due due due
to to to to
radiotherapy hypertension chemotherapy rheumatic heart disease
Answer: 3 - Heart failure due to chemotherapy Explanations: The most commonly used chemotherapy in non-Hodgkin lymphoma is the R-CHOP regimen (rituximab, cyclophosphamide, doxorubicin, vincristine, prednisolone). The primary mechanism of action of doxorubicin involves the drug’s ability to intercalate within DNA base pairs, causing breakage of DNA strands and inhibition of both DNA and RNA synthesis. Heart failure is the common side effect caused by doxorubicin and radiotherapy. Doxorubicin is also associated with significant cardiac toxicity, which limits the long-term use of the drug. The mechanism of action of doxorubicin-induced cardiac toxicity differs from the drug’s antitumor mechanism. It involves increased oxidative stress, down-regulation of cardiac-specific genes, and induction of cardiac myocyte apoptosis by doxorubicin. Adverse reactions are common after doxorubicin administration and may include fatigue, alopecia, nausea and vomiting, and oral sores. Doxorubicin-induced heart failure causes dilated cardiomyopathy with left ventricular enlargement and decreases left ventricular ejection fraction. Doxorubicininduced irreversible cardiomyopathy occurs within a few months of the end of treatment but has also been reported to occur up to twenty years after treatment termination. Radiotherapy induced heart failure causes fibrosis of the myocardium and preserved left ventricular ejection fraction. Heart failure due to rheumatic heart disease as there is no history given.
Go to the next page if you knew the correct answer, or click the link image(s) below to further research the concepts in this question (if desired).
Research Concepts: Non-Hodgkin Lymphoma
We update eBooks quarterly and Apps daily based on user feedback. Please tap flag to report any questions that need improvement.
Question 607:
A 60-year-old man is being evaluated for progressive vision and mental status changes over the past six months. Head CT shows a single mass suspicious for cancer. The mass is removed, and genetic analysis is performed. It is discovered that its DNA and histones proteins both underwent epigenetic changes, causing increased expression of multiple oncogenes. Which of the following best describes the most likely changes to the charge of the DNA and histone proteins associated with the oncogenes?
Choices: 1. DNA and histones both became more positive 2. DNA and histones both became more negative 3. DNA became more positive and histones became more negative 4. DNA became more negative and histones became more positive
Answer: 3 - DNA became more positive and histones became more negative
Explanations: DNA is negatively charged, while histones are positively charged because of high lysine and arginine content. DNA and histones interact most when they are of opposite charge. When the charge difference decreases, they become less associated, resulting in the DNA being more accessible by transcriptional enzymes. The oncogenes are more highly expressed when they are transcribed more. This is the result of fewer interactions with histone proteins. DNA becoming more positive while the histones becoming more negative causes the charge differences to become least different. At the location of oncogenes, this would result in greater expression and cause an increased risk of developing a tumor. Go to the next page if you knew the correct answer, or click the link image(s) below to further research the concepts in this question (if desired).
Research Concepts: Genetics, DNA Packaging
We update eBooks quarterly and Apps daily based on user feedback. Please tap flag to report any questions that need improvement.
Question 608:
A 33-year-old female presents to a clinic with a new breast cancer diagnosis. She is premenopausal. She has been diagnosed with invasive ductal carcinoma by core needle biopsy. The maximum size of the tumor on the core biopsy is 1 cm, and the mammogram suggests a similar size. She has no palpable axillary nodes. She undergoes sentinel lymph node biopsy and lumpectomy. The lumpectomy specimen shows adequate margins. Three sentinel nodes were removed, and one was involved with a macrometastasis that did not traverse the capsule of the node. Estrogen and progesterone receptors are positive. Which of the following would be the best choice for her?
Choices: 1. Limited axillary node dissection and adjuvant chemotherapy 2. Adjuvant chemoradiation therapy and treatment with tamoxifen 3. Breast radiation therapy and aromatase inhibitor therapy 4. Axillary radiation and adjuvant chemotherapy
Answer: 2 - Adjuvant chemoradiation therapy and treatment with tamoxifen
Explanations: The patient has early-stage breast cancer (T1, N1.) Since she has axillary involvement, she is a candidate for adjuvant chemotherapy. The Z0011 trial supports that patients with early-stage breast cancer and minimal axillary burden do not benefit from complete axillary dissection. A ten-year follow-up trial demonstrated no increased survival in patients such as these undergoing axillary dissection after positive sentinel node biopsy. The cutoff for recommending axillary radiation is three positive lymph nodes (N2 disease), so there is no need for axillary radiation in this case. As she is premenopausal, she cannot be given aromatase inhibitors. Go to the next page if you knew the correct answer, or click the link image(s) below to further research the concepts in this question (if desired).
Research Concepts: Axillary Sentinel Lymph Node Biopsy
We update eBooks quarterly and Apps daily based on user feedback. Please tap flag to report any questions that need improvement.
Question 609:
A 45-year-old male patient has been operated four times for a suprasellar and intraventricular tumor and received fractionated radiotherapy after the last surgery two years ago. He now presents to the emergency department with headaches, nausea, and vomiting. An MRI of the brain shows a large cyst extending from the suprasellar area into the third ventricle and up to the lateral ventricles through the foramen of Monro. There is no evidence of an enhancing tumor. An Ommaya reservoir is placed to aspirate the cyst periodically. What could be the most likely underlying diagnosis?
Choices: 1. 2. 3. 4.
Craniopharyngioma Suprasellar meningioma Trigeminal schwannoma Chordoma
Answer: 1 - Craniopharyngioma Explanations: Craniopharyngiomas are rare, benign tumors of the central nervous system. Craniopharyngiomas are epithelial tumors that typically arise in the suprasellar area of the brain; extending to involve the hypothalamus, optic chiasm, cranial nerves, third ventricle, and major blood vessels. Cystic tumors refractory to treatment are drained periodically using an Ommaya reservoir. Serial aspiration of fluid from these tumor cysts can help prevent symptoms and reduce the chance of obstructive hydrocephalus from compression of the third ventricle or foramen of Monro. Chordoma, meningioma, and schwannoma are usually solid tumors. Go to the next page if you knew the correct answer, or click the link image(s) below to further research the concepts in this question (if desired).
Research Concepts: Craniopharyngioma
We update eBooks quarterly and Apps daily based on user feedback. Please tap flag to report any questions that need improvement.
Question 610:
A 10-year-old boy undergoes magnetic resonance imaging (MRI) for an intracranial brain tumor. The tumor is situated within the posterior fossa, in the midline of the cerebellum. It has a large cystic component and avidly enhancing nodular mural. The nodule shows a high signal on T2 weighted imaging. He also has obstructive hydrocephalus. Histology shows low cellularity with long cellular processes. Which of the following roles does radiotherapy have in the treatment of these tumors?
Choices: 1. No role as these tumors is not sensitive to radiotherapy. 2. Not first-line as usually curative on surgical resection, however, it can be an additional treatment for unresectable components. 3. First-line treatment in order to avoid surgery in children. 4. Standard care is surgical resection followed by radiotherapy.
Answer: 2 - Not first-line as usually curative on surgical resection, however, it can be an additional treatment for unresectable components.
Explanations: Standard treatment of pilocytic astrocytoma is complete surgical resection alone, as these tumors are usually curative on resection. If some tumor remains due to the difficulty in resecting, then radiotherapy may have an additional role. Radiotherapy of the posterior fossa in this age group often caries neurological sequelae. Chemotherapy has no role in the management of pilocytic astrocytoma. Go to the next page if you knew the correct answer, or click the link image(s) below to further research the concepts in this question (if desired).
Research Concepts: Pilocytic Astrocytoma
We update eBooks quarterly and Apps daily based on user feedback. Please tap flag to report any questions that need improvement.
Question 611:
A 59-year-old woman with no significant past medical history (including no smoking history) presents to the emergency department after an episode of hemoptysis. She undergoes CT of the chest that demonstrates a right lower lobe mass measuring 3.2 x 2 cm, with spiculated margins, highly suspicious for primary lung malignancy. The CT images also picked up a left adrenal mass, 1.8 x 2 cm, suspicious for metastatic disease. She is admitted for expedited workup and undergeoes IRguided biopsy of right lung mass, with pathology confirming lung adenocarcinoma. Additional molecular studies for any targetable driver mutations are negative. Her performance status is excellent with no neurologic symptoms, and she is discharged home with an upcoming medical oncology appointment. Which of the following is the next best step in the management of this patient?
Choices: 1. 2. 3. 4.
Left adrenal mass biopsy FDG-PET/CT scan Thoracic surgery evaluation for right lobectomy Start systemic therapy using osimertinib
Answer: 2 - FDG-PET/CT scan Explanations: The patient needs to complete a staging workup and will need a PET/CT to assess for all sites of metastasis. This would help with proper tailoring of treatment for the patient. If proven to be oligometastatic (single adrenal metastasis), then based on the patient's performance status, she could be a candidate for definitive treatment with right lobectomy along with left adrenalectomy or radiation, followed by completion of 4-6 cycles of adjuvant chemotherapy. The left adrenal mass biopsy is not warranted if radiographic findings are pointing towards metastatic appearance, especially in the presence of a biopsyproven primary site. Osimertinib is currently approved for metastatic lung adenocarcinoma harboring EGFR mutations only. It was initially approved for T790M mutation, which is resistant to first and second-generation TKI therapies. Go to the next page if you knew the correct answer, or click the link image(s) below to further research the concepts in this question (if desired).
Research Concepts: Adrenal Metastasis
We update eBooks quarterly and Apps daily based on user feedback. Please tap flag to report any questions that need improvement.
Question 612:
A 54-year-old male receives chemotherapy for a refractory lymphoma and presents to the clinic for follow-up. He has struggled with nausea, vomiting, and fatigue since initiating therapy but is in good spirits. He reports noticing a strange sound while watching television that his wife did not notice, although he doesn’t seem concerned about this. His physical exam is notable for signs of mild dehydration and generalized lymphadenopathy. His most recent labs revealed leukopenia but were otherwise within normal limits. What is another common side effect of the most likely offending agent?
Choices: 1. 2. 3. 4.
Folate deficiency Nephrotoxicity Pulmonary fibrosis Skin hyperpigmentation
Answer: 2 - Nephrotoxicity Explanations: This patient presents with side effects consistent with cisplatin administration which may be utilized in refractory lymphoma cases. Tinnitus or an occasional decreased ability to hear normal conversation are indications of ototoxicity, which has been frequently observed. Abnormalities of audiometric testing are more common, and hearing loss can be unilateral or bilateral. Frequency and severity increase with repeated doses and may not be reversible, but they mainly occur in the 4,000 to 8,000 Hz range. As ototoxicity of cisplatin (cisplatinum) is cumulative, audiometric testing should be performed, if possible, prior to initiation of therapy and at regular intervals after that, particularly if the clinical symptoms of tinnitus or hearing impairment occur. Another common side effect of cisplatin is nephrotoxicity. It may also cause nausea and vomiting, myelosuppression, and neuropathy. Go to the next page if you knew the correct answer, or click the link image(s) below to further research the concepts in this question (if desired).
Research Concepts: Cisplatin
We update eBooks quarterly and Apps daily based on user feedback. Please tap flag to report any questions that need improvement.
Question 613:
A 30-year-old man with a history of metastatic rhabdomyosarcoma presents to the clinic for follow up. The patient has signs of the first relapse of his disease. The clinician initiates a regimen of temozolomide, vincristine, and irinotecan. One week later, the patient develops symptoms of obstipation and bilious vomiting, along with absent bowel sounds. An abdominal radiograph shows dilated bowel loops, and a diagnosis of paralytic ileus is considered. Which of the following is an appropriate firstline of management for this condition?
Choices: 1. Conservative, including nasogastric decompression, along with discontinuation of chemotherapy 2. Surgical along with the continuation of chemotherapy 3. Conservative, including nasogastric decompression, along with the continuation of chemotherapy 4. Surgical along with discontinuation of chemotherapy
Answer: 3 - Conservative, including nasogastric decompression, along with the continuation of chemotherapy
Explanations: Autonomic neuropathy due to vincristine may present with paralytic ileus, which may manifest with symptoms of complete bowel obstruction, or obstipation and absent bowel sounds. Anecdotal case reports describe the occurrence of adynamic ileus with vincristine and advise conservative management followed by surgery in case of no response to treatment. It does not usually warrant cessation of chemotherapy. Autonomic neuropathy with vinca alkaloids is caused by the interference with axoplasmic transport and is related directly to its underlying mechanism of action. The vinca alkaloids, which are cell cycle M phasespecific, are known to interfere with microtubular assembly due to their interaction with beta-tubulin. Vinca alkaloids show a poor ability to penetrate the blood-brain barrier, which explains their lack of central nervous system adverse effects, though the case of syndrome of inappropriate anti-diuretic hormone secretion (SIADH) is well known in the literature. Go to the next page if you knew the correct answer, or click the link image(s) below to further research the concepts in this question (if desired).
Research Concepts: Vinca Alkaloid Toxicity
We update eBooks quarterly and Apps daily based on user feedback. Please tap flag to report any questions that need improvement.
Question 614:
A 65-year-old male patient presents to the hospital with a 6-month history of skin flushing, nausea, diarrhea, and wheezing. The patient has a past medical history of hypertension and takes lisinopril. The patient’s blood pressure is 130/90 mmHg, pulse 80/min, respirations 16/min, and temperature 98.0 F (36.7 C). On pulmonary examination, wheezing is present bilaterally. Abdominal examination is significant for diffuse tenderness to palpation. The patient’s skin is erythematous and warm. The provider orders a 5-Hydroxyindoleacetic acid (5-HIAA) 24hour urine collection, which is elevated. What vitamin is likely to be deficient in this patient because of his condition?
Choices: 1. 2. 3. 4.
Niacin Riboflavin Folic acid Biotin
Answer: 1 - Niacin Explanations: Carcinoid tumors are neuroendocrine tumors most commonly seen in the gastrointestinal tract, which may cause a carcinoid syndrome of cutaneous flushing, diarrhea, wheezing, peripheral edema due to the production of serotonin. The syndrome is usually associated with liver metastasis from carcinoid tumors of the small bowel or primary carcinoid tumors of the lung or stomach. Patients may have tricuspid insufficiency or pulmonic stenosis. Diagnosis is by 24-hour urine collection for 5hydroxyindoleacetic acid (5-HIAA), a breakdown product of serotonin. Carcinoids are neuroendocrine tumors releasing serotonin. Additionally, carcinoid tumors can release many other substances like amylin, gastrin, glucagon, histamine, insulin, kinins, secretin, tachykinins, and substance P. Carcinoid tumors use tryptophan and niacin to produce serotonin. Tryptophan is normally required to produce niacin in the body. Therefore, patients with carcinoid syndrome are at risk for developing a niacin deficiency. Go to the next page if you knew the correct answer, or click the link image(s) below to further research the concepts in this question (if desired).
Research Concepts: Carcinoid Syndrome
We update eBooks quarterly and Apps daily based on user feedback. Please tap flag to report any questions that need improvement.
Question 615:
A 68-year-old woman presents to the clinic for evaluation of severe abdominal pain with associated abdominal distention. She was diagnosed with celiac disease at a young age and has had intermittent issues with diarrhea for many years due to refractory disease. However, she notes that for the past year, her symptoms have been well controlled with abstinence from gluten consumption. Most recently, however, a 3-day history of worsening abdominal distention and the development of nausea and vomiting are present. The patient undergoes a CT of the abdomen and pelvis, showing small bowel obstruction likely secondary to a possible proximal duodenal mass. The patient is admitted for further management and evaluation. Biopsy with subsequent pathology review shows T cells with monoclonal arrangements. What is the most likely diagnosis?
Choices: 1. Enteropathy associated T-cell lymphoma 2. Chronic enteropathy B-type non-Hodgkin lymphoma 3. Reactive mucosal hyperplasia secondary to acute inflammation 4. Hodgkin lymphoma and acute on chronic metaplasia
Answer: 1 - Enteropathy associated T-cell lymphoma Explanations: Patients with type 2 refractory celiac disease are at increased risk of developing malignant changes in the small bowel. Although rare, the changes noted on biopsy are most consistent with enteropathy-associated T-cell lymphoma (EATCL) of the small bowel, which is generally seen in patients with refractory celiac disease. The diagnosis of EATCL is usually preceded by an acute change in condition in a patient that was previously well controlled with standard diet modifications. Longstanding celiac disease is an independent risk factor for the development of high-grade lymphomas of the small bowel. Primarily noted to consist of nonHodgkin lymphoma subtypes. Although rare, patients with type 2 refractory celiac disease have been found to have an increased risk of enteropathy-associated T cell lymphoma (often referred to as EATL). A high index of suspicion should be considered for patients with severe celiac disease with significant issues consistent with a recurrent obstruction or evidence of steatorrhea. Despite this primarily being noted through observational studies, some suggest that the diagnosis of EATL usually is heralded by a clinical relapse after the patient is found to have an adequate response to abstinence from gluten. Go to the next page if you knew the correct answer, or click the link image(s) below to further research the concepts in this question (if desired).
Research Concepts: Small Bowel Cancer
We update eBooks quarterly and Apps daily based on user feedback. Please tap flag to report any questions that need improvement.
Question 616:
A 57-year-old woman presents with progressive numbness of the left hand and foot in addition to pain in the right retro-orbital area that gets worse when she bends forward. She describes the retroorbital pain as throbbing. The numbness started about a week ago and was accompanied by nausea and occasional vomiting. She has a past medical history significant for breast cancer for which she underwent a total mastectomy fifteen months prior. Vital signs show oxygen saturation of 98% on room air, respiratory rate of 16 per minute, heart rate of 72 beats per minute, blood pressure 140/90 mmHg, and temperature of 98 degrees F. On physical examination, her pupils are isocoric, equally reactive to light with full extraocular muscle movement. There are no visual field deficits. There is a left lower facial droop with no dysarthria. The motor examination is unremarkable. There is a partial sensory deficit to pain and light touch on the left hemibody, and she is unable to recognize stimulation on the left side on double simultaneous tactile stimulation. What is the most appropriate next step in the management of this patient?
Choices: 1. Electroencephalogram 2. Magnetic resonance imaging (MRI) of the brain with and without contrast 3. Computed tomography (CT) head with and without contrast 4. Electromyogram/nerve conduction study
Answer: 2 - Magnetic resonance imaging (MRI) of the brain with and without contrast
Explanations: An MRI is indicated in a patient with a history of malignancy and focal neurologic signs. Contrasted MRI is helpful for evaluating for enhancing lesions, including malignancy, infection, etc. CT scans are helpful for evaluating osseous anatomy as well as hemorrhage. A CT scan could be an alternative if this patient had a contraindication to undergoing an MRI scan. However, if no contraindication, such as a pacemaker, spinal cord stimulator, etc then an MRI scan would provide significantly higher resolution than a CT scan. An electroencephalogram (EEG) would be helpful for working up a suspected seizure. While brain lesions can certainly be irritating and predispose patients to seizures, her clinical presentation is more progressive and is likely due to the mass effect from the brain lesion itself rather than a persistent epileptic event. EMG/NCV can be helpful for working up peripheral neuropathies. Given her clinical presentation and history of cancer, her presentation is more concerning for a lesion within the central nervous system, specifically the brain. Go to the next page if you knew the correct answer, or click the link image(s) below to further research the concepts in this question (if desired).
Research Concepts:
Brain Metastasis
We update eBooks quarterly and Apps daily based on user feedback. Please tap flag to report any questions that need improvement.
Question 617:
A 7-year-old girl with cutaneous nodules and cafe-au-lait spots is brought to the hospital with a new constant headache. An MRI reveals a suspicious mass. Which of the following is most likely responsible for the patient's symptoms?
Choices: 1. 2. 3. 4.
Medulloblastoma Optic nerve glioma Vestibular schwannoma Meningioma
Answer: 2 - Optic nerve glioma Explanations: The patient has neurofibromatosis type 1, also called von Recklinghausen disease. It is characterized by cafe-au-lait spots, subcutaneous tumors, gliomas, bone cysts, Lisch nodules, macrocephaly, epilepsy, meningiomas, precocious puberty, and pheochromocytomas. It is an autosomal dominant condition caused by a mutation of the NF1 gene. Optic nerve gliomas are the most common brain tumor in NF1 patients. Medulloblastomas are the most common brain tumor in children in general. Go to the next page if you knew the correct answer, or click the link image(s) below to further research the concepts in this question (if desired).
Research Concepts: Optic Nerve Glioma
We update eBooks quarterly and Apps daily based on user feedback. Please tap flag to report any questions that need improvement.
Question 618:
A 53-year-old male presented with complaints of abdominal pain, nausea, indigestion, weakness, anorexia, and loss of appetite, in addition to weight loss. Ultrasound done revealed cholelithiasis, and laparoscopic cholecystectomy was performed. Pathologic evaluation showed involvement of the mucosa; however, the muscularis was not involved. Besides, carbohydrate antigen 19-9 was also found to be elevated. Which of the following is most likely to be the most appropriate treatment for this patient?
Choices: 1. 2. 3. 4.
Radiotherapy Systemic 5-FU/leucovorin Systemic gemcitabine/oxaliplatin No treatment is needed
Answer: 4 - No treatment is needed Explanations: Gallbladder cancer (GC) is detected more frequently as an incidental finding on imaging or after a surgical procedure. Early-stage GC is commonly discovered after cholecystectomy and review of the surgical pathology specimen. Physical examination may demonstrate jaundice, right upper quadrant pain, or Courvoisier sign (non-tender palpable gallbladder with jaundice) which is most likely to develop due to chronic progressive malignant obstruction rather than an intermittent gallstone obstruction. Hepatomegaly, abdominal palpable mass, ascites, and bowel obstruction on physical examination are presentations indicating advanced metastatic stage. Surgical treatment of cancer of the gallbladder is the only treatment that guarantees a cure. Disease limited to the mucosa and submucosa requires no further treatment. The disease spread into the muscularis is best treated with gemcitabine/oxaliplatin and further surgery. This should include partial regional lymphadenectomy and hepatic resection. Go to the next page if you knew the correct answer, or click the link image(s) below to further research the concepts in this question (if desired).
Research Concepts: Gallbladder Cancer
We update eBooks quarterly and Apps daily based on user feedback. Please tap flag to report any questions that need improvement.
Question 619:
A 65-year-old patient, who is a chronic smoker, presented to the clinic with complaints of blood in the stool and weight loss. After evaluation and testing, he has been diagnosed with colorectal cancer. Immunohistochemical testing on the tumor sample after resection shows MLH1 and PSM2 staining present, but MSH2 and MSH6 are absent. Upon further inquiry, the patient states that his grandfather was also diagnosed with colorectal carcinoma. Which of the following is the next best step for confirmation of the likely diagnosis of this patient?
Choices: 1. Test for CDH1 2. Test for epithelial cell adhesion molecule (EPCAM) mutation 3. Proceed to germline mutation testing for MSH2 4. Proceed to germline mutation testing for MSH6
Answer: 3 - Proceed to germline mutation testing for MSH2
Explanations: While there is a high likelihood the patient has Lynch syndrome with both MSH2 and MSH6, additional testing for germline mutation testing is required. According to the updated guidelines, Analysis can be done by immunohistochemistry (IHC) testing for the MLH1/MSH2/MSH6/PMS2 proteins and/or testing for MSI. Those tumors with loss of MLH1 should be considered for BRAF testing or analysis of MLH1 promoter hypermethylation. In order to facilitate further surgical planning, tumor testing in patients with suspicious colorectal cancers on preoperative biopsy specimens is preferred. If no germline mutation is found in MSH2, then testing for EPCAM should be considered. Although EPCAM is not a mismatch repair protein, it can affect MSH2 expression, and deletions of EPCAM have been found in patients with Lynch syndrome. If still no mutation is found, then genetic testing for MSH6 should be sent. Go to the next page if you knew the correct answer, or click the link image(s) below to further research the concepts in this question (if desired).
Research Concepts: Lynch Syndrome
We update eBooks quarterly and Apps daily based on user feedback. Please tap flag to report any questions that need improvement.
Question 620:
A 65-year-old female presents to the office with epigastric pain for three months duration. She reports that the pain is dull and unrelated to meals. She uses acetaminophen as needed, with minimal relief. Her medical history is significant for hypertension and hyperlipidemia. She takes nifedipine and atorvastatin. Vital signs show a blood pressure of 135/68 mmHg, pulse 68/minute, and respiratory rate 12/minute. Laboratory results show serum lipase 35 U/L (10–140 U/L), liver enzymes are within normal limits. She recently had an upper gastrointestinal endoscopy, which was unremarkable. A CT of the abdomen and pelvis was performed, which showed multiple cysts in the pancreas. What is the next best step in the management of this patient?
Choices: 1. Obtain an MRI and magnetic resonance cholangiopancreatography (MRCP) 2. Obtain an abdominal ultrasound 3. Obtain CEA levels 4. Reassurance, pain control, and regular followups
Answer: 1 - Obtain an MRI and magnetic resonance cholangiopancreatography (MRCP)
Explanations: Pancreatic cysts may be asymptomatic or may present with symptoms such as abdominal pain. Patients who present with symptoms are classified as high risk for possible malignancy and require further evaluation. MRI and magnetic resonance cholangiopancreatography (MRCP) are required for further evaluation of pancreatic cysts suspected to be malignant. Surgery is required for pancreatic cysts with concerning for malignancy. Go to the next page if you knew the correct answer, or click the link image(s) below to further research the concepts in this question (if desired).
Research Concepts: Pancreatic Cysts
We update eBooks quarterly and Apps daily based on user feedback. Please tap flag to report any questions that need improvement.
Question 621:
Which of the following bone disorders has been associated with tinnitus?
Choices: 1. 2. 3. 4.
Rheumatoid arthritis Paget disease Multiple myeloma Gout
Answer: 2 - Paget disease Explanations: Paget disease is a rare cause of tinnitus. When Paget disease involves the skull, patients may develop deafness, vertigo, tinnitus, vertebral insufficiency, and cranial nerve involvement. The most common cranium symptom of Paget disease is hearing loss, a complication occurring in 30% to 60% of patients who have skull involvement. The most common neurologic complication of Paget disease is deafness because of the involvement of the petrous temporal bone. Go to the next page if you knew the correct answer, or click the link image(s) below to further research the concepts in this question (if desired).
Research Concepts: Paget Disease
We update eBooks quarterly and Apps daily based on user feedback. Please tap flag to report any questions that need improvement.
Question 622:
A 47-year-old male patient presented with blood-stained nasal secretions, obstruction, and otorrhea with frequent spells of nasal regurgitation for the last six months. The patient also explained that recently, he has started noticing a rash on his upper back, chest, and shoulders with nodular swellings over his fingers. On examination, the patient has a small nodular mass localized to the apex of the posterior triangle with mild peri-orbital edema and discoloration around the eyelids. On nasopharyngoscopy, the patient has a proliferative mass obscuring the choana. On investigation, the patient has a normal complete blood count with elevated serum levels of creatine kinase. Imaging shows a heterogenous mass extending anteriorly from the roof of the nasopharynx. Which of the following best explains the patient’s recent complaints?
Choices: 1. Paraneoplastic syndrome 2. Vascular invasion with distant metastasis 3. Distant nodal involvement 4. Widespread embolic disease on the background of the nasopharyngeal primary
Answer: 1 - Paraneoplastic syndrome Explanations: The clinical vignette is most consistent with paraneoplastic syndrome on the background of nasopharyngeal carcinoma (NPC). Patients with NPC develop dermatomyositis as a paraneoplastic syndrome. Dermatomyositis presents with rashes occurring on the upper back, chest, and shoulders (‘shawl’ distribution), periorbital heliotropic rash, and Gottron papules. Dermatomyositis has a variable presentation. It develops in concurrence with the neoplastic process and can present either late after the diagnosis or months before the clinical presentation of the case. Go to the next page if you knew the correct answer, or click the link image(s) below to further research the concepts in this question (if desired).
Research Concepts: Nasopharyngeal Carcinoma
We update eBooks quarterly and Apps daily based on user feedback. Please tap flag to report any questions that need improvement.
Question 623:
A 24-year-old female patient presents to the emergency department complaining of a headache of 4 weeks duration. She reports the pain is diffuse and has been progressively worsening. She has tried over the counter tylenol and aspirin with minimal effect. A CT of the head revealed a hypodense mass without any enhancement. A biopsy is obtained, which showed a diffuse low-grade astrocytoma. Family history is unknown as she was an orphan. Which of the following would help in establishing the diagnosis?
Choices: 1. 2. 3. 4.
MRI of the liver Colonoscopy CT of the chest Iron panel
Answer: 2 - Colonoscopy Explanations: Patients with Turcot Syndrome can rarely present with an astrocytoma. Given her age, unknown family history and biopsy result, it would be prudent for her to undergo a colonoscopy to rule out Turcot. Diagnosing Turcot syndrome is helpful because making correlations between brain tumors and colorectal cancer can help guide preventative action for either condition. It also provides anticipatory guidance–to know and expect brain vs. colon cancer in patients that exhibit either clinical presentation; early diagnosis leads to early treatment. Turcot syndrome (TS) is the association of primary brain tumors to colorectal cancer. Another correct answer that may have been appropriate for her could have been to check for evidence of APC or MMR gene mutations. Go to the next page if you knew the correct answer, or click the link image(s) below to further research the concepts in this question (if desired).
Research Concepts: Turcot Syndrome
We update eBooks quarterly and Apps daily based on user feedback. Please tap flag to report any questions that
need improvement.
Question 624:
A 67-year-old woman undergoing chemotherapy for small cell lung cancer presents to the clinic with 4 days of right temporal headache, dysphagia, and a red, tender right ear. Physical examination reveals a severely inflamed right auditory canal, and close examination is not possible due to the intense pain. What is the most likely diagnosis?
Choices: 1. 2. 3. 4.
Otitis media Otitis externa Malignant otitis externa Sinusitis
Answer: 3 - Malignant otitis externa Explanations: Malignant otitis externa is usually caused by Pseudomonas aeruginosa and can progress to involve the cranial nerves. Treatment of malignant otitis externa includes intravenous antibiotics and will prevent complications like abscess and cranial nerve involvment. Cranial nerves V-XII can be affected. Diabetes mellitus and other immunocompromised states are risk factors for infection. Go to the next page if you knew the correct answer, or click the link image(s) below to further research the concepts in this question (if desired).
Research Concepts: Malignant Otitis Externa
We update eBooks quarterly and Apps daily based on user feedback. Please tap flag to report any questions that need improvement.
Question 625:
A 64-year-old male is started on the RICE regimen (rituximab, ifosfamide, carboplatin, and etoposide) as salvage therapy for refractory non-Hodgkin lymphoma with brain metastases. About five hours after receiving his first dose of ifosfamide, the patient becomes acutely confused and starts hallucinating. Vitals signs show a blood pressure of 100/70 mmHg, and the physical exam is unremarkable. What is the best next step in the management of this patient?
Choices: 1. Discontinuation of the offending drug only 2. Methylene blue administration only 3. Discontinuation of the offending drug and methylene blue administration 4. Observation
Answer: 3 - Discontinuation of the offending drug and methylene blue administration
Explanations: The drug needs to be discontinued immediately in this patient due to encephalopathy. When treating with ifosfamide, central nervous system toxicity appears as encephalopathy with varying severity (confusion, hallucinations, drowsiness, coma). It presents in about 30% of patients, and the symptoms occur when administering high doses of the drug either orally or intravenously but more with oral administration. Usually, the symptoms present within 2 to 96 hours after drug administration and are reversible within 48 to 72 hours following discontinuation of the drug. Risk factors associated with the development of neurotoxicity include oral administration, previous chemotherapy with cisplatin, impaired renal and hepatic function, low albumin, and brain metastasis. Methylene blue can be used for both treatment and prophylaxis of ifosfamide-induced encephalopathy due to the drug's ability to prevent the formation of neurotoxic metabolites that cause encephalopathy. It helps in the reversal of symptoms within 24 hours of its administration. The dosage of methylene blue is 50 mg orally in 5% glucose solution every four hours until recovery. As prophylaxis, the severity of symptoms is reduced compared to previous cycles and can restore therapy with ifosfamide. The prophylactic dose is 50 mg IV every 6 to 8 hours during ifosfamide administration.
Go to the next page if you knew the correct answer, or click the link image(s) below to further research the concepts in this question (if desired).
Research Concepts: Ifosfamide
We update eBooks quarterly and Apps daily based on user feedback. Please tap flag to report any questions that need improvement.
Question 626:
A 55-year-old male presents to the emergency department for evaluation of chest pain at rest and on exertion. Cardiac workup, including EKG and troponins, is normal. The patient has a history of a thymic neuroendocrine tumor that was resected 5 years ago. A CT chest with IV contrast is obtained, which shows a 5 cm in greatest dimension, lobulated, necrotic anterior mediastinal mass. On further scans, there is no evidence of locally invasive or distant metastatic disease. What is the first therapy option for this patient with the recurrent disease?
Choices: 1. 2. 3. 4.
Neoadjuvant chemotherapy Octreotide therapy Adjuvant radiation therapy Surgical resection
Answer: 4 - Surgical resection Explanations: Thymic neuroendocrine tumors are highly aggressive malignancies, and the rate of recurrence is higher than their gastrointestinal or lung counterparts. The benefit of neoadjuvant chemotherapy is still not clear, as the response is variable. It is used in instances of locally invasive disease with the intention to prepare the patient for surgical intervention. Even in recurrent disease, if the mass is deemed operable, surgical resection should still be attempted. Octreotide therapy is commonly used in the management of gastrointestinal neuroendocrine malignancies. Somatostatin analog therapy is reserved for tumors with positive somatostatin expression. In the rare cases of carcinoid syndrome with a thymic neuroendocrine tumor, this therapy might be considered. Adjuvant radiation therapy is utilized in the cases of microscopic residual disease. Go to the next page if you knew the correct answer, or click the link image(s) below to further research the concepts in this question (if desired).
Research Concepts: Mediastinal Carcinoid Tumors
We update eBooks quarterly and Apps daily based on user feedback. Please tap flag to report any questions that need improvement.
Question 627:
A 48-year-old female patient, nulliparous, presents to the outpatient clinic with complaints of irregular delayed menses for six months and upper abdominal pain for one week. Imaging showed a 11x8x6 cm solid mass from the right ovary with areas of hemorrhage and necrosis, compressing the right distal ureter and right external iliac vein with enlarged para-aortic nodes. Tumor markers (CA-125, CA 19-9, carcinoembryonic antigen, beta hCG, and alpha-fetoprotein) were normal, but inhibin A (141 pg/mL) and inhibin B (1261 pg/mL) were elevated. She underwent total abdominal hysterectomy with bilateral salpingo-oophorectomy, pelvic and para-aortic lymphadenectomy,infracolic omentectomy with peritoneal biopsy, and fluid cytology with endometrial biopsy. HPE showed granulosa theca cell cancer with lymph node metastases. Which of the following is the next ideal recommended treatment?
Choices: 1. Adjuvant chemotherapy cisplatin 2. Adjuvant chemotherapy bleomycin 3. Adjuvant chemotherapy cyclophosphamide 4. Adjuvant chemotherapy
with bleomycin, etoposide, and with cisplatin, vinblastin, and with cisplatin, doxorubicin, and with paclitaxel and carboplatin
Answer: 4 - Adjuvant chemotherapy with paclitaxel and carboplatin
Explanations: This patient has FIGO Stage III granulosa cell cancer, for which the recommended treatment is complete cytoreduction followed by adjuvant chemotherapy. Adjuvant chemotherapy has been associated with longer progression-free survival among advanced granulosa cell tumors [FIGO stage II to IV]. Still, there is no evidence supporting an overall survival benefit. The most commonly used regimens are bleomycin, cisplatin, plus etoposide (BEP), or paclitaxel plus carboplatin (TC). For younger, fit patients BEP regime is preferred, while the TC regime is preferred for patients >40 years old due to toxicity concerns. Go to the next page if you knew the correct answer, or click the link image(s) below to further research the concepts in this question (if desired).
Research Concepts: Granulosa Theca Cell Cancer
We update eBooks quarterly and Apps daily based on user feedback. Please tap flag to report any questions that need improvement.
Question 628:
A 60-year-old male is being evaluated at the emergency department for a frontal right-sided headache. Skull X-rays show mild opacification of the right frontal sinus. Brain magnetic resonance imaging with and without contrast is performed. It shows an incidental 2.0 cm x 1.8 cm homogeneous enhancing mass at the atrium of the left lateral ventricle without evidence of ventricular trapping or edema on adjacent brain parenchyma. The mucosa of the frontal sinus is thickened and enhanced. On the physical exam, the patient is neurologically intact. He is prescribed nasal decongestants. What is the best next step in management for the incidental mass?
Choices: 1. Follow-up brain magnetic resonance imaging with and without contrast in 6 months 2. Stereotactic needle biopsy 3. Left-sided craniotomy for tumor resection 4. Radio-oncologist consult for elective radiosurgery
Answer: 1 - Follow-up brain magnetic resonance imaging with and without contrast in 6 months
Explanations: A well-circumscribed, homogeneously enhancing mass in the atrium of the lateral ventricle is most likely an intraventricular meningioma. Intraventricular meningiomas are mainly benign, slowgrowing lesions. However, it is essential to follow-up with serial imaging due to the risk of ventricular trapping and symptomatic parenchymal edema that may warrant surgical intervention. If there is documentation of tumor growth on follow-up magnetic resonance imaging and the diagnosis of meningioma by brain magnetic resonance imaging is clear. One may proceed with treatment with radiosurgery if the lesion is under 3.0 cm in all dimensions. If the diagnosis is unclear, then a less invasive stereotactic needle biopsy vs. open tumor resection is strongly suggested. Even if meningioma is found incidentally, when there is evidence of associated parenchymal edema or trapped lateral ventricle, then urgent/emergent surgery should be performed. Go to the next page if you knew the correct answer, or click the link image(s) below to further research the concepts in this question (if desired).
Research Concepts: Intraventricular Meningioma
We update eBooks quarterly and Apps daily based on user feedback. Please tap flag to report any questions that need improvement.
Question 629:
A 65-year-old woman presents to the emergency department with a complaint of heavy vaginal bleeding. The bleeding started two days ago as vaginal spotting and has now increased, soaking a pad every 3 hours. Menarche was at age 11. Menopause at age 50 and the patient had no episodes of vaginal bleeding since then. She has smoked a pack of cigarettes daily for 30 years now. She does not use illicit drugs or alcohol. Family history is positive for endometrial cancer in the mother. Blood pressure is 110/70 mmHg, and pulse is 80/min. BMI is 36 kg/m2. An abdominal examination revealed no tenderness. There is blood in the posterior vaginal vault and a 5 cm friable lesion on the ectocervix, which bleeds on touch. A pelvic USG reveals a 3 mm endometrial stripe and no adnexal masses. Which of the following is a risk factor for the patient's condition?
Choices: 1. 2. 3. 4.
Early menarche and late menopause Obesity Tobacco use Chronic anovulation
Answer: 3 - Tobacco use Explanations: Cervical cancer is the 4th leading cause of cancer in women. It is an easily preventable disease, owing to the universal application of Papanicolaou smear screening in developed countries that has led to early detection of pre-cancerous lesions which can be removed before they progress to invasive cancer. However, it is still a very common genital cancer encountered in clinical practice in women of low and middle-income countries due to a lack of extensive screening. Cervical cancers are most often squamous cell carcinomas that arise from infection with high-risk human papillomavirus (HPV) serotypes 16 and 18. Advanced cervical cancer can present as vaginal bleeding with a cervical lesion. Smoking is a risk factor for squamous cell cervical carcinoma. Smoking exerts an oncogenic effect on the cervix through impaired immunity. Diagnosis is established by cervical biopsy or colposcopy. Go to the next page if you knew the correct answer, or click the link image(s) below to further research the concepts in this question (if desired).
Research Concepts: Cervical Intraepithelial Squamous Cell Lesion
We update eBooks quarterly and Apps daily based on user feedback. Please tap flag to report any questions that need improvement.
Question 630:
A 45-year-old man presents with a twomonth history of recurrent epistaxis from the right nostril associated with blockage of the same. He denies any nausea, vomiting, or light-headedness. Past medical history is significant for type 2 diabetes mellitus. He denies smoking tobacco, drinking alcohol, or using any illicit drugs. Vital signs reveal a blood pressure of 98/69 mmHg, a pulse of 89 beats per minute, a respiratory rate of 12 breaths per minute, and a temperature of 37.2 degrees Celsius. On rhinoscopy, there is an irregular fleshy granular mass arising from the roof of the nasal cavity. He undergoes a computed tomogram scan, which shows a mass sized 5x4 centimeters extending from the nasal cavity into the adjacent maxilla. Nuclear medicine study shows that the lesion is MIBG-avid (metaiodobenzylguanidine). He undergoes an intranasal biopsy with local anesthesia. Which of the following is the most likely histological finding in the specimen?
Choices: 1. 2. 3. 4.
Homer-Wright pseudorosette Perivascular pseudorosette Flexner–Wintersteiner rosette True rosette
Answer: 1 - Homer-Wright pseudorosette Explanations: The patient is diagnosed with esthesioneuroblastoma, as evidenced by the clinical picture and the MIBG-avid (metaiodobenzylguanidine) nuclear scan. Low-grade esthesioneuroblastoma form submucosal, sharply demarcated nests, lobules, or sheets of cells, often separated by richly vascular or hyalinized fibrous stroma. Pseudorosettes (Homer-Wright rosettes), with neoplastic cells palisading or cuffed around the central delicate fibrillar neural matrix, may be seen. The tumor cells are often uniform, with sparse cytoplasm and round or ovoid nuclei with punctate saltand-pepper chromatin with small or absent nucleoli. Esthesioneuroblastoma is characterized by fibrillary cytoplasm and interdigitating neuronal processes (neuropil), created by a syncytium of cells. The tumor cells can be arranged in gland-like rings or tight annular formations with a true lumen (FlexnerWintersteiner rosettes). Rosettes alone are not diagnostic of esthesioneuroblastoma, although the Homer-Wright rosettes are nearly pathognomonic in the nasal cavity when containing true neuropil. Go to the next page if you knew the correct answer, or click the link image(s) below to further research the concepts in this question (if desired).
Research Concepts: Esthesioneuroblastoma
We update eBooks quarterly and Apps daily based on user feedback. Please tap flag to report any questions that need improvement.
Question 631:
A 58-year-old woman presented with a 3-week history of lower abdominal discomfort. On examination, an abdominal mass was palpable. There is no family history of cancer. The computed tomographic (CT) imaging demonstrated a pelvic mass disease with mixed density. cancer antigen 125 (Ca 125) was 10 U/mL (normal, 0–35 U/mL), and a-fetoprotein (AFP) was 1847 ng/mL (normal, 0–20 ng/mL). The clinician decided to do a surgery with total abdominal hysterectomy (TAH), bilateral salpingooophorectomy (BSO), pelvic and para-aortic lymphadenectomy. Surgery reviewed a mass arising from the right ovary. The mass is yellow in color with significant necrosis and hemorrhages. Which of the following features is pathognomonic for this type of tumor?
Choices: 1. 2. 3. 4.
Psammoma bodies Schiller–Duval bodies Reed-Sternberg cells Auer rods
Answer: 2 - Schiller–Duval bodies Explanations: Yolk sac tumor is a member of germ cell tumors. Germ cell tumors usually arise in gonads but sometimes can arise extragonadal. Schiller–Duval body is pathognomonic for yolk sac tumors and resembles a glomeruloid structure with a central fibrovascular core. However, Schiller-Duvall bodies are present only in 5075% of these tumors. Yolk sac tumors are uncommon neoplasms in the ovary. They account for 15% of all ovarian germ cell tumors, they present in early life and rarely after the age of 40. The median age of diagnosis is 19 years, and 40% of patients are diagnosed in the prepubertal period. About 60% of ovarian yolk sac tumors present in a pure form, while 40% present with mixed germ cell tumors. Grossly, yolk sac tumors are poorly demarcated solid masses. They are gray to yellow in color with a gelatinous, or mucoid appearance. Significant necrosis, cystic changes, and hemorrhage are usually present. Alpha-fetoprotein (AFP) is a characteristic of yolk sac tumor, but not specific for yolk sac tumors because it can be secreted by other cancers like hepatocellular carcinoma. Go to the next page if you knew the correct answer, or click the link image(s) below to further research the concepts in this question (if desired).
Research Concepts: Yolk Sac Tumors
We update eBooks quarterly and Apps daily based on user feedback. Please tap flag to report any questions that need improvement.
Question 632:
A 75-year-old male with a history of 45 years of smoking presents to the hospital with chief complaints of polyuria, polydipsia, and severe abdominal pain. He does not have any known medical history. Which of the following could be the most likely cause?
Choices: 1. 2. 3. 4.
Diabetes insipidus Paraneoplastic presentation of occult malignancy Psychogenic polydipsia Drug-induced
Answer: 2 - Paraneoplastic presentation of occult malignancy
Explanations: This patient most likely has an occult lung malignancy, given his strong history of smoking. Paraneoplastic presentation of hypercalcemia can be the initial presentation even before the malignancy is detected. Clinicians should be aware of such presentations and identify and manage the symptoms early. Hypercalcemia of malignancy in such cases is usually in advanced disease and portends a poor prognosis. Immediate workup by measuring serum calcium levels and imaging for underlying malignancy needs to be done. The most common solid malignancies associated with the condition are renal, breast, ovarian, and lung cancers. Hematological malignancies like multiple myeloma are also associated with hypercalcemia. Go to the next page if you knew the correct answer, or click the link image(s) below to further research the concepts in this question (if desired).
Research Concepts: Malignancy-Related Hypercalcemia
We update eBooks quarterly and Apps daily based on user feedback. Please tap flag to report any questions that
need improvement.
Question 633:
A 39-year-old woman presents with a headache, blurry vision, and severe nausea. It is found she has a brain tumor. She proceeds with chemotherapy and radiotherapy, but the malignancy fails to regress. The doctor decides he wants to try an experimental drug, phenothiazine, that normally is used to treat schizophrenia and bipolar disorder for regression of cancer and symptomatic management. The patient and the family agrees to the proposal. After some time, the patient develops problems with muscle movement. What is the most likely pathology that leads to this new symptom?
Choices: 1. Inhibition of serotonin reuptake in the synaptic cleft 2. Reduction of cyclins in the brain by increasing cyclin inhibiting compounds 3. Inhibition of the dopamine D2 receptor of the mesolimbic pathway 4. Degradation of the microtubules of astrocytes in the brain
Answer: 3 - Inhibition of the dopamine D2 receptor of the mesolimbic pathway
Explanations: Phenothiazine is an aromatic tricyclic compound used in the treatment of schizophrenia and bipolar disorder. Phenothiazines are effective against glioblastomas by arresting the G1 cycle of the cancer cells. Phenothiazines reduce the cyclins by increasing cyclin inhibiting compounds, restricting DNA replication. Phenothiazines have found efficacy in treatment against glioblastomas of the brain. Their fundamental neuroleptic action connected with the dopaminergic receptors blockade leads to neuromuscular coordination, thus resulting in a movement disorder. They may cause unwanted and uncontrolled face or body movements. Prominent adverse effects include neuromuscular incoordination, photosensitization, and hemolytic anemia. Phenothiazines are used in acute psychotic episodes, schizophrenia, bipolar disorder, severe depression, and in certain patients with severe nausea and vomiting. Inhibition of serotonin reuptake in the synaptic cleft is the side effect of selective serotonin receptor inhibitors (SSRIs). Degradation of the microtubules of astrocytes in the brain is the mechanism of various neurological diseases and chemotherapeutic regimens. Go to the next page if you knew the correct answer, or click the link image(s) below to further research the concepts in this question (if desired).
Research Concepts: Phenothiazine
We update eBooks quarterly and Apps daily based on user feedback. Please tap flag to report any questions that need improvement.
Question 634:
A 2-month-old, previously healthy girl is admitted to the hospital with fever and confusion. She is tachycardic and hypotensive. On physical exam, she is jaundiced with diffuse adenopathy and bruising of her distal extremities. Lab work reveals pancytopenia, acute intrinsic liver injury, and acute renal injury. A bone marrow biopsy demonstrates normocellular bone marrow with giant macrophages engulfing healthy cells of every lineage. Which of the following describes the minimum requirement considered diagnostic for this disorder?
Choices: 1. 2. 3. 4.
Hemophagocytosis on bone marrow biopsy 8 out of 8 of the disease's diagnostic criteria Serum ferritin >10,000 mcg/L 5 out of 8 of the disease's diagnostic criteria
Answer: 4 - 5 out of 8 of the disease's diagnostic criteria Explanations: The hemophagocytic lymphohistiocytosis (HLH)-2004 diagnostic criteria includes fever, cytopenias (at a minimal two lineages), splenomegaly, hypertriglyceridemia +/- hypofibrinogenemia, biopsyproven hemophagocytosis, ferritin >500 mcg/L, low or absent NK-cell activity, and elevated sIL2Ra levels >/= 2400 U/mL. Although pathognomonic, hemophagocytosis is only present in approximately 67% of children with HLH and 85% of adults with HLH. Serum ferritin >10,000 mcg/L has a sensitivity and specificity of 90% and 98%, respectively. Despite these excellent test characteristics, the diagnosis still requires at least 5 of the HLH-2004 criteria. In any child who presents with multi-organ failure, the diagnosis of HLH should be placed on the differential. Although the disease has a relatively low incidence in the population, it has been estimated to be present in 1:3000 hospitalized pediatric cases. Go to the next page if you knew the correct answer, or click the link image(s) below to further research the concepts in this question (if desired).
Research Concepts: Lymphohistiocytosis
We update eBooks quarterly and Apps daily based on user feedback. Please tap flag to report any questions that need improvement.
Question 635:
An 81-year-old male non-smoker presents with periodic dizziness, weakness, and a nonproductive cough. His chest x-ray shows a 14 cm, welldemarcated, pleural-based mass in the right lung. The biopsy was positive for a diagnosis of a solitary fibrotic tumor of the pleura. The PET scan was positive only for the right lung mass. Cardiac and neurologic workups were negative. The patient agreed to surgery; however, surgery was postponed as repeated morning fasting blood sugars were found in the 55-65 mg/d range; (normal range 70-100 mg/dL). Additional labs revealed decreased serum free insulin, proinsulin, and serum IGF-1 levels. Screenings for blood sulfonylureas and insulin antibodies were negative. ACTH and a.m. cortisol levels were both normal. Intravenous 10% Dextrose and liberal feedings were initiated but were not entirely successful in managing the hypoglycemia. What would be the next best step in the care of this patient?
Choices: 1. 2. 3. 4.
Neoadjuvant radiation Administer embolization therapy Continuous infusion glucagon Initiate intravenous glucocorticoids
Answer: 4 - Initiate intravenous glucocorticoids Explanations: Radiation may shrink the tumor a modest amount and allow for palliation. However, neoadjuvant therapy has no definitive role in treating solitary fibrous tumors of the pleura. Embolization of the tumor is of limited use in treating this tumor. It is utilized primarily in treating massive, giant (typically > 15 cm) solitary fibrous tumors of the pleura. Continuous infusion of glucagon has the drawback of non-sustainability. The low blood sugar problem must be controlled until the tumor is resected. Once resected, the hypoglycemia will resolve. The patient has hypoglycemia compounding solitary fibrous tumor of the pleura, also known as Doege-Potter syndrome. The hypoglycemia is due to an ectopic IGF-2 produced by the tumor. Hypoglycemia must be corrected and controlled during the perioperative period; failure to do so can lead to acute neurologic damage. Steroids are very efficacious in treating hypoglycemia, where they suppress IGF-2 production by as much as 75%. The failure of IVD10 to normalize and stabilize the situation warrants steroid administration. Go to the next page if you knew the correct answer, or click the link image(s) below to further research the concepts in this question (if desired).
Research Concepts: Pleural Solitary Fibrous Tumors
We update eBooks quarterly and Apps daily based on user feedback. Please tap flag to report any questions that need improvement.
Question 636:
A 44-year-old male presents with a three-week history of abdominal pain, nausea, vomiting, and intermittent diarrhea. Additionally, he has experienced facial flushing over the past week. His blood pressure is 122/82 mmHg, his heart rate is 85 beats per minute, his respiratory rate is 15 breaths per minute, and his temperature is 98.0 F (36.7 C). On physical examination, the patient has flushing of the face and neck. Lacrimation is present. The most appropriate treatment for this patient inhibits which of the following?
Choices: 1. 2. 3. 4.
Growth hormone Mammalian target of rapamycin (mTOR) Thyroid hormone Epinephrine
Answer: 1 - Growth hormone Explanations: Some patients who have intestinal carcinoids can present with symptoms of pellagra. Somatostatin analogs are first-line agents used for the treatment of symptomatic carcinoids. Right-sided includes octreotide and lanreotide, which are highly effective in controlling symptoms with intestinal carcinoid. A depot preparation of octreotide is available in monthly intramuscular injections. These are usually well-tolerated except for mild symptoms such as nausea or bloating. The dietary tryptophan is diverted for conversion to serotonin. If carcinoid is suspected, then a 24-hour urine analysis for 5-hydroxytryptamine is recommended. mTOR inhibitors like everolimus are currently approved in the treatment of carcinoid cancer as second-line therapy for patients who have progressive symptoms despite the use of somatostatin analogs. Go to the next page if you knew the correct answer, or click the link image(s) below to further research the concepts in this question (if desired).
Research Concepts: Intestinal Carcinoid Cancer
We update eBooks quarterly and Apps daily based on user feedback. Please tap flag to report any questions that need improvement.
Question 637:
A 65-year-old man who recently immigrated from Japan presents with fever, patchy red rash, especially in the trunk, lumps, and bumps in the neck for the past week. He admits to having unprotected sexual intercourse with multiple sexual partners. Pertinent lab work includes hemoglobin 13g/dl, white count 100,000, calcium 13 mg/dl LDH 400, with peripheral blood smear showing circulating lymphocytes with abnormally convoluted nuclei. What would be the appropriate antiviral for his diagnosis?
Choices: 1. 2. 3. 4.
Acyclovir Interferon alpha with ganciclovir Interferon-alpha with zidovudine Ganciclovir
Answer: 3 - Interferon-alpha with zidovudine Explanations: The patient has T-cell lymphoma leukemia caused by Human T-cell lymphotropic virus-1. Antiviral specifically, Interferon alpha plus zidovudine combination has been approved in the acute, chronic, and smoldering stages of the lymphoma, not in the lymphoma stage of the disease. Stem cell transplant is considered after the consolidation phase, after achieving remission. Antiviral treatment is used alongside standard chemotherapy. Go to the next page if you knew the correct answer, or click the link image(s) below to further research the concepts in this question (if desired).
Research Concepts: T-Cell Lymphoma
We update eBooks quarterly and Apps daily based on user feedback. Please tap flag to report any questions that need improvement.
Question 638:
A 10-year-old boy who is undergoing treatment for leukemia is brought to the office by his mother. He is doing well with chemotherapy and has no symptoms currently, but his younger sister has a fever and vesicular rash. The vesicles are in crops and at different stages of evolution. The boy's vital signs include a pulse of 90/min, blood pressure of 111/64 mmHg, respiratory rate of 20/min, and temperature of 98.6 F (37 C). Physical examination is unremarkable. What is the most appropriate management for this child?
Choices: 1. 2. 3. 4.
Antiviral medication Observation Immune globulin Antibiotics
Answer: 3 - Immune globulin Explanations: The younger sibling has developed a vesicular rash, which is consistent with varicella-zoster infection. Because the boy is immunocompromised, he requires immune globulin to decrease his risk of infection. Varicella-zoster immunoglobulin (VZIG) provides passive immunity to susceptible patients who have been exposed to chickenpox or zoster. VZIG is the most beneficial when administered soon after the presumed exposure, but it may be effective if given up to 10 days after exposure. A live-attenuated varicella vaccine is indicated for immunocompetent, non-pregnant patients 12 months and older. There is high seroconversion following the vaccine, and immunity is long-lasting. Go to the next page if you knew the correct answer, or click the link image(s) below to further research the concepts in this question (if desired).
Research Concepts: Varicella Zoster (Chickenpox)
We update eBooks quarterly and Apps daily based on user feedback. Please tap flag to report any questions that need improvement.
Question 639:
A 74-year-old male with no medical comorbidities and a Karnofsky performance status of 90 is found to have relapsed symptoms of fatigue, weakness, and splenomegaly after he was initially treated with cladribine. A blood test is done, which shows hemoglobin 11 g/dL, platelets100000/mcL, and ANC levels of 900/mcL. Peripheral picture of the blood done shows characteristic-appearing mononuclear cells, which are typically large with circumferential hair-like cytoplasmic projections and round, well-defined nucleus. Which of the following is the best treatment choice in this patient?
Choices: 1. 2. 3. 4.
Brentuximab vedotin Induction chemotherapy with idarubicin and cytarabine Re-trial of cladribine Ibrutinib
Answer: 3 - Re-trial of cladribine Explanations: Hairy cell leukemia (HCL) is a relatively rare chronic Bcell malignancy that involves the bone marrow, spleen, and peripheral blood. The complete blood count may reveal pancytopenia, including monocytopenia. The median age at diagnosis is approximately 55. Affected patients often have non-specific symptoms, including fatigue and weakness, as well as symptoms related to cytopenias and splenomegaly. Eighty percent of patients will have significant cytopenias on presentation, with severe pancytopenia in less than 10%. While splenomegaly is a predominant feature, massive, symptomatic splenomegaly is less frequent, perhaps due to earlier detection on routine complete blood count (CBC) Diagnosis is achieved by studies on peripheral blood, including flow cytometry and review of the peripheral smear, along with bone marrow biopsy. A “dry tap,” or, inability to perform bone marrow aspiration is frequently encountered with hairy cell leukemia although not with HCL-V. The hairy cell has characteristic-appearing mononuclear cells which are typically large with circumferential hair-like cytoplasmic projections and around, well-defined nucleus. Cladribine and pentostatin, two purine analogs, are the first-line treatment for hairy cell leukemia. They are equally effective in inducing and maintaining remission. Nonetheless, cladribine is regarded as first-line chemotherapy among most hematologists due to its favorable toxicity profile. A single course of cladribine induces a complete response in approximately 90% of
individuals after one cycle (5 to 7 days). It is important to note that these patients will be immunosuppressed for many months following therapy, and attention to possible infectious complications is critical. A recent meta-analysis conducted by Andrasiak et al. showed that the most effective therapy in treatment-naive patients and the first relapse was cladribine with rituximab maintenance. Vemurafenib was also shown to be similarly efficacious; however, it is usually used for refractory or progressive cases. Go to the next page if you knew the correct answer, or click the link image(s) below to further research the concepts in this question (if desired).
Research Concepts: Hairy Cell Leukemia
We update eBooks quarterly and Apps daily based on user feedback. Please tap flag to report any questions that need improvement.
Question 640:
A 42-year-old patient comes to the clinic for her annual appointment. She currently feels well. Her vital signs are within normal limits. She is up to date on her screenings and vaccinations. She does not smoke or take illicit drugs. She drinks a glass of wine every day before dinner. She is sexually active and takes oral contraceptive pills. She has 3 children through normal vaginal deliveries, who are all healthy. Family history is insignificant. Past medical problems include anxiety and obesity. Which of the following is a protective factor in this patient for the development of ovarian cancer?
Choices: 1. 2. 3. 4.
Multiparity Obesity Age Sexual activity
Answer: 1 - Multiparity Explanations: Ovarian cancer is the second most common gynecological cancer and the leading cause of death in the United States. They can either be acute or subacute. The given patient has multiple risk factors for ovarian cancer. Multiparity is a protective factor against ovarian cancer. Multiparity decreases ovulatory cycles which in turn protects against the development of ovarian cancer. Breastfeeding and oral contraceptives have similar effects. Subacute cases may present with non-specific symptoms such as abdominal fullness, bloating, vague pelvic or abdominal pain. Acute cases present with pleural effusion, venous thrombosis, and ascites. Increasing age, alcohol intake, obesity, genetic predispositions are relevant and important factors in the development of ovarian cancer. Sexual activity has no association with ovarian cancer. Go to the next page if you knew the correct answer, or click the link image(s) below to further research the concepts in this question (if desired).
Research Concepts: Ovarian Cancer
We update eBooks quarterly and Apps daily based on user feedback. Please tap flag to report any questions that need improvement.
Question 641:
A 58-year-old female presents to the clinic with complaints of abdominal bloating, indigestion, and post-prandial fullness for four months. She has a history of ER, PR, HER2NEU –ve ductal left-sided breast carcinoma ten years back, and was treated with a bilateral mastectomy and systemic chemotherapy. BRCA 1 mutation was positive and bilateral salpingo-oophorectomy was done. The last PET scan was done two years back and showed no tumor growth. Stool examination was done two months back, which showed no cells and cysts, Helicobacter Pylori antigen test was negative and occult blood was not seen. Esophagogastroduodenoscopy (EGD) done at the time showed mild gastritis. She has been using OTC antacids for the last year. Ultrasound is advised, which demonstrates the presence of ascites. CT scan shows multiple enhanced lesions on the peritoneum. Diagnostic peritoneal tap’s cytology reveals the presence of malignant cells. Diagnostic laparoscopy is advised, and biopsies of the masses are taken. What result will most likely be seen on biopsy?
Choices: 1. Presence of dense diffuse infiltrate of marginal-zone cells in lamina propria with prominent lymphoepithelial lesions 2. Presence of central, optically clear, globoid droplet of cytoplasmic mucin with an eccentrically placed nucleus 3. Presence of mesothelial cells in a tubulo-papillary pattern with increased mitotic figures interspersed with spindle cells 4. Presence of papillary growth with fibrotic stroma, nuclear atypia, and psammoma bodies
Answer: 4 - Presence of papillary growth with fibrotic stroma, nuclear atypia, and psammoma bodies
Explanations: Peritoneal cancer is either primary (origin within the peritoneal cavity) or metastatic from some other viscera. Extraovarian primary peritoneal carcinoma (EOPPC) is the most common sub-type of primary peritoneal carcinoma and resembles serous ovarian carcinoma. This is due to the same 'Mullerian origin 'of the germinal epithelium of ovaries and mesothelial cells of the peritoneum (from the same embryonal mesoderm). It occurs exclusively in women (mean age, 56-62 years) and only in small cases in males. It can be caused by germline mutations in the BRCA 1 gene, which has been reported in 17.6% of cases. There is a set criterion for its diagnosis, defined by Gynecologic Oncology Group, as follows: i. Both ovaries should be of normal size and the enlargement, if present, is benign. ii. The malignant involvement of the surface area of extra ovarian sites should be larger than that of either of the ovaries. iii. There should be no malignancy in either ovaries or the tumor in serosa, and cortex of size less than 5X5 mm can be present. iv. The tumor in the extra ovarian site should be serosal type both histologically and cytologically. In the given scenario, the patient had already undergone bilateral salpingo-oophorectomy. Therefore EOPPC is the likely diagnosis. In both metastatic ovarian and primary peritoneal carcinoma, gross, histopathological, and immunohistochemical findings are the same. Histologically, psammoma bodies are characteristic of this tumor.
Other types of primary tumors are malignant peritoneal mesothelioma. MPM shows the presence of mesothelial cells in a tubulo-papillary pattern with increased mitotic figures interspersed with spindle cells. The presence of dense diffuse infiltrate of marginal-zone cells in lamina propria with prominent lymphoepithelial lesions is seen in MALT lymphoma, which can be seen in patients with a long term history of gastritis and is related to the presence of Helicobacter Pylori. The presence of central, optically clear vacuoles of cytoplasmic mucin with an eccentrically placed nucleus in seen signet ring cell gastric cancer. All three of them can present with a similar presentation of vague symptoms. However, BRCA 1 mutation is associated with EOPPC and serous ovarian carcinoma. Go to the next page if you knew the correct answer, or click the link image(s) below to further research the concepts in this question (if desired).
Research Concepts: Peritoneal Cancer
We update eBooks quarterly and Apps daily based on user feedback. Please tap flag to report any questions that need improvement.
Question 642:
A 65-year-old male presents to the clinic with drooping of his left eyelid. There is no history of trauma or diurnal variation of ptosis. The patient has a past medical history significant for hypertension and diabetes. He has a smoking history of 30 pack years. His initial laboratory analysis is unremarkable. On examination, there are decreased breath sounds in the middle lobe of the left lung. His visual acuity is 6/30 in both eyes. An asymmetry is noted with the left eyelid 2 mm below as compared to the right eyelid. An x-ray of the left lung reveals a centrally located mass in the left lung. What is the next step?
Choices: 1. Immediately book the patient for ptosis surgery 2. Order an immediate MRI looking for a brain tumor 3. Examine the pupils to see if Horner syndrome or third nerve palsy is present 4. Anti–striated muscle antibody looking for myasthenia gravis
Answer: 3 - Examine the pupils to see if Horner syndrome or third nerve palsy is present
Explanations: Asymmetric ptosis can be a sign of a life-threatening condition such as posterior communicating artery (PCA) aneurysm or Horner syndrome. Pupillary asymmetry can give away the diagnosis. Always look at the pupils. A ptotic eyelid with a smaller pupil indicates Horner syndrome. This can be caused by a dissecting carotid aneurysm or a lung tumor. These need to be ruled out immediately. It is important to evaluate the patient to ensure that the dehiscence of the levator is the cause of the ptosis before attempting a repair. If myasthenia gravis is considered, order acetylcholine antibodies before an anti-acetylcholine receptor antibody test is performed. Go to the next page if you knew the correct answer, or click the link image(s) below to further research the concepts in this question (if desired).
Research Concepts: Blepharoplasty Ptosis Surgery
We update eBooks quarterly and Apps daily based on user feedback. Please tap flag to report any questions that need improvement.
Question 643:
After surgery of an intraventricular tumor, the pathologist reports a spindle-shaped tumor cell, with narrow rod-shaped nuclei, and abundant collagenous or reticulum background. On immunostaining, it showed positive for epithelial membrane antigen, positive for Vimentin, and positive for S100 protein. What is the most likely genetic mutation of the mass mentioned above?
Choices: 1. 2. 3. 4.
1p19q mutation IDH-1/2 wild type Chromosome 17 mutation Chromosome 22 mutation
Answer: 4 - Chromosome 22 mutation Explanations: The most common chromosomal mutation of all meningiomas is chromosome 22, specifically with merlin tumor suppressor gene, SIS oncogene, and INI1. The most prevalent subtype of intraventricular meningioma is a fibrous meningioma. On immunostaining, fibrous meningiomas typically present with epithelial membrane antigen/vimentin/S100 positive. S100 protein positivity is more common in fibrous meningiomas, compared to other subtypes. 1p19q is seen in oligodendrogliomas, IDH wildtype is seen in primary glioblastomas, and chromosome 17 is associated with the development of neurofibromas. Go to the next page if you knew the correct answer, or click the link image(s) below to further research the concepts in this question (if desired).
Research Concepts: Intraventricular Meningioma
We update eBooks quarterly and Apps daily based on user feedback. Please tap flag to report any questions that need improvement.
Question 644:
A 67-year-old man with a 50-year pack history of smoking presents for an initial oncologic visit. He was recently diagnosed with a tumor in one of the minor salivary glands in the oral cavity. CT scan of the head and neck shows a 2 cm localized tumor without extension into neighboring structures. Analysis of the biopsy specimen shows biphasic tissue composed of myoepithelial and ductal cells. The patient inquires about his prognosis. Which of the following is the most appropriate answer from the clinician?
Choices: 1. The tumor usually cannot be removed surgically because of high vascularity, and the prognosis is generally very poor. 2. The tumor is generally slow-progressing with a 5-year survival of approximately 75%. 3. Systemic chemotherapy is the mainstay of treatment, and the prognosis depends on the tumor's response to it. 4. This is a benign tumor and will not negatively impact lifespan.
Answer: 2 - The tumor is generally slow-progressing with a 5-year survival of approximately 75%.
Explanations: The most prevalent malignant minor salivary gland tumor is adenoid cystic carcinoma. The highest incidence of malignant salivary gland tumors is found in the submandibular glands. Adenoid cystic carcinoma is generally well-differentiated and slow-growing; therefore, even patients with metastatic disease may survive for years with the average 5-year survival of approximately 75%. Treatment is primarily surgical, but complete resection can be difficult given that it tends to grow along nerves (not because of high vascularity). A delay in diagnosis and treatment is associated with a poorer prognosis. Unclear surgical margins after resection of the primary tumor, despite postoperative radiation therapy, is the single most important risk factor for local recurrence. Go to the next page if you knew the correct answer, or click the link image(s) below to further research the concepts in this question (if desired).
Research Concepts: Adenoid Cystic Cancer
We update eBooks quarterly and Apps daily based on user feedback. Please tap flag to report any questions that
need improvement.
Question 645:
A 39-year-old man is found to have a 2.5 cm thyroid nodule seen on ultrasound of the neck. Fineneedle aspiration reveals follicular cells. Which of the following is the next best step in the management of this patient?
Choices: 1. 2. 3. 4.
Thyroid uptake scan Lobectomy and isthmusectomy Total thyroidectomy Total thyroidectomy with lymph node dissection
Answer: 2 - Lobectomy and isthmusectomy Explanations: If a nodule reveals follicular cells, it is imperative to perform a lobectomy and isthmusectomy to rule out follicular carcinoma. Follicular carcinoma is the diagnosis if there is invasion of the capsule. A thyroid uptake scan will not help in assessing how active the nodule is in this case. At this point, the biopsy already shows follicular cells. Carcinoma must be ruled out. Total thyroidectomy is too aggressive and as is a lymph node dissection. Ipsilateral thyroidectomy is recommended depending on the location of the thyroid nodule. Go to the next page if you knew the correct answer, or click the link image(s) below to further research the concepts in this question (if desired).
Research Concepts: Thyroid Adenoma
We update eBooks quarterly and Apps daily based on user feedback. Please tap flag to report any questions that need improvement.
Question 646:
A 5-year-old girl with a history of hypertension treated with lisinopril is scheduled for induction of chemotherapy with high-dose cyclophosphamide for a previously resected Wilms tumor. What prophylactic regimen for chemotherapy-induced nausea and vomiting is most appropriate for this patient?
Choices: 1. An intravenous 5-HT3 antagonist and intravenous glucocorticoid one hour before the cyclophosphamide infusion 2. An intravenous or oral neurokinin-1 antagonist one hour before the cyclophosphamide infusion 3. An intravenous or oral neurokinin-1 antagonist one hour before the cyclophosphamide infusion with an intravenous 5-HT3 antagonist or intravenous glucocorticoid as a rescue therapy 4. An intravenous or oral neurokinin-1 antagonist, intravenous 5-HT3 receptor antagonist, and an intravenous glucocorticoid one hour before the cyclophosphamide infusion
Answer: 4 - An intravenous or oral neurokinin-1 antagonist, intravenous 5-HT3 receptor antagonist, and an intravenous glucocorticoid one hour before the cyclophosphamide infusion
Explanations: The Multinational Association of Supportive Care in Cancer/European Society for Medical Oncology (MASCC/ESMO), the American Society of Clinical Oncology (ASCO), and the National Comprehensive Cancer Network (NCCN) regularly update their recommendations for CINV in both adult and pediatric patients. High emetogenic chemotherapy agents are those defined as having a greater than 90% chance of developing CINV without prophylaxis. The current guidelines recommend combination therapy with a 5HT3 antagonist, a glucocorticoid, and an NK-1 receptor antagonist. Aprepitant and fosaprepitant are neurokinin-1 antagonists FDA approved for the prevention of CINV in pediatric patients over the age of 6 months. Weightbased dosing should be used below if the patient is below 30 kilograms or 12 years of age. A combination of antiemetics with different mechanisms of action is superior in prophylaxis and treatment of nausea and vomiting compared to single or multiple agents in the same therapeutic class. In general, patient satisfaction is higher when antiemetic prophylaxis is administered when compared to rescue therapy after emesis has already. Go to the next page if you knew the correct answer, or click the link image(s) below to further research the concepts in
this question (if desired).
Research Concepts: Aprepitant
We update eBooks quarterly and Apps daily based on user feedback. Please tap flag to report any questions that need improvement.
Question 647:
A 63-year-old man is being evaluated for progressive vision and mental status changes over the past six months. Head CT shows a single mass suspicious for cancer. The mass is removed, and genetic analysis is performed. It is discovered that its DNA and histones proteins both underwent epigenetic changes, causing increased expression of multiple oncogenes. Which of the following best describes the most likely changes to the charge of the DNA and histone proteins associated with the oncogenes?
Choices: 1. 2. 3. 4.
More More More More
positive DNA and histones negative DNA and histones positive DNA and more negative histones negative DNA and more positive histones
Answer: 3 - More positive DNA and more negative histones
Explanations: DNA is negatively charged, and histones are positively charged because of lysine and arginine content. Interactions between negative DNA and positive histones decrease the accessibility to the DNA by transcriptional enzymes. This means the DNA-histone interactions are strong, and the DNA is tightly bound. By increasing the charge difference between the DNA and histones, the DNA becomes less accessible. Decreasing the charge difference will loosen the interactions making the DNA more accessible. In this case with oncogenes, they are likely to be expressed more in tumors. Therefore they must be more accessible by transcriptional enzymes. To do this, the charge difference between the DNA and histones is decreased, which occurs by the DNA becoming more positive and the histones becoming more negative. Go to the next page if you knew the correct answer, or click the link image(s) below to further research the concepts in this question (if desired).
Research Concepts: Genetics, DNA Packaging
We update eBooks quarterly and Apps daily based on user feedback. Please tap flag to report any questions that need improvement.
Question 648:
A 67-year-old man presents with fatigue, abdominal discomfort, and early satiety. He reports losing 3.6 kg in the past four weeks. His past medical history is notable for osteoarthritis, hypertension, and gastroesophageal reflux disease (GERD). His medications include amlodipine 10 mg daily, omeprazole 20 mg daily, and paracetamol as needed for joint pain. He has a 40-packyear history of smoking. He denies alcohol or illicit drug use. His vital signs show oxygen saturation of 98% on room air, respiratory rate of 20 per minute, heart rate of 81 bpm, blood pressure of 133/78 mmHg, and temperature of 36.7 C (98.1 F). On examination, there are equal breath sounds in both lung fields. The liver is palpable 3 cm below the costal margin, and splenomegaly is noted. There is no lymphadenopathy. An initial set of labs shows hemoglobin 10.4 g/dL (11-14), white blood cells 44000 per microL (410010900), and platelets 490000/microL (150000-400000). Given the likely diagnosis, which of the following additional findings is most likely to be found in this patient?
Choices: 1. BCR-ABL1 fusion gene 2. t(15;17) chromosomal translocation 3. Smudge cells on the peripheral smear 4. Toxic granulation in the neutrophils and high leukocyte alkaline phosphatase (LAP score)
Answer: 1 - BCR-ABL1 fusion gene Explanations: The patient is presenting with signs and symptoms of chronic myeloid leukemia (CML). These patients usually present with malaise, fatigue, night sweat, abdominal discomfort, and early satiety secondary to splenomegaly. Patients can experience weight loss as a result of that, as seen in this patient. The diagnosis is confirmed by the demonstration of the Philadelphia chromosome, BCR-ABL1 fusion gene. T(15;17) chromosomal translocation is found in patients with acute promyelocytic leukemia, not CML. Smudge cells on peripheral smear are suggestive of chronic lymphocytic leukemia. Demonstration of toxic granulation in the neutrophils and high leukocyte alkaline phosphatase (LAP score) is consistent with leukemoid reaction. Leukemoid reaction is defined by the elevation of leukocyte count of 50,000 or more from reasons other than leukemia. Infections are the most common underlying cause. The peripheral smear typically shows toxic granulation and a high LAP score. Go to the next page if you knew the correct answer, or click the link image(s) below to further research the concepts in this question (if desired).
Research Concepts: Neutrophilia
We update eBooks quarterly and Apps daily based on user feedback. Please tap flag to report any questions that need improvement.
Question 649:
A 25-year-old nursing mother undergoes thyroidectomy for a mass. Pathologic exam demonstrates papillary thyroid cancer, 3 cm in largest dimension, lymphovascular invasion, and one positive lymph node. She is prescribed a therapeutic pill. When can the patient resume breastfeeding?
Choices: 1. 2. 3. 4.
2 to 3 months after treatment No need to stop breastfeeding Discontinue nursing the current infant A week after treatment
Answer: 3 - Discontinue nursing the current infant Explanations: As radioactive iodine is secreted through breast milk, patients should stop breastfeeding the current infant. Managing high-risk papillary thyroid cancer involves total thyroidectomy followed by radioactive iodine treatment. At least six weeks before RAI treatment, breastfeeding should be stopped. It is safe to breastfeed after future pregnancies. Go to the next page if you knew the correct answer, or click the link image(s) below to further research the concepts in this question (if desired).
Research Concepts: Radioactive Iodine For Thyroid Malignancies
We update eBooks quarterly and Apps daily based on user feedback. Please tap flag to report any questions that need improvement.
Question 650:
A 16-year-old boy originally from Bangladesh presents with his parents, as he has a 1.5 cm blue nodule on his scalp. This nodule has been present for the past three years but they have not sought medical attention until now. A biopsy report describes heavilypigmented spindle cells alternating with clear cells. How should the patient and his parents be counseled?
Choices: 1. The lesion is malignant and required immediate excision 2. The lesion is benign but can only be removed by cryotherapy 3. The patient should return for review if the nevus enlarges or otherwise changes 4. The nevus will respond well to intralesional corticosteroid treatment
Answer: 3 - The patient should return for review if the nevus enlarges or otherwise changes
Explanations: The cellular blue nevus is a benign lesion. The incidence of malignant melanoma arising from these lesions is rare, but patients should be aware to seek medical advice if the lesion changes in size or character. Incidence is highest among patients of Asian descent and usually appears during the second decade. Histologically they are encapsulated but may extend into the subdermis. Heavily-pigmented spindle cells alternate with clear cells. Multiple blue nevi may indicate a rare genetic condition known as the Carney complex. Patients with Carney complex commonly have at least one blue nevus. These lesions can be left alone or excised. They do not respond to cryotherapy or corticosteroid injections. Go to the next page if you knew the correct answer, or click the link image(s) below to further research the concepts in this question (if desired).
Research Concepts: Blue Nevus
We update eBooks quarterly and Apps daily based on user feedback. Please tap flag to report any questions that need improvement.
Question 651:
A 58-year-old hemiplegic male who had a long history of an indwelling foley catheter underwent cystoscopy for microscopic hematuria and was found to have exophytic mucosal projections on fibrous stalks. Histology of the lesion is done, which revealed inflammatory cells covered by normal-appearing epithelial cells. Which of the following is the management plan for this patient?
Choices: 1. 2. 3. 4.
Bacillus Calmette-Guerin (BCG) Trimethoprim/sulfamethoxazole Ciprofloxacin Transuretheral resection
Answer: 4 - Transuretheral resection Explanations: Papilloma is a term describing tumors with a cauliflowerlike or finger-like appearance. They often have a central vascular core and are lobulated. No matter what the cytology, epithelial neoplasms with this morphology are called papillomas. Papillomas may be benign or malignant. They can occur in many tissues, including conjunctiva, skin, gastrointestinal, breast duct, or cervix. Cystoscopy is the diagnostic procedure of choice. On cystoscopy, inverted urothelial papilloma of the bladder appears as a pedunculated or sessile mass with a relatively obvious smooth surface, or as a polypoid/papillary tumor with a smooth surface, the diameter of the mass is less than 3 cm in most cases but can sometimes be much larger, with a diameter of up to 8 cm and generally, it is a single lesion, but can rarely be multiple. Since inverted urothelial papillomas of the bladder show no tendency to infiltration, their treatment involves complete transurethral resection. Inverted urothelial papillomas of the upper urinary tracts are even less common than bladder lesions. However, when the upper urinary tracts are involved, the lesions tend to be sizeable. Treatment of smaller upper tract inverted urothelial papillomas can be with ureteroscopy, but larger lesions may require percutaneous access for direct resection, partial ureterectomy, or even nephrectomy.
Go to the next page if you knew the correct answer, or click the link image(s) below to further research the concepts in this question (if desired).
Research Concepts: Inverted Urothelial Papilloma
We update eBooks quarterly and Apps daily based on user feedback. Please tap flag to report any questions that need improvement.
Question 652:
A 44-year-old African American male patient with a past medical history significant for end-stage renal disease secondary to diabetes mellitus received a deceased donor kidney transplant 14 months ago and had an uneventful post-transplant course with a baseline creatinine of 1.2. His immunosuppression regimen includes tacrolimus, mycophenolate mofetil, and prednisone. He presents with complaints of unintentional weight loss of about 10 pounds (4.5 kg) and night sweats for one month. On examination, his vitals are stable, and physical examination reveals palpable lymph nodes in the right axillary region and palpable spleen. Imaging with PET-CT reveals increased activity in the lymph nodes and splenomegaly. What is the treatment of choice for the underlying disorder?
Choices: 1. Decrease immunosuppression and monitor 2. Rituximab and radiation therapy 3. Chemoradiation and decrease immunosuppressive therapy 4. Chemotherapy, rituximab, and decrease immunosuppressive therapy
Answer: 4 - Chemotherapy, rituximab, and decrease immunosuppressive therapy
Explanations: Following solid organ transplantation, there is a risk of post-transplant lymphoproliferative disease (PTLD) due to immunosuppressive (IS) therapy. PTLD within one year of transplant is B-cell mediated and likely due to Epstein-Barr Virus (EBV). It typically responds to decreasing the IS therapy. For patients who progress despite reducing the IS regimen, rituximab could be tried. PTLD after one year of the transplant is T cell or NK cell-mediated, and it does not respond well to decreasing the IS therapy. Monomorphic PTLD is the most common type, resembling B-cell lymphoma. Treatment involves the withdrawal of IS therapy along with rituximab +/chemotherapy. Localized polymorphic PTLD is treated with radiation alone +/- rituximab when there is no response to the withdrawal of IS therapy. Systemic polymorphic PTLD is treated with decreasing IS along with rituximab +/chemotherapy. Go to the next page if you knew the correct answer, or click the link image(s) below to further research the concepts in this question (if desired).
Research Concepts: Diffuse Large B Cell Lymphoma
We update eBooks quarterly and Apps daily based on user feedback. Please tap flag to report any questions that need improvement.
Question 653:
A 35-year-old man with a family history of Von Hippel Lindau disease presents to the clinic with a 4day history of flank pain and discolored urine. He also noticed a 10-pound (4.5 kg) unintentional weight loss over the past two months. He was diagnosed with advanced renal cell carcinoma (RCC). The specialist prescribes a multitarget tyrosine kinase inhibitor. For which of the following sets of malignancies is this agent also most appropriate?
Choices: 1. Liposarcoma and radioactive iodine-refractory differentiated thyroid cancer 2. Radioactive iodine-refractory differentiated thyroid cancer and unresectable hepatocellular carcinoma 3. Unresectable hepatocellular carcinoma and melanoma 4. Lymphomas and melanoma
Answer: 2 - Radioactive iodine-refractory differentiated thyroid cancer and unresectable hepatocellular carcinoma
Explanations: Lenvatinib is FDA-approved for radioactive iodinerefractory differentiated thyroid cancer (DTC), unresectable or advanced hepatocellular carcinoma (HCC), and advanced renal cell carcinoma (RCC). Lenvatinib exerts its mechanism of action via inhibition of multiple receptors of tyrosine kinases: VEGFR-1 (FLT1), VEGFR-2( KDR), VEGFR-3 (FLT4), FGFR-1, FGFR-2, FGFR-3, FGFR-4, PDGFRa, RET, and c-KIT. The inhibition of the VEGF receptors prevents tumor angiogenesis, and the inhibition of FGFR, RET, PDGFRa, and KIT prevent further proliferation of malignant cells. The concurrent inhibition of both receptor pathways results in the inhibition of nuclear signal transduction and concomitant suppression of the activity of factors involved in tumor growth. Lenvaitnib is available in 4 mg and 10 mg capsule dosages for oral consumption. Go to the next page if you knew the correct answer, or click the link image(s) below to further research the concepts in this question (if desired).
Research Concepts: Lenvatinib
We update eBooks quarterly and Apps daily based on user feedback. Please tap flag to report any questions that need improvement.
Question 654:
A 9-year old boy is brought to the clinic with swelling on the anterior aspect of the left thigh for 6 months. On examination, swelling is firm and mobile in one plane. MRI reveals dark T1 images and bright T2 mass. Histology reveals round cells with multinucleated giant cells. What is the most likely diagnosis?
Choices: 1. 2. 3. 4.
Giant cell tumor Rhabdomyosarcoma Ewing sarcoma Liposarcoma
Answer: 2 - Rhabdomyosarcoma Explanations: Rhabdomyosarcoma occurs mostly before 10 years of age. It originates from a muscle; therefore, mobility is mostly In one plane. Round cells with multinucleated giant cells are surrounded by dense fibrous septae on histology. Wide surgical excision with chemotherapy is the treatment of choice. Bony tumors are not mobile. Rhabdomyosarcoma is dark on the T1 image, while bright on the T2 image. Go to the next page if you knew the correct answer, or click the link image(s) below to further research the concepts in this question (if desired).
Research Concepts: Rhabdomyosarcoma
We update eBooks quarterly and Apps daily based on user feedback. Please tap flag to report any questions that need improvement.
Question 655:
A 65-year-old male presents to the clinic for evaluation of malaise, vague abdominal pain, and loss of appetite for six months. His past medical history is significant for hypertension, diabetes mellitus, and hyperlipidemia. His family history is notable for his brother dying from complications of pancreatic carcinoma. His blood pressure is 150/90 mmHg, pulse rate is 90/min, and the temperature is 99 F. On physical examination, there is the presence of a mass in the right upper quadrant, and yellowish discoloration of the sclera is noted. CT scan of the abdomen shows a large heterogeneous mass involving the head of the pancreas and the second part of the duodenum. An imaging-guided biopsy reveals sheets of malignant acinar cells with no intervening ductal structures or islets. The specimen is positive for trypsin and cytokeratin and negative for CD117 and synaptophysin. Which of the following is the appropriate management step for this patient?
Choices: 1. 2. 3. 4.
Radiation Chemotherapy Surgical resection Cryotherapy
Answer: 2 - Chemotherapy Explanations: The histopathological findings are suggestive of metastatic pancreatic acinar cell carcinoma. It is treated with chemotherapy, and various protocols have been developed which have improved the overall survival. The initial adjuvant chemotherapy for metastatic carcinoma is with 5-fluorouracil, leucovorin, and oxaliplatin. Some of the common side effects include hair loss, headaches, sores in the mouth, diarrhea, fatigue, and numbness and tingling in extremities. The more adverse rare side effects include kidney injury and scarring of lungs. It is important to discuss the side effects of chemotherapy with the patient before starting the therapy. Recent studies have highlighted the use of combination chemotherapy with platinum-based analogs, which have improved the chemosensitivity. Go to the next page if you knew the correct answer, or click the link image(s) below to further research the concepts in this question (if desired).
Research Concepts: Case Study: 24-Year-Old Male Presenting With Polyarthralgias
We update eBooks quarterly and Apps daily based on user feedback. Please tap flag to report any questions that need improvement.
Question 656:
A 55-year-old man with a history of stable chronic myelogenous leukemia (CML) is admitted to the hospital with suspected neutropenic fever. His vital signs show blood pressure 96/64 mmHg, pulse 98/min, and temperature 39 C. Urinalysis demonstrates >30 white blood cells and 3+ bacteria. He is given appropriate antibiotics and fluid resuscitation. The patient's blood pressure normalizes with fluid and antibiotics, and his fever abates. The following day, the patient is noted on morning rounds to be obtunded and jaundiced. Repeat lab work reveals newly developed transaminitis, worsening pancytopenia, and a rising creatinine. A serum ferritin level is 10,000 mcg/L. Which of the following bone marrow biopsy findings is most likely to confirm the diagnosis in this patient?
Choices: 1. 2. 3. 4.
Blast count >20% Myelofibrosis Hemophagocytosis Ringed sideroblasts
Answer: 3 - Hemophagocytosis Explanations: Hemophagocytic lymphohistiocytosis (HLH) is a hyperinflammatory disorder characterized by macrophages activation and destruction of bone marrow. Although originally described and elucidated in children, HLH can also occur in adults with some estimates as high as 1:2000 adult admissions to the ICU. The most common triggers for developing HLH in adults is an infection, followed closely by hematologic malignancies. This patient with a history of CML and presents with a complicated UTI. Despite appropriate upfront treatment, they go on to develop acute multiorgan failure that cannot be easily explained by either a resistant UTI or worsening of his CML. Extremely elevated ferritin levels can be seen in patients who develop HLH. Serum levels >500 mcg/L are part of the HLH-2004 diagnostic criteria; however, levels >10,000 mcg/L have been shown to be 90% sensitive and 98% specific in a Texas Children's Hosptial cohort of pediatric cases. Go to the next page if you knew the correct answer, or click the link image(s) below to further research the concepts in this question (if desired).
Research Concepts: Lymphohistiocytosis
We update eBooks quarterly and Apps daily based on user feedback. Please tap flag to report any questions that need improvement.
Question 657:
A 65-year-old male presents with red urine for the last 5 days. He denies any dysuria, urgency, or hesitancy. His past medical history is significant for hypertension and type 2 diabetes mellitus. He has been a smoker for the last 40 years, smoking 1 pack a day. His vital signs include a blood pressure of 142/89 mmHg, a heart rate of 80 beats per minute, and a temperature of 37.6 C (99.6 F). A physical exam reveals mild tenderness in the left side of the abdomen. No signs of scrotal swelling or masses and no lesions are seen on the penis or scrotum. Urinalysis shows 1-2 white blood cells per high power field, many red blood cells per high power field, negative leukocyte esterase, and negative nitrites. A computed tomography (CT) scan shows a solid mass within the left kidney with significant enhancement. Which of the following findings is associated with the most probable diagnosis?
Choices: 1. 2. 3. 4.
Sodium level of 125 mEq/L History of tuberous sclerosis Bladder calcifications Hemoglobin of 18 g/dL
Answer: 4 - Hemoglobin of 18 g/dL Explanations: Gross hematuria, flank tenderness, and a renal mass on a CT scan are all indicating renal cell carcinoma (RCC). RCC is associated with some paraneoplastic syndrome and polycythemia is one of them. Smoking increases the risk of renal cell carcinomas as well as urothelial cancers. The presence of a renal cell adenocarcinoma is generally silent. The classic triad of flank pain, hematuria, and flank mass are actually uncommon. Common presenting complaints include gross hematuria, weight loss, fever, hypertension, and flank pain. A CT scan is the imaging study of choice and can help stage the tumor. A CT scan can also differentiate cystic from solid lesions. Hyponatremia can be associated with lung small cell carcinoma (ADH secretion). Bladder calcifications can be associated with many conditions including schistosomiasis. Red blood cell casts and dysmorphic cells can be found in glomerulonephritis, not in renal cell carcinoma. Tuberous sclerosis is associated with angiomyolipomas. Go to the next page if you knew the correct answer, or click the link image(s) below to further research the concepts in this question (if desired).
Research Concepts: Renal Cell Cancer
We update eBooks quarterly and Apps daily based on user feedback. Please tap flag to report any questions that need improvement.
Question 658:
A 60-year-old male with a past medical history of HIV was seen by the primary care provider after presenting anal pain for 6 months and associated bleeding per rectum. He is otherwise in very good health and compliant with his antiretroviral therapy. On the anal exam, there is a concern for a mass. The patient was further evaluated by anoscopy and found to have a 2.5 cm mass in the anal canal, the biopsy of which turned out to be a squamous cell carcinoma, which is poorly differentiated. Staging studies did not show any lymph node involvement or other metastasis. The patient was treated with first-line therapy with chemo-radiation and completed therapy successfully. The patient was evaluated by an anoscopy at 12 weeks post-therapy, which showed a significant response but still some persistent disease. Which of the following is an appropriate therapy recommended for him?
Choices: 1. 2. 3. 4.
Surgery Radiation Chemotherapy with 5-FU plus mitomycin Serial exams
Answer: 4 - Serial exams Explanations: Serial exams would be appropriate for this patient, as it is likely that they may have a regression of disease on a serial exam. Anal cancers regress slowly over many weeks after the chemoradiation, sometimes up to 26 weeks following therapy. If, on the serial exams, they show persistent disease, then it is appropriate to consider surgery at that time. Early surgery will cause significant morbidity as some of these patients may be able to avoid surgery. Go to the next page if you knew the correct answer, or click the link image(s) below to further research the concepts in this question (if desired).
Research Concepts: Anal Cancer
We update eBooks quarterly and Apps daily based on user feedback. Please tap flag to report any questions that need improvement.
Question 659:
A 17-year-old male with osteosarcoma presents with oral pain, ulcerations, and erosions following recent high-dose chemotherapy. On examination, the oral mucosa appears erythematous, and a white fibrinous pseudomembrane covers the ulcers. Further inquiry reveals the pain has not affected his overall eating capacity, except for avoiding hard food. What is the most appropriate pain management therapy for this patient?
Choices: 1. 2. 3. 4.
Morphine 0.2% mouth rinse Magic mouthwash Chlorhexidine mouth wash Sucralfate topical and systemic
Answer: 1 - Morphine 0.2% mouth rinse Explanations: Oral mucositis is a debilitating condition characterized by erythema, edema, and ulceration of the oral mucosa with subsequent pain, which may affect oral intake. Oral mucositis is a complication of radiation therapy (RT) to the head and neck, chemotherapy, chemoradiotherapy, and hematopoeitic stem cell transplantation (HSCT). Morphine 0.2% mouth rinse is recommended by the MASCC/ISOO guidelines for managing pain in head and neck cancer patients receiving radiotherapy or chemotherapy. Morphine 0.2% mouth rinses are more effective in managing pain associated with oral mucositis than magic mouthwash rinses. The MASCC/ISOO guidelines do not recommend sucralfate topical or systemic for the prevention or treatment of oral mucositis. Go to the next page if you knew the correct answer, or click the link image(s) below to further research the concepts in this question (if desired).
Research Concepts: Oral Mucositis
We update eBooks quarterly and Apps daily based on user feedback. Please tap flag to report any questions that
need improvement.
Question 660:
A 25-year-old female is brought to the outpatient clinic with the complaint of one episode of focal seizures involving the right side of the body a day back. On examination, she is neurologically intact. She undergoes a magnetic resonance imaging scan of the brain, which shows a T1-hypo and T2-hyperintense lesion involving the left frontal lobe with mild mass effect and blooming on T2-star images. She undergoes craniotomy and decompression of the lesion. Loss of which of the following chromosomes on histopathological examination of the specimen is associated with increased patient survival?
Choices: 1. 2. 3. 4.
1p 1q 10p 10q
Answer: 1 - 1p Explanations: The imaging features are suggestive of oligodendroglioma. Oligodendrogliomas are usually associated with a combined loss of 1p and 19q chromosomes. A loss of 1p or a combined loss of 1p and 19q is associated with longer survival. The other chromosome losses have not been found to be associated with survival in oligodendrogliomas. Go to the next page if you knew the correct answer, or click the link image(s) below to further research the concepts in this question (if desired).
Research Concepts: Gliomas
We update eBooks quarterly and Apps daily based on user feedback. Please tap flag to report any questions that need improvement.
Question 661:
A 55-year-old patient presents with worsening jaundice, pruritus, hepatic encephalopathy, ascites, palpable mass in the upper abdomen, fever, malaise, weight loss, early satiety, abdominal distension, and cachexia. Workups are remarkable for the evidence of hepatocellular carcinoma, and trans-arterial chemoembolization (TACE) is planned. Which of the following indexes makes the patient a favorable candidate for bridging TACE?
Choices: 1. No evidence of extrahepatic spread on imaging 2. Intravenous chemotherapy with monoclonal antibodies 3. No evidence of extrahepatic spread on diagnostic laparoscopy 4. Eastern Cooperative Oncology Group (ECOG) Performance Status (PS) ECOG PS less than 5
Answer: 1 - No evidence of extrahepatic spread on imaging
Explanations: TACE intends to keep the patient a candidate for transplant ("bridging" therapy) with the following criteria; 1. no evidence of vascular invasion on imaging, 2. no evidence of extrahepatic spread on imaging, 3. ECOG PS remains 2 or less and 4. The patient meets the Milan criteria. Other than opting for no treatment, other potential mainstream therapies accepted by the National Comprehensive Cancer Network (NCCN) depending on the patient are IV chemotherapy (there are no immunotherapies as of yet), external beam radiation directed at the liver, embolization of the tumor's feeding arteries with inert particles only (bland embolization) or with radioactive particles (radioembolization). Radiofrequency or microwave ablation, unlike the therapies above, should not be used in isolation for metastatic disease from or to the liver. Radiofrequency ablation can be used in combination with TACE or other therapy. Go to the next page if you knew the correct answer, or click the link image(s) below to further research the concepts in this question (if desired).
Research Concepts: Hepatic Chemoembolization
We update eBooks quarterly and Apps daily based on user feedback. Please tap flag to report any questions that need improvement.
Question 662:
A 7-year-old boy is brought in by his parents with a pinkish lesion on his right thigh. He denies any pain, itchiness, or fever. He has no past medical history. On examination, the lesion is in the dermis, non-tender, about 4 x 2 cm in size with palpable projections in the surrounding tissue. A biopsy is performed, and the histopathology reveals an endovascular papillary proliferation projecting into the vessel lumen along with an intravascular and perivascular lymphocytic inflammatory infiltrate. Which of the following immunohistochemical markers will further aid in the diagnosis of this tumor?
Choices: 1. 2. 3. 4.
CD CD CD CD
5 20 31 99
Answer: 3 - CD 31 Explanations: Dabska tumors maintain a characteristic structure of endovascular papillary proliferations that project into the vessel lumen, as well as an intravascular and perivascular lymphocytic inflammatory infiltrate. CD31 is a good marker for vascular and endothelial tumors because of its high sensitivity and specificity for malignant tumors. The presence of highly specific lymphatic endothelial marker D2-40 and tumor marker vascular endothelial cell growth factor receptor type 3 suggest that the tumor is more similar to a tumor of lymphatic origin than a hemangioma. Other markers mentioned are T cell and B cell markers. Go to the next page if you knew the correct answer, or click the link image(s) below to further research the concepts in this question (if desired).
Research Concepts: Endovascular Papillary Angioendothelioma
We update eBooks quarterly and Apps daily based on user feedback. Please tap flag to report any questions that need improvement.
Question 663:
A 65-year-old female received a heart transplant 20 years ago, she never had an episode of rejection and is very compliant with her medications: Tacrolimus, Mycophenolate Mofetil, and Prednisone. She came to the clinic with complaints of abdominal pain, nausea, and weight loss for the last three months. Imaging shows a 3x4 cm tumor in the liver. A biopsy of the mass was performed, and pathology revealed post-transplant lymphoproliferative disorder. In a patient with the posttransplant lymphoproliferative disease, what is the first step in management?
Choices: 1. 2. 3. 4.
Administer radiation Surgical excision Reduce dose of immunosuppressant Increase dose of immunosuppressant
Answer: 3 - Reduce dose of immunosuppressant Explanations: The treatment of choice is chemotherapy. However, the first step is to reduce immunosuppression. Overimmunosuppression is responsible for Epstein-Barr virus-related post-transplant lymphoproliferative disorder. Surgery is not the treatment of choice. In most cases, surgery is not amenable due to location and recurrence. Go to the next page if you knew the correct answer, or click the link image(s) below to further research the concepts in this question (if desired).
Research Concepts: Post Transplant Lymphoproliferative Disorders
We update eBooks quarterly and Apps daily based on user feedback. Please tap flag to report any questions that need improvement.
Question 664:
A 59-year-old male with gross hematuria. Kidney biopsy reveals enlarged nuclei, with prominent nucleoli and abnormal nuclei. Staining reveals presence of mucin in the cytoplasm. The patient may have which of the following?
Choices: 1. 2. 3. 4.
Adenocarcinoma Melanoma Squamous cell cancer Transitional cell cancer
Answer: 1 - Adenocarcinoma Explanations: Renal cell cancer can have five histological subtypes at presentation. Typically renal cell cancer has clear cells with cytoplasm, which is rich in lipids and glycogen and usually show 3p deletion. Chromophilic tumors are usually bilateral, multifocal, and are linked to trisomy 7. The other histological types include oncocytoma and collecting duct cancers. Go to the next page if you knew the correct answer, or click the link image(s) below to further research the concepts in this question (if desired).
Research Concepts: Renal Cell Cancer
We update eBooks quarterly and Apps daily based on user feedback. Please tap flag to report any questions that need improvement.
Question 665:
A 45-year-old male presents to the clinic having complaints of dull, persistent, mild intensity pain over the right hypochondrium. The patient reports some weight loss with an increase in the severity of the pain on movement. Radiological examination, including a computed tomography scan of the abdomen, excluded the possibility of hepatoma and cirrhosis. Furthermore, a targeted ultrasound-guided biopsy of the lesion revealed the absence of fibrosis, with an abundant cellular component. Which of the following is a poor prognostic marker for the response of this tumor to combination chemotherapy?
Choices: 1. 2. 3. 4.
Age Raised beta-2 microglobulin levels Low serum albumin level The absence of constitutional symptoms
Answer: 2 - Raised beta-2 microglobulin levels Explanations: Raised beta-2 microglobulin levels of > 3 mg/dl is a poor prognostic marker for response to combination chemotherapy. Serum beta-2 microglobulin is secreted via blood cells, and its levels are directly correlated with the tumor burden. Presence of one pre-treatment risk factor requires to use altering combination chemotherapy, rather conventional cyclophosphamide, doxorubicin, vincristine, and prednisone (CHOP)-based regimen. Other predictors of poor outcomes in primary hepatic lymphoma are serum LDH > 10% of the upper limit of the normal, presence of constitutional symptoms such as fever, myalgia, weight loss, and lastly, tumor size of > 7 cm in the largest dimension. Go to the next page if you knew the correct answer, or click the link image(s) below to further research the concepts in this question (if desired).
Research Concepts: Hepatic Lymphoma
We update eBooks quarterly and Apps daily based on user feedback. Please tap flag to report any questions that need improvement.
Question 666:
A 67-year-old woman presents to the clinic with a palpable breast mass. Diagnostic mammography reveals a 1.4 cm spiculated mass in the upper outer right breast. Targeted ultrasound shows a hypoechoic mass measuring 1.3 x 1.1 x 0.8 cm. Breast biopsy confirms invasive ductal carcinoma that is ER/PR positive and HER2/neu negative. She undergoes lumpectomy and sentinel node biopsy demonstrating 1.5 x 1.2 cm moderately differentiated invasive ductal carcinoma, with negative margins and one negative sentinel node. What is the most appropriate next step?
Choices: 1. Right whole breast radiotherapy only 2. Adjuvant chemotherapy 3. 21-gene RT PCR assay to determine the benefit of adjuvant chemotherapy 4. Hormone therapy only
Answer: 3 - 21-gene RT PCR assay to determine the benefit of adjuvant chemotherapy
Explanations: The NCCN guidelines indicate breast tumors that are ER/PR positive, Her2/neu negative, and exceed 5 mm in size should undergo a 21-gene RT PCR assay postoperatively to determine the benefit of adjuvant chemotherapy. The gene assay provides a recurrence score, risk of distant metastasis over time, and the absolute benefit of chemotherapy. Based on the results of the TAILOR Rx study, adjuvant chemotherapy may be omitted on the basis of a low genetic risk score. In postmenopausal women, recurrence scores of 26-30 warrant at least consideration of adjuvant chemotherapy. If the recurrence score is 31 or greater, then adjuvant chemotherapy is recommended. For scores below 26, hormone therapy alone is typically recommended. In premenopausal women, the threshold for consideration of chemotherapy is typically a recurrence score exceeding 15. Patients who are candidates for adjuvant chemotherapy will typically receive this first, followed by adjuvant radiation therapy and hormonal therapy. The optimal sequencing of radiation and chemotherapy is not established. Go to the next page if you knew the correct answer, or click the link image(s) below to further research the concepts in this question (if desired).
Research Concepts:
Radiation Therapy For Early Stage Breast Cancer
We update eBooks quarterly and Apps daily based on user feedback. Please tap flag to report any questions that need improvement.
Question 667:
A 36-year-old female with acute myeloblastic leukemia is evaluated for bone marrow transplantation. She has a haploidentical sibling, and her parents are still alive. Donor specific antibody to HLA-A2 at 16,000 MFI is present. All of her potential related donors carry HLA-A2, and it is a mismatched antigen. An unrelated donor search is initiated trough National Marrow Donor Program (NMDP), and two potential NMDP donors are identified. Donor 1 is matched for loci A, B, C, DR, and DQ but mismatched at 2 DP loci. No donor-specific antibodies are identified against the mismatched loci. Donor 2 is matched for loci A, B, C, and DP but mismatched at one of the DR and DQ loci. Which of these potential donors is most suitable for this patient?
Choices: 1. 2. 3. 4.
Haploidentical sibling One of the parents Donor 1 Donor 2
Answer: 3 - Donor 1 Explanations: In bone marrow transplantation, matching for HLA loci A, B, C, DR, and DQ are important. However, DP is not one of the loci that require matching. In bone marrow transplantation degree of HLA matching correlates with increased graft survival. Usually, the match to A, B, C, DR, and DQ loci is preferred over haploidentical transplantation. The presence of donor-specific HLA antibodies is associated with decreased graft survival. If bone marrow recipients do not have related donors or related donors are not HLA matched with the recipient or are not compatible with the recipient due to the presence of HLA antibodies, NMDP donors can be used as a source for transplantation. Go to the next page if you knew the correct answer, or click the link image(s) below to further research the concepts in this question (if desired).
Research Concepts: Genetics, Human Major Histocompatibility Complex (MHC)
We update eBooks quarterly and Apps daily based on user feedback. Please tap flag to report any questions that need improvement.
Question 668:
A 58-year-old man with a past medical history of 40 pack-year smoking, gastroesophageal reflux disease (GERD), depression, chronic bacterial prostatitis, and seasonal allergies is found to have an exon 19 mutated non-small cell lung cancer with metastasis to his brain. The patient recently lost his job and does not wish to undergo surgical or radiation therapy. The decision is made to start osimertinib. Which of the following medications could most likely be continued without modification after initiating osimertinib therapy?
Choices: 1. 2. 3. 4.
Escitalopram for depression Ciprofloxacin for chronic bacterial prostatitis Cetirizine for seasonal allergies Trazadone for insomnia
Answer: 3 - Cetirizine for seasonal allergies Explanations: Cetirizine is an H1-receptor antagonist that does not affect the H2 receptors in the stomach, nor is this drug metabolized by CYP enzymes. Cetirizine is unlikely to affect the dosage and administration of osimertinib. Ciprofloxacin is a fluoroquinolone antibiotic that also inhibits some cytochrome P450 enzymes. CYP3A, in particular, plays an essential role in the metabolism of many TKIs. Inhibition of CYP3A can lead to increased TKI exposure and toxic effects. Escitalopram is an antidepressant that increases the toxicity of osimertinib by increasing the QTc interval. Go to the next page if you knew the correct answer, or click the link image(s) below to further research the concepts in this question (if desired).
Research Concepts: Tyrosine Kinase Inhibitors
We update eBooks quarterly and Apps daily based on user feedback. Please tap flag to report any questions that need improvement.
Question 669:
A 55-year-old man presents to the clinic with a solitary, erythematous, nonpruritic papule on his forehead. A skin shave biopsy is performed, which shows a proliferation of centrocytes and admixed centroblasts in a follicular pattern. The follicles are closely spaced and do not have well-defined mantle zones. The germinal centers lack tingible body macrophages. Immunohistochemical staining shows that the cells of interest stain positively for CD20 and CD79a (strong). The lesion is negative for BCL-2, cyclin D1, IRF4/MUM1, FOXP1, and cytoplasmic IgM. Which of the following, if any, is the most specific cytogenetic abnormality found in this condition?
Choices: 1. t(14;18) 2. Inactivation of CDKN2A and CDKN2B gene loci on chromosome 9p21.3 3. Rearrangement of DUSP22 or TP63 4. No specific cytogenetic abnormality
Answer: 4 - No specific cytogenetic abnormality Explanations: t(14;18) is typically NOT found in primary cutaneous follicle center lymphoma (PCFCL) as it is in typical follicular lymphoma. The correct answer, in this case, would be no specific cytogenetic abnormality as none has been documented. The most important point is that a negative t(14;18) can help differentiate it from follicular lymphoma. Inactivation of CDKN2A and CDKN2B is typically seen in diffuse large B-cell lymphoma-leg type. Rearrangements of DUSP22 and/or TP63 are typically seen in cutaneous anaplastic large cell lymphoma (T-cell neoplasm). If the lesion is positive for t(14;18) and BCL-2, suspicion for secondary involvement by follicular lymphoma should be high on the differential diagnosis. Go to the next page if you knew the correct answer, or click the link image(s) below to further research the concepts in this question (if desired).
Research Concepts: Primary Cutaneous Follicle Center Lymphoma
We update eBooks quarterly and Apps daily based on user feedback. Please tap flag to report any questions that need improvement.
Question 670:
A 32-year-old woman with a past medical history of poorly controlled HIV presents with purple lesions growing on her oral mucosa for the last 2 months. The lesions are painless and nonpruritic but do cause the patient, mild discomfort, and significant concern. A biopsy is obtained, and a diagnosis is made. In addition to educating the patient about adherence to her HIV regimen, a urine pregnancy test is ordered, and treatment is started with a drug that possesses known anti-angiogenic effects. Three weeks later, the patient returns to the clinic complaining of increased somnolence throughout the day. Proper management of this patient's drug toxicity involves which of the following recommendations?
Choices: 1. Maintain current dosage and advise taking the medication at bedtime 2. Reduce drug dosage by 50% and re-evaluate in two weeks 3. Discontinue medication and followup in two weeks 4. Prescribe a low dose stimulant to counteract druginduced somnolence
Answer: 1 - Maintain current dosage and advise taking the medication at bedtime
Explanations: Thalidomide is an extremely teratogenic medication that possesses anti-angiogenic effects, which can be used in the treatment of Kaposi sarcoma. Somnolence is a very common adverse effect related to thalidomide toxicity and is caused by the activity of the R(+) enantiomer on sleep-related receptors in the brain. Treatment of somnolence related to thalidomide toxicity begins with the continuation of the therapeutic dose and instruction to the patient to take a full daily dose before bedtime. More extreme symptoms may include obtundation, stupor, or coma. In the event of these severe symptoms, thalidomide therapy should be held until symptoms resolve and then restarted at a 50% reduction of drug dosage. Reduction of drug dosage by 50% is the first step for treating neuropathy related to thalidomide toxicity, not somnolence. Discontinuation and re-evaluation of thalidomide treatment are only required in more severe instances of somnolence related to thalidomide toxicity (obtundation, stupor, or coma). Perscription of a stimulant to counteract the effects of thalidomiderelated somnolence is not recommended as proper management of drug toxicity. Go to the next page if you knew the correct answer, or click the link image(s) below to further research the concepts in this question (if desired).
Research Concepts:
Thalidomide
We update eBooks quarterly and Apps daily based on user feedback. Please tap flag to report any questions that need improvement.
Question 671:
A 65-year-old male presents with epigastric pain. He also complains of nausea. He has also been experiencing gradual weight loss for the last six months. On presentation, he has a temperature of 98.6 F (37 C), a pulse rate of 74 bpm, a blood pressure of 130/70 mmHg, and a respiratory rate of 14 breaths/min. The physical examination reveals a palpable lymph node in the left supraclavicular area. He has a hemoglobin level of 7.7 g/dl, a total leukocyte count of 4600/mm3, and a platelet count of 408,000/mm3. Liver function tests show a bilirubin of 0.7 mg/dl (normal: 0.2-1 mg/dL), alanine aminotransferase (ALT) of 19 IU/L (normal: 5-30 IU/L), aspartate aminotransferase (AST) of 17 IU/L (normal: 6-34 IU/L), and an alkaline phosphatase level of 42 IU/L (normal: 20-140 IU/L). A serum amylase level of 44 U/L (normal: 30125 U/L) is noted. He has a history of recurrent infections with Helicobacter pylori. On endoscopic evaluation, an ulcerative lesion with raised margins is noted in the stomach. Which of the following would be the most probable diagnosis in this patient?
Choices: 1. 2. 3. 4.
Gastric cancer Chronic gastritis Gastric outlet obstruction Gastroesophageal reflux disease
Answer: 1 - Gastric cancer Explanations: Helicobacter pylori (H. pylori) infection is associated with chronic atrophic gastritis, which eventually develops into a malignancy. The intestinal-type gastric adenocarcinoma, which has an association with H. pylori infection, appears as an ulcer with heaped-up margins. The most common location for H.pylori related gastric carcinomas is the lesser curvature. Fewer than 5% of individuals who get infected with H. pylori develop gastric cancer. Go to the next page if you knew the correct answer, or click the link image(s) below to further research the concepts in this question (if desired).
Research Concepts: Helicobacter Pylori
We update eBooks quarterly and Apps daily based on user feedback. Please tap flag to report any questions that need improvement.
Question 672:
A 73-year-old patient presents to your clinic after routine colonoscopy demonstrated a 2 cm villous adenoma in her sigmoid colon. Her primary care provider ordered further imaging and referred the patient to your oncology clinic for further evaluation. The patient has always considered herself a healthy woman, and she becomes very concerned when she notices two "spots" on her liver when you are reviewing her imaging with her. What finding is generally the most worrisome?
Choices: 1. 2. 3. 4.
>2 cm in diameter Absence of an air-fluid level Poor margins A solitary lesion
Answer: 3 - Poor margins Explanations: Lung cancer, melanoma, and carcinoid can all cause cystic liver metastases. Metastatic cystic lesions are typically less than 10 mm, while primary hepatic cysts are usually larger. Cystic metastases may be due to degeneration of rapidly growing hypervascular tumors sarcoma, melanoma, carcinoid, neuroendocrine tumors, and some lung and breast tumors. Metastatic cystic lesions usually have an air-fluid level, whereas primary hepatic cysts do not. The other cause of cystic liver metastases is mucinous metastases from the colon or ovary adenocarcinomas. Metastatic cystic lesions are usually multiple, whereas primary hepatic cysts are always solitary In general, the cystic hepatic metastases are much less marginated and hence this makes it easier to differentiate them from a primary liver cyst. Go to the next page if you knew the correct answer, or click the link image(s) below to further research the concepts in this question (if desired).
Research Concepts: Liver Metastasis
We update eBooks quarterly and Apps daily based on user feedback. Please tap flag to report any questions that need improvement.
Question 673:
A 53-year-old G0 woman presents to the clinic for an annual exam. The patient has no significant past medical history and no significant gynecologic issues. She is up to date on her pap smears and has never had an abnormal result. Upon speculum exam, there is a dark blue lesion with irregular borders on the right vaginal wall. Which gene mutation is most commonly seen with this pathology?
Choices: 1. 2. 3. 4.
BRAF MLH1 PMS2 BRCA
Answer: 1 - BRAF Explanations: The large majority (~90%) of primary vaginal cancers are squamous cell carcinomas. Primary adenocarcinomas of the vagina occur but are rare. DESrelated clear cell carcinomas of the vagina occurred mainly in young women, but as the DES-exposed cohort ages, DES-related tumors are now rare. An even smaller percentage of cancers are sarcomas and melanomas. This clinical vignette describes a vaginal melanoma. BRAF proto-oncogene serine/threonine kinase is the most frequently mutated gene in vaginal melanoma. Mismatch repair protein anomalies are commonly seen in Lynch syndrome carcinomas such as colorectal, endometrial and ovarian cancers. PMS2 is a mutation seen in Lynch Syndrome which is more commonly associated with endometrial and ovarian carcinomas. BRCA is commonly associated with breast and ovarian carcinomas and not implicated in vaginal melanomas. Go to the next page if you knew the correct answer, or click the link image(s) below to further research the concepts in this question (if desired).
Research Concepts: Vaginal Cancer
We update eBooks quarterly and Apps daily based on user feedback. Please tap flag to report any questions that need improvement.
Question 674:
A 65-year-old female has an abnormal mammogram. She denies any symptoms of breast pain, lump, skin changes, nipple discharge, or bleeding. The screening mammogram demonstrates a 1.2 cm spiculated mass in the upper outer quadrant of the right breast. This is confirmed on a diagnostic mammogram. A targeted ultrasound shows a hypoechoic mass measuring 1.3 x 1.1 x 0.8 cm. The clinical exam reveals a palpable mass in the upper outer quadrant of the right breast. The axillary examination is negative. Breast biopsy reveals a moderately differentiated invasive ductal carcinoma measuring 1.2 cm that was ER/PR positive and HER2/neu negative. What is the next best step?
Choices: 1. 2. 3. 4.
Biopsy the axilla Chemotherapy Mastectomy Lumpectomy and sentinel node biopsy
Answer: 4 - Lumpectomy and sentinel node biopsy Explanations: Breast conservation therapy may include quadrantectomy, wide excision, or lumpectomy. In clinically negative axilla, sentinel node biopsy has replaced upfront axillary node dissection. Radiation therapy is recommended after lumpectomy to reduce the risk of local recurrence. Radiation options for node-negative breast cancer include whole breast radiotherapy (conventional fractionation or hypofractionation) or partial breast irradiation. Go to the next page if you knew the correct answer, or click the link image(s) below to further research the concepts in this question (if desired).
Research Concepts: Breast Cancer
We update eBooks quarterly and Apps daily based on user feedback. Please tap flag to report any questions that need improvement.
Question 675:
A 49-year-old male presented with painless lymphadenopathy in the left groin and left axilla. Lymphadenopathy was waxing and waning in nature. Flow cytometry showed a kappa restricted CD20+, CD19+, and CD10+ population. What translocation could help render a final diagnosis in this patient?
Choices: 1. 2. 3. 4.
t(14;18) t(11:18) t(2:5) t(9:22)
Answer: 1 - t(14;18) Explanations: The majority of the follicular lymphoma (FL) cases will show a t(14;18)(q32;q21); the translocation affects the IgH and BCL2. Follicular lymphoma presents with painless waxing and waning lymphadenopathy. Genetics is a useful tool for assessing FL. Most cases will show clonally rearranged immunoglobulin genes. t(14;18)(q32;q21) translocation leads to an overexpression of BCL2, which prevents cells in the follicular center from undergoing apoptosis. BCL2 is sensitive but not specific for FL and can be present in other non-Hodgkin lymphomas (NHL) such as diffuse large B-cell lymphoma (DLBCL). Diagnosis of FL needs morphological assessment, flow cytometry, and sometimes genetic testing to confirm the diagnosis. Go to the next page if you knew the correct answer, or click the link image(s) below to further research the concepts in this question (if desired).
Research Concepts: Follicular Lymphoma
We update eBooks quarterly and Apps daily based on user feedback. Please tap flag to report any questions that need improvement.
Question 676:
A 66-year-old man is being evaluated for weight loss, back pain, and chronic cough. A chest x-ray reveals a 4 cm spiculated nodule in the left upper lobe. The patient denies any smoking history but did work as a uranium miner for nearly thirty years. Which of the following is most likely responsible for the patient's disease?
Choices: 1. 2. 3. 4.
Uranium Radon gas Xenon Gamma radiation
Answer: 2 - Radon gas Explanations: Uranium mines have a high level of radon gas. Radon gas increases the likelihood of developing a lung carcinoma. Radon is made of alpha particles that can settle on alveoli tissue after being inhaled and cause cellular damage over time. Uranium exposure would have had cutaneous symptoms. Xenon is made of beta particles and would have had more systemic symptoms. Gamma radiation is usually a result of nuclear detonation. Go to the next page if you knew the correct answer, or click the link image(s) below to further research the concepts in this question (if desired).
Research Concepts: Radiation Syndrome
We update eBooks quarterly and Apps daily based on user feedback. Please tap flag to report any questions that need improvement.
Question 677:
A 56-year-old man presents to the clinic with an enlarging 2 cm left parotid mass for the past five years. On CT scan, the mass is localized in the superficial parotid. Fine needle aspiration shows varying amounts of epithelial and mesenchymal components with no malignant features, consistent with pleomorphic adenoma. What is the most appropriate management strategy for this patient?
Choices: 1. Enucleation with clean margins 2. Superficial parotidectomy 3. Total parotidectomy with facial nerve preservation due to high recurrence risk 4. Superficial parotidectomy with postoperative radiation therapy due to high recurrence risk
Answer: 2 - Superficial parotidectomy Explanations: Pleomorphic adenoma is a benign salivary tumor that consists of varying amounts of epithelial and mesenchymal components. The epithelial elements give rise to ducts, whereas the mesenchymal elements give rise to cartilaginous, osseous, hyaline, and myxoid tissues. Surgical excision with extracapsular gland dissection is the treatment of choice for benign salivary tumors. Enucleation is not advised due to the higher rates of local recurrence. Lesions of the superficial lobe of the parotid gland can be excised with a superficial parotidectomy. Total parotidectomy with facial nerve preservation is performed for large or deep parotid tumors. Surgical resection with negative margins achieves local control in over 95% of benign salivary gland tumor cases. Postoperative radiation is usually not recommended in these cases due to the small benefit in local control and potential risk for radiation morbidity. Go to the next page if you knew the correct answer, or click the link image(s) below to further research the concepts in this question (if desired).
Research Concepts: Benign Salivary Gland Tumors
We update eBooks quarterly and Apps daily based on user feedback. Please tap flag to report any questions that need improvement.
Question 678:
A 46-year-old patient presents with a cough for six months. The cough is productive of white sputum. The patient does not have any chest pain, nasal congestion, or shortness of breath. After a thorough evaluation, the patient is found to have laryngeal carcinoma. During a follow-up visit, the patient asks what would be an appropriate medication to keep the coughing to a minimum. Which drug among the following choices is most appropriate?
Choices: 1. 2. 3. 4.
Guaifenesin Paracetamol Phenylephrine Codeine
Answer: 4 - Codeine Explanations: Codeine is used in the treatment of various etiologies producing chronic cough. In addition, 46% of patients with chronic cough do not have a distinct etiology despite a proper diagnostic evaluation. Codeine produces a decrease in cough frequency and severity in cough due to an indistinct etiology. It is indicated in the management of prolonged cough, usually as 30 mg every 4 to 6 hours as needed. The most common side effects are constipation and sedation. Given the patient's cancer, codeine addiction is not a significant concern. Go to the next page if you knew the correct answer, or click the link image(s) below to further research the concepts in this question (if desired).
Research Concepts: Codeine
We update eBooks quarterly and Apps daily based on user feedback. Please tap flag to report any questions that need improvement.
Question 679:
A 68-year-old male with a 50-pack-year history of smoking presents to the clinic to establish primary care and seek advice regarding age-appropriate cancer screening. His pulse rate is 78/min, and his blood pressure is 140/80 mmHg. He has a past medical history of chronic obstructive pulmonary disease (COPD) and is currently on metered-dose inhalers. His cousin expired two years ago due to squamous cell carcinoma of the lung. He is concerned regarding his lung condition and asks about early screening options. Which of the following would be the next step in managing this patient?
Choices: 1. 2. 3. 4.
Low dose CT of the chest Annual chest X-ray No screening exam is shown to improve cancer mortality Pulmonary function test (PFT) annually
Answer: 1 - Low dose CT of the chest Explanations: The lung cancer screening program is designed for those who are 55 to 80 years old, either active smokers or those who quit within the last 15 years. Screening should be discontinued once a person has not smoked for 15 years or develops a health problem that substantially limits life expectancy or the ability or willingness to have curative lung surgery (as per US Preventive Service Task Force). Radiation exposure is another grave concern related to regular screening. The risk of radiation from medical tests is low; however, it is a theoretical possibility. The ionizing radiation can cause DNA damage leading to double-stranded breaks, which, if not repaired correctly, will lead to mutations and possible cancers. Therefore, it is important to select the right population and to have a detailed conversation with patients about the risks and benefits. As with all screening modalities, Low dose CT (LDCT) requires open and honest communication between patients and clinicians, and the clinician and patient must engage in shared decision making. Shared decision-making allows providers to give patients all the relevant information regarding a screening test, including the possibility of false-positive results, the options for further diagnostic testing, and the risks associated with undertaking each test. PFTs are useful for COPD and only need to be done once to establish the diagnosis.
Go to the next page if you knew the correct answer, or click the link image(s) below to further research the concepts in this question (if desired).
Research Concepts: Lung Cancer Screening
We update eBooks quarterly and Apps daily based on user feedback. Please tap flag to report any questions that need improvement.
Question 680:
A 65-year-old woman with a history of diabetes, hypothyroidism, gastroesophageal reflux, and malignant melanoma is admitted with severe headache, nausea, dizziness, and abdominal pain. On examination, her blood pressure is 72/58 mmHg, pulse 99/min, and temperature 96.6 F (35.8 C). The chest is clear on auscultation, with normal S1 and S2. She has mild tenderness in the abdomen. Her medications include metformin 1000 mg, levothyroxine, omeprazole, and pembrolizumab. Laboratory examination reveals sodium 132 mEq/L, potassium 3.8 mEq/L, creatinine 0.8 mg/dL, TSH 1.5 mU/L, free T4 1.1 ng/dL, and total T3 92 ng/dL. Cosyntropin stimulation test (CST) is done with a baseline and 60 min cortisol less than two mcg/dL. ACTH drawn at baseline is undetectable. Which of the following is the most likely diagnosis?
Choices: 1. She has central adrenal insufficiency secondary to pembrolizumab 2. She does not have adrenal insufficiency and CST is affected by drug interference 3. She has primary central adrenal insufficiency and should be treated with oral steroids 4. She has central adrenal insufficiency but does not need steroids
Answer: 1 - She has central adrenal insufficiency secondary to pembrolizumab
Explanations: Cosyntropin stimulation test (CST) is a dynamic test frequently used for evaluation of the Hypothalamus Pituitary Adrenal (HPA) axis in suspected adrenal insufficiency (AI). Adrenal insufficiency is potentially lifethreatening conditioning with a very variable clinical presentation and requires a high level of clinical suspicion for diagnosis. Without timely intervention, untreated AI can lead to grave complications, including death. AI can be primary, secondary, or tertiary depending on etiology. Primary AI is related to diseases related to the adrenal gland and could be autoimmune or destruction. Secondary and tertiary AI can be also referred to as central AI and are due to pituitary or hypothalamic diseases, respectively. The term Relative AI was introduced in 1991 by Rothwell et al. for decreased responsiveness of adrenals after injecting 250 mcg corticotropin in critically ill patients. The term was later changed to Critical Illness Related Corticosteroid Deficiency in 2008 by Marik et al. Administration of supraphysiological dose of synthetic ACTH or Cosyntropin stimulates the pituitary and thereby release of cortisol from the adrenal cortex as long as the adrenals cortex has a functional reserve. High dose (250 mcg) CST is standard and routinely used as dynamic testing for evaluation of the HPA axis. CST involves checking cortisol levels at baseline, 30 minutes, and 60 minutes after IV or IM administration of 250 mcg cosyntropin. A negative (normal) result is if the cortisol
level is stimulated to > 500-600 nmol/L. A positive (abnormal or subnormal) result could be from primary or central and can be further evaluated by ACTH levels. ACTH level is expected to be high with primary and low with central AI. Baseline cortisol was low suggesting the diagnosis of adrenal insufficiency (AI) and cortisol failed to get stimulated after CST, confirming the diagnosis of AI. ACTH levels were low which rules out primary AI and suggest she has central AI. Pembrolizumab can cause central AI Go to the next page if you knew the correct answer, or click the link image(s) below to further research the concepts in this question (if desired).
Research Concepts: Adrenocorticotropic Hormone Test
We update eBooks quarterly and Apps daily based on user feedback. Please tap flag to report any questions that need improvement.
Question 681:
A 44-year-old man presents to the clinic for the evaluation of hair loss for the past two months. He is currently receiving chemotherapy for stage 4 Hodgkin lymphoma. The patient does not smoke or drink alcohol. Vitals show a blood pressure of 128/70 mm Hg, a pulse of 74/min, a respiratory rate of 12/min, and a temperature of 98.6 F (37 C). Physical examination reveals conjunctival pallor and mild dryness of mucous membranes. Scalp examination may reveal which of the following?
Choices: 1. 2. 3. 4.
Exclamation point hairs Scarring of the scalp Sinus tract formation on the scalp Erythematous, scaly, ring-shaped lesions
Answer: 1 - Exclamation point hairs Explanations: Anagen effluvium is a form of nonscarring alopecia commonly associated with chemotherapy. Once chemotherapy has been initiated, anagen effluvium presents within days to a few weeks. The severity of hair loss differs between patients, but it is not unusual for a patient to have complete hair loss within 2 to 3 months of beginning chemotherapy. The hallmark physical exam finding in anagen effluvium is a tapered fracture of the hair shaft that is also known as an exclamation point hair. Anagen effluvium is non-cicatricial, and therefore any signs suggestive of an active inflammatory scarring process such as erythema, scale, or pigmentation are absent. Go to the next page if you knew the correct answer, or click the link image(s) below to further research the concepts in this question (if desired).
Research Concepts: Anagen Effluvium
We update eBooks quarterly and Apps daily based on user feedback. Please tap flag to report any questions that need improvement.
Question 682:
A 42-year-old female presents for excessive irritation in her right eye. She first noticed a whitish lesion growing in her eye 2 months ago. She has used corticosteroids as well as lubricants for the affected eye. Slit lamp examination of the right eye reveals a plaquelike, whitish, hyperkeratotic lesion at the nasal interpalpebral conjunctiva extruding over the limbus and 3 mm into the cornea. The rest of the examination is normal, and Snellen’s visual acuity is 6/6. The patient undergoes excision biopsy, and the mass is sent for histopathological evaluation. What theory best explains the pathogenesis of this condition?
Choices: 1. 2. 3. 4.
Keratocyte turnover theory Collagen remodeling theory Stem cell theory Cytokine theory
Answer: 3 - Stem cell theory Explanations: The slit lamp examination of the right eye reveals a plaque-like, whitish, hyperkeratotic lesion at the nasal interpalpebral conjunctiva extruding over the limbus and 3 mm into the cornea. The clinical picture is suggestive of ocular surface squamous neoplasia (OSSN). The most accepted theory for OSSN is stem cell theory. The nasal limbus receives the highest amount of UV ray exposure and has epithelial crypts that contain epithelial stem cells in the basal epithelial layer. The limbal stem cells are the precursor for OSSN. OSSN is the third most common tumor after melanoma and lymphoma. Go to the next page if you knew the correct answer, or click the link image(s) below to further research the concepts in this question (if desired).
Research Concepts: Ocular Surface Squamous Neoplasia
We update eBooks quarterly and Apps daily based on user feedback. Please tap flag to report any questions that need improvement.
Question 683:
A 30-year-old man presented to the clinician's office complaining of recurrent otitis media. He had three episodes of otitis media infections in the past 6 months. He denied any upper respiratory infections recently, or any history of immunodeficiency in the family. During the examination, there is no tenderness in his face, and tonsils appeared normal in size. Otoscope shows effusion behind the right tympanic membrane. Vital signs are as the following: respiratory rate: 14/minute, the pulse rate: 90/minute, blood pressure: 125/80 mmHg, O2 saturation= 99%. The patient is a heavy smoker and eats a lot of processed food. The appropriate investigation was done, and lymphadenopathy was evident at right anterior jugular station without evidence of distal metastasis. Which of the following is the best first modality of treatment?
Choices: 1. 2. 3. 4.
Nasopharyngectomy No treatment needed at this time Radiotherapy Chemotherapy alone
Answer: 3 - Radiotherapy Explanations: Nasopharyngeal carcinoma is a malignancy arising from the epithelium of the nasopharynx. An interplay of environmental factors, genetic structure, and EbsteinBarr virus (EBV) infection is involved in the etiology of the disease. This patient presented with multiple episodes of otitis media, which should increase the suspicion of obstruction of the eustachian tube. One of the causes of Eustachian tube obstruction is tumors that arise around the eustachian opening in the nasopharynx, such as nasopharyngeal carcinoma (NPC). The best modality of treatment for this tumor is radiotherapy. Radiation is the management of choice for the loco-regional lesion. Radiotherapy is effective in all cases except those for distant metastasis. NPC is highly chemo and radiosensitive. Chemotherapy alone is not the best choice, it can be used as an induction, which will lead to a better response. Go to the next page if you knew the correct answer, or click the link image(s) below to further research the concepts in this question (if desired).
Research Concepts: Nasopharyngeal Carcinoma (NPC, Lymphoepithelioma)
We update eBooks quarterly and Apps daily based on user feedback. Please tap flag to report any questions that need improvement.
Question 684:
A 54-year-old woman comes to the office to review her initial Pap smear result she did 4 weeks prior. She has 4 older children, and she is a known hypertension patient taking amlodipine, valsartan, and hydrochlorothiazide combination. She has no other comorbidity, never smoked, and no gynecological complaint. Vital signs are temperature 37 C, respiration 15/minute, pulse rate: 78 beats per minute, and blood pressure 138/90 mmHg. The present Pap smear result is atypical squamous cells of undetermined significance. Which of the following is the most appropriate action?
Choices: 1. Repeat another Pap test after 3 years and HPV testing next year 2. Repeat another Pap test together with HPV testing during this visit 3. Repeat another Pap test after three and five years 4. Do immediate colposcopy and HPV testing next year
Answer: 2 - Repeat another Pap test together with HPV testing during this visit
Explanations: Atypical squamous cells of undetermined significance (ASCUS) is not associated with significant pathology, except for the persistence of HPV infection, which is the genesis of neoplasia. The 2019 ASCCP (American Society for Colposcopy and Cervical Pathology) guidelines recommend that an initial cytological diagnosis of ASCUS be repeated immediately together with HPV testing (cotesting) to determine the presence of high-risk HPV such as HPV 16 before colposcopy. After the second cytology, together with HPV determination, would it be appropriate to refer for colposcopy, for example, the persistence of the ASCUS report and a positive high-risk HPV result. If the second cytology is negative for intraepithelial lesion or malignancy (NILM) and the HPV test is negative, the woman should be reassessed after 5 years. Colposcopy would not be necessary at this time. Go to the next page if you knew the correct answer, or click the link image(s) below to further research the concepts in this question (if desired).
Research Concepts: Atypical Squamous Cells of Undetermined Significance
We update eBooks quarterly and Apps daily based on user feedback. Please tap flag to report any questions that need improvement.
Question 685:
A 65-year-old male presents to the office with a recent onset of diarrhea. The patient states that diarrhea has made it difficult for him to leave the house. The patient denies any changes to his appetite or sleep habits. The patient recently started therapy with 5fluorouracil, leucovorin, and irinotecan. The physical exam of the patient is normal. Labwork shows that the patient is mildly neutropenic. He insists that he cannot leave without a solution to diarrhea. What can be given to this patient while taking irinotecan for his colorectal cancer treatment?
Choices: 1. 2. 3. 4.
Ondansetron Aluminum hydroxide over-the-counter Magnesium salt supplements Loperamide
Answer: 4 - Loperamide Explanations: Adverse effects of irinotecan are what limit its use in clinical settings. Neutropenia and diarrhea are two of the most common side effects. Loperamide is an opioid agonist that slows gut motility. It is the preferred opioid agonist as it does not cross the blood-brain barrier, decreasing its chances of causing addictive side effects. Ondansetron is a serotonin antagonist commonly used to decrease vomiting after chemotherapy. Magnesium sulfate would cause even more diarrhea. Aluminum hydroxide tablets would cause constipation and decrease the patient's diarrhea. The side effects of aluminum hydroxide include proximal muscle weakness and seizures, so it is less preferred than loperamide. Go to the next page if you knew the correct answer, or click the link image(s) below to further research the concepts in this question (if desired).
Research Concepts: Irinotecan
We update eBooks quarterly and Apps daily based on user feedback. Please tap flag to report any questions that need improvement.
Question 686:
A 65-year-old man with a past medical history of metastatic melanoma presents for evaluation. After discussing the risks and benefits and weighing his options, it is decided to start a new medication indicated for metastatic melanoma without using other agents. Appropriate labs are obtained, and the patient begins his treatment. At six months, the patient continues to tolerate the medication well and denies any adverse effects, and his vitals are within normal limits. The provider would like to repeat labs to assess response to medical therapy. What abnormality may be found in laboratory values as a response to the treatment?
Choices: 1. 2. 3. 4.
Hyperkalemia Elevated thyroid-stimulating hormone Neutropenia Elevated alanine aminotransferase
Answer: 4 - Elevated alanine aminotransferase Explanations: Ipilimumab can cause several endocrinopathies, including hypopituitarism (the most common), adrenal insufficiency, hypo and hyperthyroidism, hypogonadism, and Cushing syndrome. As an adverse effect of Ipilimumab, adrenal insufficiency can cause hypotension, hyponatremia, and hyperkalemia and is monitored clinically and through serial serum chemistries. Because Ipilimumab can cause adverse effects associated with almost any organ system, it is important to complete a thorough assessment of the patient before starting medical therapy, during treatment, and after completing the treatment. Complete monitoring of patients should include a thorough history and physical examination, focusing on the neurologic function, bowel habits of the patient, and a dermatologic assessment and baseline and serial monitoring of liver function tests, endocrine panel thyroid function tests, and serum chemistries. The most common adverse effects of Ipilimumab are enterocolitis, dermatitis, and hepatitis. Adverse effects associated with ipilimumab can usually be managed with supportive measures, but several uncommon side effects, such as intestinal perforation, Stevens-Johnson syndrome, toxic epidermal necrolysis, severe hepatotoxicity, and others, may rarely warrant discontinuation of the drug. Supportive measures generally include loperamide, diphenoxylate/atropine, and the occasional use of systemic corticosteroids.
Go to the next page if you knew the correct answer, or click the link image(s) below to further research the concepts in this question (if desired).
Research Concepts: Ipilimumab
We update eBooks quarterly and Apps daily based on user feedback. Please tap flag to report any questions that need improvement.
Question 687:
A 68-year-old man with no comorbidities is evaluated for hematuria. He is thin, tall, and has a healthy diet. He denies ever smoking cigarettes or doing drugs. He has never had a bladder infection or kidney stones. He has no past family history off genitourinary or other cancer. Urine cytology reveals abnormal cells with very high nuclear to cytoplasmic ratio, dark coarse chromatin, and irregular borders. Prominent nucleoli are occasionally seen in large, often single cells. Which of the following is the most relevant risk factor for the development of this patient's condition?
Choices: 1. 2. 3. 4.
Longterm alcohol use Red meat consumption Exposure to commercial dyes Presence of tattoos
Answer: 3 - Exposure to commercial dyes Explanations: 90% of bladder cancers are transitional cell carcinomas, while the remaining 10% are squamous cell carcinomas. Occupational exposures are a major risk factor for bladder cancer, and certain chemicals used in dyes, chemical, and rubber industries have been associated with transitional cell cancer. The histology of transitional cell carcinoma would reveal papillary structures lined by urothelium-like cells and nests of epithelial cells with high-grade features, pale granular cytoplasm, large nucleoli or longitudinal grooves, separated by fibrous stroma. The histology of squamous cell carcinoma would reveal abnormal cells with abundant, dense cytoplasm that can be intensely eosinophilic. Nuclei are large and hyperchromatic with irregular nuclear borders. Go to the next page if you knew the correct answer, or click the link image(s) below to further research the concepts in this question (if desired).
Research Concepts: Bladder Cancer
We update eBooks quarterly and Apps daily based on user feedback. Please tap flag to report any questions that need improvement.
Question 688:
A 57-year-old female presents with diarrhea for the past three weeks. The patient says her diarrhea is watery and contains a large amount of mucus. A colonoscopy is performed, which reveals a villous adenoma in the rectum. What metabolic imbalances would be expected in this patient?
Choices: 1. 2. 3. 4.
Hyperkalemia, hyperchloremic metabolic alkalosis Hypernatremia, hyperchloremic metabolic acidosis Hyponatremia, hyperchloremic metabolic alkalosis Hypokalemia, hyperchloremic metabolic acidosis
Answer: 4 - Hypokalemia, hyperchloremic metabolic acidosis
Explanations: In this clinical scenario, the patient has secretory diarrhea syndrome, a rare complication of villous adenoma, causing hypovolemia and metabolic imbalances like hypokalemia. Adenomatous polyps can be villous, tubulovillous, and tubular. Tubular polyps are the most common ones. However, villous adenomas are associated with larger adenomas and a more severe degree of dysplasia occurring more commonly in the rectum and the rectosigmoid. Although they may occur in any part of the colon, villous polyp usually appears as sessile structures with velvety or cauliflower-like projections where the columnar epithelium is predominant and immature; goblet cells may also be observed. The most common presenting symptom of large villous adenomas is hematochezia with anemia, most likely microcytic. Other non-specific symptoms include diarrhea, constipation, and change of stool caliber. The patient may have a positive family history of colon polyps. The patient may also be asymptomatic. Endoscopy is the most sensitive test, and biopsies can be obtained during this procedure. Fecal occult blood testing is also done, and 20 to 40 percent of patients with adenomas have positive findings. If possible, the polyp should be removed during endoscopy. Surgical resection of a polyp may be required if it is larger than 2 to 3 cm, as in the case of a villous adenoma. One of the distinguishing characteristics of hypokalemia related to lower GI losses is the higher potassium
concentration in lower intestinal losses (20 to 50 mEq/L) in most cases. Additionally, the affected patients usually present with simultaneous bicarbonate wasting and hyperchloremic metabolic acidosis. Lower gastrointestinal losses might result in hypokalemia in chronic cases, including a villous adenoma, or a vasoactive intestinal peptide secreting tumor (VIPoma). Go to the next page if you knew the correct answer, or click the link image(s) below to further research the concepts in this question (if desired).
Research Concepts: Villous Adenoma
We update eBooks quarterly and Apps daily based on user feedback. Please tap flag to report any questions that need improvement.
Question 689:
A 29-year-old woman with a history of PCOS is found to have leukocytosis and thrombocytosis during her initial evaluation at a fertility clinic. A bone marrow biopsy reveals BCR-ABL-positive chronic myelogenous leukemia (CML). The patient and her husband are interested in starting a family. What is the most appropriate management strategy for this patient?
Choices: 1. Imatinib therapy alone 2. Wait to initiate therapy until the third trimester of pregnancy 3. Wait to initiate therapy until after a successful pregnancy 4. Begin imatinib therapy with an effective contraceptive
Answer: 4 - Begin imatinib therapy with an effective contraceptive
Explanations: While several studies have shown successful, normal births while on tyrosine kinase inhibitor (TKI) therapy, fetal abnormalities are still a significant risk. No TKI has been established to be safe in pregnancy. This drug class should be avoided in female patients who are attempting to conceive or are found to be pregnant if possible. Because the couple has expressed an interest in conceiving, the patient should be counseled on the risks of becoming pregnant while taking imatinib. There is some evidence that TKIs do not cross the placenta efficiently and would likely not affect the fetus after organogenesis. However, it is best to avoid TKI whenever possible during pregnancy. Initiating TKI therapy during pregnancy might be considered in the setting of disease progress or lack of success with other treatments such as interferon. Imatinib, and other TKI therapies, have drastically expanded the life expectancy of CML patients. Delaying therapy could result in diminished life expectancy and poorer quality of life due to the progression of the disease. Therapy, therefore, should not be delayed. Due to the fetal risk of TKI therapy, pregnancy should be avoided. Once TKI therapy has been initiated, it should be continued until adequate hematologic and deep molecular responses are detected for several years. Patients should ideally avoid pregnancy until they meet standard guidelines for a stopping trial. TKIs adversely affect gonadal function, sperm and oocyte maturation, and overall fertility potential in a self-limited manner.
There are insufficient studies regarding the long-term consequences on fertility after the discontinuation of TKIs. Go to the next page if you knew the correct answer, or click the link image(s) below to further research the concepts in this question (if desired).
Research Concepts: Tyrosine Kinase Inhibitors
We update eBooks quarterly and Apps daily based on user feedback. Please tap flag to report any questions that need improvement.
Question 690:
A 50-year-old woman is diagnosed with clinical stage I invasive ductal breast cancer of her right breast that is estrogen receptor (ER) positive, progesterone receptor (PR) positive, and Her2 negative/nonamplified. Family history is significant for breast cancer in her mother and two sisters and prostate cancer in her brother. Physical examination is remarkable for a palpable 1.5 cm mass in the upper outer quadrant of her right breast. During the workup, the patient is found to have a pathogenic BRCA 1 mutation. Which treatment option will provide the most risk reduction with regards to future risk of breast and other cancers in this patient?
Choices: 1. Ipsilateral breast-conserving surgery with adjuvant radiotherapy 2. Ipsilateral mastectomy with axillary node dissection 3. Bilateral mastectomy alone 4. Bilateral mastectomy with bilateral oophorectomy
Answer: 4 - Bilateral mastectomy with bilateral oophorectomy
Explanations: A bilateral mastectomy will treat this patient's breast cancer and offer a high level of protection against ipsilateral recurrence or contralateral occurrence of new breast cancer. Unfortunately, even a bilateral mastectomy procedure does not bring the risk of developing future breast cancer to zero. Bilateral oophorectomy offers additional high-level protection against the development of ovarian cancers in women with BRCA mutation. Neoadjuvant or adjuvant therapies, including neoadjuvant or adjuvant chemotherapy, postmastectomy radiation, and adjuvant endocrine therapy, depend on the type of breast cancer (the type of breast cancer is based on ER, PR, and Her2 status). Go to the next page if you knew the correct answer, or click the link image(s) below to further research the concepts in this question (if desired).
Research Concepts: BRCA 1 and 2
We update eBooks quarterly and Apps daily based on user feedback. Please tap flag to report any questions that
need improvement.
Question 691:
A 55-year-old woman with significant past medical history presents to the clinic with a complaint of progressive rigidity and pain in her neck, upper and lower limbs. She describes bouts of severe spasms that are very painful and are often triggered by unexpected sounds or touch. On examination, she has normal cranial nerves, normal strength, generalized rigidity, and hyperreflexia in her upper and lower limbs. She has an exaggerated startle to tactile and auditory stimuli. Her sensory examination is normal. Magnetic resonance imaging (MRI) of the brain and spine are unremarkable. Laboratory analysis shows positive anti-amphiphysin antibodies. Which of the following conditions is most commonly associated with this patient's disorder?
Choices: 1. 2. 3. 4.
Breast adenocarcinoma Papillary thyroid carcinoma Renal cell carcinoma Non-Hodgkin lymphoma
Answer: 1 - Breast adenocarcinoma Explanations: Paraneoplastic stiff-person syndrome (SPS) represents 5%-10% of all SPS cases and is associated with the presence of antibodies against amphiphysin and rarely against gephyrin. Common malignancies associated with paraneoplastic SPS include carcinomas of the breast, lung, colon, and Hodgkin lymphoma. A thorough search for malignancy should be initiated as soon as a paraneoplastic neurological disorder is suspected. The mean interval from symptom onset of paraneoplastic SPS to detection of malignancy reportedly is 9 months. Treatment of underlying malignancy is essential in the management of paraneoplastic SPS as traditional symptomatic, and immunomodulating treatment used for classic SPS is insufficient for paraneoplastic SPS. Go to the next page if you knew the correct answer, or click the link image(s) below to further research the concepts in this question (if desired).
Research Concepts: Stiff Person Syndrome
We update eBooks quarterly and Apps daily based on user feedback. Please tap flag to report any questions that
need improvement.
Question 692:
A 43-year-old female patient presented with a two-month history of fever, nonproductive cough, and dyspnea. The chest CT scan evidenced a 9-mm intraluminal lesion in the left main bronchus, in correspondence to the origin of the lingular segmental bronchus. Histological examination showed tumor islands containing both cystic and solid patterns. Cystic components consisted of cytologically bland columnar cells with mucin and rare mitoses. Solid components consisted of squamoid and intermediate cells, surrounding the cystic areas. Mitoses were limited, and there was no necrosis. A left upper lobectomy was performed. The histopathological examination confirmed the preoperative diagnosis and stage (pT1N1M0). What is the most likely diagnosis?
Choices: 1. 2. 3. 4.
Adenoid cystic carcinoma Low-grade mucoepidermoid carcinoma Adenocarcinoma Adenosquamous carcinoma
Answer: 2 - Low-grade mucoepidermoid carcinoma Explanations: Low-grade mucoepidermoid carcinomas are composed of three cell types: mucin secreting, squamous, and intermediate cells, and often show cystic patterns with solid areas. Tumor islands contain both cystic and solid patterns. Necrosis is rare. The treatment of mucoepidermoid carcinomas is generally surgical, which includes traditional, or sleeve lobectomy. Poor prognostic factors of mucoepidermoid carcinomas include lymph node metastasis, as it is the case in this patient (pT1N1M0), and positive resection margins. The prognosis of low-grade mucoepidermoid carcinomas is excellent. However, high-grade mucoepidermoid carcinomas are more aggressive. Go to the next page if you knew the correct answer, or click the link image(s) below to further research the concepts in this question (if desired).
Research Concepts: Mucoepidermoid Lung Tumor
We update eBooks quarterly and Apps daily based on user feedback. Please tap flag to report any questions that need improvement.
Question 693:
A 65-year-old male patient undergoes an esophagogastroduodenoscopy (EGD) to evaluate the complaint of dyspepsia refractory to proton pump inhibitor (PPI) therapy. During the endoscopy, a 25 mm intraluminal projection is noted. Microscopy of the removed lesion demonstrates a locus of high-grade glandular dysplasia contained within clear margins. Which of the following is the most appropriate next step in management?
Choices: 1. Discontinue PPI therapy. 2. Repeat the EGD in 1 year without further monitoring. 3. Repeat the EGD at one year and again at three years from the initial endoscopy. 4. Treat empirically for Helicobacter pylori infection.
Answer: 2 - Repeat the EGD in 1 year without further monitoring.
Explanations: An adenoma is characterized histologically by low-grade glandular dysplasia, and a high-grade dysplasia indicates progression towards adenocarcinoma. Management of polyps concerning for containing carcinoma or precancerous lesions requires complete excision of the lesion with a microscopic evaluation to ensure clear margins. Gastric polyps larger than 20 mm have a much higher likelihood of progression to adenocarcinoma. Gastric cancer is the third most common cause of cancer-related death worldwide, appropriate management of lesions with precancerous potential is essential to lowering this risk. Go to the next page if you knew the correct answer, or click the link image(s) below to further research the concepts in this question (if desired).
Research Concepts: Gastric Polyp
We update eBooks quarterly and Apps daily based on user feedback. Please tap flag to report any questions that need improvement.
Question 694:
A 51-year-old perimenopausal woman presents to the clinic with symptoms of menorrhagia for six months. She has used several courses of oral progestogens with not much improvement in her symptoms. A transvaginal ultrasound (TVS) shows an increased endometrial thickness. An outpatient endometrial biopsy is performed, which showed increased gland to stroma ratio, disorganization of glands, nuclear atypia, maze-like pattern, back to back glands, and a cribriform pattern. What is the most appropriate management strategy for this patient?
Choices: 1. Medical treatment with oral progestogens for a longer duration 2. Levonorgestrel intrauterine system (LNG-IUS) 3. Discuss in a multidisciplinary meeting involving gynecologist, histopathological, and gyne-oncologist 4. Endometrial ablation
Answer: 3 - Discuss in a multidisciplinary meeting involving gynecologist, histopathological, and gyneoncologist
Explanations: The histopathological findings of the increased gland to stroma ratio, disorganization of glands, and nuclear atypia suggest endometrial hyperplasia with atypia. Histopathological features of the maze-like pattern, back to back glands, and a cribriform pattern are often suggestive of invasion. Endometrial hyperplasia with atypia has a high risk of progression (almost 27%) to endometrial malignancy. The possibility of a concurrent endometrial cancer could be as high as 43%. The endometrial biopsy shows features of invasion. It would be a good practice to discuss the case in a multidisciplinary meeting to decide about the preoperative workup and type of procedure to be performed. Endometrial ablation is not recommended for this entity because of the high risk of progression to cancer. Also, it can lead to endometrial adhesions, which can make endometrial surveillance difficult. Go to the next page if you knew the correct answer, or click the link image(s) below to further research the concepts in this question (if desired).
Research Concepts: Endometrial Hyperplasia
We update eBooks quarterly and Apps daily based on user feedback. Please tap flag to report any questions that need improvement.
Question 695:
A 60-year-old male patient presented with advanced malignancy invading the left main stem bronchus. What is the best method to predict morbidity and mortality after left-sided pneumonectomy?
Choices: 1. 2. 3. 4.
Inquiring about smoking history Renal function panel Preoperative lung imaging Preoperative lung function tests
Answer: 4 - Preoperative lung function tests Explanations: Preoperative pulmonary function tests can best predict complications after pneumonectomy. Forced expiratory volume in one second (FEV1) and diffusion capacity of the lung for carbon monoxide (DLCO) provide the most accurate risk estimates of postoperative morbidity and mortality If the estimated Postoperative FEV1 or DLCO is less than 40% of predicted, the risk of morbidity and mortality is quite high. Lung imaging alone is not used to predict complications. Cardiac risk assessment is also an important predictor of postoperative morbidity and mortality Go to the next page if you knew the correct answer, or click the link image(s) below to further research the concepts in this question (if desired).
Research Concepts: Pneumonectomy
We update eBooks quarterly and Apps daily based on user feedback. Please tap flag to report any questions that need improvement.
Question 696:
A 65-year-old woman underwent a modified radical mastectomy for cancer of the left breast (stage IIB). Invasive lobular carcinoma was diagnosed on histopathological examination. The patient received postoperative endocrine therapy and chemotherapy on an outpatient basis. One year later, the patient consulted for anorexia and abdominal pain. A computed tomography scan of the abdomen showed an enlarged duodenum and dilated intrahepatic biliary and pancreatic ducts. Upper gastrointestinal endoscopy showed a tumor of the duodenum. A duodenal biopsy of the tumor was performed. Histological examination revealed a proliferation of small cells that lack cohesion and appear individually dispersed through a fibrous connective tissue or arranged in single-file linear cords that invade the stroma. The neoplastic cells have round or notched ovoid nuclei and a thin rim of cytoplasm with an occasional intracytoplasmic lumen, often harboring a central mucoid inclusion. Immunohistochemically, the tumor cells were negative for E-cadherin and positive for cytokeratin. What is the most likely diagnosis?
Choices: 1. 2. 3. 4.
Poorly differentiated adenocarcinoma of the duodenum Duodenal small cell lymphoma Metastatic lobular carcinoma to the duodenum Signet ring cell tumor of the duodenum
Answer: 3 - Metastatic lobular carcinoma to the duodenum
Explanations: Lobular carcinoma often metastasizes to the ovary, gastrointestinal tract, uterus, serosal cavities, meninges, or bone than to the regional lymph nodes or lungs. Although many case reports have debated whether gastrointestinal and peritoneal metastases of invasive lobular carcinoma are lymphomatous or hematogenous, there is a lack of definitive evidence. Metastases of invasive lobular carcinoma show poor survival. As for treatment, palliative surgery is often performed to narrow the diagnosis and improve symptoms. In this case, the final histological diagnosis was metastatic lobular carcinoma to the duodenum because the patient had a history of invasive lobular carcinoma of the left breast. Moreover, the morphological, architectural, and immunohistochemical features of the duodenal tumor cells were highly suggestive for metastatic lobular carcinoma of the breast to the duodenum. Go to the next page if you knew the correct answer, or click the link image(s) below to further research the concepts in this question (if desired).
Research Concepts: Lobular Breast Carcinoma
We update eBooks quarterly and Apps daily based on user feedback. Please tap flag to report any questions that need improvement.
Question 697:
A 68-year-old woman presents with a 3day history of frequent urination, nausea, anorexia, and confusion. She takes acetaminophen daily for back pain. On physical examination, she is not oriented to time or place, and conjunctival pallor is noted. Her laboratory results show a hemoglobin concentration of 8.9 g/dL, serum calcium of 13.5 mg/dL, and serum creatinine of 1.9 mg/dL. Her erythrocyte sedimentation rate is 65 mm/hour. Which of the following is the most likely cause of this patient's condition?
Choices: 1. 2. 3. 4.
Increased intake of calcium Excessive proliferation of plasma cells Increased production of calcitriol Excess parathyroid hormone secretion
Answer: 2 - Excessive proliferation of plasma cells Explanations: This patient presents with features of multiple myeloma (MM). It is a plasma cell malignancy in which monoclonal plasma cells in bone marrow proliferate, resulting in the destruction of bone, an overabundance of monoclonal paraprotein, and displacement of other hematopoietic cell lines. This patient has an increased calcium level, renal dysfunction, anemia, and bone pain caused by the proliferation of plasma cells in MM. Hypercalcemia is also caused by humoral mechanisms. Isolated plasmacytomas affect 2-10% of patients and lead to hypercalcemia by producing osteoclastactivating factor. Plasma cell proliferation causes skeletal damage resulting in osteolytic lesions, bone pain, bone thinning, and hypercalcemia. Go to the next page if you knew the correct answer, or click the link image(s) below to further research the concepts in this question (if desired).
Research Concepts: Plasmacytoma
We update eBooks quarterly and Apps daily based on user feedback. Please tap flag to report any questions that
need improvement.
Question 698:
A 34-year-old female presents to the office with complaints of hearing loss for one month. She has decreased hearing in bilateral ears. She also reports early morning headaches. She denies fever, ear discharge, numbness/tingling sensation, and trauma. She has a family history of a brain tumor in her father. On examination, the vitals are stable. Weber and Rinne's tests show bilateral sensorineural hearing loss. MRI of the brain shows mass near the internal acoustic meatus on both sides. Which of the following drugs can be used in the treatment of this condition?
Choices: 1. 2. 3. 4.
Denosumab Bevacizumab Palivizumab Emicizumab
Answer: 2 - Bevacizumab Explanations: This patient has bilateral sensorineural hearing loss, most likely due to vestibular schwannoma. Bilateral vestibular schwannoma is most likely suggestive of neurofibromatosis 2. Multidisciplinary management is required in patients with NF2, including oncologists, neurologists, neuroradiologists, ophthalmologists, geneticists, and neurosurgeons. Bevacizumab, a monoclonal antibody against vascular endothelial growth factor (VEGF), is new systemic immunotherapy for a wide range of tumors. Some studies have shown tumor regression and hearing improvement in NF2 patients treated with bevacizumab. Small asymptomatic vestibular schwannomas can be managed conservatively with MRI follow-up. Surgery is the primary treatment for large symptomatic vestibular schwannomas. Go to the next page if you knew the correct answer, or click the link image(s) below to further research the concepts in this question (if desired).
Research Concepts: Neurofibromatosis Type 2
We update eBooks quarterly and Apps daily based on user feedback. Please tap flag to report any questions that need improvement.
Question 699:
A 42-year-old male patient presents to the clinic for a chief complaint of right femur pain. Lab work to include WBC is unremarkable. Physical exam demonstrates mild tenderness to palpation of the right femur. A radiograph of the right lower extremity is obtained and demonstrates a proximal right femur lesion that is welldemarcated and with a ground-glass appearance. Malignant transformation is most likely to occur when the lesion occurs in what part of the body?
Choices: 1. 2. 3. 4.
Femur Pelvis Craniofacial Tibia
Answer: 3 - Craniofacial Explanations: Fibrous dysplasia is a benign bone lesion characterized by fibro-osseous proliferation (intramedullary) secondary to altered osteogenesis. Malignant transformation in fibrous dysplasia is very rare. Malignant degeneration tends to be more common in polyostotic lesions than monostotic lesions. Malignancy is most likely to occur when the lesion occurs in the craniofacial area. The majority of malignancies are osteosarcomas and fibrosarcomas. Go to the next page if you knew the correct answer, or click the link image(s) below to further research the concepts in this question (if desired).
Research Concepts: Fibrous Dysplasia
We update eBooks quarterly and Apps daily based on user feedback. Please tap flag to report any questions that need improvement.
Question 700:
A 65-year-old man with metastatic lung cancer to the bone and brain presents to the clinic for follow-up. Initially, he was followed while receiving two rounds of aggressive chemotherapy and radiation treatments over two years, but unfortunately, his cancer was unresponsive to conventional therapies. At this time, he is seeking alternative therapies to manage his symptoms to improve his quality of life. He also expresses concern over his finances and the burden he will leave on his family after he dies. The clinician and patient are considering art therapy as an adjunct to his current palliative care regimen. According to an observational study utilizing a modified version of the Edmonton Symptom Assessment System (ESAS) self-assessment tool, which of the following is most likely to be reduced during this patient's treatment with art therapy?
Choices: 1. 2. 3. 4.
Dyspnea Anorexia Pain Cachexia
Answer: 3 - Pain Explanations: Beyond the aspects of physical illness, palliative care, as defined by the World Health Organization, strives to implement “means of early identification and impeccable assessment and treatment of pain and other problems, physical, psychosocial and spiritual.” Art therapy is a clinical intervention that utilizes the expressive qualities of art-making to improve physical, mental, and emotional well-being. Cancer patients invariably seek complementary therapies as adjuncts to their oncologic treatments. The average cancer patient reports up to a dozen symptoms, including fatigue, pain, anorexia, cachexia, dyspnea, and anxiety. An observational study found an overwhelming reduction in global distress and “reduced pain, anxiety, ill-being, tiredness, sadness, and depression,” utilizing a modified version of the well-recognized Edmonton Symptom Assessment System (ESAS) self-assessment tool before and after each art-therapy session. Women with breast cancer have been a population of focus for identifying the healing power in art therapy. Several randomized control trials have shown a recognized improvement in health-related quality of life, a decrease in symptoms of distress, improvement in coping skills in multiple psychosocial domains, future perspectives, and body image, and reductions in depression, anxiety, and somatic symptoms. Go to the next page if you knew the correct answer, or click the link image(s) below to further research the concepts in
this question (if desired).
Research Concepts: Art Therapy
We update eBooks quarterly and Apps daily based on user feedback. Please tap flag to report any questions that need improvement.
Section 8 Question 701:
A 68-year-old man presents to the oncologist with a brown nodular lesion on his right arm. Biopsy demonstrated microcystic morphology, and the pathologist is suspicious for eccrine carcinoma. Which of the following best describes the incidence of eccrine carcinoma (EC) compared to other cutaneous malignancies?
Choices: 1. 2. 3. 4.
1% 5% 5-10% >10%
Answer: 1 - 1% Explanations: EC is appendageal cancer that originates from the eccrine sweat glands. EC is a rare carcinoma of the skin and accounts for 0.005–0.01% (1%) of diagnosed cutaneous malignancies. Proper identification of EC is sometimes challenging due to the morphological similarity to other common tumors and the lack of consistent immunohistochemical markers. EC presents as brown, bluish, erythematous lesions which may appear papular or ulcerative. Go to the next page if you knew the correct answer, or click the link image(s) below to further research the concepts in this question (if desired).
Research Concepts: Eccrine Carcinoma
We update eBooks quarterly and Apps daily based on user feedback. Please tap flag to report any questions that need improvement.
Question 702:
A 70-year-old man with a history of prior myocardial infarction with the placement of drugeluting stents and diabetes presents to the emergency department with crushing chest pressure starting 2 hours ago. His electrocardiogram is notable for ST elevations in leads I, aVL, and reciprocal depressions in leads II, III, aVF. The patient is emergently taken for a left heart catheterization. He is noted to have an embolus in the proximal left anterior descending artery. His prior stent in the right coronary artery is patent. An aspiration thrombectomy is performed to remove the embolus. He is then referred for a transthoracic echocardiogram, which reveals a patent foramen ovale, and a 40 mm mass on the aortic valve on the left coronary cusp. What is the next best step in the management of this patient?
Choices: 1. Send the patient for emergent cardiac surgery to fix the patent foramen ovale (PFO) 2. Obtain transesophageal echocardiogram to evaluate the cardiac mass better 3. Implant an internal cardiac defibrillator for risk of sudden cardiac death 4. Reassurance
Answer: 2 - Obtain transesophageal echocardiogram to evaluate the cardiac mass better
Explanations: Transesophageal echocardiogram (TEE) is the correct answer because it provides better spatial resolution than a TTE and helps narrow the differential diagnosis of an aortic valve mass, which includes valve thrombus, endocarditis, and papillary fibroelastoma (PFE). On TEE, PFE should have a distinct appearance of a mass on a stalk with a shimmering border. Before surgery, you need to be sure of the diagnosis, which requires further imaging evaluation. PFO is a distractor. While a venous side thrombus can traverse potentially to the left atrium via a PFO, the probability is lower with a 40 mm mass sitting much closer to the coronaries in the aortic root. While PFEs can increase the risk of sudden cardiac death, there is no data to support the routine use of ICD implantation. The patient became symptomatic from PFE and does not need further evaluation and intervention. Go to the next page if you knew the correct answer, or click the link image(s) below to further research the concepts in this question (if desired).
Research Concepts: Papillary Fibroelastoma
We update eBooks quarterly and Apps daily based on user feedback. Please tap flag to report any questions that need improvement.
Question 703:
A 55-year-old Chinese man presents to the clinic complaining of reduced right-sided hearing and a sensation of right aural fullness for over a month. a rightsided, moderate, conductive hearing loss is demonstrated on pure tone audiometry. On further history, he mentions he has been unable to breathe through his nose for approximately 1 month, with no recent history of coryzal symptoms besides a sore throat. He works as a banker and occasionally deep-sea dives for sport. On otoscopy, there are normal external ear canals bilaterally, with an air-fluid level detected on examination of the right tympanic membrane. His left middle ear exam is normal. Which of the following best describes the pathophysiological mechanism of the patient's symptoms?
Choices: 1. 2. 3. 4.
Acute otitis media Barotrauma to the middle ear A patulous Eustachian tube Nasopharyngeal neoplasm
Answer: 4 - Nasopharyngeal neoplasm Explanations: Nasopharyngeal carcinoma should be a clinician’s differential for an adult presenting with refractory unilateral conductive hearing loss due to a middle ear effusion, with or without associated blocked nose/bloody nasal discharge. Nasopharyngeal cancer is most commonly squamous cell carcinoma on histology. Risk factors for nasopharyngeal carcinoma include Epstein Barr virus infection, family history, diets high in salt-cured meat or fish, South Chinese ethnicity, and occupational exposure to hardwood dust. Cancer of the nasopharynx obstructs the nasopharyngeal orifice of the eustachian tube, leading to eustachian tube dysfunction and its sequelae. While repeated barotrauma can lead to middle ear effusions and resultant conductive hearing loss, this is most often a bilateral phenomenon. Unilateral ear effusion in an adult, particularly with other risk factors, should prompt nasopharyngeal examination to rule out a neoplasm. Go to the next page if you knew the correct answer, or click the link image(s) below to further research the concepts in this question (if desired).
Research Concepts: Eustachian Tube Dysfunction
We update eBooks quarterly and Apps daily based on user feedback. Please tap flag to report any questions that need improvement.
Question 704:
A 45-year-old man with a history of neurofibromatosis type (NF) presents to the clinic with headaches, tremors, periodic palpitation, jitters, and diarrhea. He also reports recent unintentional weight loss. Vital signs show blood pressure 127/95 mmHg, pulse rate 130/min, and respiratory rate 15/min. On examination, the thyroid is not enlarged and the lungs are clear to auscultation bilaterally. The heart examination is noted for tachycardia, but no abnormal heart sounds. The extremities are slightly shaking with no edema. Laboratory results are shown in the table. Which one of the following tests is most helpful in making a diagnosis in this patient? Patient Reference Range Units Hb 12.1 13.2-16.5 g/dL Na 137 135-145 mmol/L K 4.0 3.5-5.0 mmol/L Cl 103 96-106 mmol/L CO2 22 22-28 mmol/L BUN 8.0 6-24 mg/dL Cr 0.7 0.6-1.2 mg/dL Ca 9.5 8.6-10.3 mg/dL TSH 0.64 0.5-4.5 IU/mL free T4 1.1 0.8-1.8 ng/dL free T3 2.4 2.0-4.9 pg/mL
Choices: 1. 2. 3. 4.
Plasma total catecholamines Plasma free metanephrines Urine vanillylmandelic acid Urine homovanillic acid
Answer: 2 - Plasma free metanephrines Explanations: Pheochromocytomas are catecholamine-producing tumors derived from chromaffin cells of the adrenal medulla. Catecholamine secretion from these tumors is often episodic, causing headaches, sweating, palpitations, and hypertension. The diagnosis relies on biochemical evidence of excess catecholamine secretion and confirmation of tumor presence by imaging studies. Many different biochemical tests have been used in screening for pheochromocytomas, measurements of the catecholamine breakdown products metanephrine and normetanephrine in plasma and urine are now regarded as first-line tests. Marked elevation of either of these metabolites is associated with a nearly 100% probability of pheochromocytoma. However, it can be challenging to differentiate between true-positive and false-positive results when metanephrine or normetanephrine concentrations are only slightly above the upper limit of the respective reference intervals. Germline mutation syndromes such as neurofibromatosis type 1 (NF1), Von Hippel-Lindau (VHL), and multiple endocrine neoplasia type 2 (MEN2) have been shown to cause pheochromocytomas, Go to the next page if you knew the correct answer, or click the link image(s) below to further research the concepts in this question (if desired).
Research Concepts:
Pheochromocytoma We update eBooks quarterly and Apps daily based on user feedback. Please tap flag to report any questions that need improvement.
Question 705:
An 82-year-old man presents with evidence of metastatic lesions and extra-hepatic disease to the lung on CT imaging. Which of the following indicates the poorest prognosis for this patient?
Choices: 1. Nodal metastasis 2. CEA of 5 ng/mL 3. Two metastatic lesions to the liver from a known colorectal primary tumor 4. Recurrence of disease following prior treatment for colon cancer
Answer: 4 - Recurrence of disease following prior treatment for colon cancer
Explanations: Recurrence of disease after prior treatment indicates the poorest prognosis for a cure. CEA greater than 10 ng/mL is considered a very poor sign. Greater than six colorectal liver metastatic lesions are also a poor sign. Nodal metastasis has an unclear prognostic value and would depend on its location (locoregional or distant) in the metastatic setting. Go to the next page if you knew the correct answer, or click the link image(s) below to further research the concepts in this question (if desired).
Research Concepts: Liver Metastasis
We update eBooks quarterly and Apps daily based on user feedback. Please tap flag to report any questions that need improvement.
Question 706:
A 65-year-old male, diagnosed with an oncological disease starts chemotherapy. After about one month from the start of the therapy, the patient comes to the oncology clinic and reports that he has alterations in the sensitivity of the hands. Which structural alteration of the nerve cell can explain this common sensitivity disorder in patients undergoing chemotherapy?
Choices: 1. 2. 3. 4.
Chemical synapses and axonal compartment Electrical synapses and axonal compartment Mitochondrial permeability transition pore system Axonal compartment
Answer: 4 - Axonal compartment Explanations: The peripheral neurotoxicity due to the anticancer agents is also referred to as chemotherapy-induced peripheral neuropathy. This disease has multiple mechanisms, involving the axon compartment. Structurally, pathological changes in the myelin sheath and, in turn, axonal degeneration are implicated. Microtubule alterations, calcium-related neurotoxicity, and impairment of axonal transport are the main pathogenetic events. Chemical or electrical synapses, as well as the mitochondrial system, are not involved. Go to the next page if you knew the correct answer, or click the link image(s) below to further research the concepts in this question (if desired).
Research Concepts: Histology, Axon
We update eBooks quarterly and Apps daily based on user feedback. Please tap flag to report any questions that need improvement.
Question 707:
A 55-year-old man is admitted to the hospital with vomiting and 20-pound (9 kg) unintentional weight loss over the last few months. During the hospital admission, he is diagnosed with stage IV pancreatic cancer. The healthcare provider recommends hospice care due to the patient's poor prognosis. Instead of being sad, the patient spends his time in the hospital looking up every internet article he can find about the disease, and he continues to ask the healthcare providers about specific details of the condition and treatment. Which of the following is seen in the patient's behavior?
Choices: 1. 2. 3. 4.
Displacement Suppression Repression Intellectualization
Answer: 4 - Intellectualization Explanations: Intellectualization is a common psychological defense mechanism when a patient uses excessive thinking to cope with an unacceptable impulse, situation, or behavior. This patient's behavior demonstrates intellectualization to avoid the stress and anxiety of his poor prognosis. According to the psychodynamic (psychoanalytic) theory, defense mechanisms are used by the ego to resolve conflicts. Suppression occurs when someone consciously chooses to block ideas or impulses that are undesirable, as opposed to repression, which is a subconscious process. Displacement occurs when someone transfers their emotional reaction from one entity to another. Go to the next page if you knew the correct answer, or click the link image(s) below to further research the concepts in this question (if desired).
Research Concepts: Defense Mechanisms
We update eBooks quarterly and Apps daily based on user feedback. Please tap flag to report any questions that need improvement.
Question 708:
A 75-year-old woman with a history of 49 pack-years smoking is diagnosed with laryngeal squamous cell cancer (T1N2M0). She undergoes successful induction chemotherapy with docetaxel, cisplatin, and 5fluorouracil for three cycles. Subsequently, she receives chemoradiation therapy with carboplatin and achieves a complete response. Unfortunately, several months later, there is a recurrence of disease as a solitary lung nodule, which is treated with wedge resection and adjuvant radiotherapy. She then has another recurrence four months later in the mediastinal nodes. She is then treated with single-agent cetuximab. After her first treatment with cetuximab, she returns to the emergency department several hours later with high-grade fevers, nausea, vomiting, neck stiffness, and excruciating headaches. She also has photophobia and phonophobia. CT of the head is unremarkable. She is started on board spectrum IV antibiotics, and a lumbar puncture is performed. CSF fluid is cloudy, with elevated protein (2.9 g/L; normal range: 0.2–0.4 g/L), leukocyte count of 49340/microL (normal range: 0– 4/microL), with 90 % neutrophils and 10% lymphocytes. Bacterial cultures and viral cultures are negative. What is the most likely diagnosis?
Choices: 1. 2. 3. 4.
Meningeal carcinamatosis Aseptic meningitis Bacterial meningitis CNS vasculitis
Answer: 2 - Aseptic meningitis Explanations: Cetuximab-induced aseptic meningitis is a rare adverse reaction, which usually occurs during the first administration, which may suggest a dose-related response. The symptoms of aseptic meningitis generally occur within the first 24 hours of starting treatment. To avoid any adverse reactions, prior to the first infusion, all patients must receive premedication with an antihistamine and a corticosteroid at least 1 hour prior to the administration of cetuximab. This premedication is recommended prior to all subsequent infusions as well. A CSF pattern with neutrophilic pleocytosis may raise concerns of an infectious/bacterial meningitis. Usually, resolution occurs within several days of drug discontinuation. Aseptic meningitis is diagnosed with sterile viral and bacterial CSF profiles. The absence of lesions on CT makes the diagnosis of meningeal carcinomatosis unlikely. Go to the next page if you knew the correct answer, or click the link image(s) below to further research the concepts in this question (if desired).
Research Concepts: Cetuximab
We update eBooks quarterly and Apps daily based on user feedback. Please tap flag to report any questions that need improvement.
Question 709:
A 65-year-old man with a previous history of prostate cancer 12 years ago treated with brachytherapy and androgen deprivation therapy presents to the clinic with mild back pain. Further evaluation shows a prostate-specific antigen (PSA) level of 12 ng/mL (which was undetectable before). Pelvic computed tomography(CT) scan shows an enlarged right iliac lymph node measuring 2.9 x 2.5 cm. The biopsy of the lesion shows metastatic castration-resistant prostate cancer. Which of the following therapies is most likely to improve the survival of this patient?
Choices: 1. 2. 3. 4.
Leuprolide Sipuleucel-T Omalizumab Rilimogene galvacirepvec/rilimogene glafolivec
Answer: 2 - Sipuleucel-T Explanations: Three pivotal, phase-3 studies, showed that the administration of Sipuleucel-T every two weeks for a total of 3 doses, in patients with androgen-independent prostate cancer, increased the median survival by four months (p=0.01) when compared to placebo. Therefore, it was approved by the FDA for asymptomatic or minimally symptomatic androgen resistant metastatic prostate cancer. After showing the benefit in mortality of patients with metastatic castration-resistant prostate cancer, Sipuleucel-T is now being studied in earlier stages of prostate cancer. Sipuleucel-T is an immunological agent composed of fusion protein combining prostatic acid phosphatase (AP) with granulocyte-macrophage colony-stimulating factor (GM-CSF). PROSTVAC (rilimogene galvacirepvec/rilimogene glafolivec) is viral vector-based immunotherapy that was developed to treat prostate cancer; however, a phase III pivotal trial showed no benefit in the overall mortality of patients with metastatic castration-resistant prostate cancer. Go to the next page if you knew the correct answer, or click the link image(s) below to further research the concepts in this question (if desired).
Research Concepts: Acid Phosphatase
We update eBooks quarterly and Apps daily based on user feedback. Please tap flag to report any questions that need improvement.
Question 710:
A 60-year-old man with a history of Paget disease is diagnosed with secondary osteosarcoma of his pelvis. He is currently undergoing neoadjuvant chemotherapy which has led to symptoms of nausea and vomiting that are not controlled by standard antiemetics. Nabilone is prescribed. The patient is informed about the possibility of nabilone causing psychoactive adverse effects. Which of the following best describes the mechanism of action of these adverse effects?
Choices: 1. THC binding with system 2. THC binding with system 3. THC binding with nervous system 4. THC binding with nervous system
cannabinoid 1 receptors in the immune cannabinoid 2 receptors in the immune cannabinoid 1 receptors in the central cannabinoid 2 receptors in the central
Answer: 3 - THC binding with cannabinoid 1 receptors in the central nervous system
Explanations: Nabilone is a cannabinoid antiemetic that acts in the central nervous system. THC acts as an agonist at both cannabinoid 1 (CB1) and cannabinoid 2 (CB2) receptors. THC binding with CB1 receptors in the central nervous system can cause psychoactivity. CB1 receptors are present throughout the body but are primarily found in the central nervous system. CB2 receptors are primarily found in the immune system and have not been linked to psychoactivity. Go to the next page if you knew the correct answer, or click the link image(s) below to further research the concepts in this question (if desired).
Research Concepts: Medical Cannabis Use In Oncology
We update eBooks quarterly and Apps daily based on user feedback. Please tap flag to report any questions that need improvement.
Question 711:
A 67-year-old female presented to the emergency department with complaints of pain in the right upper quadrant of the abdomen with a low-grade fever for three days. She also had a seven months history of dyspepsia. On physical examination, her right upper quadrant was tender, but there was no rebound tenderness, and murphy’s sign was also absent. Initial lab reports revealed mild elevation in PT and INR; however, all other lab results were normal. Additionally, ultrasound of the abdomen was performed, which demonstrated thickening of the gallbladder (GB) wall and calculus of about 2.5 cm in size. Then, computed tomography of the abdomen was done, which revealed a distended GB with thickened wall, fluid collection around GB, and calculus around the neck of GB. Open cholecystectomy was done, and the specimen was sent for histopathology. Histopathological findings were consistent with T2N0 (stage 2) malignancy. What is the most common risk factor for this disease in this patient?
Choices: 1. 2. 3. 4.
Diabetes Advanced age Gallstones Cholangitis
Answer: 3 - Gallstones Explanations: Chronic inflammation is the most important risk of gallstones (GC). The strongest correlation to develop GC is a history of gallstones (cholelithiasis), and the risk increases with gallstone size, chronicity, and burden of symptoms. Therefore, the most common risk factor for the development of gallbladder cancer is the presence of gallstones. The larger the gallstones, the higher the risk. Gallstones larger than 3 cm have a 10 fold greater risk than those, which are less than 1 cm. Despite a large number of people with gallstones, the incidence of gallbladder cancer is very low. Most of these cancers tend to occur in middle-aged females and have often spread at the time of diagnosis. Go to the next page if you knew the correct answer, or click the link image(s) below to further research the concepts in this question (if desired).
Research Concepts: Gallbladder Cancer
We update eBooks quarterly and Apps daily based on user feedback. Please tap flag to report any questions that need improvement.
Question 712:
A 4-year-old boy was referred to the pediatric urology clinic with the complaint of massive hematuria for the last couple of months. His laboratory tests are WBC: 6700 cells/mm^3, Hb: 8.8 mg/dl, and platelet: 176000 cells/microliter. Cystoscopy and biopsy reveal the diagnosis. What is the next preferred management?
Choices: 1. MRI of the brain, chest x-ray, and abdominal CT scan 2. MRI of the brain, CT scan of the chest with and without contrast 3. CT of the chest and abdominal 4. Abdominal MRI and brain CT scan
Answer: 3 - CT of the chest and abdominal Explanations: Rhabdomyosarcomas are malignancies of connective tissue, especially striated or skeletal muscle. Rhabdomyosarcomas can occur in any part of the body. Embryonal rhabdomyosarcomas occur in children and are often found either in the head or genital area. Evaluation of the rhabdomyosarcoma should exclude possible metastatic sites, especially lung, and liver, which should be examined via chest, and abdominal CT scan, respectively. Although pediatric rhabdomyosarcomas tend to metastasize to the brain more than other solid malignancies, brain MRI is not among the primary essential investigations. Go to the next page if you knew the correct answer, or click the link image(s) below to further research the concepts in this question (if desired).
Research Concepts: Rhabdomyosarcoma
We update eBooks quarterly and Apps daily based on user feedback. Please tap flag to report any questions that need improvement.
Question 713:
A 10-year-old male presents for pain and swelling of his left jaw for six months. During this period, he had intermittent fevers, for which he was treated with antibiotics. He denies trauma, gum bleeding, weight loss, and lumps or bumps anywhere. On examination of his oral cavity, there is a 4 X 4 cm hard, immobile, and tender mass on the angle of the left jaw. Multiple loose teeth are also present around the mass. No regional lymphadenopathy is appreciated. The abdominal exam is unremarkable, including no hepatosplenomegaly. Complete blood count with differential is normal. X-ray of the oral cavity shows a lytic lesion with soft tissue extension. Biopsy of the mass and subsequent staining with CD1a is positive. Which of the following mutations is the most likely cause of this patient's suspected condition?
Choices: 1. 2. 3. 4.
KRAS BRAF C-myc N-myc
Answer: 2 - BRAF Explanations: This patient with an isolated lytic lesion of the mandible and biopsy showing CD1a positive staining is most suggestive of eosinophilic granuloma of the mandible. The most common mutation causing eosinophilic granuloma is BRAF V600E mutation, which is present in more than 50% of the patients. The differential diagnosis for this lytic lesion includes osteomyelitis, lymphoma, and leukemia. Eosinophilic granuloma can be treated with curettage and bone grafting. Go to the next page if you knew the correct answer, or click the link image(s) below to further research the concepts in this question (if desired).
Research Concepts: Eosinophilic Granuloma
We update eBooks quarterly and Apps daily based on user feedback. Please tap flag to report any questions that need improvement.
Question 714:
A 12-year-old male attends the clinic with multiple hard, painless lumps inside his mouth. He has no other medical problems. He was born at term and had all of his childhood immunizations. He is estranged from his father; his maternal family history is unremarkable. On examination, minimal swelling can be seen extra-orally. Intra-oral examination shows bilateral, multiple hard bony swellings originating from a narrow base on both the buccal and lingual aspect of the mandible and a single swelling on the maxilla. Orthopantomogram shows multiple radiopaque lesions and a cotton wool appearance of the mandible. He also has multiple supernumerary teeth. What is the most important investigation to improve this patient’s prognosis?
Choices: 1. 2. 3. 4.
Core biopsy of lesions Colonoscopy Magnetic resonance imaging of the brain Positron emission tomography (PET) scan
Answer: 2 - Colonoscopy Explanations: Multiple osteomas in a child must always prompt consideration of Gardener syndrome, especially as osteoma may precede other symptoms. Gardener’s syndrome is an autosomal dominant condition caused by a mutation in the APC gene. It leads to osteomas, desmoid tumors, lipomas, supernumerary teeth, and, most importantly, widespread adenomas within the gastrointestinal tract. As the multiple gastrointestinal adenomas invariably undergo malignant transformation, surveillance with regular endoscopy is vital for early diagnosis and treatment of adenocarcinoma. Osteomas are benign and slow-growing and can be monitored or treated for functional or aesthetic concerns. Treatment involves close margin surgical excision. Go to the next page if you knew the correct answer, or click the link image(s) below to further research the concepts in this question (if desired).
Research Concepts: Non-Odontogenic Tumors Of The Jaws
We update eBooks quarterly and Apps daily based on user feedback. Please tap flag to report any questions that
need improvement.
Question 715:
A 32-year-old female patient was referred to the surgery clinic for an incidental finding in her abdominal CT scan. She has undergone an abdominal CT scan a couple of weeks ago to evaluate an acute colicky pattern of right flank pain while she was diagnosed with a 2.7 in 2.3 cm left adrenal mass and right distal ureteral 5 mm stone. She passed the stone 2 days afterward. She denied any other accompanying symptoms. What is the next preferred management?
Choices: 1. 2. 3. 4.
Obtaining a low dose dexamethasone suppression test Request an abdominal MRI with intravenous contrast Schedule for a laparoscopic adrenalectomy Check plasma renin activity and aldosterone level
Answer: 1 - Obtaining a low dose dexamethasone suppression test
Explanations: The majority of adrenal masses are discovered incidentally and thus termed adrenal incidentalomas. These masses are often seen during a radiological procedure for an unrelated procedure. Cost-effective workup of incidental adrenal masses is important and begins with a good history (including a history of past malignancies and signs/symptoms of adrenal hyperfunction). MRI and CT can usually distinguish adrenal adenomas (both lipid-rich and the less common lipid-poor variant) from metastatic disease and primary adrenal malignancy (cortical carcinoma, pheochromocytoma) using dynamic sequences. Assessment of adrenal incidentaloma might initiate with obtaining a low dose dexamethasone suppression test. If the imaging feature of the adrenal mass mostly corresponds to a malignant pathology, and or it has a greater than 4 cm diameter, adrenalectomy might be considered. Go to the next page if you knew the correct answer, or click the link image(s) below to further research the concepts in this question (if desired).
Research Concepts: Adrenal Adenoma
We update eBooks quarterly and Apps daily based on user feedback. Please tap flag to report any questions that need improvement.
Question 716:
A 79-year-old man is brought to the hospital from a nursing home for respiratory distress and loss of voice. He ends up having an emergency tracheostomy. His chart indicates progressive dysphagia and loss of phonation for the past two months. Contrastenhanced CT scan of the neck shows a large, 5 cm laryngeal tumor that has destroyed the thyroid cartilage and is involving vertebral bone as well as surrounding the left carotid artery. Multiple 2-3 cm lymph nodes are noted in bilateral levels II-IV. Biopsy confirms squamous cell carcinoma, and further imaging indicates no distant metastasis. The patient has poor performance status at baseline and is nearly entirely bedbound. What is the most appropriate treatment strategy for this patient?
Choices: 1. Palliative radiation or chemotherapy alone 2. Curative concurrent radiation and chemotherapy 3. Palliative maximal debulking surgery 4. Curative-intent laryngectomy and corpectomy, with likely adjuvant radiation and/or chemotherapy
Answer: 1 - Palliative radiation or chemotherapy alone Explanations: A 5 cm laryngeal tumor that has destroyed the thyroid cartilage and is involving vertebral bone as well as surrounding the left carotid artery, multiple 2-3 cm lymph nodes are noted in bilateral levels II-IV, and no metastatic disease indicates T4bN2cMo squamous cell carcinoma of the larynx, which is by definition inoperable. Patients with this level of disease burden amd with the described poor performance status likely cannot undergo curative-intent therapy. Palliative radiation or chemotherapy ALONE are appropriate possibilities; concurrent chemotherapy and radiation significantly increase treatment toxicity, which is of special concern when treating for palliative intent in a patient with a poor prognosis and likely short-term survival. Even for palliation, surgery would not be considered in this case. A surgery to offer palliation (i.e., to reach deep structures and relieve symptoms) would be too extensive and dangerous. Go to the next page if you knew the correct answer, or click the link image(s) below to further research the concepts in this question (if desired).
Research Concepts: Laryngeal Cancer
We update eBooks quarterly and Apps daily based on user feedback. Please tap flag to report any questions that need improvement.
Question 717:
A 42-year-old man presents to the clinic with a 3-week history of numerous ulcerating papules and nodules on his chest and bilateral upper arms. A biopsy of one of the lesions shows a dense dermal infiltrate composed of large atypical lymphocytes with anaplastic,horseshoeshaped nuclei that are positive for CD3, CD4, and CD30. When the patient returns to the clinic two weeks later, the majority of lesions have disappeared. What is the most likely diagnosis?
Choices: 1. 2. 3. 4.
Lymphomatous polyposis Lymphomatoid papulosis Mycosis fungoides with large cell transformation Primary cutaneous anaplastic large cell lymphoma
Answer: 2 - Lymphomatoid papulosis Explanations: Lymphomatoid papulosis is a chronic skin disorder most commonly seen in adult patients that are characterized by the presence of numerous cutaneous lesions in various stages of development (papules, nodules) that predominantly affect the trunk and extremities. The histologic appearance of the lesions is highly variable and is partly dependent on the age of the lesion at the time of biopsy. It is characterized by a wedgeshaped pattern, with a wide superficial base and the tip at the bottom, usually deep dermis and less frequently into the subcutaneous tissue. The most characteristic histological appearance is the presence of few to numerous large cells with a Hodgkin or Hodgkin-Reed Sternberg admixed with a reactive background of small lymphocytes and less frequently eosinophils, plasma cells, and histiocytes. The morphologic and immunophenotypic features of the lesions overlap with several types of cutaneous lymphoma. Diagnosis is aided by the tendency of the lesions to show spontaneous regression within weeks to months, leaving behind only hypopigmented or hyperpigmented scars. The disease is considered to be benign, but it may last from months to decades. Go to the next page if you knew the correct answer, or click the link image(s) below to further research the concepts in this question (if desired).
Research Concepts:
Lymphomatoid Papulosis
We update eBooks quarterly and Apps daily based on user feedback. Please tap flag to report any questions that need improvement.
Question 718:
A 30-year-old male with a past medical history of recurrent seizures presents with bilateral flank discomfort with no other complaints. His blood pressure is 120/80 mmHg, his heart rate is 90 beats per minute, and his temperature is 37 C (98.6 F). Physical examination reveals some hypopigmented skin patches. His father died of renal disease. Urinalysis results are within normal limits. Computed tomography (CT) head is performed, which shows vascular dysplastic lesions. Renal ultrasound reveals bilateral renal masses with echogenic components and acoustic shadowing. Which one of the following genes is most likely mutated?
Choices: 1. 2. 3. 4.
TSC1 gene DCC gene CFTR gene MYH9 gene
Answer: 1 - TSC1 gene Explanations: Renal angiomyolipomas (AMLs) are considered the most common benign renal neoplasms. Renal AMLs are generally composed of blood vessels, smooth muscles, and adipose tissues. Although most of these tumors are found incidentally on radiological imaging, symptomatic presentations may also exist, such as flank pain, gross hematuria, or severe retroperitoneal hemorrhage. These tumors occur as sporadic or isolated entities, which account for 80 percent of the cases; the remaining 20 percent are developed in association with pulmonary lymphangioleiomyomatosis (LAM) or tuberous sclerosis complex (TSC). Hereditary angiomyolipomas associated with tuberous sclerosis complex (TSC) or pulmonary lymphangioleiomyomatosis (LAM) are caused by a genetic mutation in the TSC1 or the TSC2 gene. Renal angiomyolipomas are observed in about 75% of patients with TSC. The DCC gene is involved in colorectal cancer, the CFTR gene is for cystic fibrosis and the MYH9 gene increases the risk of kidney failure. Go to the next page if you knew the correct answer, or click the link image(s) below to further research the concepts in this question (if desired).
Research Concepts: Renal Angiomyolipoma
We update eBooks quarterly and Apps daily based on user feedback. Please tap flag to report any questions that need improvement.
Question 719:
A 69-year-old female presents to the emergency department complaining of left groin pain for six months that has worsened over the past week. The patient ambulates typically with a walker but has had increasing difficulty ambulating during the last week due to pain with any attempts at weight-bearing. An X-ray of the left hip reveals it is a large lytic lesion that takes up greater than 2/3 the diameter of the bone at the intertrochanteric region. A biopsy is performed, confirming metastatic breast carcinoma. What is the next best treatment option?
Choices: 1. Radiation 2. Observation, weight-bearing as tolerated and proceed with surgical fixation after the fracture displaces 3. Prophylactic fixation 4. Total femur replacement
Answer: 3 - Prophylactic fixation Explanations: This patient's Mirel score is 12, calculated as follows: peri-trochanteric (3 points), functional pain (3 points), the lytic lesion (3 points), size >2/3 the diameter of bone (3 points). A score greater than 8 is at high risk for impending fracture and warrants prophylactic fixation. The Mirel criteria are based on four categories- site, pain, lesion type, and size. Up to 3 points are given per category, with the highest score being 12. Although the Mirel scoring system was first described in 1989, it is still widely used due to its simplicity and reliability. Failure to stabilize an impending pathologic fracture may lead to fracture displacement, complicating surgical intervention. Therefore observation is incorrect. Radiation may be administered for Mirel scores of 7 and lower with low risk of fracture, unlike the case in this vignette. Total femur replacement represents an overly aggressive approach, which will lead to unnecessary morbidity and loss of function and is therefore not indicated. Go to the next page if you knew the correct answer, or click the link image(s) below to further research the concepts in this question (if desired).
Research Concepts: Pathologic Fractures
We update eBooks quarterly and Apps daily based on user feedback. Please tap flag to report any questions that need improvement.
Question 720:
A 67-year-old man with a history of hypertension is being evaluated for unexplained peripheral neuropathy. The past medical history is notable for gastroesophageal reflux disease and chronic kidney disease stage 3 with e GFR 56 mL/min/1.73m2. He takes lisinopril and over-the-counter antacids. He has a 40-pack year history of smoking and consumes alcohol occasionally. His vital signs are within normal limits. Physical examination shows decreased proprioception from the knees downward. Lab data includes hemoglobin 13.5 g/dL, calcium 11.5 mg/dL (reference range: 8.8-10.2 mg/dL), creatinine 1.3 mg/dL (baseline), iPTH 60 pg/mL (reference range: 16-65 pg/mL). SPEP with immunofixation identifies a monoclonal protein band of IgA kappa 1.6 g/dL. The serum-free light chain ratio (SFLC) is 5.01 (0.26-1.65), with kappa predominance. A bone marrow biopsy shows 8% monoclonal plasma cells with kappa light chain restriction. Crosssectional imaging is within normal limits. Which of the following sets of findings predicts the highest risk of progression to multiple myeloma (MM) in this patient?
Choices: 1. Abnormal SFLC, non-IgG M protein, Serum M-protein level > 1.5g/dL 2. Hypercalcemia, renal failure, Serum M-protein> 1.5 g/dL 3. Abnormal SFLC, Non-IgM M protein, Serum M-protein level > 1.5 g/dL 4. Renal failure, hypercalcemia, and abnormal SFLC
Answer: 2 - Hypercalcemia, renal failure, Serum Mprotein> 1.5 g/dL
Explanations: This patient has IgA kappa monoclonal gammopathy of undetermined significance (MGUS). MGUS is defined by the presence of a serum monoclonal protein (M protein) at a concentration 3 g/dL, bone marrow monoclonal plasma cells of 10%, and absence of end-organ damage (HyperCalcemia, Renal insufficiency, Anemia, lytic Bone lesions- CRAB features) related to the clonal proliferation. Serum M-protein level of 1.5 g/dL, non-IgG MGUS (IgA, IgM, IgD), and abnormal SFLC ratio are the established risk factors in the risk stratification model to predict the progression of MGUS to MM, or a related malignancy. This patient has all three risk factors; therefore, he belongs to high-risk MGUS. The 20-year risk of progression to MM is 58%. Similarly, the 20-year risk for two risk factors (highintermediate risk MGUS) is 37%, one risk factor (lowintermediate risk MGUS) is 21%, No risk factors (low-risk MGUS) is 5 percent. Hypercalcemia is PTH-mediated. Though the iPTH is within normal limits, it is inappropriately elevated in the setting of hypercalcemia. The patient has stable CKD, not related to end-organ damage from proliferative process. Go to the next page if you knew the correct answer, or click the link image(s) below to further research the concepts in this question (if desired).
Research Concepts:
Monoclonal Gammopathy Of Undetermined Significance
We update eBooks quarterly and Apps daily based on user feedback. Please tap flag to report any questions that need improvement.
Question 721:
A 65-year-old man is admitted for concerns of dysphagia and weight loss. The patient has a history of Hodgkin lymphoma being treated with chemotherapy with R-CHOP for the last two months. The patient now complains of pain in swallowing solid and liquid food, and he also has been losing weight over the past two months. He also complains of heartburn and epigastric discomfort. Physical exam reveals a thin, frail male, without any rash on his skin. The remainder of the exam was unremarkable. His healthcare provider orders an endoscopy that is positive for large ulcers in the mid to distal esophagus. The pathology of the sample reveals large cells with basophilic intranuclear inclusion bodies. What is the best option for the treatment of this condition?
Choices: 1. 2. 3. 4.
Intravenous ganciclovir Intravenous foscarnet Oral acyclovir Oral pantoprazole
Answer: 1 - Intravenous ganciclovir Explanations: This patient's history is most consistent with cytomegalovirus (CMV) esophagitis. He is immunocompromised with symptoms of odynophagia and weight loss that are consistent with CMV esophagitis. The endoscopy finding of ulcers in the mid to distal esophagus is characteristic, as well as the intranuclear inclusion bodies giving those cells the name of cytomegalic cells characteristic of CMV. The induction therapy for CMV esophagitis is intravenous (IV) ganciclovir at a dose of 10-15 mg/kg per day in 2-3 divided doses daily for three weeks but up to 6 weeks. Although foscarnet may also be used, it is not as effective as ganciclovir or valganciclovir, and should only be used in cases of resistance. Acyclovir is the treatment for herpes simplex esophagitis, and not the first line for CMV esophagitis. Oral proton pump inhibitors (PPI) such as pantoprazole are most useful for functional dysphagia or gastroesophageal reflux disease, but this patient has a pathologic cause due to a viral infection, and a PPI would not be appropriate. Go to the next page if you knew the correct answer, or click the link image(s) below to further research the concepts in this question (if desired).
Research Concepts: Cytomegalovirus Esophagitis
We update eBooks quarterly and Apps daily based on user feedback. Please tap flag to report any questions that need improvement.
Question 722:
A 16-year-old male presents to the clinic with complaints of black and sticky stools for the past 2 weeks. He has also noticed bright red blood in the toilet a few times. The patient’s parents died in a car accident when he was young and was thus raised in an orphanage. His blood pressure is 125/85 mmHg, the pulse is 95/min, respiratory rate is 16/min, and the temperature is 37 C. Examination shows a soft, nontender abdomen with no palpable viscera. Cardiopulmonary examination shows no pertinent findings. Laboratory studies reveal a hemoglobin concentration of 9.0 g/dL. Fecal occult blood testing is positive, and colonoscopy shows hundreds of adenomatous colonic polyps. Other than colorectal carcinoma, which of the following malignancies is most strongly associated with this disorder?
Choices: 1. 2. 3. 4.
Basal cell carcinoma Squamous cell carcinoma of the esophagus Small cell carcinoma of the lung Adenocarcinoma of the duodenum
Answer: 4 - Adenocarcinoma of the duodenum Explanations: With the unavailable family history and presentation of hundreds of polyps on colonoscopy, the patient most likely has familial adenomatous polyposis (FAP). One hundred percent of patients with familial adenomatous polyposis (FAP) who are untreated develop colorectal cancer. As many as 12% of the patients with FAP can develop adenocarcinoma of the duodenum. Other cancers in patients with FAP include periampullary cancers, desmoid tumors, hepatoblastoma, thyroid cancer, gastric cancer, and adrenal cancer. Go to the next page if you knew the correct answer, or click the link image(s) below to further research the concepts in this question (if desired).
Research Concepts: Familial Adenomatous Polyposis
We update eBooks quarterly and Apps daily based on user feedback. Please tap flag to report any questions that need improvement.
Question 723:
A 62-year-old woman presents with fever, night sweats, weight loss, hoarseness, and anterior neck swelling. Examination is notable for an enlarged thyroid without palpable lymphadenopathy. An ultrasound shows multiple hypoechoic thyroid nodules, and a subsequent core needle biopsy demonstrates a diffuse infiltrate of lymphocytic cells that stain positive for CD20. An FDG PET/CT scan shows significant uptake in the thyroid only. Which of the following is an associated risk factor for developing this type of primary malignancy?
Choices: 1. 2. 3. 4.
Hashimoto thyroiditis Graves disease Exposure to ionizing radiation Tubercular lymphadenopathy
Answer: 1 - Hashimoto thyroiditis Explanations: The only associated risk factor for primary thyroid lymphoma is Hashimoto (chronic autoimmune) thyroiditis. The risk is almost 60 times higher in patients with thyroiditis. Hashimoto thyroiditis is more common in females; hence primary thyroid lymphoma is also more common in females. In patients presenting with a rapidly enlarging goiter, especially in the setting of Hashimoto thyroiditis, the diagnosis of primary thyroid lymphoma should be considered. There is no association of Graves disease, colloid goiter, exposure to ionizing radiation, chromosomal abnormalities, or tubercular lymphadenopathy with the development of primary thyroid lymphoma. Go to the next page if you knew the correct answer, or click the link image(s) below to further research the concepts in this question (if desired).
Research Concepts: Thyroid Lymphoma
We update eBooks quarterly and Apps daily based on user feedback. Please tap flag to report any questions that need improvement.
Question 724:
A 17-year-old female presents with a three-day history of joint pain, fever, and rash. The patient says the rash first appeared on her wrists, then spread. She has a past medical history of non–Hodgkin lymphoma, for which she recently received a rituximab infusion. On examination, her temperature is 100.4 F (38 C), and her heart rate is 92 beats per minute. There is a maculopapular skin eruption, and the mucous membranes are unaffected. What is the pathophysiology of this patient's most likely condition?
Choices: 1. 2. 3. 4.
Type II hypersensitivity reaction Type III immune-mediated hypersensitivity reaction IgE-mediated hypersensitivity reaction Type IV cell-mediated hypersensitivity reaction
Answer: 2 - Type III immune-mediated hypersensitivity reaction
Explanations: The patient in the clinical scenario presents with serum sickness, which is a type III hypersensitivity reaction. Drugs, including monoclonal antibody therapies, are common causes of serum sickness. It usually occurs 7 to 14 days after antigen exposure. The rash typically takes days to weeks to resolve once the offending agent is discontinued. Non-steroidal antiinflammatory drugs (NSAIDs), antihistamines, and corticosteroids can help alleviate symptoms in patients with moderate to severe presentations. Go to the next page if you knew the correct answer, or click the link image(s) below to further research the concepts in this question (if desired).
Research Concepts: Serum Sickness
We update eBooks quarterly and Apps daily based on user feedback. Please tap flag to report any questions that need improvement.
Question 725:
A 14-year-old boy, with a history of primary malignant intraocular tumor in infancy, is brought to the emergency department with a 2-month history of pain in his left shoulder. The pain was first noted after playing in the park with friends; however, it has since worsened and become constant. Physical examination reveals a warm, tender mass at the proximal humerus, with overlying skin erythema. The range of movement is impaired. X-ray of the left shoulder and humerus reveal a destructive osseous lesion in the proximal humerus with the "Codman triangle" configuration. Which of the following best describes the tumor and the associated gene mutation in this patient?
Choices: 1. 2. 3. 4.
Osteosarcoma; Rb1 mutation Ewing sarcoma; FET-FLI1 fusion protein Chondrosarcoma; Rb1 mutation Osteosarcoma; DNA repair/surveillance gene mutation
Answer: 1 - Osteosarcoma; Rb1 mutation Explanations: Rb1 tumor suppressor gene mutations are seen in hereditary retinoblastoma, altering cell cycle regulation and cell behavior. Hereditary retinoblastoma is characterized by a primary malignancy of the retina and is the most common eye cancer in children. Osteosarcoma is more common in patients with hereditary retinoblastoma. Ewing sarcoma is characterized by a chromosomal translocation leading to the fusion of a FET protein to an ETS transcription factor, most commonly FLI1. The resulting fusion protein acts to deregulate downstream genes, altering cell behavior. Go to the next page if you knew the correct answer, or click the link image(s) below to further research the concepts in this question (if desired).
Research Concepts: Primary Bone Cancer
We update eBooks quarterly and Apps daily based on user feedback. Please tap flag to report any questions that need improvement.
Question 726:
A 47-year-old female visited the emergency department for bright blood in stools (> 2 months), dizziness, mild shortness of breath, constipation, and anal pain. On physical examination, she was noted to have a partially obstructing palpable mass approximately 2 cm from the anal verge. She was noted to have visible and easily accessible blood vessels on the dorsum of her hands and arms. Her labs showed a hemoglobin of 6.9 mg/dL (12.7-14.5 mg/dL). She did receive 2 of PRBCs, and surgical oncology were consulted. An anoscopy clearly visualized a 3 cm (T3) mass, and a biopsy was performed. She was subsequently discharged. The pathology result showed HPV positive, moderately differentiated squamous cell carcinoma. The patient visited the oncology clinic, and CT scans of chest/abdomen/pelvis showed no metastatic disease and indicated a T2N0MO (Stage IIA). The patient has a past medical history of renal insufficiency and hypertension and has difficulty in swallowing pills due to a hiatus hernia and severe GERD. Which one of the following is the 1st step in management?
Choices: 1. Start chemotherapy soon with mitomycin/5Fu and radiation 2. Start chemotherapy soon with capecitabine 3. Gynecological exam, screening for cervical cancer, and HIV testing 4. Place a port for chemotherapy administration
Answer: 3 - Gynecological exam, screening for cervical cancer, and HIV testing
Explanations: The patient needs a gynecologic exam to rule out other concomitant malignancies such as cervical cancer, as it is also an HPV driven malignancy per NCCN guidelines. As per NCCN guidelines, she also needs HIV testing. The patient can start chemotherapy and radiation after HIV testing and a gynecologic exam completed and cervical cancer ruled out. Concurrent chemotherapy and radiation is the recommended therapy in localized anal cancer. 5-FU with mitomycin is the recommended regimen for chemotherapy used along with radiation. Go to the next page if you knew the correct answer, or click the link image(s) below to further research the concepts in this question (if desired).
Research Concepts: Anal Cancer
We update eBooks quarterly and Apps daily based on user feedback. Please tap flag to report any questions that need improvement.
Question 727:
A 25-year-old male presents for the evaluation of multiple new soft tissue swellings on his forearms and trunk. He was diagnosed with adenocarcinoma of the colon at 16 years of age and continues to receive appropriate treatment. On examination, the soft tissue swellings are soft, nontender, subcutaneous, and exhibit the 'slippage sign.' His mouth is crowded with teeth, and they overlap each other. A bone scan reveals multiple osteomas of the skull and mandible. Which of the following is the most likely diagnosis?
Choices: 1. 2. 3. 4.
Bannayan-Riley-Ruvalcaba syndrome (BRRS) Gardner syndrome Benign symmetric lipomatosis (Madelung disease) Neurofibromatosis 2
Answer: 2 - Gardner syndrome Explanations: Lipomas typically present as soft, solitary, painless, subcutaneous nodules that are mobile and not associated with epidermal change. A characteristic "slippage sign" may be elicited by gently sliding the fingers off the edge of the tumor. They are typically slow-growing and grow to a final stable size of 2 to 3 centimeters. However, they are occasionally greater than 10 centimeters and referred to as "giant lipomas." Lipomas may appear anywhere on the body but tend to favor the fatty areas of the trunk, neck, forearms, and proximal extremities. Multiple lipomas may be the presenting feature of a variety of syndromes. This patient most likely has Gardner syndrome. Gardner syndrome is due to autosomal dominant mutations in the adenomatous polyposis coli (APC) gene. Almost all patients develop adenocarcinomas of the gastrointestinal tract. Cutaneous changes include multiple lipomas or fibromas. Other associated findings include congenital hypertrophy of pigment epithelium of retina, osteomas of the skull, maxilla and mandible, supernumerary teeth, and various malignancies, including papillary thyroid carcinomas, adrenal adenomas, and hepatoblastomas. Gardner syndrome is caused by a mutation in the adenomatous polyposis coli (APC gene), located in chromosome 5q21 (band q21 on chromosome 5). This gene is also mutated in familial adenomatous polyposis (FAP), a more common disease that also predisposes to colon cancer.
Bannayan-Riley-Ruvalcaba syndrome (BRRS) is due to PTEN gene mutations and may represent a pediatric form of Cowden syndrome. Clinical findings include multiple lipomas, intestinal hamartomas, genital lentigines, macrocephaly, and mental retardation. The neurofibromas found in neurofibromatosis 2 would not demonstrate the slippage sign seen in lipomas. Go to the next page if you knew the correct answer, or click the link image(s) below to further research the concepts in this question (if desired).
Research Concepts: Lipoma
We update eBooks quarterly and Apps daily based on user feedback. Please tap flag to report any questions that need improvement.
Question 728:
A 60-year-old female with a neck mass of level III undergoes a fine needle aspiration biopsy. She has no significant smoking history, nor any significant personal or family history of endocrine diseases. She resided in Poland in the 1980s-1990s, but has since relocated to Canada. The pathologist reports amyloid in the specimen. Which condition is most likely in this patient?
Choices: 1. 2. 3. 4.
Pheochromocytoma Medullary thyroid carcinoma Papillary thyroid carcinoma Hashimoto's thyroiditis
Answer: 2 - Medullary thyroid carcinoma Explanations: Medullary thyroid cancer is a rare malignancy that produces calcitonin, and fine-needle biopsy often reveals the presence of amyloid. 25% of these malignancies are associated with a mutation in the RET proto-oncogene. Medullary cancer may also be associated with multiple endoplasmic neoplasia (MEN) 2. Residing near Chernobyl in the 1980s could have increased her risk for radiation-induced thyroid malignancy, of which papillary thyroid carcinoma is the most common. This has not been associated with medullary carcinoma, however. Go to the next page if you knew the correct answer, or click the link image(s) below to further research the concepts in this question (if desired).
Research Concepts: Medullary Thyroid Cancer
We update eBooks quarterly and Apps daily based on user feedback. Please tap flag to report any questions that need improvement.
Question 729:
A 27-year-old young, sexually active woman presents to the clinic for her regular screening for cervical cancer. After placing a speculum into the woman's vagina. the cells collected from the transformation zone of the cervix by using a brush show "atypical squamous cells of undetermined significance favor low-grade squamous intraepithelial lesion." It is positive for human papilloma virus. Which of the following is the next best step to be taken by the provider regarding this patient?
Choices: 1. 2. 3. 4.
Repeat the Pap immediately Repeat the Pap in 6 months Perform colposcopy Test for human papilloma virus
Answer: 3 - Perform colposcopy Explanations: In patients 25 to 29, the provider should proceed to colposcopy immediately when the Pap is reported as atypical squamous cells of undetermined significance, favor low-grade squamous intraepithelial lesion with positive human papillomavirus. Colposcopy is examination of the cervix under stereoscopic magnification, and includes biopsies of suspicious areas. If the biopsy reports as abnormal, definitive therapy is completed. During colposcopy, a biopsy is usually obtained. Go to the next page if you knew the correct answer, or click the link image(s) below to further research the concepts in this question (if desired).
Research Concepts: Abnormal Papanicolaou Smear
We update eBooks quarterly and Apps daily based on user feedback. Please tap flag to report any questions that need improvement.
Question 730:
A 55-year-old male patient presents to the clinic for evaluation of numerous long-standing skin lesions on his face and trunk. He has a past medical history of adenocarcinoma of the colon and multiple keratoacanthomas. The patient tells you that his father died of adenocarcinoma of the colon when he was 60-years-old and had similar skin issues. On examination, you find numerous 1-4 mm yellow papules. The lesions are asymptomatic and have been present for more than 10 years. On performing a shave biopsy of several of the small yellow lesions, which of the following is the most likely to be the origin of cells in the lesions?
Choices: 1. 2. 3. 4.
Eccrine Sebaceous Apocrine Adipose
Answer: 2 - Sebaceous Explanations: Muir-Torre syndrome is autosomal dominant and encompasses tutors of sebaceous origin including sebaceous epitheliomas, sebaceous adenomas, and sebaceous carcinomas. Additionally, patients can develop multiple keratoacanthomas and visceral malignancies. Muir-Torre syndrome is due to a mutation in DNA mismatch repair genes. The most common mutation is in MHS2. Adenocarcinoma of the colon is the most common malignancy in Muir-Torre syndrome. Patients also have an increased risk of developing genitourinary tract, lung, breast, and hematologic malignancies. Sebaceous adenomas typically appear as small yellow benign-appearing papules on the head, neck, or trunk. Go to the next page if you knew the correct answer, or click the link image(s) below to further research the concepts in this question (if desired).
Research Concepts: Muir-Torre Syndrome
We update eBooks quarterly and Apps daily based on user feedback. Please tap flag to report any questions that need improvement.
Question 731:
A 72-year-old man presents to the clinic with a thyroid nodule. His TSH is measured to be 1.9 mU/L. Fine needle aspiration shows follicular adenoma with Hurthle cells. A radioiodine scan is done which does not show any iodine avid lesions in the body. A thyroidectomy is performed, which shows Hurthle cell carcinoma with capsular and vascular invasion. Ultrasound of the neck does not show any cervical lymphadenopathy. FDG-PET done postoperatively does not show any evidence of metastatic disease. Next-generation sequencing on the tumor shows he has a RET fusion. He is advised surveillance at this time. Which of the following is the most appropriate for first-line therapy for this patient if he develops widespread metastatic disease?
Choices: 1. 2. 3. 4.
Lenvatinib Radioactive Iodine Selpercatinib Pembrolizumab
Answer: 3 - Selpercatinib Explanations: This patient has Hurthle cell carcinoma with a RET fusion. If possible, targeted therapy is always the best option for treatment since it targets specific mutations. LIBRETTO-001 clinical trial showed a remarkable response and progression-free survival benefit in patients with medullary carcinoma of thyroid with RET fusions who were given selpercatinib as first or secondline management. This data can be extrapolated to Hurthle cell carcinomas with RET fusion. The National comprehensive cancer network (NCCN) recommends using selpercatinib or pralsetinib for Hurthle cell carcinoma patients with RET fusions who need systemic therapy. If he has a progression of his cancer and needs to start on systemic therapy, the first step would be to see if his cancer is radioiodine avid. Generally, Hurthle cell carcinomas have low radioiodine avidity, and radioactive iodine treatments are not helpful. The patient had an RAI scan before surgery which did not show any iodine avidity, so RAI treatment is not useful here. Lenvatinib is a first-line therapy in most Hurthle cell carcinomas needing systemic therapy and can be used in this patient. However, with a RET fusion, it would be better to use selpercatinib first and reserve lenvatinib for future use if there is a progression on selpercatinib. Pembrolizumab is not appropriate in the first-line setting unless the patient has high microsatellite instability or tumor mutation burden. This information is not provided here.
Go to the next page if you knew the correct answer, or click the link image(s) below to further research the concepts in this question (if desired).
Research Concepts: Hurthle Cell Thyroid Carcinoma
We update eBooks quarterly and Apps daily based on user feedback. Please tap flag to report any questions that need improvement.
Question 732:
A 45-year-old male underwent lung transplantation due to severe chronic obstructive pulmonary disease (COPD). This was done at the advice of his clinicians, who deemed that he could no longer have a "normal" life if he did not opt for lung transplant surgery. A few months later, the patient developed generalized weakness and complained of losing a significant amount of weight. The complete blood count revealed anemia and thrombocytopenia. Ultrasound abdomen showed hepatosplenomegaly. On examination, enlarged lymph nodes were found. Which of the following pathogens is most commonly associated with developing this post-lungtransplant disorder?
Choices: 1. 2. 3. 4.
Cytomegalovirus Epstein-Barr virus Adenovirus Hepatitis C virus
Answer: 2 - Epstein-Barr virus Explanations: The Epstein-Barr virus is associated with post-lungtransplant lymphomas. Post-lung-transplant lymphoproliferative disorder (PTLD) is one of the most frequent post-transplant malignancies. The risk of PTLD is directly proportional to the intensity and duration of the immunosuppression used for the transplant. Cytomegalovirus is associated with post-lung transplantation syndrome but is not associated with post-lung transplant lymphoproliferative disorder. Go to the next page if you knew the correct answer, or click the link image(s) below to further research the concepts in this question (if desired).
Research Concepts: Lymphoproliferative Disorders
We update eBooks quarterly and Apps daily based on user feedback. Please tap flag to report any questions that need improvement.
Question 733:
A 48-year-old woman with a history of breast cancer undergoes a screening colonoscopy. She is found to have multiple hamartomatous polyps throughout the colon. Her past medical history is also significant for fibrocystic mammary disease, thyroid nodules, and lipomas in the back and abdomen. She reports that her father died of colon cancer. Considering hereditary colorectal cancer in the differential diagnosis, which of the following would be suspected?
Choices: 1. 2. 3. 4.
Lynch syndrome Turcot syndrome type I Cowden syndrome Hamartoma tumor syndrome
Answer: 3 - Cowden syndrome Explanations: Cowden syndrome (CS) is characterized by multiple hamartomas that can occur in any organ, macrocephaly, mucocutaneous lesions, and an increased risk of breast, thyroid, ovarian, and colon cancer. CS is usually caused by mutations in the PTEN gene, which contributes to apoptosis and the cell cycle. The International Cowden Syndrome Consortium has established major diagnostic criteria, including Lhermitte-Duclos disease, macrocephaly, thyroid carcinoma, and breast cancer. Minor criteria include genitourinary tumors or malformations, lipomas, fibromas, intellectual disability, fibrocystic breast disease, gastrointestinal hamartomas, and other thyroid lesions. CS is diagnosed with two major criteria or one major criterion plus three minor criteria. Complementing this with genetic tests helps differentiate it from other syndromes. More than 85% of patients with CS can have gastrointestinal hamartomatous polyps and associated breast cancer at any time in their life. The risk of colon cancer is slightly elevated, and the risk of endometrial cancer ranges from 20 to 30%. Go to the next page if you knew the correct answer, or click the link image(s) below to further research the concepts in this question (if desired).
Research Concepts:
A Review Of Hereditary Colorectal Cancers
We update eBooks quarterly and Apps daily based on user feedback. Please tap flag to report any questions that need improvement.
Question 734:
A 65-year-old male suffering from unresectable malignant mesothelioma was given a novel treatment with the application of arrays over the chest following which he developed an erythematous and pruritic rash over the chest. What will be the next step in the management?
Choices: 1. 2. 3. 4.
Stop the said treatment IV antibiotics Topical steroids Systemic steroids
Answer: 3 - Topical steroids Explanations: The most common complication of TTF is mild to moderate dermatitis at the site of application of transducers. In patients with malignant mesothelioma, transducer arrays are applied over the chest using special arrays and may result in local dermatitis in about half of the patients. The first line of management is the application of topical steroids and there is no need to stop the treatment. Topical and oral antibiotics may need to be added in case of superadded infection. Go to the next page if you knew the correct answer, or click the link image(s) below to further research the concepts in this question (if desired).
Research Concepts: Tumor Treatment Fields In Oncology
We update eBooks quarterly and Apps daily based on user feedback. Please tap flag to report any questions that need improvement.
Question 735:
A 65-year-old man presents to the clinic with complaints of hoarseness of voice. He also complains of difficulty in swallowing. A smoking history of 40 pack-years is present, and weight loss is also prominent. On presentation, he is afebrile; he has a pulse rate of 70 bpm, a blood pressure of 130/90 mmHg, and a respiratory rate of 16 breaths/min. The chest examination reveals reduced chest movements with a dull percussion note, on the right side. On auscultation, the right lower lung zone is found to have bronchial breath sounds. He has a Hb level of 8 g/dl, a mean corpuscular volume (MCV) of 70 fl, a TLC of 6000/mm3, and a platelet count of 170,000/mm3. Further radiographic investigations are performed adequately. An endoscopic biopsy of the esophagus reveals a poorly differentiated squamous cell carcinoma. Which of the following treatment regimens would be the most appropriate for the patient?
Choices: 1. 2. 3. 4.
Cisplatin and etoposide Cisplatin and fluorouracil Cisplatin, fluorouracil, and docetaxel Cisplatin and gemcitabine
Answer: 2 - Cisplatin and fluorouracil Explanations: The vast majority of the patients with esophageal cancer have incurable disease at presentation, and therapy in such cases is based mainly on palliation. One of the components of palliative treatment is combination chemotherapy with cisplatin and fluorouracil. Progressive dysphagia i-e first for solids, then for liquids, is the most common presenting symptom of esophageal cancer. Odynophagia and weight loss can also be present. The physical examination findings are suggestive of consolidation, as pneumonia can occur with tumor extension into the tracheobronchial tree. Hoarseness of the voice is due to the involvement of the recurrent laryngeal nerve and is indicative of the fact that the tumor can not be surgically resected. Combination chemotherapy with cisplatin and etoposide is useful in non-small cell lung cancers. Chemotherapy with cisplatin, fluorouracil, and docetaxel can be used for esophageal cancers. Still, since the patients are already in a compromised state, therapy with three chemotherapeutic agents is less preferred because of the excessive toxicity. Therapy with cisplatin and gemcitabine is used for the treatment of mesotheliomas. Go to the next page if you knew the correct answer, or click the link image(s) below to further research the concepts in this question (if desired).
Research Concepts: Esophageal Cancer
We update eBooks quarterly and Apps daily based on user feedback. Please tap flag to report any questions that need improvement.
Question 736:
In a patient with a mediastinal nonseminomatous germ cell tumor with elevated levels of human chorionic gonadotropin (HCG) and a-fetoprotein (AFP), multimodality chemotherapy reduces both HCG and AFP levels to normal. The CT scan reveals a residual mass in the chest. On exploration of the patient, what is the most likely histology of the remnant?
Choices: 1. 2. 3. 4.
Non-seminomatous germ cell Teratoma Seminoma Thymoma
Answer: 2 - Teratoma Explanations: The remnant may contain teratoma or scar tissue. Rarely, teratoma may rupture into adjacent structures and lead to lung lesions or pleuritis Calcification and pachypleuritis on chest imaging especially in patients without fever should be highly suspected of mediastinal teratoma. Pleural biopsy sometimes fails to assist in making a definitive diagnosis. Go to the next page if you knew the correct answer, or click the link image(s) below to further research the concepts in this question (if desired).
Research Concepts: Cystic Teratoma
We update eBooks quarterly and Apps daily based on user feedback. Please tap flag to report any questions that need improvement.
Question 737:
Which of these basal cell carcinoma subtypes is clinically the most aggressive?
Choices: 1. 2. 3. 4.
Morpheaform Nodular Superficial Pigmented
Answer: 1 - Morpheaform Explanations: The morpheaform (sclerosing) subtype is the most clinically aggressive form of basal cell carcinoma (BCC). The BCCs that are the highest risk are incompletely excised, recurrent, have indistinct borders, those in the mask area, those at embryonic fusion planes, functionally or cosmetically important areas, showing rapid growth, sites of prior radiation, aggressive histology, and in patients that are immunosuppressed. Histologic subtypes that are considered more aggressive include morpheaform (sclerosing), infiltrating, and micronodular variants of basal cell carcinomas. Morpheaform basal cell carcinoma commonly presents as a scar in an area without a history of trauma. Go to the next page if you knew the correct answer, or click the link image(s) below to further research the concepts in this question (if desired).
Research Concepts: Basal Cell Cancer
We update eBooks quarterly and Apps daily based on user feedback. Please tap flag to report any questions that need improvement.
Question 738:
A 65-year-old man with a medical history of adenocarcinoma of the stomach (currently on chemotherapy) presents to the clinic for follow-up. He was started on epoetin alfa treatment for chemotherapyassociated anemia for the last six months. He is taking epoetin alfa 100 units per kilogram each week. His hemoglobin has remained stable at 10.5 g/dL for the previous three months. Today, a complete blood count revealed hemoglobin of 11.5 g/dL. What would be the best next step in the management of this patient?
Choices: 1. 2. 3. 4.
Increase the dose of epoetin alfa. Start iron infusion each month. Stop epoetin alfa treatment and monitor hemoglobin. Continue epoetin alfa treatment.
Answer: 3 - Stop epoetin alfa treatment and monitor hemoglobin.
Explanations: Epoetin alfa is often used in the treatment of anemia associated with chemotherapy. Treatment should be stopped when hemoglobin reaches the goal of 11 g/dL. Epoetin alfa treatment increases thromboembolism risk. The iron profile should be checked before instituting epoetin alfa treatment. Before starting epoetin alfa treatment, intravenous iron should be supplemented if ferritin is less than 100 ng/ml and iron saturation is less than 20%. Go to the next page if you knew the correct answer, or click the link image(s) below to further research the concepts in this question (if desired).
Research Concepts: Epoetin Alfa
We update eBooks quarterly and Apps daily based on user feedback. Please tap flag to report any questions that need improvement.
Question 739:
A middle-aged premenopausal woman presents with a mass in the right upper quadrant of the right breast. Her results are negative for estrogen receptor (ER), progesterone receptor (PR), and human epidermal growth factor receptor 2 (HER2). What is the standard treatment option for this patient?
Choices: 1. 2. 3. 4.
Estrogen Chemotherapy Tamoxifen Radiation
Answer: 2 - Chemotherapy Explanations: Triple-negative breast cancer is best treated with chemotherapy. Triple-negative breast cancer cells do not have estrogen receptors (ER) or progesterone receptors (PR) and also do not make the protein human epidermal growth factor receptor 2 (HER2). Therefore, therapies targeting these receptors are not helpful. Chemotherapy is the standard treatment option for such patients. Triple-negative cancers grow and are aggressive. Because the cancer cells do not have hormone receptors, hormone therapy is not helpful. Drugs that target HER2 are not helpful either. Chemotherapy can be useful. Hormone receptor-positive breast cancer cells have either ER or PR. These cancers can be treated with hormone therapy drugs that lower estrogen levels or block estrogen receptors. This includes cancers that are ER-negative but PR-positive. Hormone receptor-positive cancers grow more slowly than those that are receptor negative. Women with hormone receptor-positive cancers have a better short-term prognosis, but these cancers often reoccur many years after treatment. Go to the next page if you knew the correct answer, or click the link image(s) below to further research the concepts in this question (if desired).
Research Concepts: Breast Cancer
We update eBooks quarterly and Apps daily based on user feedback. Please tap flag to report any questions that need improvement.
Question 740:
A 68-year-old male with a history of prostate cancer presents to the emergency department with acute onset low back pain, which is constant and worst at nighttime. The appropriate imaging test to evaluate for metastatic involvement of the spine is ordered. Which of the following is the most rational pathogenesis underlying the metastatic spread of prostate cancer postulated in the patient?
Choices: 1. 2. 3. 4.
Arterial hematogenous dissemination Lymphatic spread Direct invasion of the spine Spread through Batson's venous plexus
Answer: 4 - Spread through Batson's venous plexus Explanations: Along with lung cancer and multiple myeloma, prostate cancer is one of the top three malignancies associated with metastatic spinal cord compression. Metastatic spread of prostate cancer is hematogenous in nature. Unlike other sources of hematogenous spread, pelvic tumors, including prostatic malignancies, are postulated to metastasize via Batson venous plexus, a network of valveless veins that shunts venous blood to the spinal epidural venous plexus during a Valsalva maneuver such as sneezing, coughing, or bearing down. Patients with metastatic spinal cord compression most commonly present with isolated back pain. Any patient with active malignancy and new-onset back pain should be evaluated for the possibility of metastatic spinal disease. Direct extension of paraspinal malignancy into the spinal canal is most commonly seen with lymphoma, which accesses the spinal canal via neural foramina. Go to the next page if you knew the correct answer, or click the link image(s) below to further research the concepts in this question (if desired).
Research Concepts: Spinal Cord Compression
We update eBooks quarterly and Apps daily based on user feedback. Please tap flag to report any questions that need improvement.
Question 741:
A 72-year-old woman presents to the clinic with complaints of an ulcer on the superior aspect of her right labia majora. She has no other complaints at this time, except for some swelling in her right groin region. She reports no medical history but notes that she was diagnosed in her mid-40s with “inflammation” near her vagina but never had it followed up. On physical exam, a 1 cm patch with ulceration and increased surrounding vascularity is noted in the area of concern, and there is widespread erythematous vulvar skin that is paper-thin. A biopsy of the lesion is taken and squamous cell carcinoma is diagnosed. What is the most likely molecular origin of this patient’s disease?
Choices: 1. 2. 3. 4.
Gain of function in an oncoprotein Gain of function in a tumor suppressor protein Loss of function in an oncoprotein Loss of function in a tumor suppressor protein
Answer: 4 - Loss of function in a tumor suppressor protein
Explanations: In both the usual type and differentiated type of vulvar dysplasia development, there is a loss of function in a tumor suppressor protein. In usual type, the HPV virus has oncoproteins E6 and E7 which are known to inhibit the p53 and Rb tumor suppressor proteins. In the differentiated type, p16 is inhibited. The inhibition of these tumor suppressor proteins leads to decreased expression and thus cell proliferation, immortality, and eventually cancer. Go to the next page if you knew the correct answer, or click the link image(s) below to further research the concepts in this question (if desired).
Research Concepts: Vulva Cancer
We update eBooks quarterly and Apps daily based on user feedback. Please tap flag to report any questions that need improvement.
Question 742:
A 55-year-old piping insulation worker from Bangladesh presents to the clinic with five months of difficulty breathing and chest pain. He says that these symptoms are associated with fatigue and lethargy. He also has an unrelenting cough, which started as a dry cough, progressed to productive cough, and now even has streaks of blood in it. He works as a plumber and has been insulating leaky pipes for the past 35 years and has never had any such complaint before this. He has a history of smoking one pack of cigarettes for the past 35 years. However, he has never had alcohol before. He also has a collection of parrots and pigeons at his home, which he cares for in his free time. On examination, breath sounds are decreased on the left lower lobe of the lung. Percussion note is dull, and vocal fremitus is reduced. X-ray imaging of the left lung shows blunting of the costophrenic angle along with small areas of calcifications on the margins of the lung. A diagnosis of occupational disease is made. Which of the following is the most likely culprit for his presentation?
Choices: 1. 2. 3. 4.
Arsenic Lead Asbestos Bird feces
Answer: 3 - Asbestos Explanations: Pleural mesothelioma is the most likely cause of this patient's presentation. Pleural mesotheliomas are caused by exposure to asbestos. It has a very rapid downhill course and is usually seen in middle-aged men. While exposure to asbestosis is the leading risk factor, other cases have been linked to radiation, talc, or mica exposure. In this patient, occupational exposure to asbestos can be expected due to his line of work. Bird feces, lead, and arsenic isn't linked to developing malignant mesothelioma. Go to the next page if you knew the correct answer, or click the link image(s) below to further research the concepts in this question (if desired).
Research Concepts: Malignant Mesothelioma
We update eBooks quarterly and Apps daily based on user feedback. Please tap flag to report any questions that need improvement.
Question 743:
A 16-year-old female patient presents to the oral maxillofacial clinic with an 8-month history of right cheek swelling. She states the swelling has suddenly been increasing in size over the last several weeks and now causing significant pain, swelling, and deformity of the right side of her face. Upon examining the patient, the swelling appears to be firm, not cystic, non-tender, and not mobile. Intra-orally, a 2-centimeter ulcerated mass was seen involving the cheek mucosa. There is no palpable neck mass. Computed tomography imaging was performed to determine the extent of the mass, the patient was sent for a biopsy which showed peripheral palisading nuclei, polarization, and stellate reticulum-like cells. Which of the following is the appropriate management for the patient's condition?
Choices: 1. 2. 3. 4.
Radiotherapy Chemotherapy Surgery Concurrent chemoradiation
Answer: 3 - Surgery Explanations: Malignant ameloblastoma comprises both metastasizing ameloblastoma and ameloblastic carcinoma. In terms of natural history and histopathologic features, metastasizing ameloblastoma is closer to benign ameloblastoma while ameloblastic carcinoma has a very poor prognosis in late disease. Malignant ameloblastoma is generally found in a slightly greater ratio in males than females. Benign ameloblastoma has a slow-growing natural history and rarely transforms into malignant ameloblastoma. However, the sudden nature of the increasing size of the swelling along with the intraoral swelling should begin investigation for a malignant cause. Go to the next page if you knew the correct answer, or click the link image(s) below to further research the concepts in this question (if desired).
Research Concepts: Ameloblastoma
We update eBooks quarterly and Apps daily based on user feedback. Please tap flag to report any questions that need improvement.
Question 744:
A 13-year old male presents with complaints of foot pain for the past 3 months that is worse at night and relieved with an over-the-counter NSAID. He denies any history of trauma. On examination, there is diffuse tenderness over the dorsum of the foot. Which of the following is the most accurate diagnostic test for the early diagnosis of this condition?
Choices: 1. 2. 3. 4.
Plain radiography Thin-slice CT imaging Triple-phase bone scan MRI
Answer: 2 - Thin-slice CT imaging Explanations: The clinical scenario suggests an osteoid osteoma in the foot. Plain radiography is the initial test of choice but not considered the most accurate for diagnostic purposes. Thin-slice CT imaging is the imaging modality of choice. It identifies the anatomic location of the nidus. Triple-phase bone scans are an option if plain radiographs are inconclusive. However, they are not the most accurate diagnostic test. Bone marrow edema around the nidus may lead to misdiagnosis by MRI. MRI is more accurate than CT imaging in diagnosing cancellous lesions. Go to the next page if you knew the correct answer, or click the link image(s) below to further research the concepts in this question (if desired).
Research Concepts: Osteoid Osteoma
We update eBooks quarterly and Apps daily based on user feedback. Please tap flag to report any questions that need improvement.
Question 745:
A 56-year-old male presented to the clinic with complaints of bright red bleed per rectum and alternating constipation and diarrhea. He was diagnosed with rectal adenocarcinoma on rectal sigmoidoscopy after a transrectal ultrasound revealed T2N1 disease at 5 cm from the anal verge. The patient is diagnosed with a RAS/BRAF wild-type metastatic rectal adenocarcinoma. What is the most appropriate first-line treatment option in this patient?
Choices: 1. Sequential capecitabine, irinotecan, and oxaliplatin (CAPOX) 2. Folinic acid, fluorouracil, and oxaliplatin (FOLFOX) plus bevacizumab 3. FOLFOX plus cetuximab 4. Irinotecan with fluorouracil and folinic acid (FOLFIRI) plus bevacizumab
Answer: 3 - FOLFOX plus cetuximab Explanations: For fit candidates with metastatic colorectal cancer (CRC), the standard first-line treatment approach is to start with a FOLFOX, CAPOX, or FOLFIRI regimen plus anti-EGFR (Cetuximab or Panitumumab) when the tumor is RAS/BRAF wild-type and left-sided. For fit candidates with metastatic CRC, the standard first-line treatment approach is to start with a FOLFOX, CAPOX, or FOLFIRI regimen plus anti-VGFR (Bevacizumab) when the tumor is RAS/BRAF mutated or right-sided. At progression of the disease or unacceptable toxicity of first-line therapy, switch to the opposite second-line regimen with either continuation of prior anti-VGFR or new biological agent regardless of the tumor location. Sequential single agents remains a valid option for mCRC patients compared to combination treatments demonstrated in the United Kingdom Medical Research Council's FOCUS (Fluorouracil, Oxaliplatin, CPT-11: Use and Sequencing) and the Dutch Colorectal Group CAIRO (CApecitabine, IRinotecan, Oxaliplatin) trials, although biological agents were not used in both trials and most of all patients did not receive all three drugs. Go to the next page if you knew the correct answer, or click the link image(s) below to further research the concepts in this question (if desired).
Research Concepts: Rectal Cancer
We update eBooks quarterly and Apps daily based on user feedback. Please tap flag to report any questions that need improvement.
Question 746:
A 38-year-old woman presents to the clinic for follow-up due to an abnormal Pap smear result. Her last Pap smear was reported to contain atypical glandular epithelial cells of undetermined significance. She reports a history of heavy periods for the past year. On examination, external genitalia and vulva are normal. Per speculum examination reveals no vaginal or cervical lesions. On bimanual palpation, the uterus is slightly bulky, adnexa, and the ovaries are normal. A pregnancy test is negative. Which of the following is most likely to provide information for definitive management of this patient?
Choices: 1. 2. 3. 4.
Pelvic ultrasonography Colposcopy, hysteroscopy, and biopsies Pelvic computed tomography (CT) Abdominal and vaginal ultrasonography
Answer: 2 - Colposcopy, hysteroscopy, and biopsies Explanations: Patients with endometrial hyperplasia most frequently present with abnormal uterine bleeding. Some patients may be detected due to abnormal glandular or endometrial cells in the Pap smear. A detailed history and physical examination are required to determine and rule out other sources and causes of the presenting complaints or bleeding. The final diagnosis of endometrial hyperplasia that differentiates simple hyperplasia from atypical hyperplasia and endometrial carcinoma can only be done on histopathology. This is done by hysteroscopy and direct biopsy sampling of the endometrium. Imaging the endometrium for the diagnosis of endometrial hyperplasia can be done on days 5 to 10 of the patient’s cycle. Endometrial hyperplasia can be reliably excluded if the endometrium measures less than 6 mm in thickness in pre-menopausal women and less than 5 mm thick in post-menopausal women. However, it is not possible to reliably differentiate endometrial hyperplasia from atypical hyperplasia by ultrasonography or CT. Endometrial carcinoma also can not be differentiated from endometrial hyperplasia by ultrasonography or CT unless frank myometrium invasion is present. Management of all three conditions is different. Go to the next page if you knew the correct answer, or click the link image(s) below to further research the concepts in this question (if desired).
Research Concepts: Premalignant Lesions Of The Endometrium
We update eBooks quarterly and Apps daily based on user feedback. Please tap flag to report any questions that need improvement.
Question 747:
A 65-year-old man presents with chest heaviness and dyspnea. Vital signs, physical examination, ECG, and troponin levels are normal. A CT angiogram of the chest done to rule out pulmonary embolism shows an anterior mediastinal mass measuring 4.5 cm in the greatest dimension. Thoracoscopic tumor resection is performed, and microscopic evaluation of the specimen confirms the diagnosis of an intermediate-grade neuroendocrine tumor of the thymus. The specimen is also notable for positive margins. What is the most appropriate management for this patient?
Choices: 1. 2. 3. 4.
Open surgical resection Chemotherapy Radiation with or without chemotherapy Wait and watch
Answer: 3 - Radiation with or without chemotherapy Explanations: The majority of carcinoids in the mediastinum occur in the thymus. Radiation is the treatment of choice for resected tumors with a positive or close margin. As this patient has positive margins, radiation is indicated. Postoperative radiotherapy is also administered in patients with an invasive lesion. Induction chemotherapy is usually administered for advanced neoplasms or locally advanced disease to achieve tumor shrinkage. Chemotherapy has no role in this patient as the tumor was localized and resected. Go to the next page if you knew the correct answer, or click the link image(s) below to further research the concepts in this question (if desired).
Research Concepts: Mediastinal Carcinoid Tumors
We update eBooks quarterly and Apps daily based on user feedback. Please tap flag to report any questions that need improvement.
Question 748:
A 42-year-old man presents to his primary care provider for a health maintenance exam. On examination, the clinician discovers a hard, firm testicular mass on the left testis. On further questioning, the patient informs that he first noticed it 5 months ago, but did not think he needed to have it examined since he was not in pain. Transillumination of the scrotum was negative for translucency. Blood work shows normal alpha-fetoprotein (AFP) and beta-human chorionic gonadotropin (B-HCG). Chest radiograph and CT scans were negative for lymph nodes or distant metastases. Testicular ultrasound shows a solid hypoechoic intratesticular mass. Which of the following is the most appropriate next step?
Choices: 1. Radical inguinal orchiectomy with possible retroperitoneal lymph node dissection to evaluate for metastases 2. Testicular biopsy 3. Chemotherapy followed by follow up imaging and biopsy 4. Serial ultrasonography
Answer: 1 - Radical inguinal orchiectomy with possible retroperitoneal lymph node dissection to evaluate for metastases
Explanations: Testicular cancer is an uncommon malignancy of the males. It is divided into germ-cell tumors & sex cordstromal tumors. Germ cell tumors are the commonest type, accounting for about 95% of testicular malignancies. Testicular cancer usually presents as a painless testicular mass. On physical examination, a testicular lump or swelling with no translucency may be found. All testicular cancers should undergo radical orchiectomy and if there is a possibility of metastases, retroperitoneal lymph node dissection (RPLND) should be done. Testicular biopsy should always be avoided in order to avoid the spread of cancer cells in the suspicious lesion through the needle tract and in the surrounding testicular tissues. This seems to be a case of sex cord stroma testicular cancer and needs to be further evaluated with the help of imaging and oncology referral. Go to the next page if you knew the correct answer, or click the link image(s) below to further research the concepts in this question (if desired).
Research Concepts: Sex Cord Stromal Testicular Tumor
We update eBooks quarterly and Apps daily based on user feedback. Please tap flag to report any questions that need improvement.
Question 749:
A 65-year-old truck driver presents with an annular plaque with an elevated border on his left arm. The lesion has been present for several years and periodically itches. Dermoscopy shows central brown pigmentation with blue-gray dots surrounded by a single hypopigmented band with a white track at the periphery. What is the histologic hallmark of the most likely diagnosis?
Choices: 1. 2. 3. 4.
Cornoid lamella Thickened granular layer Peripheral palisading of basal cells Aberrant deep keratinization
Answer: 1 - Cornoid lamella Explanations: The histologic hallmark of porokeratosis is a cornoid lamella, which is a column of tightly fitted parakeratotic cells. Clinically, porokeratosis will present in older patients in sun-exposed areas such as keratotic papules or annular plaques that expand centrifugally with an elevated keratotic border. This cornoid lamella overlies the epidermis with an absent granular layer. Dyskeratotic squamous cells may be seen in the upper spinous layer. Dermoscopy will show central brown pigmentation with blue-gray dots surrounded by a single hypopigmented band with a white track at the periphery. While the cornoid lamella is the histologic hallmark of porokeratosis, it is not pathognomonic, and the clinical picture must be considered. Go to the next page if you knew the correct answer, or click the link image(s) below to further research the concepts in this question (if desired).
Research Concepts: Porokeratosis
We update eBooks quarterly and Apps daily based on user feedback. Please tap flag to report any questions that need improvement.
Question 750:
A 65-year-old man presents to the clinic with symptoms of bone pain, fatigue, anorexia, loss of weight of 20 pounds. On examination, he has widespread lymphadenopathy and hepatosplenomegaly. Detailed evaluation reveals a diagnosis of T cell Leukemia. Further testing shows calcium 12 mg/dL (reference range: 9 - 10.5 mg/dL), phosphorous 2.7 mg/dL (reference range: 3 - 4.5 mg/dL), parathyroid hormone 10 pg/mL (reference range: 10 - 65pg/mL). Which of the following tests is most likely to reveal the cause of the patient's hypercalcemia?
Choices: 1. 2. 3. 4.
Alkaline phosphatase level 1, 25 hydroxycholecalciferol Parathrome related protein level Cortisol level
Answer: 3 - Parathrome related protein level Explanations: Adult T cell Leukemia (ATL) is frequently associated with hypercalcemia, which can be severe. The hypercalcemia seen in ATL is paraneoplastic in origin. The mechanisms thought to be causes of hypercalcemia in patients with adult T cell leukemia include production of parathyroid hormone-related protein (PTH-RP), tumor necrosis factor-beta or interleukin-1, and increased expression of RANKL (receptor activator of nuclear factor-kB ligand). The frequency of hypercalcemia differs among the variants of ATL. In the acute variant, 70% of patients with ATL will have hypercalcemia at some point in their disease course, while 40% will have lytic bone lesions. Hypercalcemia can be severe, with calcium levels as high as 21 mg/dL. Signs and symptoms related to hypercalcemia such as renal dysfunction or neuropsychiatric disturbances may be prominent among the presenting clinical features. There are mainly two mechanisms of hypercalcemia associated with malignancy: osteolytic and humoral. When lytic bone metastases are present, hypercalcemia is due to the increased mobilization of calcium from the bone. In such cases, the serum alkaline phosphatase level is elevated. Humoral hypercalcemia is less common and occurs when the tumor itself produces a parathyroid-related protein (PTHrP) that binds to and activates the parathyroid receptor, raising serum calcium levels, and parathormone level is suppressed.
Go to the next page if you knew the correct answer, or click the link image(s) below to further research the concepts in this question (if desired).
Research Concepts: Adult T Cell Leukemia
We update eBooks quarterly and Apps daily based on user feedback. Please tap flag to report any questions that need improvement.
Question 751:
A 62-year-old male has a one-week history of nausea, vomiting, and paresthesias of the right hand and foot. On the day of admission, he had a generalized seizure followed by a headache that increases when he bends forward. There is right facial droop and loss of double simultaneous tactile stimulation. MRI shows a left posterior frontal, intraparenchymal, non-enhancing mass with a local mass-effect on the lateral ventricle consistent with a brain tumor. Transformation of which brain cell type is most likely the etiology of the tumor in this patient?
Choices: 1. 2. 3. 4.
Lymphocyte Neuron Star-shaped glial cells Oligodendrocyte
Answer: 3 - Star-shaped glial cells Explanations: The most common primary brain tumors in adults are in the family of astrocytomas. Within the astrocytomas. Glioblastoma Multiforme, WHO grade 4 is most common with a peak incidence in ages 60-70 with a male predominance. Astrocytoma originates in astrocytes, which are a kind of glial cells in the cerebrum which are star-shaped. It is the most common glioma, usually affecting the brain and sometimes the spinal cord. Amongst brain tumors, glial tumors comprise 60% of the tumors. Symptoms can be divided into two categories: general & focal. General symptoms include headache (usually early morning), nausea, vomiting, cognitive difficulties, personality changes, and gait disorders. Localizing symptoms include seizures, aphasia, or visual field defects. A visual field defect is commonly unnoticed by the patient, may be revealed after it leads to injury such as automobile accidents. The only test necessary to diagnose a brain tumor is neuroimaging. MRI is the best imaging for the same. Gadolinium contrast-enhanced MR imaging should be used whenever possible. If there is any contraindication for MRI, such as joint implants or pacemakers in situ, computed tomography or CT may be done. Lower-grade gliomas aren't contrasting enhancing, so fluidattenuated inversion recovery (FLAIR) sequences of MRI are done. If a tumor is found, a biopsy must be performed.
Go to the next page if you knew the correct answer, or click the link image(s) below to further research the concepts in this question (if desired).
Research Concepts: Astrocytoma
We update eBooks quarterly and Apps daily based on user feedback. Please tap flag to report any questions that need improvement.
Question 752:
A 17-year-old male patient presents to the neurology clinic with complaints of hoarseness of voice. He has a history of acute leukemia and is being treated with the modified Berlin-Frankfurt-Munster (BFM) protocol. Neurological examination shows sensory loss in a glove and stocking distribution and areflexia. An indirect laryngoscopy is suggestive of restricted mobility of the right vocal cord. Which of the following drug is most likely responsible for this condition?
Choices: 1. 2. 3. 4.
L -Asparaginase Cytosine arabinoside Duanomycin Vincristine
Answer: 4 - Vincristine Explanations: Berlin-Frankfurt-Munster (BFM) protocol is used in patients with childhood and adult acute lymphoblastic leukemia. The protocol comprises of the following chemotherapeutic drugs: vincristine, methotrexate, daunomycin, cytosine arabinoside, L-asparaginase, and prednisolone. Vincristine associated vocal cord palsy usually presents with hoarseness of voice. The lesion is known to localize to the recurrent laryngeal nerve or the vagal nerve nucleus. While both pyridoxine and pyridostigmine are associated with early recovery, the role of glutamic acid in the treatment of this adverse neurological outcome remains a matter of future research. It is known to be reversible, with recovery expected within a duration of 6 to 9 months. Pre-existing hepatic dysfunction, higher than recommended dosing of the drug, hypersensitivity to the drug, hereditary sensory neuropathy, and concomitant usage of other medications such as allopurinol, erythromycin, mitomycin C, phenytoin and itraconazole have been reported to predispose to vocal cord palsy. Vincristine has also been associated with the occurrence of jaw pain, which has an underlying neuropathic component, and it is known to be resistant to usual analgesic treatment. It is said to represent the cranial nerve (trigeminal) involvement. Go to the next page if you knew the correct answer, or click the link image(s) below to further research the concepts in
this question (if desired).
Research Concepts: Vinca Alkaloid Toxicity
We update eBooks quarterly and Apps daily based on user feedback. Please tap flag to report any questions that need improvement.
Question 753:
A 70-year-old male smoker who occasionally drinks alcohol presents with an isolated varicocele. He has a waist-hip ratio of 2.0 and past history of hypertension, recently immigrated from the Czech Republic. Contrast-enhanced computed tomography (CECT) abdomen is suggestive of a retroperitoneal mass. A biopsy is suggestive of nests of clear cells along with eosinophilic granular cells rich in glycogen in the background of fine branching vessels. Given the most common differential, which is the most common determinant of the histological grading of these tumors?
Choices: 1. 2. 3. 4.
Ratio of plasma cells to normal cells Proportion of lymphocytes Cellular atypia Nuclear grade
Answer: 4 - Nuclear grade Explanations: Though a renal cell carcinoma, which is also known as the internist’s tumor has been associated with the typical triad of hypertension, haematuria, and flank mass, this typical presentation is seen rarely. Renal tumors are usually associated with a myriad of manifestations, which have an underlying paraneoplastic etiology. This needs to be differentiated from other manifestations of advanced disease, which may include constitutional symptoms, hematological findings such as raised erythrocyte sedimentation rate and anemia, and cancer-associated syndrome clusters such as cancer anorexia-cachexia syndrome. Nuclear morphology has been used as the basis for grading these tumors. Fuhrmann’s system, which is based upon an assessment of nuclear size, nuclear irregularity, and degree of prominence of the nucleoli, is among the most commonly used. This system classifies the tumor into four grades. Grade 4 tumors characterized by the presence of large, pleomorphic, multilobed giant cells with heavy chromatin lumps and the presence of extremely irregular outlines, are considered the most aggressive. The criticisms leveled at this system include the difficulty associated with the simultaneous examination of three morphological features and an absence of guidelines to characterize the tumor when at least one of these features may not be prominent. Poor interobserver variability and difficulty in standardizing the outcome based upon the three pre-defined parameters have also been mentioned as another
critique. The World Health Organization/International Society of urological pathology has introduced a grading system that is based on nucleolar morphology. Nucleolar size assumes importance in the wake of the observation that it correlates with ribosomal biogenesis. Nucleolar grade has also been shown to correlate with patient outcomes. An isolated varicocele or a deep vein thrombosis might be the only manifestation of a retroperitoneal mass. Other conditions that may present with retroperitoneal masses include lymphomas, sarcomas, lipomas, and Castleman's disease. Go to the next page if you knew the correct answer, or click the link image(s) below to further research the concepts in this question (if desired).
Research Concepts: Renal Clear Cell Cancer
We update eBooks quarterly and Apps daily based on user feedback. Please tap flag to report any questions that need improvement.
Question 754:
A 29-year-old presents with a history of several days of pain in the right flank. Yesterday he noticed pink-colored urine. He denies any history of stones and has no other significant past medical history. Family history is significant for Ewing sarcoma in a first cousin. He has mild tenderness in the right flank, but the physical exam is unremarkable. Urinalysis shows 3+ blood on the dipstick, and microscopy shows 3456 red blood cells. Other labs are unremarkable. CT scan of the abdomen shows a 7 x 6 cm solid mass in the retroperitoneum infiltrating the right renal pelvis. The rest of the imaging does not show any other abnormal masses. A biopsy of the mass is done, and pathology shows a very poorly differentiated tumor which is negative for CD45, S-100, CK20, and HMB-45. The immunohistochemistry stains are positive for CK7 and CK19, and the cytoplasm of the tumor cells stains positive for CD99. Additional immunohistochemistry staining shows that the tumor is negative for TTF-1 and SOX-10. Which of the following molecular studies will help clinch the diagnosis?
Choices: 1. Balanced and reciprocal t(X;18)(p11.2;q11.2) translocation 2. t(12;15)(p13;q25) translocation which generates ETV6NTRK3 fusion gene transcripts 3. t(11;22)(q24;q12) generating EWSR1-FL1 4. t(12;22)(q13;q12) generating EWSR1-ATF1
Answer: 1 - Balanced and reciprocal t(X;18) (p11.2;q11.2) translocation
Explanations: This patient has synovial sarcoma. The positive stains for CK7, CK19, and the cytoplasm staining with CD99 suggest synovial sarcoma. In addition, the balanced and reciprocal t(X;18)(p11.2;q11.2) translocation is pathognomonic of synovial sarcoma. The negative results for TTF-1 rule out lung cancer. Negative stain for SOX-10 rules out malignant peripheral nerve sheath tumors. Negative stain for S-100 and HMB45 rules out melanoma. Negative stain for CK20 rules out the GI origin of the tumor. Synovial sarcoma can originate anywhere in the body. The word "synovial" is a misnomer. t(12;15)(p13;q25) translocation, which generates ETV6NTRK3 fusion gene transcripts, is characteristic of Infantile Fibrosarcoma - this is generally seen in very young babies. t(11;22)(q24;q12) generating EWSR1-FL1 is characteristic of Ewing Sarcoma. The positive cytoplasmic stain of CD99 is not seen in Ewing Sarcoma t(12;22)(q13;q12) generating EWSR1-ATF1 is characteristic of clear cell sarcoma. Clear cell sarcoma is usually positive for S-100. Go to the next page if you knew the correct answer, or click the link image(s) below to further research the concepts in this question (if desired).
Research Concepts: Synovial Cell Sarcoma
We update eBooks quarterly and Apps daily based on user feedback. Please tap flag to report any questions that need improvement.
Question 755:
A 50-years-old man presented with a four-month history of progressive right-sided nasal obstruction, headache, and occasional epistaxis. On rhinoscopy, there was an irregular fleshy granular mass arising from the roof of the nasal cavity. He undergoes a computed tomogram scan, which shows that the 2x4 cm sized tumor is limited to the nasal cavity alone. He undergoes an intranasal biopsy with local anesthesia. Histopathological examination of the biopsy specimen with immunohistochemistry shows diffuse staining for neuronspecific enolase, synaptophysin, chromogranin A, CD56, and beta-tubulin; and negative staining for CD45RB, CD99, p63, and FLl1. Physical examination of the patient did not disclose any cervical lymphadenopathy, and there were no distant metastases. What will be the optimal treatment for this patient?
Choices: 1. 2. 3. 4.
Surgical resection and follow-up Neoadjuvant chemotherapy followed by surgery Surgery and adjuvant chemotherapy Surgery and adjuvant chemotherapy and radiotherapy
Answer: 1 - Surgical resection and follow-up Explanations: The three options used for the treatment of esthesioneuroblastoma are surgery, external beam radiation, and chemotherapy. Often, a combination of these modalities is used. For small tumors with no sign of regional or local metastasis, surgical resection is advocated. More extensive surgery, typically an open or endoscopic-assisted craniofacial resection is recommended along with postoperative radiation therapy, as the disease becomes more locally advanced. Chemoradiation is an approach for patients with unresectable local disease, which could subsequently improve the resectability for locally advanced esthesioneuroblastoma. Go to the next page if you knew the correct answer, or click the link image(s) below to further research the concepts in this question (if desired).
Research Concepts: Esthesioneuroblastoma
We update eBooks quarterly and Apps daily based on user feedback. Please tap flag to report any questions that need improvement.
Question 756:
A 68-year-old man presents to the clinic with a painless swelling on the left side of the face. The swelling is firm and fixed to the underlying structures and the overlying skin on palpation. There are no palpable lymph nodes in the neck. The patient reports a history of smoking for 30 years and, recently, a three weeks history of inability to close the left eye even while asleep. Findings on fine-needle aspirations show the presence of epidermoid cells with very few mucous cells and minimal cystic changes, suggestive of mucoepidermoid carcinoma. Which of the following is the most appropriate surgical approach for this patient?
Choices: 1. 2. 3. 4.
Superficial parotidectomy Radical parotidectomy Extracapsular dissection Radiochemotherapy
Answer: 2 - Radical parotidectomy Explanations: Radical parotidectomy is a surgical procedure that includes the removal of the entire parotid gland, along with resection of the facial nerve. Radical parotidectomy is indicated in facial nerve involvement by the tumor. Facial nerve involvement by the tumor manifests with limited ability to wrinkle the forehead, eye closure, whistling, and blowing. Superficial parotidectomy and extracapsular dissection are carried out for benign parotid tumors. Radiochemotherapy is recommended in cases of cervical lymph nodes involvement by the tumor. Go to the next page if you knew the correct answer, or click the link image(s) below to further research the concepts in this question (if desired).
Research Concepts: Parotidectomy
We update eBooks quarterly and Apps daily based on user feedback. Please tap flag to report any questions that need improvement.
Question 757:
A 64-year-old man with a 40 pack-year history of smoking presents to the clinic with recent unprovoked weight loss. His chest x-ray shows a calcified mass in the right lower lobe. The lung mass is biopsied. Genetic analysis of the mass shows different levels of histone proteins that have undergone methylation posttranscriptionally compared to normal lung tissue. If the biopsy is confirmed to be carcinogenic, which of the following best describes the most likely methylation levels of the histone proteins and subsequent effects on the tissue's tumor suppressor genes?
Choices: 1. Histone hypomethylation causing tumor suppressor gene overexpression 2. Histone hypomethylation causing tumor suppressor gene underexpression 3. Histone hypermethylation causing tumor suppressor gene overexpression 4. Histone hypermethylation causing tumor suppressor gene underexpression
Answer: 4 - Histone hypermethylation causing tumor suppressor gene underexpression
Explanations: Histone methylation generally increases the interactions between the histone and the DNA wrapped around it. By increasing the methylation levels, the histones interact more with the DNA resulting in less accessibility by transcriptional enzymes. If the histone associated with a tumor suppressor gene is suppressed because of increased interactions with its histone, the gene is expressed less, thus predisposing to potential tumor growth. Histone hypermethylation is the best answer because this is most associated with cancers as well as resultant tumor suppressor gene underexpression. Go to the next page if you knew the correct answer, or click the link image(s) below to further research the concepts in this question (if desired).
Research Concepts: Genetics, DNA Packaging
We update eBooks quarterly and Apps daily based on user feedback. Please tap flag to report any questions that need improvement.
Question 758:
A 66-year-old man presents to the emergency department complaining of persistent epigastric pain with associated nausea and early satiety. On computed tomography (CT) of the abdomen and pelvis with contrast, a 3 cm solid, contrast-enhancing mass with smooth margins is seen protruding off the first part of the duodenum. No other abnormal lesions are seen throughout the rest of the imaging. Which of the following treatment options is most appropriate for this patient?
Choices: 1. Endoscopic ultrasound 2. Surgical resection of the mass 3. Neo-adjuvant imatinib therapy followed by resection of the mass 4. Follow-up CT scan in 3 months to assess for progression
Answer: 2 - Surgical resection of the mass Explanations: Contrast-enhanced CT using oral and intravenous contrast is the preferred imaging modality for noninvasive assessment of gastrointestinal stromal tumors (GIST). On CT, a GIST will appear as a solid, contrast-enhancing mass with smooth margins. For patients in whom CT is highly suggestive of GIST, and preoperative therapy is not required, the tumor can be resected and sent for pathologic diagnosis without a biopsy being obtained prior. Surgical resection is the cornerstone of treatment for easily resectable GISTs greater than 2 cm. Go to the next page if you knew the correct answer, or click the link image(s) below to further research the concepts in this question (if desired).
Research Concepts: Gastrointestinal Stromal Cancer
We update eBooks quarterly and Apps daily based on user feedback. Please tap flag to report any questions that need improvement.
Question 759:
A 71-year-old white man presents to the hematologist with new lesions in his right neck, left elbow, and right groin region. He was diagnosed six months back with angioimmunoblastic lymphoma (AITL) when he had presented then with an isolated left groin lesion. A biopsy later had revealed a CD30 negative AITL, PET was negative for any disseminated disease then, and he was treated with six cycles of CHOP (Cyclophosphamide, Hydroxydaunorubicin, Vincristine, Prednisone). He went into partial remission but declined bone marrow transplant despite being transplant eligible due to "personal reasons." He was scheduled for a PET scan during a clinic visit today, which reveals disseminated disease with metastatic lesions in the liver, spleen, and several lymph nodes conglomerates. What would be the best next step in the management of this patient?
Choices: 1. Brentuximab Vedotin plus CHP (Cyclophosphamide, Hydroxydaunorubicin, Prednisone) 2. Repeat CHOP (Cyclophosphamide, Hydroxydaunorubicin, Vincristine, Prednisone) 3. Rebiopsy 4. Flow cytometry
Answer: 3 - Rebiopsy Explanations: It is important to repeat biopsy at relapse as patients with AITL have been known to harbor EBV positive B cells which can predispose them to develop one of the most common non-Hodgkin lymphoma - Diffuse large B cell lymphoma (DLBCL). A diagnosis of DLBCL will alter management significantly as they are responsive to Rituximab, which is a monoclonal antibody against CD20 positive B cells. It is always better to reconfirm the diagnosis. Brentuximab Vedotin is a monoclonal antibody conjugate against CD30 positive cells. This patient will likely not respond. Moreover, it is pertinent to confirm the diagnosis first. Repeat the same regimen without confirming the diagnosis is inappropriate. Flow cytometry will only provide information on clonality, hence the incorrect choice. Go to the next page if you knew the correct answer, or click the link image(s) below to further research the concepts in this question (if desired).
Research Concepts: Peripheral T-Cell Lymphoma
We update eBooks quarterly and Apps daily based on user feedback. Please tap flag to report any questions that
need improvement.
Question 760:
A 65-year-old male presents to the office for evaluation of a lesion on his left cheek. The patient states the lesion first appeared ten months ago and has slowly increased in size. On examination, the lesion is present on the left cheek, 5 mm in diameter, light brown to dark brown in color, and it has a fried-egg appearance. A biopsy is performed that shows changes in melanocytes and lamellar and concentric fibroplasia. What are the most likely changes in melanocyte structure?
Choices: 1. 2. 3. 4.
Hyperplasia and cytological atypia of melanocytes Hypoplasia and cytological atypia of melanocytes Hyperplasia of melanocytes Hypoplasia of melanocytes
Answer: 1 - Hyperplasia and cytological atypia of melanocytes
Explanations: A dysplastic nevus, or Clark nevus, is an acquired mole that demonstrates a unique clinical and histopathologic appearance compared to a common mole. The histopathologic features seen in dysplastic nevi include melanocytic intraepidermal lentiginous hyperplasia, cytologic atypia of melanocytes, lamellar fibroplasia (stromal response), and architectural disorder. The "shoulder sign" in dysplastic nevi refers to the histopathologic finding of an epidermal component that extends beyond or "shoulders" the dermal component on both sides of the compound nevus. The cytological atypic may include enlarged, dark nuclei with large nucleoli and is often graded from mild to severe. Go to the next page if you knew the correct answer, or click the link image(s) below to further research the concepts in this question (if desired).
Research Concepts: Dysplastic Nevi
We update eBooks quarterly and Apps daily based on user feedback. Please tap flag to report any questions that
need improvement.
Question 761:
A 65-year-old woman presented with an indurated right breast mass of 6-months duration. Physical exam showed a large mass with induration palpated on the left breast and a large lymph node in the right axilla. Initial mammography and ultrasound showed a left breast mass measuring 5.5 cm with speculated margins. An irregular mass was seen in the right axilla measuring 3.2 cm. Biopsy of the left breast mass showed grade II invasive lobular carcinoma, and biopsy of the right axillary mass showed metastatic spread. A bilateral breast MRI was performed to evaluate the right breast. Significant background enhancement was seen in the right breast, but no suspicious enhancing mass was identified. The case was reviewed in multidisciplinary tumor board, and a blind biopsy of the significant background enhancement in the right breast was requested based on consensus opinion. MRI-guided biopsy of the right breast showed rare foci of lymphatic tumor emboli in a background of normal fibroglandular breast tissue. What is the most likely diagnosis?
Choices: 1. The lymphangitic spread of invasive lobular carcinoma of the right breast to the contralateral breast 2. The lymphangitic spread of invasive lobular carcinoma of the left breast to the contralateral breast 3. Lobular carcinoma in situ of the right breast 4. Invasive carcinoma of no special type of the right breast
Answer: 2 - The lymphangitic spread of invasive lobular carcinoma of the left breast to the contralateral breast
Explanations: Lymphangitic carcinomatosis, also called carcinomatous lymphangitis, is a pattern of metastasis characterized by the spread of tumor emboli to lymphatic vessels of distant sites. Invasive lobular carcinoma spreads first to the ipsilateral lymph nodes and then metastasizes most commonly to the bone, liver, and lung through hematogenous pathways. Rarely, breast cancer can metastasize to the lungs via the lymphangitic spread, causing pulmonary lymphangitic carcinomatosis and metastasizes to the contralateral breast via the lymphangitic spread. In this case, the lymphangitic spread to the contralateral breast was discovered incidentally on biopsy and was not associated with any specific symptoms. If the lymphangitic spread of cancer is discovered, further evaluation for other sites of distant metastasis is essential. Go to the next page if you knew the correct answer, or click the link image(s) below to further research the concepts in this question (if desired).
Research Concepts: Lobular Breast Carcinoma
We update eBooks quarterly and Apps daily based on user feedback. Please tap flag to report any questions that need improvement.
Question 762:
A 60-year-old man is evaluated during a routine examination. He has hepatitis B virus-related cirrhosis with decompensation, including previous hepatic encephalopathy and variceal bleeding. He currently takes spironolactone, furosemide, lactulose, and rifaximin. Current vital signs show a temperature of 36.5 C (97.7 F), heart rate 60/minute, blood pressure 110/65 mmHg, and BMI 20 kg/m2. He has clubbing of the fingers and toes, and palmar erythema is noted. The remainder of the examination is normal. Ultrasound shows 2 new liver lesions in the left hepatic lobe. Multiphasic contrast CT demonstrates 2 lesions (2 cm and 2.8 cm in size) with hyperenhancement in the arterial phase and rapid washout during the portal venous phase. Macrovascular invasion is noted on the CT scan as well as splenomegaly. The patient has a European cooperative oncologic group performance status (ECOG-PS) score of 1. What is the most appropriate next step?
Choices: 1. 2. 3. 4.
Sorafenib Radiofrequency ablation (RFA) Surgical resection Liver transplantation
Answer: 1 - Sorafenib Explanations: Hepatocellular carcinoma (HCC) is a malignant tumor of the hepatocytes. Cytological features depend on the differentiation of hepatocytes, from well-differentiated to poorly differentiated HCC. This patient has a Barcelona clinic liver cancer (BCLC) advanced stage (C). Sorafenib is the first-line treatment for the patient with BCLC advanced stage (C) with preserved liver function, European cooperative oncologic group performance status (ECOG-PS) score of 1-2, and macrovascular invasion or extrahepatic spread. Sorafenib is a multikinase inhibitor. Sorafenib hepatocellular carcinoma assessment randomized protocol (SHARP) trial, demonstrated a median survival of 10.7 months patients receiving sorafenib as compared to 7.9 months in the placebo group. In addition, sorafenib was shown to be effective in the Asia-Pacific region with patients with advanced HCC. Patients with BCLC classification of very early (0) and early-stage (A) who do not meet surgical resection criteria are appropriate for ablation. Patients with BCLC classification of very early (0) and early-stage (A) are ideal candidates for surgical resection. Go to the next page if you knew the correct answer, or click the link image(s) below to further research the concepts in this question (if desired).
Research Concepts: Hepatocellular Carcinoma
We update eBooks quarterly and Apps daily based on user feedback. Please tap flag to report any questions that need improvement.
Question 763:
A 65-year-old female with a history of hypertension complained of recurrent emergency department visits because of episodes of dizziness, blurred vision, palpitations, and loss of consciousness. Her random blood glucose levels were in the range of 15 to 40 mg/dl on every visit, which improved by the administration of dextrose. She denied any medications other than amlodipine 10 mg for hypertension. She has no history of diabetes. Fasting insulin levels were 25 microU/mL and Cpeptides 10 ng/mL. MRI of the abdomen showed 4 cm pancreatic mass, and the Ki-67 index was 10%. What is the best treatment modality for this patient?
Choices: 1. 2. 3. 4.
Everolimus Whipple surgery Octreotide Peptide receptor radiotherapy (PRRT)
Answer: 2 - Whipple surgery Explanations: Pancreaticoduodenectomy or Whipple surgery and distal pancreatectomy are procedures typically used for pNETs greater than 2 cm, symptomatic disease, and low to intermediate grade. Surgical resection is the potentially curative treatment for localized and locally advanced NETs. On the other hand, surgical removal of pNETs less than 2cm in size, non-functioning, and low grade remains controversial. When surgical removal of the tumor is not accessible due to tumor size, a debulking surgery is recommended. Platinum-based chemotherapy is recommended following surgery in surgically excised, poorly differentiated tumors. Everolimus and octreotide are systemic therapies for more advanced NETs. Go to the next page if you knew the correct answer, or click the link image(s) below to further research the concepts in this question (if desired).
Research Concepts: APUDoma
We update eBooks quarterly and Apps daily based on user feedback. Please tap flag to report any questions that need improvement.
Question 764:
A 31-year-old female with a history of hoarseness presents with painless neck swelling. She has a family history of hypothyroidism in her mother and a sister. Physical examination shows a palpable swelling in the midline of the neck that is not tender. After ultrasound follow-up and fine-needle aspiration, the patient undergoes a total thyroidectomy. A diagnostic radioactive iodine wholebody scan four weeks after surgery shows a metastatic lymph node in the right neck. A therapeutic pill is prescribed. What is the most likely etiology?
Choices: 1. 2. 3. 4.
Poorly differentiated follicular thyroid cancer Well-differentiated papillary thyroid cancer Medullary thyroid cancer Anaplastic thyroid cancer
Answer: 2 - Well-differentiated papillary thyroid cancer Explanations: Similar to the normal thyroid follicular cells, welldifferentiated (WD) thyroid cancer cells will have the sodium iodide symporter (NIS), which aids absorb radioiodine into the cells. Hence WD thyroid cancer responds well to radioiodine therapy. As cancer cells become de-differentiated, they lose the function of NIS and their ability to concentrate radioiodine. Papillary thyroid cancer is the most common type of thyroid cancer. Differentiated thyroid cancer includes papillary thyroid cancer, follicular thyroid cancer, and Hurthle cell thyroid cancer. Papillary thyroid cancer shows predominantly lymphatic spread. Follicular thyroid cancer demonstrates predominant hematogenous spread. Medullary thyroid cancer arises from the parafollicular C cells of the thyroid, and anaplastic thyroid cancer is an aggressive cancer. These do not respond to RAI treatment. Go to the next page if you knew the correct answer, or click the link image(s) below to further research the concepts in this question (if desired).
Research Concepts: Radioactive Iodine For Thyroid Malignancies
We update eBooks quarterly and Apps daily based on user feedback. Please tap flag to report any questions that need improvement.
Question 765:
A 65-year-old man with a history of hypertension, diabetes mellitus, and colon cancer that was surgically removed ten years ago, presents to the clinic for shortness of breath and a recent 15-pound (7 kg) weight loss. The shortness of breath is at rest and worse on exertion. He has a 20 pack history of cigarette smoking but he stopped three years ago. He does not have any cough, chest pain, or hemoptysis. Medications are lisinopril 20 mg daily and metformin 500 mg twice a day. Vital signs include a temperature of 98 F orally, blood pressure of 135/85 mmHg, a pulse of 84/min, respirations of 16/min, and oxygen saturation of 92% on room air. Physical exam reveals no sub-clavicular or axillary lymph nodes; there are no cardiac murmurs or gallops, lung fields are clear and there is no egophony. Labs show a white blood cell count of 9000/microL and a hematocrit of 48%. Chest x-ray demonstrated clear lung fields except for a left upper lobe 9 mm ground-glass nodule and a small left pleural effusion. Which of the following is the next best diagnostic study to evaluate the pulmonary nodule and pleural effusion?
Choices: 1. 2. 3. 4.
Left-sided thoracentesis Left upper lobectomy A full-body PET scan Perform a CT scan at six months
Answer: 3 - A full-body PET scan Explanations: This patient's history of cancer, new weight loss, and location of the pulmonary nodule in the upper lobe is highly concerning for malignancy. PET scan would be able to determine if there are any other lesions/nodules in the body; this is useful given the patient's history of cancer and concern for metastasis. A contrast-enhanced CT scan at three months or obtaining a biopsy of the nodule would be other acceptable forms of management in this scenario. Malignant appearing nodules need to be dealt with quickly as delay in treatment may lead to an increase in morbidity and mortality. Go to the next page if you knew the correct answer, or click the link image(s) below to further research the concepts in this question (if desired).
Research Concepts: Solitary Pulmonary Nodule
We update eBooks quarterly and Apps daily based on user feedback. Please tap flag to report any questions that need improvement.
Question 766:
A 30-year-old man presents to the clinic for follow up. He recently underwent an IR guided biopsy of a large superior mediastinal mass that shows a mediastinal seminoma. The patient has no past medical history and does not take any medications. A physical exam is within normal limits. What is the most appropriate management strategy for this patient?
Choices: 1. 2. 3. 4.
Radiotherapy only Chemotherapy only Chemotherapy and radiotherapy Surgical resection only
Answer: 3 - Chemotherapy and radiotherapy Explanations: Multimodality treatment with chemotherapy followed by radiation is the first line treatment for large mediastinal seminomas. Chemotherapy agents include cisplatin, bleomycin, and etoposide for 4-6 cycles. Intermittent pulmonary function tests need to be checked while on bleomycin. Multimodal treatment with chemotherapy and radiotherapy exerts much better results when combined. After completion of the regimen, a CT chest should be obtained. If the tumor responds well to the treatment and shrinks, surgical resection can be performed. Large mediastinal seminomas need to be treated with combo therapy of chemo and radiation. Each regimen alone does not yield the best results as the combined treatment. Surgical resection is not the primary treatment for large seminomas. More than 50% of patients have unresectable tumors. Surgery is usually reserved for small mediastinal masses in patients with no symptoms. Surgery is often associated with a high recurrence rate and must be accompanied by a form of adjuvant therapy if it is considered. Go to the next page if you knew the correct answer, or click the link image(s) below to further research the concepts in this question (if desired).
Research Concepts: Mediastinal Seminoma
We update eBooks quarterly and Apps daily based on user feedback. Please tap flag to report any questions that need improvement.
Question 767:
A mother of a 4-day-old newborn presents to your clinic as a follow-up appointment due to an elevated Succinylacetone level found on routine examination. The mother says that her father died in his 50's due to complications from a condition in which his "protein levels were elevated." Without early intervention and treatment, what is the most likely cause of death in this patient?
Choices: 1. 2. 3. 4.
Cardiac arrest Hepatocellular carcinoma Bladder cancer Pancreatic cancer
Answer: 2 - Hepatocellular carcinoma Explanations: An elevated succinylacetone level is highly specific and sensitive for the condition hypertyrosinemia. Hypertyrosinemia is due to enzyme deficiencies in the tyrosine degradation pathway. Early diagnosis and treatment can reduce the chance of developing hepatocellular carcinoma. Other complications of this condition include intellectual disability, kidney disease, and multi-organ failure. Go to the next page if you knew the correct answer, or click the link image(s) below to further research the concepts in this question (if desired).
Research Concepts: Hypertyrosinemia
We update eBooks quarterly and Apps daily based on user feedback. Please tap flag to report any questions that need improvement.
Question 768:
A 66-year-old woman with metastatic breast cancer secondary to a knockout of PTENP1, the PTEN pseudogene, has volunteered to a genetic study of her genome. Their PTENP1 pseudogene is compared to conserved sequences within pseudogenes of other species. How does this study utilize the relatively conserved sequences of pseudogenes across species?
Choices: 1. Pseudogenes have conserved sequences that can reveal the evolution of certain genes for phylogenetics 2. Pseudogenes with conserved sequences can reveal the evolution of cancer 3. Pseudogenes have conserved sequences and can reveal the types of proteins produced from them in proteogenomic studies 4. Pseudogenes have conserved sequences and can reveal how antibiotic resistance genes developed from viral transmission in cDNA libraries
Answer: 1 - Pseudogenes have conserved sequences that can reveal the evolution of certain genes for phylogenetics
Explanations: Pseudogenes share many conserved sequences amongst varying species. The established, nondeleterious, changes to pseudogenes throughout evolution is a powerful tool for phylogenetics in studying the evolution of genes. Most of these conserved sequences may be interrupted by minor mutations such as silence mutations or missense mutations. However, these interruptions still allow for the coding of RNA transcripts that can go on to perform various regulatory roles. Pseudogenes do not ode for protein products. Rather, they exist as RNA transcripts if they are functional and perform regulatory roles as RNA. Pseudogenes may display palindromic-like sequences that allow them to act as retrotransposons and insert themselves into another locus or chromosome. However, because they do not code for physical protein, antibiotic resistance is not a method displayed or studied within pseudogenes. Go to the next page if you knew the correct answer, or click the link image(s) below to further research the concepts in this question (if desired).
Research Concepts: Biochemistry, Pseudogenes
We update eBooks quarterly and Apps daily based on user feedback. Please tap flag to report any questions that need improvement.
Question 769:
A 65-year-old male patient with a 25 pack-year history of smoking presents with a 9-month history of pain in the groin. Radiographic workup demonstrates a lesion within the right acetabulum. Staging studies do not reveal any other sites of disease. A bone biopsy is performed at the referring institution and reveals chondrocytes with variable size and shape that contain enlarged hyperchromatic nuclei with rings and arcs of cartilage. What is the most favorable prognostic factor for survival?
Choices: 1. Response to chemotherapy (percent tumor necrosis) on resection specimen 2. Tumor volume of 100 cubic centimeters 3. Pelvic location of the tumor 4. Older patient age
Answer: 2 - Tumor volume of 100 cubic centimeters Explanations: Good prognostic factors for local recurrence and overall survival of a patient with chondrosarcoma include small tumor size or tumor volume (less than 100 cubic centimeters). Patients with a large tumor volume, axial (pelvic) tumor location, old age, and a high grade imply a poorer prognosis. Low to intermediate grade chondrosarcomas are chemotherapy and radiation-resistant, and the treatment of a grade 2 chondrosarcoma is surgery alone. The use of radiation and chemotherapy for dedifferentiated chondrosarcoma is currently being investigated. Go to the next page if you knew the correct answer, or click the link image(s) below to further research the concepts in this question (if desired).
Research Concepts: Chondrosarcoma
We update eBooks quarterly and Apps daily based on user feedback. Please tap flag to report any questions that need improvement.
Question 770:
A 33-year-old woman presents to the clinic for newly developed erythema involving about 80% to 90% of her body surface area. The lesion is biopsied, revealing a superficial lichenoid lymphoid infiltrate with the malignant appearing lymphocytes revealing cerebriform nuclei, exhibiting 'epidermotropism,' and associated with 'Pautrier microabscesses.' Immunophenotyping is performed, this reveals CD3+, CD4+, CD45RO+, and CD8cells. The positron emission tomography scan does not reveal any visceral organ or lymph node involvement. Flow cytometry and bloodwork are negative for atypical circulating cells. The provider plans to start her on systemic therapy with an oral retinoid, which of the following preexisting conditions should be confirmed before initiating therapy?
Choices: 1. 2. 3. 4.
Asthma Retinitis pigmentosa Hyperparathyroidism Pregnancy
Answer: 4 - Pregnancy Explanations: This patient has stage IIIA mycosis fungoides and warrants treatment with a systemic agent. The hematologist is using an oral retinoid. Bexarotene is the oral retinoid used in this condition. It should never be used in pregnancy as it is teratogenic and is categorized as X for pregnancy. Hypertriglyceridemia and central hypothyroidism are the major adverse effects of systemic retinoid therapy. Retinitis pigmentosa is not recognized as a side effect of oral retinoids. It is, in fact, a group of inherited disorders. It does not affect patients with asthma, hyperthyroidism. Go to the next page if you knew the correct answer, or click the link image(s) below to further research the concepts in this question (if desired).
Research Concepts: Peripheral T-Cell Lymphoma
We update eBooks quarterly and Apps daily based on user feedback. Please tap flag to report any questions that need improvement.
Question 771:
A 41-year-old woman presents to the clinic 6 months after a diagnosis of ER+ breast cancer with worsening right upper quadrant pain. She had a bilateral mastectomy and is currently undergoing adjuvant chemotherapy with tamoxifen. Laboratory studies are normal; however, given concern for metastatic disease, a CT scan of her abdomen is obtained demonstrating hypoattenuating lesions within the liver. Subsequent biopsy reveals findings consistent with peliosis hepatis. Which of the following is most likely responsible for this patient's condition?
Choices: 1. 2. 3. 4.
Tamoxifen Breast cancer Hepatocellular carcinoma Ascending cholangitis
Answer: 1 - Tamoxifen Explanations: Tamoxifen works through binding to estrogen receptors. Typically used in the treatment of hormone-sensitive ER+ breast cancer, tamoxifen is known to cause hepatotoxicity. Though uncommon, case reports have described circumstances where patients receiving tamoxifen therapy develop peliosis hepatis. Tamoxifen hepatotoxicity often arises within the first 6 months of therapy. Go to the next page if you knew the correct answer, or click the link image(s) below to further research the concepts in this question (if desired).
Research Concepts: Peliosis Hepatis
We update eBooks quarterly and Apps daily based on user feedback. Please tap flag to report any questions that need improvement.
Question 772:
A 65-year-old male presents to a clinic with complaints of nocturia, urgency, and hesitancy for the past two months. He denies any history of hematuria, suprapubic pain, or dysuria. On a digital rectal examination, there is a small palpable nodule in the area corresponding to the prostate. Suprapubic ultrasound shows a prostate volume of 45 ml. Magnetic resonance imaging reveals a PIRADS 3 lesion. A biopsy of the lesion demonstrates predominantly poorly-formed, fused, or cribriform glands with a lesser component of well-formed glands. What is the Gleason grade group of the lesion?
Choices: 1. 2. 3. 4.
Grade Grade Grade Grade
group group group group
2 3 4 5
(Gleason (Gleason (Gleason (Gleason
score 3 + 4 = 7) score 4 + 3 = 7) score 8) scores 9 or 10)
Answer: 2 - Grade group 3 (Gleason score 4 + 3 = 7) Explanations: Grade group 3 (Gleason score 4 + 3 = 7) lesion shows predominantly poorly-formed, fused, or cribriform glands with a lesser component of well-formed glands. In clinical practice, group 1 is considered "low grade," Group 2 is "intermediate grade," and group 3 or higher is "high grade" disease. The newer classification system introduced by the world health organization (WHO) is based on clinical experience with the old Gleason scoring system. It suggests very little difference in clinical outcomes in lower Gleason score patients, but somewhat different ones in the higher grades. Grade group 2 (Gleason score 3 + 4 = 7) shows predominantly well-formed glands with a lesser component of poorly-formed, fused, or cribriform glands. Grade group 4 (Gleason score 8) indicates only poorlyformed/fused/cribriform glands, or predominantly wellformed glands with a lesser component lacking glands, or predominantly lacking glands with a lesser component of well-formed glands. Grade group 5 (Gleason scores 9 or 10) lesion lacks gland formation (or with necrosis) with or without poorly-formed, fused, or cribriform glands. Go to the next page if you knew the correct answer, or click the link image(s) below to further research the concepts in this question (if desired).
Research Concepts:
Prostate Cancer
We update eBooks quarterly and Apps daily based on user feedback. Please tap flag to report any questions that need improvement.
Question 773:
A 74-year-old patient presents 25 months after completing primary chemotherapy and radiation therapy for stage 3 oropharyngeal squamous cell carcinoma. She continues to smoke. She began having pain when chewing one month ago, which has become progressively worse, and she has noticed that her teeth "do not fit together normally" anymore. She denies hemoptysis, weight loss, trismus, or neck mass. On examination, there is a palpable pseudoarthrosis in the body of her left mandible, but no exposed bone. Her remaining teeth are in good repair. After a biopsy excludes recurrent cancer, which of the following would be a component of her next step in treatment?
Choices: 1. 2. 3. 4.
30 hyperbaric oxygen treatments 10 hyperbaric oxygen treatments Surgical removal of necrotic bone Empiric antibiotics
Answer: 1 - 30 hyperbaric oxygen treatments Explanations: This patient most likely has osteoradionecrosis. Patients see the most improvement of healing and symptom relief when 30 preoperative hyperbaric oxygen (HBO) treatments are delivered. Surgical intervention should take place after 30 HBO treatments have been done for improved healing of the bone and improvement of symptoms. All cases of osteoradionecrosis of the mandible require surgical intervention with mild bony debridement as a minimum requirement. To prevent osteoradionecrosis (ORN), HBO treatments are always indicated before the surgical intervention or dental extraction in a previously irradiated area of bone. ORN of the mandible is primarily an aseptic disease of the bone and only requires antibiotics if an apparent secondary bacterial infection is present. Go to the next page if you knew the correct answer, or click the link image(s) below to further research the concepts in this question (if desired).
Research Concepts: Mandible Osteoradionecrosis
We update eBooks quarterly and Apps daily based on user feedback. Please tap flag to report any questions that
need improvement.
Question 774:
A 55-year-old woman with a history of rheumatoid arthritis presents to the emergency department with fever, chills, and a productive cough of 2 days duration. The physical exam is notable for a temperature of 101 F and right lower lobe crackles. Chest x-ray shows right lower lobe consolidation. CBC shows WBC count of 5000/microL with 10% neutrophils and 70% lymphocytes on differential. Hemoglobin is 8 g/dL, and platelet count is 80000/microL. The peripheral blood smear shows increased large granular lymphocytes. Peripheral blood flow cytometry shows a monoclonal population of lymphocytes positive for CD3, CD8, CD16, CD57, and negative for CD56. Which of the following is an additional feature of the most likely diagnosis?
Choices: 1. T cell receptor (TCR) gene rearrangement 2. Spontaneous recovery of cell counts 3. Lack of spleen involvement 4. Most patients are diagnosed incidentally and are asymptomatic at presentation
Answer: 1 - T cell receptor (TCR) gene rearrangement Explanations: T cell large granular lymphocyte (LGL) leukemia is a disease characterized by clonal expansion of LGL. T-LGL leukemia patients commonly present with pancytopenia, among which symptoms related to neutropenia are most common. Only one-third of patients may be asymptomatic at presentation. Typical immunophenotype of T-cell LGL Leukemia is CD3+, CD8+, CD16+, CD57+, CD56 negative, and identification of TCR gene rearrangement is a key diagnostic feature. Spleen infiltration is a common feature. Go to the next page if you knew the correct answer, or click the link image(s) below to further research the concepts in this question (if desired).
Research Concepts: Lymphocytosis
We update eBooks quarterly and Apps daily based on user feedback. Please tap flag to report any questions that need improvement.
Question 775:
A 75-year-old man with a history of HIV presents to the clinic with a right midline palate mass. On physical exam, the mass is non-tender, and he has bilateral cervical lymphadenopathy. Fine needle biopsy showed lymphocytes mixed with plasma cells and histiocytes. Immunophenotypic showed CD20+ B-cells. What is the most appropriate management strategy for this patient?
Choices: 1. Wide surgical resection with neck dissection 2. Wide surgical resection with neck dissection and adjuvant radiation 3. Wide surgical resection with neck dissection and adjuvant chemoradiation 4. Chemoradiation
Answer: 4 - Chemoradiation Explanations: The risk factors for developing Non-Hodgkin lymphoma (NHL) is multifactorial, including immunosuppression (HIV, congenital immunodeficiency, organ transplant, chemotherapy, and radiation), viral infections (EBV, HTLV-1, herpes, hepatitis C), bacterial infections (H. pylori, Lyme disease), tobacco, animal fat, obesity, hair dyes, ultraviolet radiation pesticides and occupational toxin exposure, and genetic predisposition for B-cell survival and growth. NHL often presents with a painless, submucosal mass without ulceration and cervical lymphadenopathy. NHL histopathology shows irregular lymphocytes mixed with plasma cells, histiocytes, and lymphoid follicles. (Medeiros) On immunophenotyping, there is a predominant CD20+ B-cells with T-cells positive for CD3+. NHL is treated with a combination of radiation and chemotherapy. Radiation was reported to be given at a range of 24-45 Gy over 16-25 fractions. NHL is most commonly treated with a standard chemotherapy regiment involving cyclophosphamide, doxorubicin, vincristine, and prednisolone (CHOP) for 8 cycles over a duration of 3 weeks. Go to the next page if you knew the correct answer, or click the link image(s) below to further research the concepts in this question (if desired).
Research Concepts:
Malignant Tumors of the Palate
We update eBooks quarterly and Apps daily based on user feedback. Please tap flag to report any questions that need improvement.
Question 776:
A 34-year-old female patient presents to the office with her husband for a follow up of her acute promyelocytic leukemia treatment. Her last follow up 10 days ago showed that she is in remission and she is cautiously optimistic of a good outcome. She is still on tretinoin therapy and will be for the next 20 days. The patient appears upset and states that all she has always wanted a child of her own and is afraid that may never happen. She and her husband are interested in conceiving a child but are afraid that her leukemia may come back or that the child may experience adverse effects of the medication. Which of the following is the most appropriate recommendation?
Choices: 1. It is safe to conceive now since she is in remission 2. The patient may conceive once she has completed oral tretinoin therapy in 20 days 3. The patient may safely conceive after at least 50 more days 4. The patient should wait for 9 months before conceiving to give her time to recover from tretinoin therapy
Answer: 3 - The patient may safely conceive after at least 50 more days
Explanations: The patient has been in remission for 10 days while taking oral tretinoin therapy. It is recommended that she continue treatment with tretinoin for 30 days (total) after being diagnosed with remission. While taking oral tretinoin therapy, pregnancy is absolutely contraindicated. It is recommended that contraception continues for one month following remission therapy. While on oral tretinoin, patients are to be on two forms of contraception or remain completely abstinent due to severe teratogenic effects. Go to the next page if you knew the correct answer, or click the link image(s) below to further research the concepts in this question (if desired).
Research Concepts: Tretinoin
We update eBooks quarterly and Apps daily based on user feedback. Please tap flag to report any questions that need improvement.
Question 777:
A 5-year-old boy is being treated with intrathecal methotrexate as a part of his treatment for acute myeloid leukemia (AML). The patient seems to be doing fine. However, it is noted that the patient had vomited twice in the morning. A methotrexate overdose is suspected. What is the most appropriate route for the antidote for this patient?
Choices: 1. 2. 3. 4.
Intrathecal Intravenous Oral Subcutaneous
Answer: 2 - Intravenous Explanations: Methotrexate toxicity is treated with leucovorin, hydration, and urine alkalinization. In the setting of toxicity, intravenous oral or intramuscular leucovorin is indicated. The patient is nauseated and hence it would be prudent to avoid oral drug administration. Intrathecal administration of leucovorin is dangerous and may be fatal. Go to the next page if you knew the correct answer, or click the link image(s) below to further research the concepts in this question (if desired).
Research Concepts: Leucovorin
We update eBooks quarterly and Apps daily based on user feedback. Please tap flag to report any questions that need improvement.
Question 778:
A 68-year-old man presents to the clinic with one week of hematuria. He denies any dysuria, frequency, abdominal pain, or back pain. He has a history of prostate cancer treated with radiation therapy to the prostate for a total dose of 6500 cGy completed last week. What is the most appropriate time to initiate hyperbaric oxygen therapy for this patient?
Choices: 1. 2. 3. 4.
Immediately after the completion of radiation therapy 1 to 3 months after radiation therapy 3 to 6 months after radiation therapy More than 6 months after radiation therapy
Answer: 4 - More than 6 months after radiation therapy Explanations: Soft tissue radionecrosis refers to the delayed effects of radiation therapy, which result in tissue breakdown from the impaired blood supply radiation-damaged tissue. Radiation can damage capillary beds and arterioles, which lead to relative tissue hypoxia resulting in characteristic fibrosis, and these tissue changes can develop over time, remote from the time of the original radiation exposure. The soft tissue radionecrosis can develop 6 months to several years after the exposure. Prior to 6 months, the diagnosis is acute or subacute radiation injury. Go to the next page if you knew the correct answer, or click the link image(s) below to further research the concepts in this question (if desired).
Research Concepts: Hyperbaric Soft Tissue Radionecrosis
We update eBooks quarterly and Apps daily based on user feedback. Please tap flag to report any questions that need improvement.
Question 779:
A 65-year-old man undergoes exploratory laparoscopy for abdominal pain and is diagnosed with a generalized colorectal tumor at the peritoneum. He is being considered for elective treatment with cytoreductive surgery in combination with hyperthermic intraperitoneal chemotherapy (HIPEC). He has a past medical history of coronary artery disease, diabetes mellitus, and hyperlipidemia. Which of the following factors is most likely to delay this procedure for this patient?
Choices: 1. 2. 3. 4.
Age Coronary artery disease Diabetes mellitus Hyperlipidemia
Answer: 2 - Coronary artery disease Explanations: Hyperthermic intraperitoneal chemotherapy (HIPEC) represents a concept of multimodal therapy increasingly used in the management of peritoneal carcinoma. This involves administering cytotoxic agents into the peritoneal cavity at an elevated temperature (41 to 43 ° C) in order to promote their absorption by neoplastic nodules. HIPEC has become the treatment of choice for treating peritoneal metastases from colorectal cancers. In colorectal cancer, the peritoneum is the second site most common metastatic after liver. About 7% to 15% of colorectal cancer patients have synchronous peritoneal metastases, and 4%-19% will develop one afterward. This procedure is not devoid of serious cardiovascular and respiratory complications as described in the literature. According to recent studies (meta-analysis), cytoreductive surgery and HIPEC are associated with morbidity and mortality of approximately 33% and 2.8%, respectively. The presence of co-morbid factors such as severe cardiovascular disorders and serious lung disease may complicate the procedure and should contraindicate the procedure. It should be remembered that heavy and complex surgical procedures likely lead to increased toxicity of HIPEC through changes in pharmacokinetics, protein losses, hepatic and renal metabolic restrictions, and stress-related bone marrow suppression. During the procedure, the use of cytotoxic drugs like 5-fluorouracil and oxaliplatin can be deleterious for heart function because they are cardiotoxic drugs.
Diabetes and dyslipidemia are not contraindications for HIPEC. Go to the next page if you knew the correct answer, or click the link image(s) below to further research the concepts in this question (if desired).
Research Concepts: Hyperthermic Intraperitoneal Chemotherapy
We update eBooks quarterly and Apps daily based on user feedback. Please tap flag to report any questions that need improvement.
Question 780:
A 79-year-old woman presents to the clinic with a chief complaint of a 1 cm right breast lump, which she first noticed 2 weeks ago. On exam, it is firm, well-circumscribed, tender, and just under the breast surface. There is some mild overlying erythema without drainage. Topical warm soaks and oral antibiotic therapy is initiated as a treatment trial for a presumed sebaceous cyst. She returns 1 week later for follow up on the response to treatment. Upon repeat breast exam, the tenderness and erythema have resolved, but the lump is unchanged. Which of the following is the next best step in the management of this patient?
Choices: 1. Schedule mammogram and ultrasound 2. Reassure the patient that the lump will resolve over the next 3 months 3. Fine needle aspiration cytology 4. Repeat another 7-day course of antibiotics
Answer: 1 - Schedule mammogram and ultrasound Explanations: The initial physical exam presented an important differential diagnosis for the breast lump, which included both inflammatory and structural breast disease. A treatment trial based on your differential diagnosis of the breast lump with short term follow up was reasonable, but an inadequate response to treatment as evidenced by the follow-up exam changes the differential diagnosis towards a complex inflammatory and/or structural issue, and further evaluation is warranted. Imaging clarification is the next best step in preparation for radiology intervention biopsy vs. direct surgical referral depending on imaging results. Reassurance is premature at this point despite the improvement in pain and erythema and could potentially delay a diagnosis of breast cancer. An unresolved breast lump should not be allowed to remain unevaluated or undiagnosed. Even if the lesion is a cyst or abscess, repeating a course of antibiotics will only delay definitive diagnosis and/or therapy. Go to the next page if you knew the correct answer, or click the link image(s) below to further research the concepts in this question (if desired).
Research Concepts: Breast Examination Techniques
We update eBooks quarterly and Apps daily based on user feedback. Please tap flag to report any questions that need improvement.
Question 781:
An 18-year-old male presents with a firm and fluctuant swelling on the right side of the face. The patient says the swelling has progressively increased in size for the past six months. The swelling is painless, but he reports a recently decreased sensation on the right side of the face. An ultrasound shows a well-circumscribed hypoechoic lesion with a partial cystic appearance. Fineneedle aspiration cytology (FNAC) reveals an infiltrative growth pattern of atypical epidermoid and intermediate cells with cytoplasmic clearing and a small number of mucinous cells. What is the most likely diagnosis?
Choices: 1. 2. 3. 4.
Non-Hodgkin Lymphoma Primary squamous cell carcinoma Mucoepidermoid carcinoma Warthin tumor
Answer: 3 - Mucoepidermoid carcinoma Explanations: This patient in the clinical scenario most likely presents with mucoepidermoid carcinoma (MEC), the most common salivary gland malignancy in adults and children (about 89% of cases are found in the parotid). Mucoepidermoid carcinoma often presents as a painless and firm mass and facial paralysis. However, some patients describe occasional pain and first bite syndrome. On biopsy, mucoepidermoid carcinoma is described as a non-encapsulated, poorly circumscribed mass with cystic components that can often be mistaken for mucoceles. It can be composed of up to 6 different types of cells, including maternal, intermediate, epidermoid, clear, columnar, and mucous cells. MEC can occur as low, intermediate, or high-grade disease depending on the quantity of aggressive, epidermoid tissue. Go to the next page if you knew the correct answer, or click the link image(s) below to further research the concepts in this question (if desired).
Research Concepts: Malignant Salivary Gland Tumors
We update eBooks quarterly and Apps daily based on user feedback. Please tap flag to report any questions that need improvement.
Question 782:
A 28-year-old female, gravida 0, presents to a reproductive endocrinologist's office desiring fertility. She was recently diagnosed with polycystic ovarian syndrome (PCOS) based on chronic anovulation, signs of hirsutism (facial hair), and an ultrasound that was significant for multiple follicles (>10mm) in each ovary. At her initial visit, her body mass index (BMI) was noted to be 38 kg/m2, and her heart rate was 120 beats per minute. All of her other vital signs were within normal limits. Her past medical history is significant for diabetes mellitus type 2 which is controlled with metformin 500 mg twice daily. She denies any past surgical history. Family history is significant for obesity, hypertension, and dyslipidemia in her mother and father, and PCOS in her older sister. Upon questioning, she received all childhood vaccinations, including the HPV series as a teenager. What type of cancer is she at the most significant risk for developing in her lifetime?
Choices: 1. 2. 3. 4.
Ovarian Cervical Endometrial Anal
Answer: 3 - Endometrial Explanations: Unopposed estrogen is a risk factor for endometrial hyperplasia. Endometrial hyperplasia is a precursor to endometrial cancer. Unopposed estrogen can occur in granulosa cell tumors, hormone replacement therapy, and polycystic ovarian disease. Obesity increases endogenous estrogen because the fatty tissues convert androstenedione to estrogen at a high rate. Go to the next page if you knew the correct answer, or click the link image(s) below to further research the concepts in this question (if desired).
Research Concepts: Endometrial Cancer
We update eBooks quarterly and Apps daily based on user feedback. Please tap flag to report any questions that need improvement.
Question 783:
A 43-year-old male with a history of hypertension and diabetes mellitus presents to the emergency department with three months of worsening progressive shortness of breath. He has never smoked and reports no family history of lung cancer. Current medications are hydrochlorothiazide 25 mg daily and amlodipine 5 mg daily. His vital signs include a temperature of 98°F (36.7°C) orally, blood pressure of 140/88 mmHg, a heart rate of 95 beats per minute, a respiratory rate of 22 breaths per minute, and oxygen saturation of 88% on room air. Physical examination reveals an obese male with bilateral crackles at the bases of both lungs. The patient is placed on oxygen, and a computed tomography (CT) scan of his chest with contrast was performed, which demonstrated a 10 mm solid homogenous nodule in the left upper lobe. Bilateral interstitial edema and trace pleural effusions are also noted. Echocardiography reveals heart failure with reduced ejection fraction. What is the most appropriate next step in the management of this patient's pulmonary nodule?
Choices: 1. 2. 3. 4.
Surgical wedge resection Repeat CT scan in three months Repeat CT scan in twelve months Perform a percutaneous needle biopsy of the nodule
Answer: 2 - Repeat CT scan in three months Explanations: A pulmonary nodule is a well-defined lesion that is surrounded by pulmonary parenchyma and is less than 30 mm. Pulmonary nodules can be benign or malignant. Although this patient presents with shortness of breath, he has a clear alternate etiology of his hypoxia. The size of the nodule is a predictor of malignancy. Data from retrospective studies show that, when assessed on a CT scan, nodules less than 5 mm have a less than 1% chance of being malignant. Nodules that are 5 to 9 mm have a 2% to 6% chance of being malignant. Nodules that are 10 to 20 mm have an 18% chance of being malignant, and nodules that are greater than 20 mm have a greater than 50% chance of being malignant. The presence of fat or calcification on the border of the nodule is a reliable indicator of a benign pulmonary nodule. Patients with nodules larger than 8 mm should be further evaluated for cancer. A nodule that has a low probability of being malignant should be followed with a CT scan after three months. A nodule that has a high risk of being malignant should be evaluated with a positron emission tomography (PET) scan, with or without a biopsy. This patient is relatively young and has no risk factors or family history of lung cancer and thus has a low probability of lung cancer. He should be followed up with a CT scan in three months. Go to the next page if you knew the correct answer, or click the link image(s) below to further research the concepts in this question (if desired).
Research Concepts: Solitary Pulmonary Nodule
We update eBooks quarterly and Apps daily based on user feedback. Please tap flag to report any questions that need improvement.
Question 784:
A 44 year old male complains of sudden loss of hearing and a persistent headache. A few weeks prior he had been having episodes of dizziness and vertigo. A CT scan revealed a mass in the internal acoustic meatus. If this is a primary tumor, which is most likely?
Choices: 1. 2. 3. 4.
Melanoma Schwannoma Glioblastoma multiforme Meningioma
Answer: 2 - Schwannoma Explanations: Because of the high number of nerves, the internal acoustic meatus is often prone to nerve malignancies like schwannomas and neuromas. These malignancies are best visualized by CT or MRI. The individuals often present with ear pain, dizziness, vertigo, or loss of hearing. Surgical excision can provide a cure but hearing loss may be permanent. Stereotactic radiotherapy is another option. Go to the next page if you knew the correct answer, or click the link image(s) below to further research the concepts in this question (if desired).
Research Concepts: Schwannoma
We update eBooks quarterly and Apps daily based on user feedback. Please tap flag to report any questions that need improvement.
Question 785:
A 65-year-old female with a past medical history of hypertension and diabetes mellitus type 2 presents for the evaluation of weakness. She reports that over the last two months, she has noticed progressive weakness and fatigue. Due to these symptoms, she reports difficulty getting out of a chair and climbing the stairs. She also reports gait disturbances in the last two weeks. She denies any fever, shortness of breath, or chest pain. She has smoked 1/2 pack per day for the past 20 years. She does not drink alcohol and does not use illicit drugs. Physical examination is notable for ataxic gait and hyporeflexia. Bilateral proximal muscle weakness in the lower extremities is evident. The cardiopulmonary examination is unremarkable. Serologic testing for anti-voltage-gated calcium channel antibodies is positive. What is the next best step in the management of this patient?
Choices: 1. 2. 3. 4.
Start amifampridine therapy Obtain chest X-ray Order mammogram Start intravenous immune globulin (IVIG) therapy
Answer: 2 - Obtain chest X-ray Explanations: The most crucial step in managing Lambert-Eaton myasthenic syndrome (LEMS) is to evaluate for an underlying malignancy. Treatment of the underlying malignancy usually improves symptoms of LEMS, and specific therapy may not be required. Small cell lung cancer is the most common tumor associated with LEMS. In this patient, who presents with LEMS and has an underlying smoking history with advanced age, the likelihood of small cell lung cancer is very high. A chest X-ray followed by dedicated radiographic imaging of the chest is warranted. Approximately one-half of the LEMS cases are associated with small cell lung cancer. In patients with severe weakness that interferes with daily function, amifampridine therapy may be tried. In patients with refractory weakness, immunomodulatory therapy with IVIG, corticosteroids, and/or other immunomodulatory agents may be tried. The other malignancies associated with LEMS include lymphoproliferative disorders (Hodgkin lymphoma), atypical carcinoid tumor, thymic neuroendocrine carcinoma, thymic small cell carcinoma, and neuroblastoma. Go to the next page if you knew the correct answer, or click the link image(s) below to further research the concepts in this question (if desired).
Research Concepts:
Paraneoplastic Syndromes
We update eBooks quarterly and Apps daily based on user feedback. Please tap flag to report any questions that need improvement.
Question 786:
Which of the following is associated with the CD10 antigen?
Choices: 1. 2. 3. 4.
Acute myeloid leukemia Acute lymphocytic leukemia Chronic myeloid leukemia Hodgkin lymphoma
Answer: 2 - Acute lymphocytic leukemia Explanations: The CD10 antigen is also known as neprilysin or membrane metalloendopeptidase. The CD10 antigen is a zinc dependent enzyme that degrades many peptides and is often associated with acute lymphoblastic leukemias. In some types of cancers, the CD10 is over expressed. Mutation in the CD 10 antigen has also been linked to Alzheimer disease. Go to the next page if you knew the correct answer, or click the link image(s) below to further research the concepts in this question (if desired).
Research Concepts: Acute Lymphocytic Leukemia
We update eBooks quarterly and Apps daily based on user feedback. Please tap flag to report any questions that need improvement.
Question 787:
A 45-year-old woman presents for her routine breast cancer ultrasound screening. The examination reveals bilateral microcalcifications. On physical examination, there is no abnormal breast mass, nipple retraction, or skin changes. A biopsy is done on two lesions showing lobular carcinoma in situ. She is nulliparous, and her menarche was when she was 14 years old. What is the most appropriate next step in the management of this patient?
Choices: 1. 2. 3. 4.
Simple mastectomy Modified radical mastectomy with adjuvant tamoxifen Life long surveillance with prophylactic tamoxifen Lumpectomy followed by radiation therapy
Answer: 3 - Life long surveillance with prophylactic tamoxifen
Explanations: Lobular carcinoma in situ (LCIS) is considered a risk factor for invasive breast cancer; therefore, follow-up with prophylactic tamoxifen is the best course of action for this patient. Life-long surveillance is necessary as LCIS increases the risk of invasive ductal carcinoma eightfold. Tamoxifen has been proved to reduce the risk of breast cancer by 50% if used by patients with LCIS. Technically, LCIS is not a disease, so follow-up is needed. Mastectomy and radiation therapy are not indicated. Go to the next page if you knew the correct answer, or click the link image(s) below to further research the concepts in this question (if desired).
Research Concepts: Breast Cancer
We update eBooks quarterly and Apps daily based on user feedback. Please tap flag to report any questions that need improvement.
Question 788:
A 65-year-old man with metastatic prostate cancer to bone presents with bone pain. He has had no pain relief with aspirin. The clinician decides to choose a drug from the next level of the analgesic ladder. What is the most appropriate next drug on the WHO ladder for cancer pain relief in adults?
Choices: 1. 2. 3. 4.
Methadone Hydrocodone Fentanyl Morphine
Answer: 2 - Hydrocodone Explanations: According to the World Health Organization (WHO) ladder for cancer pain relief in adults, prompt oral medication administration is warranted. Nonopioids, such as aspirin and acetaminophen, should be given first. If necessary, mild opioids, such as codeine, hydrocodone, or tramadol, should be given next. If mild opioids are unable to bring adequate pain relief, then strong opioids, such as morphine, methadone, or fentanyl, should be given until the patient is free of pain. Adjunct medications for pain relief such as gabapentin, corticosteroids, or bisphosphonates should be considered at every level. The WHO also recommends adjuvant medications "to calm fears and anxiety" as needed. Go to the next page if you knew the correct answer, or click the link image(s) below to further research the concepts in this question (if desired).
Research Concepts: WHO Analgesic Ladder
We update eBooks quarterly and Apps daily based on user feedback. Please tap flag to report any questions that need improvement.
Question 789:
A 10-year-old boy is under treatment for acute lymphoblastic leukemia and is planned for intrathecal methotrexate to prevent CNS relapse. Where is the most appropriate point of access to the intrathecal space in this patient?
Choices: 1. 2. 3. 4.
Lumbar puncture Right frontal Right parietal Ventriculoperitoneal shunt
Answer: 2 - Right frontal Explanations: Ommaya reservoir, a dome-shaped port with an intraventricular catheter attached to it, is commonly used for delivery for antineoplastic drugs into the intrathecal space. Ommaya reservoir is routinely placed subcutaneously over the right Kocher's point, which is approximately 2-3 cms away from the midline and 1 cm anterior to the coronal suture. Unless indicated for a tumor cyst or in the presence of anatomical distortion, the intraventricular catheter is placed through a burr hole over right Kocher's point. The bulge over the scalp is palpated, and under aseptic precautions, a butterfly cannula is inserted at an oblique angle to penetrate the reservoir. Ommaya reservoir helps prevent repeated lumbar punctures. Unless indicated for a tumor cyst or by the presence of anatomical difficulty, the intraventricular catheter is placed through a burr hole over right Kocher's point. Go to the next page if you knew the correct answer, or click the link image(s) below to further research the concepts in this question (if desired).
Research Concepts: Ommaya Reservoir
We update eBooks quarterly and Apps daily based on user feedback. Please tap flag to report any questions that need improvement.
Question 790:
A 65-year-old woman presents with dysphagia to solids for the past two months. She also reports weight loss and a decrease in appetite. Her past medical history is significant for heavy smoking and chronic alcohol use. On examination, there is mild conjunctival pallor, and she looks emaciated. Upper endoscopy demonstrates an ulcerated gastric mass, and endoscopic ultrasound shows infiltration into adventitia. Biopsy analysis shows large cells with a high nuclear-cytoplasmic ratio, disrupted nuclei, and glandular cells. CT scans of the chest, abdomen, and pelvis are negative for metastasis, including nodal involvement. The patient's overall performance status is very good. What is the standard of care for this patient?
Choices: 1. Endoscopic resection 2. Neoadjuvant chemoradiation followed by surgical resection 3. Perioperative chemotherapy followed by surgical resection 4. Palliative systemic therapy
Answer: 3 - Perioperative chemotherapy followed by surgical resection
Explanations: This vignette depicts T3N0M0 gastric cancer, based on endoscopic ultrasound and CT staging. Endoscopic resection is limited to superficial, limited mucosa disease (less than or equal to T1a). Surgical resection with lymphadenectomy is recommended for lesions penetrating the submucosa with negative lymph nodes (more than or equal to T1b). The current standard of care with a category 1 recommendation as per NCCN guidelines for tumors that are T2 or higher with or without lymph node involvement is perioperative chemotherapy (FLOT regimen) with surgical resection. Neoadjuvant chemoradiation has a category 2B status recommendation based on available data. Palliative systemic therapy is recommended for patients with metastatic disease. Go to the next page if you knew the correct answer, or click the link image(s) below to further research the concepts in this question (if desired).
Research Concepts: Gastric Cancer
We update eBooks quarterly and Apps daily based on user feedback. Please tap flag to report any questions that need improvement.
Question 791:
A 55-year-old male with a history of Paget disease and severe osteoarthritis presents to the emergency department with erythema, warmth, and swelling of the surgical incision. CT of the hip reveals no fluid formation. He has a history of total hip arthroplasty 4 months ago as well as resection of heterotopic ossification (HO) followed by radiation therapy (RT) 2 weeks ago. What is the most likely diagnosis?
Choices: 1. 2. 3. 4.
Wound healing complication Re-development of heterotopic ossification Septic arthritis Psoriasis
Answer: 1 - Wound healing complication Explanations: The clinical vignette is most consistent with wound complications following radiation therapy for heterotopic ossification. Treatment for HO consists of resection of the ossification followed by postoperative radiation therapy within 72 hours of surgery. Although the radiation given to patients is a low dose with benefits likely outweighing the potential risks, side effects can include fatigue, wound healing delays, joint swelling, and extremely low chance of secondary cancer from the radiation. For male patients, RT dose to the testicles is a concern as there can be a reduction in sperm count, so the use of testicular shielding during radiation is recommended. Go to the next page if you knew the correct answer, or click the link image(s) below to further research the concepts in this question (if desired).
Research Concepts: Radiation Therapy For Heterotopic Ossification Prophylaxis
We update eBooks quarterly and Apps daily based on user feedback. Please tap flag to report any questions that need improvement.
Question 792:
A 45-year-old woman presents to the hospital with a 4-month history of a lump on her neck. The lump is moderately painful. She appears well. The examination shows swelling on the left side of her neck that moves on swallowing. Cardiopulmonary examination shows no abnormalities. Vital signs are unremarkable. Her thyroidstimulating hormone (TSH) is 3.5 mU/mL. Ultrasound shows a 0.4 cm hypoechoic mass in the left thyroid lobe. Fineneedle aspiration of the mass shows neoplastic follicular cells. What is the most appropriate next step in management?
Choices: 1. 2. 3. 4.
Chemotherapy Watchful waiting TSH suppression therapy Thyroid lobectomy
Answer: 4 - Thyroid lobectomy Explanations: An ultrasound showing a hypoechoic mass and fineneedle aspiration showing neoplastic follicular cells should raise concern for follicular thyroid carcinoma (FTC). Thyroid lobectomy is the most suitable next step in management for this patient, as it allows for definitive diagnosis and is the preferred first-line treatment for patients with well-differentiated intrathyroidal cancers that are less than 1 cm in size. Diagnosis of FTC depends on abnormal positioning and spread of thyroid follicles, including extracapsular and vascular invasions. Nuclear atypia itself may or may not be present, although it does indicate a worse prognosis when present. Given that individual cellular morphology is insufficient for FTC diagnosis, a fine-needle aspiration (FNA) biopsy cannot be used to diagnose FTC fully. FNA cannot be used to distinguish between follicular adenoma and follicular carcinoma, as it does not show whether neoplastic cells have invaded the basement membrane. This distinction is made based on the histologic analysis of the tissue following surgery. Watchful waiting is inappropriate in patients with an FNA, and treatment delays may lead to cancer growth and invasion into the surrounding structures. Chemotherapy is used for cancers that are poorly differentiated. TSH suppression will be done after the surgical procedure is done. Go to the next page if you knew the correct answer, or click the link image(s) below to further research the concepts in this question (if desired).
Research Concepts: Thyroid Cancer
We update eBooks quarterly and Apps daily based on user feedback. Please tap flag to report any questions that need improvement.
Question 793:
A 66-year-old woman presents to the clinic with a right cheek mass. Physical exam shows a 4 cm fixed tender mass without any overlying skin changes. Fine needle aspiration shows mucoepidermoid carcinoma. Which of the following possible factors is most likely to be associated with a favorable prognosis in this patient?
Choices: 1. MECT1-MAML2 gene fusion 2. Limited mucous cells with large amounts of solid, squamous cells on histology 3. House-Brackmann 4/6 of the right face 4. Cervical lymphadenopathy
Answer: 1 - MECT1-MAML2 gene fusion Explanations: Mucoepidermoid carcinoma with MECT1-MAML2 gene fusion has a good prognosis with a 5-year survival rate of more than 90%. One of the most common genetic findings for mucoepidermoid carcinoma is the chromosomal translocation t(11;19)(q21:p13), leading to MECT1 and MAML2 gene fusion, which is responsible for disrupting the NOTCH signaling pathway. This translocation is found in 50-70% of patients with MEC and is more often seen in low-grade tumors with better prognosis. Positive prognostic factors include lack of cervical metastasis, no perineural or facial nerve involvement, low-grade histology, young age, and absence of comorbidities. Low-grade mucoepidermoid carcinoma has a better prognosis and is characterized by well-formed cystic spaces with plentiful glandular composition. High-grade mucoepidermoid carcinoma has a worse prognosis with limited mucous cells with large amounts of solid, squamous cells that can often be misdiagnosed as squamous cell carcinoma. Go to the next page if you knew the correct answer, or click the link image(s) below to further research the concepts in this question (if desired).
Research Concepts: Malignant Salivary Gland Tumors
We update eBooks quarterly and Apps daily based on user feedback. Please tap flag to report any questions that need improvement.
Question 794:
A 75-year-old man presents with a onemonth history of swallowing difficulty and mouth bleeding. He also reports difficulty in moving the tongue. He has a past medical history of coronary artery disease and a 35 pack-year smoking history. On examination, there is a 1.5 cm, indurated, slightly elevated, and velvety patch in the posterior part of the dorsal aspect of the tongue that the patient believes has been there for at least two months. The patch has an ulcerated surface. A biopsy shows nests of squamous epithelial cells arising from the epithelium and extending into the connective tissue. There are moderate dysplastic changes. The patient undergoes surgical excision and radiotherapy, but after one year the tumor remitted. Which of the following can be used as prognostic markers of oral malignant tumors?
Choices: 1. 2. 3. 4.
CDKN2A CEA 19-9 AFP MAP kinase
Answer: 1 - CDKN2A Explanations: The patient in the clinical scenario presents with oral squamous cell carcinoma. Consuming tobacco in any form and drinking alcohol constitute an increased risk for developing oral malignancies. Local and regional recurrence of the tumor is a major problem with oral cancers. CDKN2A and CDKN2B can be used as a prognostic marker. Oral cancer screening is helpful in the early diagnosis of oral cancers. Typical clinical findings include painful mouth ulcers that do not heal within several weeks. Persistent lumps in the mouth and nodes in the lymph glands in the neck that do no disappear. Other symptoms include dysphagia, unintentional weight loss, bleeding or numbness in the mouth, and difficulty speaking or moving the mandibula. Go to the next page if you knew the correct answer, or click the link image(s) below to further research the concepts in this question (if desired).
Research Concepts: Oral Mucosa Cancer
We update eBooks quarterly and Apps daily based on user feedback. Please tap flag to report any questions that need improvement.
Question 795:
A 58-year-old man with a history of alcohol use disorder and hepatitis B undergoes a gallium-68DOTATATE PET/CT scan. The scan shows diffuse liver and spleen uptake, with a small area of intense uptake in the pancreas and a large irregular area of intense uptake in the liver. What is the most likely reason for this finding?
Choices: 1. 2. 3. 4.
Hepatocellular carcinoma Insulinoma that metastasized to his liver Normal physiologic uptake for this patient Pancreatitis and liver cirrhosis
Answer: 2 - Insulinoma that metastasized to his liver Explanations: Insulinoma is a rare form of a neuroendocrine tumor in the pancreas. It is most likely the primary tumor that has metastasized to the liver. Gallium-68 DOTATATE is largely specific to neuroendocrine tumors. The Gallium-68 DOTATATE does not detect liver cirrhosis or pancreatitis. Hepatocellular carcinoma does not metastasize to the pancreas. Focal lesions should raise suspicions of pathology on PET scans. Go to the next page if you knew the correct answer, or click the link image(s) below to further research the concepts in this question (if desired).
Research Concepts: Gallium Scan
We update eBooks quarterly and Apps daily based on user feedback. Please tap flag to report any questions that need improvement.
Question 796:
A 12-year-old boy is brought to the emergency department due to odd behavior, lethargy, and headache. On examination, he has ataxia, nystagmus, and papilloedema. An MRI head shows a posterior fossa tumor focused on the midline of the cerebellum. It has a large cystic component and an enhancing nodular mural. A surgical biopsy is taken. Microscopy of a biopsy shows long bipolar cellular processes. Which of the following sets of molecular markers is most consistent with the patient's tumor?
Choices: 1. 2. 3. 4.
IDH mutant, BRAF and S-100 positive BRAF and GFAP positive with IDH mutation present BRAF, GFAP, and S-100 positive CEA and AFP positive
Answer: 3 - BRAF, GFAP, and S-100 positive Explanations: A pediatric posterior fossa tumor with a cystic and nodular mural is consistent with pilocytic astrocytoma. Pilocytic astrocytoma gets its name from long cellular processes that resemble hair-like structures. They tend to be positive for BRAF, GFAP, and S-100 while being negative for IDH mutation. Treatment is surgical resection. Go to the next page if you knew the correct answer, or click the link image(s) below to further research the concepts in this question (if desired).
Research Concepts: Pilocytic Astrocytoma
We update eBooks quarterly and Apps daily based on user feedback. Please tap flag to report any questions that need improvement.
Question 797:
A 53-year-old man with a history of metastatic melanoma presents with headaches, photophobia, and visual field defects. For the last six months ago, he has been receiving ipilimumab and nivolumab every three weeks, with his last treatment being one week ago. MRI brain shows swelling and enhancement of the pituitary gland. Further investigations show low cortisol, low T4, low ACTH, and low TSH. What is the most likely diagnosis?
Choices: 1. 2. 3. 4.
Prolactinoma Immune-mediated hypophysitis Adrenal insufficiency Carcinoid tumor
Answer: 2 - Immune-mediated hypophysitis Explanations: Immune-mediated hypophysitis is seen with combination chemotherapy with nivolumab and ipilimumab. Patients present with symptoms of fatigue and headache. The evaluation shows low cortisol, T4, ACTH, TSH, FSH, LH, GH, and prolactin. Imagining shows swelling and enhancement of the pituitary gland. Treatment is steroids and supplementation of the affected hormones. Go to the next page if you knew the correct answer, or click the link image(s) below to further research the concepts in this question (if desired).
Research Concepts: Lymphocytic Hypophysitis
We update eBooks quarterly and Apps daily based on user feedback. Please tap flag to report any questions that need improvement.
Question 798:
A 65-year-old male patient was diagnosed with glioblastoma multiforme and underwent surgical resection followed by chemoradiotherapy. Repeat MRI done six weeks after initiation of therapy showed the increase in size with reduced cerebral blood volume, low choline ratio Cho/NAA ratio of less than1.48, increased lactate peak, increased lipid peak, and elevated ADC values due to cell death. On evaluation, the patient denies any new symptoms. What is the next best step in management?
Choices: 1. Discontinue the intensive treatment due to the progression of the disease and recommend palliative measures. 2. Continue with therapy followed by repeat imaging in 6-8 weeks. 3. Proceed with surgery again. 4. Get a PET scan to assess for progression of the disease.
Answer: 2 - Continue with therapy followed by repeat imaging in 6-8 weeks.
Explanations: Continuing with therapy followed by repeat imaging in 2 months is recommended as the imaging finding is likely a treatment-related effect called pseudoprogression. Pseuoprogression is usually asymptomatic, which is the case in this scenario. Pseudoprogression is a phenomenon that is usually seen four to six weeks after the completion of chemoradiotherapy. Pseudoprogression is a treatment-related effect with imaging findings similar to tumor progression. It usually occurs within three months of completion of chemoradiation in glioblastoma patients. Distinguishing between pseudoprogression due to treatment and true progression of the disease is important to avoid inappropriate discontinuation of treatment in patients. Go to the next page if you knew the correct answer, or click the link image(s) below to further research the concepts in this question (if desired).
Research Concepts: Glioblastoma Multiforme
We update eBooks quarterly and Apps daily based on user feedback. Please tap flag to report any questions that need improvement.
Question 799:
A 25-year-old male is recently diagnosed with medullary thyroid carcinoma, and his metabolic panel is significant for hypercalcemia and hypophosphatemia. He also reports episodes of dizziness accompanied by sweating and feeling lightheaded. He says his mother had her thyroid gland removed when he was a child, but he does not remember the exact diagnosis. The patient's clinical findings suggest a possible mutation in which of the following genes?
Choices: 1. 2. 3. 4.
RET BRAF p53 RAS
Answer: 1 - RET Explanations: The findings described in the question are typically found in patients with multiple endocrine neoplasia (MEN) type 2, (Sipple syndrome). The MEN syndromes have an autosomal dominant pattern of inheritance, and MEN-2A and MEN-2B are both linked to mutations in the RET proto-oncogene. MEN type 2A is characterized by parathyroid gland hyperplasia or tumor leading to hypercalcemia, medullary carcinoma of the thyroid, and pheochromocytoma (which commonly causes elevated plasma catecholamine levels). The patient’s episodes of lightheadedness and sweating can be attributed to the increased catecholamine level found in pheochromocytoma. MEN type 2B is characterized by pheochromocytoma, multiple mucocutaneous neuromas, and medullary carcinoma of the thyroid and is also associated with RET mutations. Mutations in BRAF are associated with papillary carcinoma of the thyroid. RAS gene mutations are associated with follicular thyroid carcinoma, not medullary carcinoma of the thyroid. Go to the next page if you knew the correct answer, or click the link image(s) below to further research the concepts in this question (if desired).
Research Concepts: Multiple Endocrine Neoplasias Type 2
We update eBooks quarterly and Apps daily based on user feedback. Please tap flag to report any questions that need improvement.
Question 800:
A 61-year-old man with a painless, round, yellow-tan nodule on his upper eyelid is diagnosed with chalazion. Despite treatment with warm compresses, the lesion has slowly grown and is now ulcerated. Which therapy is the most appropriate for this patient after a biopsy?
Choices: 1. 2. 3. 4.
Cryotherapy Topical 5-fluorouracil Mohs micrographic surgery IV clindamycin
Answer: 3 - Mohs micrographic surgery Explanations: The patient's clinical presentation is consistent with a sebaceous carcinoma. Sebaceous carcinomas are most often present on the eyelids. Sebaceous carcinomas are frequently misdiagnosed as a chalazion. Wide local excision or Mohs micrographic surgery is first-line therapy for the treatment of sebaceous carcinomas. Go to the next page if you knew the correct answer, or click the link image(s) below to further research the concepts in this question (if desired).
Research Concepts: Sebaceous Gland Carcinoma
We update eBooks quarterly and Apps daily based on user feedback. Please tap flag to report any questions that need improvement.
Section 9 Question 801:
A 5-year-old African American female presents with complaints of abdominal pain and gross hematuria. On physical examination, she is afebrile, her blood pressure is 126/78 mmHg, and her heart rate is 92 beats per minute. The abdomen was soft and non-tender, but there was a palpable abdominal mass on the left side of the abdominal wall. Ultrasound of the abdomen revealed a mass with cystic areas adjacent to the lower pole of the left kidney. An ultrasound-guided biopsy of the mass was performed, which showed blastema tissue, undifferentiated mesenchymal cells, and primitive epithelial rosette-like structures. Cytogenetic studies revealed 1p and 16q deletions. Blood workup showed hemoglobin of 9.5 mg/dL, a mean corpuscular volume (MCV) of 62.5 fL, white blood cell count of 3,250 cells/microliter, and platelet count of 226,000 platelets/microliter. Which of the following features of the disease is associated with a poorer prognosis?
Choices: 1. 2. 3. 4.
Female gender Presence of 1p and 16q gene deletions African American descent Presence of a rare hematological abnormality
Answer: 2 - Presence of 1p and 16q gene deletions Explanations: Ninety percent of Wilms tumors will demonstrate "favorable" histology, which generally has a better prognosis. Classical histological features of a "favorable" Wilms tumor include a triphasic pattern of a blastema, epithelial, and stromal tissues. Even with "favorable" histology, the loss of heterozygosity at 1p and 16q loci tends to have a worse prognosis. For this reason, when Wilms tumor tissue is available, it should be checked cytogenetically for 1p and 16q deletions. Wilms tumor is more common in Africans and African Americans, while it is least common in East Asians. Asian patients also have fewer unfavorable histology tumors, tend to have a lower-stage disease, and generally enjoy better survival outcomes, but this does not necessarily mean that the former will always have a poor prognosis. A poorer prognosis has also been linked to TP53 and is associated with the loss of heterozygosity at chromosomes 1p, 1q, 11p15, and 16q. Go to the next page if you knew the correct answer, or click the link image(s) below to further research the concepts in this question (if desired).
Research Concepts: Wilms Tumor
We update eBooks quarterly and Apps daily based on user feedback. Please tap flag to report any questions that need improvement.
Question 802:
A 71-year-old male presents with complaints of fatigue for the past few weeks. He denies any bloody stools. His last colonoscopy was a year ago and was unremarkable. He does not have any other medical conditions, and he does not take any medications. He does not smoke or drink alcohol. Vitals include a temperature of 37.2 C (99 F), blood pressure of 130/75 mmHg, and respiratory rate of 18/min. Physical examination reveals pallor. No other abnormal findings on the physical exam. Laboratory results show hemoglobin 8 g/dl, MCV 100 fL, leukocyte count 3000/microL, platelets 150,000/microL, vitamin B12 500 picogram/ml, folate 15 ng/dl, and TSH 4 mIU/L. Peripheral smear shows hyposegmented neutrophils. Bone marrow biopsy shows hypercellular marrow with dysplastic myeloid progenitor cells. Which of the following is a complication of the above condition?
Choices: 1. 2. 3. 4.
Acute myeloid leukemia Myelofibrosis Chronic myeloid leukemia Chronic lymphocytic leukemia
Answer: 1 - Acute myeloid leukemia Explanations: The patient presents with pancytopenia, normal vitamin B12, TSH, and folate levels. Besides, his peripheral blood smear shows hyposegmented neutrophils, and bone marrow biopsy shows hypercellular marrow. The mentioned findings in an older male point towards myelodysplastic syndrome (MDS). Myelodysplastic syndrome is a condition affecting the hematopoietic stem cells leading to ineffective hematopoiesis. This can result in cytopenia affecting anywhere from 1 to 3 cell lines. Symptoms typically depend on the cell line(s) affected. Myelodysplastic syndrome typically causes macrocytic anemia. Peripheral smear shows dysplastic cells such as ovalomacrocytosis, hypogranular hyposegmented neutrophils. Bone marrow biopsy is diagnostic and shows hypercellular marrow with 1 or 3 lineage dysplasia. Patients with few blasts in the bone marrow and mild to moderate cytopenia have a good prognosis. However, patients with more than 10% blasts and severe pancytopenia have a poor prognosis. The myelodysplastic syndrome may progress to acute myeloid leukemia (AML). Treatment goals for myelodysplastic syndrome include symptom relief and prevention of progression to acute myeloid leukemia. Patients may need transfusions as needed. Agents such as azacytidine and decitabine can reduce transfusions and progression to acute myeloid leukemia. Allogenic hematopoietic stem cell transplant is the curative treatment. However, this is not usually preferred in older patients.
Go to the next page if you knew the correct answer, or click the link image(s) below to further research the concepts in this question (if desired).
Research Concepts: Pancytopenia
We update eBooks quarterly and Apps daily based on user feedback. Please tap flag to report any questions that need improvement.
Question 803:
A 71-year-old male comes to the endocrinology clinic for a follow-up of his Cushing disease. He required bilateral adrenalectomy three years prior for non-responsive medical management. His new brain magnetic resonance imaging shows a new large 2.3 cm sellar and suprasellar tumor. He is presenting bitemporal hemianopia. Four months ago, he suffered a myocardial infarct, which was treated with a stent. Which is the best treatment for his new tumor?
Choices: 1. 2. 3. 4.
Somatostatin analogs Radiation therapy Surgical treatment - microscopic Surgical treatment - endoscopic
Answer: 4 - Surgical treatment - endoscopic Explanations: The best treatment for Nelson syndrome is the surgical removal of the tumor. This can be performed microsurgically or endoscopically transsphenoidal, the latter being preferred these days. It is prudent to wait for 8 weeks after myocardial infarction before performing elective surgery. Since the cardiac event occurred 4 months ago, this patient can go ahead with elective surgery if the ejection fraction is adequate. Radiotherapy in the form of fractionated radiotherapy or stereotactic radiosurgery can be given to the tumor if the patient is a poor surgical candidate. Some patients may have already received radiation as treatment for the Cushing disease, and the second course of radiotherapy may be contraindicated. The effects of radiotherapy will cause hypopituitarism in the next 5-10 years, so that hormonal replacement may be needed. Medical treatment is limited, but some results have been found with somatostatin-analogs, cabergoline, and temozolomide. Go to the next page if you knew the correct answer, or click the link image(s) below to further research the concepts in this question (if desired).
Research Concepts: Nelson Syndrome
We update eBooks quarterly and Apps daily based on user feedback. Please tap flag to report any questions that need improvement.
Question 804:
A 51-year-old female with a personal history of clear cell carcinoma of the ovary eight years earlier underwent radical mastectomy for a breast mass initially diagnosed as invasive carcinoma of no special type on core biopsy. On histologic examination of the surgical specimen, the breast tumor was well-circumscribed with a pushing border. The neoplasm was composed of malignant cells that are not forming glands but, instead, large sheets with no architectural pattern. In addition, there is a rim of lymphocytes and mononuclear cells. At high magnification, highly atypical cells with vesicular nuclei and prominent single or multiple nucleoli are apparent. There are nuclear pleomorphism and numerous mitotic figures. Lymphocytes are very prominent and are mixed in with the tumor cells. What is the most likely immunohistochemical profile of this tumor?
Choices: 1. Ki67: 5%; P53: positive; estrogen receptor-negative, progesterone receptor-negative, and HER2/neu negative 2. Ki67: 80%; P53: positive; estrogen receptor-negative, progesterone receptor-negative, and HER2/neu negative 3. Ki67: 80%; P53: negative; estrogen receptor-negative, progesterone receptor-negative, and HER2/neu negative 4. Ki67: 80%; P53: positive; estrogen receptor-positive, progesterone receptor-positive, and HER2/neu negative
Answer: 2 - Ki67: 80%; P53: positive; estrogen receptornegative, progesterone receptor-negative, and HER2/neu negative
Explanations: Medullary carcinomas are often estrogen receptor, progesterone receptor, and HER2/neu negative. Overexpression of p53 protein results from TP53 mutation. Medullary breast carcinomas are often P53 positive. Medullary carcinomas have a high Ki-67 proliferation index and mitoses are numerous. The diagnosis of medullary breast carcinoma is established in the majority of cases on H&E sections using histologic criteria without the need for immunohistochemical study. Thus, the diagnosis of medullary breast carcinoma does not rely on the immunohistochemical profile, which usually shows a high Ki-67 proliferation index, P53 positivity, estrogen receptor negativity, progesterone receptor negativity, and HER2/neu negativity. Go to the next page if you knew the correct answer, or click the link image(s) below to further research the concepts in this question (if desired).
Research Concepts: Medullary Breast Carcinoma
We update eBooks quarterly and Apps daily based on user feedback. Please tap flag to report any questions that need improvement.
Question 805:
A 16-year-old male deceased donor renal transplant recipient presents 6 months after his transplant complaining of constitutional symptoms including cyclical fevers, night sweats, and malaise. His EBV viral load is 40,000/mL. What is the most likely diagnosis?
Choices: 1. 2. 3. 4.
Burkitt cell lymphoma Peripheral T-cell lymphoma Diffuse large B-cell lymphoma Plasma cell lymphoma
Answer: 3 - Diffuse large B-cell lymphoma Explanations: Post transplant lymphoproliferative disorder (PTLD) is most commonly associated with EBV viremia as the virus transforms B-cells. More than 80% occur in the first year after transplant. Those most at risk include young patients, those that are more heavily immunosuppressed, and those who were seronegative pre-transplant and received a seropositive donor allograft. Diffuse large B-cell lymphoma is the most common B-cell lymphoma seen in monomorphic PTLD. T-cell lymphomas and Plasma cell lymphomas are rare. Go to the next page if you knew the correct answer, or click the link image(s) below to further research the concepts in this question (if desired).
Research Concepts: Kidney Transplantation
We update eBooks quarterly and Apps daily based on user feedback. Please tap flag to report any questions that need improvement.
Question 806:
A 55-year-old woman presents after coughing up blood. She has had a dry cough for 1 month, which she put down to seasonal allergies. She is a smoker with 12 pack-year smoking history. A lung biopsy shows atypical squamous and glandular cells. There is no evidence of cells of neuroendocrine origin. On chest radiography, which of the following would be the most likely earliest presentation of this cancer?
Choices: 1. 2. 3. 4.
Cavity Solitary nodule Hilar adenopathy Consolidation
Answer: 2 - Solitary nodule Explanations: A solitary pulmonary nodule (SPN) is seen in the majority of non-small cell cancer. Ninety-five percent of patients with non-small cell lung cancer (NSCLC) are symptomatic at presentation, and most have some evidence of lung pathology on chest xray. The majority of solitary pulmonary nodules are not malignant, but depending on clinical and imaging context, many require workup. The workup of an SPN may vary and can involve careful history, comparison to old studies, CT (including dynamic contrast enhancement characteristics), and positron emission tomography-computed tomography (PET/CT). Go to the next page if you knew the correct answer, or click the link image(s) below to further research the concepts in this question (if desired).
Research Concepts: Non Small Cell Lung Cancer
We update eBooks quarterly and Apps daily based on user feedback. Please tap flag to report any questions that need improvement.
Question 807:
An 18-year-old male presents with swelling of the upper end of the tibia. Local examination demonstrates redness, warmth, and tenderness to touch. He is subsequently planned for chemotherapy with the methotrexate, adriamycin, and cisplatin. He is administered methotrexate over 4 to 6 hours on day 1. He presents with complaints of diarrhea, oral mucositis, and decreased urine output on day 2 of treatment. Blood methotrexate levels at the time of presentation are 450 umol/L. Which of the following is the best treatment recommendations for the management of this toxicity?
Choices: 1. The decrease in fluid intake, leucovorin rescue, hemodialysis 2. Increase in fluid intake, increase in leucovorin administered, carboxypeptidase G2 and hemodialysis 3. Increase in fluid intake, naproxen, increase in the amount of leucovorin, and hemodialysis 4. Maintenance fluids, aspirin, initiation of leucovorin rescue, and hemodialysis
Answer: 2 - Increase in fluid intake, increase in leucovorin administered, carboxypeptidase G2 and hemodialysis
Explanations: Methotrexate is an irreversible inhibitor of dihydrofolate reductase, which leads to inhibition of thymidylate synthesis and subsequent cell death. Leucovorin or folinic acid rescue is used along with methotrexate, to replenish tetrahydrofolate stores and ameliorate the toxicity on normal cells. Leucovorin is usually administered 24 hours after the initiation of methotrexate therapy. Methotrexate levels at 24 hours post-infusion should be within the range of 30-300 umol/L. A normal hepatic and renal function, normal complete blood count, and absence of infection are prerequisites before starting methotrexate therapy. Methotrexate toxicity manifests as oral and mucosal ulceration, hepatorenal failure, and myelosuppression. The deposition of methotrexate crystals is responsible for the nephrotoxicity, which may be exacerbated by the concomitant administration of other potentially nephrotoxic agents such as cisplatin. Methotrexate toxicity is generally treated with an increase in fluid intake, an increase in leucovorin dose to 50 -100 mg every six-hourly, carboxypeptidase G2 for oliguria/anuria and hemodialysis as indicated. Go to the next page if you knew the correct answer, or click the link image(s) below to further research the concepts in this question (if desired).
Research Concepts:
Osteogenic Sarcoma
We update eBooks quarterly and Apps daily based on user feedback. Please tap flag to report any questions that need improvement.
Question 808:
A 55-year-old man presents to the clinic with complaints of a mass in his right thigh. The patient states the mass started developing 18 months ago. At first, the mass was painless; however, whenever the patient walks, he experiences a pulling sensation and pain in his thigh. An x-ray of the right thigh reveals a soft tissue mass with no attachment to the underlying bones. A fine-needle aspiration biopsy reveals a dense myxoid background with delicate branching vessels, cells with multiple cytoplasmic vacuoles, and round to spindle cells with scant cytoplasm. Which of the following cytogenetic abnormalities is most likely to be found in this patient?
Choices: 1. 2. 3. 4.
t(X;17)(p11;q25) t(11;22)(q24;q12) t(12;16)(q13;p11) t(X;18)(p11;q11)
Answer: 3 - t(12;16)(q13;p11) Explanations: This patient has a myxoid liposarcoma. The relative frequency of liposarcoma at various body sites is much dependent on the tumor subtypes. For example, dedifferentiated liposarcoma is much more common in retroperitoneal location, while myxoid liposarcoma occurs in the lower extremities. In myxoid liposarcoma (MLS), reciprocal translocation occurs between chromosomes 12 and 16, leading to the fusion of genes DDIT3 and FUS, and subsequently, activating some downstream targets PPAR-gamma2 and C/EBPalpha, which promotes cell cycle proliferation. Fluorescence in situ hybridization (FISH) is frequently used to demonstrate t(12;16)(q13;p11). Low-grade MLS have low cellularity with bland nuclei and a rich, prominent network of curving capillaries, resembling a chicken-wire pattern. High-grade MLS are markedly hypercellular with solid sheets of round cells comprising at least 5% of the tumor. Go to the next page if you knew the correct answer, or click the link image(s) below to further research the concepts in this question (if desired).
Research Concepts: Liposarcoma
We update eBooks quarterly and Apps daily based on user feedback. Please tap flag to report any questions that need improvement.
Question 809:
A 65-year-old man with hypertension presents for an annual check-up. His blood pressure is 124/78 mmHg, heart rate 76/minute, respiratory rate 12/minute. Abdominal examination demonstrates a palpable splenic tip. On further questioning, the patient reveals having episodes of shortness of breath with exertion. A complete blood count is notable for hemoglobin of 10.3 g/dL. Ultrasound of the abdomen shows an enlarged spleen with heterogeneous texture and no focal lesions. He is referred to a hematologist, and after extensive workup, splenectomy with tissue diagnosis is recommended. Histological examination of the biopsy specimen shows sinus-like anastomosing channels with an irregular lumen, and immunohistochemical staining is positive for CD31, CD34, and CD68. Given the likely diagnosis, which of the following best describes this patient's condition?
Choices: 1. The patient needs a bone marrow biopsy. 2. The patient should be started on rituximab-based chemotherapy. 3. The patient has an excellent prognosis because the condition is likely benign. 4. The patient needs full-body FDG-PET imaging.
Answer: 3 - The patient has an excellent prognosis because the condition is likely benign.
Explanations: Littoral cell angioma can present as an incidental finding of splenomegaly; however, patients may have abdominal pain, anemia, and thrombocytopenia. Imaging studies are nonspecific and not diagnostic for littoral cell angioma. Histological examination and immunohistochemical analysis of the splenic specimen are essential to make a diagnosis. Typical histopathological findings of littoral cell angioma include sinus-like anastomosing channels with an irregular lumen that resemble splenic sinusoidal architecture. The endothelial cells lining the sinuses can have hemophagocytosis and lack features of nuclear atypia or mitotic activity. The immunohistochemical staining for littoral cells usually reveals a dual differentiation pattern. The cells stain positive for endothelial markers, e.g., FVIII, CD31, vWF, CD34, and histiocytic markers, e.g., CD 68. Histological and immunohistochemical analysis is important to differentiate littoral cell angioma from other splenic vascular lesions. Currently, splenectomy followed by long-term follow-up is the recommended treatment for littoral cell angioma, given its malignant potential. Go to the next page if you knew the correct answer, or click the link image(s) below to further research the concepts in this question (if desired).
Research Concepts:
Littoral Cell Splenic Angioma
We update eBooks quarterly and Apps daily based on user feedback. Please tap flag to report any questions that need improvement.
Question 810:
A 65-year-old man with a history of pancreatic cancer presents to the clinic for follow up. He has had chemotherapy-induced nausea and vomiting (CINV) and is still experiencing symptoms after being treated with ondansetron. The patient's medical history is significant for well-controlled schizophrenia managed with monthly depot antipsychotic injections. A drug that targets the CB1 receptor for the treatment of CINV is being considered. Which of the following complications is most likely to occur as a result of this treatment in this patient?
Choices: 1. 2. 3. 4.
Serotonin syndrome Exacerbation of psychiatric symptoms Neuroleptic malignant syndrome Torsades des pointes
Answer: 2 - Exacerbation of psychiatric symptoms Explanations: The antiemetic and appetite-stimulating effects of the CB1 receptor are believed to mediate the therapeutic effects of dronabinol. In chemotherapy-induced nausea and vomiting in patients who have failed to respond to conventional antiemetics, the recommended starting adult dosage is 5 mg orally 1 to 3 hours prior to chemotherapy, followed by 5 mg every 2 to 4 hours (for a total of 4 - 6 doses in a day). The first dose should be on an empty stomach (at least 30 minutes before eating). The following doses can be taken with or without meals. Similar to dosing for anorexia, elderly patients may be started in a lower dose of 2.5 mg once daily prior to chemotherapy to reduce CNS symptoms. It can be titrated up in increments of 2.5 mg with a maximum dose of 15 mg per dose (for 4 to 6 doses per day). Dronabinol can cause psychiatric and cognitive effects (may need to be avoided in those with psychiatric history). It may also result in mental or physical impairment. Patients should not operate heavy machinery or motor vehicles until certain that drug does not impair their ability to operate. Avoid drugs with similar effects. Dronabinol is formulated in sesame oil. As such, this drug is contraindicated in patients who have a history of hypersensitivity reaction to sesame oil or THC derivatives. Careful attention should be paid to signs of hypersensitivity reactions.
Go to the next page if you knew the correct answer, or click the link image(s) below to further research the concepts in this question (if desired).
Research Concepts: Dronabinol
We update eBooks quarterly and Apps daily based on user feedback. Please tap flag to report any questions that need improvement.
Question 811:
A 68-year old female receiving systemic fluorouracil (5-FU) for the treatment of pancreatic adenocarcinoma develops neutropenia during the course of her treatment. What is the mechanism of action of the reversal agent used in cases of 5-FU toxicity?
Choices: 1. Blocking the facilitated transport pump allowing 5-FU into cells 2. Thymine analog that competes with 5-FU during DNA replication and transcription 3. Selective activation of the enzyme thymidylate synthase 4. High concentrations of uridine compete with 5-FU metabolites for binding sites
Answer: 4 - High concentrations of uridine compete with 5-FU metabolites for binding sites
Explanations: Vistogard is a pyrimidine analog composed of uridine triacetate that can be used for patients experiencing adverse effects from 5-FU treatment. Uridine triacetate is an acetylated prodrug of uridine that is administered orally. Uridine triacetate has been shown to be safe and efficacious for treating 5-FU toxicity if given within four days of the most recent dose of 5-FU. Uridine triacetate works by providing high concentrations of uridine that compete with 5-FU metabolites for binding sites in the body. Go to the next page if you knew the correct answer, or click the link image(s) below to further research the concepts in this question (if desired).
Research Concepts: Fluorouracil
We update eBooks quarterly and Apps daily based on user feedback. Please tap flag to report any questions that need improvement.
Question 812:
A 65-year-old man presents to the hospital with hearing loss and numbness in his hands. On examination, he is afebrile; his blood pressure is 133/81 mmHg, the pulse of 81 beats per minute, and a respiratory rate of 24 breaths per minute. Furthermore, there is lymphadenopathy, splenomegaly, peripheral neuropathy in both hands, and bilateral hearing loss. The patient’s laboratory values are sodium of 141 mmol/L, potassium of 4.5 mmol/L, magnesium of 1.2 mmol/L, and glucose of 4.4 mmol/L. Immunoglobin plasma concentrations are IgA of 150 mg/dL, IgD of 1.1 mg/dL, IgE of 0.19 mg/dL, IgG of 250 mg/dL , and an IgM of 355 mg/dL. The patient has been prescribed a medication that induces apoptosis of cancer cells. Which of the following is the most likely diagnosis?
Choices: 1. 2. 3. 4.
Hair cell leukemia Waldenstrom macroglobulinemia Prostate cancer Mantle-cell lymphoma
Answer: 2 - Waldenstrom macroglobulinemia Explanations: Bortezomib is used to treat B-cell malignancies like Waldenstrom macroglobulinemia. Bortezomib is FDA approved as front-line therapy for multiple myeloma. It is approved for relapsed or refractory mantle-cell lymphoma. Bortezomib is a proteasome inhibitor. The proteasomal system plays a vital role in cellular protein turnover, which is essential for the homeostasis of cells. Bortezomib reversibly binds to the chymotrypsin-like subunit of the 26S proteasome, resulting in its inhibition and preventing the degradation of various pro-apoptotic factors. Bortezomib has not been therapeutically effective in solid tumors, e.g., prostate cancer. Go to the next page if you knew the correct answer, or click the link image(s) below to further research the concepts in this question (if desired).
Research Concepts: Bortezomib
We update eBooks quarterly and Apps daily based on user feedback. Please tap flag to report any questions that need improvement.
Question 813:
A 66-year-old man is diagnosed with a neuroendocrine tumor in the small intestine after undergoing a magnetic resonance image (MRI). He presented to the clinic for routine follow up. Presently, he is asymptomatic and vitally stable. An octreotide scan was advised by the provider which showed uptake of the tracer more than in the liver. Which of the following is the next course of management?
Choices: 1. 2. 3. 4.
Peptide receptor radionuclide therapy Positron emission tomography (PET) scan Magnetic resonance image (MRI) Histopathology is required
Answer: 1 - Peptide receptor radionuclide therapy Explanations: Krenning score is used in grading uptake intensity of neuroendocrine tumors on somatostatin receptor imaging modalities like octreotide scan. The grading is as follows: grade 1: uptake less than a normal liver, grade 2: uptake equal to normal liver, grade 3: uptake greater than a normal liver, and grade 4: uptake greater than the spleen or kidneys. Typically PRRT is taken into consideration if this score is greater than 2. As the uptake is more than the liver, in this case, PPRT is indicated. Go to the next page if you knew the correct answer, or click the link image(s) below to further research the concepts in this question (if desired).
Research Concepts: Octreotide Scan
We update eBooks quarterly and Apps daily based on user feedback. Please tap flag to report any questions that need improvement.
Question 814:
A 30-year-old woman presents to the clinic for a follow-up. She was recently found to have a spiculated mass in the upper outer left breast. Her biopsy shows moderately differentiated ductal carcinoma with estrogen receptor (ER) and progesterone receptor (PR) positivity, and negative for HER2/neu. She also reports that she has not had her periods in the last two months, and yesterday her urine pregnancy test came positive. She expresses her desire to continue her pregnancy as the previous two pregnancies were spontaneously aborted. Which of the following is the most appropriate management strategy for this patient?
Choices: 1. PET-CT, radical mastectomy, and neo-adjuvant chemotherapy 2. Hypofractionated whole breast radiation therapy and endocrine therapy after the first trimester 3. Mastectomy and axillary staging, adjuvant chemotherapy in the second trimester, and possible adjuvant radiation and endocrine therapy postpartum 4. Recommend aborting the pregnancy for whole-body external beam irradiation and radical mastectomy
Answer: 3 - Mastectomy and axillary staging, adjuvant chemotherapy in the second trimester, and possible adjuvant radiation and endocrine therapy postpartum
Explanations: The fetus is highly susceptible to ionizing radiations during the first trimester, and if exposed to radiation above 0.5 Gy, it will develop certain malformations. The National Comprehensive Cancer Network recommends mastectomy and axillary staging, adjuvant chemotherapy in the second trimester, and possible adjuvant radiation and endocrine therapy postpartum Aborting ongoing pregnancy should be the last resort, only after weighing various modalities against the possible outcomes, and should be left to the patient and her partner. Chemotherapy has shown no substantial implications on fetal growth parameters, irrespective of fetal age. Hence it should be the first line of therapy while surgical or radiotherapeutic regimens are awaited. It's important to conduct a thorough obstetrical history and exam in patients of reproductive age group as pregnancy can complicate the net outcomes for both mother and fetus, especially when radiotherapy is required. Go to the next page if you knew the correct answer, or click the link image(s) below to further research the concepts in this question (if desired).
Research Concepts: Radiation Effects On The Fetus
We update eBooks quarterly and Apps daily based on user feedback. Please tap flag to report any questions that need improvement.
Question 815:
Which imaging modality is the most sensitive at identifying the radiologic findings of extensive intraductal component (EIC), which are common in ductal carcinoma in situ?
Choices: 1. 2. 3. 4.
Full field digital mammography Breast ultrasound MRI Breast scintigraphic gamma imaging
Answer: 3 - MRI Explanations: Based on a 2004 Radiology paper by Berg, extensive intraductal component (EIC) is common with invasive ductal carcinoma (IDC), seen in over 20 percent of cases. Mammography underestimates lesion size by > 2 cm in 37 percent. Breast ultrasound also underestimates EIC in 37 percent. MRI underestimates EIC in only 5 percent. Go to the next page if you knew the correct answer, or click the link image(s) below to further research the concepts in this question (if desired).
Research Concepts: Breast Ductal Carcinoma in Situ
We update eBooks quarterly and Apps daily based on user feedback. Please tap flag to report any questions that need improvement.
Question 816:
A patient presents with a neck mass which has been confirmed as a medullary thyroid carcinoma (MTC) with a needle aspiration biopsy. What is the next step in the management of this patient?
Choices: 1. 2. 3. 4.
Total thyroidectomy Send urine for catecholamine analysis Perform an open thyroid biopsy Obtain a CT scan of the neck
Answer: 2 - Send urine for catecholamine analysis Explanations: Medullary carcinoma of the thyroid originates in C cells of the thyroid. C cells produce calcitonin. Isolated cases of medullary cancer account for most tumors, but at least 25% occur in association with multiple endocrine neoplasia. In a patient with medullary cancer, it is essential to rule out pheochromocytoma, which can affect the morbidity. Serum calcitonin levels are a marker for medullary thyroid carcinoma, but to make a diagnosis of pheochromocytoma the urine needs to be analyzed for metanephrines. Go to the next page if you knew the correct answer, or click the link image(s) below to further research the concepts in this question (if desired).
Research Concepts: Medullary Thyroid Cancer
We update eBooks quarterly and Apps daily based on user feedback. Please tap flag to report any questions that need improvement.
Question 817:
A 79-year-old African American male presents with a 2-week history of intermittent, painless gross hematuria. His past medical history is significant for coronary artery disease status post coronary artery bypass grafting and multiple cardiac stents, chronic stable angina, hypertension, diabetes mellitus, peripheral vascular disease, and chronic kidney disease. The patient denies any history of renal stones or any symptoms suggestive of a urinary tract infection. His vital signs revealed a heart rate of 70 beats per minute, blood pressure of 155/86 mmHg, a temperature of 36.7 C (98 F), respiratory rate of 14 breaths per minute, oxygen saturation of 97% on room air, and BMI of 23. On physical examination, the patient appears chronically ill, but not in any acute distress. Abdominal examination revealed mild right-sided flank tenderness. Respiratory and cardiovascular examinations were normal. Laboratory data revealed a white blood cell (WBC) count of 5,000 per microliter, hemoglobin of 10 g/dL, platelets of 210 per microliter, sodium 135 mEq/L, potassium 5 mEq/L, chloride 106 mEq/L, CO2 21mEq/L, blood urea nitrogen 30 mg/dL, creatinine 2.1 mg/dL, calcium 8.4 mEq/L, and albumin 3.1 mg/dL. Urinalysis revealed specific gravity of 1.010, blood ++, 30 protein, 30 red blood cells per high power field, and negative for WBC, nitrites, and leukocyte esterase. A computed tomography (CT) of the abdomen/pelvis revealed a 4.9 cm mass in the right kidney suspicious for renal cell carcinoma. What is the next best step?
Choices: 1. Partial nephrectomy 2. Observation and CT scan in 3 months 3. Radiofrequency ablation and cryotherapy
. Sunitinib 4
Answer: 3 - Radiofrequency ablation and cryotherapy Explanations: The patient has stage I (T1b) for which a partial nephrectomy is usually recommended. But, given his age and co-morbidities, he is not a good surgical candidate. For non-surgical candidates with T1 lesions, radiofrequency ablation and cryotherapy are recommended. Observation and reimaging are not an option. The patient does not qualify for chemotherapy. Compared to partial nephrectomy and radical nephrectomy, the rate of disease recurrence is higher in patients who undergo radiofrequency ablation and cryotherapy. Go to the next page if you knew the correct answer, or click the link image(s) below to further research the concepts in this question (if desired).
Research Concepts: Clear Cell Renal Carcinoma
We update eBooks quarterly and Apps daily based on user feedback. Please tap flag to report any questions that need improvement.
Question 818:
A 38-year-old single mother of 2 daughters presents to the clinic with a report from a CT scan of her abdomen revealing a 3.2 cm mass in her right adrenal gland. The scans were performed two weeks ago as part of her preoperative evaluation for a proposed elective hysterectomy to manage her large, symptomatic submucosal uterine leiomyoma. She denies having any nausea, headache, or palpitation. Currently, she is afebrile with a blood pressure of 129/86 mmHg and a pulse of 101 beats per minute. Physical examination is unremarkable. What is the next best step in the management of this patient?
Choices: 1. 2. 3. 4.
Repeat abdominal CT scan in 6 months Pituitary MRI Plasma fractionated metanephrine levels Selective venous sampling of catecholamines
Answer: 3 - Plasma fractionated metanephrine levels Explanations: Although its common to discover an adrenal incidentaloma on an abdominal CT scan, all such adrenal masses must be evaluated for hormone hypersecretion and malignancy before deciding on definitive management. As a rule in endocrinology, laboratory testing precedes imaging. In any patient with an adrenal mass, essential lab test for assessment of hormone hypersecretion including: 24-hour urine free cortisol levels, plasma fractionated metanephrine levels, vanillylmandelic acid, and 17-keto steroids assay, may be performed. If plasma fractionated metanephrine levels come out high in this patient, then further imaging with a PET or octreotide scan can be performed to assess for any extra-adrenal locations of a suspected pheochromocytoma. Functional studies and assessment for a suspected adrenal malignancy must be done when an incidentaloma is identified on imaging before deciding on simple observation and followup. Selective venous sampling for catecholamines is no longer recommended due to the high variability of the results and the risk of accidentally inciting a hypertensive crisis. Go to the next page if you knew the correct answer, or click the link image(s) below to further research the concepts in this question (if desired).
Research Concepts:
Chromaffin Cell Cancer
We update eBooks quarterly and Apps daily based on user feedback. Please tap flag to report any questions that need improvement.
Question 819:
A 45-year-old woman presents to the clinic with a hyperpigmented, leathery, thick plaque across the posterolateral neck. She states the plaque is very itchy and has bothered her for years, but she has never sought treatment. Which of the following conditions is most likely to be present in this patient?
Choices: 1. 2. 3. 4.
Diabetes mellitus Hodgkin lymphoma Dermatomyositis Anemia
Answer: 2 - Hodgkin lymphoma Explanations: Chronic pruritus leads to lichenification and should raise suspicion of a paraneoplastic phenomenon and necessitates thorough screening in cases with a high index of suspicion. Lymphomas, leukemias, and carcinomas have all been documented to cause paraneoplastic pruritis causing chronic skin changes such as lichenification. Diffuse lichenification can be a manifestation of cutaneous T-cell lymphoma and Sezary syndrome, particularly the folliculotropic type. Lichenification points to chronic pruritis and may be a clue to psychiatric, neoplastic, or other systemic disorders. Go to the next page if you knew the correct answer, or click the link image(s) below to further research the concepts in this question (if desired).
Research Concepts: Lichenification
We update eBooks quarterly and Apps daily based on user feedback. Please tap flag to report any questions that need improvement.
Question 820:
A 10-year-old girl suffered from intermittent abdominal pain and fever for 2 weeks. On physical examination, lower abdomen tenderness is noted. The ultrasonography (US) showed a pelvic tumor 8 × 7 cm in size with cysts and ascites over the lower abdominal area. There was no family history of cancer. The clinical diagnosis was pelvic mass, possibly neoplastic in nature. Alphafetoprotein (AFP) was elevated, CA125, Lactate dehydrogenase (LDH), and beta-human chorionic gonadotropin (hCG) were withing the normal ranges. Microscopic examination of the mass tissue will most likely reveal which of the following?
Choices: 1. 2. 3. 4.
Presence of intratubular germ cell neoplasia (IGCN) Prominently associated granuloma formation Presence of three cell types Glomeruloid structures with a central fibrovascular core
Answer: 4 - Glomeruloid structures with a central fibrovascular core
Explanations: Yolk sac tumors (also known as primitive endodermal tumors or endodermal sinus tumors) are malignant primitive germ cell tumors that histologically resemble the mesenchyme of the primitive yolk sac. Yolk sac tumor is a member of germ cell tumors; germ cell tumors usually arise in gonads but sometimes can arise extragonadal. Schiller–Duval body is pathognomonic for yolk sac tumors and resembles a glomeruloid structure with a central fibrovascular core. However, Schiller-Duvall bodies are present only in 50-75% of these tumors. Grossly, yolk sac tumors are poorly demarcated solid masses. They are gray to yellow in color with a gelatinous, or mucoid appearance. Significant necrosis, cystic changes, and hemorrhage are usually present. Microscopically, yolk sac tumors are composed of primitive tumor cells and have many histological patterns. The most common pattern is the reticular microcystic pattern, which is formed by vacuolated cytoplasm of tumor cells; it appears like a honeycomb under a microscope. Go to the next page if you knew the correct answer, or click the link image(s) below to further research the concepts in this question (if desired).
Research Concepts: Yolk Sac Tumors
We update eBooks quarterly and Apps daily based on user feedback. Please tap flag to report any questions that need improvement.
Question 821:
A 20-year-old male of Asian origin presents to the office complaining of recurrent epistaxis and continuous feeling of right-sided nasal obstruction. The patient denies any recent history of upper respiratory tract infection or any bleeding tendency in the family. He reports multiple episodes of otitis media before, all of which were treated with antibiotics. He is a heavy smoker and drinks alcohol occasionally. While interviewing the patient, the clinician notices a hyponasal voice. On physical examination, vital signs include a respiratory rate of 16 breaths per minute, a heart rate of 90 beats per minute, blood pressure of 120/80 mmHg, and oxygen saturation of 99%. The oral and nasal examination is normal, but there is painless cervical lymphadenopathy bilaterally. Nasal endoscopy reveals a mass filling the nasopharynx with a friable surface. Which of the following is associated with the most likely diagnosis?
Choices: 1. 2. 3. 4.
Epstein-Barr virus Human papillomavirus 18 Mutation of the APC gene Hepatitis C virus
Answer: 1 - Epstein-Barr virus Explanations: This patient presented with recurrent epistaxis, the feeling of nasal obstruction, and multiple episodes of otitis media, which should raise the suspension of eustachian tube obstruction. In addition, neck lymphadenopathy with the given history should raise suspicion of nasopharyngeal carcinoma. This tumor is highly endemic to the Chinese, Malay, and Indonesian populations, along with people from southeast Asia. The Epstein-Barr virus (EBV) may be associated with the malignant transformation into nasopharyngeal carcinoma. HPV subtypes 16, 18, 31, 33, as well as others, are associated with oropharyngeal squamous cell carcinoma, rather than nasopharyngeal carcinoma. HCV is not directly associated with nasopharyngeal tumors. Mutations in the APC tumor suppressor gene are found in patients with familial adenomatous polyposis, which has been posited to increase the risk of juvenile nasopharyngeal angiofibroma. Go to the next page if you knew the correct answer, or click the link image(s) below to further research the concepts in this question (if desired).
Research Concepts: Nasopharyngeal Carcinoma (NPC, Lymphoepithelioma)
We update eBooks quarterly and Apps daily based on user feedback. Please tap flag to report any questions that need improvement.
Question 822:
A 35-year-old man presents to the clinic with a soft tissue swelling on his right ankle. The patient states the swelling has been present for the past 2 years and has gradually increased in size. There is no pain, discharge, or pruritus associated with the swelling. However, the patient has difficulty wearing a closed shoe due to the swelling. A biopsy demonstrates polygonal and fusiform cells with clear cytoplasm and a centrally located round nucleus. Periodic acid-Schiff stain reveals intracellular glycogen accumulation. Pigmented and unpigmented melanosomes, scattered multinucleated giant cells, and areas of focal necrosis are also present. Which of the following genetic alteration is responsible for this patient's condition?
Choices: 1. 2. 3. 4.
t(12;22) translocation involving the EWS and ATF1 gene t(9;22) translocation involving the BCR and ABL gene t(9;22) translocation involving the EWS and ATF1 gene t(12;22) translocation involving the BCR and ABL gene
Answer: 1 - t(12;22) translocation involving the EWS and ATF1 gene
Explanations: This patient most likely has clear cell sarcoma. Clear cell sarcoma of soft tissue (CCSST) is an exceedingly rare tumor that originates from neural crest cells and is histologically characterized by clear cells representing intracellular glycogen accumulation. CCSST typically harbors the t(12;22) translocation involving the EWS and ATF1 gene. Clear cell sarcoma was initially described as “malignant melanoma of soft parts” due to their histological similarities. Both share immunohistochemical markers, and their gene expression profiling suggests that CCSST is a genomic subtype of MM. Unlike MM, however, most clear cell sarcomas are associated with a t(12;22)(q13-14;q12) translocation. Go to the next page if you knew the correct answer, or click the link image(s) below to further research the concepts in this question (if desired).
Research Concepts: Soft Tissue Clear Cell Sarcoma
We update eBooks quarterly and Apps daily based on user feedback. Please tap flag to report any questions that need improvement.
Question 823:
A 65-year-old woman presents with a 4week history of hoarseness and dysphagia. She has smoked fifteen cigarettes a day for 40 years and 2-4 units of alcohol per week. She takes losartan for high blood pressure and bisoprolol for atrial fibrillation. She is referred to an otolaryngologist who performed a flexible laryngoscopy. It revealed a small irregular lesion on the true right cord extending to the subglottis, with impaired vocal cord movement. Which of the following is the next best step in management?
Choices: 1. Computed tomography scan of the head and thorax 2. Computed tomography neck and chest with direct laryngoscopy and biopsy 3. Positron emission tomography-computed tomography scan 4. A magnetic resonance imaging of the neck
Answer: 2 - Computed tomography neck and chest with direct laryngoscopy and biopsy
Explanations: Computed tomography scans of the head and thorax are commonly used to accurately assess local invasion, nodal disease, and distant metastasis. However, computed tomography scans are unlikely to be of any diagnostic benefit in early T1 cancers that do not involve the anterior commissure, so it should be used in conjunction with direct laryngoscopy and biopsy under general anesthetic for definitive diagnosis. T2a/b cancers are more advanced, and usually, the best response is seen with radiotherapy of higher doses per fraction and concurrent chemotherapy such as cisplatin. Compared to transoral laser microsurgery, radiotherapy is said to have superior voice-related outcomes, although limited by short term radiation toxicity sideeffects. Prior to the 90s total laryngectomy used to be the treatment of choice for more advanced glottic cancers however now the preferred choice if possible is organ preservation therapy. Go to the next page if you knew the correct answer, or click the link image(s) below to further research the concepts in this question (if desired).
Research Concepts: Glottic Cancer
We update eBooks quarterly and Apps daily based on user feedback. Please tap flag to report any questions that need improvement.
Question 824:
A 62-year-old male is scheduled to undergo a bone marrow aspiration and biopsy for suspected multiple myeloma. He takes atorvastatin, clopidogrel, and aspirin and has a past medical history of coronary artery disease. In order to best access the most commonly chosen sampling site, how should the patient be positioned on the table?
Choices: 1. 2. 3. 4.
Lateral decubitus Supine Seated, leaning forward Bone marrow aspiration and biopsy are contraindicated
Answer: 1 - Lateral decubitus Explanations: The preferred site of sampling for a bone marrow aspiration and biopsy is the posterior superior iliac crest due to distance from vascular and nervous structures as well as for patient comfort. The second most commonly used location is the anterior superior iliac crest. The prone and lateral decubitus positions are the most ideal for accessing the posterior superior iliac crest. Patient positioning during bone marrow aspiration and biopsy is key for easy access to the sampling site as well as maximizing patient and operator comfort. The use of antiplatelet agents is not a contraindication to performing a bone marrow aspiration or biopsy. Go to the next page if you knew the correct answer, or click the link image(s) below to further research the concepts in this question (if desired).
Research Concepts: Bone Marrow Aspiration And Biopsy
We update eBooks quarterly and Apps daily based on user feedback. Please tap flag to report any questions that need improvement.
Question 825:
A postmenopausal 68-year-old gravida 2, para 2 female presented to the emergency department with the passage of clots, heavy vaginal bleeding, and presyncope. Her blood pressure was 87/56 mmHg, her heart rate was 120 beats per minute, her respiration rate was 18 breaths per minute, and her temperature was 98 F (36.6 C). Her hemoglobin was noted to be 7.6 g/dl. She received 1 unit of packed red blood cells with symptomatic improvement. Further history reveals she underwent menarche at the age of 11 and menopause at the age of 60. She is obese, but otherwise her past medical, surgical, family, and social history were noncontributory. Pelvic examination was concerning for blood coming from the cervical os without any other masses or abnormalities. Further evaluation with pelvic ultrasound demonstrated a 7 mm heterogenous endometrial stripe. Follow-up with endometrial biopsy was concerning for high-grade endometrial carcinoma, and she underwent a total hysterectomy, bilateral salpingo-oophorectomy, omentectomy, and aortic and pelvic lymph node dissection. The final pathology report was significant for papillary serous endometrial carcinoma. The patient asks about the potential route by which her cancer can metastasize. What is the appropriate response to her inquiry?
Choices: 1. Papillary serous carcinomas are low-grade and do not typically metastasize 2. Hematogenously 3. Transit through the fallopian lumens 4. Direct bone marrow metastasis
Answer: 3 - Transit through the fallopian lumens Explanations: Papillary serous carcinomas are high grade and commonly metastasize via the lymphatic system and through the fallopian lumens to disseminate throughout the pelvis and abdomen. Hematogenous spread is most commonly associated with choriocarcinoma, not papillary serous carcinoma. Direct tissue spread is more common with cervical cancer. Low-risk, type 1 endometrioid, carcinomas tend to remain confined to the uterus and therefore have a favorable prognosis. Go to the next page if you knew the correct answer, or click the link image(s) below to further research the concepts in this question (if desired).
Research Concepts: Endometrial Cancer
We update eBooks quarterly and Apps daily based on user feedback. Please tap flag to report any questions that need improvement.
Question 826:
A 25-year-old Caucasian female presents with a mole that is rapidly growing. She noticed that the mole on her neck changed color from black to pink and was growing fast for the past 2 months. She works as a farmer and does not apply sunscreen. The physical exam shows a pink nevus which was previously black, asymmetrical, with irregular borders, and with a diameter change of 2 mm to 7 mm. Which of the following procedures is the best method to diagnose her condition?
Choices: 1. 2. 3. 4.
Excision biopsy Incision biopsy Punch biopsy Shave biopsy
Answer: 1 - Excision biopsy Explanations: Melanoma is a malignant tumor produced from the malignant transformation of melanocytes. This can be sporadic or arise from a preexisting premalignant lesion. It has the highest incidence in areas with populations with fair skin and in locations with lots of sun exposure. Melanoma is the most common cancer in Caucasian women age 25 to 29 and the second most common cancer of women age 30 to 34. In melanoma of the head and neck, a thorough examination is warranted as several important considerations are different from management elsewhere. Head and neck melanoma have a worse prognosis than other sites; the location of the lesion has a significant impact on prognosis. Scalp location has the worst prognosis, followed by ear, cheek, and neck, all in respected order. Excisional biopsy is the most acceptable method with small lesions and used with 1mm to 2mm margins. If pathological examination results in malignant melanoma, then wide excision is necessary. Incisional or punch biopsy allows the examination of the depth of the tumor and it does not disrupt lymphatic drainage or its borders. A shave biopsy should never be performed on a pigmented lesion that is suspicious for melanoma as the depth is necessary for the evaluation. Go to the next page if you knew the correct answer, or click the link image(s) below to further research the concepts in this question (if desired).
Research Concepts:
Melanoma Of The Head And Neck
We update eBooks quarterly and Apps daily based on user feedback. Please tap flag to report any questions that need improvement.
Question 827:
A 31-year-old male athlete presents to the hospital with palpitations and exercise-induced dyspnea for three months. There is no significant medical history. A physical examination and chest X-ray are both normal. An electrocardiogram (ECG) shows deep, inverted T waves. Transthoracic echocardiography (TTE) shows a homogeneous intracardial mass of 40 × 50 mm on the posterolateral wall of the left ventricle. There are no significant lab findings. After further investigations with MRI and ventriculography, a benign heart tumor interfering with the left ventricular filling is suspected. Surgery is planned for resection of the mass. Which of the following best describes the most likely expected findings on biopsy?
Choices: 1. Complex structures resembling cords, nests, rings, or poorly formed glands with abundant mucopolysaccharide (myxoid) ground substance 2. Disordered, hypertrophied muscle fibers containing myocytes with large nuclei, surrounded by fibrous and adipose tissue 3. Solid and cystic areas lined by nonciliated, epithelialappearing cuboidal cells (main cells) mixed with occasional clear cells (neuroendocrine or C cells) 4. Clear cells and large, rounded, polygonal cells ("spider cells") with glycogen vacuoles separated by strands of cytoplasm extending between the cell membrane and nucleus
Answer: 2 - Disordered, hypertrophied muscle fibers containing myocytes with large nuclei, surrounded by fibrous and adipose tissue
Explanations: Option 2 describes the benign hamartomatous growth of mature cardiac myocytes. It is a rare non-neoplastic tumor and tumor-like lesion of the heart that can mimic various cardiac lesions and tumors. Histopathological examination is the only way to a definitive diagnosis. Complete surgical excision with clear margins is the treatment of choice, with excellent prognosis and minimal recurrence rates. Option 1 describes atrial myxoma. Although it can be present in any of the heart chambers, in 75% of cases, it is present in the left atrium. Option 3 describes a cystic tumor. Cystic tumor of the AV node is the most common primary cardiac tumor causing sudden cardiac death. Given the lack of microscopic clues to its presence, it is the wrong choice. Option 4 describes a cardiac rhabdomyoma. Cardiac rhabdomyomas are the most common primary tumor of the heart in infants and children. It has an association with tuberous sclerosis. Given the patient history and symptoms, cardiac hamartoma is most likely the diagnosis. Go to the next page if you knew the correct answer, or click the link image(s) below to further research the concepts in this question (if desired).
Research Concepts:
Hamartoma
We update eBooks quarterly and Apps daily based on user feedback. Please tap flag to report any questions that need improvement.
Question 828:
A 67-year-old man with a past medical history of long-standing diabetes mellitus and prostatic adenocarcinoma presents with constant and sharp back pain. He also undergoes hemodialysis twice a week. His blood pressure is 170/100 mmHg, and his body mass index (BMI) is 48 kg/m^2. His serum creatinine is 6.0 mg/dL. What is the mechanism of action of the analgesic drug that is best for this patient?
Choices: 1. Agonist at mu and kappa opiate receptors 2. Inhibition of prostaglandin synthesis by competitive blocking of the enzyme cyclo-oxygenase 3. Antagonist at mu and kappa-opiate receptors 4. Non-selective and irreversible inhibitor of COX-1 and COX2
Answer: 1 - Agonist at mu and kappa opiate receptors Explanations: Fentanyl is used in severe cases of pain in patients with renal failure owing to its primarily hepatic elimination. Fentanyl is an opiate drug that acts as an agonist at mu and kappa-opiate receptors. It is used as analgesia and in combination with other medications in anesthesia. Fentanyl patches are an excellent option for pain relief in patients with poor kidney function who need longterm pain management. This patient requires palliative care and comfort. Nonsteroidal anti-inflammatory drugs (NSAIDs) would not be appropriate for patients with poor kidney function. Go to the next page if you knew the correct answer, or click the link image(s) below to further research the concepts in this question (if desired).
Research Concepts: Fentanyl
We update eBooks quarterly and Apps daily based on user feedback. Please tap flag to report any questions that need improvement.
Question 829:
A 65-year-old male presents for the evaluation of cough and unexplained weight loss. He also reports a small amount of hemoptysis for the last few weeks. He has a 40-pack-year history of smoking. A chest Xray shows a 2 cm spiculated mass in the right para hilar region. On further evaluation, which of the following is most likely to be present?
Choices: 1. 2. 3. 4.
Cushing syndrome Symptomatic liver metastases Symptomatic bone metastases Limbic encephalitis
Answer: 3 - Symptomatic bone metastases Explanations: Lung cancer or bronchogenic carcinoma refers to tumors originating in the lung parenchyma or within the bronchi. Smoking is the most common cause of lung cancer. Cough, dyspnea, and hemoptysis are the most common presenting symptoms. Metastasis from lung cancer to bone is frequently symptomatic, and patients may present with bone pain at the site of metastasis in the setting of elevated serum alkaline phosphatase and hypercalcemia. Up to 20% of the patients with non-small cell lung cancer may have bone pain secondary to metastasis on initial presentation; whereas the percentage is as high as 30% to 40% in patients with small-cell lung cancer. Imaging usually reveals osteolytic lesions with vertebral bodies as the most common site of metastasis. Adrenal metastases also occur in lung cancer, but they are rarely symptomatic. Not all adrenal lesions are malignant lesions, and positron emission tomography (PET) scanning is recommended to differentiate benign from malignant adrenal lesions in these patients. Brain metastasis is another common feature of lung cancer in small cell lung cancers (SCLC) and non-small cell lung cancers (NSCLC). In SCLC, brain metastases may be present in as high as 20% to 30% of the patients at the time of diagnosis. Other common sites of metastases in lung cancer include the liver, which is usually only symptomatic in advanced disease. Go to the next page if you knew the correct answer, or click the link image(s) below to further research the concepts in
this question (if desired).
Research Concepts: Lung Cancer
We update eBooks quarterly and Apps daily based on user feedback. Please tap flag to report any questions that need improvement.
Question 830: A 59-year-old man with a history of diabetes, hypertension, and multiple myeloma presents to the clinic for chest pain. The patient takes metformin, lisinopril, and ixazomib. Which of the following lab values is most appropriate to be monitored in this patient periodically?
Choices: 1. 2. 3. 4.
Brain natriuretic peptide (BNP) Erythrocyte sedimentation rate Glucose Liver function tests
Answer: 4 - Liver function tests Explanations: Ixazomib is FDA approved to treat multiple myeloma. Ixazomib is FDA approved to only be used when at least one prior treatment has been tried/given. Ixazomib is an oral proteasome inhibitor, which causes the accumulation of paraproteins, which ultimately leads to apoptosis. Patients on Ixazomib should have liver function tests done while on treatment due to the risk of hepatotoxicity. Go to the next page if you knew the correct answer, or click the link image(s) below to further research the concepts in this question (if desired).
Research Concepts: Ixazomib
We update eBooks quarterly and Apps daily based on user feedback. Please tap flag to report any questions that need improvement.
Question 831:
A 68-year-old female presents with complaints of sudden, severe pain in her right lower quadrant. It is accompanied by nausea and vomiting. She states the pain is 10/10 and extends towards the pubic region. She is currently receiving chemotherapy for breast cancer. She has no prior history of any similar events. Vital signs are unremarkable. Urinalysis shows large microscopic blood and is otherwise normal. Which of the following is the most likely underlying chemical cause of her disorder?
Choices: 1. 2. 3. 4.
Aciduria Hypercalciuria Hyperoxaluria Hyperuricosuria
Answer: 4 - Hyperuricosuria Explanations: Chemotherapy for malignancies can often increase uric acid levels in the blood and urine, ultimately resulting in a urinary uric acid stone. During chemotherapy, an increased degree of malignant cell destruction will occur. This release of cellular nuclear material including purine nucleotides ultimately results in increased uric acid production. When this process occurs in excess, it can cause tumor lysis syndrome. While aciduria is the most common cause of uric acid urolithiasis, in this case, it is most likely caused by hyperuricosuria. Go to the next page if you knew the correct answer, or click the link image(s) below to further research the concepts in this question (if desired).
Research Concepts: Hyperuricosuria
We update eBooks quarterly and Apps daily based on user feedback. Please tap flag to report any questions that need improvement.
Question 832:
A 49-year-old man presents with complaints of fatigue, headache, generalized body aches, and low-grade fevers. The patient has noticed neck swelling while he was taking a shower a few days ago. He denies chest pain, abdominal pain, or pedal edema. There is no history of travel or contact with ill patients. There is no significant past medical history. On physical examination, he has two posterior cervical lymph nodes, the largest being 2.5 cm x 2.5 cm in size. They are immobile, attached to deep structures, and have a firm consistency. The rest of his physical examination is unremarkable. His blood pressure is 100/70 mmHg, the pulse is 100/min, and the temperature is 38 C (100.4 F). The chest x-ray is clear. The laboratory tests show hemoglobin of 12 g/dl and leukocyte count 9000/mm3. Which of the following is most appropriate to make a diagnosis of this condition?
Choices: 1. 2. 3. 4.
Lymph node biopsy Blood culture and film Bone marrow biopsy Fine needle aspiration of lymph nodes
Answer: 1 - Lymph node biopsy Explanations: Non-Hodgkin lymphoma has symptoms like fatigue, lowgrade fever, night sweats, and weight loss. Painless peripheral lymphadenopathy is present in up to two-third cases of non-Hodgkin lymphoma. The most commonly involved lymph nodes are Waldeyer’s ring (base of the tongue, tonsils, nasopharynx), cervical, supraclavicular, axillary, inguinal, and femoral. Lymph node biopsy confirms the diagnosis of nonHodgkin lymphoma. Fine needle aspiration does not provide an accurate diagnosis. It is used for screening purposes. Go to the next page if you knew the correct answer, or click the link image(s) below to further research the concepts in this question (if desired).
Research Concepts: Non-Hodgkin Lymphoma
We update eBooks quarterly and Apps daily based on user feedback. Please tap flag to report any questions that need improvement.
Question 833:
A 55-year-old man presents to the office for a health maintenance visit. He has not had any problems and is feeling healthy. He follows a balanced diet and exercises regularly. He takes daily multivitamins. His father died due to heart disease at age 59. He does not have a family history of cancer. He has never smoked and drinks alcohol occasionally. Vital signs are within normal limits. The patient is referred for screening colonoscopy, which reveals a single 7-mm polyp in the sigmoid colon. It is excised and sent for pathologic evaluation. Which of the following histological findings requires the most intensive follow-up colonoscopic surveillance?
Choices: 1. 2. 3. 4.
Hyperplastic polyp Submucosal lymphoid polyp Tubular adenoma Villous adenoma
Answer: 4 - Villous adenoma Explanations: Colonic adenomas are raised protrusion of colonic mucosa i.e. polyps formed by glandular tissue. Although a tumor of benign nature, these adenomas are usually considered pre-cancerous and can develop malignant structures, in contrast to hyperplastic polyps with no malignant potential. Adenomatous polyps carry increased malignant potential and warrant enhanced colonoscopic surveillance. Large polyps (1 cm) and adenomas with high-grade dysplasia or villous features carry an even more significant malignant potential. Hyperplastic polyps have no malignant potential. A lymphoid polyp (lymphoid hyperplasia, benign lymphoma) is a benign, focal, or diffuse condition that occurs typically where clusters of lymphoid follicles are present (terminal ileum, rectum). Tubular adenomas are less likely to become malignant. Go to the next page if you knew the correct answer, or click the link image(s) below to further research the concepts in this question (if desired).
Research Concepts: Tubular Adenoma
We update eBooks quarterly and Apps daily based on user feedback. Please tap flag to report any questions that need improvement.
Question 834:
A 48-year-old man with no significant past medical history presents to the clinic for a recently found high total protein at 9 g/dl. Subsequently, serum protein electrophoresis shows IgA paraprotein at 3.1 g/dL. She denies any symptoms, including bone pain, weight loss, or fevers. Family history is unremarkable. He has no past surgical history. Vital signs show temperature 36.5 C, blood pressure 130/80 mmHg, and pulse 85/min. The physical exam is unremarkable. Laboratory results show no abnormalities. The skeletal survey is normal as well. Bone marrow biopsy shows 25% of clonal plasma cells. Which of the following is the next best step in the management of this patient?
Choices: 1. Induction therapy in preparation for autologous stem cell transplant 2. Observation every 6 months 3. Enrollment in a clinical trial 4. Whole-body MRI
Answer: 4 - Whole-body MRI Explanations: Smoldering myeloma (SMM) is characterized by serum M-protein 3 g/L and/or 10% monoclonal plasma cells in the bone marrow with the absence of myeloma-related events, end-organ damage, or amyloidosis. In this case, the patient has a negative skeletal survey and needs to undergo imaging tests such as whole-body MRI, FDG PET/CT, or low dose CT as a part of the initial diagnostic workup to assess for bone disease. SMM is a premalignant condition, and the progression rate is estimated at around 10% in one year. Therefore, patients who receive an SMM diagnosis need frequent follow-up. Based on the current guidelines, observation with repeat tests (basic laboratory, SPEP, serum immunofixation, UPEP, urine immunofixation, bone marrow aspirate and biopsy, advanced imaging) should be done at every 3-6 months. For high-risk SMM patients, clinical trials showed improved overall survival and progression-free survival with lenalidomide with or without dexamethasone. Before considering clinical trial enrollment, patients should undergo imaging tests to assess for bone diseases. Especially for high-risk SMM patients, clinical trial enrollment is highly encouraged. Go to the next page if you knew the correct answer, or click the link image(s) below to further research the concepts in this question (if desired).
Research Concepts:
Plasma Cell Cancer
We update eBooks quarterly and Apps daily based on user feedback. Please tap flag to report any questions that need improvement.
Question 835:
A 69-year old man was just diagnosed with locally advanced prostate cancer. The family is asking about germline testing as there is prostate cancer in the family. According to the NCCN guidelines, which of the following would be the strongest indicator of the need for germline testing?
Choices: 1. 2. 3. 4.
FIrst degree relative with breast cancer before age 60 Father or brother with prostate cancer at any age Sephardic Jewish ancestry Metastatic prostate cancer at any age
Answer: 4 - Metastatic prostate cancer at any age Explanations: The NCCN guidelines recommend germline testing for all patients with metastatic disease or where there is a father or brother with prostate cancer before age 60, a first degree relative with breast cancer before age 50 and for patients with Ashkenazi Jewish descent. Germline testing is used for treatment optimization but is most helpful in family counseling to modify screeinings for family members who may be at higher risk. Specific germline mutations recommended by the NCCN for prostate cancer include ATM, BRCA1, BRCA2, CHEK2, PALB2, PMS2, MLH1, MSH2, and MSH6. We also suggest adding the following if possible: EpCAM, HOXB13 (especially in African Americans), FANCA, P53, and NBN Go to the next page if you knew the correct answer, or click the link image(s) below to further research the concepts in this question (if desired).
Research Concepts: Prostate Cancer
We update eBooks quarterly and Apps daily based on user feedback. Please tap flag to report any questions that need improvement.
Question 836:
An 80-year-old female presents with a growing papule on her nasal tip, which has been present for at least two years. The lesion was initially asymptomatic, but within the past three months has become sore and occasionally bleeds. Physical examination reveals a pink, pearly papule with overlying telangiectasias and rolled borders. A biopsy is performed, and the histopathologic examination reveals basaloid islands with peripheral palisading. What treatment has the highest cure rate for this lesion?
Choices: 1. 2. 3. 4.
Electrodesiccation and curettage Cryotherapy Mohs micrographic surgery Surgical excision
Answer: 3 - Mohs micrographic surgery Explanations: The highest cure rate for basal cell carcinoma is seen with Mohs micrographic surgery. It also preserves the most normal tissue. The five-year recurrence rate is 1% to 2%. Recurrence rates for other treatments are as follows: standard excision (10.1%), radiation (8.7%), electrodesiccation and curettage (7.7%), and cryotherapy (7.5%). Go to the next page if you knew the correct answer, or click the link image(s) below to further research the concepts in this question (if desired).
Research Concepts: Basal Cell Cancer
We update eBooks quarterly and Apps daily based on user feedback. Please tap flag to report any questions that need improvement.
Question 837:
A 55-year-old man is diagnosed with adult T-cell leukemia and scheduled to receive multi-agent chemotherapy, including vincristine, cyclophosphamide, doxorubicin, etoposide, carboplatin, and prednisone. His past medical history is notable for a 10-pack-year history of smoking. Which of the following is the most appropriate for pretreatment evaluation and monitoring of this patient's condition during chemotherapy?
Choices: 1. 2. 3. 4.
Coronary angiography Pulmonary function testing Multigated acquisition (MUGA) scan Ambulatory 24-hour EKG monitoring
Answer: 3 - Multigated acquisition (MUGA) scan Explanations: The most commonly used regimen for adult T cell leukemia is VCAP-AMP-VECP (also known as LSG15), which includes treatment with vincristine, cyclophosphamide, doxorubicin, prednisone, ranimustine, vindesine, etoposide, and carboplatin. Anthracycline (e.g., doxorubicin, daunorubicin) administration is often accompanied by adverse drug reactions that limit their use. Anthracycline-mediated cardiotoxicity is dose-dependent and cumulative; the risk is increased in patients with preexisting cardiac diseases and with low cardiac ejection fraction. Patients receiving anthracyclines should be evaluated with a multigated acquisition (MUGA) scan or echocardiography to measure ejection fraction (EF). MUGA scan is highly accurate for quantifying EF. It is performed before starting chemotherapy and each subsequent dose of chemotherapy. Anthracycline chemotherapy is contraindicated in patients with an EF of less than 30%, modified dosing is required in patients with an EF of less than 50%, and a decrease in EF of 10% or more requires discontinuation of the drug. Go to the next page if you knew the correct answer, or click the link image(s) below to further research the concepts in this question (if desired).
Research Concepts: Adult T Cell Leukemia
We update eBooks quarterly and Apps daily based on user feedback. Please tap flag to report any questions that need improvement.
Question 838:
A 70-year-old man presents to the clinic with a two-month history of weight loss, fever, night sweats, and fatigue. Physical examination reveals generalized, painless lymphadenopathy and splenomegaly. Complete blood count shows lymphocytosis, low red blood cell count, and a low platelet count. Which of the following sets of immunophenotypes is most likely to be seen on flow cytometry in this patient?
Choices: 1. 2. 3. 4.
CD5, CD200, FMC7 CD5, CD19, FMC7 CD10, CD19, CD200 CD10, CD19, FMC7
Answer: 1 - CD5, CD200, FMC7 Explanations: Lymphadenopathy suggests a lymphoid malignancy. A patient more than 50 years of age with lymphocytosis suggests chronic lymphocytic leukemia/small lymphocytic lymphoma (CLL/SLL). Anemia, thrombocytopenia, and splenomegaly suggest a possible leukemia. Flow cytometry will demonstrate a monoclonal CD5 positive B-cell population that is positive for CD200, which is reliably negative in mantle cell lymphoma. FMC7 is positive in mantle cell lymphoma and negative in CLL/SLL. Other helpful clues to CLL/SLL are dim CD20 expression and dim light chain expression. Go to the next page if you knew the correct answer, or click the link image(s) below to further research the concepts in this question (if desired).
Research Concepts: Immunophenotyping
We update eBooks quarterly and Apps daily based on user feedback. Please tap flag to report any questions that need improvement.
Question 839:
A 17-year-old male presents to the clinic with a history of a solid mass in his testicle. He discovered it 3 weeks ago and denies any history of trauma. He denies any other symptoms and the mass is not painful. He is otherwise healthy and takes no medications. His blood pressure is 122/80 mmHg, pulse 80/min, respirations 16/min, and temperature 98.0 F (36.7 C). On examination, the patient has a hard, nodular mass that is non-tender. The biopsy confirms the suspected diagnosis. Blood tests indicated increased levels of placental alkaline phosphatase. Microscopic examination shows a sheet-like pattern of cells surrounded by a fibrous stromal network, which stains PASpositive. Which of the following is most closely associated with an increased risk for the patient’s condition?
Choices: 1. 2. 3. 4.
HIV infection Cryptorchidism Maternal exposure to DES during pregnancy Klinefelter syndrome
Answer: 2 - Cryptorchidism Explanations: Testicular cancers are classified based on their cell of origin: seminomatous, non-seminomatous, Leydig, Sertoli, choriocarcinoma, embryonal, teratoma, and yolk-sac derivatives. Seminoma and non-seminomatous lesions are frequently grouped as germ cell tumors and are notable for their responsiveness to chemotherapy in comparison to the other types. Seminoma accounts for about a third of all testicular germ cell malignancies and is one of the most treatable cancers with a survival rate of 98% to 99% in early-stage disease. While the overall incidence of testicular germ cell tumors is low, at only 1% to 2% of all male malignancies, it remains the most common cancer in the 15 to 35 year age group. Testicular seminomas are associated with placental alkaline phosphatase, detectable in the serum. Microscopic examination of a testicular seminoma shows a sheet-like pattern of cells surrounded by a fibrous stromal network, which stains PAS-positive due to glycogen content. HIV infection may increase the risk of seminoma slightly, but not nearly as much as a history of cryptorchidism. The remaining answers have no known significant effect on the incidence of germ cell testicular tumors. The risk of developing a testicular seminoma is substantially increased in individuals with a history of cryptorchidism. The risk is even greater if the undescended testis was in the abdomen versus the inguinal region. The subsequent development of cancer is not affected by surgical repair of cryptorchidism, but it does allow for easier inspection.
On physical exam, there is usually a unilateral, firm to hard palpable mass in the scrotum, which is localized to the testis. This may be associated with a hydrocele that would preclude palpation, so testicular ultrasonography may be needed to identify the mass. Metastatic disease, although rare, can present initially with lymphadenopathy in the retroperitoneum and the anterior mediastinum. Even massive metastatic disease can be produced from a non-palpable testicular malignancy. Abdominal CT scans are used to identify metastatic disease in the retroperitoneal lymph nodes. Go to the next page if you knew the correct answer, or click the link image(s) below to further research the concepts in this question (if desired).
Research Concepts: Testicular Seminoma
We update eBooks quarterly and Apps daily based on user feedback. Please tap flag to report any questions that need improvement.
Question 840:
A 65-year-old man is brought to the hospital with an episode of confusion. He was diagnosed with colorectal cancer recently. He previously had radical resection of the tumor followed by chemotherapy with 5fluorouracil and leucovorin. His latest imaging revealed the systemic spread of cancer. An adjuvant platinum-based antineoplastic agent was added to his chemotherapy regimen to work synergistically with 5-fluorouracil to control the spread of cancer. On the fifth day after his first dose of chemotherapy, he developed headaches and blurry vision. On the seventh day, he was found on the floor. He does not remember anything before or during the fall. What is the most appropriate management strategy for this patient?
Choices: 1. Continue the chemotherapeutic regimen and administer levetiracetam 2. Decrease the dosage of the new agent and administer levetiracetam 3. Decrease the dosage of the new agent, order EEG, administer levetiracetam, and perform serial neurological examinations 4. Discontinue the new agent, order MRI, administer levetiracetam, and perform serial neurological examinations
Answer: 4 - Discontinue the new agent, order MRI, administer levetiracetam, and perform serial neurological examinations
Explanations: This patient has posterior reversible encephalopathy syndrome (PRES), which requires immediate cessation of oxaliplatin. The pathophysiology of PRES is unclear. However, two main hypotheses have been proposed: 1) failure of cerebrovascular autoregulation leading to vasogenic edema and 2) endothelial and blood-brain barrier dysfunction leading to extravasation of fluid and protein. PRES is diagnosed based on clinical and radiological findings. Clinical findings of PRES are headaches, seizures, visual disturbances (blurriness, homonymous hemianopsia, cortical blindness), altered consciousness, and focal neurological deficits. CT and MRI of the brain will show bihemispheric subcortical white matter lesions and focal vasogenic edema in a watershed distribution predominantly in the parieto-occipital region with some involvement of the frontotemporal lobe. T2-weighted MRI will show hyperintense lesions, indicating vasogenic edema. The major clinical conditions associated with PRES are hypertension, preeclampsia, eclampsia, infections, sepsis, autoimmune diseases, cancer chemotherapy, and organ transplantation. Early diagnosis and prompt treatment of PRES can pave the way for early recovery. The treatment of PRES is symptomatic and focused on targeting the underlying cause. Periodic neurological examinations and
neuroimaging must be done to assess the progress of recovery. Go to the next page if you knew the correct answer, or click the link image(s) below to further research the concepts in this question (if desired).
Research Concepts: Oxaliplatin
We update eBooks quarterly and Apps daily based on user feedback. Please tap flag to report any questions that need improvement.
Question 841:
A 45-year-old woman presents for a follow-up. She had a right-sided mastectomy for breast cancer ten weeks ago. She reports a gradual onset of swelling and a sense of heaviness in her right arm. Which of the following is the most likely cause of this patient's new symptoms?
Choices: 1. 2. 3. 4.
Venous congestion Interruption of the lymphatic drainage Damage to the arterial supply Blockage of the lymphatic drainage
Answer: 2 - Interruption of the lymphatic drainage Explanations: Many patients with breast cancer may undergo an axillary lymph node dissection (ALND). This is associated with complications such as lymphedema. Lymphedema presents with swelling, pain, and heaviness of the affected region. It typically develops over several weeks, as it takes time for the lymph to accumulate. The lymph from the arm drains to the axillary nodes, which can also be affected in breast cancer. In patients that have had an ALND performed, lymphedema occurs due to the removal of lymph vessels and interruption of lymph drainage. Go to the next page if you knew the correct answer, or click the link image(s) below to further research the concepts in this question (if desired).
Research Concepts: Anatomy, Shoulder and Upper Limb, Axillary Lymph Nodes
We update eBooks quarterly and Apps daily based on user feedback. Please tap flag to report any questions that need improvement.
Question 842:
A 59-year-old woman undergoes an elective splenectomy for evaluation of her anemia and moderate splenomegaly. The gross pathologic examination shows the spleen measuring 12.5 x 11 x 6 cm. The cut surface showed two sponge-like vascular spaces measuring 3 x 2.5 x 1.0 cm and 2 x 3.5 x 2 cm close to the hilum. Microscopic examination reveals that the tumors have clear boundaries without any fibrous capsule, and anastomosing vascular channels as seen in splenic sinusoids. No nuclear atypia or mitotic activity is noted. Immunophenotyping of the cells lining the vascular channels is strongly positive for CD34 but negative for CD31; some lining cells are weakly positive for factor VIII; CD8 and lysozyme are also noted. Which of the following best describes the pathogenesis of the patient's most likely diagnosis?
Choices: 1. Dual endothelial and histiocytic differentiation 2. B-cell lymphocyte origin with a possible role of EpsteinBarr virus (EBV) 3. T cell lymphocyte origin with a possible role of human T cell Lymphocyte virus (HTLV) 4. Unknown mechanism
Answer: 1 - Dual endothelial and histiocytic differentiation
Explanations: Littoral cell splenic angioma (LCA) is a distinct entity from other splenic vascular lesions, eg. angiosarcoma, as suggested by the immunohistochemical staining for littoral cells revealing a dual differentiation pattern. The cells stain positive for both endothelial markers, e.g., factor VIII, CD31, von Willebrand factor, CD34, as well as histiocytic markers, e.g., CD 68. Littoral cell angioma is a primary vascular tumor arising from the cells lining the splenic red pulp venous sinuses, hence the name littoral cells. LCA does not originate from B or T cells. It is correct that the etiology of littoral cell angioma remains largely unknown. However, there seems to be a role of immune dysregulation. The above mentioned immunophenotyping does not highlight this aspect. Go to the next page if you knew the correct answer, or click the link image(s) below to further research the concepts in this question (if desired).
Research Concepts: Littoral Cell Splenic Angioma
We update eBooks quarterly and Apps daily based on user feedback. Please tap flag to report any questions that
need improvement.
Question 843:
A 25-year-old male patient presents to the clinic with complaints of aberrant lesions present on different parts of his body but mainly on his tongue and lips. He also complains of headaches, double vision, and tremors. Physical examination reveals an enlarged head and diffuse, painless, moveable swellings on his upper back and shoulder regions. MRI scan shows diffuse hypertrophy of the granular layer of the cerebellum. Which gene is most likely to be mutated in this disease?
Choices: 1. 2. 3. 4.
p53 Phosphatase and tensin homolog gene ABCC6 BRAF
Answer: 2 - Phosphatase and tensin homolog gene Explanations: The presence of hamartomas along with signs and symptoms and MRI findings consistent with Lhermitte– Duclos disease are suggestive of Cowden disease. In addition, the patient has macrocephaly which is also one of the major criteria for the diagnosis of Cowden disease. The protein encoded by the phosphatase and tensin (PTEN) homolog gene contributes to the control of apoptosis and the cell cycle. Specifically, the phosphatidylinositol 3-kinase (PI3K)/AKT/mammalian target of rapamycin (mTOR) pathway is down-regulated by the PTEN gene product resulting in decreased cellular proliferation and survival. Autosomally dominant inherited mutations in the tumor suppressor gene phosphatase, and tensin homolog gene (PTEN) are responsible for Cowden disease. Some studies have shown a female predominance in Cowden syndrome, and most patients reported in the literature are white. An autosomal dominant disease with mutations in the SDHB/C/D genes can present with similar findings as Cowden syndrome such as thyroid, breast, and renal cancers. Go to the next page if you knew the correct answer, or click the link image(s) below to further research the concepts in this question (if desired).
Research Concepts:
Cowden Disease
We update eBooks quarterly and Apps daily based on user feedback. Please tap flag to report any questions that need improvement.
Question 844:
A 58-year-old man presents for a routine follow-up at the clinic. He has a past medical history of Stage 1 hepatocellular carcinoma (HCC) for which he underwent liver transplantation 15 months ago. He does not have any complaints at this time but does report worrying about a recurrence of his liver cancer. His other medical problems include gastroesophageal reflux disease and hyperlipidemia. The patient does not use tobacco or alcohol. He currently lives with his wife and is retired. His current medications include prednisone, cyclosporine, mycophenolate, acyclovir, sulfamethoxazole-trimethoprim, atorvastatin, and omeprazole. He underwent a screening colonoscopy at age 54, which was unremarkable. He does not report any drug allergies. His physical examination is unremarkable. A thorough oral and skin examination does not reveal any abnormalities. Cardiovascular and abdominal examination is within normal limits. Which of the following is recommended as an essential part of post-transplantation surveillance in this patient?
Choices: 1. Abdominal ultrasound and serum alpha-fetoprotein (AFP) every 6 months 2. Annual colonoscopy 3. Annual brain MRI 4. Upper endoscopy every 6 months
Answer: 1 - Abdominal ultrasound and serum alphafetoprotein (AFP) every 6 months
Explanations: Both skin and non-skin malignancies are more common in liver transplant patients than in the general population. Physicians must maintain a low clinical threshold for the evaluation of suspicious symptoms. Guidelines recommend close surveillance and screening for hepatocellular carcinoma (HCC) in liver-transplant patients with chronic hepatitis B or C, cirrhosis, or if they were transplanted for hepatocellular carcinoma. The recommended method of HCC surveillance is abdominal ultrasound and serum alpha-fetoprotein every 6 months. Routine upper endoscopy and brain MRIs are not recommended for surveillance in asymptomatic liver transplant patients. The patient has already undergone guideline-based screening for colon cancer. Go to the next page if you knew the correct answer, or click the link image(s) below to further research the concepts in this question (if desired).
Research Concepts: Post Transplantation Cancer
We update eBooks quarterly and Apps daily based on user feedback. Please tap flag to report any questions that
need improvement.
Question 845:
The patient is a fifty-year-old white male who recently presented with complaints of fatigue. He had dismissed his fatigue as the result of his depression, financial problems, and work-related stressors. He is the oldest of nine children. He denied fever, night sweats, significant weight loss, and neurologic symptoms. His physical examination showed modest splenomegaly with a modicum of (small) lymphadenopathy. He has no skin lesions. His CT scan of the chest, abdomen, and pelvis revealed no effusions or hepatomegaly. His LDH and Beta-2microglobulin were slightly elevated. His blood laboratories revealed a leukocyte count of 90,000 /uL (normal 4-11,000 /uL) with 90% lymphocytes. The hemoglobin was 10 g/dL (normal 13.5 to 16.5 g/dL), the hematocrit was 30% (normal 38 to 48%), and the platelet count was 100,000 /uL (normal 150 to 450,000 /uL). The peripheral blood contained a dominant population of moderately sized cells with a high nuclear/cytoplasmic ratio, a single nucleolus, basophilic cytoplasm, and cytoplasmic protrusions. Flow cytometry revealed the following positive/negative results - Positive: TCL1, CD2, CD3, CD5, CD7, and CD4/CD8 overexpression. Negative: TdT, HTLV1 serology, CD1a, CD16, CD30, and CD56. Cytogenetics revealed + 11q23 and (inv14)(q11q32). The clinical and laboratory findings were felt to be commensurate with a diagnosis of T-cell prolymphocytic leukemia. Which of the following would be unnecessary in the evaluation of this patient?
Choices: 1. 2. 3. 4.
Bone marrow biopsy CD52 status HLA typing of siblings CMV status
Answer: 1 - Bone marrow biopsy Explanations: In the diagnostic workup of T-PLL, the bone marrow biopsy has been largely supplanted by the use of flow cytometry and cytogenetics. CD52 is highly expressed in T-PLL: the main reason for alemtuzumab efficacy. The patient has eight other siblings, so the likelihood of a match is significant. Since allogeneic bone marrow transplant could be this patient's consolidation therapy, this process should be completed as soon as possible. CMV can reactivate during alemtuzumab therapy. Knowing and monitoring its status is required. Go to the next page if you knew the correct answer, or click the link image(s) below to further research the concepts in this question (if desired).
Research Concepts: T Cell Prolymphocytic Leukemia
We update eBooks quarterly and Apps daily based on user feedback. Please tap flag to report any questions that need improvement.
Question 846:
A 22-year-old male in the bone marrow transplant unit begins to complain overnight of rectal bleeding. The on-call physician is paged and presents to the bedside for further evaluation. The patient states he woke up in the middle of the night feeling bloated, went to the bathroom, then had a bowel movement that was limited in volume but pink and appeared that it might have had some blood clots. What is the next best step in evaluation?
Choices: 1. Perform a rectal exam 2. Obtain labs, including complete blood counts 3. Arrange for further evaluation by colonoscopy in the morning 4. Defer further management to the day team of physicians
Answer: 2 - Obtain labs, including complete blood counts
Explanations: The key point here is that a patient in the bone marrow transplant unit is likely to be immunosuppressed. A rectal exam is contraindicated in immunosuppressed patients, as this could introduce a nidus for infection. Obtaining blood counts first would help determine the patient's absolute neutrophil count, and also provide information about the hemoglobin and hematocrit, and this has dropped compared to previously. A colonoscopy may be needed for further evaluation, but adequate resuscitation (blood transfusions if needed), as well as ensuring that the patient is not profoundly neutropenic first is important. Go to the next page if you knew the correct answer, or click the link image(s) below to further research the concepts in this question (if desired).
Research Concepts: Rectal Exam
We update eBooks quarterly and Apps daily based on user feedback. Please tap flag to report any questions that need improvement.
Question 847:
A 28-year old White male presents to the office with a chief complaint of sensitive "bumps" on his left arm that have been present for 5 years. Physical examination reveals skin-tone nodules ranging in size from 2 mm to 20 mm. Histopathology reveals interweaving bundles of smooth muscle cells with characteristic "bluntended cigar-shaped nuclei" and eosinophilic cytoplasm admixed with collagen bundles. Which of the following is the gold standard treatment in this patient?
Choices: 1. 2. 3. 4.
Nifedipine Injection of botulinum toxin Doxazosin Surgical excision
Answer: 4 - Surgical excision Explanations: The sign and symptoms, along with histopathology findings, are suggestive of multiple piloleiomyomas that may indicate the presence of an underlying mutation in fumarate hydratase, which is associated with the diagnosis of hereditary leiomyomatosis and renal cell carcinoma (HLRCC) or Reed syndrome. Cutaneous leiomyomas are rare, benign, smooth muscle tumors that are subcategorized based on the origin of the smooth muscle within the tumor. The most common type of cutaneous leiomyomas, angioleiomyomas, arise from the tunica media of blood vessels. The 2 other subtypes, piloleiomyomas, and genital leiomyomas, arise from the arrector pili musculature of the hair follicle and from the smooth muscle found in the scrotum, labia, or nipple, respectively. Histological examination of piloleiomyomas reveals interweaving bundles of smooth muscle cells with characteristic “blunt-ended cigar-shaped nuclei” and eosinophilic cytoplasm admixed with collagen bundles. Piloleiomyomas are typically found in the reticular dermis, close to a hair follicle. Mitotic figures are rarely seen, but if present or abundant, should raise suspicion for leiomyosarcoma. Usually, piloleiomyomas are easily identified on H and E staining, but “special stains” may occasionally be needed to confirm smooth muscle origin. The gold standard of therapy is surgical excision if a patient has a small, localized amount of piloleiomyomas. An in-depth discussion regarding the significant risk of local recurrence, sometimes as early
as six weeks after removal, should be held with the patient prior to implementing treatment. Additional ablative treatment options include cryosurgery and carbon dioxide (CO2) laser. Go to the next page if you knew the correct answer, or click the link image(s) below to further research the concepts in this question (if desired).
Research Concepts: Cutaneous Leiomyomas
We update eBooks quarterly and Apps daily based on user feedback. Please tap flag to report any questions that need improvement.
Question 848:
A 40-year-old woman presents for chemotherapy. She has a past medical history of HERpositive left breast cancer. A highly selective antagonist of the G-protein coupled neurokinin-1 receptor, and a 5 HT3 receptor antagonist are prescribed to prevent nausea and vomiting. In which of the following cases is the combination of this medication and dexamethasone recommended?
Choices: 1. 2. 3. 4.
Over 30 and with up to 90 percent risk of emesis 20 to 30 percent risk of emesis 10 to 20 percent risk of emesis Under 10 percent risk of emesis
Answer: 1 - Over 30 and with up to 90 percent risk of emesis
Explanations: Aprepitant is indicated for the prevention of chemotherapy-induced and postoperative nausea and vomiting. Chemotherapy agents are broadly divided into five emetogenic levels. Each level states the expected frequency of emesis if the patient receives no prophylactic antiemetic. An (NK-1) receptor antagonist is recommended in combination with a 5-HT3 receptor antagonist (e.g., ondansetron) and a glucocorticoid (often dexamethasone) for highly emetogenic chemotherapy (HECT) and moderately emetogenic chemotherapy (MECT). High emetogenic chemotherapy is over 90 percent risk of emesis, and a moderate one is over 30 and up to 90 percent risk of emesis. Go to the next page if you knew the correct answer, or click the link image(s) below to further research the concepts in this question (if desired).
Research Concepts: Aprepitant
We update eBooks quarterly and Apps daily based on user feedback. Please tap flag to report any questions that need improvement.
Question 849:
A 61-year-old woman with a medical history of ovarian cancer treated initially by hysterectomy with bilateral oophorectomy 3 years ago presents with abdominal pain for a few weeks. On clinical examination, the patient presents stable vital parameters with a distended abdomen. The percussion is painful with a positive fluid shift test. The full-body CT scan shows several small peritoneal nodules. Biopsy reveals positive cytology for neoplastic cells of ovarian origin. Which of the following is the most appropriate management strategy for this patient?
Choices: 1. Cytoreductive surgery of all peritoneal lesions only 2. Systemic non-platinum-based chemotherapy 3. Cytoreductive surgery of all peritoneal lesions combined with hyperthermic intraperitoneal chemotherapy in a clinical trial 4. Localized radiation therapy in areas rich in peritoneal nodules
Answer: 3 - Cytoreductive surgery of all peritoneal lesions combined with hyperthermic intraperitoneal chemotherapy in a clinical trial
Explanations: First described by Spratt in 1980 in animal experiments, hyperthermic intraperitoneal chemotherapy (HIPEC) has become the treatment of choice for treating peritoneal metastases from ovarian, stomach, or colorectal cancers. This involves administering cytotoxic agents into the peritoneal cavity at an elevated temperature (41 to 43°C) in order to promote their absorption by neoplastic nodules. In ovarian cancer, the risk of peritoneal infiltration is very high due to the anatomical proximity to the peritoneal surfaces and its similar mesodermal embryonic origin of the cells. In ovarian neoplasia with peritoneal carcinosis, treatment should be multimodal and include preoperative systemic chemotherapy, complete cytoreductive surgery, and intraabdominal hyperthermia combined with intraoperative chemotherapy. According to a review of the literature and metaanalysis, cytoreduction surgery with HIPEC prolongs overall survival and up to eight years afterward. It will also control ascites better than other measures. It should optimally be done within a clinical trial. If systemic chemotherapy were to be used, platinumbased would be preferred as there is no evidence that the patient has platinum-resistant disease. Radiotherapy does not play any role in this context and could be deleterious for the patient. Systemic chemoradiotherapy alone is not indicated at this stage
of metastatic disease because surgical cytoreduction is necessary first. Go to the next page if you knew the correct answer, or click the link image(s) below to further research the concepts in this question (if desired).
Research Concepts: Hyperthermic Intraperitoneal Chemotherapy
We update eBooks quarterly and Apps daily based on user feedback. Please tap flag to report any questions that need improvement.
Question 850:
A 35-year-old primigravid female presents to the clinic for her prenatal visit at 10 weeks of gestation. She reports vaginal spotting, nausea, and vomiting. Her menarche occurred at 13 years of age, and she has had 8 sexual partners since. She has no medical history and takes no medications except for prenatal vitamins. She smokes one pack of cigarettes daily for the past 6 years but quit as soon as she learned she was pregnant. She does not drink alcohol. Her blood pressure is 115/75 mmHg, and her pulse is 78/min. The fetal heart rate is 140/min, as shown by Doppler. Physical examination reveals a well-appearing female with gravid, nontender uterus. The cervix is long, closed, firm, and posterior. The Pap test shows a high-grade squamous intraepithelial lesion. Which of the following is the best next step in the management of this patient?
Choices: 1. 2. 3. 4.
Loop electrosurgical excision procedure Colposcopy Repeat co-testing in three years Repeat co-testing after delivery
Answer: 2 - Colposcopy Explanations: The patient is pregnant, and her Pap smear shows a high-grade squamous intraepithelial lesion (HSIL). HSIL in women under 24 years requires colposcopy. Women over 24 years are recommended immediate excisional treatment or colposcopy. The patient is pregnant. Therefore we cannot proceed with excisional treatment. Hence, we choose colposcopy. Excisional treatments include loop electrosurgical excision and laser and cold knife conization. Cervical excision is done only if evidence of invasive cancer is found on colposcopy. Immediate surgical excision based on Pap testing alone is contraindicated due to the risk of preterm delivery. Go to the next page if you knew the correct answer, or click the link image(s) below to further research the concepts in this question (if desired).
Research Concepts: Papanicolaou Smear
We update eBooks quarterly and Apps daily based on user feedback. Please tap flag to report any questions that need improvement.
Question 851:
A 76-year-old man is planned for surgical resection, with adjuvant chemo- and radiotherapy for a squamous cell malignancy in his anal canal. Which of the following best explains the primary benefit of the addition of mitomycin-C to 5-fluorouracil in the chemotherapy regimen?
Choices: 1. 2. 3. 4.
Reduced grade 4 and 5 toxicity Reduced brain metastasis Improved 5-year local control Improved quality of life
Answer: 3 - Improved 5-year local control Explanations: Clinical trials in anal cancer have demonstrated that concurrent chemotherapy and radiation yield results superior to those of radiation alone or surgery alone. The RTOG 87-04 trial demonstrated a benefit to chemoradiation with mitomycin-C (MMC) and 5fluorouracil (5FU) versus chemoradiation with 5FU alone. This study showed an improvement in 5-year local control from 64% to 83%. There was also a decreased colostomy rate from 22% to 9% (p = 0.002) with the addition of MMC to 5FU. Go to the next page if you knew the correct answer, or click the link image(s) below to further research the concepts in this question (if desired).
Research Concepts: Radiation Therapy For Anal Cancer
We update eBooks quarterly and Apps daily based on user feedback. Please tap flag to report any questions that need improvement.
Question 852:
A 65-year-old man who has recently immigrated from China presents to the clinic with a left earache. He has a past medical history of hypertension and hyperlipidemia. His blood pressure is 145/95 mmHg, heart rate is 88 beats/min, respiratory rate is 17 breaths/minute, and temperature is 36.7 C (98.2 F). His otoscopic examination of the left ear reveals a loss of light reflex and decreased tympanic membrane mobility. The right ear shows no abnormalities. Based on these findings, which region should be examined in this patient?
Choices: 1. 2. 3. 4.
Nasopharynx Inferior nasal turbinate Oropharynx Nasal septum
Answer: 1 - Nasopharynx Explanations: He has most likely nasopharyngeal carcinoma (NPC). NPC is a malignant epithelial tumor most commonly found in southern Chinese males. Predisposing factors appear to be host genetics, environmental factors, and Epstein-Barr virus (EBV) infection. Nasopharyngeal carcinomas (NPC) most commonly originate from the fossa of Rosenmuller, within the lateral wall of the nasopharynx. NPC is known to spread to adjacent structures; presenting signs and symptoms typically depend on which structures are involved. If the tumor involves the Eustachian tube opening, the patient can present with serous otitis media, typically unilateral. Therefore, considering this patient’s demographics and physical examination findings, the provider should perform a nasal endoscopy to look for abnormal lesions or masses in the nasopharynx. Regarding nodal metastasis, cervical lymphadenopathy is the most common presenting symptom (75% of cases) of nasopharyngeal carcinomas (NPC). Typically, the retropharyngeal nodes are the first to be involved. Distant metastasis is thought to occur at higher rates in NPC than in other head and neck cancers. Bone and lungs are the most common sites of distant metastasis. Go to the next page if you knew the correct answer, or click the link image(s) below to further research the concepts in this question (if desired).
Research Concepts:
Nasopharyngeal Carcinoma
We update eBooks quarterly and Apps daily based on user feedback. Please tap flag to report any questions that need improvement.
Question 853:
A 31-year-old man is brought to the outpatient clinic with the complaint of one episode of focal seizures involving the left side of the body 2 days back. On examination, he is neurologically intact. He undergoes a magnetic resonance imaging scan of the brain, which shows a T1-hypo and T2-hyperintense lesion involving the right frontal lobe with mild mass effect and blooming on T2-star images. He undergoes craniotomy and decompression of the lesion. Which of the following histological features is expected in this lesion?
Choices: 1. 2. 3. 4.
Fried egg appearance Positive staining for GFAP True rosettes Pseudo-rosettes
Answer: 1 - Fried egg appearance Explanations: The imaging features are suggestive of oligodendroglioma. Lucent perinuclear halos give the fried egg appearance on histology. This appearance is an artifact of fixation with formalin. Since oligodendrogliomas contain mostly microtubules, they usually do not stain positive for GFAP. Go to the next page if you knew the correct answer, or click the link image(s) below to further research the concepts in this question (if desired).
Research Concepts: Gliomas
We update eBooks quarterly and Apps daily based on user feedback. Please tap flag to report any questions that need improvement.
Question 854:
A 59-year-old man with chronic lymphocytic leukemia (currently taking acalabrutinib) is admitted to the hospital for acute respiratory failure secondary to pneumonia. This is his third ICU admission this year for the same presentation. He is treated with IV antibiotics and breathing treatments. He is discharged and later follows up in the clinic. Labs show serum IgG level 370 mg/dL (reference range: 700-1600 mg/dL), serum IgA level 31 mg/dL (reference range 70-400 mg/dL), and serum IgM level 12 mg/dL (reference range: 40-230 mg/dL). When compared to initial lab values at diagnosis of CLL 3 years ago, there has been a steady decline with immunoglobulins. Which of the following best approximates the patient's chances of developing this complication long-term?
Choices: 1. 2. 3. 4.
10% 25% 33% 50%
Answer: 2 - 25% Explanations: Hypogammaglobulinemia is a disorder caused by low serum immunoglobulin or antibody levels. Immunoglobulins are the main components of the humoral immune system and are able to recognize antigens to trigger a biological response and eradicate the infectious source. Hypogammaglobulinemia is the most common primary immunodeficiency and encompasses a majority of immune-compromised patients. Approximately 25% of patients diagnosed with CLL develop hypogammaglobulinemia long term. IgG levels are subsequently lower during the course of the disease when compared to initial levels during diagnosis. Go to the next page if you knew the correct answer, or click the link image(s) below to further research the concepts in this question (if desired).
Research Concepts: Hypogammaglobulinemia
We update eBooks quarterly and Apps daily based on user feedback. Please tap flag to report any questions that need improvement.
Question 855:
An 86-year-old male was begrudgingly brought to the clinic by his daughter-in-law after his continuous complaints of fatigue ongoing the past few months. He denies a medical history but has refused to see a physician in over 20 years since his wife passed away following an appendectomy that had gone awry. Aside from the fatigue, his only complaints are mild back pain and stomach pain, which has occurred on and off for years for which he takes non-steroidal anti-inflammatory drugs (NSAIDs) with some relief. Physical examination is significant for mild epigastric tenderness to palpation and + 1 pedal edema of the lower extremities bilaterally. Testing of para-amino hippuric acid clearance (a conjugate measure for renal plasma flow) and glomerular filtration rate (GFR) on lab results helped the physician conclude that the patient’s disease involved a significant decrease in the renal filtration fraction. Which of the following study results is diagnostic of this patient’s underlying disease.
Choices: 1. 2. 3. 4.
Echocardiography SPEP (serum protein electrophoresis) 24-hour urine collection Upper GI endoscopy
Answer: 2 - SPEP (serum protein electrophoresis) Explanations: Echocardiography is an imaging technique used to assess for congestive heart failure. Clinically speaking a patient with congestive heart failure is considered hypervolemic. However, there is a reduction in effective circulating blood volume leading to a significantly decreased renal plasma flow. Although the GFR is also decreased in congestive heart failure, it is not to the extent of renal plasma flow, which leads to the overall increase in the filtration fraction. A monoclonal spike (M-spike) on serum protein electrophoresis suggests that the patient may have multiple myeloma. The multiple myeloma is a disease in which abnormal plasma cells produce a significant amount of abnormal antibodies, increasing the serum protein. The filtration fraction (FF) is the fraction of renal plasma flow (RPF) filtered across the glomerulus, estimated with the glomerular filtration rate (GFR). The equation to calculate the filtration fraction is as follows. FF = GFR/RPF Para-amino hippuric acid testing is used as a conjugate measure for RPF. The physician in question uses the eGFR (determined on lab studies) and RPF (via conjugate testing of para-amino hippuric acid clearance) to discover the patient to have a low filtration fraction. According to the Starling equation used in association with GFR, the driving force for filtration across the glomerulus is the net ultrafiltration pressure across the capillaries, which includes the hydrostatic and oncotic pressures. The multiple myeloma is a state of increased plasma protein concentration. The GFR is, therefore decreased due to
significantly increased glomerular capillary oncotic pressure attributed to elevated serum protein. There is no change in RPF, resulting in decreased FF. A 24-hour urine collection can be utilized to make the diagnosis of nephrotic syndrome. Nephrotic syndrome is a state of decreased plasma protein concentration due to proteinuria. The decrease in plasma oncotic pressure leads to an increase in GFR with no change in RPF, resulting in increased FF. An upper GI endoscopy may be utilized to assess for peptic ulcer disease secondary to extensive NSAID use. Prostaglandins are involved in afferent arteriole dilation. NSAID use inhibits prostaglandins leading to decreased arteriole dilation and therefore decrease GFR. This also leads to a relatively equal decrease in RPF, leading to no change in filtration fraction with chronic NSAID use. Go to the next page if you knew the correct answer, or click the link image(s) below to further research the concepts in this question (if desired).
Research Concepts: Physiology, Renal Blood Flow and Filtration
We update eBooks quarterly and Apps daily based on user feedback. Please tap flag to report any questions that need improvement.
Question 856:
A 19-year-old man presents to the clinic with pain in his lower right arm. The pain has been present for the last 2 months, and now is starting to interfere with work. An extensive workup is suggestive of a giant cell tumor of bone. The patient meets with the treatment team and undergoes the recommended treatment without the local adjuvant. There are no complications. Which of the following best describes the prognosis for this patient at this point?
Choices: 1. Recurrence of the tumor is rare and does not require postop imaging 2. Post-op radiation is needed to reduce the rate of recurrence 3. Recurrence is high as 50% after surgery 4. Recurrence is high as 22% after surgery
Answer: 3 - Recurrence is high as 50% after surgery Explanations: Intralesional curettage without a local adjuvant has a recurrence rate as high as 50%, three-fourths within two years. Local adjuvants have been shown in retrospective studies reduced recurrence rates ranging from 13-22%, not yet confirmed by a randomized control trial. Local adjuvants include aqueous zinc chloride, bone cement (PMMA), phenol, argon beam coagulation or cryotherapy. Bone cement the preferred adjuvant to reduces recurrence rates when performing an intralesional curettage. Go to the next page if you knew the correct answer, or click the link image(s) below to further research the concepts in this question (if desired).
Research Concepts: Osteoclastoma
We update eBooks quarterly and Apps daily based on user feedback. Please tap flag to report any questions that need improvement.
Question 857:
A 55-year-old with a significant medical history of cancer presents with multiple violaceous nodules and tender plaques distributed mainly in the trunk and upper extremities. Vital signs are within normal limits. The patient denies other associated symptoms. Upon further questioning, the patient revealed that he received his first cycle of cytarabine one week ago. He had a similar rash in the past after chemotherapy, which was biopsied. The biopsy specimens revealed a dense neutrophilic infiltrate surrounding and infiltrating within and around eccrine glands, with necrotic eccrine epithelial cells. Which of the following cancers does this patient most likely have?
Choices: 1. 2. 3. 4.
Acute myelogenous leukemia (AML) Lung Cancer Skin cancer Prostrate cancer
Answer: 1 - Acute myelogenous leukemia (AML) Explanations: Neutrophilic eccrine hidradenitis (NEH) is most frequently seen in AML patients receiving systemic chemotherapy. NEH lesions favor the trunk and extremities. Half of NEH patients are asymptomatic. Fever and lesional tenderness upon palpation are the most common complaints in those who are symptomatic. NEH lesions are best described as dark red, violaceous macules, papules, nodules, or plaques. Neutrophilic eccrine hidradenitis (NEH) is a benign, rare condition that was initially used to describe the skin eruptions observed in acute myelogenous leukemia (AML) patients receiving systemic chemotherapy. It can be considered a reactive disorder and is commonly associated with malignancies that include other leukemias, Hodgkin lymphoma, and solid tumors. Go to the next page if you knew the correct answer, or click the link image(s) below to further research the concepts in this question (if desired).
Research Concepts: Neutrophilic Eccrine Hidradenitis
We update eBooks quarterly and Apps daily based on user feedback. Please tap flag to report any questions that
need improvement.
Question 858:
A 26-year-old man presents with a worsening cough and shortness of breath for two weeks. His medical history is significant for lung transplantation a few months ago. His vital signs show a heart rate of 100/min, respiratory rate of 22/min, blood pressure of 100/70 mmHg, and a temperature of 99 F (37.2 C). Physical examination is remarkable for features consistent with consolidation in the transplanted lung. Based on CT thorax and bronchoscopy, a diagnosis of locally invasive Aspergillus infection at the bronchial anastomosis is made. After administering inhaled amphotericin, he is discharged on voriconazole as daily prophylaxis for the first 12 months. In the follow-up period, this patient will need to be monitored as he is at risk for developing which malignancy?
Choices: 1. 2. 3. 4.
Adenocarcinoma of the lung Squamous cell cancer of the skin Hodgkin lymphoma Kaposi sarcoma
Answer: 2 - Squamous cell cancer of the skin Explanations: Both locally invasive and disseminated Aspergillus infections occur in lung transplant patients. The infection can occur at the site of the bronchial anastomosis and cause ulceration. The treatment is inhaled amphotericin followed by 12 months of prophylaxis with voriconazole. Patients on voriconazole need to be monitored as they are at an increased risk of developing squamous cell cancer of the skin. Voriconazole is also associated with numerous other skin conditions, including photosensitivity, DRESS syndrome, melanoma, etc. Go to the next page if you knew the correct answer, or click the link image(s) below to further research the concepts in this question (if desired).
Research Concepts: Aspergillosis
We update eBooks quarterly and Apps daily based on user feedback. Please tap flag to report any questions that need improvement.
Question 859:
A 68-year-old woman presents to the clinic after a recent diagnosis of acute myeloid leukemia. She is otherwise healthy. A decision is made to proceed with induction therapy with cytarabine and daunorubicin. Bone marrow biopsy on day 14 showed no blasts and indicated clinical remission. She then undergoes myeloablative chemotherapy with melphalan and busulfan, followed by an allogeneic bone marrow transplantation from a matched related donor. Which of the following lab findings best indicates engraftment in this patient?
Choices: 1. Persistent for 3 days 2. Persistent 3. Persistent 4. Persistent
ANC (absolute neutrophils count) >500/microL ANC >500/microL for 7 days ANC >1000/microL for 3 days ANC >250/microL for 3 days
Answer: 1 - Persistent ANC (absolute neutrophils count) >500/microL for 3 days
Explanations: Engraftment defined as the first of three consecutive days of achieving a sustained peripheral blood neutrophil count of >500/mircoL. Platelet engraftment is usually defined as independence from platelet transfusion for at least 7 days with a platelet count of more than >20000/microL. ANC >1000/microL is not required to confirm engraftment. ANC >250/microL but less than 500/microL is not an indication for engraftment. Go to the next page if you knew the correct answer, or click the link image(s) below to further research the concepts in this question (if desired).
Research Concepts: Hematopoietic Stem Cell Transplantation
We update eBooks quarterly and Apps daily based on user feedback. Please tap flag to report any questions that need improvement.
Question 860:
A 66-year-old male presented to the emergency department regarding the one-week history of hemoptysis. He says that he is coughing more than he used before. But over the last week, he started to see blood in sputum. He has hypertension and a longed history of heavy smoking 2 packs a day for the last 42 years. He also reports some recent weight loss. On examination, he appears anorexic, cardiac examination shows normal heart sounds, and the lungs examination shows right-sided coarse breathing sounds. The chest X-ray shows a right-sided central mass, confirmed with a CT-scan that showed a mass of 3 cm in size. The biopsy shows keratin pearls. Which of the following is a possible finding in this patient condition?
Choices: 1. 2. 3. 4.
Hyperkalemia Hypercalcemia Hypernatremia Hypoglycemia
Answer: 2 - Hypercalcemia Explanations: This patient presentation fits lung cancer. Squamous cell carcinoma shows keratin pearls in the biopsy. Squamous cell carcinoma can cause hypercalcemia, as the tumor secretes parathyroid-hormone related peptide, which can cause hypercalcemia. Squamous cell carcinoma presents with positive sputum and is the most common occult cancer. Recently, photochemical dyes with the use of lasers have helped in improving the localization and treatment of superficial cancers. Usually, the treatment is surgical and lesser resections are not possible. Recurrences are rare, but newer primaries do occur. These patients need follow-up at 612 month intervals. Go to the next page if you knew the correct answer, or click the link image(s) below to further research the concepts in this question (if desired).
Research Concepts: Squamous Cell Lung Cancer
We update eBooks quarterly and Apps daily based on user feedback. Please tap flag to report any questions that need improvement.
Question 861:
A 59-year-old male presents to the healthcare provider with the complaint of blurring of vision over the past two months. He also complains of intermittent mild frontal headaches. On examination, he has bitemporal hemianopia. A magnetic resonance imaging of the brain is done, which shows a sella-suprasellar mass of size 3x2 cm, with a predominant sellar mass, cystic with calcification. What is the likely WHO grade of this lesion?
Choices: 1. 2. 3. 4.
WHO WHO WHO WHO
grade grade grade grade
I II III IV
Answer: 1 - WHO grade I Explanations: The patient is most likely having a diagnosis of craniopharyngioma. Most of the craniopharyngiomas are benign and belong to WHO grade I. WHO grade I tumors grow slowly and do not invade the surrounding tissues. It may be possible to remove a WHO grade I tumor completely. However, this may not always be the case as many vital structures are situated in the vicinity of the suprasellar region. Go to the next page if you knew the correct answer, or click the link image(s) below to further research the concepts in this question (if desired).
Research Concepts: Craniopharyngioma
We update eBooks quarterly and Apps daily based on user feedback. Please tap flag to report any questions that need improvement.
Question 862:
A 35-year-old male presents to the clinic for evaluation of malaise, vague abdominal pain, and loss of appetite for three months. His family history is notable for his brother dying from complications of pancreatic carcinoma. His blood pressure is 130/90 mmHg, pulse rate is 80/min, and the temperature is 99 F. On physical examination, there is the presence of a mass in the right upper quadrant, and yellowish discoloration of the sclera is noted. CT scan of the abdomen shows a large heterogeneous mass involving the head of the pancreas and the second part of the duodenum. An imaging-guided biopsy reveals sheets of malignant acinar cells with no intervening ductal structures or islets. The specimen is positive for trypsin and cytokeratin and negative for CD117 and synaptophysin. Which of the following is the most common site of metastasis of this patient's malignancy?
Choices: 1. 2. 3. 4.
Lungs Stomach Liver Colon
Answer: 3 - Liver Explanations: The most common site of metastasis with all pancreatic tumors is the liver. Patients present with liver metastasis (stage 4) have a very poor prognosis and survive an average survival of less than 14 months. Clinical diagnosis is by nonspecific symptoms such as nausea, weight loss, and abdominal pain. Liver metastases are identified on a CT scan as multiple small areas of abnormality. The metastases can affect the lymph nodes, leading to disruption of the immune system and causing recurrent infections. Other complications include gastrointestinal and biliary tract obstruction. About 50% of the patients have liver metastasis at the initial time of diagnosis. The 5-year survival rate of pancreatic acinar cell carcinoma is about 10%. Multiple factors affect the prognosis, including age, recurrence of disease, and response to treatment. Few cases have been reported of metastatic acinar cell carcinoma having an objective response following chemotherapy. Go to the next page if you knew the correct answer, or click the link image(s) below to further research the concepts in this question (if desired).
Research Concepts: Case Study: 24-Year-Old Male Presenting With Polyarthralgias
We update eBooks quarterly and Apps daily based on user feedback. Please tap flag to report any questions that need improvement.
Question 863:
A 54-year-old woman presents to the hospital with malaise, fever, and a facial rash. Her vital signs are blood pressure 130/81 mmHg, temperature 99.1 F, pulse 74/minute, and respiratory rate 22/minute. The physical exam reveals purpuric macules and patches forming on the skin and mucous membranes of the face, including keratoconjunctivitis. The patient was diagnosed with breast cancer a month ago, and she is receiving treatment with methotrexate, 5-fluorouracil, and cyclophosphamide. A medication that affects the neurokinin-1 receptor was recently added to her antiemetic regimen. Which of the following is the most likely diagnosis?
Choices: 1. 2. 3. 4.
Systemic lupus erythematosus Steven-Johnson syndrome Erythema nodosum leprosum Palmar-plantar erythrodysesthesia
Answer: 2 - Steven-Johnson syndrome Explanations: Neurokinin-1 receptor blockers, e.g., fosaprepitant can cause Steven-Johnson syndrome. Patients with Steven-Johnson syndrome can present with purpuric macules and patches forming on the skin and mucous membranes including keratoconjunctivitis. Fosaprepitant is a substance P/neurokinin 1 receptor antagonist. It is indicated for the prevention of chemotherapyinduced nausea and vomiting. Go to the next page if you knew the correct answer, or click the link image(s) below to further research the concepts in this question (if desired).
Research Concepts: Antiemetic Neurokinin-1 Receptor Blockers
We update eBooks quarterly and Apps daily based on user feedback. Please tap flag to report any questions that need improvement.
Question 864:
Following colonoscopy for hematochezia, a patient has an appointment with a surgical oncologist for further evaluation and recommendation regarding a 4 cm mid-rectal tumor, 8 cm from the anal verge. Computed tomography imaging provided with him demonstrates liver metastases that appear resectable with no associated lymphadenopathy. However, the rectal tumor is invading into the surrounding tissue. With regards to preoperative planning, which of the following best distinguishes this patient’s cancer compared to a primary colon tumor?
Choices: 1. He will require a greater duration of preoperative chemotherapy 2. He is more likely to have tumor burden necessitating radiotherapy 3. His morbidity and mortality is such that surgery is not a possibility as a result of his clinical stage, and he should undergo chemotherapy only 4. He can undergo total mesorectal excision with concomitant localized hepatectomy without need for neoadjuvant chemotherapy
Answer: 2 - He is more likely to have tumor burden necessitating radiotherapy
Explanations: Preoperative radiotherapy, along with chemotherapy, is recommended for mid to low rectal tumors with associated hepatic metastases. Resection of symptomatic rectal tumors may necessitate diverting ostomy for palliation if unable to safely proceed with low anterior resection (LAR) or abdominoperineal resection (APR). A complete response to neoadjuvant treatment is low, and it is lower in the setting of liver metastases. Total mesorectal excisions are limited to more localized disease. Go to the next page if you knew the correct answer, or click the link image(s) below to further research the concepts in this question (if desired).
Research Concepts: Liver Metastasis
We update eBooks quarterly and Apps daily based on user feedback. Please tap flag to report any questions that need improvement.
Question 865:
A 42-year-old woman presents to the emergency department with acute onset of abnormal conjugate eye movements, which are involuntary and multidirectional. The remainder of her neurological exam is normal. She has a history of stage 2 breast cancer and reports being cancer-free for the past two years. A follow-up mammogram, performed two weeks ago, was interpreted as normal. A CT scan and MRI of the brain with contrast performed in the emergency department are normal. Her workup should include which of the following investigations?
Choices: 1. 2. 3. 4.
Toxic screen Repeat mammogram Paraneoplastic antibody screening Lumbar puncture for oligoclonal bands
Answer: 3 - Paraneoplastic antibody screening Explanations: Opsoclonus often presents as a paraneoplastic manifestation of underlying malignancy and can be the first manifestation of tumor recurrence. Anti-Ri antibodies are paraneoplastic antibodies that are strongly associated with breast cancer and have been found to be associated with the humoral immunopathogenesis of opsoclonus/opsoclonusmyoclonus syndrome. Paraneoplastic antibody testing for anti-Ri antibodies, in this case, should be pursued next, given the history of breast cancer. Lumbar puncture for CSF analysis should also be considered to exclude infection and malignancy; however, investigation for oligoclonal bands is not warranted here as there are no other clinical indicators of multiple sclerosis, and the patient’s MRI brain was unremarkable. Go to the next page if you knew the correct answer, or click the link image(s) below to further research the concepts in this question (if desired).
Research Concepts: Opsoclonus
We update eBooks quarterly and Apps daily based on user feedback. Please tap flag to report any questions that need improvement.
Question 866:
A 74-year-old male presents with a request for a prostate-specific antigen (PSA) test based on a newspaper article he has read. He has no significant urological symptoms, has diet-controlled type 2 diabetes mellitus (T2DM), a BMI of 32, and treated hypertension. A digital rectal examination reveals a firm nodule in the right lobe of his prostate. His PSA test comes back at 4.2 ng/mL, and he is referred for a multiparametric magnetic resonance imaging (MRI) scan. This confirms a LIKERT 4 lesion measuring approximately 12 mm in diameter in the peripheral zone of the right lobe of the prostate without breach of the capsule. Cognitive-fusion TRUS-guided biopsy reveals 2 cores with 3 mm each of Gleason 3+4 (Grade group 2) from the region of interest. Which of the following statements is accurate about his treatment options?
Choices: 1. A robotic-assisted radical prostatectomy represents his best chance of surviving 10 years 2. Radical external beam radiotherapy is not an option given his T2DM and elevated BMI 3. A period of active surveillance/monitoring is an appropriate option 4. Any significant delay in active or definitive treatment, either in the form of radiation therapy or radical prostatectomy, will significantly adversely affect his cancerspecific mortality at 10 years
Answer: 3 - A period of active surveillance/monitoring is an appropriate option
Explanations: This patient has intermediate-risk disease without any features of high-risk disease. Trial data (e.g., ProtecT study, Hamdy et al.) demonstrates that 10-year outcomes are the same for intermediate-risk prostate cancer treated with active surveillance, radical prostatectomy, or definitive radiation therapy. Active surveillance does not preclude treatment - the patient is closely monitored and any change suggesting progression of the disease will lead to an offer of definitive therapy. Attention to his diet and weight loss while on active surveillance might well reduce the risk of progression of his prostate cancer as well as reduce the risks associated with active treatment. The patient's co-morbidities increase his surgical risk, and these would need to be discussed with him. His comorbidities are not a contraindication to radiotherapy, and in fact, radiotherapy may have a less negative impact on the quality of life than surgery. Go to the next page if you knew the correct answer, or click the link image(s) below to further research the concepts in this question (if desired).
Research Concepts: Localized Prostate Cancer
We update eBooks quarterly and Apps daily based on user feedback. Please tap flag to report any questions that need improvement.
Question 867:
A 66-year-old male presents to his hematologist with new skin lesions in his right neck, left elbow, and right groin region. He was diagnosed three months back with angioimmunoblastic lymphoma (AITL) after presenting with an isolated left groin lesion. Biopsy then had revealed a CD30 negative AITL, positron emission tomography (PET) was negative for any disseminated disease then, and he was treated with three cycles of CHOP (cyclophosphamide, hydroxydaunorubicin, vincristine, prednisone). He subsequently undergoes a repeat biopsy of one of the superficial lesions, and the diagnosis is consistent with primary progressive CD30 negative angioimmunoblastic T cell lymphoma. PET scan reveals disseminated disease with metastatic lesions in the liver, spleen, and several lymph node conglomerates. His performance status is an eastern cooperative oncology group (ECOG) 3, and his comorbidities include advanced chronic obstructive pulmonary disease (COPD), heart failure with preserved ejection fraction, and type 2 diabetes mellitus on Insulin. What would be the best treatment modality for this patient?
Choices: 1. 2. 3. 4.
Brentuximab Vedotin plus bone marrow transplant Romidepsin DHAP (dexamethasone, cisplatin, cytarabine) ICE (ifosfamide, carboplatin, etoposide)
Answer: 2 - Romidepsin Explanations: Romidepsin is the correct answer. This is a histone deacetylase inhibitor and is FDA approved as monotherapy for progressive/relapsed peripheral T cell lymphoma. This is a patient with poor performance status, multiple comorbidities, and is thus transplant ineligible. The ideal option for him would be a medication that would be less toxic and continuous. This is a CD-30 negative angioimmunoblastic lymphoma (AITL), hence no role for brentuximab vedotin, as it requires the presence of CD30 in order to be effective. Options 3 and 4 are aggressive short term chemotherapy. With his performance status and comorbid conditions, the risks of chemotherapy toxicity due to aggressive treatment combinations (ICE and DHAP) would outweigh the benefits. Go to the next page if you knew the correct answer, or click the link image(s) below to further research the concepts in this question (if desired).
Research Concepts: Peripheral T-Cell Lymphoma
We update eBooks quarterly and Apps daily based on user feedback. Please tap flag to report any questions that
need improvement.
Question 868:
A 66-year-old male with a past medical history of type 2 diabetes mellitus controlled with diet and exercise presented with a 6-month history of worsening abdominal pain, weight loss, fever, night sweats, and early satiety. What is the next best step in management?
Choices: 1. 2. 3. 4.
Helicobacter pylori stool antigen test Start proton pump inhibitor Upper endoscopy with biopsies Evaluate with scintigraphic gastric emptying study
Answer: 3 - Upper endoscopy with biopsies Explanations: Gastric lymphoma is responsible for 70% of gastrointestinal primary extra-nodal lymphomas but makes up 5% of gastric cancers. The most common symptoms of gastric lymphoma are abdominal pain, loss of appetite, weight loss, gastrointestinal bleeding, nausea, vomiting, and early satiety, in decreasing order. The diagnostic evaluation of choice for gastric lymphoma is an esophagogastroduodenoscopy with biopsies of the gastric mucosa. Laparotomy and laparoscopy are reserved for perforations and/or obstructions. Most pathology evaluations of gastric lymphomas reveal extra-nodal marginal zone B cell lymphoma of mucosaassociated lymphoid tissue (MALT) and diffuse large B cell lymphoma. Go to the next page if you knew the correct answer, or click the link image(s) below to further research the concepts in this question (if desired).
Research Concepts: Gastric Lymphoma
We update eBooks quarterly and Apps daily based on user feedback. Please tap flag to report any questions that
need improvement.
Question 869:
A 67-year-old Vietnamese veteran presented with intractable diarrhea and weight loss for the past 2 months. He was evaluated by his primary care physician and the gastroenterologist. Initial work-up for infection and inflammatory diseases was unrevealing. The radiological assessment with CT scans identified a 3.2 cm x 1.2 cm appendix mass concerning for carcinoid tumor and multiple lesions in the liver suggestive of metastasis. The chromogranin level was elevated at 1200. He underwent a colonoscopy that confirmed a moderately-differentiated intermediate-grade neuroendocrine tumor. He was started on somatostatin analogs. After three months, the patient underwent reassessment with imaging that showed worsening liver metastasis and increased size of the appendiceal tumor. The gallium-68 dotatate imaging did not trace the tumors (no PET avidity). What is the most appropriate treatment strategy for this patient?
Choices: 1. 2. 3. 4.
Trans-arteria chemoembolization (TACE) Lu-177 dotatate therapy Increase the dose of somatostatin analogs Everolimus
Answer: 4 - Everolimus Explanations: The patient has a recent progression of both primary and metastasis. At this point, the optimal treatment strategy is to use systemic therapy to gain control over his tumor. TACE is the liver-directed intervention, that could be considered once the stability of his tumor is achieved. The patient has no avidity on Ga 68-Dotatate PET. Therefore, not a candidate for Lu-177 therapy. The patient did not respond to initial doses of somatostatin analogs. There is no definitive evidence of improved outcomes with dose escalations. The intermediate-grade histology is the catch here, where the tumors are moderately differentiated with KI67 high, that is not PET avid on Ga-68 imaging, and do not respond well to Lu-177-dotatate therapy. The second-line intervention for these patients is molecularly targeted therapy such as everolimus. Go to the next page if you knew the correct answer, or click the link image(s) below to further research the concepts in this question (if desired).
Research Concepts: Neuroendocrine Tumor Lu-177-Dotatate Therapy We update eBooks quarterly and Apps daily based on user feedback. Please tap flag to report any questions that need improvement.
Question 870:
A 12-year-old female presents with a Spitz juvenile melanoma on her lower leg. Which of the following is the most accurate characterization of Spitz nevi?
Choices: 1. Spitz tumors are typically brown to black 2. A Spitz tumor is melanoma in situ occurring in a young patient 3. Although technically benign, there is strong histological resemblance to melanoma 4. Most Spitz nevi occur in white females
Answer: 3 - Although technically benign, there is strong histological resemblance to melanoma
Explanations: A Spitz tumor is typically presented as a sharply circumscribed, dome-shaped, pink-red papule most commonly on the face or lower extremities. Juvenile, in this case, refers not to the patient but the early stage of the tumor. Spitz tumors that do not show atypical clinical or dermoscopic features may be monitored clinically. Concerning Spitz nevi are excised with 3 to 5 mm margins. Spitz tumors occur in all ethnic groups. It equally affects males and females, although there may be a female predominance in young adults. Go to the next page if you knew the correct answer, or click the link image(s) below to further research the concepts in this question (if desired).
Research Concepts: Melanocytic Nevi
We update eBooks quarterly and Apps daily based on user feedback. Please tap flag to report any questions that need improvement.
Question 871:
A 71-year-old male presents to the clinic complaining of jaundice, epigastric discomfort and anorexia for the past four weeks. A detailed medical history revealed that the patient has been diagnosed with diabetes mellitus six months ago and is currently taking metformin. The patient also gives a history of passing pale stools and dark urine. He is a chronic smoker with a history of 35 packyears. On abdominal examination, the liver is palpable 4 cm below the costal margin. The patient is being taken for an ultrasound scan. Which of the following is the most likely finding on examination?
Choices: 1. 2. 3. 4.
Palpable spleen Palpable gallbladder Ascites Pleural effusion
Answer: 2 - Palpable gallbladder Explanations: Carcinoma of the head of the pancreas presents with jaundice that is painless and epigastric discomfort. Courvoisier first drew attention to the association of an enlarged gallbladder and a pancreatic tumor, when he noted that, when the common duct is obstructed by a stone, distension of the gallbladder is rare. In contrast, when the duct is obstructed by a tumor, the distention of the gallbladder is common. Hence this is called Courvoisier's law. Patients above 50 years who are newly diagnosed with diabetes and upper abdominal symptoms with no family history and obesity should raise the suspicion of a pancreatic tumor. Tumor of the pancreatic head causes obstruction of the common bile duct leading to conjugated hyperbilirubinemia which eventually results in pale stools and dark urine. Go to the next page if you knew the correct answer, or click the link image(s) below to further research the concepts in this question (if desired).
Research Concepts: Pancreatic Cancer
We update eBooks quarterly and Apps daily based on user feedback. Please tap flag to report any questions that need improvement.
Question 872:
A 25-year-old woman with American Thyroid Association (ATA) high-risk thyroid carcinoma presents to the clinic for her Radioactive Iodine (RAI) dose of 150 mCi I-131. She denies recent sexual activity and is currently on oral contraceptives. Her past medical history is significant for diabetes mellitus type 1 and a family history significant for endocrine neoplasia, including first degree relatives with parathyroid adenomas and prolactinomas. Which of the following is the most appropriate test to perform prior to RAI therapy?
Choices: 1. 2. 3. 4.
Fingerstick glucose Pregnancy test Testing for MEN1 gene mutations Testing for RET gene mutation
Answer: 2 - Pregnancy test Explanations: The ATA recommends pregnancy testing for all women of childbearing age prior to RAI therapy regardless of sexual history and contraception use. The only two absolute contraindications for RAI therapy are pregnancy and breastfeeding. Radioiodine use in pregnancy is associated with miscarriage, infantile hypothyroidism, and poor neurocognitive development. Mutations in MEN1 and RET are associated with subtypes of multiple endocrine neoplasia (MEN), however, these hereditary neoplastic disorders are more frequently associated with medullary thyroid carcinoma, which is not responsive to radioiodine therapy. Go to the next page if you knew the correct answer, or click the link image(s) below to further research the concepts in this question (if desired).
Research Concepts: 131 I Sodium Iodide
We update eBooks quarterly and Apps daily based on user feedback. Please tap flag to report any questions that need improvement.
Question 873:
A 14-year-old male patient presents to the hospital for the evaluation of nausea, vomiting, and generalized itching. He is currently on chemotherapy started for acute leukemia. Vital signs are pulse rate 90/min, blood pressure 110/65 mmHg, respiratory rate 18/min, and temperature 98.6 F (37 C). Physical examination reveals muffled heart sounds with bilateral pitting edema. Lab reveals sodium 135 mEq/L, potassium 5.5 mEq/L, calcium 7.5 mg/dL, creatinine 1.7 mg/dL, and uric acid 9.0 mg/dL. What is the most appropriate medication to treat the underlying cause of illness in this patient?
Choices: 1. 2. 3. 4.
Probenecid Furosemide Rasburicase Colchicine
Answer: 3 - Rasburicase Explanations: Rasburicase is a new medication for the prevention and treatment of tumor lysis syndrome in patients receiving chemotherapy for hematologic malignancies like lymphomas and leukemias. For a patient with a high-risk tumor, the overall consensus is to start on prophylactic urate oxidase inhibitor therapy before the initiation of chemotherapy. It is advisable to start rasburicase for hyperuricemia. Rasburicase catalyzes the metabolism of uric acid to allantoin. Allantoin is an inactive metabolite of purine metabolism and is ten times more soluble than uric acid. Therefore, it is more rapidly excreted by the kidneys. The drug metabolizes uric acid into hydrogen peroxide, a potent oxidizing agent that can cause severe methemoglobinemia or hemolytic anemia in patients with glucose 6 phosphate dehydrogenase (G6PD) deficiency. It is contraindicated in patients with G6PD deficiency. Go to the next page if you knew the correct answer, or click the link image(s) below to further research the concepts in this question (if desired).
Research Concepts: Tumor Lysis Syndrome
We update eBooks quarterly and Apps daily based on user feedback. Please tap flag to report any questions that need improvement.
Question 874:
A 45-year female with a previously diagnosed 3 cm cystic lesion in the head of the pancreas presents for evaluation. A recent magnetic resonance imaging (MRI) scan of the abdomen shows an increase in size to 5 cm, and endoscopic ultrasonography (EUS) showed papillary projections with no communication to the pancreatic duct. Cyst fluid aspiration revealed a carcinoembryonic antigen level of 210 ng/mL. Which of the following is the next step in the management of this patient?
Choices: 1. Duodenum preserving pancreatic head resection 2. Continued surveillance 3. Whipple's pancreaticoduodenectomy 4. Enucleation of the pancreatic cyst with frozen section biopsy
Answer: 3 - Whipple's pancreaticoduodenectomy Explanations: Mucinous cystadenoma occurs in middle-aged females and is slow-growing. These tumors can be resected and have a five-year survival of about 70%. Larger cystadenoma tends to be malignant and is usually located in the tail of the pancreas. Papillary projections, thickened walls are suspicious features of malignancy. Surveillance is an accepted standard of care in cystic lesions of size less than 4 cm. Go to the next page if you knew the correct answer, or click the link image(s) below to further research the concepts in this question (if desired).
Research Concepts: Pancreatic Mucinous Cystadenoma
We update eBooks quarterly and Apps daily based on user feedback. Please tap flag to report any questions that need improvement.
Question 875:
A 55-year-old male presents to the gastroenterology clinic with complaints of persistent diarrhea. He has had multiple episodes of watery, teacolored loose stools for the past eight months. He often has to get up in the middle of the night and states that fasting does not improve his symptoms. There is no history of blood in stools. However, he has lost about 16 pounds (7.3 kg) in weight during the same period. He has a history of diabetes mellitus, for which he uses subcutaneous insulin. He also has a history of cholecystectomy ten years back, for symptomatic cholelithiasis. His examination reveals a blood pressure of 100/50 mmHg, a pulse of 105 beats per minute, a respiratory rate of 12 per minute, and a temperature of 98 F. His systemic examination is unremarkable. His investigations reveal a WBC count of 6,000 per microL, a hemoglobin of 11gm/dl, a platelet count 250,000 per microL, serum ALT 35 IU/L, serum AST 30 IU/L, serum creatinine 0.8 mg /dl, serum sodium 135 meq/l , serum potassium 2.8 meq/l , serum chloride 110 meq/l, serum bicarbonate 18 meq/l, arterial pH 7.2, HbA1c 7.1% , blood sugar random 150 mg/dl and serum CRP 3.5 mg/l. His stool examination is negative for ova and cysts, and stool culture is negative. The stool osmolar gap is calculated to be 40 mOsm/kg. A colonoscopy with biopsy is normal. A contrastenhanced CT scan reveals the presence of a 3 cm solitary mass in the tail of the pancreas. Urinary 5 hydroxytryptamine level is normal. The patient is treated with fluid and electrolyte supplementation. Which of the following is an appropriate treatment option in this patient?
Choices: 1. Rifaxamine 2. Clonidine
. Octreotide 3 4. Cholestyramine
Answer: 3 - Octreotide Explanations: This patient has presented with chronic, voluminous diarrhea which does not resolve on fasting. The laboratory findings of hypokalemia, hyperchloremia, and normal anion gap acidosis along with normal stool osmolar gap make the diagnosis of secretory diarrhea. The presence of a solitary pancreatic lesion indicates the presence of pancreatic VIPoma. Somatostatin analogs are useful for the management of VIPoma. VIPomas are functional neuroendocrine tumors that secrete vasoactive intestinal peptide. The signs and symptoms are due to hormonal activation. Somatostatin analogs such as octreotide and lanreotide are highly effective in controlling diarrhea in VIPoma. The initial dose of octreotide is 50-100 mcg subcutaneously every 8 hours, followed by titration as required. Monthy depot intramuscular formulations are available once symptom control is adequate. Octreotide is usually well tolerated but can have some important side effects such as gall bladder hypomotility. Treatment with glucocorticoids can be considered in individuals with resistant diarrhea. Clonidine is useful in diarrhea associated with diabetic autonomic enteropathy, which is unlikely given the good glycemic control. Diarrhea can be present post-cholecystectomy due to malabsorption of bile salts; it is mainly osmotic and responds to cholestyramine. Small bowel overgrowth can be managed with oral antibiotics such as rifaxamine.
Go to the next page if you knew the correct answer, or click the link image(s) below to further research the concepts in this question (if desired).
Research Concepts: ViPoma
We update eBooks quarterly and Apps daily based on user feedback. Please tap flag to report any questions that need improvement.
Question 876:
A 67-year-old African-American female patient presents with decreased appetite and unintentional weight loss of 1-month duration. Her past medical history is significant for hypertension and diabetes mellitus, with no known complications. Upon further questioning, she also complains of decreased urine output in the last three days. Vital signs reveal heart rate 110 beats per minute, blood pressure 105/60 mmHg, respiratory rate 14 breaths per minute, oxygen saturation 96% on room air, and temperature 37 C (98.6 F). On physical examination, she looks chronically ill, drowsy but rousable, and is oriented. Lung and heart sounds are normal. She has 1+ bilateral lower extremity edema. Lab tests show white blood cell count of 8 x 10^9/L (4-10), hemoglobin 8 g/dL (12-15), platelets 165 x 10^9/L (150-400), sodium 135 mmol/L (135145), potassium 5.4 mmol/L (3.5-5.0), chloride 108 mmol/L (95-105), blood urea nitrogen 44mg/dL (8-21), creatinine 4.7 (baseline 1.4), calcium 9.5 mmol/L (1.03-1.23), albumin 2.8 g/L (35-50). A renal ultrasound reveals a 10.8 cm right kidney and 11 cm left kidney without any mass or obstruction. Urine analysis shows protein 300 mg/dL. A 24hour urine protein and creatinine ratio was 4200 mg/24 hours. What is the most appropriate next step in the diagnosis of this patient's condition?
Choices: 1. 2. 3. 4.
Renal biopsy Serum immunofixation electrophoresis Serum protein electrophoresis Serum-free light chains
Answer: 4 - Serum-free light chains Explanations: The patient likely has multiple myeloma, and the possible etiology for the renal failure appears secondary to myeloma kidney or cast nephropathy. There is a dissociation between the protein on urine analysis and the 24-hour sample. The next best step would be to obtain the serum-free light chains (SFLC) ratio. This test has higher sensitivity when compared to serum protein electrophoresis (SPEP) and serum immunofixation electrophoresis (SIFE). Eventually, the patient may need a kidney biopsy depending upon her renal recovery and bone marrow findings. The patient meets the criteria for possible myeloma by age, anemia, and renal abnormalities. Go to the next page if you knew the correct answer, or click the link image(s) below to further research the concepts in this question (if desired).
Research Concepts: Myeloma Kidney
We update eBooks quarterly and Apps daily based on user feedback. Please tap flag to report any questions that need improvement.
Question 877:
A 65-year-old male has an acute onset of low back pain. A CT scan shows an L3 burst fracture. He also reports a 30-pound (13.6 kg) weight loss over the past three months. He complains of numbness of the inguinal and perineal areas. Ankle jerk is absent bilaterally. Motor strength below the knee area is significantly reduced. What is the most likely etiology of his burst fracture?
Choices: 1. 2. 3. 4.
Pathologic fracture due to metastatic disease Osteoporosis L3 schwannoma Lymphoma
Answer: 1 - Pathologic fracture due to metastatic disease
Explanations: The most common extradural and extramedullary lesions to the spine are metastatic lesions. Common spine metastases originate from the lung, breast, prostate, and kidney. Metastatic disease to the spine can cause bony destruction and weakening of the vertebrae, thus leading to pathologic fracture. Retropulsed bone fragments and tumor extension can cause neurological deficits along with back pain. Symptoms of cauda equina syndrome are common. Many lesions in the conus will present with similar history and physical exam as those in the cauda because the nerve roots compressed in the cauda originate in the conus area. Deep tendon reflexes may be diminished in the lower extremities. The lesion in the cauda will only have the Achilles tendon involved, while those with conus lesion will have also the patellar knee reflex involved. Go to the next page if you knew the correct answer, or click the link image(s) below to further research the concepts in this question (if desired).
Research Concepts: Conus And Cauda Equina Tumors
We update eBooks quarterly and Apps daily based on user feedback. Please tap flag to report any questions that need improvement.
Question 878:
A 68-year-old man presents to the clinic with dysphagia, weight loss, hoarseness of voice, and palpable nodules in the neck. He has a 40-pack-year history of smoking. Laryngoscopy reveals a large, exophytic mass emanating from the infrahyoid supraglottis and extending inferiorly to involve the true glottis, which is nearly completely effaced and is immobile; multiple, firm, immobile, 1-2 cm lymph nodes corresponding to levels II and III. In addition, a contrast-enhanced CT scan of the neck shows extensive thyroid cartilage destruction, with tumor visibly transgressing the full thickness of the thyroid cartilage. A biopsy confirms squamous cell carcinoma, and further imaging indicates no distant metastasis. What is the most appropriate treatment strategy for this patient?
Choices: 1. Definitive external beam radiation alone 2. Total laryngectomy and bilateral neck dissections 3. Total laryngectomy, bilateral neck dissections, and postoperative radiation therapy 4. Concurrent external beam radiation and chemotherapy
Answer: 3 - Total laryngectomy, bilateral neck dissections, and postoperative radiation therapy
Explanations: Symptomatology beginning with dysphagia and progressive hoarseness often indicates a supraglottic primary cancer that has grown to invade the glottis. Additionally, weight loss is a poor prognostic sign and often a surrogate for advanced disease and/or significant mass-effect, causing dysphagia and decreased oral intake. Major invasion of the thyroid cartilage, in addition to the multiple, bilateral lymph nodes less than 6 cm, render this a T4aN2cM0 squamous cell carcinoma of the supraglottic larynx. In locally advanced laryngeal cancer, the optimal treatment is based on whether the larynx can be preserved and on patient comorbidities. T4a disease precludes laryngeal preservation, and thus concurrent chemoradiation, or monotherapy with radiation or chemotherapy alone, is not appropriate. Thus, laryngectomy, possibly also entailing a thyroidectomy due to the local invasion, is the best choice in a patient who can undergo surgery. Adjuvant radiation and/or chemotherapy may be offered depending on the presence of high-risk features on final surgical pathology. Go to the next page if you knew the correct answer, or click the link image(s) below to further research the concepts in this question (if desired).
Research Concepts:
Laryngeal Cancer
We update eBooks quarterly and Apps daily based on user feedback. Please tap flag to report any questions that need improvement.
Question 879:
A 37-year-old man presents to the clinic after recently starting treatment with an R-CHOP (rituximab, cyclophosphamide, doxorubicin, vincristine [Oncovin], and prednisolone) regimen for Hodgkin lymphoma. The patient complains of recent frequent urinary urgency, lower back and penile pain, and the sensation of incomplete urinary emptying. The patient is diagnosed with bladder spasms, which lead to referred pain in the glans penis and lower back. Which of the following best describes the pathophysiology of the patient's symptoms?
Choices: 1. Immunosuppressive effects of cyclophosphamide irritating the epithelial cells in the ureter 2. DNA damage in the bladder due to DNA intercalating effects of hydroxydaunorubicin 3. Blood clot retention precipitated from hemorrhagic cystitis brought on by cyclophosphamide 4. Anti-mitotic effects of oncovin on cells within the bladder mucosa
Answer: 3 - Blood clot retention precipitated from hemorrhagic cystitis brought on by cyclophosphamide
Explanations: Cyclophosphamide is metabolized by the liver's microsomal enzymes to hydroxy-cyclophosphamide. Target cells convert it to active phosphamide mustard and acrolein that is excreted in the urine. Acrolein binds to the bladder mucosa causing inflammation and possible ulceration. About 10% of patients develop microscopic hematuria. Patients should be monitored for red-tinged urine by dipstick. Cystoscopy may be needed and can show bleeding points, mucosal ulceration, hemorrhage, and necrosis. Go to the next page if you knew the correct answer, or click the link image(s) below to further research the concepts in this question (if desired).
Research Concepts: Cyclophosphamide
We update eBooks quarterly and Apps daily based on user feedback. Please tap flag to report any questions that need improvement.
Question 880:
A 60-year-old woman is taken to the operating room for a simple vulvectomy with sentinel lymph node biopsy secondary to biopsy confirmed invasive vulvar squamous cell carcinoma. During the procedure, the sentinel lymph node is identified and frozen sections of the sentinel lymph node and nearby lymph nodes are sent to pathology. The biopsy result confirms disease extension to three lymph nodes. Which of the following is the next best step in the management of this patient?
Choices: 1. Finish vulvectomy and start radiation therapy 2. Finish vulvectomy and start combination radiation and chemotherapy 3. Perform inguinofemoral lymph node dissection and start radiation and chemotherapy 4. Perform inguinofemoral lymph node dissection and initiate palliative care
Answer: 3 - Perform inguinofemoral lymph node dissection and start radiation and chemotherapy
Explanations: In this patient with 2 inguinal lymph nodes positive for spread of vulvar malignancy, it is recommended to proceed with completion of the inguinofemoral lymph node dissection and to start external beam radiation therapy with, or without, chemotherapy. Radiation with or without chemotherapy would be an option for positive sentinel lymph node biopsy, but it would not be appropriate due to 2 positive lymph nodes in this situation. Radiation is recommended for vulvar cancer as adjuvant therapy for histology-confirmed metastatic disease and as primary therapy for locally advanced disease followed by radical resection or the residual tumor. This patient without evidence of distal metastasis should be given chemotherapy and radiation treatment before considering the transition to palliative care. Go to the next page if you knew the correct answer, or click the link image(s) below to further research the concepts in this question (if desired).
Research Concepts: Vulva Cancer
We update eBooks quarterly and Apps daily based on user feedback. Please tap flag to report any questions that need improvement.
Question 881:
A 56-year-old male presents with a complaint of progressive behavioral changes and memory issues for the past one month. His past medical history is significant for diabetes. He is an accountant by profession but now has difficulty in performing simple arithmetic calculations and stays at home. He has also become quite aggressive and has involved in many arguments with his wife and those in the neighborhood. He denies smoking or the use of illicit drugs. He drinks alcohol occasionally. Cranial MRI reveals mild temporal T2-hyperintense signals. Electroencephalographic (EEG) findings include epileptiform activity, maximal in the temporal regions. Serum antibody testing detected anti-NMDA receptor antibodies. What is the most appropriate treatment of this condition?
Choices: 1. 2. 3. 4.
Acyclovir Rituximab Methylprednisolone Bortezomib
Answer: 3 - Methylprednisolone Explanations: The patient is having autoimmune limbic encephalitis. It is is a rare paraneoplastic syndrome that affects the medial temporal lobe and may be presented with cognitive dysfunction with a subacute beginning, change in the personality, seizure in partial and generalized type, irritability, hallucinations, disorientation, limbic paresis, and disruption of consciousness and short-term memory loss. Common first-line treatment for autoimmune encephalitis include corticosteroids, intravenous immune globulin (IVIG), and plasmapheresis. Some of the patients have an associated tumor but it not present in all cases. If a tumor is present, the treatment of an underlying tumor is first-line management. Rituximab and Bortezomib may be considered a secondline treatment options for autoimmune encephalitis with anti-NMDA antibodies. Go to the next page if you knew the correct answer, or click the link image(s) below to further research the concepts in this question (if desired).
Research Concepts: Paraneoplastic Limbic Encephalitis
We update eBooks quarterly and Apps daily based on user feedback. Please tap flag to report any questions that need improvement.
Question 882:
A 50-year-old man presents to the clinic with pain in his right lower limb for 3 months. There is no history of trauma, fever, or discharge. Physical examination shows leonine facies. The radiograph reveals mixed increased and decreased density in the tibia. Serum markers are normal except for high alkaline phosphatase levels. What is the most likely diagnosis?
Choices: 1. 2. 3. 4.
Osteosarcoma Paget disease Chondrosarcoma Osteoid osteoma
Answer: 2 - Paget disease Explanations: In the above vignette, leonine facies with a mixed pattern of disease on radiograph and high alkaline phosphatase levels point to Paget disease. Paget disease presents as bony pains, fractures, deformities, and sometimes high cardiac output failures. NSAIDs, bisphosphonates, and joint replacements are treatment opinions, depending upon the presentation. Osteosarcoma has a sunburst appearance or Codman triangle on the radiograph. Go to the next page if you knew the correct answer, or click the link image(s) below to further research the concepts in this question (if desired).
Research Concepts: Paget Disease
We update eBooks quarterly and Apps daily based on user feedback. Please tap flag to report any questions that need improvement.
Question 883:
A 39-year-old woman is admitted for the treatment of breast cancer. She was diagnosed with triple-negative, BRCA gene mutation-positive, right-sided breast cancer. She has undergone lumpectomy for cancer with level 2 axillary lymph node clearance. She is due to start on a combination of chemotherapy. Which among the following additional therapy is required in this patient?
Choices: 1. 2. 3. 4.
Vancomycin for 5 days Amoxicillin Filgrastim Amphotericin
Answer: 3 - Filgrastim Explanations: One of the adverse effects of chemotherapy is pancytopenia. Thus, the patient on combination chemotherapy is given an injection of filgrastim. Filgrastim, a granulocyte colony-stimulating factor (GCSF), is used to accelerate the production, maturation, migration, and cytotoxicity of the neutrophils. Agranulocytosis may be due to various medications, chemicals, autoimmune conditions, and infections. Chemotherapy is the leading cause of pancytopenia, including agranulocytosis in cancer patients. Prophylactic vancomycin, oral amoxicillin, and amphotericin are generally not added in the cancer chemotherapy regime without signs and symptoms of infection. Go to the next page if you knew the correct answer, or click the link image(s) below to further research the concepts in this question (if desired).
Research Concepts: Agranulocytosis
We update eBooks quarterly and Apps daily based on user feedback. Please tap flag to report any questions that need improvement.
Question 884:
A 65-year-old female has a 2-cm mass in her left breast. Fine-needle aspiration was done, but the results were inconclusive. Two months later, the patient returns with the same mass, which appears unchanged. What is the next step in the management of the patient?
Choices: 1. 2. 3. 4.
Continued observation Mammogram Repeat fine needle aspiration Core biopsy
Answer: 4 - Core biopsy Explanations: Confirmation of breast mass, especially in a 55-year-old, is mandatory. If the first fine needle aspiration was inconclusive, a core or open biopsy is warranted. A mammogram on a patient with a palpable mass is redundant. The test does not provide confirmation of malignancy. Observation of any breast mass in a 55-year-old is not the standard of care. Go to the next page if you knew the correct answer, or click the link image(s) below to further research the concepts in this question (if desired).
Research Concepts: Breast Abscess
We update eBooks quarterly and Apps daily based on user feedback. Please tap flag to report any questions that need improvement.
Question 885:
A 59-year-old woman was recently diagnosed with breast cancer. Her medical history includes hypertension, rate-controlled atrial fibrillation, and peripheral vascular disease. She presents for preoperative evaluation of a palliative mastectomy. Her functional capacity at baseline is 4 METs. Her most recent echocardiogram, which was performed during the last six months, revealed a 55 percent ejection fraction. She denies a history of orthopnea or nocturnal dyspnea. She has a heart rate of 95 bpm, a blood pressure of 135/88 mmHg, a respiratory rate of 16/min, and a 98% oxygen saturation on room air. Which of the following is the best course of action for her surgical procedure?
Choices: 1. Proceed with planned surgery under general anesthesia. 2. Order noninvasive cardiac testing. 3. Proceed with planned surgery using thoracic segmental spinal anesthesia. 4. Refer to a cardiology provider for further management.
Answer: 1 - Proceed with planned surgery under general anesthesia.
Explanations: Simple or toilet palliative mastectomy is a procedure having low inherent cardiac risk. Based on the American College of Cardiology/American Heart Association (ACC/AHA) guidelines on perioperative cardiovascular evaluation and care for noncardiac surgery, this patient should proceed with the planned procedure under general anesthesia. Important to note, this patient does not have any “active cardiac conditions,” as defined by the ACC/AHA guidelines. She does not have decompensated heart failure, unstable or severe angina, and her atrial fibrillation is not associated with an uncontrolled ventricular rate. The 2014 ACC/AHA guidelines recommend no further testing in low-risk patients without any active cardiac conditions. Go to the next page if you knew the correct answer, or click the link image(s) below to further research the concepts in this question (if desired).
Research Concepts: Perioperative Cardiac Management
We update eBooks quarterly and Apps daily based on user feedback. Please tap flag to report any questions that need improvement.
Question 886:
A 56-year-old female lawyer complains of worsening vision in both eyes over the last nine months. The patient has a history of hypertension and a family history of diabetes, heart disease, and lymphoma. She has a BMI of 18.9 and mentions that she has lost some weight recently. She states that she has developed occasional febrile episodes and easy fatigability. Diagnostic workup reveals serum protein of 10g/dl, the calcium of 12.5 mg/dl, hemoglobin of 11.1 g/dl, creatinine of 2.8 mg/dl, and M protein on SPEP. What is the most appropriate next step in the treatment of her symptoms?
Choices: 1. 2. 3. 4.
Corneal transplant Systemic chemotherapy Topical steroids Phototherapeutic keratectomy
Answer: 2 - Systemic chemotherapy Explanations: This patient is experiencing symptoms and history concerning for multiple myeloma with accompanying paraproteinemic keratopathy. The patient shows several signs and symptoms of multiple myeloma, including hypercalcemia, increased creatinine, anemia, bone pain, as well as B-type symptoms (fatigue and weight loss) and M proteins on SPEP. The mnemonic “CRAB” (hypercalcemia, renal dysfunction, anemia, and bone disease) helps remember key findings in this disease. While corneal transplant may eventually be considered for severe cases of paraproteinemic keratopathy, the transplant should be deferred until the systemic malignancy is treated. Stopping the underlying process and reducing systemic immunoglobulin levels are critical both for the cessation of ocular damage and the prevention of system-wide sequelae. Patients with paraproteinemic keratopathy have a high likelihood of recurrence, especially if immunoglobulin levels remain elevated. Systemic chemotherapy for the treatment of multiple myeloma is the correct answer. Not only is the lifethreatening malignancy the highest priority in treatment, but the elevated levels of immunoglobulin in the serum is likely the cause of the corneal pathology. Treatment of the systemic disease is critical and should be the initial step of treatment for paraproteinemic keratopathy. Paraproteinemic keratopathy a disease of corneal immunoglobulin deposition associated with multiple myeloma and other gammopathies. Topical steroids
have shown no recognizable benefit in the treatment of paraproteinemic keratopathy. Paraproteinemic keratopathy a disease of corneal immunoglobulin deposition associated with multiple myeloma and other gammopathies. Topical steroids have shown no recognizable benefit in the treatment of paraproteinemic keratopathy. Go to the next page if you knew the correct answer, or click the link image(s) below to further research the concepts in this question (if desired).
Research Concepts: Paraproteinemic Keratopathy
We update eBooks quarterly and Apps daily based on user feedback. Please tap flag to report any questions that need improvement.
Question 887:
A 75-year-old male with prostate cancer has suboptimally responded to management despite surgical castration. Oral anti-androgen therapy is being considered for him. The patient also gives a history of irritable bowel syndrome (IBS) with diarrhea and almost daily alcohol intake. His history includes epilepsy that started with generalized tonic-clonic seizures around three years back for which he is on oral phenytoin. Occupationally he has worked for 40 years in a glass manufacturing factory. His chest computed tomography scan and spirometry confirmed interstitial lung disease (ILD), although his breathing is well-controlled on a salmeterol - budesonide combination inhaler. Which of the following antiandrogens is preferred for his further medical management?
Choices: 1. 2. 3. 4.
Flutamide Nilutamide Bicalutamide Enzalutamide
Answer: 3 - Bicalutamide Explanations: All these agents, i.e., flutamide, nilutamide, bicalutamide, and enzalutamide are nonsteroidal antiandrogens that competitively bind androgen receptors throughout the body. They share certain adverse effects like hot flashes, decreased libido, and the propensity for hepatotoxicity. Bicalutamide has a lower potential for diarrhea and hepatotoxicity and no known interaction with alcohol (like nilutamide) or worsening of ILD or seizures. Enzalutamide's adverse effects include the induction of seizures, especially in those with a predisposition. Flutamide is the most hepatotoxic and has a much higher potential of causing diarrhea than other options. While nilutamide has a longer half-life and better adverse effect profile than flutamide, one of its significant adverse effects is alcohol intolerance, presenting as facial flushing, malaise, and hypotension. Go to the next page if you knew the correct answer, or click the link image(s) below to further research the concepts in this question (if desired).
Research Concepts: Flutamide
We update eBooks quarterly and Apps daily based on user feedback. Please tap flag to report any questions that need improvement.
Question 888:
A 12-year-old male being treated for Ewings sarcoma, known to have a translocation [t(11;22)] on tissue biopsy. He is being administered dose-dense chemotherapy comprising of vincristine, ifosfamide, and doxorubicin in the neoadjuvant setting. Which particular genotype predisposes to increased risk of vinca alkaloid toxicity?
Choices: 1. 2. 3. 4.
CYP CYP CYP CYP
3A4 2C19 3A5 2E1
Answer: 3 - CYP 3A5 Explanations: Vinca alkaloids are class of antitumor agents derived from vinca rosea, which mediate the inhibition of the mitotic spindle apparatus by binding to microtubules resulting in cell cycle mitotic arrest and cell apoptosis. Neurotoxicity, which is known to be cumulative and dose dependant, is the most common side effect of vincristine, which is seen in 35-45 percent of patients. Polymorphic CYP3A5 expression is known to predispose to vincristine-induced neurotoxicity. Expressers of CYP 3A5 experience less vincristine related neurotoxicity as compared to those who do not express this cytochrome. CYP 3A5 genotype causes increased vincristine exposure, leading to a higher incidence of neuropathy and severity. Vincristine induced neuropathy is known to be reversible, with mild sensory neuropathy resolving within a period of two months. Neurotoxicity has been known to occur with cumulative doses ranging from 2-6 mg/m2. It has also been shown to be related to single doses, with a dose of 4 mg associated with the development of acute neurotoxicity. A single dose usually does not exceed 2 mg, regardless of the body surface area, as higher doses have been associated with a higher incidence of neurotoxicity. Coasting, or the worsening of symptoms of peripheral neuropathy after cessation of chemotherapy, is seen in 30 percent of patients. Go to the next page if you knew the correct answer, or click the link image(s) below to further research the concepts in
this question (if desired).
Research Concepts: Vinca Alkaloid Toxicity
We update eBooks quarterly and Apps daily based on user feedback. Please tap flag to report any questions that need improvement.
Question 889:
A 57-year-old male with a past medical history of diabetes and hyperlipidemia presents with persistent abdominal pain, diarrhea, and progressive weight loss over several months. Physical examination showed blood pressure 154/90 mmHg, heart rate 89 beats/min, and temperature 37.2 C. The heart examination showed normal S1 and S2. Epigastric and right upper quadrant tenderness noted. Imaging showed a pancreatic mass and hepatic metastasis. A mitotic rate of more than 20 per 10 HPF found. The patient received medication, which improved the symptoms, but later on, the patient developed redness, swelling, and ulcers of the hands and feet. What of the following medications is most likely taken by the patient?
Choices: 1. 2. 3. 4.
Sunitinib Octreotide Everolimus Bevacizumab
Answer: 1 - Sunitinib Explanations: Sunitinib is a tyrosine kinase inhibitor that acts by inhibiting angiogenesis and affecting the PDGF receptor, and VEGF receptor types 1,2, and 3 expressed by the tumor cells. Toxicities of sunitinib include diarrhea, nausea, vomiting, hypertension, palmar-plantar erythrodysesthesia (handfoot syndrome), and hypothyroidism. Sunitinib is approved for the management of progressive pNETs. It improves progression-free survival compared to placebo. Hand-foot syndrome or so-called palmar-plantar erythrodysesthesia is a known side effect to sunitinib, which manifests by swelling, redness, and ulcers in both hands and feet. It is not usually caused by everolimus, octreotide, or bevacizumab. Go to the next page if you knew the correct answer, or click the link image(s) below to further research the concepts in this question (if desired).
Research Concepts: APUDoma
We update eBooks quarterly and Apps daily based on user feedback. Please tap flag to report any questions that need improvement.
Question 890:
A 16-year-old girl presents to the clinic with a sensation of epigastric fullness. On physical examination, she has conjunctival pallor and the tip of the spleen is palpable 3 cm below the left costal margin. Her labs are significant for hemoglobin 9.7 g/dL. MRI examination of the abdomen shows an anterior spleen tumor, suggestive of hemangioma. She undergoes an elective splenectomy. Histological examination of the specimen shows the tumor has clear boundaries without any fibrous capsule. Vascular channels with irregular lumen lined by cuboid-shaped endothelial cells and anastomosis resembling splenic sinusoids are seen. The endothelial cells also show hemophagocytosis. Which of the following findings on immunohistochemical staining is most likely to be seen in this case?
Choices: 1. 2. 3. 4.
CD21, CD5 Factor VIII, CD31, CD68 CD3, CD45, CD25 CD34
Answer: 2 - Factor VIII, CD31, CD68 Explanations: Littoral cell angioma can be differentiated from other splenic neoplasms only by histological and immunohistochemical staining analysis. Littoral cell angioma is characterized by the presence of dual differentiation patterns on immunohistochemical staining with both endothelial as well as histiocytic markers. The littoral cell angiomas commonly stain positive for endothelial markers like Factor VIII, CD31, von Willebrand factor (vWF), CD34, as well as histiocytic markers, eg. CD 68. Immunohistochemical staining positive only for endothelial factors (like CD34) goes in favor of a diagnosis of other splenic tumors. Go to the next page if you knew the correct answer, or click the link image(s) below to further research the concepts in this question (if desired).
Research Concepts: Littoral Cell Splenic Angioma
We update eBooks quarterly and Apps daily based on user feedback. Please tap flag to report any questions that need improvement.
Question 891:
A 50-year-old gentleman who is a high school teacher presents to the primary care physician with the complaint of headaches 2/10 and weight loss over a 2 month period. Further evaluation by imaging of the brain showed almost 20 lesions concerning for metastatic cancer. CT of the chest, abdomen, and pelvis showed multiple lesions in the liver. The biopsy of the lesion showed a BRAFwild type melanoma. The patient mentioned his preference to continue to teach as long as possible and would not prefer therapy that has effects on cognition. His performance status is excellent. Which of the following treatment would be recommended?
Choices: 1. 2. 3. 4.
Stereotactic radiosurgery Dabrafenib and trametinib Whole-brain radiotherapy Ipilimumab and nivolumab
Answer: 4 - Ipilimumab and nivolumab Explanations: Ipilimumab and nivolumab as a combination were studied in untreated brain metastases patients and showed a response rate of more than double compared to single-agent nivolumab. Intracranial complete responses were noted in about 17% of patients who were treated with ipilimumab and nivolumab. BRAF wild type tumors do not respond well to the dabrafenib and trametinib. Whole-brain radiotherapy causes a decrease in neurocognitive function and would not be a reasonable option for this patient. Stereotactic radiosurgery is not a practical choice for him with multiple lesions. Go to the next page if you knew the correct answer, or click the link image(s) below to further research the concepts in this question (if desired).
Research Concepts: Malignant Melanoma Metastatic To The Central Nervous System
We update eBooks quarterly and Apps daily based on user feedback. Please tap flag to report any questions that need improvement.
Question 892:
A 66-year-old man presents to the clinic accompanied by his daughter with a complaint of gradual onset of fatigue, anorexia, and constipation over the last five weeks. He also feels that he has been urinating more often and with a larger volume. Over the last two days, he has experienced some confusion and lethargy. She also adds that her dad has a nagging cough with some tinge of blood that has not improved with over-the-counter medications and has lost 10 kg during that interval. He has a history of hypertension and for which he takes hydrochlorothiazide. He has smoked three packs of cigarettes daily for 40 years and drinks socially. On physical examination, he appears somnolent and disoriented. His reflexes are reduced. Chest and cardiovascular examinations appear normal. Temporal wasting is noted. His serum calcium is 18.3 mg/dL, glucose 98 mg/dL, serum sodium 142 mEq/L, serum potassium 4.2 mEq/L. Which other metabolic abnormality do you expect in this patient, given the underlying diagnosis?
Choices: 1. 2. 3. 4.
Low phosphate levels High phosphate levels High sodium levels Increased serum creatinine
Answer: 1 - Low phosphate levels Explanations: This patient has elevated serum calcium levels, polyuria, neurologic symptoms, and a long history of smoking. The most likely diagnosis is humoral hypercalcemia of malignancy (HHM) from an underlying squamous cell carcinoma of the lung. PTHrP can be high in several malignancies, such as squamous cell carcinoma of the lung, clear type renal cell carcinoma, and even bladder cancer. Some cases of PTHrp have also been seen in breast and ovarian malignancies. Parathyroid hormone-related protein (PTHrP) is structurally similar to parathyroid hormone (PTH). It can act on the PTH-1 receptor, thus increasing sodium calcium levels via an increase in bone resorption. PTHrP can also lead to more calcium reabsorption in the distal renal tubule. Phosphate levels can drop with PTHrP as it can act on the proximal tubules and prevent phosphate reabsorption. Beta-HCG can be elevated secondary to large cell lung cancer. This can lead to gynecomastia in men. Go to the next page if you knew the correct answer, or click the link image(s) below to further research the concepts in this question (if desired).
Research Concepts: Paraneoplastic Syndromes
We update eBooks quarterly and Apps daily based on user feedback. Please tap flag to report any questions that need improvement.
Question 893:
A 45-year-old female patient with a history of chronic alcohol use disorder presents to the hospital with increasing abdominal distension. She noticed an increase in abdominal girth over six months. On examination, she has spider nevi, jaundiced sclera, and ascites. Her blood pressure is 100/60 mmHg, and her pulse rate is 88/min regular. Laboratory investigations reveal a hemoglobin of 10.5 g/dl, a mean cell volume of 110 fl, and CA-125 72 U/ml ( normal 35 U/ml). What is the most likely cause of her laboratory variation?
Choices: 1. 2. 3. 4.
Ovarian carcinoma Colonic carcinoma Hepatocellular carcinoma Cirrhosis
Answer: 4 - Cirrhosis Explanations: This patient appears to be most likely suffering from alcoholic cirrhosis. Several conditions including cirrhosis can lead to falsely raised CA-125 levels. AST (aspartate aminotransferase) is markedly raised as opposed to ALT (alanine aminotransferase) in alcoholic liver disease. There is hypoalbuminemia, hyperbilirubinemia, and hypertriglyceridemia. Also, GGT (gamma-glutamyl transpeptidase) is usually raised. Abdominal imaging (abdominal ultrasonography) is useful in looking for biliary obstruction and liver tumors. A liver biopsy can lead to a definitive diagnosis in cases where the diagnosis is uncertain. More often than not, it is used for evaluation of severity, prognosis, staging, and treatment monitoring. For an accurate diagnosis of fibrosis, at least a 1.5 to 2 cm long sample of liver tissue is necessary. Go to the next page if you knew the correct answer, or click the link image(s) below to further research the concepts in this question (if desired).
Research Concepts: Alcoholic Liver Disease
We update eBooks quarterly and Apps daily based on user feedback. Please tap flag to report any questions that
need improvement.
Question 894:
A 43-year-old man presents to the office with complaints of a rash for one month. The rash is present in his bilateral lower extremities in the form of purple/red macules and papules. There is no itching associated with the rash. He also reports anorexia and weight loss of 19 lbs (8.6 kg) in the last two months. He has had multiple sexual partners in the past, including men. He denies any fever, cough, lumps or bumps, or any contact history with animals. On examination, his vitals are stable. Chest and abdominal examinations are normal. Biopsy of the mass reveals spindle cell vascular proliferation in the dermis. What is the most likely causative agent of this condition?
Choices: 1. 2. 3. 4.
Human T-cell lymphotropic virus-1 (HTLV-1) Bartonella henselae Human herpesvirus-8 Epstein Barr virus
Answer: 3 - Human herpesvirus-8 Explanations: This patient with rash along with high-risk sexual behavior and biopsy finding is most likely suffering from Kaposi sarcoma. Human herpesvirus-8 is responsible for Kaposi sarcoma. The virus is usually acquired through saliva. It often presents in patients with HIV. Since this patient has high-risk sexual behavior, he should be further evaluated for HIV. Patients with HIV-related Kaposi sarcoma respond well to HAART, which can cause regression or complete treatment of their sarcoma. Bartonella henselae causes Bacillary angiomatosis, which is confused with Kaposi sarcoma. The patient should have a contact history with cat and lymphadenopathy. Go to the next page if you knew the correct answer, or click the link image(s) below to further research the concepts in this question (if desired).
Research Concepts: Kaposi Sarcoma
We update eBooks quarterly and Apps daily based on user feedback. Please tap flag to report any questions that need improvement.
Question 895:
A 43-year-old male patient with a past medical history significant for smoking and recently diagnosed renal cell carcinoma presents with waddling gait and severe bilateral hip pain. Radiographs show generalized osteopenia and regions of banded sclerosis (stress fractures) at the femoral necks. Which lab value is the most suggestive of a disease process with increased bone turnover?
Choices: 1. 2. 3. 4.
Hypercalcemia Hypocalciuria Elevated alkaline phosphatase (ALP) Hypophosphatemia
Answer: 3 - Elevated alkaline phosphatase (ALP) Explanations: Many biochemical markers, including bone ALP, are now available. They are specific and sensitive in assessing disease processes that involve the bone formation and resorption. In long-term osteomalacia, decreased urine calcium excretion can be seen, however, this is not necessarily suggestive of bone turnover. Serum calcium can be increased or decreased in diseases of increased bone turnover. Serum phosphate can be increased or decreased in diseases of increased bone turnover. In renal osteodystrophy, one would expect hyperphosphatemia. However, due to renal wasting in osteomalacia, hypophosphatemia is expected. Go to the next page if you knew the correct answer, or click the link image(s) below to further research the concepts in this question (if desired).
Research Concepts: Osteomalacia
We update eBooks quarterly and Apps daily based on user feedback. Please tap flag to report any questions that need improvement.
Question 896:
A 65-year-old male with a history of hypertension and type 2 diabetes presents with a 9-month history of pain in the right groin. X-ray and CT scan of the pelvis shows a large lytic lesion with endosteal scalloping, located in the right acetabulum. Imaging guided bone biopsy is performed, with features consistent with grade II conventional chondrosarcoma. Which of the following is the most favorable prognostic survival factor?
Choices: 1. 2. 3. 4.
Response to chemotherapy on resection specimen Pelvic location of the tumor Old age Tumor volume of less than 100 cm3
Answer: 4 - Tumor volume of less than 100 cm3 Explanations: Chondrosarcoma is primarily a disease of adults, most frequently diagnosed between the ages of 30 to 60 years. They are generally slow-growing tumors of intermediate malignancy, rarely metastasizing. Tumor volume of less than 100 cm3 has a 5-year survival of 92% in conventional chondrosarcoma, compared to 50% if tumor volume is more than 100 cm3. The treatment of choice for chondrosarcoma is primarily surgical resection, with tumors typically resistant to chemotherapy and radiotherapy. The histological grade is the most potent negative prognostic factor in chondrosarcoma. Other factors include metastatic disease at presentation, increased age, and pelvic tumor location. Go to the next page if you knew the correct answer, or click the link image(s) below to further research the concepts in this question (if desired).
Research Concepts: Primary Bone Cancer
We update eBooks quarterly and Apps daily based on user feedback. Please tap flag to report any questions that need improvement.
Question 897:
A 37-year-old female presented with persistent left upper abdominal quadrant (LUQ) discomfort for four months. She denied any change in bowel habits or weight loss. The physical examination revealed mild epigastric tenderness. Routine laboratory tests, including tumor marker levels, were within the normal range. Abdominal ultrasonography (US) reported a heterogeneous semisolid mass with cystic components. Abdominal computerized tomography (CT) revealed a well-defined 12×10×8 cm mass involving the pancreatic tail and was heterogeneous and abuts splenic vein. It was tentatively diagnosed solid papillary cystic tumor of the pancreas. What is the next step in the management?
Choices: 1. 2. 3. 4.
Systemic chemotherapy Surgery Locoregional Radiation Watchful waiting
Answer: 2 - Surgery Explanations: Solid papillary tumors of the pancreas are commonly benign, and chemotherapy is not indicated unless there is malignant transformation. SPEN stands for Solid and Papillary Epithelial Neoplasm (SPEN) of the pancreas. The tumor has also been referred with other names, including solid pseudopapillary tumor of the pancreas, solid pseudopapillary neoplasm. These are indolent and benign neoplasms and primarily managed by surgery that offers five-year survival of up to 95%. Solid papillary cystic tumors are not amenable for radiation. Primary treatment is surgery. Solid papillary cystic tumors of the pancreas are commonly benign and curative by surgery alone. Rarely they present as malignant. Therefore wait and watch is not the recommended strategy. Go to the next page if you knew the correct answer, or click the link image(s) below to further research the concepts in this question (if desired).
Research Concepts: Solid and Papillary Epithelial Neoplasm
We update eBooks quarterly and Apps daily based on user feedback. Please tap flag to report any questions that
need improvement.
Question 898:
A 67-year-old man recently diagnosed with metastatic lung cancer undergoes staging with a whole-body 18F-fluorodeoxyglucose positron emission tomography scan. There are hypermetabolic lesions both at the known primary site of malignancy within the lungs, as well as several mediastinal lymph nodes. There are no other areas of abnormal hypermetabolic activity identified outside of the chest. However, a pituitary mass is incidentally identified within the sella turcica. Which of the following is the next best step in the management of this patient?
Choices: 1. 2. 3. 4.
CT sella protocol with IV contrast MRI brain without and with intravenous contrast Transcranial ultrasound No follow-up imaging is required
Answer: 2 - MRI brain without and with intravenous contrast
Explanations: Pituitary mass lesions are occasionally discovered incidentally on staging 18F-FDG PET imaging studies. Due to the high metabolic uptake of the 18F-FDG radiotracer by the brain, mass lesions of the pituitary are incompletely evaluated on these studies. Furthermore, the CT images which are obtained during these imaging studies use a large slice thickness. Further evaluation should be performed ideally with an MRI of the brain with and without intravenous contrast. Because of this patient’s concurrent lung cancer, an MRI of the brain is also warranted to assess for metastatic disease. Evaluation of pituitary lesions is optimally performed using an MRI of the brain with and without intravenous contrast. CT protocols have been developed to evaluate pituitary lesions, but are both less sensitive and specific for pituitary lesions. Transcranial ultrasonography is a noninvasive tool that can be used to measure intracranial blood flow. This tool cannot provide images and cannot be used to image the pituitary region. Thus, transcranial ultrasound does not play a role in evaluating sellar masses. Metastatic disease infrequently involves the pituitary gland, but both breast cancer and lung cancers have been shown to have propensities for this area. Any patient with an underlying malignancy should receive a full imaging workup for pituitary lesions.
Go to the next page if you knew the correct answer, or click the link image(s) below to further research the concepts in this question (if desired).
Research Concepts: Pituitary Gland Imaging
We update eBooks quarterly and Apps daily based on user feedback. Please tap flag to report any questions that need improvement.
Question 899:
A 66-year-old male presents to the clinic for evaluation. He was diagnosed with advanced prostate cancer, for which he underwent radiotherapy. At the site of radiotherapy, he has developed a lesion. A biopsy of the lesion is performed, which on light microscopy shows atypical, pleomorphic spindle cells with abundant mitotic figures that extend through the deep dermis. It appears to have a sheet-like configuration. Which of the following complications of radiation therapy has most likely occurred in this patient?
Choices: 1. 2. 3. 4.
Osteosarcoma Fibrosarcoma Undifferentiated pleomorphic sarcoma Ewing sarcoma
Answer: 3 - Undifferentiated pleomorphic sarcoma Explanations: The undifferentiated/unclassified pleomorphic sarcoma (UPS), formerly known as malignant fibrous histiocytoma, is a high-grade aggressive neoplasm probably from mesenchymal or fibroblastic cells rather than histiocytes. It can affect soft tissues, bones, and retroperitoneum and metastasize to several organs. Undifferentiated/unclassified pleomorphic sarcoma (UPS) develops most commonly in sites of prior radiation in adults. UPS exhibits atypical, pleomorphic spindle cells with abundant mitotic figures extending through the deep dermis on light microscopy. The tumor may display a storiform, fascicular, or sheet-like configuration. Precisely distinguishing between superficial versus deep UPS may serve as a prognostic indicator. Visualization of deep-tissue invasion and lack of solar elastosis in UPS is useful to differentiate from atypical fibroxanthoma. A complete postoperative histopathology report should address the location of the tumor, depth of invasion, size, histologic grade, presence or absence of necrosis, excision margins status, vascular involvement, mitotic rate, type and extension of the inflammatory infiltrate, as well as the tumor, node, and metastasis (TNM) staging. Go to the next page if you knew the correct answer, or click the link image(s) below to further research the concepts in this question (if desired).
Research Concepts:
Adverse Effects Of Radiation Therapy
We update eBooks quarterly and Apps daily based on user feedback. Please tap flag to report any questions that need improvement.
Question 900:
A postmenopausal 68 year-old female, gravida 2, para 2. presents to the emergency department with the passage of clots, heavy vaginal bleeding, and feeling pre-syncopal. Her blood pressure was 86/55 mmHg, and pulse was 123 beats per minute. Her hemoglobin was noted to be 7.6 g/dl. She received 1 unit of packed red blood cells with symptomatic improvement. Further evaluation reveals she underwent menarche at the age of 11 and menopause at the age of 60. She is obese, but otherwise, her past medical, past surgical, family and social history are noncontributory. Pelvic examination was concerning for blood coming from the cervical os without any other masses or abnormalities. A pelvic ultrasound demonstrated at 7mm heterogenous endometrial stripe. What is the most common type of uterine cancer in a woman of this age-group?
Choices: 1. 2. 3. 4.
Serous carcinoma Papillary serous carcinoma Low-grade endometrioid carcinoma Clear cell carcinoma
Answer: 3 - Low-grade endometrioid carcinoma Explanations: Endometrioid carcinomas compose approximately 83% of uterine cancers. Most commonly, endometrioid cancers are low grade. More virulent serous and papillary serous carcinomas compose 4-6% of endometrial carcinomas and are considered high grade. High-grade carcinomas necessitate pelvic and para-aortic lymph node dissection as well as omentectomy. Clear cell carcinoma is another high-grade endometrial carcinoma, and it composes approximately 1-2% of endometrial cancers. It is essential to differentiate between type 1 and type 2 endometrial cancers for the management and prevention of these cancers. Low-grade endometrioid cancer is considered type 1 and is the most common type of endometrial cancer. Go to the next page if you knew the correct answer, or click the link image(s) below to further research the concepts in this question (if desired).
Research Concepts: Endometrial Cancer
We update eBooks quarterly and Apps daily based on user feedback. Please tap flag to report any questions that
need improvement.
Section 10 Question 901:
A 60-year-old African American male with a significant smoking history is hospitalized for abdominal pain and gross hematuria. He currently takes no medication. An abdominal single-phase CT scan with contrast shows a perfectly round, homogeneous, smoothlymarginated mass in the kidney. Additionally, on the computerized tomography density measurement, the mass is 90 Hounsfield units. What is the best next step in the management of this patient?
Choices: 1. This is a cyst, and no further workup is required. 2. Obtain delayed images and a noncontrast CT for further evaluation of tumor characteristics. 3. The lesion is probably malignant and requires a nephrectomy. 4. The lesion has malignant potential and requires a biopsy.
Answer: 2 - Obtain delayed images and a noncontrast CT for further evaluation of tumor characteristics.
Explanations: Uniform homogeneous masses >70 Hounsfield units (HU) in patients on nifurtimox-eflornithine combination therapy are benign high-density cysts. However, this patient does not have a history of this combination therapy. If a lesion is >70 HU but 110 HU on contrast-enhanced computed tomography, it is not possible to definitively differentiate between a high-density cyst that did not enhance and a non-enhancing renal cell carcinoma. A delayed contrast-enhanced CT is needed before any intervention. A noncontrast CT of the abdomen is also recommended. Uniform homogeneous masses >110 HU on contrastenhanced CT scans usually are malignant renal cell carcinomas. Go to the next page if you knew the correct answer, or click the link image(s) below to further research the concepts in this question (if desired).
Research Concepts: Renal Mass
We update eBooks quarterly and Apps daily based on user feedback. Please tap flag to report any questions that
need improvement.
Question 902:
Which of the following drugs can be used to treat chronic myelogenous leukemia?
Choices: 1. 2. 3. 4.
Adalimumab Tositumomab Bevacizumab Imatinib
Answer: 4 - Imatinib Explanations: Imatinib is used for Philadelphia chromosome-positive (Ph+) chronic myeloid leukemia (CML) in the chronic phase. Ph+ acute lymphoblastic leukemia can also be treated with this condition. Imatinib is an inhibitor of Bcr-Abl tyrosine kinase. This is the product of the Philadelphia chromosome in patients with CML, so inhibition blocks proliferation and causes apoptosis. Go to the next page if you knew the correct answer, or click the link image(s) below to further research the concepts in this question (if desired).
Research Concepts: Imatinib
We update eBooks quarterly and Apps daily based on user feedback. Please tap flag to report any questions that need improvement.
Question 903:
A 69-year-old man originally from India presents with a one-month history of sore throat and swallowing difficulty. He has a past medical history of hypertension, which is well controlled on lisinopril. The patient mentions that he has had the habit of chewing betel quid for the last 40 years; however, he does not smoke or drink alcohol. On examination, there is a 2.5 cm ulcer at the posterior third of the tongue and poor dental hygiene. Oral brush biopsy reveals dysplastic changes in the cells of the affected area. Which of the following is the treatment of choice for this patient's most likely condition?
Choices: 1. 2. 3. 4.
Cisplatin-based chemotherapy Transoral laser microsurgery Paclitaxel-based chemotherapy Radiotherapy
Answer: 2 - Transoral laser microsurgery Explanations: The sign and symptoms, along with the history of betel quid chewing in addition to the site of the ulcer, are highly suggestive of oropharyngeal squamous cell carcinoma. Surgery and radiotherapy are the two principle modalities used the treatment of the patients suffering from oropharyngeal cancers. The use of minimally invasive procedures, such as transoral laser microsurgery (TLM), as a first-line treatment modality for oropharyngeal carcinomas, has been found safe and efficient. Other resection methods include transoral robotic surgery, transoral video laryngoscopic surgery, transoral ultrasound surgery, and endoscopic laryngopharyngeal surgery. Go to the next page if you knew the correct answer, or click the link image(s) below to further research the concepts in this question (if desired).
Research Concepts: Oropharyngeal Squamous Cell Carcinoma
We update eBooks quarterly and Apps daily based on user feedback. Please tap flag to report any questions that need improvement.
Question 904:
A 60-year-old male presents with hematuria, back pain, and a left-sided flank mass. He has noted fatigue, weight loss, and a low-grade fever for the last few days. Last year, when he presented with the same complaints, a computed tomography (CT) scan with IV contrast was done, which showed a 3.4 cm complex cystic mass in the lower pole with multiple thin enhancing septations. Follow up CT scan with IV contrast today shows a 4.3 cm lesion with progressive septal thickening and nodular enhancement. Which of the following is the most likely management plan for this patient?
Choices: 1. Percutaneous radiofrequency ablation 2. Follow up dedicated kidney CT scan before and after IV contrast in 1 year 3. No further follow-up or imaging 4. Surgical removal with a partial nephrectomy
Answer: 4 - Surgical removal with a partial nephrectomy Explanations: The signs and symptoms in this patient are suggestive of renal cell carcinoma. Renal cell carcinomas (RCCs), which originate within the renal cortex, are responsible for 80% to 85% of all primary renal neoplasms. Symptoms may include hematuria, back pain, flank mass, fatigue, weight loss, anemia, fever, and/or a high serum calcium level. If RCC is suspected, renal and bladder ultrasound is often the first radiographic test. If the renal ultrasound shows a solid mass or a complex cyst with septations or nodules, the next test should be a dedicated CT scan of the kidneys, ureters, and bladder before and after IV contrast with delayed imaging of the entire abdomen and pelvis. This tumor is too large for optimal radiofrequency ablation and the patient appears fit enough for partial nephrectomy. Treatment depends on the stage of the tumor: For stage I renal cell cancer measuring less than 7 centimeters and confined to the kidney, nephrectomy or partial nephrectomy is the treatment of choice and is usually curative. Radiofrequency ablation or cryotherapy is an option in patients with bilateral tumors and small cortical tumors. Imaging surveillance is an option in elderly patients with a short life expectancy who are not good surgical candidates, as many renal cell cancers are slow-growing. For stage II renal cell cancer, laparoscopic or robotic radical nephrectomy is the treatment of choice. For stage III renal cell cancer, open radical nephrectomy is the standard of care. Adrenalectomy or extensive lymph node dissection is only recommended
when abdominal CT shows evidence of adrenal or lymph node invasion. Stage IV renal cell cancer is not curable. Treatment is generally palliative. Palliative nephrectomy with tumor spread does not appear to increase survival except in a few, rare circumstances where the metastases are limited. Go to the next page if you knew the correct answer, or click the link image(s) below to further research the concepts in this question (if desired).
Research Concepts: Renal Cell Cancer
We update eBooks quarterly and Apps daily based on user feedback. Please tap flag to report any questions that need improvement.
Question 905:
A 75-year-old man presents to the clinic with weight loss, early satiety, and an enlarged node in the left supraclavicular area. On physical examination, he is cachectic and has reduced muscle mass in his legs. Which of the following pathways is most commonly responsible for the reduced muscle mass in this patient's legs?
Choices: 1. 2. 3. 4.
Wnt signaling pathway JAK-STAT pathway Hedgehog signaling pathway cAMP-dependent pathway
Answer: 2 - JAK-STAT pathway Explanations: Weight loss, early satiety, and enlarged node in the left supraclavicular area are highly suggestive of gastric cancer in this patient. The pathway involved in cancerrelated muscle wasting is the JAK-STAT pathway. Janus kinases (JAKs) are a family of kinases that include JAK1, JAK2, JAK3, and tyrosine kinase 2 (TYK2). The JAKSTAT pathway is thought to be involved in cancerinduced muscle wasting via multiple different cytokines. Cytokines commonly involved in cancer-related muscle wasting include IL-1, IL-6, IL-8, and TNF-alfa. Evidence for the involvement of this pathway comes from a study evaluating ruxolitinib in pancreatic cancer, where patients treated with the drug gained greater than or equal to 5% of their body weight vs. only 2% in the placebo group. Go to the next page if you knew the correct answer, or click the link image(s) below to further research the concepts in this question (if desired).
Research Concepts: Anorexia and Cachexia
We update eBooks quarterly and Apps daily based on user feedback. Please tap flag to report any questions that need improvement.
Question 906:
A 41-year-old man presents to the clinic with a five-month history of low back pain and bilateral sciatica. Magnetic resonance imaging of the lumbar region demonstrates an intradural tumor with L4 to S2 extension. The patient undergoes L3–S3 laminoplasty. An intradural tumor highly adherent to the lumbosacral nerve roots is found and partially removed. Histologically, the tumor is composed of low-grade ependymal cells surrounding blood vessels, producing the papillary appearance and perivascular myxoid material between blood vessels and ependymal cells, creating the myxopapillary appearance. Which of the following sets of tests is most likely to be positive in this neoplasm?
Choices: 1. Glial fibrillary acidic protein (GFAP); CK19, CD30, and HMB45 2. Glial fibrillary acidic protein (GFAP); S-100, epithelial membrane antigen (EMA) and vimentin 3. CK7, desmin, P63 and myogenin 4. MDM2, chromogranin, p53, and synaptophysin
Answer: 2 - Glial fibrillary acidic protein (GFAP); S-100, epithelial membrane antigen (EMA) and vimentin
Explanations: In the present case, histological examination showed the typical features of myxopapillary ependymoma. Immunohistochemically, the tumor cells are usually positive for GFAP, S-100, and vimentin but are EMAnegative. The histological features of myxopapillary ependymoma are usually typical and characteristic. We use immunohistochemistry in some cases in order to exclude the differential diagnoses. The differential diagnosis of small myxopapillary ependymoma comprises paraganglioma and schwannoma. The differential diagnosis of large myxopapillary ependymoma that induces sacral destruction includes aneurysmal bone cyst, chordoma, and giant cell tumor. The immunohistochemical profile of myxopapillary ependymoma does not predict prognosis. When histology is typical, the immunohistochemical study is not mandatory. In case of doubt, it is useful to carry out an immunohistochemical study using the antibodies glial fibrillary acidic protein (GFAP), S-100, epithelial membrane antigen (EMA), and vimentin. Go to the next page if you knew the correct answer, or click the link image(s) below to further research the concepts in this question (if desired).
Research Concepts:
Myxopapillary Ependymoma
We update eBooks quarterly and Apps daily based on user feedback. Please tap flag to report any questions that need improvement.
Question 907:
A 67-year-old comes to the office with a right upper quadrant pain. He has a history of type 2 diabetes mellitus, hepatitis C, and hypertension. The patient also states that he has lost weight over the period of the last two months. The patient is vitally stable. Upon examination, he has slight tenderness in the right upper quadrant. An ultrasound is ordered, which raises suspicion of a hepatic tumor. After further testing, the patient undergoes the removal of the tumor, and the specimen is sent for the biopsy. What type of capillaries are likely to be seen on the specimen?
Choices: 1. 2. 3. 4.
Continuous capillaries Fenestrated capillaries Arteriovenous anastomoses Sinusoidal capillaries
Answer: 4 - Sinusoidal capillaries Explanations: Sinusoidal capillaries are also referred to as discontinuous capillaries. These capillaries have endothelial linings with multiple openings and are around 30 to 40 nm in diameter. These capillaries have no diaphragm and a non-existent basal lamina, allowing blood cells and proteins to pass through the capillary wall. Sinusoidal capillaries are mainly found between epithelial cells and hepatocytes of the liver. Sinusoids can also be seen in the spleen and lymph nodes. Fenestrated capillaries can be seen in kidneys, endocrine glands, and small intestine, whereas continuous capillaries are generally found in the nervous system. Go to the next page if you knew the correct answer, or click the link image(s) below to further research the concepts in this question (if desired).
Research Concepts: Histology, Capillary
We update eBooks quarterly and Apps daily based on user feedback. Please tap flag to report any questions that need improvement.
Question 908:
A 67-year-old male with a recent diagnosis of stage 3 non-small cell carcinoma presents to the emergency department with left leg swelling for the past 3 days. Venous duplex ultrasound is performed, and the patient is found to have a left lower limb deep venous thrombosis. What is the next step regarding the treatment of this deep venous thrombosis?
Choices: 1. 2. 3. 4.
Low molecular weight heparin Rivaroxaban Intravenous unfractionated heparin Inferior vena cava filter
Answer: 1 - Low molecular weight heparin Explanations: Patients presenting with venous thromboembolism (VTE) and malignancy are ideally treated with lower molecular weight heparin for immediate anticoagulation. Lower extremity DVT is the most common type of DVT in patients presenting with malignancy. Patients with no contraindications for low molecular weight heparin (LMWH) and no renal insufficiency are started on LMWH for immediate anticoagulation. Direct oral anticoagulants are preferred by some experts, with rivaroxaban or apixaban as an alternative. There is not enough data present for the use of direct oral anticoagulants (DOACs) in a setting where immediate anticoagulation is required. However, some experts use it. Unfractionated heparin can be used in patients with renal insufficiency. Go to the next page if you knew the correct answer, or click the link image(s) below to further research the concepts in this question (if desired).
Research Concepts: Hypercoagulability
We update eBooks quarterly and Apps daily based on user feedback. Please tap flag to report any questions that need improvement.
Question 909:
An 11-year-old boy is brought to the clinic for a swelling on his right shin. X-ray is suggestive of a tumor in the shaft of the tibia. MRI does not show soft tissue extension, and the differential diagnosis of osteofibrous dysplasia or admantinoma is considered. Which of the following is the next best step in the management of this patient?
Choices: 1. 2. 3. 4.
Repeat x-ray in six months Alendronate Bone biopsy Extraperiosteal resection
Answer: 3 - Bone biopsy Explanations: Surgical biopsy is needed for this patient. The primary management of OFD is conservative treatment. It is a benign lesion and asymptomatic. It is progressive and may lead to severe defects and lesions in bone and skin and need surgical intervention. However, if MRI suspects adamantinoma, which is more aggressive and need surgical excision, a biopsy should be done. Go to the next page if you knew the correct answer, or click the link image(s) below to further research the concepts in this question (if desired).
Research Concepts: Osteofibrous Dysplasia
We update eBooks quarterly and Apps daily based on user feedback. Please tap flag to report any questions that need improvement.
Question 910:
A 45-year-old male presents with a history of abdominal pain and diarrhea for the past three months. His symptoms are gradually and progressively getting worse. He describes the pain as "a dull aching type." His stools are watery, without blood or mucus. He reports a 16 kg weight loss during this period. He denies any history of fever or per-rectal bleeding. He recalls occasional episodes of vomiting soon after ingestion of food in the past. He is an avid football player, but he quickly gets tired, and therefore now plays less often. His past medical history is significant for the diagnosis of malabsorption in childhood, for which he was started on a gluten-free diet. He responded well to these dietary modifications and never had any issues until now. He works as a lawyer and has never used tobacco, alcohol, or recreational drugs. He is in a monogamous relationship with his wife. He currently takes no medications, and he has no known drug allergies. Physical examination reveals an emaciated man with pallor. Which of the following is the most likely new diagnosis?
Choices: 1. 2. 3. 4.
Whipple disease Chronic nonalcoholic pancreatitis Lactase deficiency Intestinal lymphoma
Answer: 4 - Intestinal lymphoma Explanations: Celiac disease usually presents with symptoms of malabsorption, such as voluminous and malodorous stools. There may be associated abdominal pain and bloating. With time, patients may develop either adenocarcinoma or lymphoma of the small bowel. Although Whipple disease and chronic pancreatitis are associated with similar symptoms, this patient's history of celiac sprue makes intestinal lymphoma the more likely diagnosis. Lactase deficiency does not usually cause weight loss or fatigue. Intestinal lymphoma must be suspected in any patient with a history of celiac disease who presents with gastrointestinal symptoms despite complying with a gluten-free diet. Go to the next page if you knew the correct answer, or click the link image(s) below to further research the concepts in this question (if desired).
Research Concepts: Celiac Disease
We update eBooks quarterly and Apps daily based on user feedback. Please tap flag to report any questions that need improvement.
Question 911:
A 55-year-old male patient presented with a 3-week history of poor appetite, intermittent fever, and fatigue. His physical exam is significant for multiple bilateral tender cervical lymphadenopathies. Pertinent laboratory findings showed leukocytosis: 80,800/mm3, hypercalcemia:17 mg/dL, elevated ionized calcium:11 mg/dL, serum phosphorus:2.5 mg/dL, potassium level:4 mEq/L, creatinine: 1 mg/dL, uric acid: 4.8 mg/dL, elevated PTH-related protein: 50 pg/mL, HIV antibody test: negative. A chest radiograph was significant for diffuse lytic lesions in ribs. CT of the chest, abdomen, and pelvis showed massive right axillary adenopathy and mesenteric nodes. Peripheral blood smear showed large flower cells with polylobated nuclei. The bone marrow biopsy finding was consistent with the peripheral blood smear. Flow cytometry analysis of bone marrow showed 18% aberrant T-cells CD3+, CD4+, CD5+, CD25+, CD7-, CD8-, CD30-. HTLV-1 antibody was positive by enzyme-linked immunosorbent assay (ELISA)and Western Blot. The diagnosis was made, and he was started on doseadjusted EPOCH (etoposide, prednisone, vincristine, cyclophosphamide, doxorubicin). After 24 hours of treatment with chemotherapy, he was confused with decreased urine output. Laboratory findings showed serum phosphorus: 5.8 mg/dL, potassium: 6 mg/dL, creatinine level: 5.5 mg/dL, uric acid: 9 mg/dL. Which of the following is the next best step in management?
Choices: 1. 2. 3. 4.
Allopurinol Aggressive intravenous hydration Hemodialysis Peritoneal dialysis
Answer: 2 - Aggressive intravenous hydration Explanations: Tumor lysis syndrome (TLS) could be a lethal complication from anti-neoplastic treatment. Thus, prevention is a crucial strategy to avoid TLS during anticancer therapy. This patient is suffering from tumor lysis syndrome with elevated uric acid: 8.0 mg/dL, potassium: 6.0 mmol/L, phosphorus: 4.5 mg/dL, and creatinine: 1.5 times the upper limit of normal. The most appropriate next best step is to initiate aggressive intravenous fluids immediately that helps dilute any toxins in the blood and increase the urine output. Dialysis is not indicated for this patient. Dialysis should be considered for patients with worsening renal function despite optimal medical intervention. It should also be considered for patients who have failed to have sufficient diuresis after aggressive fluids and or the use of loop diuretics. Hyperkalemia could cause lethal cardiac arrhythmias if it is not treated rapidly and aggressively. Administration of 50% dextrose in water with ten units of regular insulin IV is the treatment of choice. If hyperkalemia is not responding to treatment or serum level is greater than 6 mEq/L, hemodialysis is indicated. Hemodialysis may also be necessary if serum creatinine and phosphorus level are greater than 10 mEq/L, and or serum uric acid level is greater than 10 mEq/L. Allopurinol does not remove the preexisting uric acid in the body. It reduces the production of new uric acid. A urinary catheter may be placed for strict monitoring of intake and output. However, the initial best next step, in this case, would be aggressive hydration.
Go to the next page if you knew the correct answer, or click the link image(s) below to further research the concepts in this question (if desired).
Research Concepts: Adult T Cell Leukemia
We update eBooks quarterly and Apps daily based on user feedback. Please tap flag to report any questions that need improvement.
Question 912:
A 65-year-old man with a past medical history of COPD, hypertension, and sleep apnea presents to the clinic with complaints of excessive fatigue limiting his daily activity, increasing dyspnea, a 15-pound (7 kg) unintentional weight loss over the last 6 months, and an episode of hemoptysis which occurred one day before presentation to the clinic. He has poor follow-up with the clinic and is generally nonadherent with prescriptions. He smokes and has a 45 pack-year smoking history. CT scan of the chest demonstrates a 6 cm irregular mass with spiculated borders in the right upper lobe. A small portion of the liver is caught on the chest imaging, which demonstrates multiple suspicious lesions. A positron emission tomography (PET) scan shows widespread lesions in the liver with increased uptake in addition to increased uptake in the lung mass. A biopsy of the lung mass is performed, which reveals squamous cell carcinoma of the lung. Testing for gene mutations is negative. The patient is given cisplatin, gemcitabine, and a monoclonal antibody. Which of the following best classifies the origin of the appropriate monoclonal antibody, as per the basis of the suffix utilized in the nomenclature?
Choices: 1. 2. 3. 4.
Murine Humanized Human Chimeric
Answer: 2 - Humanized Explanations: The nomenclature of mAbs depends on the origin of each respective mAb. Common suffixes include -omab, ximab, -zumab, and -umab, which represent murine, chimeric, humanized, and human agents, respectively. Bevacizumab, a VEGF-A inhibitor, is a humanized monoclonal antibody as suggested by the suffix "zumab". Inhibition of VEGF-A (as by bevacizumab) inhibits the neovascularization associated with malignancies. It can help decrease tumor burden through a variety of mechanisms, including inhibition of neovascularization, regression of recent vasculature, and alteration of blood vessel function. Possible symptoms raising the possibility of malignancy including excessive fatigue, increasing dyspnea, unintentional weight loss, and hemoptysis in patients with a compatible history (for example, family history of cancers, significant smoking history, etc.). Go to the next page if you knew the correct answer, or click the link image(s) below to further research the concepts in this question (if desired).
Research Concepts: Understanding How Monoclonal Antibodies Work
We update eBooks quarterly and Apps daily based on user feedback. Please tap flag to report any questions that need improvement.
Question 913:
A 61-year-old female has been followed for the last few months for progressive muscle stiffness and weight loss. Her son reports that it first started in both of her arms and now is more generalized and limiting her mobility that she is almost chair bound. He further mentions that whenever there is a loud sound around the house, she gets these “spasms,” and he tries to calm her down. She has not seen a provider for more than ten years before that, and there is no known family history of Parkinsonism. Later on, she was diagnosed with Stiff-person syndrome, and a paraneoplastic panel was sent to rule minor secondary causes. If the expected antibody from the panel comes back positive, which of the following would be more expected?
Choices: 1. 2. 3. 4.
Thymoma Cervical cancer Lymphoma Breast cancer
Answer: 4 - Breast cancer Explanations: It is important to consider common malignancies in patients who are not following regular age-appropriate screening guidelines. Paraneoplastic Stiff-Person syndrome is associated mainly with anti-amphiphysin. Anti-amphiphysin antibodies are mainly associated with small cell lung cancer and breast cancer. Thymoma, however, is associated with anti-CRMP5, while Hodgin's lymphoma is associated with anti-Tr antibodies. Go to the next page if you knew the correct answer, or click the link image(s) below to further research the concepts in this question (if desired).
Research Concepts: Paraneoplastic Encephalomyelitis
We update eBooks quarterly and Apps daily based on user feedback. Please tap flag to report any questions that need improvement.
Question 914:
A 25-year-old man presents to the clinic with a 6-months history of right knee pain. He reports that there has been a gradual onset. Physical exam is remarkable for swelling and pain on palpation of the right knee. X-rays demonstrate a lytic lesion with nonsclerotic margins. Pathology reveals elongated, oval, or polygonal mononuclear cells. Which of the following is the most likely location of this lesion in this patient population?
Choices: 1. 2. 3. 4.
Epiphysis Physis Diaphysis Metaphysis
Answer: 4 - Metaphysis Explanations: Giant cell tumor of bone (GCTB) typically arises in the metaphysis and extends into the epiphysis of long bones. Imaging of the primary site with plain films and CT is recommended. Establishing a diagnosis of GCTB is difficult, oftentimes requiring a biopsy in addition to imaging. Histopathology of a GCTB is composed of elongated, oval or polygonal mononuclear cells with a typical uniform distribution. GCTB is often a low-grade tumor even when they appear aggressive on imaging. Go to the next page if you knew the correct answer, or click the link image(s) below to further research the concepts in this question (if desired).
Research Concepts: Osteoclastoma
We update eBooks quarterly and Apps daily based on user feedback. Please tap flag to report any questions that need improvement.
Question 915:
A 55-year-old woman presents with a 2.1 cm predominantly ground glass lung nodule in the right upper lobe (RUL) that has grown in size from 1.0 cm compared to a CT scan done 6 months ago. She undergoes a CT-guided transthoracic core needle biopsy, complicated by iatrogenic alveolar hemorrhage. The histopathology shows atypical cellular proliferation replacing normal alveolar epithelia with nuclear and nucleolar prominence. There is no disruption of the alveolar basement membrane. Immunostaining is positive for thyroid transcription factor-1 (TTF-1), cytokeratin-7 (CK-7), and negative for cytokeratin20 (CK-20). She also has a 1.2 cm pure ground glass nodule in the right lower lobe (RLL), which was only 3 mm on a prior CT scan. For an appropriate treatment decision, a biopsy of the RLL nodule is offered, but she is traumatized by the previous complication and wants to know the physician's expert opinion about the nature of the second lesion. What is the most likely etiology of the second lesion?
Choices: 1. 2. 3. 4.
It It It It
is is is is
a a a a
metastatic lesion through aerogenous spread. synchronous malignant lesion. metastatic lesion through lymphatic spread. metastatic lesion through vascular spread.
Answer: 2 - It is a synchronous malignant lesion. Explanations: The histology description from the right upper lobe (RUL) lesion fits the criteria for noninvasive adenocarcinoma with lepidic growth. Immunostaining is suggestive of a non-mucinous subtype. Adenocarcinoma with lepidic growth rarely metastasizes through vascular or lymphatic invasion. Therefore, multiple lesions are either through aerogenous spread or are synchronous lesions. Skip lesions through aerogenous spread is a common finding. The presence of mucinous subtype or significant solid component is more indicative of aerogenous metastasis than a synchronous disease. Consolidation tumor ratio, defined as a ratio of the maximum diameter of the area of consolidation to the tumor, has also been used as a marker for metastasis and prognosis in non-solid adenocarcinomas with lepidic growth. Multiple synchronous lesions can be found on CT. Go to the next page if you knew the correct answer, or click the link image(s) below to further research the concepts in this question (if desired).
Research Concepts: Bronchoalveolar Cancer
We update eBooks quarterly and Apps daily based on user feedback. Please tap flag to report any questions that need improvement.
Question 916:
A 65-year-old man presents to the clinic with a left-sided hearing loss. He has a 45 pack-year history of smoking. Tuning fork testing reveals the hearing impairment is conductive. The otoscopic exam is suggestive of a dull retracted left tympanic membrane with a clear fluid-level behind it. Examination of which of the following structures is most likely to reveal the lesion in this patient?
Choices: 1. 2. 3. 4.
Nasal vestibule Larynx Oral cavity Nasopharynx
Answer: 4 - Nasopharynx Explanations: A dull retracted tympanic membrane with a fluid level behind it is diagnostic of otitis media with effusion (OME). A tympanogram, if performed, would produce a 'type B' trace. Otitis media with effusion (OME) in adults can be secondary to nasopharyngeal carcinoma. The lesion can disrupt the function of the Eustachian tube, leading to poor ventilation of the middle ear and subsequent otitis media. Examination of the nasopharynx is most commonly performed with flexible nasendoscopy. Any lesion found in the nasopharynx would subsequently usually require a biopsy. Risk factors for nasopharyngeal carcinoma include male sex, Chinese or Southeastern ethnicity, advanced age, consumption of salt-cured foods, Epstein-Barr virus infection, smoking, and excess alcohol consumption. Go to the next page if you knew the correct answer, or click the link image(s) below to further research the concepts in this question (if desired).
Research Concepts: Ear Examination
We update eBooks quarterly and Apps daily based on user feedback. Please tap flag to report any questions that need improvement.
Question 917:
A 65-year-old woman presents to the clinic for follow up. She has a history of a resected gastrointestinal stromal tumor of the stomach (GIST) and has been using imatinib 400 mg orally daily for 2 months. Since starting the drug, she has had multiple, debilitating side effects such as muscle cramping, fatigue, and peripheral edema. She would like to stop the treatment. What is the most appropriate recommendation for this patient?
Choices: 1. 2. 3. 4.
Stop the treatment Decrease the dose Shift to another tyrosine kinase inhibitor Reassurance that these symptoms are not from the drug
Answer: 2 - Decrease the dose Explanations: Some physicians will stop the treatment if the patient’s lifestyle is affected severely. Studies have shown that decreasing the dose may alleviate severe symptoms. A trial dose of 200 mg per day may be used. One of the other tyrosine kinase inhibitors like sunitinib may be used. Substitution is usually advised if imatinib therapy fails. In this case, the symptoms could be controlled by lowering the dose. There is no evidence of treatment failure yet to warrant shifting to another TKI. Symptoms will continue or even worsen if that dose is maintained. Go to the next page if you knew the correct answer, or click the link image(s) below to further research the concepts in this question (if desired).
Research Concepts: Tyrosine Kinase Inhibitors
We update eBooks quarterly and Apps daily based on user feedback. Please tap flag to report any questions that need improvement.
Question 918:
A 26-years-old female presents to her primary care provider complaining of chronic cough and nasal voice. She recently moved to the United States from China. She was diagnosed with a postnasal drip four months ago. She denies any recent respiratory infections, but she reports tinnitus that is present most of the time. She denies fever, shortness of breath, chest pain, or blood in sputum. She has a sedentary lifestyle and eats a lot of smoked food. She smokes more than two packs of cigarettes daily. Her respiratory rate is 14/minute, the pulse is 96/minute, blood pressure is 130/90 mmHg, the temperature is 37.1 C (98.78 F), and oxygen saturation is 98%. On Auscultation, lungs are clear with vesicular sounds bilaterally, the heart shows no murmurs with normal S1 and S2 sounds. A nasal endoscopic investigation is done, and a mass in the nasopharynx area was discovered, a biopsy was taken then for more investigation and evaluation. If this tumor metastasizes intracranially, which of the following will be most likely to be affected?
Choices: 1. 2. 3. 4.
Cranial Cranial Cranial Cranial
nerve nerve nerve nerve
II IV X VI
Answer: 4 - Cranial nerve VI Explanations: Giving the patient's ethnicity and presentation, along with the history of heavy smoking and consuming a lot of smoked food, raise the suspicion of nasopharyngeal carcinoma (NPC). Patients with NPC can have variable presentations depending on the area of involvement, nasal symptoms ranging from nasal obstruction, blood-tinged nasal discharge, and post-nasal drip to denasalization of voice and cacosmia. The intracranial extension is prevalent among 8% to 12% of the demographic various forms of cranial nerve involvement present with the associated symptom. The most commonly involved cranial nerve is the abducens nerve. Go to the next page if you knew the correct answer, or click the link image(s) below to further research the concepts in this question (if desired).
Research Concepts: Nasopharyngeal Carcinoma (NPC, Lymphoepithelioma)
We update eBooks quarterly and Apps daily based on user feedback. Please tap flag to report any questions that need improvement.
Question 919:
A 58-year-old woman presents to the hospital with increasingly frequent headaches waking her from sleep for the past six months. She also reports occasional dizzy episodes lasting 30 minutes at a time, along with tinnitus in the left ear. CT brain shows a heterogenous 2 cm mass lateral to the left putamen. MRI with contrast shows a peripherally enhancing mass deep to the temporal operculum. Surgical resection is recommended alongside adjuvant chemoradiation. Which of the following preoperative tests is most likely to reduce neurological morbidity associated with the procedure?
Choices: 1. 2. 3. 4.
Plain film skull radiograph Electroencephalogram (EEG) Functional magnetic resonance imaging Visual field perimetry
Answer: 3 - Functional magnetic resonance imaging Explanations: This patient is likely to harbor an insular malignant glioma based on their age, clinical exam, and radiographic features. Final WHO 2016 pathologic and molecular diagnosis can only be made after surgical resection or biopsy. For suspected malignant gliomas in adults, maximal safe resection + chemoradiation (Stupp protocol) is generally first-line therapy. Although surgical resection is preferred, insular gliomas are particularly challenging to access given their location, delicate vasculature, and adjacent eloquent cortex. Given this patient's likely Berger-Sinai Zone II/III insular glioma, a transcortical approach may be preferred. However, significant care should be taken to preserve the language and auditory structures of the superior temporal gyrus, including Wernicke's area (Brodmann area 22) and the auditory cortex (Brodmann areas 41 & 42). There is also a concern for the corticospinal tract, basal ganglia, and various fasciculi, which wrap around the insula. Although population-level anatomy and physiology may not differ significantly, functional MRI (fMRI) measures cerebral activity based on blood flow and can help to determine where eloquent areas of the brain lie for a particular patient. This is especially vital for lesions like insular gliomas, where concern for numerous white and grey matter structures will govern management. Tractography is also beneficial in this context. The other preoperative tests may be useful but are not as important as fMRI and similar tests.
Go to the next page if you knew the correct answer, or click the link image(s) below to further research the concepts in this question (if desired).
Research Concepts: Insular Cortex
We update eBooks quarterly and Apps daily based on user feedback. Please tap flag to report any questions that need improvement.
Question 920:
A 33-year-old woman is scheduled for islet cell transplantation. Before the procedure, the patient discusses the risks and benefits of the surgery with the surgeon. She asks the surgeon if her disease could be cured, and if there is a chance, she will never have to use exogenous insulin again. The surgeon informs the patient that patients experience varying levels of graft survival and function. Which one of the mentioned techniques would give this patient the best chance at achieving insulin independence?
Choices: 1. Letting the graft sit in Hank solution at least 72 hours after harvest before infusion 2. Infusing the patient with grafts pooled from multiple donors 3. Placing the patient on antibiotics postoperatively 4. Bathing the patient's pancreas in a digestive enzyme solution to assist new islet cell graft incorporating into the patient's pancreas
Answer: 2 - Infusing the patient with grafts pooled from multiple donors
Explanations: Only a portion of islet cells infused will take to the recipient; many of the cells injected do not survive. Islet cells are pooled from multiple donors to create a graft with an adequate number of cells. Because the majority of cells that are infused do not survive, harvesting cells from a single donor will not produce a functional graft. Grafts must be infused within 72 hours of harvesting to ensure graft survival. Go to the next page if you knew the correct answer, or click the link image(s) below to further research the concepts in this question (if desired).
Research Concepts: Islets Transplantation
We update eBooks quarterly and Apps daily based on user feedback. Please tap flag to report any questions that need improvement.
Question 921:
A 65-year-old man has metastatic prostate cancer. He has refused all types of treatment. Routine blood work reveals that his hematocrit has been dropping, and a smear reveals normochromic, anisocytosis, poikilocytosis, a few nucleated red blood cells, and immature myeloid cells. What has he developed?
Choices: 1. 2. 3. 4.
Coombs hemolytic anemia Hereditary spherocytosis Myelophthisic anemia Vitamin B12 deficiency
Answer: 3 - Myelophthisic anemia Explanations: Myelophthisic anemia is caused by the replacement of marrow by other constituents, either abnormal hematopoietic cells or non-hematopoietic cells. These, in effect, "push out" the normal marrow constituents leading to anemia. The fragmented and distorted cells seen in anemias due to this type of process are the result of trauma to erythrocytes going through distorted and compressed capillary beds. If the underlying disease process is myelofibrosis (not very common) or a hereditary storage disease (rare), splenomegaly (sometimes massive) related to either extramedullary hematopoiesis or the underlying disease process may be present. When the underlying disease process is end-stage cancer (common) or granulomatous disease (less common), splenomegaly is usually not seen because little extramedullary hematopoiesis occurs. Treatment involves management of the underlying disease, if possible, with transfusions, if the anemia produces cardiovascular symptoms. Erythropoietin is often tried in these settings but typically provides only a modest response since the underlying problem is a lack of marrow room rather than stagnant erythrocyte development. Go to the next page if you knew the correct answer, or click the link image(s) below to further research the concepts in this question (if desired).
Research Concepts: Myelophthisic Anemia
We update eBooks quarterly and Apps daily based on user feedback. Please tap flag to report any questions that need improvement.
Question 922:
A 48-year-old female has undergone bronchoscopy for a biopsy of a 2 cm central lesion of the bronchus identified on computed tomography. The lesion is observed to be 2 cm, round, and symmetrical lesion. After a biopsy of the lesion, her blood pressure increases to 210/108 mm Hg, the temperature is 100.7 degrees Fahrenheit, and the heart rate is 166 beats per minute. Facial flushing is noted on the physical exam. Periprocedural treatment with which of the following would be beneficial to the patient?
Choices: 1. 2. 3. 4.
Corticosteroid therapy Metoprolol tartrate Somatostatin analog Acetaminophen
Answer: 3 - Somatostatin analog Explanations: This patient is presenting with carcinoid syndrome as a result of a bronchial carcinoid tumor biopsy. Although rare, when a biopsy is performed, neuroamines from the tumor can enter systemically into the body, creating carcinoid syndrome. Somatostatin analog use can stabilize and decrease tumor mass. The use of somatostatin analogs also provides symptomatic relief. Go to the next page if you knew the correct answer, or click the link image(s) below to further research the concepts in this question (if desired).
Research Concepts: Bronchial Carcinoid Tumors
We update eBooks quarterly and Apps daily based on user feedback. Please tap flag to report any questions that need improvement.
Question 923:
A 52-year-old woman comes to the clinic for assessment after medical testing for insurance applications revealed elevated liver enzymes. She has not seen a physician for several years. Upon asking, the patient admits that she has noticed some change in the color of stool and urine, but other than that, she has no issue to report. The patient has not traveled recently. She smokes a pack of cigarettes and drinks 2-3 glasses of wine daily. Temperature is 37.2 C (99 F), blood pressure is 130/90 mm Hg, and pulse rate is 87 beats per minute. Body mass index (BMI) is 21 kg/m2. On physical examination, the patient appears tired; there is mild hepatomegaly but no splenomegaly on abdominal palpation. Bowel sounds are normal. Abdominal ultrasound reveals increased echogenicity of the liver without any masses. Laboratory results show: hemoglobin: 12 g/dL, total white cells count: 7500/mm3, platelets count: 280,000/mm3, albumin: 4 g/dl, total bilirubin: 12.5 g/dl, alanine aminotransferase (ALT, SGPT): 124 U/L, aspartate aminotransferase (AST, SGOT): 119 U/L, alkaline phosphatase (ALP): 120 U/L, gammaglutamyltransferase (GGT): 100 U/L, hepatitis A virus antibody: negative, hepatitis B surface antibody: positive, hepatitis B surface antigen: negative, hepatitis C virus antibody: positive, hepatitis C virus RNA: 1,988,655 IU/mL. Which of the following is the best next step?
Choices: 1. 2. 3. 4.
Exploratory laparotomy Endoscopic retrograde cholangiopancreatography Abdominal X-ray Hepatitis B vaccination
Answer: 2 - Endoscopic retrograde cholangiopancreatography
Explanations: Hepatitis B and C virus infections leading to cirrhosis are documented risk factors for the development of intrahepatic cholangiocarcinoma, which is evidenced by the symptoms of obstructive jaundice in this patient, in the setting of hepatitis C virus RNA load and ultrasound findings. Given the findings on ultrasound and laboratory results, the next best step would be performing either an endoscopic retrograde cholangiopancreatography (ERCP) or magnetic resonance cholangiopancreatography (MRCP) to find out the cause of obstructive jaundice. Endoscopic retrograde cholangiopancreatography is an invasive procedure which can be used for both diagnostic and therapeutic purposes, such as, taking of biopsy sample and placing a stent to relieve the cholestatic symptoms, respectively. The urine becomes dark, and the stool becomes light (acholic) as a result of obstruction to the bile flow, which occurs in people with biliary tract carcinoma, as the mass obstructs the bile flow. Ultrasound and CT scans are considered primary image modalities for diagnosis. Still, direct visualization of the tumor is often necessary, which is carried out with either an invasive procedure as endoscopic retrograde cholangiopancreatography or non-invasive magnetic resonance cholangiopancreatography.
Go to the next page if you knew the correct answer, or click the link image(s) below to further research the concepts in this question (if desired).
Research Concepts: Biliary Tract Cancer
We update eBooks quarterly and Apps daily based on user feedback. Please tap flag to report any questions that need improvement.
Question 924:
A 50-year-old woman presents to the clinic with complaints of low-grade fever, night sweats, and increased urination over the past ten days. She also has mild constipation, decreased appetite, and bone pain. She further states that she is irritated easily and cannot focus on her work. Her past medical history is significant for diabetes mellitus for 15 years, well-controlled with medications. She visits her clinician regularly, and her blood sugar is under control. Her temperature is 36.9 C, her blood pressure is 135/70 mmHg, and her heart rate is 85/min. On physical examination, a mobile mass is palpable in the left mandible with a palpable lymph node. Current laboratory results reveal glucose 119 mg/dL, HbA1c 6.2%, serum calcium 19.5 mg/dL, urine calcium 275 mg, serum creatinine 1.3 mg/dL, and serum sodium 141 mEq/L. What is the most likely cause of her symptoms?
Choices: 1. High level of parathyroid hormone 2. High level of parathyroid hormone-related protein (PTHrP) 3. High level of 1,25-dihydroxy vitamin D 4. Mutations in the calcium-sensing receptor in parathyroid and kidney
Answer: 2 - High level of parathyroid hormone-related protein (PTHrP)
Explanations: Hypercalcemia occurs in 20% to 30% of patients with cancer. Cancer represents the most common etiology of hypercalcemia in the inpatient setting. Malignant-associated hypercalcemia (MAH) is classified into four groups: humoral hypercalcemia of malignancy (HHM), local osteolytic hypercalcemia (LOH), excess 1,25-dihydroxy vitamin D, and ectopic parathyroid hormone (PTH) secretion. Humoral hypercalcemia of malignancy is associated with 80 percent of MAHs and is caused by the effects of the oversecretion of parathyroid hormone-related peptide (PTHrP). In hyperparathyroidism, there is an elevated Ca, PTH, and serum 1,25 dihydroxy vitamin D, as well as hyperchloremic acidosis with a decreased pH confirmed by bicarbonate and chloride lab measurements. In HHM, typical lab values include elevated Ca and PTHrP, decreased PTH, and suppressed serum 1,25 dihydroxyvitamin D levels. Go to the next page if you knew the correct answer, or click the link image(s) below to further research the concepts in this question (if desired).
Research Concepts: Malignancy-Related Hypercalcemia
We update eBooks quarterly and Apps daily based on user feedback. Please tap flag to report any questions that need improvement.
Question 925:
A 33-year-old man with a mediastinal seminoma measuring 7.2 cm was treated with chemotherapy and radiation. He is currently asymptomatic but smokes tobacco. He reports smoking one pack per day for the last seven years. On a follow up computed tomography (CT) scan, he has a 3.5 cm mass in the anterior mediastinum. He reports no associated symptoms and reports he feels much better since he stopped his chemotherapy regimen. What is the most appropriate way to manage the residual mass found on the CT scan?
Choices: 1. Follow-up in one years time to assess symptoms and progression 2. Administer a second round of chemotherapy 3. Administer a second round of radiotherapy to the mass 4. Obtain an open biopsy for tissue diagnosis
Answer: 4 - Obtain an open biopsy for tissue diagnosis Explanations: After patients undergo treatment of their mediastinal seminoma, it is not uncommon to have a residual mass. The residual mass is not always cancer. The remaining mass may be a fibrotic scar, a desmoplastic reaction, or an area of necrosis. After treatment, residual masses in the mediastinum are usually monitored with serial CT scans if less than 3 cm in size. Masses larger than 3cm may contain residual disease in up to 30% of patients. If the mass is larger than 3cm, an open biopsy should be obtained instead of getting serial CT scans versus a positron emission tomography scan. Before starting any more chemotherapy or radiation, a tissue biopsy should be taken to determine if the mass is cancer. The residual mass should be further examined since it is larger than 3cm in size; this can harbor malignancy. Follow-up in one years time is not adequate. Starting chemotherapy or radiation without a tissue biopsy would be incorrect as it is first necessary to determine if this is cancer. Go to the next page if you knew the correct answer, or click the link image(s) below to further research the concepts in this question (if desired).
Research Concepts: Mediastinal Seminoma
We update eBooks quarterly and Apps daily based on user feedback. Please tap flag to report any questions that need improvement.
Question 926:
A 14-year-old boy is brought to the emergency department complaining of bone pain near his upper knee that is not relieved by aspirin. An x-ray shows a metaphyseal tumor arising from his distal femur. The patient's mother was diagnosed with breast cancer at the age of 30 years, and his mother’s sister passed away from a lethal brain tumor. Genetic analysis shows a germline mutation in the TP53 gene. Which of the following is the most likely underlying diagnosis?
Choices: 1. 2. 3. 4.
Retinoblastoma Li-Fraumeni syndrome Gardner syndrome Lynch syndrome
Answer: 2 - Li-Fraumeni syndrome Explanations: P53 is a major cell cycle regulator. Many tumors have a TP53 mutation. When a germline or inherited mutation in TP53 is present, patients have a syndrome called Li-Fraumeni syndrome and are genetically predisposed to a variety of cancers. Patients with Li-Fraumeni syndrome are predisposed to osteosarcomas, soft tissue sarcomas, brain tumors, premenopausal breast cancers, and adrenal cortical carcinomas. Go to the next page if you knew the correct answer, or click the link image(s) below to further research the concepts in this question (if desired).
Research Concepts: Genetics, Mitosis
We update eBooks quarterly and Apps daily based on user feedback. Please tap flag to report any questions that need improvement.
Question 927:
A 23-year-old woman undergoes her first screening colonoscopy due to the recently diagnosed colon cancer in her 35-year-old brother. Colonoscopy is remarkable for hundreds of adenomatous polyps and an ulcerated mass at the rectosigmoid junction. She is concerned about the potential for other malignancies and whether she is supposed to undergo a standard surveillance schedule. Which of the following statements is the most appropriate response to her inquiry?
Choices: 1. 2. 3. 4.
Annual upper GI endoscopy is indicated Annual abdominal sonography is recommended Desmoid tumors can occur in up to15 percent of patients There is no risk of thyroid malignancy
Answer: 3 - Desmoid tumors can occur in up to15 percent of patients
Explanations: The patient has familial adenomatous polyposis (FAP), diagnosed when more than one hundred polyps are seen on colonoscopy. Duodenal adenomas might occur in up to 90 percent of patients with FAP. Patients with a family history of FAP should undergo screening beginning at age 12. Desmoid tumors can occur in up to 15 percent of FAP patients. Considering the natural course of desmoid tumors, despite their slow-growing nature, they might cause severe morbidities. The morbidities are mainly related to the major vascular structure encasement. Up to fifty percent of patients with FAP have thyroid nodules. Their thyroid nodules are most commonly benign. However, there is a great propensity for malignancy in female patients in comparison with males (ratio: 19 to 1). Gastric and duodenal adenomas are also associated with FAP. The incidence of duodenal adenoma is more than 80% in FAP patients. Upper endoscopy should begin at 20 to 25 years of age. Go to the next page if you knew the correct answer, or click the link image(s) below to further research the concepts in this question (if desired).
Research Concepts: Familial Adenomatous Polyposis
We update eBooks quarterly and Apps daily based on user feedback. Please tap flag to report any questions that need improvement.
Question 928:
A 25-year-old patient presents to the clinic with complaints of abdominal discomfort, nausea, vomiting, fever, weight loss, and night sweats. Laboratory workup reveals cells that are CD10 positive but BCL-2 negative. FISH reveals an MYC translocation on chromosome 8. This patient's condition most commonly involves which part of the gastrointestinal tract?
Choices: 1. 2. 3. 4.
Duodenum Gastric antrum Terminal ileum Sigmoid colon
Answer: 3 - Terminal ileum Explanations: This clinical scenario is most consistent with Burkitt lymphoma, which frequently affects the ileocecal region. Burkitt lymphoma (BL) is an aggressive non-Hodgkin Bcell lymphoma. The disease is associated with EpsteinBarr virus (EBV), human immunodeficiency virus (HIV), and chromosomal translocations that cause the overexpression of oncogene C-MYC. BL of the intestine can present with nausea, vomiting, anorexia, a change in bowel habits, gastrointestinal bleeding, and intestinal perforation. Adult patients are more likely to present with constitutional symptoms (i.e., fever, weight loss, night sweats). Infrequently, patients have jaw or bone marrow involvement. Immunohistochemistry and cytogenetics play a significant role in the diagnosis and management of BL. The malignant B-cells express surface IgM. The cells are positive for B-cell markers, including CD19, CD20, CD79a, and PAX5. They are positive for germinal center markers CD10 and BCL-6 but are negative for BCL-2. Go to the next page if you knew the correct answer, or click the link image(s) below to further research the concepts in this question (if desired).
Research Concepts: Burkitt Lymphoma
We update eBooks quarterly and Apps daily based on user feedback. Please tap flag to report any questions that need improvement.
Question 929:
A 66-year-old man presents to the emergency department with seizures, headache, blurred vision, and speech difficulties. His blood pressure is 136/84 mmHg, temperature 37.1 C (98.8 F) and pulse 88/min. Magnetic resonance spectroscopy (MRS) is planned to rule out a glioma. Which of the following abnormalities is most likely to be associated with this tumor in this patient?
Choices: 1. 2. 3. 4.
Isocitrate dehydrogenase mutations Thiamine deficiency Fumarase mutations Oxaloacetate deficiency
Answer: 1 - Isocitrate dehydrogenase mutations Explanations: Mutations of isocitrate dehydrogenase (IDH) have been found in several types of cancers including leukemia, gliomas, and sarcomas. IDH mutations can be useful for the differential diagnosis and subclassification of human gliomas. The normal function of IDH is to catalyze the oxidative decarboxylation of isocitrate to alpha-ketoglutarate. Mutant IDH catalyzes the formation of 2hydroxyglutarate instead of alpha-ketoglutarate. 2-hydroxyglutarate is an oncometabolite that causes DNA and histone hypermethylation leading to neoplasia. 2-hydroxyglutarate can be used as a biomarker for cancer in patients with inborn errors of metabolism. Magnetic resonance spectroscopy (MRS) is a noninvasive technique that is used for measuring the concentrations of different chemical components within tissues. A missense mutation leads to the neomorphic reduction of isocitrate dehydrogenase leading to the accumulation of the oncometabolite 2-hydroxyglutarate. So the detection of 2-hydroxyglutarate can serve as a surrogate biomarker for these mutations. Go to the next page if you knew the correct answer, or click the link image(s) below to further research the concepts in this question (if desired).
Research Concepts: Physiology, Krebs Cycle
We update eBooks quarterly and Apps daily based on user feedback. Please tap flag to report any questions that need improvement.
Question 930:
A 52-year-old man comes to the outpatient department with the complaint of vague rightsided headache for the past six months. On physical examination, he does not have any neurological deficits. There is a hard swelling of size 5 x 3 cm in the right parietal region. He undergoes a computed tomogram scan, which shows a right temporoparietal hemispheric mass with hyperostosis of the overlying skull, which is eroded by the mass. He undergoes craniotomy and complete excision of the lesion. The pathology report states that the Ki-67 index is 11%. What is the chance of tumor recurrence?
Choices: 1. 2. 3. 4.
10% 20% 30% 50%
Answer: 4 - 50% Explanations: The patient has the diagnosis of right temporoparietal meningioma. Meningiomas are known to occur more commonly in elderly individuals. They cause thickening of the bone, in some instances, known as hyperostosis. Ki-67/MIB-1 proliferation index is used to assess the prognosis of meningioma. The recurrence rates for tumors with Ki-67 index 0.7%, 2.1%, and 11% are 9%, 29%, and 50% respectively. Go to the next page if you knew the correct answer, or click the link image(s) below to further research the concepts in this question (if desired).
Research Concepts: Meningioma
We update eBooks quarterly and Apps daily based on user feedback. Please tap flag to report any questions that need improvement.
Question 931:
Following colonoscopy for hematochezia, a patient has an appointment with a surgical oncologist for further evaluation and recommendation regarding a 4 cm mid-rectal tumor 8 cm from the anal verge. CT imaging provided with him demonstrates liver metastases that appear resectable with no associated lymphadenopathy. However, the rectal tumor is invading into the surrounding tissue. With regards to pre-operative planning, what distinguishes this patient’s cancer compared to a primary colon tumor?
Choices: 1. He will require a greater duration of pre-operative chemotherapy 2. He is more likely to have tumor burden necessitating radiotherapy 3. His morbidity and mortality is such that surgery is not a possibility as a result of his clinical stage, and he should undergo chemotherapy only 4. He can undergo total mesorectal excision with concomitant localized hepatectomy without need for neoadjuvant chemotherapy
Answer: 2 - He is more likely to have tumor burden necessitating radiotherapy
Explanations: Preoperative radiotherapy, along with chemotherapy, is recommended for mid to low rectal tumors with associated hepatic metastases. A palliative diverting ostomy may be required for rectal cancers deemed unresectable. Tumor location within the rectum will dictate the surgical approach. Rectal cancer management is different than colon cancer. Go to the next page if you knew the correct answer, or click the link image(s) below to further research the concepts in this question (if desired).
Research Concepts: Liver Metastasis
We update eBooks quarterly and Apps daily based on user feedback. Please tap flag to report any questions that need improvement.
Question 932:
A 65-year-old male with a past medical history of gastroesophageal reflux disease presents to the clinic for continued dyspepsia and reflux symptoms. He reports taking once-daily omeprazole for many months without improvement. Physical examination is unremarkable. Complete blood count and the basic metabolic panel are significant only for mild hypokalemia of 3.4 mEq/L. He undergoes esophagogastroduodenoscopy which shows Barrett esophagus. The biopsy reveals dysplasia. In which of the following is endoluminal therapy contraindicated?
Choices: 1. 2. 3. 4.
High-grade dysplasia Low-grade dysplasia Dysplasia with the invasion of submucosa Dysplasia with flat circumferential lesions
Answer: 3 - Dysplasia with the invasion of submucosa Explanations: Endoscopic eradication therapies have become the mainstay of treatment for patients with Barrett's esophagus with dysplasia. Application of heat through laser, radiofrequency, argon plasma coagulation, or cold through cryotherapy or photodynamic therapy (PDT) is used to destroy the abnormal epithelium. Endoluminal therapy is not recommended in cases of submucosal invasion because it is associated with a much higher risk of metastatic spread. The other lesions listed can be treated with endoluminal therapy. Ablation therapies are indicated for flat lesions. Any nodular lesions should be removed using endoscopic mucosal resection, which also provides a substantial sample for determining the depth of invasion. Go to the next page if you knew the correct answer, or click the link image(s) below to further research the concepts in this question (if desired).
Research Concepts: Barrett Metaplasia
We update eBooks quarterly and Apps daily based on user feedback. Please tap flag to report any questions that
need improvement.
Question 933:
A 43-year-old woman presents to the clinic for a routine pap screen. The results come back as “atypical glandular cells, endometrial.” Which of the following is the next best step in the management of this patient?
Choices: 1. Colposcopy 2. Repeat cervical cytology with HPV DNA testing in 3 months 3. Cone biopsy 4. Endocervical and endometrial sampling with or without colposcopy
Answer: 4 - Endocervical and endometrial sampling with or without colposcopy
Explanations: In patients with a diagnosis of “atypical glandular cells, endometrial,” the recommended management is endocervical and endometrial sampling with or without colposcopy. If no pathology is found, colposcopy is recommended. Cone biopsy with colposcopy is recommended for patients diagnosed with atypical glandular cells, NOS category. The patients diagnosed with atypical glandular cells should be followed up with repeat cytology along with High-risk HPV DNA testing every six months if the workup is negative. Go to the next page if you knew the correct answer, or click the link image(s) below to further research the concepts in this question (if desired).
Research Concepts: Atypical Glandular Cells (AGS)
We update eBooks quarterly and Apps daily based on user feedback. Please tap flag to report any questions that need improvement.
Question 934:
In which of the following types of acute myeloid leukemia is disseminated intravascular coagulation common?
Choices: 1. 2. 3. 4.
Acute Acute Acute Acute
myelomonocytic leukemia erythroid leukemia promyelocytic leukemia megakaryocytic leukemia
Answer: 3 - Acute promyelocytic leukemia Explanations: Disseminated intravascular coagulation is common in patients with acute promyelocytic leukemia (APL). Patients are at high risk of bleeding complications. Without treatment, patients with APL die due to bleeding complications in less than a week. Treatment with tretinoin may be initiated prior to the confirmation of diagnosis. Invasive procedures and leukapheresis are contraindicated in APL. Go to the next page if you knew the correct answer, or click the link image(s) below to further research the concepts in this question (if desired).
Research Concepts: Acute Myeloid Leukemia
We update eBooks quarterly and Apps daily based on user feedback. Please tap flag to report any questions that need improvement.
Question 935:
A 76-year-old man with a history of rectal cancer cured with chemotherapy and radiation therapy 15 years ago presents to the clinic for one week of increased urinary frequency. He states he noticed a moderate amount of blood while urinating this morning. He reports feeling well until this started one week ago and has not noticed any other symptoms. A physical exam is unremarkable. Urinalysis, culture, and cytology are performed and rule out infection and bladder cancer. Urinalysis is negative for RBCs currently and does not indicate that a urinary tract infection is present. Cystoscopy shows moderate generalized telangiectasia without petechiae, erythema, and no blood clots. Which of the following is the next best step in the management of this patient?
Choices: 1. Bladder function testing 2. CT of the abdomen/pelvis 3. Complete blood count, chemistry panel, and coagulation studies 4. Electrocautery
Answer: 3 - Complete blood count, chemistry panel, and coagulation studies
Explanations: The history of rectal cancer with radiation therapy indicates radiation cystitis should be on the differential with this patient's presentation. With his presentation of increased urinary frequency and new-onset hematuria in the setting of prior pelvic radiation therapy, the first line of evaluation should be urinalysis, urine culture, and urine cytology to evaluate for signs of infection or presence of RBCs to further direct treatment. After this has been performed, it is appropriate to get basic labs, including a CBC, CMP, and coagulation studies. CBC can be used to evaluate WBC for infection, hemoglobin for monitoring for anemia, and platelets for the etiology of hematuria. CMP can be used to evaluate electrolytes and renal function. Coagulation studies again can be used to evaluate the etiology of hematuria. The level of severity of symptoms is graded by Radiation Therapy Oncology Group (RTOG) as follows: Grade 1 - any evidence of epithelial damage or atrophy, telangiectasia, microscopic hematuria; Grade 2 - any moderate frequency, generalized telangiectasia, intermittent macroscopic hematuria, intermittent urinary incontinence; Grade 3 - any severe frequency or urgency, severe telangiectasia, persistent, incontinence, reduced bladder capacity 150 mL, frequent hematuria; Grade 4 - any necrosis, fistula, hemorrhagic cystitis, reduced bladder capacity 100 mL, refractory incontinence requiring either catheter or surgical intervention.
The other options listed may all be appropriate eventually in the workup and treatment of this patient pending symptom progression but would not be the most appropriate next step in the initial evaluation. Go to the next page if you knew the correct answer, or click the link image(s) below to further research the concepts in this question (if desired).
Research Concepts: Radiation Cystitis And Hyperbaric Management
We update eBooks quarterly and Apps daily based on user feedback. Please tap flag to report any questions that need improvement.
Question 936:
A 55-year-old woman presents to the clinic for routine follow-up. She has a history of cervical cancer treated with chemoradiation, followed by a hysterectomy. Today, she was found to have a central pelvic mass with no symptoms. Biopsy of the mass demonstrates recurrent cervical cancer. Preoperative evaluation included a computed tomography scan, magnetic resonance imaging, and positron emission tomography scan. The mass involved the bladder, vagina, and rectum. No distant metastases were identified on imaging. A pelvic exenteration was performed. The resected specimen was identified to be R0. Which of the following factors best indicates the potential for overall survival in this patient?
Choices: 1. 2. 3. 4.
Prior chemoradiation Lack of symptoms No distant metastasis on imaging R0 resection
Answer: 4 - R0 resection Explanations: Preoperative evaluation to include computed tomography scan, magnetic resonance imaging, and positron emission tomography scan is important to both, excluding distant metastasis and determining an R0 resection potential. With preoperative evaluation, an R0 resection is accomplished in approximately 65% of cases. For recurrent malignancy, overall survival is generally 50% or greater when an R0 resection is accomplished. R0 resection is the best predictor of outcomes for pelvic exenteration. Prior treatments and symptoms have no major bearing on outcomes for pelvic exenteration. A search for distant metastasis is important to exclude a patient for pelvic exenteration, but having involved surgical margins with malignancy indicates poor overall survival. Go to the next page if you knew the correct answer, or click the link image(s) below to further research the concepts in this question (if desired).
Research Concepts: Pelvic Exenteration
We update eBooks quarterly and Apps daily based on user feedback. Please tap flag to report any questions that
need improvement.
Question 937:
A 35-year-old female presents to the outpatient department with the complaint of occasional mild to moderate frontal headaches over the past few months. She also gives a history of blurring of vision. She does not have galactorrhea or amenorrhea. She comes with a magnetic resonance imaging scan of the head, which shows a sellar lesion extending to the suprasellar location and displacing the chiasm. Which of the following is the most sensitive ophthalmologic test for detecting optic pathway compromise secondary to this tumor?
Choices: 1. 2. 3. 4.
Peripheral visual field testing Central visual field testing Examination of the fundus Color vision testing
Answer: 1 - Peripheral visual field testing Explanations: The patient is having a pituitary macroadenoma. The compression of the chiasm causes visual field abnormalities in a pituitary adenoma. The most sensitive test for detecting optic pathway compromise due to a pituitary tumor is peripheral visual field testing. Decreases in color perception are much less sensitive in the diagnosis of optic nerve compression. Go to the next page if you knew the correct answer, or click the link image(s) below to further research the concepts in this question (if desired).
Research Concepts: Pituitary Cancer
We update eBooks quarterly and Apps daily based on user feedback. Please tap flag to report any questions that need improvement.
Question 938:
A 70-year-old female undergoes radiographic imaging of the chest for surgical clearance prior to an elective orthopedic surgery. She is found to have a pulmonary nodule. Further questioning reveals poor functional capacity and tolerance for the last several months. She also reported poor appetite and unintentional weight loss but thought this was related to "old age." Which of the following is the most likely cause of this pulmonary lesion?
Choices: 1. 2. 3. 4.
Malignant melanoma Esophageal cancer Uterine leiomyoma Giant cell tumor of the bone
Answer: 1 - Malignant melanoma Explanations: The most common cancers that metastasize to the lungs are malignant melanoma, sarcoma, bronchogenic carcinoma, colon cancer, renal cell carcinoma, breast cancer, and testicular cancer. The probability of a pulmonary nodule being a malignant metastatic nodule in a patient with prior history of extrathoracic malignancy is 25%. Pulmonary metastases usually present as round variable-sized nodules with diffuse thickening of the interstitium. However, atypical features of pulmonary metastases are frequently seen which makes it difficult to differentiate between metastatic malignant, primary malignant, and benign lesions based on radiographic evidence alone. Calcifications, for example, can occur in metastatic sarcoma or adenocarcinoma. Calcifications are also seen in benign granulomas and hamartomas. Uterine leiomyomas and giant cell tumors of the bone are benign tumors that rarely metastasize to the lung. Go to the next page if you knew the correct answer, or click the link image(s) below to further research the concepts in this question (if desired).
Research Concepts: Lung Metastasis
We update eBooks quarterly and Apps daily based on user feedback. Please tap flag to report any questions that need improvement.
Question 939:
A 38-year-old woman is found to have an incidental 2-cm fluid-cyst branch-duct on the tail of the pancreas after visiting the emergency department two weeks ago for back pain. She had an abdominal computed tomography (CT) showing a renal stone and was treated for a urinary tract infection. Now, the patient presents to the clinic to address this concerning pancreatic lesion, fearing her father had died of pancreatic cancer at age 54. She has no active complaints and has no past medical history. Physical examination is unremarkable. Routine labs and appropriately ordered tumor markers are unremarkable. Prior contrast-enhanced CT showed no concerning lymphadenopathy, no vascular involvement, and no liver lesions. An endoscopic ultrasound (EUS) with fine-needle aspiration (FNA) confirms a 20 mm fluid-cystic lesion, no main duct dilation, a thin wall of 5 mm, and with negative for atypical or malignant cells cytology. What is the next best step in the management of this patient?
Choices: 1. 2. 3. 4.
Surgical resection Reevaluate in 3 weeks Direct EUS-guided alcohol ablation Serial surveillance of the cystic lesion every 2 years
Answer: 4 - Serial surveillance of the cystic lesion every 2 years
Explanations: Cystic lesions associated with an increased risk of malignancy included cyst size greater than 3 cm, a solid component within the cyst, an irregular and thick septum in branch-duct, main pancreatic duct dilation more than 7 mm, and mural nodules. None of these are present in our clinical scenario, further supported with negative cytology. Branch-duct IPMN without high-risk features can be managed with surveillance. The best surveillance recommendation is unclear; guidelines suggest an MRI in one year and then every two years for a total of five years for patients with unchanged characteristics of the pancreatic cyst. Surgery is a treatment option in patients with high-risk features IPMN of the pancreas with high-grade dysplasia or invasive carcinoma on biopsy or cytology. EUS-guided alcohol ablation is an option for non-surgical candidates with symptomatic lesion and/or high-risk malignancy suspicion; otherwise, it is not considered the standard of care. Go to the next page if you knew the correct answer, or click the link image(s) below to further research the concepts in this question (if desired).
Research Concepts: Mucinous Cystic Pancreatic Neoplasms
We update eBooks quarterly and Apps daily based on user feedback. Please tap flag to report any questions that need improvement.
Question 940:
A 66-year-old woman being treated for stage IV colorectal adenocarcinoma was recently started on chemotherapy. She presents to your office for the sudden eruption of a tender red papule on her right index finger that bleeds profusely with minor trauma. Pyogenic granuloma is suspected based on the exam. What medication is the most likely cuprite in this case?
Choices: 1. 2. 3. 4.
Tamoxifen Bortezomib Methotrexate Capecitabine
Answer: 4 - Capecitabine Explanations: Digital pyogenic granuloma (PG) has a strong association with medications. Reports suggest up to 30% of periungual PG is associated with medication use. Medication-induced pyogenic granuloma can be solitary but are often multiple. Antineoplastic drugs are frequently implicated. Most common include pyrimidine analogs (capecitabine and systemic 5-fluorouracil), taxanes (docetaxel and paclitaxel), epidermal growth factor receptor (EGFR) inhibitors (monoclonal antibodies against the EGFR (cetuximab, panitumumab) and EGFR tyrosine kinase inhibitors (gefitinib, erlotinib)) and tyrosine kinase inhibitor (Imatinib). When a therapeutic regimen includes the above medications, patients should be counseled about this possible adverse reaction. Patients should be advised to avoid trauma to the feet and contact the provider to evaluate and treat lesions if they occur. Options include weekly chemical cauterization with silver nitrate. Go to the next page if you knew the correct answer, or click the link image(s) below to further research the concepts in this question (if desired).
Research Concepts: Pyogenic Granuloma
We update eBooks quarterly and Apps daily based on user feedback. Please tap flag to report any questions that need improvement.
Question 941:
A 16-year-old boy with a history of attention-deficit hyperactivity disorder (ADHD) and an eye tumor in infancy is brought to the hospital with a 3-month history of right knee pain. X-ray of the knee shows a destructive lesion in the proximal tibial metaphysis. A biopsy specimen shows poorly differentiated malignant cells with abnormal mitotic figures and a high nuclear to cytoplasmic ratio. Which of the following best identifies the most likely bone tumor and the genetic abnormality in this patient?
Choices: 1. 2. 3. 4.
Osteosarcoma; pRB-1 mutation Ewing sarcoma; t(2;13) translocation Chondrosarcoma; translocation of the RB gene Osteosarcoma; t(11;22) mutation
Answer: 1 - Osteosarcoma; pRB-1 mutation Explanations: This question stem describes an adolescent male patient with a history of retinoblastoma (eye cancer). Mutations in the tumor suppressor genes p53 and retinoblastoma (RB) are associated with the development of osteosarcoma. The most common primary bone cancer in this age group is osteosarcoma. The translocation between the Ewing sarcoma gene EWS and transcription factor FLI t(11;22) is associated with Ewing sarcoma. Commonly, Ewing sarcoma occurs in the diaphysis of long bones. Ewing sarcoma is the second most common primary bone cancer in adolescents. The translocation t(2;13) is associated with rhabdomyosarcoma. The etiology of chondrosarcoma is still under investigation. Mutations in the IDH genes are associated with developing some forms of chondrosarcoma. Go to the next page if you knew the correct answer, or click the link image(s) below to further research the concepts in this question (if desired).
Research Concepts: Primary Bone Cancer
We update eBooks quarterly and Apps daily based on user feedback. Please tap flag to report any questions that need improvement.
Question 942:
A 2-year-old male is being seen in the emergency department for jaundice and itching. The initial exam was normal, with the exception of some mild right upper quadrant tenderness. Labs revealed elevated transaminases. An ultrasound was obtained, which showed a large hepatic mass. What are the risks of long term treatment for this condition?
Choices: 1. 2. 3. 4.
Obesity Delayed puberty Sensorineural hearing impairment Short stature
Answer: 3 - Sensorineural hearing impairment Explanations: The patient gives a clinical and radiological presentation of hepatoblastoma. Treatment for hepatoblastoma includes cisplatin, which can lead to sensorineural hearing impairment. Long term complications of hepatoblastoma include cardiomyopathy, hearing loss, and secondary malignancies. Hepatoblastoma treatment will not increase the patient's risk for short stature, obesity, or delayed puberty. Go to the next page if you knew the correct answer, or click the link image(s) below to further research the concepts in this question (if desired).
Research Concepts: Hepatoblastoma
We update eBooks quarterly and Apps daily based on user feedback. Please tap flag to report any questions that need improvement.
Question 943:
A 52-year-old man presents to the clinic with fatigue and night sweats. On examination, he has evidence of splenomegaly. Complete blood count shows a WBC count of 200,000/microL, and 70% of cells are reported to be lymphocytes. Flow cytometry shows a monoclonal B cell population that is CD5 positive. FISH studies are positive for t(11:14). What's the most likely diagnosis?
Choices: 1. 2. 3. 4.
Chronic lymphocytic leukemia Mantle cell lymphoma Small lymphocytic lymphoma Follicular lymphoma
Answer: 2 - Mantle cell lymphoma Explanations: FISH studies help diagnose and distinguish mantle cell lymphoma from CLL. Mantle cell lymphoma patients can present with lymphocytosis. FISH for (11:14) is helpful to diagnose mantle cell lymphoma, and FISH can provide useful information in addition to prognosis. In this case, it helped us diagnose mantle cell lymphoma. Flow cytometry can sometimes confuse mantle cell lymphoma from CLL. Go to the next page if you knew the correct answer, or click the link image(s) below to further research the concepts in this question (if desired).
Research Concepts: Chronic Lymphocytic Leukemia With Variant Genetics
We update eBooks quarterly and Apps daily based on user feedback. Please tap flag to report any questions that need improvement.
Question 944:
A 55-year-old male with a 75 pack-year history of smoking presents to the clinic to establish primary care and discuss age-appropriate screening options for cancer. His pulse rate is 78/min, blood pressure is 140/80 mmHg, and saturation at room air is 96%. Air entry is decreased bilaterally with occasional rhonchi and crepitations on chest auscultation. He is advised by the primary care provider to undergo a low-dose CT scan (LDCT) for screening. Which of the following are the criteria for this patient's screening test?
Choices: 1. All adults between ages 55 and 80 years 2. Adults more than 40 years of age with a 20 pack-year smoking history 3. Adults aged 55 to 80 years with a 30 pack-year smoking history 4. Adults less than 60 years of age who are active smokers and have no other risk factors
Answer: 3 - Adults aged 55 to 80 years with a 30 packyear smoking history
Explanations: All adults aged 55 to 80 years do not meet the criteria for a high-risk population. As lung cancer screening involves exposure to possible harmful radiation and potential other risks to screening, identifying a high-risk population is essential. According to the National Comprehensive Cancer Network (NCCN), there is a utility in screening patients with a 20 pack-year history of smoking. However, recommendations are for patients over 55 years of age with additional risk factors such as family history, occupational exposure to carcinogens, or personal history of chronic obstructive pulmonary disease (COPD). According to the National Lung Screening Trial, patients aged 55 to 80 years with a 30 pack-year history of smoking who are active smokers or quit within the last 15 years qualify for annual low-dose CT screening. Patients with a life-limiting condition should not be screened. Go to the next page if you knew the correct answer, or click the link image(s) below to further research the concepts in this question (if desired).
Research Concepts: Lung Cancer Screening
We update eBooks quarterly and Apps daily based on user feedback. Please tap flag to report any questions that need improvement.
Question 945:
A 68-year-old female with a past medical history significant for diabetes mellitus, hypertension, and tobacco use, presents to the emergency department with abdominal pain, weakness, and weight loss of approximately 15 pounds (7 kg) in the past two months. She states that lately, she has had a decreased appetite, which is what she believes caused her weight loss. The abdominal pain has acutely worsened and became more constant, prompting her to come to the emergency department for further evaluation. She admits to having small, thin caliber bowel movements every few days and occasionally notes that her stool is black. She has been taking iron supplements in addition to her multivitamin because she thought her weakness was secondary to her doctor telling her she is anemic at her last check-up over six months ago. She assumed the iron was also causing her stool discoloration. She denies any bleeding per rectum, nausea, or vomiting. She has never had an endoscopy or colonoscopy before. She denies any recent travel or sick contacts. Her family history is significant for her mother's death from ovarian cancer. She drinks socially on the weekends and has been smoking a pack per day since she was 17. On exam, her vitals are within normal limits. She appears pale and thin with a mildly distended abdomen. A rectal exam shows grade 2 internal hemorrhoids. A full set of labs, imaging, and gastroenterology consult are ordered as it is suspect she may have colon cancer. At what age should she have gotten a colonoscopy?
Choices: 1. 21 years old 2. 25 years old 3. 45 years old
. 50 years old 4
Answer: 3 - 45 years old Explanations: In the absence of risk factors, adults should undergo screening for colon cancer starting at age 45. Recommended procedures include colonoscopy, sigmoidoscopy, or high-sensitivity fecal occult blood testing. Other screening options include a flexible sigmoidoscopy every five years and a double-contrast barium enema every five years. If no polyps are found in the first screening colonoscopy, the next one should be done after ten years. If the patient has a first-degree relative with diagnosed colorectal cancer or advanced adenoma before 60 years of age or two first-degree relatives diagnosed, then screening should begin at 40 years old or ten years before the earliest diagnosis in the patient's family; whichever comes first. Testing should be repeated every five years. Go to the next page if you knew the correct answer, or click the link image(s) below to further research the concepts in this question (if desired).
Research Concepts: Colon Cancer Screening
We update eBooks quarterly and Apps daily based on user feedback. Please tap flag to report any questions that need improvement.
Question 946:
A 67-year-old male with HIV presents with fevers, night sweats, and 20 lb (9 kg) weight loss in the past three months. His physical exam reveals inguinal lymphadenopathy. CT of the chest, abdomen, and pelvis reveals a gastric mass. Biopsy of the gastric mass reveals plasmablasts, and immunophenotype is positive for CD 19, CD 38, MUM-1, and MYC gene rearrangement is present. What is the standard of care for this tumor?
Choices: 1. Rituximab, cyclophosphamide, doxorubicin, vincristine, prednisone (R-CHOP) 2. Dose-adjusted rituximab, etoposide, vincristine, prednisone, cyclophosphamide, doxorubicin (da-R-EPOCH) 3. There is no current standard of care 4. Rituximab, cyclophosphamide, vincristine, prednisone (RCVP)
Answer: 3 - There is no current standard of care Explanations: Plasmablastic Lymphoma (PBL) is a very aggressive lymphoma that is both hard to diagnose and treat. There is no current standard of care for the treatment of PBL. da-R-EPOCH, R-CHOP, and R-CVP are the most common regimens used to treat this condition. da-R-EPOCH has been shown to be more effective than R-CHOP in patients with HIV-associated lymphomas. RCHOP and R-CVP can also be used in elderly and frail patients who cannot tolerate very intensive chemotherapy. Go to the next page if you knew the correct answer, or click the link image(s) below to further research the concepts in this question (if desired).
Research Concepts: Plasmablastic Lymphoma
We update eBooks quarterly and Apps daily based on user feedback. Please tap flag to report any questions that need improvement.
Question 947:
Radioactive iodine uptake scan in a 26year-old white woman who presented with a 2-month history of thyroid mass reveals a solitary, discrete area of decreased uptake. Which of the following statements is correct with respect to these neck findings?
Choices: 1. 2. 3. 4.
This is a hot nodule Hashimoto is the most likely cause This finding has ruled out cancer The mass should be evaluated for possible cancer
Answer: 4 - The mass should be evaluated for possible cancer
Explanations: Decreased uptake on a radioiodine scan is otherwise known as a "cold nodule". 10% of cold nodules are malignant. In contradistinction to hot nodules which are usually benign. A discrete enlargement in the thyroid is more likely to be an adenoma than Hashimotos, which would present with diffuse swelling. This patient would require further evaluation to rule out a cancer. Go to the next page if you knew the correct answer, or click the link image(s) below to further research the concepts in this question (if desired).
Research Concepts: Thyroid Uptake and Scan
We update eBooks quarterly and Apps daily based on user feedback. Please tap flag to report any questions that need improvement.
Question 948:
A 65-year-old male patient presents to the dermatology clinic for the evaluation of a 3-cm pigmented patch on his right heel that was discovered by his primary care provider. Past medical history is only significant for hypertension. Focal blue-black nodules are seen within the brown patch. A punch biopsy of the lesion reveals lentiginous malignant melanocytes at the dermalepidermal junction, but no invasion is identified in the specimen. What is the next best step?
Choices: 1. Excise the entire lesion with 2 cm margins 2. Amputate the foot at the ankle 3. Observe the patient and have them return to the clinic in 6 months 4. Re-biopsy the nodular portion of the lesion
Answer: 4 - Re-biopsy the nodular portion of the lesion Explanations: A 3-cm nodular melanocytic lesion on an acral surface is highly suspicious for invasive melanoma. The nodular portions of the lesion most likely represent foci of invasion and should be preferentially biopsied. Clinicopathologic correlation is imperative in acral lentiginous melanoma and requires diligent communication between multiple disciplines. For acral lentiginous melanoma in-situ, the American Academy of Dermatology recommended surgical margins are 0.5 cm. However, appropriate definitive treatment is not clear if an invasive lesion is expected, yet not confirmed on pathologic evaluation. Patients with suspected invasive acral lentiginous melanoma should be actively engaged with the healthcare team to ensure rapid treatment of the lesion. Go to the next page if you knew the correct answer, or click the link image(s) below to further research the concepts in this question (if desired).
Research Concepts: Acral Lentiginous Melanoma
We update eBooks quarterly and Apps daily based on user feedback. Please tap flag to report any questions that need improvement.
Question 949:
A 22-year-old female with no significant past medical history presented with one month of intermittent fever, night sweats, and 15-pound (7 kg) weight loss without intention. She was worked up for possible underlying malignancy and was referred to the hematology clinic by her primary care provider due to "lymphoblastic cells in the peripheral blood smear." An extensive workup was done, including flow cytometry of peripheral blood smear and a bone marrow biopsy. She was diagnosed with B-cell acute lymphoblastic leukemia (ALL) with a positive Philadelphia chromosome. She was started on imatinib and went into partial remission but relapsed in the following months. Treatment was then switched to dasatinib, another tyrosine kinase inhibitor, with which no remission was achieved. She was recommended to start chimeric antigen receptor T-cells (CAR T-cells) therapy and started on tisagenlecleucel. Two days after the infusion of medication, she became short of breath and progressively worse. She was sent to the emergency department of her cancer center and intubated due to acute hypoxemic respiratory failure. She became hypotensive and was admitted to the intensive care unit. Hematology/oncology was consulted. Besides the ventilation and blood pressure support, which of the following medication is the best next step?
Choices: 1. 2. 3. 4.
Acetaminophen Oral prednisone Tocilizumab Tocilizumab and IV hydrocortisone 100 mg every 8 hours
Answer: 4 - Tocilizumab and IV hydrocortisone 100 mg every 8 hours
Explanations: Cytokine release syndrome is the most common side effect of CAR T-cell therapy. Mild symptoms of low-grade fever, fatigue, anorexia can be managed supportively with acetaminophen, intravenous fluid, etc. Severe CRS with hemodynamic instability despite IV fluids and vasopressor support or worsening respiratory distress requiring ventilation support, an IL-6 antagonist, tocilizumab should be given immediately along with steroids. Tocilizumab can be repeated with a maximal of 4 doses in total, with at least an 8-hour interval between consecutive doses. Go to the next page if you knew the correct answer, or click the link image(s) below to further research the concepts in this question (if desired).
Research Concepts: Chimeric Antigen Receptor T-Cell Therapy
We update eBooks quarterly and Apps daily based on user feedback. Please tap flag to report any questions that need improvement.
Question 950:
A 38-year-old woman presents to the clinic with a one-year history of progressive monoparesthesia and monoparesia of her left leg. Neurological examination reveals mild paresis of the left quadriceps and iliopsoas muscles and the absence of the Achilles reflex on the left. Lumbar spinal magnetic resonance imaging (MRI) reveals an intradural extramedullary mass at L1-L2. The lesion shows intermediate signal intensity on T1-weighted (W) and T2W images and is homogeneously enhanced after intravenous contrast injection. The patient undergoes a lumbar laminectomy at the level of L1-L2 with gross total resection of the dominant lumbar tumor. Histologically, the tumor is composed of low-grade ependymal cells surrounding blood vessels, producing the papillary appearance and perivascular myxoid material between blood vessels and ependymal cells, creating the myxopapillary appearance. Focally, hypercellularity, pseudopalisading necrosis, vascular proliferation, with frequent pleomorphic cells and atypical mitotic figures are noted. The tumor cells show positive immunostaining for vimentin and glial fibrillary acidic protein (GFAP); but are negative for epithelial membrane antigen (EMA), CAM5.2. The MIB-1 index (Ki-67) is 30%. What is the most likely diagnosis?
Choices: 1. 2. 3. 4.
Anaplastic myxopapillary ependymoma Myxopapillary ependymoma (grade I) Anaplastic ependymoma (grade III) Classic ependymoma (grade II)
Answer: 1 - Anaplastic myxopapillary ependymoma Explanations: Anaplasia in myxopapillary ependymomas is characterized by the presence of hypercellularity, pseudopalisading necrosis, brisk mitotic activity, vascular proliferation, and nuclear pleomorphism. In the present case, histological examination reveal hypercellularity, pseudopalisading necrosis, vascular proliferation with frequent pleomorphic cells and atypical mitotic figures. Moreover, the MIB-1 index (Ki67) is 30%. Such features are seen in anaplastic myxopapillary ependymoma. Anaplastic myxopapillary ependymomas are locally invasive, with a higher rate of recurrence and spread to other areas of the neuraxis via the cerebrospinal fluid. The survival rate of anaplastic myxopapillary ependymomas and prognosis is poorer than conventional myxopapillary ependymoma grade I. Anaplastic myxopapillary ependymoma differs from the classical ependymomas morphologically and biologically. The immunohistochemical study is realized to rule out differential diagnoses such as chordomas or chondrosarcomas. Go to the next page if you knew the correct answer, or click the link image(s) below to further research the concepts in this question (if desired).
Research Concepts: Myxopapillary Ependymoma
We update eBooks quarterly and Apps daily based on user feedback. Please tap flag to report any questions that need improvement.
Question 951:
A 67-year-old woman with a past medical history of stage 4 breast carcinoma is inpatient receiving hospice care. The patient mentions that she is feeling tearful and anxious about death. She regrets many things she did in her life. However, there is no complaint of low mood. Which of the following medications is most appropriate to relieve her anxiety?
Choices: 1. 2. 3. 4.
Propranolol Alprazolam Sertraline Haloperidol
Answer: 2 - Alprazolam Explanations: Psychiatric symptoms such as depression and anxiety affect the psychological health domain of the dying patient. These symptoms can range from moderate to severe and should be assessed using valid tools to direct treatment. The General Anxiety Disorder (GAD-7) and Hamilton Anxiety Rating Scale can be used to measure symptoms of anxiety. Among the most common pharmacological interventions to treat anxiety symptoms are benzodiazepines. Sertraline, a selective serotonin reuptake inhibitor, may be effective in the long term but take weeks to take effect. Cognitive-behavioral therapy, as well as meditation, is effective in reducing anxiety symptoms. Go to the next page if you knew the correct answer, or click the link image(s) below to further research the concepts in this question (if desired).
Research Concepts: End of Life Care
We update eBooks quarterly and Apps daily based on user feedback. Please tap flag to report any questions that need improvement.
Question 952:
A 69-year-old male with a previous history of transsphenoidal surgery for the management of Cushing disease five years prior is evaluated by the endocrinologist and the general surgeon for treatment failure. He is now being considered for bilateral adrenalectomy, as he is not controlled with medical therapy. Which postadrenalectomy factor in this patient is most predictive for developing a new pituitary tumor?
Choices: 1. 2. 3. 4.
High levels of plasma adrenocorticotropic hormone High levels of plasma cortisol Old age Male gender
Answer: 1 - High levels of plasma adrenocorticotropic hormone
Explanations: Risk factors for developing Nelson syndrome have been studied with means of achieving early detection and treatment. High levels of plasma adrenocorticotropic hormone following bilateral adrenalectomy have the strongest predictive capabilities. Most of the Nelson syndrome cases are seen as a result of bilateral adrenalectomy for the treatment of Cushing disease. It is hypothesized that the loss of feedback inhibition of the hypothalamic-pituitary-adrenal axis leads to the development of an adrenocorticotropic hormonesecreting pituitary tumor. Age and sex have not been significantly associated with increased risk; however, younger patients undergoing bilateral adrenalectomy have a higher lifetime risk of developing Nelson syndrome. Go to the next page if you knew the correct answer, or click the link image(s) below to further research the concepts in this question (if desired).
Research Concepts: Nelson Syndrome
We update eBooks quarterly and Apps daily based on user feedback. Please tap flag to report any questions that need improvement.
Question 953:
A 47-year-old man presents to the emergency department complaining of cough, subjective fevers, and night sweats for the past week. He has recently immigrated to the United States from Russia where he worked as a computer programmer. He eats a diet high in fish and reports drinking one to two beers once a week during the weekend. On physical examination, he is noted to have abnormally enlarged fingertips and curvature of the nails. Which disease process is associated with his condition?
Choices: 1. 2. 3. 4.
Vitamin D deficiency Bronchogenic carcinoma Calcium deficiency Iron deficiency
Answer: 2 - Bronchogenic carcinoma Explanations: Nail clubbing is seen in lung cancer and other chronic pulmonary and cardiac diseases. Despite the broad literature on the subject of clubbing, there have never been well-controlled studies to determine exactly which conditions produce clubbing. Most of the data we have on clubbing is based on case reports and everyone is extrapolating these findings to many disorders. Clubbing is known to occur in many disorders besides just lung and heart pathology. Many GI disorders (Inflammatory bowel disease) also produce clubbing. Go to the next page if you knew the correct answer, or click the link image(s) below to further research the concepts in this question (if desired).
Research Concepts: Nail Clubbing
We update eBooks quarterly and Apps daily based on user feedback. Please tap flag to report any questions that need improvement.
Question 954:
A 65-year-old woman who is a farmer presents with a papule on her neck. The patient reports an itching and burning sensation associated with the lesion. She has no past medical history and takes no medications. On examination, there is a pink-white 1 cm papule on her neck. A punch biopsy shows small keratocysts on the top. There are cords and strands of basaloid epithelioid cells with ductal lumina that invade the dermis and the sub-dermis. There is a zone of separation between the epidermis and the tumor. Which of the following is an immunohistochemical feature of this tumor that differentiates it from basal cell and squamous cell carcinomas?
Choices: 1. 2. 3. 4.
Carcinoembryonic antigen stain (CEA) is positive Carcinoembryonic antigen stain (CEA) is negative It stains positive for eccrine and sebaceous structures It stains negative for apocrine and sebaceous structures
Answer: 1 - Carcinoembryonic antigen stain (CEA) is positive
Explanations: Microcystic adnexal carcinoma (MAC) is a slow-growing sweat gland carcinoma. It is distinguished from basal cell carcinoma and desmoplastic squamous cell carcinoma on the basis of ductal differentiation. Immunohistochemistry (IHC) is utilized in the pathological evaluation of MAC to help support the diagnosis. In particular, the carcinoembryonic antigen stain (CEA) is positive in approximately 50% of MAC cases. This helps distinguish MAC from basal cell carcinoma and squamous cell carcinoma where CEA is negative. CEA stains apocrine and eccrine structures positive while negative for sebaceous structures. Go to the next page if you knew the correct answer, or click the link image(s) below to further research the concepts in this question (if desired).
Research Concepts: Microcystic Adnexal Carcinoma
We update eBooks quarterly and Apps daily based on user feedback. Please tap flag to report any questions that need improvement.
Question 955:
A 41-year-old man presents to the hospital with generalized weakness. He has a history of appendicular pseudomyxoma treated 5 weeks ago with extensive cytoreductive surgery and intraperitoneal chemotherapy. Vital signs show a blood pressure of 90/60 mmHg, a temperature of 38.7 C, a pulse rate of 110/min, respiratory rate of 23/min, and oxygen saturation of 96% on room air. The abdominal examination does not reveal any abnormalities apart from a clean surgical scar. Which of the following is the best initial test in the evaluation of this patient?
Choices: 1. 2. 3. 4.
Complete blood count Abdominal CT scan Abdominal x-ray Urine microscopy
Answer: 1 - Complete blood count Explanations: Hyperthermic intraperitoneal chemotherapy (HIPEC) represents a concept of multimodal therapy increasingly used in the management of peritoneal carcinoma and more in the following decades. It is now an integral part of the treatment of primary and secondary peritoneal tumors. This involves administering cytotoxic agents into the peritoneal cavity at an elevated temperature (41 to 43°C) in order to promote their absorption by neoplastic nodules. Cytoreduction surgery (CRS) with HIPEC has been well established as the only treatment for mucinous tumors with peritoneal dissemination, based on retrospective and comparative studies (Chua et al.). It should be remembered that heavy and complex surgical procedures likely lead to increased toxicity of HIPEC through changes in pharmacokinetics, protein losses, hepatic and renal metabolic restrictions, and stress-related bone marrow suppression. This increased toxicity is manifested by hematotoxicity and nephrotoxicity with respective frequencies of 5.6% and 1.7% according to the literature. This hematotoxicity related to cytotoxic drugs of HIPEC manifests as pancytopenia and febrile neutropenia linked to microbial agents. An abdominal CT scan is not urgent because the abdominal examination is not worrying. Go to the next page if you knew the correct answer, or click the link image(s) below to further research the concepts in this question (if desired).
Research Concepts: Hyperthermic Intraperitoneal Chemotherapy
We update eBooks quarterly and Apps daily based on user feedback. Please tap flag to report any questions that need improvement.
Question 956:
A 48-year-old Caucasian woman with a past medical history of hypertension is evaluated because her right breast has been "bothering her." She has no other symptoms. She is afebrile at the time of the presentation. On physical examination, the right breast is diffusely erythematous, and the skin is thickened and indurated. No palpable axillary adenopathy was appreciated bilaterally. A diagnostic mammogram showed right breast skin thickening and trabecular thickening of the stroma. The left breast was normal. A punch biopsy of the right breast showed changes consistent with inflammatory breast cancer. She subsequently underwent a contrast-enhanced breast magnetic resonance imaging, which confirmed the absence of a discrete breast mass and absence of chest wall invasion. What is the next best step in management?
Choices: 1. Whole-body FDG-positron emission tomography/computed tomography 2. Nuclear scintigraphy for sentinel lymph node mapping and biopsy 3. Right radical mastectomy with axillary lymph node dissection 4. Initiate therapy with tamoxifen
Answer: 1 - Whole-body FDG-positron emission tomography/computed tomography
Explanations: Although the magnetic resonance imaging did not show any discrete mass within the breast or locally advanced disease, the patient requires further staging to assess for a possible distant metastatic disease, which would determine treatment approach and prognosis. The main role of positron emission tomographycomputed tomography is to stage inflammatory breast cancer, differentiate between curable and incurable disease (stage IV), and assess response and manage therapy. Sentinel lymph node biopsy is not a standard of care for inflammatory breast cancer. Patients who are surgical candidates should undergo a more aggressive approach, usually with radical mastectomy and lymph node dissection. The treatment approach should be tailored according to patient staging, among other factors. Additionally, tamoxifen would rarely be the treatment of choice, as most inflammatory breast cancers are hormonereceptor negative. Go to the next page if you knew the correct answer, or click the link image(s) below to further research the concepts in this question (if desired).
Research Concepts: Inflammatory Breast Cancer
We update eBooks quarterly and Apps daily based on user feedback. Please tap flag to report any questions that need improvement.
Question 957:
A 45-year-old female is seen in the outpatient clinic due to the incidental finding of a 1.7 cm x 1.9 cm isodense left lateral ventricle mass seen on the head computed tomographic scan with contrast done for workup for chronic sinusitis. On magnetic resonance imaging, the mass is located adjacent to the choroid plexus of the atrium of the left lateral ventricle and is isointense to gray matter on T1/T2 and has homogenous/vividly enhancing on postcontrast images. Also, there is evidence of fine calcified borders on the gradient echo sequence. No evidence of hydrocephalus or parenchymal edema is seen. On the physical exam, the patient is neurologically intact and only complains of a bifrontal headache. What is the best next step in management?
Choices: 1. Consult neurosurgery for an urgent operation 2. Observation 3. Radio-oncologist consult for elective radiosurgery 4. Obtain serum beta-human chorionic gonadotropin, alphafetoprotein, and placental alkaline phosphatase
Answer: 2 - Observation Explanations: Meningiomas are usually isointense to grey matter (6090%) on T1 and T2 sequences. Intraventricular meningiomas are mainly benign, slow-growing lesions. However, it is important to follow-up with serial imaging due to the risk of ventricular trapping and symptomatic parenchymal edema that may warrant surgical intervention. Meningiomas appear more common in the atrium on the left lateral ventricle and are well-circumscribed, homogeneously enhancing masses. Compared to extra-axial meningiomas, intraventricular meningiomas have a higher percentage of calcification (50 vs. 20%). Due to the absence of hydrocephalus, choroid plexus papilloma is unlikely. Due to the absence of adjacent brain parenchymal edema, choroid plexus carcinoma and brain metastasis are unlikely. There is no need for urgent surgery or radiosurgery. Go to the next page if you knew the correct answer, or click the link image(s) below to further research the concepts in this question (if desired).
Research Concepts: Intraventricular Meningioma
We update eBooks quarterly and Apps daily based on user feedback. Please tap flag to report any questions that need improvement.
Question 958:
A 42-year-old woman presents to the clinic for results from her core biopsy. She had initially presented with a solitary, painless, firm, and freely mobile 3 cm lesion, which on ultrasound appeared to be a homogeneously hypoechoic well-circumscribed solid mass. The results of the biopsy state that this is mass is a collagenized/fibrous, cellular, lipomatous, infiltrative, myxoid, epithelioid, and deciduoid-like variant microscopically, devoid of any lobule or ductal involvement; and macroscopically shows a well-demarcated pale pink or tan round mass. Which of the following is the most likely diagnosis?
Choices: 1. 2. 3. 4.
Lobular carcinoma in situ Ductal carcinoma in situ Invasive ductal carcinoma Myofibroblastoma
Answer: 4 - Myofibroblastoma Explanations: Histology indicators of benign tumors include - welldermacated lesions and an extracellular matrix of collagen. A wide local excision relates to a cancer or precancerous operation. An excision biopsy relates to benign breast disease. Myofibroblastomas are benign mesenchymal tumors that do not carry premalignant potential. Further information is required for axilla surgery, such as axillary ultrasound, grade, and staging of cancer. Myofibroblastomas are benign tumors with no malignant potential and therefore do not require axillary surgery, such as a sentinel lymph node biopsy. Go to the next page if you knew the correct answer, or click the link image(s) below to further research the concepts in this question (if desired).
Research Concepts: Breast Myofibroblastoma
We update eBooks quarterly and Apps daily based on user feedback. Please tap flag to report any questions that need improvement.
Question 959:
A 30-year-old man presents to the clinic with a 5- month history of left knee swelling and limited range of motion. He reports no obvious trauma but recalls falling 2 months ago. He decided to get it checked out because it was starting to interfere with his daily activities. He has no family history of similar incidences. On examination, there is a mild swelling and limited range of motion of the left knee, but no associated weakness. X-ray demonstrates eccentric, lytic lesion arising from the metaphysis. Pathology reveals polygonal mononuclear cells with a typical uniform distribution and was positive for H3FA. What is the next best step in the management of this patient?
Choices: 1. 2. 3. 4.
Neoadjuvant treatment Additional imaging Proceed with the surgery Obtain PTH level
Answer: 2 - Additional imaging Explanations: For patients diagnosed with giant cell tumor of bone (GCTB), the National Comprehensive Cancer Network (NCCN) recommends a chest CT. This is because it is a common site of metastasis. When imaged on x-ray and CT, GCTB tends to have four radiographic findings when located on a long bone. They are usually eccentric, have well defined non-sclerotic margins and occurs at closed growth plates and near the articular surface. MRI may be useful when evaluating unusual bone lesions. GCTB have low to intermediate signal intensity on T1-weighted images, heterogenous high signal intensity on T2, and demonstrates enhancement. Bone scintigraphy can be considered when suspecting GCTB. Particularly in early GCTB, bone scintigraphy is more sensitive than x-rays. Go to the next page if you knew the correct answer, or click the link image(s) below to further research the concepts in this question (if desired).
Research Concepts: Osteoclastoma
We update eBooks quarterly and Apps daily based on user feedback. Please tap flag to report any questions that
need improvement.
Question 960:
A 71-year-old man presents to the clinic for evaluation. He was diagnosed with stage 4 pancreatic cancer four months ago and has been pursuing palliative care options. His current medications include a regular dose of morphine and as needed hydromorphone/acetaminophen for cancer-related pain. His primary complaint today is incapacitating fatigue. Which of the following is the next best step in the management of this patient?
Choices: 1. 2. 3. 4.
Sertraline Stop morphine Midazolam Prednisone
Answer: 4 - Prednisone Explanations: The patient is describing cancer-related fatigue, a common bothersome symptom for end-stage cancer patients. Commonly prescribed therapies include prednisone, methylphenidate, and modafinil. Of these three agents, prednisone is tried first. Persistent symptoms may be managed with methylphenidate or modafinil. Stopping morphine would likely result in a worsening of pain for the patient. Sertraline and midazolam are not used for cancer fatigue. Go to the next page if you knew the correct answer, or click the link image(s) below to further research the concepts in this question (if desired).
Research Concepts: End of Life Care
We update eBooks quarterly and Apps daily based on user feedback. Please tap flag to report any questions that need improvement.
Question 961:
A 65-year-old man with a history of carcinoid tumor presents to the clinic with excessive sweating, heat intolerance, weight loss, fatigue, and weakness. He has been on interferon-alpha therapy for the past year. Which of the following therapies is most appropriate for this patient?
Choices: 1. 2. 3. 4.
Radioactive iodine Acetaminophen Methylphenidate Ticagrelor
Answer: 1 - Radioactive iodine Explanations: This patient is experiencing symptoms of hyperthyroidism (excessive sweating, heat intolerance, weight loss, fatigue, and weakness). Hyperthyroidism is commonly treated with radioactive iodine. Interferon-alpha therapy is commonly associated with thyroid dysfunction (hyper and hypothyroidism). Therefore, it is critical to monitor the thyroid hormone levels of patients on Interferon-alpha therapy. Acetaminophen may be given to this patient for pain relief and symptomatic care. However, it would not directly treat this patient's hyperthyroidism. Methylphenidate is used to treat ADHD, and ticagrelor is an antiplatelet agent. Go to the next page if you knew the correct answer, or click the link image(s) below to further research the concepts in this question (if desired).
Research Concepts: Interferon
We update eBooks quarterly and Apps daily based on user feedback. Please tap flag to report any questions that need improvement.
Question 962:
A 75-year-old man presented to the emergency department with a 2-month history of a large right-sided nodal mass, throat pain, and hoarseness. He is a life-long smoker (started at age 15) with a history of significant chronic obstructive pulmonary disease, congestive cardiac failure, liver cirrhosis (due to continued alcohol consumption). He is known to have tested positive for human papillomavirus. He is capable of limited self-care. Referred to otolaryngology, a flexible laryngoscopy and CT scan of neck and thorax demonstrated a large glottic mass with extension through the thyroid cartilage and prevertebral space, in addition to multiple bilateral lymph nodes with indistinct nodal margins. Which of the following are the two most significant independent risk factors in this patients' history to consider for laryngeal cancer?
Choices: 1. 2. 3. 4.
Smoking and alcohol consumption Chronic obstructive pulmonary disease and smoking Human papillomavirus Liver cirrhosis and low socioeconomic status
Answer: 1 - Smoking and alcohol consumption Explanations: Smoking and alcohol have the most significant impact on laryngeal cancer, and in combination, can have a multiplicative effect. Although human papillomavirus is implicated in oropharyngeal cancer, no strong associations between this disease and laryngeal cancer have been demonstrated. Low socioeconomic status is associated with a later stage of presentation for laryngeal cancer. The presence of HPV may significantly alter the nodal (N) status of oropharyngeal cancer, but this is not reflected in laryngeal TNM staging. Go to the next page if you knew the correct answer, or click the link image(s) below to further research the concepts in this question (if desired).
Research Concepts: Glottic Cancer
We update eBooks quarterly and Apps daily based on user feedback. Please tap flag to report any questions that need improvement.
Question 963:
A 65-year-old female presents to the hospital with unexplained fever, drenching night sweats, and significant weight loss over the past four months. On physical examination, her vital signs are normal. There is no peripheral lymphadenopathy or hepatosplenomegaly. Computed tomography (CT) scan reveals retroperitoneal lymphadenopathy. Histopathological examination of the enlarged lymph nodes reveals malignant cells with abundant slightly basophilic cytoplasm and a distinctive binucleate morphology resembling an owl's eye appearance. Which of the following cluster of differentiation (CD) markers is likely to be seen on the immunohistochemical evaluation of these cells?
Choices: 1. 2. 3. 4.
CD45 CD20 CD30 CD56
Answer: 3 - CD30 Explanations: Classic diagnostic Reed-Sternberg cells for Hodgkin lymphoma are malignant cells with abundant slightly basophilic cytoplasm and a distinctive bi-nucleate morphology resembling an owl's eye appearance. On immunohistochemical staining, these cells are positive for CD30 in all cases. Most are positive for CD15 as well. The characteristic immunophenotype of classic Hodgkin lymphoma is the expression of CD30 and CD15, but not CD20. This distinguishes classic Hodgkin lymphoma from normal B lymphocytes and from other types of lymphoma. Characteristic immunophenotype also notes an absence of CD45 in these patients. An excisional lymph node biopsy has been recommended to confirm the diagnosis. A core-needle biopsy may be performed if excisional lymph node biopsy is not feasible, and may be adequate if diagnostic. Immunohistochemical stains for other B cell markers and T cell antigens are usually negative in these cases. Go to the next page if you knew the correct answer, or click the link image(s) below to further research the concepts in this question (if desired).
Research Concepts: Lymphocyte Depleted Hodgkin Lymphoma
We update eBooks quarterly and Apps daily based on user feedback. Please tap flag to report any questions that need improvement.
Question 964:
A 66-year-old man with a past medical history of metastatic colon cancer, heart failure of reduced ejection fraction (EF 15%), and poorly controlled diabetes is brought to the emergency department for severe chest pain. He is found to be having unstable angina (UA) and decompensated heart failure. He currently has ongoing issues with central chest pain and shortness of breath at rest. The patient himself cannot speak due to his shortness of breath. Given his extensive comorbidities and poor prognosis, his family, including the medical power of attorney is interested in "keeping him comfortable" rather than additional interventions. Which of the following is the next best step in the management of this patient?
Choices: 1. Heparin infusion only 2. Cardiac catheterization 3. Supplemental oxygen only 4. Patient-controlled analgesia with hydromorphone and supplemental oxygen
Answer: 4 - Patient-controlled analgesia with hydromorphone and supplemental oxygen
Explanations: The patient has unstable angina and decompensated heart failure. However, the patient's family (acting as surrogates) has decided to pursue comfort measures only and their decision must be honored. Controlling the patient's pain and shortness of breath without any other interventions with patient-controlled analgesia and the use of supplemental oxygen would be most prudent as it is more in line with their wishes. Although death is imminent without medical management or cardiology evaluation for stenting, the goal, in this case, is keeping the patient comfortable until he passes away. Patients do not necessarily need to given consent for palliative care and hospice by themselves. If patients are obtained or nonverbal, legal surrogates and family members can have an end of life discussion with physicians. Heparin infusion would be the next step in a patient in whom we are pursuing full medical management. Go to the next page if you knew the correct answer, or click the link image(s) below to further research the concepts in this question (if desired).
Research Concepts: Palliative Sedation In Patients With Terminal Illness
We update eBooks quarterly and Apps daily based on user feedback. Please tap flag to report any questions that need improvement.
Question 965:
A 65-year-old man presents to the clinic for a skin rash. Examination shows violaceous, psoriasiform plaques over his fingers, ears, and nose. He also reports a recent history of dyspnea and significant weight loss. The patient is a chronic smoker. A skin biopsy shows spongiosis, interface dermatitis, and mild dyskeratosis. Further evaluation shows that the patient has squamous cell carcinoma of the lung. Which of the following is the most appropriate investigation to confirm the dermatologic diagnosis in this patient?
Choices: 1. 2. 3. 4.
Immunofluorescence/ immunohistochemistry Fungal stains Special stains for mucin Special stains for Iron
Answer: 1 - Immunofluorescence/ immunohistochemistry
Explanations: The morphology of the skin lesions, location and the presence of associated squamous cell carcinoma of the lung, strongly suggests the possibility of acrokeratosis paraneoplastica (AP). Acrokeratosis paraneoplastica, also called arokeratosis neoplastica or Bazex syndrome, is characterized by psoriasiform lesions on the acral areas of the body, associated with internal malignancy. The most common associated malignancy is squamous cell carcinoma of the upper aero-digestive tract. While the clinical features, histology and the associated internal malignancy is enough to confirm the diagnosis, an additional investigation which can further corroborate the diagnosis is the demonstration of C3 and Immunoglobulins, along the basement membrane. Go to the next page if you knew the correct answer, or click the link image(s) below to further research the concepts in this question (if desired).
Research Concepts: Acrokeratosis Paraneoplastica
We update eBooks quarterly and Apps daily based on user feedback. Please tap flag to report any questions that
need improvement.
Question 966:
A 50-year-old man presents with his family after he is found with an altered state of consciousness. His past medical history includes malignant melanoma that was diagnosed two years ago. He was treated with surgical removal of the tumor, followed by six monthly PET scans. However, he missed the last two of his PET scan appointments. His vital signs are within normal limits. A brain MRI is ordered to rule out cerebral metastasis. In what part of the brain would metastatic disease most likely be found in this patient?
Choices: 1. 2. 3. 4.
Frontal lobes Gray-white junction Periventricular areas Cerebellum
Answer: 2 - Gray-white junction Explanations: Melanoma often metastasizes via the vascular spread and can deposit in the brain. It is usually found at the gray-white junction where vascular networks deposit blood-borne cells. Brain metastasis correlates to a poor prognosis. If present, melanoma often reveals multiple areas of metastasis in the brain. They are seen on MRI as hyperintense lesions on T1 weighted images and hypointense lesions on T2 weighted images. Tumor cells are highly metabolic and often outgrow their blood supply leading to central necrosis and hemorrhage. Hemorrhages can be seen both on CT and MRI images. The cerebral hemispheres and cerebellar hemispheres are equally affected by the metastatic spread. Go to the next page if you knew the correct answer, or click the link image(s) below to further research the concepts in this question (if desired).
Research Concepts: Malignant Melanoma
We update eBooks quarterly and Apps daily based on user feedback. Please tap flag to report any questions that need improvement.
Question 967:
A 49-year-old patient presents to the dermatology clinic with a lesion on their arm that is growing in size, has irregular borders, and variegated pigmentation. Physical examination reveals that it is about 0.5 cm in diameter. The general body examination is performed to check the entire body of the patient for any suspicious lesions, but no significant abnormality could be found. Which of the following measures is most likely to diagnose this lesion?
Choices: 1. Shave biopsy 2. Curettage 3. Punch biopsy 4. Take the patient to the operating room and excise with 1 cm margins with sentinel lymph node biopsy
Answer: 3 - Punch biopsy Explanations: The sign and symptoms in this patient are suggestive of a melanoma. Melanoma is a skin cancer of the melanocytes that occurs after DNA mutation, most often secondary to excess sun exposure. Fair-skinned and light-haired persons living in high sun-exposure environments are at greatest risk. Clinically, melanoma exhibits shape irregularity, irregular color, and asymmetry. Sometimes, melanoma exhibits ulceration and bleeding, which is associated with a poorer prognosis. A punch biopsy often reveals atypical nests of melanocytes that accumulate and coalesce at the dermo-epidermal junction. The depth of melanoma is the most important prognostic factor. The physical exam should consist of checking the entire body of the patient for any suspicious lesions. Specifically, the ABCDEs should be assessed: asymmetry, border irregularity, color (variation), diameter ( more than 5 mm), and/or erythema. Any positivity in the exam should raise suspicion for malignant melanoma, and then the practitioner should obtain a biopsy of the lesion. It is important not to perform a shave biopsy on lesions suspected to have malignant melanoma but rather perform a punch biopsy or an excisional biopsy if the lesion is less than 7 mm. Shave biopsy does not allow to assess the entire depth and may ruin any chance of proper staging. Go to the next page if you knew the correct answer, or click the link image(s) below to further research the concepts in
this question (if desired).
Research Concepts: Melanoma Pathology
We update eBooks quarterly and Apps daily based on user feedback. Please tap flag to report any questions that need improvement.
Question 968:
A 62-year-old male is brought to the emergency department following a witnessed seizure in the grocery store. He is currently alert, and his orientation is improving. Finger-stick blood glucose upon arrival was 112 mg/dL, with no dextrose administered en route. Upon questioning, the clinician discovers this has never happened before, and he has no other significant medical or family history. He undergoes a CT of the head that demonstrates a frontal lobe parenchymal mass with scattered coarse and gyriform calcifications. After it is determined there isn't any enhancement on MRI, what is the next test that will aid the most in making the diagnosis?
Choices: 1. 2. 3. 4.
Magnetic resonance spectroscopy Tumor genetic testing Tumor histologic markers Tumor cytology and architecture
Answer: 2 - Tumor genetic testing Explanations: A nonenhancing, parenchymal, seizure producing central nervous system tumor is most likely a low-grade glioma in a patient without a history of cancer. Of these, a frontal lobe mass with coarse calcifications is most likely an oligodendroglioma. The best way to determine this from other gliomas is through genetic marker testing. Oligodendrogliomas are defined by an isocitrate dehydrogenase (IDH) mutation and a 1p/19q codeletion. This combination is absent in the other gliomas such as astrocytoma and glioblastoma. Genetic testing plays a prognostic role in diagnosing these tumors, as those that have a mutated IDH and 1p/19q codeletion respond better to chemotherapy, and are less aggressive than those with wild-type IDH and lacking the codeletion. Oligodendrogliomas often share many histological markers with the other glial tumors, such as glial fibrillary acidic protein (GFAP) found in astrocytomas. Their cytology and cell architecture is not definitive in diagnosing these tumors. Go to the next page if you knew the correct answer, or click the link image(s) below to further research the concepts in this question (if desired).
Research Concepts: Oligodendroglioma
We update eBooks quarterly and Apps daily based on user feedback. Please tap flag to report any questions that need improvement.
Question 969:
A 72-year-old man with no significant history presented with worsening fatigue, weight loss, and night sweats. Physical exam showed cervical, axillary, and inguinal lymph node enlargement with no tenderness. Excisional biopsy from the inguinal lymph node and microscopic exam showed monomorphic large atypical cells with a high N:C ratio, prominent nucleoli, and some necrosis. Immunohistochemistry staining was positive for CD20, CD5, and cyclin D1 with Ki-67 of 82%. CD3, CD10, CD30, and ALK-1 were negative. What is the diagnosis of this patient?
Choices: 1. 2. 3. 4.
Diffuse large B-cell lymphoma Mantle cell lymphoma Follicular lymphoma Anaplastic large cell lymphoma
Answer: 2 - Mantle cell lymphoma Explanations: CD20, CD5, and cyclin D1 positivity highly suggest mantle cell lymphoma. Immunohistochemistry findings are consistent with mantle cell lymphoma. Follicular lymphoma is low-grade lymphoma and cyclin D1 negative. CD30 is negative, suggesting anaplastic large cell lymphoma is less likely. Go to the next page if you knew the correct answer, or click the link image(s) below to further research the concepts in this question (if desired).
Research Concepts: Mantle Cell Lymphoma
We update eBooks quarterly and Apps daily based on user feedback. Please tap flag to report any questions that need improvement.
Question 970:
A 65-year-old male currently being treated for chronic cancer pain is brought to the emergency department by EMS after being found unresponsive by family members at home. The family reports that the patient currently takes polyethylene glycol for constipation, erythromycin for recent pneumonia, and transdermal fentanyl in addition to the cancer regime. His heart rate is 102 bpm, respiratory rate is 6/min, the temperature is 98.6 F, blood pressure is 118/76 mmHg. On physical exam, the patient is non-responsive, pupils are midline and pinpoint bilaterally. The transdermal fentanyl patch is intact on the right chest. What is the primary differential the clinician should consider?
Choices: 1. 2. 3. 4.
Opioid toxicity Intentional opioid overdose Stroke Inappropriate transdermal fentanyl patch administration
Answer: 1 - Opioid toxicity Explanations: The patient is exhibiting symptoms of opioid toxicity. The respiratory rate is 6/min, demonstrating respiratory depression, and pupils are pinpoint. In addition, the patient recently began a trial of erythromycin, a strong CYP3A4 inhibitor. CYP3A4 inhibitors reduce the metabolism of fentanyl and cause increased concentrations of fentanyl in the blood, which will lead to toxicity. Intentional opioid overdose is unlikely in this patient. Only one patch was applied to clean, intact, hairless skin. Also, the scenario upon which this has occurred points towards inhibition of CYP3A4 by erythromycin leading to increased fentanyl concentration, causing toxicity over intentional overdose. Hypercoagulability caused by malignancy is a cause for concern. However, the patient's pupils are pinpoint and midline. If the primary differential were to be a stroke, the patient's pupils would be enlarged. The transdermal fentanyl patch was administered properly. The patch must be applied to intact, dry, clean skin, with minimal hair. This patient has it placed in an optimal location (the chest). Patches that are placed on heated surfaces, or to broken skin would be considered suboptimal and must not be used. Go to the next page if you knew the correct answer, or click the link image(s) below to further research the concepts in this question (if desired).
Research Concepts:
Fentanyl Transdermal
We update eBooks quarterly and Apps daily based on user feedback. Please tap flag to report any questions that need improvement.
Question 971:
A 65-year-old woman presents to the hospital with complaints of fever, chills, faintness, chest pain, and shortness of breath for one day. The patient has been recently diagnosed with Hodgkin lymphoma and is on chemotherapy since then. The clinical symptoms of the patient are most likely due to the adverse effects of the drug. What test is routinely performed before starting the patient on this drug?
Choices: 1. 2. 3. 4.
Chest imaging Renal function tests Pulmonary function tests Complete blood count
Answer: 3 - Pulmonary function tests Explanations: Bleomycin belongs to a subfamily of glycopeptide antibiotics and is utilized primarily as an antineoplastic agent. Bleomycin is used clinically as part of combination cytotoxic chemotherapy regimens including ABVD (Adriamycin, Bleomycin, Vinblastine, Dacarbazine). It was first approved by the FDA in 1975 for the treatment of squamous cell carcinomas, malignant lymphomas, and testicular cancers. Bleomycin has since been shown to have a wide array of FDA-approved therapeutic indications, including germinal cell tumors, gestational trophoblastic disease, Hodgkin lymphoma, and non-Hodgkin lymphoma. NonFDA approved indications include AIDS-associated Kaposi sarcoma, osteosarcoma, malignant melanoma, and advanced stages of mycosis fungoides. It is recommended liver enzymes, blood counts, plasma proteins, and electrolytes, it is advisable that providers acquire periodic chest imaging of patients receiving bleomycin. Pulmonary function tests are performed before administering the drug and are not recommended as a routine investigation. Renal function tests are performed in patients having renal toxicity or showing signs of renal failure. Go to the next page if you knew the correct answer, or click the link image(s) below to further research the concepts in this question (if desired).
Research Concepts:
Bleomycin
We update eBooks quarterly and Apps daily based on user feedback. Please tap flag to report any questions that need improvement.
Question 972:
A 50-year-old woman found to have calcifications on screening mammography undergoes a core needle biopsy. Subsequent excisional biopsy of her left breast confirms the same pathology. The findings on the pathology slide demonstrate cytonuclear and architectural atypia with partial involvement of the ducts and a small size of 2 mm. There is no central necrosis or significant nuclear atypia. Which of the following best describes her breast cancer risk?
Choices: 1. 100% in the left breast, 5-fold increase in future cancer risk in the right breast 2. 100% the left breast, no increase in future cancer risk in the right breast 3. 5-fold increase in future cancer risk in the left breast, no increased risk in the right breast 4. 5-fold increase in future cancer risk in either breast
Answer: 4 - 5-fold increase in future cancer risk in either breast
Explanations: Atypical ductal hyperplasia is the patient's diagnosis. It is not cancer. Atypical ductal hyperplasia is a high-risk lesion and NOT a precancerous lesion. Therefore her increased risk is not at the site of the biopsy. There is a 5-fold increased risk of breast cancer in patients with confirmed ADH. The increased risk of cancer is cancer in any breast and any portion of the breast. Go to the next page if you knew the correct answer, or click the link image(s) below to further research the concepts in this question (if desired).
Research Concepts: Atypical Ductal Hyperplasia
We update eBooks quarterly and Apps daily based on user feedback. Please tap flag to report any questions that need improvement.
Question 973:
A 45-year-old female presented with a complaint of chronic abdominal pain, abdominal distention, and malaise for the past 2 months. She also reports unintentional weight loss. On abdominal examination, percussion revealed tympanic resonance in the hypogastric region. Abdominopelvic ultrasonography reveals echogenic peritoneal masses, scalloping of the liver, and an appendiceal mucocele. Magnetic resonance imaging (MRI) shows a 10 cm x 8 cm x 5 cm fluid mass covering the uterus and the ovaries as well as a dilated appendix filled with fluid. A laparoscopic biopsy is done. Which histological finding would have the worst prognosis?
Choices: 1. Disseminated peritoneal adenomucinosis 2. Peritoneal mucinous carcinomatosis without signet ring cells 3. Ovarian cancer 4. Peritoneal mucinous carcinomatosis with signet ring cells
Answer: 4 - Peritoneal mucinous carcinomatosis with signet ring cells
Explanations: Pseudomyxoma peritonei is a rare clinical entity, characterized by diffuse intra-abdominal gelatinous ascites, the so-called “jelly belly,” with mucinous implants on peritoneal surfaces. Any lesion with a component of signet ring cells is classified separately because of their worse prognosis compared to other subgroups of otherwise similar histology. Signet ring cells mainly come from GI malignancies including stomach, colon, and appendiceal adenocarcinoma. Peritoneal mucinous carcinomatosis generally presents with a worse prognosis than disseminated peritoneal adenomucinosis and acellular mucin because of a higher grade of cellular components with atypia and mitotic activities. Go to the next page if you knew the correct answer, or click the link image(s) below to further research the concepts in this question (if desired).
Research Concepts: Pseudomyxoma Peritonei
We update eBooks quarterly and Apps daily based on user feedback. Please tap flag to report any questions that need improvement.
Question 974:
Using the American Joint Committee on Cancer classification for head and neck lymph node location, what is the level of a submental node?
Choices: 1. 2. 3. 4.
I II III IV
Answer: 1 - I Explanations: The AJCC lymph node classification system has ten levels. Levels II, III, IV, and V are bilaterally symmetric. Levels I, VI, and VII are midline. Level I is submental and divided into A and B, A being medial to the anterior belly of the digastric muscle, formerly termed "submental lymph nodes" IB, more lateral, were formerly termed "submandibular lymph nodes." Go to the next page if you knew the correct answer, or click the link image(s) below to further research the concepts in this question (if desired).
Research Concepts: Anatomy, Head and Neck, Lymph Nodes
We update eBooks quarterly and Apps daily based on user feedback. Please tap flag to report any questions that need improvement.
Question 975:
A 32-year-old man presents to the hospital with complaints of weight gain despite a restricted diet and episodes of lightheadedness and sweating for the past six months. The episodic events of sweating and lightheadedness have increased in frequency over the past four months. The patient also admits to blurred vision during these episodes. Each episode lasts about 5 to 10 minutes and improves after the patient eats a quick snack. He drinks red wine once a month and does not smoke. He does not take any medication, and his family history is unremarkable. His blood pressure is 130/85 mmHg, and his pulse is 82/min and regular. Physical examination is within normal limits. What is the best next step in managing this patient's condition?
Choices: 1. Measurement of plasma fractionated metanephrine levels during the episodes 2. Measurement of cortisol levels during the episodes 3. Measurement of thyroid-stimulating hormone during the episodes 4. Measurement of patient's blood glucose levels during the episodes
Answer: 4 - Measurement of patient's blood glucose levels during the episodes
Explanations: Insulinomas are predominantly derived from pancreatic islet cells. It is a neuroendocrine tumor that produces insulin in excess. The hormones may be secreted in short bursts resulting in fluctuations in blood glucose. Presenting complaints in about 85% of patients are episodic blurred vision, diplopia, sweating, weakness, and palpitations. The combination of weight gain and hypoglycemia makes it important to investigate insulinoma as a potential culprit. The best next step is measuring blood glucose levels during the episodes. Only 10% of insulinomas are malignant. Normally insulin and C-peptide are secreted in equal quantities as they are products of proinsulin. Some insulinomas also secrete gastrin, ACTH, 5-hydroxyindole acid, hCG, and somatostatin. Hyperthyroidism can cause symptoms of episodic sweating, blurred vision, and palpitations, but it would not explain the weight gain and resolution of symptoms after having a snack. Go to the next page if you knew the correct answer, or click the link image(s) below to further research the concepts in this question (if desired).
Research Concepts: Insulinoma
We update eBooks quarterly and Apps daily based on user feedback. Please tap flag to report any questions that need improvement.
Question 976:
A 24-year-old man reports to the clinic with a complaint of vague pain in the thigh for the past two years. His father has died of leukemia, and his brother has adrenal carcinoma. His 31-years-old sister has been diagnosed to have breast carcinoma. The patient is investigated and found to have a bone tumor related to a genetic syndrome. Which of the following tumors is most likely in this patient?
Choices: 1. 2. 3. 4.
Osteosarcoma Chondrosarcoma Giant cell tumor Ewing sarcoma
Answer: 1 - Osteosarcoma Explanations: The patient is having a familial cancer syndrome, probably Li-Fraumeni syndrome. It is also known as the sarcoma, breast, leukemia, and adrenal gland (SBLA) cancer syndrome. P53-related cancers usually include soft tissue sarcoma, osteosarcoma (bone cancer), leukemia, brain tumors, adrenocortical carcinoma (cancer of the adrenal glands), and other cancers. Li-Fraumeni syndrome (LFS) is an inherited autosomal dominant disorder that is usually associated with abnormalities in the tumor suppressor protein P53 gene (TP53) located on chromosome 17p13. Go to the next page if you knew the correct answer, or click the link image(s) below to further research the concepts in this question (if desired).
Research Concepts: Li-Fraumeni Syndrome
We update eBooks quarterly and Apps daily based on user feedback. Please tap flag to report any questions that need improvement.
Question 977:
A 64-year-old male walked to the emergency department to report an acute onset of severe low back pain. He was barely able to stand upright. His past medical history is remarkable for prostate cancer. On exam, he had weakness of his legs at 3 to 4 out of 5, limited by pain and proximal worse than distal. He had no rectal tone. MRI of the spine showed multiple enhancing lesions in the lumbar spine with obliteration of the central canal at the L4L5 area. What is the most likely diagnosis in the patient?
Choices: 1. 2. 3. 4.
Extruded disc Compression fracture Metastatic deposits on the spine Vertebral body hemangiomas
Answer: 3 - Metastatic deposits on the spine Explanations: A cauda equina syndrome is a clinical diagnosis. Metastatic deposits are the most common type of malignancy observed in the spine. For patients with acute back pain, acute neurological deficit, and history of cancer, metastatic cancer should be considered high in the differential diagnosis. For patients with acute neurological deficit and radiographic findings of the spinal cord or nerve root compression, emergent surgical consultation is indicated. Go to the next page if you knew the correct answer, or click the link image(s) below to further research the concepts in this question (if desired).
Research Concepts: Spinal Metastasis
We update eBooks quarterly and Apps daily based on user feedback. Please tap flag to report any questions that need improvement.
Question 978:
A 65-year-old female presents to the clinic with a history of a non-tender, palpable left breast mass. It has been present for 3 years, and it has not changed in size. She has no records of prior breast mammograms or ultrasounds. On physical exam, there is a palpable lump in the upper inner quadrant of the left breast. There are no overlying skin changes, and no abnormalities are found in the right breast. There is no axillary or cervical lymphadenopathy. The patient undergoes a mammogram and breast ultrasound for evaluation. The left breast mammogram demonstrates a lobulated, heterogeneous 5.0 cm spiculated mass with irregular margins in the superior medial quadrant without any calcifications. Ultrasound reveals a hypoechoic mass with posterior shadowing and irregular margins but no hypervascularity. The lesion is classified as a breast imaging-reporting and data system (BIRADS): 4, and the lesion is biopsied. Histopathology demonstrates solid nests of tumor cells with abundant, eosinophilic granular cytoplasm separated by fine fibrous stroma. The cells stain positively for S-100 and CD68 and negatively for Beta-hydroxy-beta-methyl butyrate (HMB-45). The granules are periodic acid-Schiff positive and diastaseresistant. What additional diagnostic findings or history will dictate surgical management for this patient?
Choices: 1. History of Noonan syndrome, neurofibromatosis I, or lentigines on the head and neck, electrocardiogram abnormalities, ocular hypertelorism, pulmonary stenosis, abnormal genitalia, retardation of growth, deafness (LEOPARD syndrome) 2. Molecular tests on tissue revealing the presence of mutations in Ras/MAP signaling pathway
. The monotony of tumor cells, lack of tumor capsule, focal 3 stromal infiltration, negative staining with cytokeratins, and location of the tumor near the supraclavicular nerve 4. Increased number of mitotic figures, presence of necrosis, large areas of spindled cells, increased nuclear to cytoplasmic ratios, atypical or pleomorphic nuclei, presence of nucleoli, and large size of the tumor
Answer: 4 - Increased number of mitotic figures, presence of necrosis, large areas of spindled cells, increased nuclear to cytoplasmic ratios, atypical or pleomorphic nuclei, presence of nucleoli, and large size of the tumor
Explanations: This patient has histologic findings diagnostic for a granular cell tumor. It is not yet clear, however, whether the patient has a benign or malignant granular cell tumor. The presence of a malignant granular cell tumor will prompt the surgeon to consider a sentinel lymph node biopsy to evaluate for metastasis, as the malignant tumors behave aggressively and metastasize often. The features which are classically used to distinguish benign from malignant granular cell tumors include (1) increased nuclear to cytoplasmic ratio, (2) necrosis, (3) spindled cells, (4) vacuolated nucleus with the large nucleolus, (5) nuclear pleomorphism, and (6) increased mitotic figures (greater than 2 per 10 highpower fields). Meeting 3 of these criteria warrants a diagnosis of malignancy, 1 or 2 warrants a diagnosis of atypical, and a 0 warrants a diagnosis of benign. Some authors have proposed formal consideration of Ki-67 and clinical presentation (presence of metastasis at diagnosis) in diagnosing malignant lesions. Granular cell tumors of the breast account for up to 15% of all granular cell tumors and most commonly occur in women from 40 to 60 years of age. They frequently occur as painless, palpable masses in the upper inner quadrant, presumably due to its proximity to the supraclavicular nerve. Clinically, these tumors can be mistaken for invasive mammary carcinoma, as they can cause skin retraction or nipple inversion. Benign lesions
are locally excised with clear margins, while malignant tumors prompt additional consideration of a lymph node biopsy. Radiologically, granular cell tumors of the breast are often confused for invasive carcinomas. They are frequently seen on mammography as spiculated or stellate masses with irregular borders. On ultrasound, they present as hypoechoic or hyperechoic masses with posterior shadowing and indistinct margins. It is often not until a biopsy that granular cell tumors are diagnosed. Although Noonan syndrome, neurofibromatosis I, and LEOPARD syndrome are reportedly associated with multiple granular cell tumors, the presence or absence of these syndromes would not influence the immediate surgical planning in this patient. The presence of mutations in the Ras/MAP pathway has been implicated in patients with multiple benign granular cell tumors. Finding these mutations in the biopsy sample may point the surgeon toward an extensive physical exam for additional synchronous tumors but would not upstage the current breast mass from benign to malignant. Finally, cellular monotony, lack of tumor capsule, focal stromal infiltration, negative cytokeratins, and location near the supraclavicular nerve are all found in benign granular cell tumors and would not upgrade this tumor to its malignant counterpart. Go to the next page if you knew the correct answer, or click the link image(s) below to further research the concepts in this question (if desired).
Research Concepts: Granular Cell Tumor
We update eBooks quarterly and Apps daily based on user feedback. Please tap flag to report any questions that need improvement.
Question 979:
A 20-year-old woman presents to the clinic for a wellness visit. She has no complaints but is concerned because her brother was recently diagnosed with a giant cell tumor of bone (GCTB). The physical exam is unremarkable, and there is no additional history that is concerning. The patient is reassured, and risk factors are reviewed with the patient. What abnormal laboratory finding is most often associated with the primary risk factor of GCTB?
Choices: 1. 2. 3. 4.
Elevated Elevated Elevated Elevated
acid phosphatase serum calcium parathyroid hormone alkaline phosphatase
Answer: 4 - Elevated alkaline phosphatase Explanations: While risk factors for giant cell tumor of bone (GCTB) are not fully understood, there are increased incidents in patients with Paget disease. Paget disease is associated with elevated alkaline phosphate levels. In Paget disease, GCTB typically develops in the pelvis or skull. Multiple GCTB lesions are rare and, when they occur, are usually associated with Paget disease. Go to the next page if you knew the correct answer, or click the link image(s) below to further research the concepts in this question (if desired).
Research Concepts: Osteoclastoma
We update eBooks quarterly and Apps daily based on user feedback. Please tap flag to report any questions that need improvement.
Question 980:
A 65-year-old woman presents to the clinic with a one-year history of fatigue and appetite loss. She denies any other symptoms, and there she has no significant past medical history. On examination, multiple enlarged lymph nodes are noted in the supraclavicular and axillary areas. An abdominal examination reveals hepatosplenomegaly. A diagnosis of chronic lymphocytic leukemia is made after work-up, and she is started on a combination of fludarabine, cyclophosphamide, and rituximab (FCR). Chest x-ray is normal, and baseline serum immunoglobulin levels are within the normal range. Serologies for hepatitis B, C, and HIV are negative. What prophylactic therapy should be initiated for this patient?
Choices: 1. 2. 3. 4.
Isoniazid Entecavir Trimethoprim-sulfamethoxazole No antimicrobial prophylaxis is necessary
Answer: 3 - Trimethoprim-sulfamethoxazole Explanations: Pneumocystis jirovecii prophylaxis is recommended in patients with chronic lymphocytic leukemia who are recieving FCR (fludarabine, cyclophosphamide, rituximab). Trimethoprim-sulfamethoxazole is the first choice for primary prophylaxis of Pneumocystis jirovecii during FCR treatment and for up to a year after completion. In patients with granulomatosis with polyangiitis (GPA) and microscopic polyangiitis (MPA), prophylaxis is recommended during FCR treatment to six months after treatment completion. Entecavir is recommended if a patient is positive for HBV DNA. Go to the next page if you knew the correct answer, or click the link image(s) below to further research the concepts in this question (if desired).
Research Concepts: Rituximab
We update eBooks quarterly and Apps daily based on user feedback. Please tap flag to report any questions that need improvement.
Question 981:
A 20-year-old man presents to the clinic with a 10-month history of swelling and pain in the right knee. On examination, there is a mild tenderness on the knee joint. The distal neurovascular status is intact. Radiographs reveal a subchondral lytic lesion. Biopsy shows a benign lesion with multiple giant cells. CT chest is suggestive of pulmonary metastasis. Which of the following is the most likely diagnosis?
Choices: 1. 2. 3. 4.
Tenosynovial giant cell tumor Giant cell tumor Osteochondroma Chrondromyxoid fibroma
Answer: 2 - Giant cell tumor Explanations: Giant cell tumor is aggressive benign lesions according to Enneking classification. Giant cells are the hallmark of this tumor. Giant cell tumors metastasize to the lungs. The incidence of spread is between 1-2%. Excision and chemotherapy are the treatment option for metastasis. Osteochondroma and fibroma do not spread to the viscera. Tenosynovial giant cell tumor does not metastasize to the lungs. Go to the next page if you knew the correct answer, or click the link image(s) below to further research the concepts in this question (if desired).
Research Concepts: Giant Cell Tumor
We update eBooks quarterly and Apps daily based on user feedback. Please tap flag to report any questions that need improvement.
Question 982:
A 75-year-old white man presents to dermatology with a 5-month history of an ulcerated erythematous plaque on his forehead. Biopsy examination reveals a dermal-based pleomorphic and atypical spindle cell tumor with numerous atypical mitoses. By immunohistochemistry, the lesion expresses CD-10 and procollagen and is negative for cytokeratin, CD34, SRY-Box 10 (SOX10), and melanoma antigen recognized T cells–1 (MART-1). Which of the following is the least likely characteristic of this disease?
Choices: 1. 2. 3. 4.
Severe solar-induced damage to the skin is a risk factor Erythematous dome-like growth is typical These tumors often contain melanin pigment These tumors are rarely metastatic at diagnosis
Answer: 3 - These tumors often contain melanin pigment
Explanations: Atypical fibroxanthoma (AFX) is a spindle cell tumor that arises in sun-exposed areas in the elderly. It clinically presents with an erythematous nodule and histologically is strikingly atypical and pleomorphic. Before the clinical entity of atypical fibroxanthoma was defined, many of the lesions were mistakenly termed atypical spindle cell melanoma. AFX generally exhibits indolent behavior, and excision is curative. It must be differentiated from pleomorphic dermal sarcoma, which is histologically similar but deeper and is more aggressive. Immunoperoxidase studies are useful in clarifying the diagnosis. Atypical fibroxanthoma marks with antibodies to CD10 and procollagen and is negative for cytokeratin and melanocyte markers. Excluding melanoma and squamous cell carcinoma is crucial to making the diagnosis. These tumors may contain hemosiderin pigment but not melanin, as could occur in melanoma. Go to the next page if you knew the correct answer, or click the link image(s) below to further research the concepts in this question (if desired).
Research Concepts: Atypical Fibroxanthoma
We update eBooks quarterly and Apps daily based on user feedback. Please tap flag to report any questions that need improvement.
Question 983:
A 6-year-old boy is brought to his pediatrician for an annual visit. The patient has a history of advanced osteosarcoma and finished treatment last week. He has a history of asthma and is treated with an albuterol rescue inhaler as needed. He is recovering from acute otitis media one month prior. He has no pertinent family history and lives at home with his mother, father, and sisters in a carpeted home with a pet dog. His mother mentions during the visit that she often has to say his name multiple times to get his attention and that sometimes “he seems oblivious to what is happening around him.” She wonders if he has trouble with attention like his sister, who was diagnosed with ADHD. You have to speak loudly to get his attention, and he continuously tries to play with his toys instead of participating in the examination. The physical exam is unremarkable. He is afebrile and reports no ear pain when manipulating the pinna, and denies discharge from the ear. What is the next best step in the assessment?
Choices: 1. 2. 3. 4.
Repeat course of antibiotics Multimodality assessement for ADHD Audiometry testing MRI of the brain
Answer: 3 - Audiometry testing Explanations: This patient has a history of advanced osteosarcoma and has just finished treatment. Advanced osteosarcoma is sometimes treated with a regimen containing cisplatin, which is known to be ototoxic. Ototoxicity may be manifested by tinnitus, loss of highfrequency hearing, and occasionally deafness. The index of suspicion should be high in children demonstrating signs of ototoxicity such as hearing loss. Pediatric patients should receive audiometry testing to indicate a baseline, and testing should be repeated before each subsequent round of chemotherapy. Loss of hearing acuity can be detrimental to language development in pediatric populations and should be assessed with audiometry. Go to the next page if you knew the correct answer, or click the link image(s) below to further research the concepts in this question (if desired).
Research Concepts: Cisplatin
We update eBooks quarterly and Apps daily based on user feedback. Please tap flag to report any questions that need improvement.
Question 984:
A 52-year-old man with a past medical history of hepatic cirrhosis secondary to hepatitis C infection status-post curative treatment with antiviral therapy presents to the hospital with a complaint of fatigue and abdominal pain. The initial workup includes a CT of the abdomen and pelvis with intravenous contrast. The scan reveals multiple enhancing hepatic lesions measuring 1.2 cm, 1.7 cm, and 2.2 cm, respectively. Laboratory analysis reveals mild elevation of liver function tests but no frank liver failure. His Child-Pugh score is B, and his ECOG score is 0. The patient undergoes CT-guided biopsy, the pathology for which reveals hepatocellular carcinoma. Which of the following is the most appropriate curative therapy for this patient?
Choices: 1. 2. 3. 4.
Surgical resection Transplantation Percutaneous radiofrequency ablation Transarterial chemoembolization
Answer: 2 - Transplantation Explanations: Hepatocellular carcinoma is highly associated with both cirrhosis and hepatitis C infection. Many options exist for the treatment of hepatocellular carcinoma. However, resection and/or transplantation represent the best chance for a complete cure. In the case of a patient with hepatocellular carcinoma on a background of cirrhosis, transplantation is the best option because it addresses both cancer and underlying liver dysfunction. Radiofrequency ablation is an excellent alternative treatment for hepatocellular carcinoma in patients who are not potential transplant/surgical candidates. Go to the next page if you knew the correct answer, or click the link image(s) below to further research the concepts in this question (if desired).
Research Concepts: Percutaneous Radiofrequency Ablation Of Liver Tumors
We update eBooks quarterly and Apps daily based on user feedback. Please tap flag to report any questions that need improvement.
Question 985:
A 67-year-old female presents to the clinic with a two-day history of right lower quadrant abdominal pain, fever, and anorexia. Her blood pressure is 121/71 mmHg, her temperature is 38.4 C (101.2 F), and her pulse is 79 beats per minute. Physical exam reveals moderate tenderness to palpation in the right lower quadrant without guarding or rigidity. Laboratory studies are significant for a white blood cell count of 13.2 x 10^9cells/L. CT scan shows a blind-ended tubular structure in the right lower quadrant with wall thickening, which measures seven millimeters in diameter, an ill-defined mass at its tip, and adjacent fat stranding. With regard to appendiceal neoplasms, which of the following statements is true?
Choices: 1. Tumor size determines the prognosis of adenocarcinomas 2. Carcinoids are less common than adenocarcinomas 3. Of all intestinal carcinoids, appendiceal carcinoids have the worst prognosis 4. The initial presentation of mucus-producing cystadenomas can be localized pseudomyxoma peritonei
Answer: 4 - The initial presentation of mucus-producing cystadenomas can be localized pseudomyxoma peritonei
Explanations: As many as 5% of routine specimens sent from appendectomies find neoplastic appendiceal lesions. Most neoplastic appendiceal lesions are benign and include mucosal metaplasia or hyperplasia, lipomas, leiomyomas, neuromas, and angiomas. For localized pseudomyxoma peritonei, appendiceal mucocele, most appendiceal carcinoids, and other benign neoplasms, the appendectomy itself may be curative. Definitive management of appendiceal masses encountered intraoperatively depends on the tumor’s size, location, whether or not it is carcinoid, if metastatic disease is present, and immunohistochemical and histologic findings. Go to the next page if you knew the correct answer, or click the link image(s) below to further research the concepts in this question (if desired).
Research Concepts: Appendectomy
We update eBooks quarterly and Apps daily based on user feedback. Please tap flag to report any questions that need improvement.
Question 986:
A 40-year-old male patient presented with shortness of breath and dry cough for the past few months. He works in a library and does not smoke or drink alcohol. He has a history of major depression treated with psychotherapy. He had a cane, which "helps to prevent a fall as he walks downstairs." The patient denies any change in weight or appetite and appears healthy. His medical history is insignificant, and his family history is not known as he was adopted. On examination, his breath sounds are normal. However, the patient coughed as he drank water from his water bottle. He has carried this bottle for one month as his tongue feels dry. The evaluation includes a CT chest, which demonstrates a mass in the anterior mediastinum. Which of the following statements is true regarding this mass?
Choices: 1. It is most often treated with local resection and radiation therapy 2. The most common paraneoplastic syndrome associated is Lambert-Eaton myasthenic syndrome 3. Adjunctive chemotherapy is recommended for low-grade tumors 4. It is more common in young males
Answer: 1 - It is most often treated with local resection and radiation therapy
Explanations: The patient presents with an anterior mediastinal mass, muscle weakness, and autonomic symptoms of dry mouth. This points to the diagnosis of thymoma. Generally, a thymoma is slow-growing and can become locally invasive. Stage IA tumors can be treated with local resection alone, while other thymomas are treated with surgical resection and radiation therapy. The most common paraneoplastic syndrome associated is myasthenia gravis, although Lambert-Eaton myasthenic syndrome can also occur as in this case. Other paraneoplastic syndromes include Addison disease, Cushing syndrome, pure red cell aplasia, and nephrotic syndrome. Chemotherapy with platinum-based drugs is used in recurrent thymomas. Thymomas are more common between 40 and 60 years of age, and there is a similar incidence in both sexes. Lambert-Eaton syndrome, characterized by muscle weakness, is correlated with small cell lung cancer in almost half of the affected patients. Go to the next page if you knew the correct answer, or click the link image(s) below to further research the concepts in this question (if desired).
Research Concepts: Anterior Mediastinal Mass
We update eBooks quarterly and Apps daily based on user feedback. Please tap flag to report any questions that need improvement.
Question 987:
A 58-year old male with no medical comorbidities is diagnosed with a 3.5 cm squamous cell carcinoma of the anal canal originating 1 cm from the anal verge. Positron emission tomography-computed tomography (PET/CT) shows no lymph node involvement or distant metastases. What is the recommended treatment?
Choices: 1. Abdominoperineal resection alone 2. Radiation alone 3. Definitive chemoradiation 4. Neoadjuvant chemoradiation followed by abdominoperineal resection
Answer: 3 - Definitive chemoradiation Explanations: Historically, abdominoperineal resection was standard of care for anal cancers, which results in a permanent colostomy. However, abdominoperineal resection alone yields a poor overall survival rate of 50% and local recurrence rates as high as 30%. For tumors of the anal canal that are 2 cm or larger (T2 or higher), the standard of care is definitive chemoradiation, with surgery reserved for salvage. Rectal adenocarcinomas are treated with neoadjuvant chemoradiation followed by abdominoperineal resection. Radiation alone is an appropriate option for anal canal tumors less than 2 cm with negative nodes (T1N0) in patients who may have medical comorbidities. However, T1N0 tumors also can be managed with definitive chemoradiation. Go to the next page if you knew the correct answer, or click the link image(s) below to further research the concepts in this question (if desired).
Research Concepts: Radiation Therapy For Anal Cancer
We update eBooks quarterly and Apps daily based on user feedback. Please tap flag to report any questions that
need improvement.
Question 988:
A 66-year-old woman presents to the clinic with a lower eyelid mass. The lesion measures 2 cm and has pearly, rolled borders with telangiectasias and a central ulcer. There is also a focal loss of eyelashes. She also complains of some irritation and itching. What is the most common tumor type of this patient's lesion?
Choices: 1. 2. 3. 4.
Superficial Nodular Morpheaform Noduloulcerative
Answer: 2 - Nodular Explanations: This patient has signs of basal cell carcinoma (BCC). Basal cell carcinoma is the most common skin cancer in humans. The most prime etiological factor in developing BCC is exposure to UV light. Nodular BCC is the most common subtype seen. This presents as a pearly papule or nodule with overlying telangiectasias and a rolled border. Ulceration may also be seen. The majority of basal cell carcinomas are amelanotic. Superficial BCC presents as a scaly erythematous patch or plaque, which can mimic psoriasis, discoid eczema, intraepidermal carcinoma, squamous cell carcinoma. Morpheaform is a less common subtype seen. This type typically presents with an indurated, whitish, scar-like plaque with indistinct margins. This subtype has a more aggressive clinical course. Noduloulcerative is less common. This presents as a nodule with overlying ulceration, also known as a "rodent ulcer." These typically indicate a longer duration of the tumor. Go to the next page if you knew the correct answer, or click the link image(s) below to further research the concepts in this question (if desired).
Research Concepts: Diseases of the Eyelashes
We update eBooks quarterly and Apps daily based on user feedback. Please tap flag to report any questions that need improvement.
Question 989:
A 44-year-old male patient has noticed progressive weakness in both legs and difficulty in climbing steps during the last month. He recalls doing a lot of flexion and extension movements 2 months ago while helping his sister move to another apartment. He is an intravenous illegal drug user with negative HIV testing. He has numbness in the testicular area. On exam, he has weakness of plantar flexion on the left foot and dorsiflexion on the right. Cremasteric reflex is absent. Upon further questioning, he says that he is unable to have an erection in the last month. A magnetic resonance imaging is ordered and is reported with a lesion in the L2 spinal canal area with enhancement upon contrast administration. It has a dumbbell-shape appearance extending to the right paraspinal area. Which is the most probable diagnosis?
Choices: 1. 2. 3. 4.
Spinal epidural abscess Meningioma Massive herniated lumbar disc Schwannoma
Answer: 4 - Schwannoma Explanations: Schwannoma commonly have a dumbbell-shape due to their propensity to grow out through the neural foramina. A spinal epidural abscess usually arises as a complication of discitis or vertebral osteomyelitis. Symptoms can be acute as they can cause venous thrombophlebitis with ischemia of the spinal cord. Massive herniated lumbar disc is one of the principal causes of cauda equina syndrome, but it is usually associated with the intervertebral disc space and does not exit through the foramen forming a dumbbell-shape appearance. Meningioma may arise anywhere along the lumbar spinal canal and often have a dural tail. Go to the next page if you knew the correct answer, or click the link image(s) below to further research the concepts in this question (if desired).
Research Concepts: Conus And Cauda Equina Tumors
We update eBooks quarterly and Apps daily based on user feedback. Please tap flag to report any questions that need improvement.
Question 990:
Which of the following gene mutations has been implicated as a potential trigger of human mammary oncogenesis leading to male breast cancer?
Choices: 1. 2. 3. 4.
PTEN tumor suppressor gene PTPN11 protein-coding gene WT1 tumor suppressor gene NF1 tumor suppressor gene
Answer: 1 - PTEN tumor suppressor gene Explanations: It has been shown that germline mutations of PTEN have been implicated as a potential trigger of human mammary oncogenesis. This is seen in Cowden disease, an autosomal dominant condition characterized by benign tumors called hamartomas. These have the potential for malignancy through the mutation of the PTEN tumor suppressor gene. The most common malignancies associated with Cowden disease are adenocarcinoma of the breast (20%). Once diagnosed with Cowden disease, females require annual mammography and breast MRI starting at age 30 to 35 as opposed to males who only require comprehensive physical exams with attention to the breasts. Go to the next page if you knew the correct answer, or click the link image(s) below to further research the concepts in this question (if desired).
Research Concepts: Male Breast Cancer
We update eBooks quarterly and Apps daily based on user feedback. Please tap flag to report any questions that
need improvement.
Question 991:
A patient is here for a wellness exam. He is a 75-year-old with a past medical history of congestive heart failure with reduced ejection fraction (EF of 15 %), who is concerned if he should be screened for prostate cancer. He has had dribbling of urine and has to strain with urination. He denies any dysuria or hematuria. What is the current recommendation from USPSTF for prostate cancer screening in this age group?
Choices: 1. Do a digital rectal exam (DRE) now 2. Prostate-specific antigen (PSA) level annually 3. DRE with PSA level every year 4. USPSTF recommends against screening for prostate cancer
Answer: 4 - USPSTF recommends against screening for prostate cancer
Explanations: The USPSTF continues to recommend against prostate cancer screening in men aged 70 years and older with a PSA-based test (D recommendation). Screening leads to false-positive PSA results and causes more harm, over-diagnosis, biopsies, and treatment in older men. Based on the review of the evidence of prostate cancerrelated issues, the USPSTF deduced that there is a small net benefit of screening for prostate cancer with PSAbased tests in men aged 55 to 69 years. As per American Cancer Society guideline, asymptomatic men with life expectancy based upon age and health status of less than a 10-years should not be offered to screen for prostate cancer. Go to the next page if you knew the correct answer, or click the link image(s) below to further research the concepts in this question (if desired).
Research Concepts: Cancer Screening
We update eBooks quarterly and Apps daily based on user feedback. Please tap flag to report any questions that need improvement.
Question 992:
A 7-year-old female is brought by her mother to the clinic because of a high-grade fever for the past 12 hours. The mother further reveals that her daughter has been very irritable and is crying a lot for the past 3 days. She is also pulling her ear and not eating well. The patient has been generally well, other than the occasional sore throat this season. Her temperature is 38.8° C 101 F, blood pressure is 90/60 mm Hg, a pulse is 122/min, and respirations are 22/min. She appears well-nourished but is irritable. Physical examination reveals enlarged cervical lymph nodes and splenomegaly. The tympanic membranes are inflamed. Lab work reveals a white cell count of 73,020 cell/mm3, hemoglobin of 8 g/dl, platelets level of 23,000 cell/mm3, and blast forms 75 %. The blast cells have condensed nuclear chromatin, small nucleoli, and scant agranular cytoplasm. Which of the following genetic abnormalities in B-acute lymphoblastic leukemia (B-ALL) is associated with a favorable outcome?
Choices: 1. 2. 3. 4.
t(12;21) Hypodiploidy MLL gene rearrangement t(9;22)
Answer: 1 - t(12;21) Explanations: Genetic studies are a useful tool in assessing prognosis in B-ALL. The following genetic abnormalities are associated with a favorable outcome: High hyperdiploidy and the t(12;21) ETV6-RUNX1. The following genetic abnormalities are associated with an unfavorable outcome in B-ALL: Hypodiploidy (less than 44 chromosomes), MLL rearrangement, BCR-ABL1 Ph-like ALL, CRLF2 rearrangement, intrachromosomal amplification of chromosome 21. Genetic testing results to aid in stratifying B-ALL patients into different risk groups. Go to the next page if you knew the correct answer, or click the link image(s) below to further research the concepts in this question (if desired).
Research Concepts: Lymphoblastic Lymphoma
We update eBooks quarterly and Apps daily based on user feedback. Please tap flag to report any questions that need improvement.
Question 993:
A 45-year-old man presents with fatigue, weight loss, and muscle weakness, and double vision at the end of the day. He has a past medical history of Hodgkin lymphoma, which was treated with mantle irradiation during childhood. He denies any fever, night sweats, shortness of breath, or chest pain. He drinks 2 to 3 glasses of wine per day and has a 30 pack-year smoking history. On examination, there is muscle weakness that improves on repeated movements. A CT scan reveals a 2 x 3 cm mass in the right lobe of the lung. Blood tests show positive antibodies against SOX1. Which of the following interventions would have reduced the risk of this complication?
Choices: 1. 2. 3. 4.
Chest physiotherapy Smoking cessation Moderation of alcohol use Albuterol inhaler use
Answer: 2 - Smoking cessation Explanations: The patient is presenting with secondary lung cancer years after mantle irradiation of the chest for treatment of Hodgkin lymphoma. Muscle weakness that improves with activity and autoantibodies against the SOX1 gene suggests Lambert-Eaton myasthenic syndrome, which is seen in lung cancers. The risk of secondary lung cancer is increased by 20 times if associated with smoking. Therefore, smoking cessation can reduce the risk of secondary lung malignancies in lung cancer patients. Alcohol use is associated with cirrhosis and subsequently, hepatocellular carcinoma. It does not affect lung cancer risk. Go to the next page if you knew the correct answer, or click the link image(s) below to further research the concepts in this question (if desired).
Research Concepts: Adverse Effects Of Radiation Therapy
We update eBooks quarterly and Apps daily based on user feedback. Please tap flag to report any questions that need improvement.
Question 994:
A 50-year-old man presents to the clinic with flushing of the face, diarrhea, and 10 lbs weight loss over one month. The clinician is suspecting neuroendocrine tumour. Whole body MRI and CT imaging reveal no lesions. Which of the following is the next best step in the management of this patient?
Choices: 1. 2. 3. 4.
111Ln-Octreoscan 68-Ga DOTATATE PET-CT Biochemical evaluation 18-FDG PET
Answer: 2 - 68-Ga DOTATATE PET-CT Explanations: 68-Gallium DOTATATE PET-CT can detect significantly more lesions in patients with negative anatomical imaging compared with 111-ln-octreoscan (30 vs. 2; p = 0.028). 68-Gallium DOTATATE PET-CT scans detect 74.3% of the lesions, whereas 111-ln-octreoscan detect 12% of the lesions. 68-Gallium DOTATATE PET-CT is helpful in the detection of NETs in symptomatic patients with no evidence of disease on the basis of anatomical imaging and endoscopic evaluation, with or without biochemical evidence of disease. Patients with a difficult diagnosis should be offered 68Gallium DOTATATE PET-CT. Go to the next page if you knew the correct answer, or click the link image(s) below to further research the concepts in this question (if desired).
Research Concepts: PET Scanning
We update eBooks quarterly and Apps daily based on user feedback. Please tap flag to report any questions that need improvement.
Question 995:
A 28-year-old otherwise healthy woman presents to the clinic with a new right-sided breast lump on a self-breast exam. Bedside ultrasound shows a small 0.5 cm hypoechoic, compressible cystic lesion, wider than tall. It does not cause her pain or discomfort. There are no other abnormalities. Which of the following is the next best step in the management of this patient?
Choices: 1. 2. 3. 4.
Reassurance and reevaluation in 2-6 months Fine needle aspiration Core needle biopsy Diagnostic mammogram
Answer: 1 - Reassurance and reevaluation in 2-6 months Explanations: Small simple cysts that do not cause discomfort do not require fine needle aspiration. They do require reevaluation, however, to ensure they are fully resorbed and not progressing into something more serious. For cysts that are large, complex, or causing discomfort, a fine needle aspiration may be performed. There is no indication for a core needle biopsy in a simple cyst. Ultrasound is the preferred modality in women under age 30, not mammogram. Go to the next page if you knew the correct answer, or click the link image(s) below to further research the concepts in this question (if desired).
Research Concepts: Fine Needle Aspiration Of Breast Masses
We update eBooks quarterly and Apps daily based on user feedback. Please tap flag to report any questions that need improvement.
Question 996:
A 30-year-old nulligravid woman presents to the healthcare provider with primary infertility. On examination, she has a 14 cm uterus. Sonogram confirms a 13 cm × 8 cm × 7 cm uterus with a single dominant leiomyoma measuring 9 cm distorting the endometrial cavity. She undergoes abdominal myomectomy without morcellation and no entry into the endometrial cavity. The pathologic diagnosis is “smooth muscle tumor of uncertain malignant potential (STUMP).” Which of the following is the best next step in the management of this patient?
Choices: 1. 2. 3. 4.
Hysterectomy Chemotherapy Radiation Surveillance
Answer: 4 - Surveillance Explanations: Leiomyosarcomas (LMS) represent 1% of uterine cancers that behave aggressively with a high rate of recurrence. The risk of LMS in patients diagnosed with STUMP is 1 in 350-1000 women. STUMP rarely recur or metastasize. Complete resection is the treatment of choice with no further work-up. Hysterectomy would only be recommended for the treatment of invasive sarcoma or incomplete resection of the isolated lesion. Chemotherapy and radiation are indicated for the treatment of confirmed invasive sarcoma. Go to the next page if you knew the correct answer, or click the link image(s) below to further research the concepts in this question (if desired).
Research Concepts: Leiomyoma
We update eBooks quarterly and Apps daily based on user feedback. Please tap flag to report any questions that need improvement.
Question 997:
A 30-year-old man presents to the clinic with a solitary tender lymph node swelling in his cervical region that has developed in the past eight months. The size of the lymph node is roughly 5 to 6 cm. He has a threemonth history of unintentional weight loss of 5 kg and pain in the neck around the swelling. There is no associated shortness of breath, malaise, weakness, or night sweats. No other systemic abnormalities are observed. Laboratory investigations are within range. Biopsy of the lymph node revealed an onion-skin appearance of the mantle zone around the germinal center, having a lollipop appearance with endothelial proliferation, hyalinization of the interfollicular zones, and a high number of immature plasma cells and follicular dendritic cells. What is the most common complication of the suspected disease?
Choices: 1. 2. 3. 4.
Thromboembolic disorders Renal insufficiency Paraneoplastic pemphigus Non-Hodgkin lymphoma
Answer: 3 - Paraneoplastic pemphigus Explanations: Castleman disease is a lymphoproliferative disorder that can be divided into types based on the involvement of single (Unicentric Castleman disease-UCD) or multiple lymph nodes (Multicentric Castleman disease-MCD) and whether systemic manifestations are present or not. Etiology is not yet established for UCD, but MCD can be caused by abnormal cytokine and interleukin production or associated with HIV or HHV-8. Laboratory investigations in UCD are normal, but MCD has abnormal levels of CRP, ESR, Hb. UCD has no clinical manifestations, unlike MCD, and presents as a solitary lymph node swelling that only causes pain when it grows and suppresses other tissues. MCD can cause systemic malfunctions. The most common complications with UCD involve the skin (paraneoplastic pemphigus) and lung abnormalities (bronchiolitis obliterans). As MCD has systemic abnormalities, the most deadly complications are hematopoietic disorders and multiple organ failure (especially kidneys are involved). Go to the next page if you knew the correct answer, or click the link image(s) below to further research the concepts in this question (if desired).
Research Concepts: Castleman Disease
We update eBooks quarterly and Apps daily based on user feedback. Please tap flag to report any questions that need improvement.
Question 998:
A 60-year-old man presents with a sixmonth history of insidious nonproductive cough. A chest xray is performed that shows a solitary mass of approximately 1 cm. He has no significant past medical history and has a 30 pack-year smoking history. Which of the following statements best describes the use of positron emission tomography (PET) scanning in this patient?
Choices: 1. PET scanning is not helpful in the workup of a solitary pulmonary nodule if there is a high pretest probability of malignancy. 2. PET scanning is not helpful as it cannot distinguish scar tissue from a tumor. 3. Because lung tissue attenuates 511-KeV photons, PET scanning will be ineffective. 4. A CT with contrast is more accurate than PET scanning for mediastinal staging of non-small cell lung cancer.
Answer: 1 - PET scanning is not helpful in the workup of a solitary pulmonary nodule if there is a high pretest probability of malignancy.
Explanations: While Positron emission tomography (PET) imaging has high sensitivity and specificity in evaluating a solitary pulmonary nodule, if the pretest probability of malignancy is already high, PET adds little. CT scanning of the chest is recommended following the identification of a pulmonary nodule on a chest radiograph. If suspicion of malignancy is high, then biopsy with histopathological evaluation should be performed. When 511-Kev photons come in contact with dense material, there is a decrease in the force or effect of the photons (attenuation). Lung tissue does not attenuate high-energy photons, as it is mostly air. Tumor is seen on PET scans as an area of abnormal 18 FDG uptake while scar tissue has no or very little uptake of the contrast. The sensitivity and specificity of PET scanning for lung tumor versus scar tissue is 97% and 100%. PET is more accurate than contrast CT for staging non-small cell lung cancer. Abnormally high uptake of 18 FDG by a lung mass is highly suggestive of malignancy. Go to the next page if you knew the correct answer, or click the link image(s) below to further research the concepts in this question (if desired).
Research Concepts: PET Scanning
We update eBooks quarterly and Apps daily based on user feedback. Please tap flag to report any questions that need improvement.
Question 999:
A 65-year-old woman presents to the office for evaluation of a swelling in front of her right ear. The patient says the swelling first appeared 2 years ago and has progressively increased in size since then. There is no pain, pruritus, or discharge associated with the swelling. On examination, there is a mass 6 x 5 x 5 cm in size, present in front of the right ear and extending to the angle of the mandible. A fine-needle aspiration biopsy demonstrates the presence of abundant, granular, lightly basophilic cytoplasm and round cellular nuclei with the presence of high mitotic activity. Staining with which of the following is most likely to help confirm the most likely diagnosis in this patient?
Choices: 1. 2. 3. 4.
Periodic acid Schiff with diastase Mucicarmine India ink Calponin
Answer: 1 - Periodic acid Schiff with diastase Explanations: This patient most likely has an acinar cell carcinoma of the parotid gland. Periodic acid Schiff with diastase (PASD) will highlight the zymogen granules in the serous cells. Although acinic cell carcinoma is usually considered a low-grade malignancy, it can undergo high-grade transformation into high-grade adenocarcinoma or undifferentiated carcinoma and present with complete loss of acinar differentiation, desmoplasia, tumor necrosis, frequent mitosis, and cervical or distant metastasis. Acinic cell carcinoma is located in the parotid gland in more than 80% of all cases. Go to the next page if you knew the correct answer, or click the link image(s) below to further research the concepts in this question (if desired).
Research Concepts: Malignant Salivary Gland Tumors
We update eBooks quarterly and Apps daily based on user feedback. Please tap flag to report any questions that need improvement.
Question 1000:
A 55-year-old woman presents to the office with complaints of low-grade fever, night sweats, and increased urination over the past 15 days. She also has mild constipation, a decrease in appetite, and bone pain. She further states that she is irritated easily, and she is not able to focus on her work. Because of this behavior, she was fired from her job. Her past medical history is significant for diabetes mellitus for 10 years, well-controlled with medications. She visits her clinician regularly, and her blood sugar is under control. On physical examination, a mobile mass is palpable in the right mandible with a palpable lymph node. Her temperature is 36.7 C, blood pressure 130/70 mmHg, and heart rate 82/min. Current laboratory results reveal glucose 120 mg/dL, HbA1c 5.5%, serum calcium 20 mg/dL, urine calcium 270 mg, serum creatinine 1.2 mg/dL, and serum sodium 137 mEq/L. What is the most likely cause of her symptoms?
Choices: 1. Increase in the parathyroid hormone in the blood 2. Increase parathyroid hormone-related protein (PTHrP) in the blood due to malignancy 3. Renal failure due to diabetes mellitus 4. Mutations in the calcium-sensing receptor in parathyroid and kidney
Answer: 2 - Increase parathyroid hormone-related protein (PTHrP) in the blood due to malignancy
Explanations: Non–Hodgkin lymphoma presents with systemic B symptoms (low-grade fever, night sweats, weight loss), and painless peripheral lymphadenopathy in up to twothird patient. Hypercalcemia due to non-Hodgkin lymphoma is due to increase parathyroid hormone-related protein (PTHrP) or 1,25-dihydroxycholecalciferol (calcitriol). Symptoms of hypercalcemia include agitation, constipation, and increased urination. The serum calcium level can rise to 21 mg/dL. Urine calcium rises above 250 mg in hypercalcemia due to malignancy. Go to the next page if you knew the correct answer, or click the link image(s) below to further research the concepts in this question (if desired).
Research Concepts: Non-Hodgkin Lymphoma
We update eBooks quarterly and Apps daily based on user feedback. Please tap flag to report any questions that need improvement.
About StatPearls
StatPearls develops Kindle eBooks for Medical Professionals. Here is a list of other books available on Kindle. Please visit our website at http://www.StatPearls.com for more information. Allied Health Specialties Audiology Cardiac Interventional Technology Certified Clinical Nutritionist (CCN)
Certified Medical Assistant Certified Nursing Assistant Neuropsychology Registered Dietitian EKG Technician EMS-EMT Basic EMS-AEMT Intermediate EMS-Paramedic Naturopathic Part II - Clinical Sciences Occupational Therapist Lab-Phlebotomy Technician (PBT) Physical Therapy Radiology Technology Radiology Technology-CT Radiology Tech-Mammography Radiology Technology-MRI Radiology Tech-Nuclear Medicine Radiology Technology-Vascular Respiratory Therapist Social Work Speech Pathology Certified Surgical Technologist Sonography Technology EMS-Flight Paramedic EMS-Critical Care Paramedic EMS-Tactical Paramedic Physical Therapy-Neurology Physical Therapy-Geriatrics Physical Therapy-Orthopedics Physical Therapy-Pediatrics Physical Therapy-Sports Lab-Chemistry Lab-Blood Banking Lab-Microbiology Lab-Hematology EMR-First Medical Responder
EMS-Community Paramedic Lab-Medical Laboratory Scientist (MLS) Lab-Medical Laboratory Technician Occupational Therapy Assistant Certified Surgical 1st Assistant Certified Hyperbaric Technologist (CHT) Naturopathic Part I - Basic Sciences Physical Therapy-Assistant Ob/Gyn - CREOG® In-Service Emergency Medicine - MRCEM SBA Optometry-Clinical Science Certified Anesthesiology Assistants Clinical Nurse Specialties CNS-Diabetes Management CNS-Adult-Gerontology CNS-Pediatric CNS-Neonatal Dental Doctor Specialties Graduate Specialties ACLS PALS International Trauma-ITLS Medical Doctor Specialties CNS-Anesthetist Adolescent Medicine Aerospace Medicine Allergy and Immunology Ambulatory and Urgent Care Anesthesiology Anesthesiology-Cardiothoracic
Anesthesiology-Pediatric Advanced Trauma Support Cardiology Cardiology-Electrophysiology Cardiology-Failure Transplant Cardiology-Interventional Critical Care Dermatology Emergency Medicine Endocrinology Family Medicine Gastroenterology Genetics Geriatrics Hematology Hospice And Palliative Medicine Infectious Disease Internal Medicine Nephrology Neurology Neurology-Headache Neurodevelopmental Disabilities Neuromuscular Medicine Obesity Medicine Occupational Medicine Oncology Pain Medicine Pathology-Anatomic Pediatric Pediatric-Cardiology Pediatric-Child Abuse Pediatric-Critical Care Pediatric-Developmental Pediatric-Emergency Medicine Pediatric-Endocrinology Pediatric-Gastroenterology
Pediatric-Hematology-Oncology Pediatric-Infectious Diseases Pediatric-Neonatal Perinatal Pediatric-Nephrology Pediatric-Neurology Pediatric-Pulmonology Pediatric-Rehabilitation Pediatric-Rheumatology Physical Medicine Psychiatry Addiction Medicine Psychiatry - Child and Adolescent Psychiatry-Geriatric Psychiatry - Consultation-Liaison Public and Preventive Medicine Pulmonary Radiation Oncology Radiology - Diagnostic Radiology - Musculoskeletal Radiology - Neuroradiology Nuclear Medicine Radiology - Pediatric Radiology - Physics and Safety Radiology - Vascular and Interventional Rheumatology Sleep SPEX Spinal Cord Injury Sports Medicine Cardiothoracic Surgery Colorectal Surgery Complex Surgical Oncology Craniofacial Surgery General Surgery Gynecologic Oncology Hand Surgery
Ob/Gyn - Maternal-Fetal Medicine Neurosurgery Obstetrics and Gynecology Ophthalmology Oral And Maxillofacial Surgery Orthopedic Surgery Otolaryngology Pediatric Surgery Plastic Surgery Reproductive Endocrinology & Infertility Thoracic Surgery Urology Urology - Pediatric Vascular Surgery Toxicology Transplant Hepatology Undersea and Hyperbaric Radiology - Cardiac Radiology - Gastrointestinal Radiology - Breast Radiology - Genitourinary Radiology-Thoracic Radiology - Ultrasound Emergency Medicine EMS Compliance Physician Clinical Tropical Medicine Dermatopathology Dermatology-Pediatric Dermatologic Micrographic Surgery Pathology-Laboratory Medicine Pathology-Blood Banking Transfusion Medicine Pathology-Chemistry Pathology-Cytopathology Pathology-Forensics Pathology-Hematology Pathology-Microbiology
Pathology-Molecular Genetic Pathology-Neuropathology Pathology-Pediatric Neurocritical Care Pediatric-Hospital Medicine Neurology - Epilepsy Medicine Neurology - Clinical Neurophysiology Wound Ophthalmology - Oculoplastic Surgery Brain Injury Medicine Vascular and Stroke Neurology Podiatry Medicine Podiatry Surgery Pelvic Medicine and Reconstructive Surgery Osteopathic Neuromusculoskeletal Medicine Neurology - Behavioral Neuropsychiatry Cardiology-Nuclear Cardiology-Echocardiography Cardiology-CV Computed Tomography Vascular Medicine Nutrition Medicine Quality Assurance and Utilization Facial Plastic Surgery Certified Hypertension Clinician Integrative Medicine Simulation In Healthcare Internal Medicine - MRCP UK and Ireland Part 2 Internal Medicine - MRCP UK and Ireland Part 1 General Surgery - MRCS Part A Family Medicine - MRCGP-Applied Knowledge Test Advanced Diabetes Management Spine Surgery COMVEX General Surgery - Inservice ABSITE® Correctional Medicine Pediatric - MRCPCH
Emergency Medicine - FRCEM Primary Certified Hypertension Specialist Ophthalmology - Cornea & External Eye Disease Ophthalmology - Glaucoma Ophthalmology - Neuro Ophthalmology - Pediatric & Strabismus Ophthalmology - Cataract UK PLAB 1 Australian Medical Council-IMG Canada MCC-QE 1 Optometry-Basic Science Optometry-Advanced Medical Care National Board Dental Examination (NBDE) Part I Pakistan Medical Commission NLE Medical Student Specialties USMLE Step 1 USMLE Step 2 USMLE Step 3 COMLEX Level 1 COMLEX Level 2 COMLEX Level 3 Ambulatory Care-Medical Student Neurology-Medical Student Family Medicine-Medical Student Internal Medicine-Med Student Ob Gyn-Medical Student Pediatrics-Medical Student Psychiatry-Medical Student Surgery-Medical Student Emergency Medicine-Med Student Internal Medicine Advanced-Med Student Indian PG Entrance Exam-NEETPG Dermatology-Medical Student Clinical Diagnosis
Anatomy-Medical Student Behavioral Sci-Medical Student Biochemistry-Medical Student Embryology-Medical Student Genetics-Medical Student Histology-Medical Student Microbiology-Medical Student Neuroanatomy-Medical Student Path Clin Lab-Medical Student Pathology-Medical Student Pharmacology-Medical Student Physiology-Medical Student Philippine Physician Exam (PLE) Statistics and Healthcare Economics Law, Medicine, and Ethics Pakistan-1st Professional Pakistan-3rd Professional Pakistan-4th Professional Pakistan-Final Professional Ophthalmology-Medical Student Indonesia CBT Final Examination Nepal Medical Council Exam Medical Council Nigeria (MDCN) Allergy - Medical Student Anesthesiology - Medical Student Cardiology - Medical Student Critical Care-Medical Student Endocrinology-Medical Student Gastroenterology-Medical Student Infectious Disease-Medical Student Nephrology-Medical Student Oncology-Medical Student Peds-Cardiology Medical Student Peds-Critical Care Medical Student Peds-Endocrinology Med Student Peds-GI Medical Student
Peds-Hem Onc-Medical Student Peds-ID-Medical Student Peds-Neonatal Medical Student Peds-Nephrology Medical Student Peds-Pulmonology Medical Student Physical Medicine-Medical Student Pulmonary-Medical Student Diagnostic Radiology-Medical Student Rheumatology-Medical Student Neurosurgery-Medical Student Orthopedic-Medical Student Urology-Medical Student Otolaryngology-Medical Student Cardiothoracic-Medical Student Plastic Surgery-Medical Student Vascular Surgery-Medical Student Caribbean Medical Licensing Exam Peds-Surgery Medical Student Peds-Neurology Medical Student Osteopathic Manipulation Hematology-Medical Student Hand-Medical Student Sports Medicine-Medical Student ENARM Mexico Exam Pakistan FCPS (Basic Science) Pakistan-2nd Professional Nurse Practitioner Specialties NP-Acute Adult and Gerontology NP-Diabetes Management NP-Family NP-Psychiatric Mental Health NP-Adult-Geriatric Primary Care NP-Midwife NP-Pediatric Primary Care
NP-Emergency Medicine NP-Women's Health Care NP-Acute Care Pediatric NP-Neonatal NP-Dermatology NP-Addiction NP-Nephrology NP-Oncology NP-Orthopedic NP-Advanced Health Assessment NP-Advanced Pharmacology NP-Advanced Pathophysiology NP-Advanced Physiology NP-Urology NP-Wound, Continence, And Ostomy NP-Advanced Neurovascular Practitioner Physician Assistant Specialties PA-PANCE PA-PANRE PA-Cardiovascular Thoracic PA-Emergency Medicine PA-Hospital Medicine PA-Nephrology PA-Orthopedic PA-Pediatric PA-Psychiatry Pharmacist Specialties Pharmacy-Technician (PTCB) Pharmacy-NAPLEX Review Pharmacy-Ambulatory Care Pharmacy-Critical Care Pharmacy-Nutrition Support Pharmacy-Oncology
Pharmacy-Pediatric Pharmacy-Pharmacotherapy Pharmacy-Psychiatric Pharmacy-Infectious Disease Pharmacy-Geriatric Pharmacy-Cardiac Pharmacy-Applied Toxicology Nursing Specialties NP-Pediatric Mental Health Specialist (PMHS) LPN LVN NCLEX PN-Nursing Student NCLEX RN-Nursing Student Community Health-Nursing Student Adult Medical Surgical RN-Nursing Student Maternal Newborn PN-Nursing Student Child Health RN-Nursing Student Mental Health RN-Nursing Student Fundamentals-Nursing Student Mental Health PN-Nursing Student Pharmacology RN-Nursing Student Professional Ethics-Nursing Student Calculations-Nursing Student Maternal Newborn RN-Nursing Student Pharmacology PN-Nursing Student Adult Medical Surgical PN-Nursing Student Child Health PN-Nursing Student Anatomy-Nursing Student Physiology-Nursing Student Microbiology-Nursing Student Nutrition-Nursing Student Lab and Diagnostic Testing-Nursing Student Elder Adult Care-Nursing Student Critical Care Rhythms-Nursing Student Pathophysiology-Nursing Student Health Assessment-Nursing Student
Nurse-Addiction (CARN) Nurse-Administrator NE NEA Nurse-AIDS-HIV (ACRN AACRN) Nurse-Ambulatory Care Nurse-Perianesthesia CAPA-CPAN Nurse-Anticoagulation Nurse-Bariatric (CBN) Nurse-Breast Care CBCN© Review Nurse-Cardiovascular CVN-CVRN Nurse-Case Management Nurse-Continence Care Nurse-Corrections (CCHP) Nurse-Critical Care (CCRN) Nurse-Dermatology (DNC) Nurse-Certified Diabetes Specialist Nurse-Emergency Certification Nurse-Flight Registered (CFRN) Nurse-Foot-Nail Care (CFCN) Nurse-Gastroenterology (CGRN) Nurse-Gerontology (NCA) Nurse-Hemodialysis (CHN) Nurse-Hospice-Palliative CHPN Nurse-Hyperbaric (CHRN) Nurse-Infection Control (CIC) Nurse-Infusion (CRNI) Nurse-Lactation (CLA) Nurse-Low Risk Neonatal (LRN) Nurse-Maternal Newborn (MNN) Nurse-Medical Surgical Nurse-Neonatal Intensive Care Nurse-Nephrology (CNN) Nurse-Neuroscience (CNRN) Nurse-Occupational (COHN) Nurse-Oncology (OCN) Nurse-Operating Room (CNOR) Nurse-Ophthalmology (CRNO)
Nurse-Orthopedic (ONC) Nurse-Otolaryngology (CORLN) Nurse-Pain Management (PMCN) Nurse-Pediatric (CPN) Nurse-Pediatric Critical Care Nurse-Pediatric Emergency CPEN Nurse-Pediatric Hem Onc (CPHON) Nurse-Inpatient Obstetric RNC Nurse-Plastic Surgery (CPSN) Nurse-Professional Development Nurse-Healthcare Quality Nurse-Progressive Care (PCCN) Nurse-Psychiatric Health Nurse-Radiologic (CRN) Nurse-Rehabilitation Registered Nurse-Risk Management (CPHRM) Nurse-Sexual Assault (SANE) Nurse-Transplant (CCTN-CCTC)) Nurse-Transport (CTRN) Nurse-Urologic Registered Nurse-Wound Care Nurse-Eating Disorders (CEDRN) Nurse-Neonatal Transport C-NPT Nurse-Trauma (TCRN) Nurse-Emergency Pediatric Certification Nurse-Stroke Certified (SC-RN) Nurse-Wound Care and Ostomy Nurse-Wound, Ostomy, and Continence Nurse-Clinical Leader Nurse-Assisted Living Nurse-Vascular Access (VABC) Nurse-School (NBCSN)